Você está na página 1de 377

Zenaide Auxiliadora Pachegas Bruno Chieregatti e João de Sá Brasil, Heitor Ferreira,

Ovidio Lopes da Cruz Netto, Ricardo Razaboni

Prefeitura do Município de São José dos Pinhais do Estado do Paraná

SÃO JOSÉ DOS PINHAIS - PR


Guarda Municipal Masculino

A apostila preparatória é elaborada antes da publicação do Edital Oficial com base

no edital anterior, para que o aluno antecipe seus estudos.

MR046-19
Todos os direitos autorais desta obra são protegidos pela Lei nº 9.610, de 19/12/1998.
Proibida a reprodução, total ou parcialmente, sem autorização prévia expressa por escrito da editora e do autor. Se você
conhece algum caso de “pirataria” de nossos materiais, denuncie pelo sac@novaconcursos.com.br.

OBRA

Prefeitura do Município de São José dos Pinhais do Estado do Paraná

Guarda Municipal Masculino

Atualizada até 03/2019

AUTORES
Português - Profª Zenaide Auxiliadora Pachegas Branco
Matemática - Prof°Bruno Chieregatti e João de Sá Brasil
Conhecimentos Gerais - Prof° Heitor Ferreira
Noções de Informática - Prof° Ovídio Lopes da Cruz Netto
Legislação Específica - Prof° Ricardo Razaboni

PRODUÇÃO EDITORIAL/REVISÃO
Elaine Cristina
Érica Duarte
Leando Filho
Karina Fávaro

DIAGRAMAÇÃO
Elaine Cristina
Thais Regis
Danna Silva

CAPA
Joel Ferreira dos Santos

www.novaconcursos.com.br

sac@novaconcursos.com.br
SUMÁRIO
PORTUGUÊS
Compreensão e interpretação de textos; Localização de informações explícitas no texto; Interpretação de linguagem
não-verbal (tabelas, fotos, quadrinhos, etc); .................................................................................................................................................... 01
Reconhecimento das relações lógico-discursivas presentes no texto, marcadas por conjunções, advérbios, preposições
argumentativas, locuções, etc;................................................................................................................................................................................ 12
Reconhecimento de efeitos de sentido decorrentes do uso de pontuação, da exploração de recursos ortográficos, de
campos semânticos, e de outras notações;....................................................................................................................................................... 59
Identificação de diferentes estratégias que contribuem para a continuidade do texto (figuras de linguagem, pronomes
relativos, demonstrativos, etc); Compreensão de estruturas temática e lexical complexas........................................................... .68

MATEMÁTICA
Resolução de problemas envolvendo: Raciocínio lógico;............................................................................................................................. 01
Porcentagem e juros;................................................................................................................................................................................................... 23
Razões e proporções; ................................................................................................................................................................................................. 30
Medidas de tempo; ..................................................................................................................................................................................................... 34
Equações de primeiro e segundo grau; .............................................................................................................................................................. 39
Sistemas de equações; .............................................................................................................................................................................................. 39
Sistema de medidas de tempo; ............................................................................................................................................................................. 45
Sistema métrico decimal; ......................................................................................................................................................................................... 45
Formas geométricas básicas; .................................................................................................................................................................................. 46
Perímetro, área e volume de figuras geométricas; ......................................................................................................................................... 46
Regra de três simples e composta......................................................................................................................................................................... 71

CONHECIMENTOS GERAIS
São José dos Pinhais: fundação, colonização, emancipação política; fatos e aspectos históricos, características geográfi-
cas, culturais, econômicos e étnicos. Atualidades políticas, econômicas e sociais do Brasil e do Paraná................................. 01

NOÇÕES DE INFORMÁTICA
Correio Eletrônico: conceitos; Gerenciador de e-mail (Outlook Express);.............................................................................................. 01
BrOffice Documento Texto (Writer): Atalhos e barra de ferramentas; Modos de seleção de texto; Formatação de texto;
Formatação de parágrafos; Alinhamento;........................................................................................................................................................... 03
BrOffice Planilha (Calc): Atalhos e barra de ferramentas; Formatação de dados; Seleção de células; Atributos de
caracteres..............................................................................................................................................................................................................................23
SUMÁRIO
LEGISLAÇÃO ESPECÍFICA
Lei Orgânica do Município de São José dos Pinhais;.......................................................................................................................................... 01
Constituição da República Federativa do Brasil; Dos direitos e garantias fundamentais - Capítulo I (dos direitos e deveres
individuais e coletivos); Da organização do Estado; Da defesa do Estado e das Instituições Democráticas - Capítulo III (da
segurança pública); .......................................................................................................................................................................................................... 14
Declaração Universal dos Direitos Humanos de 1948;....................................................................................................................................... 37
Lei Ordinária nº 10.741, de 1º de outubro de 2003. (Estatuto do Idoso); Dos direitos fundamentais; Das medidas de
proteção; Dos crimes; ...................................................................................................................................................................................................... 47
Lei Ordinária nº 10.826, de 22 de dezembro de 2003. (Estatuto do Desarmamento); ........................................................................ 56
Decreto nº 5.123, de 1º de Julho de 2004; ............................................................................................................................................................. 63
Portaria Interministerial 4.226, de 31 de Dezembro de 2010; ........................................................................................................................ 13
Lei nº 11.340, de 7 de Agosto de 2006 (Lei Maria da Penha); ........................................................................................................................ 74
Lei Ordinária nº 8.069, de 13 de julho de 1990 (Estatuto da Criança e do Adolescente); Dos direitos fundamentais; Da
prática do ato infracional;............................................................................................................................................................................................... 83
Lei 9.503, de 23 de setembro de 1997. (Código de Trânsito Brasileiro); Disposições preliminares; Do sistema nacional de
trânsito; Das normas gerais de circulação e conduta; Da habilitação; Dos crimes de trânsito.......................................................138
ÍNDICE

PORTUGUÊS
Compreensão e interpretação de textos; Localização de informações explícitas no texto; Interpretação de linguagem não-ver-
bal (tabelas, fotos, quadrinhos, etc); ...............................................................................................................................................................................01
Reconhecimento das relações lógico-discursivas presentes no texto, marcadas por conjunções, advérbios, preposições argu-
mentativas, locuções, etc;.................................................................................................................................................................................................... 12
Reconhecimento de efeitos de sentido decorrentes do uso de pontuação, da exploração de recursos ortográficos, de campos
semânticos, e de outras notações;................................................................................................................................................................................... 59
Identificação de diferentes estratégias que contribuem para a continuidade do texto (figuras de linguagem, pronomes relati-
vos, demonstrativos, etc); Compreensão de estruturas temática e lexical complexas.................................................................................68
Compreender significa
COMPREENSÃO E INTERPRETAÇÃO DE Entendimento, atenção ao que realmente está escrito.
TEXTOS; LOCALIZAÇÃO DE INFORMAÇÕES O texto diz que...
EXPLÍCITAS NO TEXTO; INTERPRETAÇÃO É sugerido pelo autor que...
DE LINGUAGEM NÃO-VERBAL (TABELAS, De acordo com o texto, é correta ou errada a afir-
mação...
FOTOS, QUADRINHOS, ETC);
O narrador afirma...

3. Erros de interpretação
COMPREENSÃO E INTERPRETAÇÃO DE TEXTO
 Extrapolação (“viagem”) = ocorre quando se sai
Texto – é um conjunto de ideias organizadas e rela- do contexto, acrescentando ideias que não estão
cionadas entre si, formando um todo significativo capaz no texto, quer por conhecimento prévio do tema
de produzir interação comunicativa (capacidade de codi- quer pela imaginação.
ficar e decodificar).  Redução = é o oposto da extrapolação. Dá-se
Contexto – um texto é constituído por diversas frases. atenção apenas a um aspecto (esquecendo que
Em cada uma delas, há uma informação que se liga com um texto é um conjunto de ideias), o que pode ser
a anterior e/ou com a posterior, criando condições para insuficiente para o entendimento do tema desen-
a estruturação do conteúdo a ser transmitido. A essa in- volvido.
terligação dá-se o nome de contexto. O relacionamento  Contradição = às vezes o texto apresenta ideias
entre as frases é tão grande que, se uma frase for retirada contrárias às do candidato, fazendo-o tirar con-
de seu contexto original e analisada separadamente, po- clusões equivocadas e, consequentemente, errar a
derá ter um significado diferente daquele inicial. questão.
Intertexto - comumente, os textos apresentam refe-
rências diretas ou indiretas a outros autores através de Observação:
citações. Esse tipo de recurso denomina-se intertexto. Muitos pensam que existem a ótica do escritor e a
Interpretação de texto - o objetivo da interpretação ótica do leitor. Pode ser que existam, mas em uma prova
de um texto é a identificação de sua ideia principal. A de concurso, o que deve ser levado em consideração é o
partir daí, localizam-se as ideias secundárias (ou fun- que o autor diz e nada mais.
damentações), as argumentações (ou explicações), que
levam ao esclarecimento das questões apresentadas na Coesão - é o emprego de mecanismo de sintaxe que
prova. relaciona palavras, orações, frases e/ou parágrafos entre
si. Em outras palavras, a coesão dá-se quando, através de
Normalmente, em uma prova, o candidato deve: um pronome relativo, uma conjunção (NEXOS), ou um
 Identificar os elementos fundamentais de uma pronome oblíquo átono, há uma relação correta entre o
argumentação, de um processo, de uma época que se vai dizer e o que já foi dito.
(neste caso, procuram-se os verbos e os advérbios,
os quais definem o tempo). São muitos os erros de coesão no dia a dia e, entre
 Comparar as relações de semelhança ou de dife- eles, está o mau uso do pronome relativo e do prono-
renças entre as situações do texto. me oblíquo átono. Este depende da regência do verbo;
 Comentar/relacionar o conteúdo apresentado aquele, do seu antecedente. Não se pode esquecer tam-
com uma realidade. bém de que os pronomes relativos têm, cada um, valor
 Resumir as ideias centrais e/ou secundárias. semântico, por isso a necessidade de adequação ao an-
 Parafrasear = reescrever o texto com outras pa- tecedente.
lavras. Os pronomes relativos são muito importantes na in-
terpretação de texto, pois seu uso incorreto traz erros de
1. Condições básicas para interpretar coesão. Assim sendo, deve-se levar em consideração que
existe um pronome relativo adequado a cada circunstân-
Fazem-se necessários: conhecimento histórico-literá- cia, a saber:
rio (escolas e gêneros literários, estrutura do texto), lei- que (neutro) - relaciona-se com qualquer anteceden-
te, mas depende das condições da frase.
tura e prática; conhecimento gramatical, estilístico (qua-
qual (neutro) idem ao anterior.
lidades do texto) e semântico; capacidade de observação
quem (pessoa)
e de síntese; capacidade de raciocínio.
cujo (posse) - antes dele aparece o possuidor e depois
o objeto possuído.
2. Interpretar/Compreender como (modo)
onde (lugar)
Interpretar significa: quando (tempo)
Explicar, comentar, julgar, tirar conclusões, deduzir.
PORTUGUÊS

quanto (montante)
Através do texto, infere-se que... Exemplo:
É possível deduzir que... Falou tudo QUANTO queria (correto)
O autor permite concluir que... Falou tudo QUE queria (errado - antes do QUE, deveria
Qual é a intenção do autor ao afirmar que... aparecer o demonstrativo O).

1
3. Dicas para melhorar a interpretação de textos

 Leia todo o texto, procurando ter uma visão geral


EXERCÍCIOS COMENTADOS
do assunto. Se ele for longo, não desista! Há muitos
candidatos na disputa, portanto, quanto mais infor- 1. (PCJ-MT – Delegado Substituto – Superior – Ces-
mação você absorver com a leitura, mais chances pe – 2017)
terá de resolver as questões.
 Se encontrar palavras desconhecidas, não inter- Texto CG1A1AAA
rompa a leitura.
 Leia o texto, pelo menos, duas vezes – ou quantas A valorização do direito à vida digna preserva as duas
forem necessárias. faces do homem: a do indivíduo e a do ser político; a
 Procure fazer inferências, deduções (chegar a uma do ser em si e a do ser com o outro. O homem é inteiro
conclusão). em sua dimensão plural e faz-se único em sua condição
 Volte ao texto quantas vezes precisar. social. Igual em sua humanidade, o homem desiguala-se,
 Não permita que prevaleçam suas ideias sobre singulariza-se em sua individualidade. O direito é o ins-
as do autor. trumento da fraternização racional e rigorosa.
 Fragmente o texto (parágrafos, partes) para me- O direito à vida é a substância em torno da qual todos os
lhor compreensão. direitos se conjugam, se desdobram, se somam para que
 Verifique, com atenção e cuidado, o enunciado o sistema fique mais e mais próximo da ideia concretizá-
de cada questão. vel de justiça social.
 O autor defende ideias e você deve percebê-las. Mais valeria que a vida atravessasse as páginas da Lei
 Observe as relações interparágrafos. Um parágra- Maior a se traduzir em palavras que fossem apenas a re-
fo geralmente mantém com outro uma relação de velação da justiça. Quando os descaminhos não condu-
continuação, conclusão ou falsa oposição. Identifi- zirem a isso, competirá ao homem transformar a lei na
que muito bem essas relações. vida mais digna para que a convivência política seja mais
 Sublinhe, em cada parágrafo, o tópico frasal, ou fecunda e humana.
seja, a ideia mais importante. Cármen Lúcia Antunes Rocha. Comentário ao artigo 3.º.
 Nos enunciados, grife palavras como “correto” In: 50 anos da Declaração Universal dos Direitos Hu-
ou “incorreto”, evitando, assim, uma confusão manos 1948-1998: conquistas e desafios. Brasília: OAB,
na hora da resposta – o que vale não somente Comissão Nacional de Direitos Humanos, 1998, p. 50-1
para Interpretação de Texto, mas para todas as de- (com adaptações).
mais questões!
 Se o foco do enunciado for o tema ou a ideia prin- Compreende-se do texto CG1A1AAA que o ser humano
cipal, leia com atenção a introdução e/ou a con- tem direito
clusão.
 Olhe com especial atenção os pronomes relativos, a) de agir de forma autônoma, em nome da lei da sobre-
pronomes pessoais, pronomes demonstrativos, vivência das espécies.
etc., chamados vocábulos relatores, porque reme- b) de ignorar o direito do outro se isso lhe for necessário
tem a outros vocábulos do texto. para defender seus interesses.
c) de demandar ao sistema judicial a concretização de
SITES seus direitos.
http://www.tudosobreconcursos.com/materiais/por- d) à institucionalização do seu direito em detrimento dos
tugues/como-interpretar-textos direitos de outros.
http://portuguesemfoco.com/pf/09-dicas-para-me- e) a uma vida plena e adequada, direito esse que está na
lhorar-a-interpretacao-de-textos-em-provas essência de todos os direitos.
http://www.portuguesnarede.com/2014/03/dicas-
-para-voce-interpretar-melhor-um.html Resposta: Letra E. O ser humano tem direito a uma
http://vestibular.uol.com.br/cursinho/questoes/ques- vida digna, adequada, para que consiga gozar de seus
tao-117-portugues.htm direitos – saúde, educação, segurança – e exercer seus
deveres plenamente, como prescrevem todos os di-
reitos: (...) O direito à vida é a substância em torno da
qual todos os direitos se conjugam (...).

2. (PCJ-MT – Delegado Substituto – Superior – Ces-


pe – 2017)

Texto CG1A1BBB

Segundo o parágrafo único do art. 1.º da Constituição


PORTUGUÊS

da República Federativa do Brasil, “Todo o poder emana


do povo, que o exerce por meio de representantes elei-
tos ou diretamente, nos termos desta Constituição.” Em
virtude desse comando, afirma-se que o poder dos juízes

2
emana do povo e em seu nome é exercido. A forma de 1. As tipologias textuais se caracterizam pelos as-
sua investidura é legitimada pela compatibilidade com as pectos de ordem linguística
regras do Estado de direito e eles são, assim, autênticos
agentes do poder popular, que o Estado polariza e exer- Os tipos textuais designam uma sequência definida
ce. Na Itália, isso é constantemente lembrado, porque pela natureza linguística de sua composição. São observa-
toda sentença é dedicada (intestata) ao povo italiano, em dos aspectos lexicais, sintáticos, tempos verbais, relações
nome do qual é pronunciada. logicas. Os tipos textuais são o narrativo, descritivo, argu-
mentativo/dissertativo, injuntivo e expositivo.
Cândido Rangel Dinamarco. A instrumentalidade do pro- A) Textos narrativos – constituem-se de verbos de
cesso. São Paulo: Revista dos Tribunais, 1987, p. 195 (com ação demarcados no tempo do universo narrado,
adaptações). como também de advérbios, como é o caso de an-
Conforme as ideias do texto CG1A1BBB, tes, agora, depois, entre outros: Ela entrava em seu
a) o Poder Judiciário brasileiro desempenha seu papel com carro quando ele apareceu. Depois de muita conver-
fundamento no princípio da soberania popular. sa, resolveram...
b) os magistrados do Brasil deveriam ser escolhidos pelo B) Textos descritivos – como o próprio nome indica,
voto popular, como ocorre com os representantes dos
descrevem características tanto físicas quanto psi-
demais poderes.
cológicas acerca de um determinado indivíduo ou
c) os magistrados italianos, ao contrário dos brasileiros,
objeto. Os tempos verbais aparecem demarcados
exercem o poder que lhes é conferido em nome de seus
nacionais. no presente ou no pretérito imperfeito: “Tinha os
d) há incompatibilidade entre o autogoverno da magistra- cabelos mais negros como a asa da graúna...”
tura e o sistema democrático. C) Textos expositivos – Têm por finalidade explicar
e) os magistrados brasileiros exercem o poder constitucio- um assunto ou uma determinada situação que se
nal que lhes é atribuído em nome do governo federal. almeje desenvolvê-la, enfatizando acerca das ra-
zões de ela acontecer, como em: O cadastramento
Resposta: Letra A. A questão deve ser respondida se- irá se prorrogar até o dia 02 de dezembro, portanto,
gundo o texto: (...) “Todo o poder emana do povo, que não se esqueça de fazê-lo, sob pena de perder o be-
o exerce por meio de representantes eleitos ou direta- nefício.
mente, nos termos desta Constituição.” Em virtude desse D) Textos injuntivos (instrucional) – Trata-se de
comando, afirma-se que o poder dos juízes emana do uma modalidade na qual as ações são prescritas de
povo e em seu nome é exercido (...). forma sequencial, utilizando-se de verbos expres-
sos no imperativo, infinitivo ou futuro do presente:
3. (PCJ-MT – DELEGADO SUBSTITUTO – SUPERIOR Misture todos os ingrediente e bata no liquidificador
– CESPE – 2017 – ADAPTADA) No texto CG1A1BBB, o até criar uma massa homogênea.
vocábulo ‘emana’ foi empregado com o sentido de E) Textos argumentativos (dissertativo) – Demar-
cam-se pelo predomínio de operadores argumen-
a) trata. tativos, revelados por uma carga ideológica cons-
b) provém. tituída de argumentos e contra-argumentos que
c) manifesta. justificam a posição assumida acerca de um deter-
d) pertence. minado assunto: A mulher do mundo contemporâ-
e) cabe. neo luta cada vez mais para conquistar seu espaço
no mercado de trabalho, o que significa que os gê-
Resposta: Letra B. Dentro do contexto, “emana” tem o neros estão em complementação, não em disputa.
sentido de “provém”.
2. Gêneros Textuais
TIPOLOGIA E GÊNERO TEXTUAL

A todo o momento nos deparamos com vários textos, São os textos materializados que encontramos em
sejam eles verbais ou não verbais. Em todos há a presen- nosso cotidiano; tais textos apresentam características
ça do discurso, isto é, a ideia intrínseca, a essência daquilo sócio-comunicativas definidas por seu estilo, função,
que está sendo transmitido entre os interlocutores. Estes composição, conteúdo e canal. Como exemplos, temos:
interlocutores são as peças principais em um diálogo ou receita culinária, e-mail, reportagem, monografia, poema,
em um texto escrito. editorial, piada, debate, agenda, inquérito policial, fórum,
É de fundamental importância sabermos classificar os blog, etc.
textos com os quais travamos convivência no nosso dia a A escolha de um determinado gênero discursivo de-
dia. Para isso, precisamos saber que existem tipos textuais pende, em grande parte, da situação de produção, ou
e gêneros textuais. seja, a finalidade do texto a ser produzido, quem são os
Comumente relatamos sobre um acontecimento, um locutores e os interlocutores, o meio disponível para vei-
fato presenciado ou ocorrido conosco, expomos nossa opi- cular o texto, etc.
nião sobre determinado assunto, descrevemos algum lugar Os gêneros discursivos geralmente estão ligados a
PORTUGUÊS

que visitamos, fazemos um retrato verbal sobre alguém que esferas de circulação. Assim, na esfera jornalística, por
acabamos de conhecer ou ver. É exatamente nessas situa- exemplo, são comuns gêneros como notícias, reporta-
ções corriqueiras que classificamos os nossos textos naquela gens, editoriais, entrevistas e outros; na esfera de divul-
tradicional tipologia: Narração, Descrição e Dissertação. gação científica são comuns gêneros como verbete de

3
dicionário ou de enciclopédia, artigo ou ensaio científico, sendo, portanto, um texto informativo.
seminário, conferência. Argumentativo: Os textos argumentativos, ao contrá-
rio, têm por finalidade principal persuadir o leitor sobre o
REFERÊNCIAS BIBLIOGRÁFICAS ponto de vista do autor a respeito do assunto. Quando o
Português linguagens: volume 1 / Wiliam Roberto texto, além de explicar, também persuade o interlocutor
Cereja, Thereza Cochar Magalhães. – 7.ª ed. Reform. – e modifica seu comportamento, temos um texto disser-
São Paulo: Saraiva, 2010. tativo-argumentativo.
Português – Literatura, Produção de Textos & Gra- Exemplos: texto de opinião, carta do leitor, carta de
mática – volume único / Samira Yousseff Campedelli, solicitação, deliberação informal, discurso de defesa e
Jésus Barbosa Souza. – 3.ª ed. – São Paulo: Saraiva, 2002. acusação (advocacia), resenha crítica, artigos de opinião
ou assinados, editorial.
SITE Exposição: Apresenta informações sobre assuntos,
http://www.brasilescola.com/redacao/tipologia-tex- expõe ideias; explica, avalia, reflete. (analisa ideias). Es-
tual.htm trutura básica; ideia principal; desenvolvimento; conclu-
são. Uso de linguagem clara. Exemplo: ensaios, artigos
Observação: Não foram encontradas questões abran- científicos, exposições.
gendo tal conteúdo. Injunção: Indica como realizar uma ação. É também
utilizado para predizer acontecimentos e comportamen-
tos. Utiliza linguagem objetiva e simples. Os verbos são,
ESTUDO DE TEXTO(S)
na sua maioria, empregados no modo imperativo. Há
também o uso do futuro do presente. Exemplo: Receita
Texto Literário: expressa a opinião pessoal do autor
de um bolo e manuais.
que também é transmitida através de figuras, impreg-
Diálogo: é uma conversação estabelecida entre duas
nado de subjetivismo. Exemplo: um romance, um conto, ou mais pessoas. Pode conter marcas da linguagem oral,
uma poesia... (Conotação, Figurado, Subjetivo, Pessoal). como pausas e retomadas.
Texto Não-Literário: preocupa-se em transmitir uma Entrevista: é uma conversação entre duas ou mais
mensagem da forma mais clara e objetiva possível. Exem- pessoas (o entrevistador e o entrevistado), na qual per-
plo: uma notícia de jornal, uma bula de medicamento. guntas são feitas pelo entrevistador para obter informa-
(Denotação, Claro, Objetivo, Informativo). ção do entrevistado. Os repórteres entrevistam as suas
O objetivo do texto é passar conhecimento para o fontes para obter declarações que validem as informa-
leitor. Nesse tipo textual, não se faz a defesa de uma ções apuradas ou que relatem situações vividas por
ideia. Exemplos de textos explicativos são os encontra- personagens. Antes de ir para a rua, o repórter recebe
dos em manuais de instruções. uma pauta que contém informações que o ajudarão a
Informativo: Tem a função de informar o leitor a res- construir a matéria. Além das informações, a pauta su-
peito de algo ou alguém, é o texto de uma notícia de gere o enfoque a ser trabalhado assim como as fontes a
jornal, de revista, folhetos informativos, propagandas. serem entrevistadas. Antes da entrevista o repórter cos-
Uso da função referencial da linguagem, 3ª pessoa do tuma reunir o máximo de informações disponíveis sobre
singular. o assunto a ser abordado e sobre a pessoa que será en-
Descrição: Um texto em que se faz um retrato por es- trevistada. Munido deste material, ele formula perguntas
crito de um lugar, uma pessoa, um animal ou um objeto. que levem o entrevistado a fornecer informações novas
A classe de palavras mais utilizada nessa produção é o e relevantes. O repórter também deve ser perspicaz para
adjetivo, pela sua função caracterizadora. Numa aborda- perceber se o entrevistado mente ou manipula dados
gem mais abstrata, pode-se até descrever sensações ou nas suas respostas, fato que costuma acontecer princi-
sentimentos. Não há relação de anterioridade e posterio- palmente com as fontes oficiais do tema. Por exemplo,
ridade. Significa “criar” com palavras a imagem do objeto quando o repórter vai entrevistar o presidente de uma
descrito. É fazer uma descrição minuciosa do objeto ou instituição pública sobre um problema que está a afe-
da personagem a que o texto se refere. tar o fornecimento de serviços à população, ele tende a
Narração: Modalidade em que se conta um fato, fic- evitar as perguntas e a querer reverter a resposta para o
tício ou não, que ocorreu num determinado tempo e lu- que considera positivo na instituição. É importante que o
gar, envolvendo certos personagens. Refere-se a objetos repórter seja insistente. O entrevistador deve conquistar
do mundo real. Há uma relação de anterioridade e pos- a confiança do entrevistado, mas não tentar dominá-lo,
terioridade. O tempo verbal predominante é o passado. nem ser por ele dominado. Caso contrário, acabará indu-
Estamos cercados de narrações desde as que nos contam zindo as respostas ou perdendo a objetividade.
histórias infantis, como o “Chapeuzinho Vermelho” ou a As entrevistas apresentam com frequência alguns
“Bela Adormecida”, até as picantes piadas do cotidiano. sinais de pontuação como o ponto de interrogação, o
Dissertação: Dissertar é o mesmo que desenvolver travessão, aspas, reticências, parêntese e as vezes col-
ou explicar um assunto, discorrer sobre ele. Assim, o chetes, que servem para dar ao leitor maior informações
texto dissertativo pertence ao grupo dos textos expositi- que ele supostamente desconhece. O título da entrevista
PORTUGUÊS

vos, juntamente com o texto de apresentação científica, é um enunciado curto que chama a atenção do leitor e
o relatório, o texto didático, o artigo enciclopédico. Em resume a ideia básica da entrevista. Pode estar todo em
princípio, o texto dissertativo não está preocupado com letra maiúscula e recebe maior destaque da página. Na
a persuasão e sim, com a transmissão de conhecimento, maioria dos casos, apenas as preposições ficam com a

4
letra minúscula. O subtítulo introduz o objetivo principal O que é linguagem?
da entrevista e não vem seguido de ponto final. É um É o uso da língua como forma de expressão e comuni-
pequeno texto e vem em destaque também. A fotografia cação entre as pessoas. Agora, a linguagem não é somen-
do entrevistado aparece normalmente na primeira pá- te um conjunto de palavras faladas ou escritas, mas tam-
gina da entrevista e pode estar acompanhada por uma bém de gestos e imagens. Afinal, não nos comunicamos
frase dita por ele. As frases importantes ditas pelo entre- apenas pela fala ou escrita, não é verdade?
vistado e que aparecem em destaque nas outras páginas Então, a linguagem pode ser verbalizada, e daí vem a
da entrevista são chamadas de “olho”. analogia ao verbo. Você já tentou se pronunciar sem uti-
Crônica: Assim como a fábula e o enigma, a crônica lizar o verbo? Se não, tente, e verá que é impossível se ter
é um gênero narrativo. Como diz a origem da palavra algo fundamentado e coerente! Assim, a linguagem verbal
(Cronos é o deus grego do tempo), narra fatos históricos é que se utiliza de palavras quando se fala ou quando se
em ordem cronológica, ou trata de temas da atualidade. escreve.
Mas não é só isso. Lendo esse texto, você conhecerá as A linguagem pode ser não verbal, ao contrário da ver-
principais características da crônica, técnicas de sua reda- bal, não se utiliza do vocábulo, das palavras para se comu-
ção e terá exemplos. nicar. O objetivo, neste caso, não é de expor verbalmente
Uma das mais famosas crônicas da história da lite- o que se quer dizer ou o que se está pensando, mas se uti-
ratura luso-brasileira corresponde à definição de crônica lizar de outros meios comunicativos, como: placas, figuras,
como “narração histórica”. É a “Carta de Achamento do gestos, objetos, cores, ou seja, dos signos visuais.
Brasil”, de Pero Vaz de Caminha”, na qual são narrados Vejamos: um texto narrativo, uma carta, o diálogo,
ao rei português, D. Manuel, o descobrimento do Brasil uma entrevista, uma reportagem no jornal escrito ou tele-
e como foram os primeiros dias que os marinheiros por- visionado, um bilhete? Linguagem verbal!
tugueses passaram aqui. Mas trataremos, sobretudo, da Agora: o semáforo, o apito do juiz numa partida de
crônica como gênero que comenta assuntos do dia a dia. futebol, o cartão vermelho, o cartão amarelo, uma dança,
Para começar, uma crônica sobre a crônica, de Machado o aviso de “não fume” ou de “silêncio”, o bocejo, a identi-
de Assis: ficação de “feminino” e “masculino” através de figuras na
porta do banheiro, as placas de trânsito? Linguagem não
1. O nascimento da crônica verbal!
A linguagem pode ser ainda verbal e não verbal ao
“Há um meio certo de começar a crônica por uma tri- mesmo tempo, como nos casos das charges, cartoons e
vialidade. É dizer: Que calor! Que desenfreado calor! Diz- anúncios publicitários.
-se isto, agitando as pontas do lenço, bufando como um Observe alguns exemplos:
touro, ou simplesmente sacudindo a sobrecasaca. Resva-
la-se do calor aos fenômenos atmosféricos, fazem-se al-
gumas conjeturas acerca do sol e da lua, outras sobre a fe-
bre amarela, manda-se um suspiro a Petrópolis, e la glace
est rompue está começada a crônica. (...)
Machado de Assis. Crônicas Escolhidas. São Paulo:
Editora Ática, 1994.

Publicada em jornal ou revista onde é publicada, des-


tina-se à leitura diária ou semanal e trata de acontecimen-
tos cotidianos. A crônica se diferencia no jornal por não
buscar exatidão da informação. Diferente da notícia, que Cartão vermelho – denúncia de falta grave no futebol.
procura relatar os fatos que acontecem, a crônica os ana-
lisa, dá-lhes um colorido emocional, mostrando aos olhos
do leitor uma situação comum, vista por outro ângulo,
singular.
O leitor pressuposto da crônica é urbano e, em princí-
pio, um leitor de jornal ou de revista. A preocupação com
esse leitor é que faz com que, dentre os assuntos tratados,
o cronista dê maior atenção aos problemas do modo de
vida urbano, do mundo contemporâneo, dos pequenos
acontecimentos do dia a dia comuns nas grandes cidades.
Jornalismo e literatura: É assim que podemos dizer
que a crônica é uma mistura de jornalismo e literatura. De
um recebe a observação atenta da realidade cotidiana e
do outro, a construção da linguagem, o jogo verbal. Al- Charge do autor Tacho – exemplo de linguagem ver-
gumas crônicas são editadas em livro, para garantir sua bal (óxente, polo Norte 2100) e não verbal (imagem: sol,
PORTUGUÊS

durabilidade no tempo. cactos, pinguim).

5
1. Linguagem como forma de Ação e Interação

A concepção da linguagem ainda é bastante debatida


entre os linguistas, que até hoje ainda não chegaram a um
consenso da sua concepção. Ela é sintetizada de três ma-
neiras diferentes, sendo que o modo que mais utilizamos
em nossos relacionamentos com outras pessoas é a de
ação e interação.
Neste modo, a linguagem funciona como uma “ativi-
dade” de ação/interação entre os envolvidos nesta comu-
nicação. Os interlocutores expõem algo ao outro para que
este seja induzido a interagir na conversa, que pode ser
Placas de trânsito – à frente “proibido andar de bici- verbal, não verbal, mista e digital.
cleta”, atrás “quebra-molas”. A atividade de comunicação é indispensável ao ser hu-
mano e aos animais. Existem diversos meios de comuni-
cação:
→ Entre os animais: a dança das abelhas, os odores, as
produções vocais (como no caso das aves)
→ Entre os homens: a dança, a pintura, a mímica, os
gestos, os sinais de trânsito, os símbolos, a linguagem
dos surdos-mudos, a dos deficientes visuais, a linguagem
computacional, a linguagem matemática, as línguas natu-
rais etc.
Símbolo que se coloca na porta para indicar “sanitá-
De um modo geral, dá-se o nome de linguagem a to-
rio masculino”.
dos os meios de comunicação: linguagem animal e lin-
guagem humana, em linguagem não verbal e linguagem
verbal. O termo é, pois, empregado à aptidão humana de
associar os sons produzidos pelo aparelho fonador huma-
no a um conteúdo significativo e utilizar o resultado dessa
associação para a interação verbal. Fala-se, pois, em lin-
guagem verbal.
É um termo muito amplo: as línguas naturais (portu-
guês, inglês, por exemplo) são manifestações desse algo
mais geral. Saussure, o linguista genebrino, concebia a lin-
guagem em duas partes: a língua e a fala, e era a primeira
o seu objeto de estudo; embora, reconhecesse a interde-
pendência entre elas.
Enquanto sistema de signos, a linguagem é um códi-
go - um conjunto de signos sujeitos a regras de combi-
nação e utilizado na produção e na compreensão de uma
mensagem.
Imagem indicativa de “silêncio”.
Signo é compreendido por:
- Significante, veículo do significado (parte percep-
tível, sensível);
- Significado, o que se entende quando se usa o
signo (parte inteligível).

Os linguistas compreendem que há no processo de


comunicação seis elementos:
- Emissor (remetente), envia a mensagem;
- Receptor (destinatário), recebe a mensagem;
- Mensagem - informação veiculada;
- Código, sistema de signos utilizados para codifi-
Semáforo com sinal amarelo advertindo “atenção”. car a mensagem;
- Contexto (referente), aquilo a que a mensagem
se refere;
PORTUGUÊS

- Contato (canal), veículo, meio físico utilizado para


transmitir a mensagem.

1.1. Concepção de linguagem

6
De acordo com o prof. Luiz C. Travaglia, no seu li- No ato de comunicação, percebemos a existência de
vro Gramática e Interação, admite-se para a linguagem alguns elementos, são eles:
admite três concepções: ▪ Emissor: é aquele que envia a mensagem (pode
ser uma única pessoa ou um grupo de pessoas).
1.2. Linguagem como expressão do pensamento ▪ Mensagem: é o conteúdo (assunto) das informa-
ções que ora são transmitidas.
Se a linguagem é expressão do pensamento, quando ▪ Receptor: é aquele a quem a mensagem é ende-
as pessoas não se expressam bem é porque não sabem reçada (um indivíduo ou um grupo), também conhecido
elaborar o pensamento. Se o enunciador expressa o que como destinatário.
pensa, sua fala é resultado da sua maneira própria de or- ▪ Canal de comunicação: é o meio pelo qual a
ganizar as suas ideias. O texto, dessa forma, nada tem a mensagem é transmitida.
ver com o leitor ou com quem se fala, e sim, somente com ▪ Código: é o conjunto de signos e de regras de
o enunciador. Nessa linha de pensamento encontra-se a combinação desses signos utilizados para elaborar a
gramática normativa ou tradicional. mensagem: o emissor codifica aquilo que o receptor irá
decodificar.
1.3. Linguagem como instrumento de comunicação ▪ Contexto: é o objeto ou a situação a que a men-
sagem se refere.
Neste conceito a língua é vista como um código, que
deve ser dominado pelos falantes para que as comuni- Partindo desses seis elementos, Roman Jakobson,
cações sejam efetivadas. A comunicação, pois, depende linguista russo, elaborou estudos acerca das funções da
do grau de domínio que o falante tem da língua como linguagem, os quais são muito úteis para a análise e pro-
sistema. O falante utiliza-se dos conceitos estruturais que dução de textos. As seis funções são:
conhece para expressar o pensamento; o ouvinte decodi- 1. Função referencial: referente é o objeto ou situa-
fica os sinais codificados por ele e transforma-os em nova ção de que a mensagem trata. A função referencial pri-
mensagem. Essa linha de pensamento pertence ao estru- vilegia justamente o referente da mensagem, buscando
turalismo e também ao gerativismo. transmitir informações objetivas sobre ele. Essa função
predomina nos textos de caráter científico e é privilegia-
1.4. Linguagem como forma ou processo de inte- do nos textos jornalísticos.
ração 2. Função emotiva: através dessa função, o emissor
imprime no texto as marcas de sua atitude pessoal: emo-
Nesta concepção o falante realiza ações, age e intera- ções, avaliações, opiniões. O leitor sente no texto a pre-
ge com o outro (com quem ele fala). Dessa forma, a lin- sença do emissor.
guagem toma uma dimensão mais ampla e não uniforme, Exemplo: “[…] Mas quem sou eu para censurar os
pois inserindo num contexto ideológico e sociocultural, culpados? O pior é que preciso perdoá-los. É necessário
ela não tem direção preestabelecida vai depender uni- chegar a tal nada que indiferentemente se ame ou não se
camente da interação entre os dois sujeitos. Fazem parte ame o criminoso que nos mata. Mas não estou seguro de
dessa corrente a Teoria do Discurso, Linguística Textual, mim mesmo: preciso amar aquele que me trucida e per-
Semântica Argumentativa, Análise do Discurso, Análise guntar quem de vós me trucida. E minha vida, mais forte
da Conversação. do que eu, responde que quer porque quer vingança e
responde que devo lutar como quem se afoga, mesmo
1.5. As funções da linguagem que eu morra depois. Se assim for, que assim seja [...]”.
(Fragmento de A hora da estrela, de Clarice Lispector)
Para entendermos com clareza as funções da lingua-
gem, é bom primeiramente conhecermos as etapas da 3. Função conativa: essa função procura organizar
comunicação. o texto de forma que se imponha sobre o receptor da
Ao contrário do que muitos pensam, a comunicação mensagem, persuadindo-o, seduzindo-o. Nas mensa-
não acontece somente quando falamos, estabelecemos gens em que predomina essa função, busca-se envolver
um diálogo ou redigimos um texto, ela se faz presente o leitor com o conteúdo transmitido, levando-o a adotar
em todos (ou quase todos) os momentos. este ou aquele comportamento. Nos tipos de textos em
Comunicamo-nos com nossos colegas de trabalho, que a função conativa predomina, é possível perceber o
com o livro que lemos, com a revista, com os documen- uso da 2ª pessoa como maneira de interpelar alguém,
tos que manuseamos, através de nossos gestos, ações, além do emprego dos verbos no imperativo para con-
até mesmo através de um beijo de “boa-noite”. vencer o interlocutor:
É o que diz Bordenave quando se refere à comuni-
cação: “A comunicação confunde-se com a própria vida.
Temos tanta consciência de que comunicamos como de
que respiramos ou andamos. Somente percebemos a
sua essencial importância quando, por acidente ou uma
PORTUGUÊS

doença, perdemos a capacidade de nos comunicar. (Bor-


denave, 1986. p.17-9)”

7
Exemplo: Exemplo:

4. Função fática: a palavra fática significa “ruído, ru-


mor”. Foi utilizada inicialmente para designar cer-
tas formas usadas para chamar a atenção (ruídos
como psiu, ahn, ei). Essa função ocorre quando a
mensagem se orienta sobre o canal de comuni-
cação ou contato, buscando verificar e fortalecer
sua eficiência. Tipo de mensagem cujo objetivo é
prolongar ou interromper uma conversa. Nela, o 6. Função poética: quando a mensagem é elaborada
emissor utiliza procedimentos para manter contato de forma inovadora e imprevista, utilizando com-
físico ou psicológico com o interlocutor: binações sonoras ou rítmicas, jogos de imagem ou
de ideias, temos a manifestação da função poética
Exemplo: da linguagem. Essa função é capaz de despertar no
“(...) Olá, como vai? leitor prazer estético e surpresa. É muito encontra-
Eu vou indo e você, tudo bem? da na Literatura, especialmente na poesia, a função
Tudo bem eu vou indo correndo poética apresenta um texto no qual a função está
Pegar meu lugar no futuro, e você? centrada na própria mensagem, rompendo com o
Tudo bem, eu vou indo em busca modo tradicional com o vemos a linguagem.
De um sono tranquilo, quem sabe …
Quanto tempo... pois é... Exemplo:
Quanto tempo...
Me perdoe a pressa
É a alma dos nossos negócios
Oh! Não tem de quê
Eu também só ando a cem
Quando é que você telefona?
Precisamos nos ver por aí (…)”.
(Trecho da música Sinal Fechado, de Paulinho da Viola).

5. Função metalinguística: quando a linguagem se


volta sobre si mesma, transformando-se em seu
próprio referente, ocorre a função metalinguística.
Linguagem utilizada para falar, explicar ou descrever
o próprio código: esse é o principal objetivo da
função metalinguística. Nas situações em que ela é
empregada, geralmente na poesia e na publicida-
de, a atenção está voltada para o próprio código:
PORTUGUÊS

Essas funções não são exploradas isoladamente; de


modo geral, ocorre a superposição de várias delas. Há,
no entanto, aquela que se sobressai, assim podemos
identificar a finalidade principal do texto.

8
Variações Linguísticas tos grupos, como os surfistas, cantores de rap, tatuado-
res, entre outros.
A linguagem é a característica que nos difere dos de- Os jargões estão relacionados ao profissionalismo,
mais seres, permitindo-nos a oportunidade de expressar caracterizando um linguajar técnico. Representando a
sentimentos, revelar conhecimentos, expor nossa opinião classe, podemos citar os médicos, advogados, profissio-
frente aos assuntos relacionados ao nosso cotidiano, e, so- nais da área de informática, dentre outros.
bretudo, promovendo nossa inserção ao convívio social. Vejamos um poema e o trecho de uma música para
E dentre os fatores que a ela se relacionam desta- entendermos melhor sobre o assunto:
cam-se os níveis da fala, que são basicamente dois: “Vício na fala
- O nível de formalidade; Para dizerem milho dizem mio
- O de informalidade. Para melhor dizem mió
Para pior pió
O padrão formal está diretamente ligado à lingua- Para telha dizem teia
gem escrita, restringindo-se às normas gramaticais de Para telhado dizem teiado
um modo geral. Razão pela qual nunca escrevemos da E vão fazendo telhados”.
mesma maneira que falamos. Este fator foi determinante (Oswald de Andrade).
para a que a mesma pudesse exercer total soberania so-
bre as demais. Chopis Centis
Quanto ao nível informal, este por sua vez represen- “Eu “di” um beijo nela
ta o estilo considerado “de menor prestígio”, e isto tem E chamei pra passear.
gerado controvérsias entre os estudos da língua, uma vez A gente fomos no shopping
que para a sociedade, aquela pessoa que fala ou escreve Pra “mode” a gente lanchar.
de maneira errônea é considerada “inculta”, tornando-se Comi uns bicho estranho, com um tal de gergelim.
desta forma um estigma. Até que “tava” gostoso, mas eu prefiro aipim.
Compondo o quadro do padrão informal da lingua- Quanta gente, Quanta alegria, A minha felicidade é
gem, estão as chamadas variedades linguísticas, as quais um crediário nas Casas Bahia.
representam as variações de acordo com as condições Esse tal Chopis Centis é muito legalzinho.
sociais, culturais, regionais e históricas em que é utiliza- Pra levar a namorada e dar uns “rolezinho”,
da. Dentre elas destacam-se: Quando eu estou no trabalho,
Não vejo a hora de descer dos andaime.
a) Variações históricas Pra pegar um cinema, ver Schwarzneger,
Dado o dinamismo que a língua apresenta, a mesma E também o Van Damme.
sofre transformações ao longo do tempo. Um exemplo (Dinho e Júlio Rasec, encarte CD Mamonas Assassi-
bastante representativo é a questão da ortografia, se nas, 1995).
levarmos em consideração a palavra farmácia, uma vez
que a mesma era grafada com “ph”, contrapondo-se à Para expandir a capacidade de compreensão e, prin-
linguagem dos internautas, a qual fundamenta-se pela cipalmente, de interpretação, é importante acostumar-se
supressão dos vocábulos. à leitura, seja de um texto ou um objeto, figura ou fato.
Analisemos, pois, o fragmento exposto: Ao se ler algo, deve-se notar aspectos particulares
Antigamente que permitam associar o elemento da leitura ao tempo
“Antigamente, as moças chamavam-se mademoisel- e a acontecimentos, destacar o essencial e o secundá-
les e eram todas mimosas e muito prendadas. Não fa- rio relacionando-o a outros já lidos. Semanticamente, o
ziam anos: completavam primaveras, em geral dezoito. elemento da leitura encerra em si todas as indagações,
Os janotas, mesmo sendo rapagões, faziam-lhes pé-de- permitindo uma análise e oferecendo os subsídios da
-alferes, arrastando a asa, mas ficavam longos meses de- resposta. Compreender um texto é entender o seu senti-
baixo do balaio.” Carlos Drummond de Andrade do; apreender a situação, o fato, a narração, a tese a nós
Comparando-o à modernidade, percebemos um vo- expostos. Interpretar um texto é conseguir desenvolver
cabulário antiquado. em outras palavras a ideia do texto, é, portanto, parafra-
seá-lo ou reescrevê-lo. Para se interpretar bem um texto,
b) Variações regionais é de suma importância uma boa leitura. Para se entender
São os chamados dialetos, que são as marcas deter- um texto é necessária uma leitura atenta, em que se no-
minantes referentes a diferentes regiões. Como exemplo, tem as suas sutilezas e superficialidades. É conveniente
citamos a palavra mandioca que, em certos lugares, rece- marcar as ideias principais e estar atento as entrelinhas,
be outras nomenclaturas, tais como: macaxeira e aipim. aos detalhes e a todo o contexto.
Figurando também esta modalidade estão os sotaques, - Paráfrase: é o desenvolvimento ou citação de um
ligados às características orais da linguagem. texto ou parte dele, com palavras diferentes, mas
c) Variações sociais ou culturais de igual significação. Não há alteração da ideia
Estão diretamente ligadas aos grupos sociais de uma central.
PORTUGUÊS

maneira geral e também ao grau de instrução de uma - Perífrase: é um circunlóquio, um rodeio de pala-
determinada pessoa. Como exemplo, citamos as gírias, vras; a exposição de ideias é feito usando-se de
os jargões e o linguajar caipira. muitas palavras. Usam se mais palavras do que o
As gírias pertencem ao vocabulário específico de cer- texto original, isto é, usam-se mais palavras que o

9
necessário. LÍNGUA PADRÃO, LÍNGUA NÃO PADRÃO
- Intertextualidade: é a relação entre dois ou mais
textos, cujo tema seja o mesmo, porém tratado de A língua é um código de que se serve o homem para
forma diferente. elaborar mensagens, para se comunicar. Existem basica-
- Síntese: é uma resenha, que é feita utilizando-se mente duas modalidades de língua, ou seja, duas línguas
das palavras do texto. Aparecem apenas as ideias funcionais:
principais.
- Resumo: é uma representação do texto em que só A) a língua funcional de modalidade culta, língua
aparecem as ideias principais, não é necessário que culta ou língua-padrão, que compreende a língua literária,
seja com as mesmas palavras do texto. tem por base a norma culta, forma linguística utilizada pelo
- Inferência: é uma informação que não está no tex- segmento mais culto e influente de uma sociedade. Cons-
to, mas sim fora dele; está nas entrelinhas ou exige titui, em suma, a língua utilizada pelos veículos de comu-
um conhecimento extra textual. nicação de massa (emissoras de rádio e televisão, jornais,
- Tipologia Textual Descrição: ato de descrever ca- revistas, painéis, anúncios, etc.), cuja função é a de serem
racterísticas. Pode ser: Objetiva: descrição com aliados da escola, prestando serviço à sociedade, colabo-
caráter universal. Subjetiva: descrição com caráter rando na educação;
particular, pessoal.
- Técnica: descrição com caráter próprio de um ofí- B) a língua funcional de modalidade popular; língua
cio, profissão. popular ou língua cotidiana, que apresenta gradações as
- Narração: é o relato de um fato, de um aconteci- mais diversas, tem o seu limite na gíria e no calão.
mento. Seus elementos são:
- Fato: é o acontecimento; o encadeamento das 1. Norma culta
ações forma o enredo.
- Personagens: são os participantes do aconteci- A norma culta, forma linguística que todo povo civiliza-
do possui, é a que assegura a unidade da língua nacional.
mento. Principais (mais atuantes) Secundários
E justamente em nome dessa unidade, tão importante do
- Ambiente ou cenário: é o lugar onde ocorrem os
ponto de vista político--cultural, que é ensinada nas escolas
acontecimentos.
e difundida nas gramáticas. Sendo mais espontânea e cria-
- Tempo: é a localização cronológica do aconteci-
tiva, a língua popular afigura-se mais expressiva e dinâmica.
mento. Foco Narrativo (narrador): a narração pode
Temos, assim, à guisa de exemplificação:
ser em: 1ª pessoa: narrador-personagem 3ª pes-
Estou preocupado. (norma culta)
soa: narrador-onisciente/narrador-observador.
Tô preocupado. (língua popular)
Dissertação: ato de discorrer sobre um tema. A Tô grilado. (gíria, limite da língua popular)
dissertação divide-se em três partes: Introdução:
apresenta a ideia principal a ser discutida. Não basta conhecer apenas uma modalidade de língua;
- Desenvolvimento: é o desdobramento da ideia urge conhecer a língua popular, captando-lhe a esponta-
central, a exposição dos argumentos. neidade, expressividade e enorme criatividade, para viver;
- Conclusão: retoma ou resume os principais aspec- urge conhecer a língua culta para conviver.
tos do texto e confirma a tese inicial. Podemos, agora, definir gramática: é o estudo das nor-
d) Tipos de Discurso mas da língua culta.
- Discurso Direto: a fala do personagem é, geral- 2. O conceito de erro em língua
mente, acompanhada por um verbo de elocução (dizer,
falar, responder, perguntar, afirmar etc.), não havendo co- Em rigor, ninguém comete erro em língua, exceto nos
nectivo, porém uma pausa marcada por sinal de pontuação. casos de ortografia. O que normalmente se comete são
Exemplo.: A mãe perguntou-lhe atarantada: - Onde transgressões da norma culta. De fato, aquele que, num
você pensa que vai? momento íntimo do discurso, diz: “Ninguém deixou ele
falar”, não comete propriamente erro; na verdade, trans-
- Discurso Indireto: o personagem não fala com suas gride a norma culta.
palavras, é o narrador que reproduz com suas palavras o dis-
Um repórter, ao cometer uma transgressão em sua
curso do personagem. Os verbos de elocução são núcleos do
fala, transgride tanto quanto um indivíduo que compare-
predicado da oração principal seguido de oração subordinada.
ce a um banquete trajando xortes ou quanto um banhis-
Exemplo: Ele respondeu que sempre saía sozinho.
ta, numa praia, vestido de fraque e cartola.
- Indireto Livre: é um discurso misto, pois há carac- Releva considerar, assim, o momento do discurso, que
terísticas do discurso direto e do indireto. A fala do perso- pode ser íntimo, neutro ou solene. O momento íntimo é
nagem se insere no discurso do narrador, são perceptíveis o das liberdades da fala. No recesso do lar, na fala entre
aspectos psicológicos e fluxos do pensamento do persona- amigos, parentes, namorados, etc., portanto, são consi-
gem. deradas perfeitamente normais construções do tipo:
Exemplo: Naquele dia o rapaz havia se declarado à sua Eu não vi ela hoje.
PORTUGUÊS

vizinha. Ele já tinha sofrido muito. Custava à moça acreditar? Ninguém deixou ele falar.
Não sabia se tinha feito à coisa certa. Deixe eu ver isso!
Eu te amo, sim, mas não abuse!
Não assisti o filme nem vou assisti-lo.

10
Sou teu pai, por isso vou perdoá-lo. 3. Língua escrita e língua falada - Nível de lingua-
gem
Nesse momento, a informalidade prevalece sobre a
norma culta, deixando mais livres os interlocutores. A língua escrita, estática, mais elaborada e menos
O momento neutro é o do uso da língua-padrão, que econômica, não dispõe dos recursos próprios da língua
é a língua da Nação. Como forma de respeito, tomam-se falada.
por base aqui as normas estabelecidas na gramática, ou A acentuação (relevo de sílaba ou sílabas), a entoação
seja, a norma culta. Assim, aquelas mesmas construções (melodia da frase), as pausas (intervalos significativos no
se alteram: decorrer do discurso), além da possibilidade de gestos,
Eu não a vi hoje. olhares, piscadas, etc., fazem da língua falada a moda-
Ninguém o deixou falar. lidade mais expressiva, mais criativa, mais espontânea e
Deixe-me ver isso! natural, estando, por isso mesmo, mais sujeita a transfor-
Eu te amo, sim, mas não abuses! mações e a evoluções.
Não assisti ao filme nem vou assistir a ele. Nenhuma, porém, sobrepõe-se a outra em impor-
Sou seu pai, por isso vou perdoar-lhe. tância. Nas escolas, principalmente, costuma se ensinar
a língua falada com base na língua escrita, considerada
Considera-se momento neutro o utilizado nos veí- superior. Decorrem daí as correções, as retificações, as
culos de comunicação de massa (rádio, televisão, jornal, emendas, a que os professores sempre estão atentos.
revista, etc.). Daí o fato de não se admitirem deslizes ou Ao professor cabe ensinar as duas modalidades, mos-
transgressões da norma culta na pena ou na boca de trando as características e as vantagens de uma e outra,
jornalistas, quando no exercício do trabalho, que deve sem deixar transparecer nenhum caráter de superiorida-
refletir serviço à causa do ensino. de ou inferioridade, que em verdade inexiste.
O momento solene, acessível a poucos, é o da arte Isso não implica dizer que se deve admitir tudo na
língua falada. A nenhum povo interessa a multiplicação
poética, caracterizado por construções de rara beleza.
de línguas. A nenhuma nação convém o surgimento de
Vale lembrar, finalmente, que a língua é um costume.
dialetos, consequência natural do enorme distanciamen-
Como tal, qualquer transgressão, ou chamado erro, deixa
to entre uma modalidade e outra.
de sê-lo no exato instante em que a maioria absoluta
A língua escrita é, foi e sempre será mais bem-ela-
o comete, passando, assim, a constituir fato linguístico
borada que a língua falada, porque é a modalidade que
registro de linguagem definitivamente consagrado pelo
mantém a unidade linguística de um povo, além de ser
uso, ainda que não tenha amparo gramatical. Exemplos:
a que faz o pensamento atravessar o espaço e o tem-
Olha eu aqui! (Substituiu: Olha-me aqui!) po. Nenhuma reflexão, nenhuma análise mais detida será
Vamos nos reunir. (Substituiu: Vamo-nos reunir) possível sem a língua escrita, cujas transformações, por
Não vamos nos dispersar. (Substituiu: Não nos vamos isso mesmo, processam-se lentamente e em número
dispersar e Não vamos dispersar-nos) consideravelmente menor, quando cotejada com a mo-
Tenho que sair daqui depressinha. (Substituiu: Tenho dalidade falada.
de sair daqui bem depressa) Importante é fazer o educando perceber que o nível
O soldado está a postos. (Substituiu: O soldado está no da linguagem, a norma linguística, deve variar de acordo
seu posto) com a situação em que se desenvolve o discurso.
O ambiente sociocultural determina o nível da lingua-
As formas impeço, despeço e desimpeço, dos verbos gem a ser empregado. O vocabulário, a sintaxe, a pro-
impedir, despedir e desimpedir, respectivamente, são núncia e até a entoação variam segundo esse nível. Um
exemplos também de transgressões ou “erros” que se padre não fala com uma criança como se estivesse em
tornaram fatos linguísticos, já que só correm hoje por- uma missa, assim como uma criança não fala como um
que a maioria viu tais verbos como derivados de pedir, adulto. Um engenheiro não usará um mesmo discurso,
que tem início, na sua conjugação, com peço. Tanto bas- ou um mesmo nível de fala, para colegas e para pedrei-
tou para se arcaizarem as formas então legítimas impido, ros, assim como nenhum professor utiliza o mesmo nível
despido e desimpido, que hoje nenhuma pessoa bem-es- de fala no recesso do lar e na sala de aula.
colarizada tem coragem de usar. Existem, portanto, vários níveis de linguagem e, entre
Em vista do exposto, será útil eliminar do vocabulário esses níveis, destacam-se em importância o culto e o co-
escolar palavras como corrigir e correto, quando nos refe- tidiano, a que já fizemos referência.
rimos a frases. “Corrija estas frases” é uma expressão que
deve dar lugar a esta, por exemplo: “Converta estas frases
da língua popular para a língua culta”.
Uma frase correta não é aquela que se contrapõe a
uma frase “errada”; é, na verdade, uma frase elaborada
conforme as normas gramaticais; em suma, conforme a
norma culta.
PORTUGUÊS

11
RECONHECIMENTO DAS RELAÇÕES de lago lacustre
LÓGICO-DISCURSIVAS PRESENTES NO de leão leonino
TEXTO, MARCADAS POR CONJUNÇÕES, de lebre l eporino
ADVÉRBIOS, PREPOSIÇÕES
de lua lunar ou selênico
ARGUMENTATIVAS, LOCUÇÕES, ETC;
de madeira lígneo
de mestre magistral
Adjetivo de ouro áureo

É a palavra que expressa uma qualidade ou caracterís- de paixão passional


tica do ser e se relaciona com o substantivo, concordando de pâncreas pancreático
com este em gênero e número.
de porco suíno ou porcino
As praias brasileiras estão poluídas.
Praias = substantivo; brasileiras/poluídas = adjetivos dos quadris ciático
(plural e feminino, pois concordam com “praias”). de rio fluvial
1. Locução adjetiva de sonho onírico
Locução = reunião de palavras. Sempre que são neces- de velho senil
sárias duas ou mais palavras para falar sobre a mesma coisa,
tem-se locução. Às vezes, uma preposição + substantivo tem de vento eólico
o mesmo valor de um adjetivo: é a Locução Adjetiva (expres- de vidro vítreo ou hialino
são que equivale a um adjetivo). Por exemplo: aves da noite
de virilha inguinal
(aves noturnas), paixão sem freio (paixão desenfreada).
de visão óptico ou ótico
Observe outros exemplos:
Observação:
de águia aquilino Nem toda locução adjetiva possui um adjetivo corres-
pondente, com o mesmo significado: Vi as alunas da 5ª
de aluno discente série. / O muro de tijolos caiu.
de anjo angelical
2 Morfossintaxe do Adjetivo (Função Sintática):
de ano anual O adjetivo exerce sempre funções sintáticas (função
de aranha aracnídeo dentro de uma oração) relativas aos substantivos, atuan-
de boi bovino do como adjunto adnominal ou como predicativo (do
sujeito ou do objeto).
de cabelo capilar
de cabra caprino 3 Adjetivo Pátrio (ou gentílico)
Indica a nacionalidade ou o lugar de origem do ser.
de campo campestre ou rural Observe alguns deles:
de chuva pluvial
Estados e cidades brasileiras:
de criança pueril
de dedo digital
Alagoas alagoano
de estômago estomacal ou gástrico
Amapá amapaense
de falcão falconídeo
Aracaju aracajuano ou aracajuense
de farinha farináceo
Amazonas amazonense ou baré
de fera ferino
Belo Horizonte belo-horizontino
de ferro férreo
Brasília brasiliense
de fogo ígneo
Cabo Frio cabo-friense
de garganta gutural
Campinas campineiro ou campinense
de gelo glacial
de guerra bélico 4 Adjetivo Pátrio Composto
de homem viril ou humano
PORTUGUÊS

Na formação do adjetivo pátrio composto, o primei-


de ilha insular ro elemento aparece na forma reduzida e, normalmente,
de inverno hibernal ou invernal erudita. Observe alguns exemplos:

12
África afro- / Cultura afro-americana
Alemanha germano- ou teuto-/Competições teuto-inglesas
América américo- / Companhia américo-africana
Bélgica belgo- / Acampamentos belgo-franceses
China sino- / Acordos sino-japoneses
Espanha hispano- / Mercado hispano-português
Europa euro- / Negociações euro-americanas
França franco- ou galo- / Reuniões franco-italianas
Grécia greco- / Filmes greco-romanos
Inglaterra anglo- / Letras anglo-portuguesas
Itália ítalo- / Sociedade ítalo-portuguesa
Japão nipo- / Associações nipo-brasileiras
Portugal luso- / Acordos luso-brasileiros

5 Flexão dos adjetivos

O adjetivo varia em gênero, número e grau.

6. Gênero dos Adjetivos

Os adjetivos concordam com o substantivo a que se referem (masculino e feminino). De forma semelhante aos
substantivos, classificam-se em:

A) Biformes - têm duas formas, sendo uma para o masculino e outra para o feminino: ativo e ativa, mau e má.
Se o adjetivo é composto e biforme, ele flexiona no feminino somente o último elemento: o moço norte-americano,
a moça norte-americana.
Exceção: surdo-mudo e surda-muda.

B) Uniformes - têm uma só forma tanto para o masculino como para o feminino: homem feliz e mulher feliz.
Se o adjetivo é composto e uniforme, fica invariável no feminino: conflito político-social e desavença político-social.

7 Número dos Adjetivos

A) Plural dos adjetivos simples


Os adjetivos simples se flexionam no plural de acordo com as regras estabelecidas para a flexão numérica dos subs-
tantivos simples: mau e maus, feliz e felizes, ruim e ruins, boa e boas.
Caso o adjetivo seja uma palavra que também exerça função de substantivo, ficará invariável, ou seja, se a palavra
que estiver qualificando um elemento for, originalmente, um substantivo, ela manterá sua forma primitiva. Exemplo: a
palavra cinza é, originalmente, um substantivo; porém, se estiver qualificando um elemento, funcionará como adjetivo.
Ficará, então, invariável. Logo: camisas cinza, ternos cinza.
Motos vinho (mas: motos verdes)
Paredes musgo (mas: paredes brancas).
Comícios monstro (mas: comícios grandiosos).

B) Adjetivo Composto
É aquele formado por dois ou mais elementos. Normalmente, esses elementos são ligados por hífen. Apenas o
último elemento concorda com o substantivo a que se refere; os demais ficam na forma masculina, singular. Caso um
dos elementos que formam o adjetivo composto seja um substantivo adjetivado, todo o adjetivo composto ficará in-
variável. Por exemplo: a palavra “rosa” é, originalmente, um substantivo, porém, se estiver qualificando um elemento,
funcionará como adjetivo. Caso se ligue a outra palavra por hífen, formará um adjetivo composto; como é um substan-
tivo adjetivado, o adjetivo composto inteiro ficará invariável. Veja:
Camisas rosa-claro.
PORTUGUÊS

Ternos rosa-claro.
Olhos verde-claros.
Calças azul-escuras e camisas verde-mar.
Telhados marrom-café e paredes verde-claras.

13
Observação:  Sintética: nessa, há o acréscimo de sufixos. Por
Azul-marinho, azul-celeste, ultravioleta e qualquer exemplo: O concurseiro é esforçadíssimo.
adjetivo composto iniciado por “cor-de-...” são sempre
invariáveis: roupas azul-marinho, tecidos azul-celeste, Observe alguns superlativos sintéticos:
vestidos cor-de-rosa.
O adjetivo composto surdo-mudo tem os dois ele-
benéfico - beneficentíssimo
mentos flexionados: crianças surdas-mudas.
bom - boníssimo ou ótimo
8 Grau do Adjetivo comum - comuníssimo
Os adjetivos se flexionam em grau para indicar a in- cruel - crudelíssimo
tensidade da qualidade do ser. São dois os graus do ad- difícil - dificílimo
jetivo: o comparativo e o superlativo.
doce - dulcíssimo
A) Comparativo fácil - facílimo
Nesse grau, comparam-se a mesma característica
fiel - fidelíssimo
atribuída a dois ou mais seres ou duas ou mais caracte-
rísticas atribuídas ao mesmo ser. O comparativo pode ser
de igualdade, de superioridade ou de inferioridade. B.2 Superlativo Relativo: ocorre quando a qualidade
Sou tão alto como você. = Comparativo de Igualdade de um ser é intensificada em relação a um conjunto de
No comparativo de igualdade, o segundo termo da seres. Essa relação pode ser:
comparação é introduzido pelas palavras como, quanto  De Superioridade: Essa matéria é a mais fácil de
ou quão. todas.
 De Inferioridade: Essa matéria é a menos fácil de
Sou mais alto (do) que você. = Comparativo de Su- todas.
perioridade
O superlativo absoluto analítico é expresso por meio
Sílvia é menos alta que Tiago. = Comparativo de In- dos advérbios muito, extremamente, excepcionalmente,
ferioridade antepostos ao adjetivo.
O superlativo absoluto sintético se apresenta sob
Alguns adjetivos possuem, para o comparativo de duas formas: uma erudita - de origem latina – e outra
superioridade, formas sintéticas, herdadas do latim. São popular - de origem vernácula. A forma erudita é cons-
eles: bom /melhor, pequeno/menor, mau/pior, alto/supe- tituída pelo radical do adjetivo latino + um dos sufixos
rior, grande/maior, baixo/inferior. -íssimo, -imo ou érrimo: fidelíssimo, facílimo, paupérrimo;
a popular é constituída do radical do adjetivo português
Observe que: + o sufixo -íssimo: pobríssimo, agilíssimo.
 As formas menor e pior são comparativos de su- Os adjetivos terminados em –io fazem o superlativo
perioridade, pois equivalem a mais pequeno e mais com dois “ii”: frio – friíssimo, sério – seriíssimo; os termi-
mau, respectivamente. nados em –eio, com apenas um “i”: feio - feíssimo, cheio
 Bom, mau, grande e pequeno têm formas sintéticas – cheíssimo.
(melhor, pior, maior e menor), porém, em compa-
rações feitas entre duas qualidades de um mesmo REFERÊNCIAS BIBLIOGRÁFICAS
elemento, deve-se usar as formas analíticas mais Português linguagens: volume 2 / Wiliam Roberto Ce-
bom, mais mau,mais grande e mais pequeno. Por reja, Thereza Cochar Magalhães. – 7.ª ed. Reform. – São
exemplo: Paulo: Saraiva, 2010.
Pedro é maior do que Paulo - Comparação de dois SACCONI, Luiz Antônio. Nossa gramática completa
elementos. Sacconi. 30.ª ed. Rev. São Paulo: Nova Geração, 2010.
Pedro é mais grande que pequeno - comparação de Português: novas palavras: literatura, gramática, reda-
duas qualidades de um mesmo elemento. ção / Emília Amaral... [et al.]. – São Paulo: FTD, 2000.
Sou menos alto (do) que você. = Comparativo de In-
ferioridade SITE
Sou menos passivo (do) que tolerante. http://www.soportugues.com.br/secoes/morf/
morf32.php
B) Superlativo
O superlativo expressa qualidades num grau muito
elevado ou em grau máximo. Pode ser absoluto ou rela-
tivo e apresenta as seguintes modalidades:
B.1 Superlativo Absoluto: ocorre quando a quali-
dade de um ser é intensificada, sem relação com outros
PORTUGUÊS

seres. Apresenta-se nas formas:


 Analítica: a intensificação é feita com o auxílio de
palavras que dão ideia de intensidade (advérbios).
Por exemplo: O concurseiro é muito esforçado.

14
Advérbio 2. Classificação dos Advérbios

Compare estes exemplos: De acordo com a circunstância que exprime, o advér-


O ônibus chegou. bio pode ser de:
O ônibus chegou ontem. A) Lugar: aqui, antes, dentro, ali, adiante, fora, aco-
lá, atrás, além, lá, detrás, aquém, cá, acima, onde,
Advérbio é uma palavra invariável que modifica o perto, aí, abaixo, aonde, longe, debaixo, algures, de-
sentido do verbo (acrescentando-lhe circunstâncias de fronte, nenhures, adentro, afora, alhures, nenhures,
tempo, de modo, de lugar, de intensidade), do adjetivo e aquém, embaixo, externamente, à distância, à dis-
do próprio advérbio. tância de, de longe, de perto, em cima, à direita, à
Estudei bastante. = modificando o verbo estudei esquerda, ao lado, em volta.
Ele canta muito bem! = intensificando outro advérbio B) Tempo: hoje, logo, primeiro, ontem, tarde, outrora,
(bem) amanhã, cedo, dantes, depois, ainda, antigamente,
Ela tem os olhos muito claros. = relação com um ad- antes, doravante, nunca, então, ora, jamais, agora,
jetivo (claros) sempre, já, enfim, afinal, amiúde, breve, constan-
temente, entrementes, imediatamente, primeira-
Quando modifica um verbo, o advérbio pode acres- mente, provisoriamente, sucessivamente, às vezes,
centar ideia de: à tarde, à noite, de manhã, de repente, de vez em
Tempo: Ela chegou tarde. quando, de quando em quando, a qualquer mo-
Lugar: Ele mora aqui. mento, de tempos em tempos, em breve, hoje em
Modo: Eles agiram mal. dia.
Negação: Ela não saiu de casa. C) Modo: bem, mal, assim, adrede, melhor, pior, de-
Dúvida: Talvez ele volte. pressa, acinte, debalde, devagar, às pressas, às cla-
ras, às cegas, à toa, à vontade, às escondidas, aos
1. Flexão do Advérbio poucos, desse jeito, desse modo, dessa maneira, em
geral, frente a frente, lado a lado, a pé, de cor, em
Os advérbios são palavras invariáveis, isto é, não apre- vão e a maior parte dos que terminam em “-men-
sentam variação em gênero e número. Alguns advérbios, te”: calmamente, tristemente, propositadamente,
porém, admitem a variação em grau. Observe: pacientemente, amorosamente, docemente, escan-
dalosamente, bondosamente, generosamente.
A) Grau Comparativo D) Afirmação: sim, certamente, realmente, decerto,
Forma-se o comparativo do advérbio do mesmo efetivamente, certo, decididamente, deveras, indu-
modo que o comparativo do adjetivo: bitavelmente.
 de igualdade: tão + advérbio + quanto (como): E) Negação: não, nem, nunca, jamais, de modo algum,
Renato fala tão alto quanto João. de forma nenhuma, tampouco, de jeito nenhum.
 de inferioridade: menos + advérbio + que (do F) Dúvida: acaso, porventura, possivelmente, pro-
que): Renato fala menos alto do que João. vavelmente, quiçá, talvez, casualmente, por certo,
 de superioridade: quem sabe.
A.1 Analítico: mais + advérbio + que (do que): Renato G) Intensidade: muito, demais, pouco, tão, em ex-
fala mais alto do que João. cesso, bastante, mais, menos, demasiado, quanto,
A.2 Sintético: melhor ou pior que (do que): Renato quão, tanto, assaz, que (equivale a quão), tudo,
fala melhor que João. nada, todo, quase, de todo, de muito, por completo,
extremamente, intensamente, grandemente, bem
B) Grau Superlativo (quando aplicado a propriedades graduáveis).
O superlativo pode ser analítico ou sintético: H) Exclusão: apenas, exclusivamente, salvo, senão, so-
B.1 Analítico: acompanhado de outro advérbio: Re- mente, simplesmente, só, unicamente. Por exemplo:
nato fala muito alto. Brando, o vento apenas move a copa das árvores.
muito = advérbio de intensidade / alto = advérbio I) Inclusão: ainda, até, mesmo, inclusivamente, tam-
de modo bém. Por exemplo: O indivíduo também amadurece
B.2 Sintético: formado com sufixos: Renato fala al- durante a adolescência.
tíssimo. J) Ordem: depois, primeiramente, ultimamente. Por
exemplo: Primeiramente, eu gostaria de agradecer
Observação: aos meus amigos por comparecerem à festa.
As formas diminutivas (cedinho, pertinho, etc.) são
comuns na língua popular. Saiba que:
Maria mora pertinho daqui. (muito perto) Para se exprimir o limite de possibilidade, antepõe-se
A criança levantou cedinho. (muito cedo) ao advérbio “o mais” ou “o menos”. Por exemplo: Ficarei
o mais longe que puder daquele garoto. Voltarei o menos
PORTUGUÊS

tarde possível.
Quando ocorrem dois ou mais advérbios em -mente,
em geral sufixamos apenas o último: O aluno respondeu
calma e respeitosamente.

15
3. Distinção entre Advérbio e Pronome Indefinido Usam-se, de preferência, as formas mais bem e mais
mal antes de adjetivos ou de verbos no particípio:
Há palavras como muito, bastante, que podem apare- Essa matéria é mais bem interessante que aquela.
cer como advérbio e como pronome indefinido. Nosso aluno foi o mais bem colocado no concurso!
O numeral “primeiro”, ao modificar o verbo, é advér-
Advérbio: refere-se a um verbo, adjetivo, ou a outro bio: Cheguei primeiro.
advérbio e não sofre flexões. Por exemplo: Eu corri muito.
Pronome Indefinido: relaciona-se a um substantivo Quanto a sua função sintática: o advérbio e a locução
e sofre flexões. Por exemplo: Eu corri muitos quilômetros. adverbial desempenham na oração a função de adjunto
adverbial, classificando-se de acordo com as circunstân-
#FicaDica cias que acrescentam ao verbo, ao adjetivo ou ao advér-
Como saber se a palavra bastante é advérbio bio. Exemplo:
(não varia, não se flexiona) ou pronome Meio cansada, a candidata saiu da sala. = adjunto ad-
indefinido (varia, sofre flexão)? Se der, na verbial de intensidade (ligado ao adjetivo “cansada”)
frase, para substituir o “bastante” por “muito”, Trovejou muito ontem. = adjunto adverbial de intensi-
estamos diante de um advérbio; se der para dade e de tempo, respectivamente.
substituir por “muitos” (ou muitas), é um
pronome. Veja: REFERÊNCIAS BIBLIOGRÁFICAS
1. Estudei bastante para o concurso. (estudei Português linguagens: volume 2 / Wiliam Roberto Ce-
muito, pois “muitos” não dá!) = advérbio reja, Thereza Cochar Magalhães. – 7.ª ed. Reform. – São
2. Estudei bastantes capítulos para o concurso. Paulo: Saraiva, 2010.
(estudei muitos capítulos) = pronome Português: novas palavras: literatura, gramática, reda-
indefinido ção / Emília Amaral... [et al.]. – São Paulo: FTD, 2000.
SACCONI, Luiz Antônio. Nossa gramática completa
Sacconi. 30.ª ed. Rev. São Paulo: Nova Geração, 2010.

4. Advérbios Interrogativos SITE


http://www.soportugues.com.br/secoes/morf/
São as palavras: onde? aonde? donde? quando? como? morf75.php
por quê? nas interrogações diretas ou indiretas, referen-
tes às circunstâncias de lugar, tempo, modo e causa. Veja: Artigo

Interrogação Direta Interrogação Indireta O artigo integra as dez classes gramaticais, definindo-
-se como o termo variável que serve para individualizar
Como aprendeu? Perguntei como aprendeu. ou generalizar o substantivo, indicando, também, o gê-
Onde mora? Indaguei onde morava. nero (masculino/feminino) e o número (singular/plural).
Os artigos se subdividem em definidos (“o” e as va-
Por que choras? Não sei por que choras. riações “a”[as] e [os]) e indefinidos (“um” e as variações
Aonde vai? Perguntei aonde ia. “uma”[s] e “uns]).
Donde vens? Pergunto donde vens.
A) Artigos definidos – São usados para indicar seres
Quando voltas? Pergunto quando voltas. determinados, expressos de forma individual: O
concurseiro estuda muito. Os concurseiros estudam
5. Locução Adverbial muito.
B) Artigos indefinidos – usados para indicar seres de
Quando há duas ou mais palavras que exercem fun- modo vago, impreciso: Uma candidata foi aprova-
ção de advérbio, temos a locução adverbial, que pode da! Umas candidatas foram aprovadas!
expressar as mesmas noções dos advérbios. Iniciam ordi-
nariamente por uma preposição. Veja: 1. Circunstâncias em que os artigos se manifestam:
A) lugar: à esquerda, à direita, de longe, de perto,
para dentro, por aqui, etc. Considera-se obrigatório o uso do artigo depois do
B) afirmação: por certo, sem dúvida, etc. numeral “ambos”: Ambos os concursos cobrarão tal con-
C) modo: às pressas, passo a passo, de cor, em vão, teúdo.
em geral, frente a frente, etc. Nomes próprios indicativos de lugar (ou topônimos)
D) tempo: de noite, de dia, de vez em quando, à tarde, admitem o uso do artigo, outros não: São Paulo, O Rio de
hoje em dia, nunca mais, etc. Janeiro, Veneza, A Bahia...
A locução adverbial e o advérbio modificam o verbo, Quando indicado no singular, o artigo definido pode
o adjetivo e outro advérbio: indicar toda uma espécie: O trabalho dignifica o homem.
PORTUGUÊS

Chegou muito cedo. (advérbio) No caso de nomes próprios personativos, denotando


Joana é muito bela. (adjetivo) a ideia de familiaridade ou afetividade, é facultativo o uso
De repente correram para a rua. (verbo) do artigo: Marcela é a mais extrovertida das irmãs. / O
Pedro é o xodó da família.

16
No caso de os nomes próprios personativos estarem 1. Morfossintaxe da Conjunção
no plural, são determinados pelo uso do artigo: Os Maias,
os Incas, Os Astecas... As conjunções, a exemplo das preposições, não exer-
Usa-se o artigo depois do pronome indefinido todo(a) cem propriamente uma função sintática: são conectivos.
para conferir uma ideia de totalidade. Sem o uso dele (do
artigo), o pronome assume a noção de “qualquer”. 2. Classificação da Conjunção
Toda a classe parabenizou o professor. (a sala toda)
Toda classe possui alunos interessados e desinteressa- De acordo com o tipo de relação que estabelecem,
dos. (qualquer classe) as conjunções podem ser classificadas em coordenati-
Antes de pronomes possessivos, o uso do artigo é fa- vas e subordinativas. No primeiro caso, os elementos
cultativo: Preparei o meu curso. Preparei meu curso. ligados pela conjunção podem ser isolados um do outro.
A utilização do artigo indefinido pode indicar uma Esse isolamento, no entanto, não acarreta perda da uni-
ideia de aproximação numérica: O máximo que ele deve dade de sentido que cada um dos elementos possui. Já
ter é uns vinte anos. no segundo caso, cada um dos elementos ligados pela
O artigo também é usado para substantivar palavras conjunção depende da existência do outro. Veja:
pertencentes a outras classes gramaticais: Não sei o por- Estudei muito, mas ainda não compreendi o conteúdo.
quê de tudo isso. / O bem vence o mal. Podemos separá-las por ponto:
Estudei muito. Ainda não compreendi o conteúdo.
2. Há casos em que o artigo definido não pode ser
usado: Temos acima um exemplo de conjunção (e, conse-
Antes de nomes de cidade (topônimo) e de pessoas quentemente, orações coordenadas) coordenativa –
conhecidas: O professor visitará Roma. “mas”. Já em:
Espero que eu seja aprovada no concurso!
Mas, se o nome apresentar um caracterizador, a pre- Não conseguimos separar uma oração da outra, pois
sença do artigo será obrigatória: O professor visitará a a segunda “completa” o sentido da primeira (da oração
bela Roma. principal): Espero o quê? Ser aprovada. Nesse período te-
mos uma oração subordinada substantiva objetiva direta
(ela exerce a função de objeto direto do verbo da oração
Antes de pronomes de tratamento: Vossa Senhoria
principal).
sairá agora?
Exceção: O senhor vai à festa?
3. Conjunções Coordenativas
Após o pronome relativo “cujo” e suas variações: Esse
São aquelas que ligam orações de sentido completo
é o concurso cujas provas foram anuladas?/ Este é o can-
e independente ou termos da oração que têm a mesma
didato cuja nota foi a mais alta. função gramatical. Subdividem-se em:
A) Aditivas: ligam orações ou palavras, expressando
REFERÊNCIAS BIBLIOGRÁFICAS ideia de acréscimo ou adição. São elas: e, nem (= e
Português linguagens: volume 2 / Wiliam Roberto Ce- não), não só... mas também, não só... como também,
reja, Thereza Cochar Magalhães. – 7.ª ed. Reform. – São bem como, não só... mas ainda.
Paulo: Saraiva, 2010. A sua pesquisa é clara e objetiva.
Português: novas palavras: literatura, gramática, reda- Não só dança, mas também canta.
ção / Emília Amaral... [et al.]. – São Paulo: FTD, 2000.SAC-
CONI, Luiz Antônio. Nossa gramática completa Sacconi. B) Adversativas: ligam duas orações ou palavras,
30.ª ed. Rev. São Paulo: Nova Geração, 2010. expressando ideia de contraste ou compensação.
Português linguagens: volume 1 / Wiliam Roberto Ce- São elas: mas, porém, contudo, todavia, entretanto,
reja, Thereza Cochar Magalhães. – 7.ª ed. Reform. – São no entanto, não obstante.
Paulo: Saraiva, 2010. Tentei chegar mais cedo, porém não consegui.

SITE
http://www.brasilescola.com/gramatica/artigo.htm C) Alternativas: ligam orações ou palavras, expres-
sando ideia de alternância ou escolha, indicando
fatos que se realizam separadamente. São elas: ou,
Conjunção ou... ou, ora... ora, já... já, quer... quer, seja... seja, tal-
vez... talvez.
Além da preposição, há outra palavra também inva- Ou escolho agora, ou fico sem presente de aniversário.
riável que, na frase, é usada como elemento de ligação:
a conjunção. Ela serve para ligar duas orações ou duas D) Conclusivas: ligam a oração anterior a uma oração
palavras de mesma função em uma oração: que expressa ideia de conclusão ou consequência.
O concurso será realizado nas cidades de Campinas e São elas: logo, pois (depois do verbo), portanto, por
PORTUGUÊS

São Paulo. conseguinte, por isso, assim.


A prova não será fácil, por isso estou estudando muito. Marta estava bem preparada para o teste, portanto
não ficou nervosa.
Você nos ajudou muito; terá, pois, nossa gratidão.

17
E) Explicativas: ligam a oração anterior a uma oração
que a explica, que justifica a ideia nela contida. São #FicaDica
elas: que, porque, pois (antes do verbo), porquanto. Você deve ter percebido que a conjunção con-
Não demore, que o filme já vai começar. dicional “se” também é conjunção integrante.
Falei muito, pois não gosto do silêncio! A diferença é clara ao ler as orações que são
introduzidas por ela. Acima, ela nos dá a ideia
4. Conjunções Subordinativas da condição para que recebamos um telefo-
nema (se for preciso ajuda). Já na oração: Não
São aquelas que ligam duas orações, sendo uma de- sei se farei o concurso. Não há ideia de
las dependente da outra. A oração dependente, intro- condição alguma, há? Outra coisa: o verbo da
duzida pelas conjunções subordinativas, recebe o nome oração principal (sei) pede complemento (ob-
de oração subordinada. Veja o exemplo: O baile já tinha jeto direto, já que “quem não sabe, não sabe
começado quando ela chegou. algo”). Portanto, a oração em destaque exerce
O baile já tinha começado: oração principal a função de objeto direto da oração principal,
quando: conjunção subordinativa (adverbial tempo- sendo classificada como oração subordinada
ral) substantiva objetiva direta.
ela chegou: oração subordinada

As conjunções subordinativas subdividem-se em in- D) Conformativas: introduzem uma oração que ex-
tegrantes e adverbiais: prime a conformidade de um fato com outro. São
elas: conforme, como (= conforme), segundo, con-
Integrantes - Indicam que a oração subordinada por soante, etc.
elas introduzida completa ou integra o sentido da prin- O passeio ocorreu como havíamos planejado.
cipal. Introduzem orações que equivalem a substantivos,
ou seja, as orações subordinadas substantivas. São elas: E) Finais: introduzem uma oração que expressa a fi-
que, se. nalidade ou o objetivo com que se realiza a oração
Quero que você volte. (Quero sua volta) principal. São elas: para que, a fim de que, que, por-
que (= para que), que, etc.
Adverbiais - Indicam que a oração subordinada exer- Toque o sinal para que todos entrem no salão.
ce a função de adjunto adverbial da principal. De acordo
com a circunstância que expressam, classificam-se em: F) Proporcionais: introduzem uma oração que ex-
pressa um fato relacionado proporcionalmente
A) Causais: introduzem uma oração que é causa da à ocorrência do expresso na principal. São elas:
ocorrência da oração principal. São elas: porque, à medida que, à proporção que, ao passo que e
que, como (= porque, no início da frase), pois que, as combinações quanto mais... (mais), quanto me-
visto que, uma vez que, porquanto, já que, desde nos... (menos), quanto menos... (mais), quanto me-
que, etc. nos... (menos), etc.
Ele não fez a pesquisa porque não dispunha de meios. O preço fica mais caro à medida que os produtos es-
casseiam.
B) Concessivas: introduzem uma oração que expres-
sa ideia contrária à da principal, sem, no entanto, Observação:
São incorretas as locuções proporcionais à medida
impedir sua realização. São elas: embora, ainda
em que, na medida que e na medida em que.
que, apesar de que, se bem que, mesmo que, por
mais que, posto que, conquanto, etc.
G) Temporais: introduzem uma oração que acrescen-
Embora fosse tarde, fomos visitá-lo.
ta uma circunstância de tempo ao fato expresso na
oração principal. São elas: quando, enquanto, antes
C) Condicionais: introduzem uma oração que indica
que, depois que, logo que, todas as vezes que, desde
a hipótese ou a condição para ocorrência da prin-
que, sempre que, assim que, agora que, mal (= as-
cipal. São elas: se, caso, contanto que, salvo se, a sim que), etc.
não ser que, desde que, a menos que, sem que, etc. A briga começou assim que saímos da festa.
Se precisar de minha ajuda, telefone-me.
H) Comparativas: introduzem uma oração que ex-
pressa ideia de comparação com referência à ora-
ção principal. São elas: como, assim como, tal como,
como se, (tão)... como, tanto como, tanto quanto, do
que, quanto, tal, qual, tal qual, que nem, que (com-
PORTUGUÊS

binado com menos ou mais), etc.


O jogo de hoje será mais difícil que o de ontem.

18
I) Consecutivas: introduzem uma oração que expressa As interjeições cumprem, normalmente, duas funções:
a consequência da principal. São elas: de sorte que, A) Sintetizar uma frase exclamativa, exprimindo ale-
de modo que, sem que (= que não), de forma que, gria, tristeza, dor, etc.: Ah, deve ser muito interes-
de jeito que, que (tendo como antecedente na oração sante!
principal uma palavra como tal, tão, cada, tanto, ta- B) Sintetizar uma frase apelativa: Cuidado! Saia da
manho), etc. minha frente.
Estudou tanto durante a noite que dormiu na hora do
exame. As interjeições podem ser formadas por:
 simples sons vocálicos: Oh!, Ah!, Ó, Ô
FIQUE ATENTO!  palavras: Oba! Olá! Claro!
Muitas conjunções não têm classificação úni-  grupos de palavras (locuções interjetivas): Meu
ca, imutável, devendo, portanto, ser classifica- Deus! Ora bolas!
das de acordo com o sentido que apresentam
no contexto (destaque da Zê!). 1. Classificação das Interjeições

Comumente, as interjeições expressam sentido de:


A) Advertência: Cuidado! Devagar! Calma! Sentido!
REFERÊNCIAS BIBLIOGRÁFICAS Atenção! Olha! Alerta!
SACCONI, Luiz Antônio. Nossa gramática completa B) Afugentamento: Fora! Passa! Rua!
Sacconi. 30.ª ed. Rev. São Paulo: Nova Geração, 2010. C) Alegria ou Satisfação: Oh! Ah! Eh! Oba! Viva!
Português linguagens: volume 2 / Wiliam Roberto Ce- D) Alívio: Arre! Uf! Ufa! Ah!
reja, Thereza Cochar Magalhães. – 7.ª ed. Reform. – São E) Animação ou Estímulo: Vamos! Força! Coragem!
Paulo: Saraiva, 2010. Ânimo! Adiante!
Português: novas palavras: literatura, gramática, reda- F) Aplauso ou Aprovação: Bravo! Bis! Apoiado! Viva!
ção / Emília Amaral... [et al.]. – São Paulo: FTD, 2000.
G) Concordância: Claro! Sim! Pois não! Tá!
H) Repulsa ou Desaprovação: Credo! Ih! Francamen-
SITE
te! Essa não! Chega! Basta!
http://www.soportugues.com.br/secoes/morf/morf84.
I) Desejo ou Intenção: Pudera! Tomara! Oxalá! Quei-
php
ra Deus!
J) Desculpa: Perdão!
Interjeição
K) Dor ou Tristeza: Ai! Ui! Ai de mim! Que pena!
Interjeição é a palavra invariável que exprime emo- L) Dúvida ou Incredulidade: Que nada! Qual o quê!
ções, sensações, estados de espírito. É um recurso da lin- M) Espanto ou Admiração: Oh! Ah! Uai! Puxa! Céus!
guagem afetiva, em que não há uma ideia organizada de Quê! Caramba! Opa! Nossa! Hein? Cruz! Putz!
maneira lógica, como são as sentenças da língua, mas sim N) Impaciência ou Contrariedade: Hum! Raios!
a manifestação de um suspiro, um estado da alma decor- Puxa! Pô! Ora!
rente de uma situação particular, um momento ou um O) Pedido de Auxílio: Socorro! Aqui! Piedade!
contexto específico. Exemplos: P) Saudação, Chamamento ou Invocação: Salve!
Ah, como eu queria voltar a ser criança! Viva! Olá! Alô! Tchau! Psiu! Socorro! Valha-me,
ah: expressão de um estado emotivo = interjeição Deus!
Hum! Esse pudim estava maravilhoso! Q) Silêncio: Psiu! Silêncio!
hum: expressão de um pensamento súbito = interjei- R) Terror ou Medo: Credo! Cruzes! Minha nossa!
ção
O significado das interjeições está vinculado à maneira Saiba que:
como elas são proferidas. O tom da fala é que dita o senti- As interjeições são palavras invariáveis, isto é, não so-
do que a expressão vai adquirir em cada contexto em que frem variação em gênero, número e grau como os no-
for utilizada. Exemplos: mes, nem de número, pessoa, tempo, modo, aspecto e
voz como os verbos. No entanto, em uso específico, al-
Psiu! gumas interjeições sofrem variação em grau. Não se trata
contexto: alguém pronunciando esta expressão na rua; de um processo natural desta classe de palavra, mas tão
significado da interjeição (sugestão): “Estou te chamando! só uma variação que a linguagem afetiva permite. Exem-
Ei, espere!” plos: oizinho, bravíssimo, até loguinho.

Psiu! 2. Locução Interjetiva


contexto: alguém pronunciando em um hospital; sig-
nificado da interjeição (sugestão): “Por favor, faça silêncio!” Ocorre quando duas ou mais palavras formam uma
expressão com sentido de interjeição: Ora bolas!, Virgem
Puxa! Ganhei o maior prêmio do sorteio! Maria!, Meu Deus!, Ó de casa!, Ai de mim!, Graças a Deus!
PORTUGUÊS

puxa: interjeição; tom da fala: euforia Toda frase mais ou menos breve dita em tom excla-
mativo torna-se uma locução interjetiva, dispensando
Puxa! Hoje não foi meu dia de sorte! análise dos termos que a compõem: Macacos me mor-
puxa: interjeição; tom da fala: decepção dam!, Valha-me Deus!, Quem me dera!

19
1. As interjeições são como frases resumidas, sinté-
ticas. Por exemplo: Ué! (= Eu não esperava por #FicaDica
essa!) / Perdão! (= Peço-lhe que me desculpe)
As palavras anterior, posterior, último, antepe-
2. Além do contexto, o que caracteriza a interjeição é
núltimo, final e penúltimo também indicam
o seu tom exclamativo; por isso, palavras de outras
posição dos seres, mas são classificadas como
classes gramaticais podem aparecer como inter-
adjetivos, não ordinais.
jeições. Por exemplo: Viva! Basta! (Verbos) / Fora!
Francamente! (Advérbios)
3. A interjeição pode ser considerada uma “palavra-
-frase” porque sozinha pode constituir uma men- C) Fracionários: indicam parte de uma quantidade,
sagem. Por exemplo: Socorro! Ajudem-me! Silêncio! ou seja, uma divisão dos seres: meio, terço, dois
Fique quieto! quintos, etc.
4. Há, também, as interjeições onomatopaicas ou imi- D) Multiplicativos: expressam ideia de multiplicação
tativas, que exprimem ruídos e vozes. Por exemplo: dos seres, indicando quantas vezes a quantidade
Miau! Bumba! Zás! Plaft! Pof! Catapimba! Tique-ta- foi aumentada: dobro, triplo, quíntuplo, etc.
que! Quá-quá-quá!, etc.
5. Não se deve confundir a interjeição de apelo “ó” 2. Flexão dos numerais
com a sua homônima “oh!”, que exprime admira-
ção, alegria, tristeza, etc. Faz-se uma pausa depois Os numerais cardinais que variam em gênero são um/
do “oh!” exclamativo e não a fazemos depois do uma, dois/duas e os que indicam centenas de duzentos/
“ó” vocativo. Por exemplo: “Ó natureza! ó mãe pie- duzentas em diante: trezentos/trezentas, quatrocentos/
dosa e pura!” (Olavo Bilac) quatrocentas, etc. Cardinais como milhão, bilhão, trilhão,
variam em número: milhões, bilhões, trilhões. Os demais
REFERÊNCIAS BIBLIOGRÁFICAS cardinais são invariáveis.
SACCONI, Luiz Antônio. Nossa gramática completa Os numerais ordinais variam em gênero e número:
Sacconi. 30.ª ed. Rev. São Paulo: Nova Geração, 2010.
Português – Literatura, Produção de Textos & Gramá- primeiro segundo milésimo
tica – volume único / Samira Yousseff Campedelli, Jésus
Barbosa Souza. – 3. Ed. – São Paulo: Saraiva, 2002. primeira segunda milésima
primeiros segundos milésimos
SITE
http://www.soportugues.com.br/secoes/morf/ primeiras segundas milésimas
morf89.php
Os numerais multiplicativos são invariáveis quando
NUMERAL atuam em funções substantivas: Fizeram o dobro do es-
forço e conseguiram o triplo de produção.
Numeral é a palavra variável que indica quantidade Quando atuam em funções adjetivas, esses numerais
numérica ou ordem; expressa a quantidade exata de pes- flexionam-se em gênero e número: Teve de tomar doses
soas ou coisas ou o lugar que elas ocupam numa deter- triplas do medicamento.
minada sequência. Os numerais fracionários flexionam-se em gênero e
número. Observe: um terço/dois terços, uma terça parte/
Os numerais traduzem, em palavras, o que os núme- duas terças partes.
ros indicam em relação aos seres. Assim, quando a ex-
pressão é colocada em números (1, 1.º, 1/3, etc.) não se Os numerais coletivos flexionam-se em número: uma
trata de numerais, mas sim de algarismos. dúzia, um milheiro, duas dúzias, dois milheiros.
Além dos numerais mais conhecidos, já que refletem É comum na linguagem coloquial a indicação de grau
a ideia expressa pelos números, existem mais algumas nos numerais, traduzindo afetividade ou especialização
palavras consideradas numerais porque denotam quan- de sentido. É o que ocorre em frases como:
“Me empresta duzentinho...”
tidade, proporção ou ordenação. São alguns exemplos:
É artigo de primeiríssima qualidade!
década, dúzia, par, ambos(as), novena.
O time está arriscado por ter caído na segundona. (=
segunda divisão de futebol)
1. Classificação dos Numerais
3. Emprego e Leitura dos Numerais
A) Cardinais: indicam quantidade exata ou determi-
nada de seres: um, dois, cem mil, etc. Alguns car-
Os numerais são escritos em conjunto de três algaris-
dinais têm sentido coletivo, como por exemplo: mos, contados da direita para a esquerda, em forma de
século, par, dúzia, década, bimestre. centenas, dezenas e unidades, tendo cada conjunto uma
B) Ordinais: indicam a ordem, a posição que alguém
PORTUGUÊS

separação através de ponto ou espaço correspondente a


ou alguma coisa ocupa numa determinada se- um ponto: 8.234.456 ou 8 234 456.
quência: primeiro, segundo, centésimo, etc. Em sentido figurado, usa-se o numeral para indicar
exagero intencional, constituindo a figura de linguagem
conhecida como hipérbole: Já li esse texto mil vezes.

20
No português contemporâneo, não se usa a conjunção “e” após “mil”, seguido de centena: Nasci em mil novecentos
e noventa e dois.
Seu salário será de mil quinhentos e cinquenta reais.

Mas, se a centena começa por “zero” ou termina por dois zeros, usa-se o “e”: Seu salário será de mil e quinhentos
reais. (R$1.500,00)
Gastamos mil e quarenta reais. (R$1.040,00)

Para designar papas, reis, imperadores, séculos e partes em que se divide uma obra, utilizam-se os ordinais até
décimo e, a partir daí, os cardinais, desde que o numeral venha depois do substantivo;

Ordinais Cardinais
João Paulo II (segundo) Tomo XV (quinze)
D. Pedro II (segundo) Luís XVI (dezesseis)
Ato II (segundo) Capítulo XX (vinte)
Século VIII (oitavo) Século XX (vinte)
Canto IX (nono) João XXIII ( vinte e três)

Se o numeral aparece antes do substantivo, será lido como ordinal: XXX Feira do Bordado. (trigésima)

#FicaDica
Ordinal lembra ordem. Memorize assim, por associação. Ficará mais fácil!

Para designar leis, decretos e portarias, utiliza-se o ordinal até nono e o cardinal de dez em diante:
Artigo 1.° (primeiro) Artigo 10 (dez)
Artigo 9.° (nono) Artigo 21 (vinte e um)

Ambos/ambas = numeral dual, porque sempre se refere a dois seres. Significam “um e outro”, “os dois” (ou “uma
e outra”, “as duas”) e são largamente empregados para retomar pares de seres aos quais já se fez referência. Sua uti-
lização exige a presença do artigo posposto: Ambos os concursos realizarão suas provas no mesmo dia. O artigo só é
dispensado caso haja um pronome demonstrativo: Ambos esses ministros falarão à imprensa.

Quadro de alguns numerais

Cardinais Ordinais Multiplicativos Fracionários


um primeiro - -
dois segundo dobro, duplo meio
três terceiro triplo, tríplice terço
quatro quarto quádruplo quarto
cinco quinto quíntuplo quinto
seis sexto sêxtuplo sexto
sete sétimo sétuplo sétimo
oito oitavo óctuplo oitavo
nove nono nônuplo nono
dez décimo décuplo décimo
onze décimo primeiro - onze avos
doze décimo segundo - doze avos
PORTUGUÊS

treze décimo terceiro - treze avos


catorze décimo quarto - catorze avos
quinze décimo quinto - quinze avos

21
dezesseis décimo sexto - dezesseis avos
dezessete décimo sétimo - dezessete avos
dezoito décimo oitavo - dezoito avos
dezenove décimo nono - dezenove avos
vinte vigésimo - vinte avos
trinta trigésimo - trinta avos
quarenta quadragésimo - quarenta avos
cinqüenta quinquagésimo - cinquenta avos
sessenta sexagésimo - sessenta avos
setenta septuagésimo - setenta avos
oitenta octogésimo - oitenta avos
noventa nonagésimo - noventa avos
cem centésimo cêntuplo centésimo
duzentos ducentésimo - ducentésimo
trezentos trecentésimo - trecentésimo
quatrocentos quadringentésimo - quadringentésimo
quinhentos quingentésimo - quingentésimo
seiscentos sexcentésimo - sexcentésimo
setecentos septingentésimo - septingentésimo
oitocentos octingentésimo - octingentésimo
novecentos nongentésimo
ou noningentésimo - nongentésimo
mil milésimo - milésimo
milhão milionésimo - milionésimo
bilhão bilionésimo - bilionésimo

REFERÊNCIAS BIBLIOGRÁFICAS
SACCONI, Luiz Antônio. Nossa gramática completa Sacconi. 30.ª ed. Rev. São Paulo: Nova Geração, 2010.
Português linguagens: volume 2 / Wiliam Roberto Cereja, Thereza Cochar Magalhães. – 7.ª ed. Reform. – São Paulo:
Saraiva, 2010.
Português: novas palavras: literatura, gramática, redação / Emília Amaral... [et al.]. – São Paulo: FTD, 2000.

SITE
http://www.soportugues.com.br/secoes/morf/morf40.php

Preposição

Preposição é uma palavra invariável que serve para ligar termos ou orações. Quando esta ligação acontece, nor-
malmente há uma subordinação do segundo termo em relação ao primeiro. As preposições são muito importantes na
estrutura da língua, pois estabelecem a coesão textual e possuem valores semânticos indispensáveis para a compreen-
são do texto.

1. Tipos de Preposição

A) Preposições essenciais: palavras que atuam exclusivamente como preposições: a, ante, perante, após, até, com,
contra, de, desde, em, entre, para, por, sem, sob, sobre, trás, atrás de, dentro de, para com.
B) Preposições acidentais: palavras de outras classes gramaticais que podem atuar como preposições, ou seja,
PORTUGUÊS

formadas por uma derivação imprópria: como, durante, exceto, fora, mediante, salvo, segundo, senão, visto.
C) Locuções prepositivas: duas ou mais palavras valendo como uma preposição, sendo que a última palavra é uma
(preposição): abaixo de, acerca de, acima de, ao lado de, a respeito de, de acordo com, em cima de, embaixo de, em
frente a, ao redor de, graças a, junto a, com, perto de, por causa de, por cima de, por trás de.

22
A preposição é invariável e, no entanto, pode unir-se REFERÊNCIAS BIBLIOGRÁFICAS
a outras palavras e, assim, estabelecer concordância em SACCONI, Luiz Antônio. Nossa gramática completa
gênero ou em número. Exemplo: por + o = pelo / por + Sacconi. 30.ª ed. Rev. São Paulo: Nova Geração, 2010.
a = pela. Português linguagens: volume 2 / Wiliam Roberto Ce-
Essa concordância não é característica da preposição, reja, Thereza Cochar Magalhães. – 7.ª ed. Reform. – São
mas das palavras às quais ela se une. Paulo: Saraiva, 2010.
Esse processo de junção de uma preposição com ou- Português: novas palavras: literatura, gramática, reda-
tra palavra pode se dar a partir dos processos de: ção / Emília Amaral... [et al.]. – São Paulo: FTD, 2000.
 Combinação: união da preposição “a” com o ar-
tigo “o”(s), ou com o advérbio “onde”: ao, aonde, SITE
aos. Os vocábulos não sofrem alteração. http://www.infoescola.com/portugues/preposicao/
 Contração: união de uma preposição com outra
palavra, ocorrendo perda ou transformação de fo- Substantivo
nema: de + o = do, em + a = na, per + os = pelos,
de + aquele = daquele, em + isso = nisso. Substantivo é a classe gramatical de palavras variá-
 Crase: é a fusão de vogais idênticas: à (“a” preposi- veis, as quais denominam todos os seres que existem,
ção + “a” artigo), àquilo (“a” preposição + 1.ª vogal sejam reais ou imaginários. Além de objetos, pessoas e
do pronome “aquilo”). fenômenos, os substantivos também nomeiam:
 lugares: Alemanha, Portugal
#FicaDica  sentimentos: amor, saudade
 estados: alegria, tristeza
O “a” pode funcionar como preposição, pro-  qualidades: honestidade, sinceridade
nome pessoal oblíquo e artigo. Como dis-  ações: corrida, pescaria
tingui-los? Caso o “a” seja um artigo, virá
precedendo um substantivo, servindo para 1. Morfossintaxe do substantivo
determiná-lo como um substantivo singular e
feminino: A matéria que estudei é fácil! Nas orações, geralmente o substantivo exerce fun-
ções diretamente relacionadas com o verbo: atua como
núcleo do sujeito, dos complementos verbais (objeto di-
reto ou indireto) e do agente da passiva, podendo, ainda,
Quando é preposição, além de ser invariável, liga dois
funcionar como núcleo do complemento nominal ou do
termos e estabelece relação de subordinação entre eles.
aposto, como núcleo do predicativo do sujeito, do obje-
Irei à festa sozinha.
Entregamos a flor à professora! = o primeiro “a” é arti- to ou como núcleo do vocativo. Também encontramos
go; o segundo, preposição. substantivos como núcleos de adjuntos adnominais e de
Se for pronome pessoal oblíquo estará ocupando o adjuntos adverbiais - quando essas funções são desem-
lugar e/ou a função de um substantivo: Nós trouxemos a penhadas por grupos de palavras.
apostila. = Nós a trouxemos.
2. Classificação dos Substantivos
2. Relações semânticas (= de sentido) estabeleci-
das por meio das preposições: A) Substantivos Comuns e Próprios
Observe a definição:
Destino = Irei a Salvador.
Modo = Saiu aos prantos. Cidade: s.f. 1. Povoação maior que vila, com muitas
Lugar = Sempre a seu lado. casas e edifícios, dispostos em ruas e avenidas (no Brasil,
Assunto = Falemos sobre futebol. toda a sede de município é cidade). 2. O centro de uma
Tempo = Chegarei em instantes. cidade (em oposição aos bairros).
Causa = Chorei de saudade. Qualquer “povoação maior que vila, com muitas casas
Fim ou finalidade = Vim para ficar. e edifícios, dispostos em ruas e avenidas” será chamada
Instrumento = Escreveu a lápis. cidade. Isso significa que a palavra cidade é um substan-
Posse = Vi as roupas da mamãe. tivo comum.
Autoria = livro de Machado de Assis Substantivo Comum é aquele que designa os seres de
Companhia = Estarei com ele amanhã. uma mesma espécie de forma genérica: cidade, menino,
Matéria = copo de cristal. homem, mulher, país, cachorro.
Meio = passeio de barco. Estamos voando para Barcelona.
Origem = Nós somos do Nordeste.
Conteúdo = frascos de perfume. O substantivo Barcelona designa apenas um ser da
Oposição = Esse movimento é contra o que eu penso. espécie cidade. Barcelona é um substantivo próprio –
Preço = Essa roupa sai por cinquenta reais. aquele que designa os seres de uma mesma espécie de
PORTUGUÊS

forma particular: Londres, Paulinho, Pedro, Tietê, Brasil.


Quanto à preposição “trás”: não se usa senão nas
locuções adverbiais (para trás ou por trás) e na locução
prepositiva por trás de.

23
B) Substantivos Concretos e Abstratos cancioneiro canções, poesias líricas
B.1 Substantivo Concreto: é aquele que designa o
ser que existe, independentemente de outros seres. colmeia abelhas
concílio bispos
Observação:
Os substantivos concretos designam seres do mundo congresso parlamentares, cientistas
real e do mundo imaginário. elenco atores de uma peça ou filme
Seres do mundo real: homem, mulher, cadeira, cobra, esquadra navios de guerra
Brasília.
Seres do mundo imaginário: saci, mãe-d’água, fantas- enxoval roupas
ma. falange soldados, anjos

B.2 Substantivo Abstrato: é aquele que designa se- fauna animais de uma região
res que dependem de outros para se manifestarem ou feixe lenha, capim
existirem. Por exemplo: a beleza não existe por si só, flora vegetais de uma região
não pode ser observada. Só podemos observar a beleza
numa pessoa ou coisa que seja bela. A beleza depende frota navios mercantes, ônibus
de outro ser para se manifestar. Portanto, a palavra bele- girândola fogos de artifício
za é um substantivo abstrato.
Os substantivos abstratos designam estados, quali- horda bandidos, invasores
dades, ações e sentimentos dos seres, dos quais podem junta médicos, bois, credores, exa-
ser abstraídos, e sem os quais não podem existir: vida minadores
(estado), rapidez (qualidade), viagem (ação), saudade júri jurados
(sentimento).
legião soldados, anjos, demônios
 Substantivos Coletivos leva presos, recrutas

Ele vinha pela estrada e foi picado por uma abelha, malta malfeitores ou desordeiros
outra abelha, mais outra abelha. manada búfalos, bois, elefantes,
Ele vinha pela estrada e foi picado por várias abelhas. matilha cães de raça
Ele vinha pela estrada e foi picado por um enxame.
molho chaves, verduras
Note que, no primeiro caso, para indicar plural, foi ne- multidão pessoas em geral
cessário repetir o substantivo: uma abelha, outra abelha,
mais outra abelha. No segundo caso, utilizaram-se duas nuvem insetos (gafanhotos, mosqui-
palavras no plural. No terceiro, empregou-se um subs- tos, etc.)
tantivo no singular (enxame) para designar um conjunto penca bananas, chaves
de seres da mesma espécie (abelhas). pinacoteca pinturas, quadros
O substantivo enxame é um substantivo coletivo.
Substantivo Coletivo: é o substantivo comum que, quadrilha ladrões, bandidos
mesmo estando no singular, designa um conjunto de se- ramalhete flores
res da mesma espécie.
rebanho ovelhas

Substantivo coletivo Conjunto de: repertório peças teatrais, obras musicais

assembleia pessoas reunidas réstia alhos ou cebolas

alcateia lobos romanceiro poesias narrativas

acervo livros revoada pássaros

antologia trechos literários selecionados sínodo párocos

arquipélago ilhas talha lenha

banda músicos tropa muares, soldados

bando desordeiros ou malfeitores turma estudantes, trabalhadores

banca examinadores vara porcos

batalhão soldados
PORTUGUÊS

cardume peixes
caravana viajantes peregrinos
cacho frutas

24
3. Formação dos Substantivos A) Epicenos: referentes a animais. A distinção de sexo
se faz mediante a utilização das palavras “macho”
A) Substantivos Simples e Compostos e “fêmea”: a cobra macho e a cobra fêmea, o jacaré
Chuva - subst. Fem. 1 - água caindo em gotas sobre a terra. macho e o jacaré fêmea.
O substantivo chuva é formado por um único ele- B) Sobrecomuns: substantivos uniformes referentes
mento ou radical. É um substantivo simples. a pessoas de ambos os sexos: a criança, a teste-
A.1 Substantivo Simples: é aquele formado por um munha, a vítima, o cônjuge, o gênio, o ídolo, o in-
único elemento. divíduo.
Outros substantivos simples: tempo, sol, sofá, etc. Veja C) Comuns de Dois ou Comum de Dois Gêneros:
agora: O substantivo guarda-chuva é formado por dois ele- indicam o sexo das pessoas por meio do artigo: o
mentos (guarda + chuva). Esse substantivo é composto. colega e a colega, o doente e a doente, o artista e
A.2 Substantivo Composto: é aquele formado por a artista.
dois ou mais elementos. Outros exemplos: beija-flor, pas-
satempo. Substantivos de origem grega terminados em ema
ou oma são masculinos: o fonema, o poema, o sistema, o
sintoma, o teorema.
B) Substantivos Primitivos e Derivados  Existem certos substantivos que, variando de
B.1 Substantivo Primitivo: é aquele que não deriva gênero, variam em seu significado:
de nenhuma outra palavra da própria língua por- o águia (vigarista) e a águia (ave; perspicaz); o cabeça
tuguesa. (líder) e a cabeça (parte do corpo); o capital (dinheiro) e
B.2 Substantivo Derivado: é aquele que se origi- a capital (cidade); o coma (sono mórbido) e a coma (ca-
na de outra palavra. O substantivo limoeiro, por beleira, juba); o lente (professor) e a lente (vidro de au-
exemplo, é derivado, pois se originou a partir da mento); o moral (estado de espírito) e a moral (ética; con-
palavra limão. clusão); o praça (soldado raso) e a praça (área pública);
o rádio (aparelho receptor) e a rádio (estação emissora).
4. Flexão dos substantivos
6. Formação do Feminino dos Substantivos Bifor-
O substantivo é uma classe variável. A palavra é variá- mes
vel quando sofre flexão (variação). A palavra menino, por
exemplo, pode sofrer variações para indicar: Regra geral: troca-se a terminação -o por –a: aluno
Plural: meninos / Feminino: menina / Aumentativo: - aluna.
meninão / Diminutivo: menininho  Substantivos terminados em -ês: acrescenta-se -a
ao masculino: freguês - freguesa
A) Flexão de Gênero  Substantivos terminados em -ão: fazem o femini-
Gênero é um princípio puramente linguístico, não de- no de três formas:
vendo ser confundido com “sexo”. O gênero diz respeito 1. troca-se -ão por -oa. = patrão – patroa
a todos os substantivos de nossa língua, quer se refiram 2. troca-se -ão por -ã. = campeão - campeã
a seres animais providos de sexo, quer designem apenas 3. troca-se -ão por ona. = solteirão - solteirona
“coisas”: o gato/a gata; o banco, a casa. Exceções: barão – baronesa, ladrão - ladra, sultão -
Na língua portuguesa, há dois gêneros: masculino e sultana
feminino. Pertencem ao gênero masculino os substanti-
vos que podem vir precedidos dos artigos o, os, um, uns.  Substantivos terminados em -or:
Veja estes títulos de filmes: acrescenta-se -a ao masculino = doutor – doutora
O velho e o mar troca-se -or por -triz: = imperador – imperatriz
Um Natal inesquecível  Substantivos com feminino em -esa, -essa, -isa:
Os reis da praia cônsul - consulesa / abade - abadessa / poeta - poe-
tisa / duque - duquesa / conde - condessa / profeta
Pertencem ao gênero feminino os substantivos que - profetisa
podem vir precedidos dos artigos a, as, uma, umas:  Substantivos que formam o feminino trocando o
A história sem fim -e final por -a: elefante - elefanta
Uma cidade sem passado  Substantivos que têm radicais diferentes no mas-
As tartarugas ninjas culino e no feminino: bode – cabra / boi - vaca
 Substantivos que formam o feminino de maneira
5. Substantivos Biformes e Substantivos Unifor- especial, isto é, não seguem nenhuma das regras
mes anteriores: czar – czarina, réu - ré

1. Substantivos Biformes (= duas formas): apresen-


tam uma forma para cada gênero: gato – gata, ho-
PORTUGUÊS

mem – mulher, poeta – poetisa, prefeito - prefeita


2. Substantivos Uniformes: apresentam uma única
forma, que serve tanto para o masculino quanto
para o feminino. Classificam-se em:

25
7. Formação do Feminino dos Substantivos Uni- Femininos: a dinamite, a derme, a hélice, a omoplata,
formes a cataplasma, a pane, a mascote, a gênese, a entorse, a
libido, a cal, a faringe, a cólera (doença), a ubá (canoa).
Epicenos:
Novo jacaré escapa de policiais no rio Pinheiros. São geralmente masculinos os substantivos de ori-
gem grega terminados em -ma: o grama (peso), o quilo-
Não é possível saber o sexo do jacaré em questão. grama, o plasma, o apostema, o diagrama, o epigrama, o
Isso ocorre porque o substantivo jacaré tem apenas uma telefonema, o estratagema, o dilema, o teorema, o trema,
forma para indicar o masculino e o feminino. o eczema, o edema, o magma, o estigma, o axioma, o tra-
Alguns nomes de animais apresentam uma só for- coma, o hematoma.
ma para designar os dois sexos. Esses substantivos são Exceções: a cataplasma, a celeuma, a fleuma, etc.
chamados de epicenos. No caso dos epicenos, quando
houver a necessidade de especificar o sexo, utilizam-se Gênero dos Nomes de Cidades - Com raras exce-
palavras macho e fêmea. ções, nomes de cidades são femininos: A histórica Ouro
A cobra macho picou o marinheiro. Preto. / A dinâmica São Paulo. / A acolhedora Porto Ale-
A cobra fêmea escondeu-se na bananeira. gre. / Uma Londres imensa e triste.
Exceções: o Rio de Janeiro, o Cairo, o Porto, o Havre.
8. Sobrecomuns:
Entregue as crianças à natureza. 10. Gênero e Significação

A palavra crianças se refere tanto a seres do sexo Muitos substantivos, como já mencionado anterior-
masculino, quanto a seres do sexo feminino. Nesse caso, mente, têm uma significação no masculino e outra no fe-
nem o artigo nem um possível adjetivo permitem identi- minino. Observe: o baliza (soldado que à frente da tropa,
ficar o sexo dos seres a que se refere a palavra. Veja: indica os movimentos que se deve realizar em conjunto; o
A criança chorona chamava-se João. que vai à frente de um bloco carnavalesco, manejando um
A criança chorona chamava-se Maria. bastão), a baliza (marco, estaca; sinal que marca um limite
ou proibição de trânsito), o cabeça (chefe), a cabeça (par-
Outros substantivos sobrecomuns: te do corpo), o cisma (separação religiosa, dissidência), a
a criatura = João é uma boa criatura. Maria é uma cisma (ato de cismar, desconfiança), o cinza (a cor cinzen-
boa criatura. ta), a cinza (resíduos de combustão), o capital (dinheiro),
o cônjuge = O cônjuge de João faleceu. O cônjuge de a capital (cidade), o coma (perda dos sentidos), a coma
Marcela faleceu (cabeleira), o coral (pólipo, a cor vermelha, canto em coro),
a coral (cobra venenosa), o crisma (óleo sagrado, usado
9. Comuns de Dois Gêneros: na administração da crisma e de outros sacramentos), a
Motorista tem acidente idêntico 23 anos depois. crisma (sacramento da confirmação), o cura (pároco), a
cura (ato de curar), o estepe (pneu sobressalente), a estepe
Quem sofreu o acidente: um homem ou uma mulher? (vasta planície de vegetação), o guia (pessoa que guia ou-
É impossível saber apenas pelo título da notícia, uma tras), a guia (documento, pena grande das asas das aves),
vez que a palavra motorista é um substantivo uniforme. o grama (unidade de peso), a grama (relva), o caixa (fun-
A distinção de gênero pode ser feita através da análi- cionário da caixa), a caixa (recipiente, setor de pagamen-
se do artigo ou adjetivo, quando acompanharem o subs- tos), o lente (professor), a lente (vidro de aumento), o mo-
tantivo: o colega - a colega; o imigrante - a imigrante; ral (ânimo), a moral (honestidade, bons costumes, ética),
um jovem - uma jovem; artista famoso - artista famosa; o nascente (lado onde nasce o Sol), a nascente (a fonte),
repórter francês - repórter francesa. o maria-fumaça (trem como locomotiva a vapor), maria-
-fumaça (locomotiva movida a vapor), o pala (poncho), a
A palavra personagem é usada indistintamente nos pala (parte anterior do boné ou quepe, anteparo), o rádio
dois gêneros. Entre os escritores modernos nota-se (aparelho receptor), a rádio (emissora), o voga (remador),
acentuada preferência pelo masculino: O menino desco- a voga (moda).
briu nas nuvens os personagens dos contos de carochinha.
Com referência à mulher, deve-se preferir o feminino: B) Flexão de Número do Substantivo
O problema está nas mulheres de mais idade, que não
aceitam a personagem. Em português, há dois números gramaticais: o singu-
lar, que indica um ser ou um grupo de seres, e o plural,
Diz-se: o (ou a) manequim Marcela, o (ou a) modelo que indica mais de um ser ou grupo de seres. A caracte-
fotográfico Ana Belmonte. rística do plural é o “s” final.

Masculinos: o tapa, o eclipse, o lança-perfume, o dó 11. Plural dos Substantivos Simples


)pena), o sanduíche, o clarinete, o champanha, o sósia, o
PORTUGUÊS

maracajá, o clã, o herpes, o pijama, o suéter, o soprano, o Os substantivos terminados em vogal, ditongo oral e
proclama, o pernoite, o púbis. “n” fazem o plural pelo acréscimo de “s”: pai – pais; ímã –
ímãs; hífen - hifens (sem acento, no plural).
Exceção: cânon - cânones.

26
Os substantivos terminados em “m” fazem o plural A) Flexionam-se os dois elementos, quando for-
em “ns”: homem - homens. mados de:
Os substantivos terminados em “r” e “z” fazem o substantivo + substantivo = couve-flor e couves-flores
plural pelo acréscimo de “es”: revólver – revólveres; raiz substantivo + adjetivo = amor-perfeito e amores-per-
- raízes. feitos
adjetivo + substantivo = gentil-homem e gentis-ho-
Atenção: mens
O plural de caráter é caracteres. numeral + substantivo = quinta-feira e quintas-feiras

Os substantivos terminados em al, el, ol, ul flexio- B) Flexiona-se somente o segundo elemento,
nam-se no plural, trocando o “l” por “is”: quintal - quin- quando formados de:
tais; caracol – caracóis; hotel - hotéis. Exceções: mal e verbo + substantivo = guarda-roupa e guarda-roupas
males, cônsul e cônsules. palavra invariável + palavra variável = alto-falante e
Os substantivos terminados em “il” fazem o plural alto-falantes
de duas maneiras: palavras repetidas ou imitativas = reco-reco e reco-
1. Quando oxítonos, em “is”: canil - canis -recos
2. Quando paroxítonos, em “eis”: míssil - mísseis.
C) Flexiona-se somente o primeiro elemento,
Observação: quando formados de:
A palavra réptil pode formar seu plural de duas ma- substantivo + preposição clara + substantivo = água-
neiras: répteis ou reptis (pouco usada). -de-colônia e águas-de-colônia
substantivo + preposição oculta + substantivo = ca-
Os substantivos terminados em “s” fazem o plural valo-vapor e cavalos-vapor
de duas maneiras: substantivo + substantivo que funciona como deter-
1. Quando monossilábicos ou oxítonos, mediante o minante do primeiro, ou seja, especifica a função ou o
acréscimo de “es”: ás – ases / retrós - retroses tipo do termo anterior: palavra-chave - palavras-chave,
2. Quando paroxítonos ou proparoxítonos, ficam in- bomba-relógio - bombas-relógio, homem-rã - homens-
variáveis: o lápis - os lápis / o ônibus - os ônibus. -rã, peixe-espada - peixes-espada.

Os substantivos terminados em “ão” fazem o plural D) Permanecem invariáveis, quando formados de:
de três maneiras. verbo + advérbio = o bota-fora e os bota-fora
1. substituindo o -ão por -ões: ação - ações verbo + substantivo no plural = o saca-rolhas e os
2. substituindo o -ão por -ães: cão - cães saca-rolhas
3. substituindo o -ão por -ãos: grão - grãos
13. Casos Especiais
Observação:
Muitos substantivos terminados em “ão” apresen- o louva-a-deus e os louva-a-deus
tam dois – e até três – plurais:
aldeão – aldeões/aldeães/aldeãos an- o bem-te-vi e os bem-te-vis
cião – anciões/anciães/anciãos o bem-me-quer e os bem-me-queres
charlatão – charlatões/charlatães cor-
o joão-ninguém e os joões-ninguém.
rimão – corrimãos/corrimões
guardião – guardiões/guardiães vilão
14. Plural das Palavras Substantivadas
– vilãos/vilões/vilães
As palavras substantivadas, isto é, palavras de outras
Os substantivos terminados em “x” ficam invariá-
classes gramaticais usadas como substantivo apresen-
veis: o látex - os látex.
tam, no plural, as flexões próprias dos substantivos.
Pese bem os prós e os contras.
12. Plural dos Substantivos Compostos
O aluno errou na prova dos noves.
Ouça com a mesma serenidade os sins e os nãos.
A formação do plural dos substantivos compostos
depende da forma como são grafados, do tipo de pa-
Observação:
lavras que formam o composto e da relação que esta-
Numerais substantivados terminados em “s” ou “z”
belecem entre si. Aqueles que são grafados sem hífen
não variam no plural: Nas provas mensais consegui mui-
comportam-se como os substantivos simples: aguar-
tos seis e alguns dez.
dente/aguardentes, girassol/girassóis, pontapé/ponta-
pés, malmequer/malmequeres.
PORTUGUÊS

O plural dos substantivos compostos cujos elemen-


tos são ligados por hífen costuma provocar muitas dú-
vidas e discussões. Algumas orientações são dadas a
seguir:

27
15. Plural dos Diminutivos fosso fossos
Flexiona-se o substantivo no plural, retira-se o “s” fi- imposto impostos
nal e acrescenta-se o sufixo diminutivo. olho olhos
osso (ô) ossos (ó)
pãe(s) + zinhos = pãezinhos
ovo ovos
animai(s) + zinhos = animaizinhos
poço poços
botõe(s) + zinhos = botõezinhos
porto portos
chapéu(s) + zinhos = chapeuzinhos
posto postos
farói(s) + zinhos = faroizinhos
tijolo tijolos
tren(s) + zinhos = trenzinhos
colhere(s) + zinhas = colherezinhas Têm a vogal tônica fechada (ô): adornos, almoços,
bolsos, esposos, estojos, globos, gostos, polvos, rolos, so-
flore(s) + zinhas = florezinhas
ros, etc.
mão(s) + zinhas = mãozinhas
papéi(s) + zinhos = papeizinhos Observação:
Distinga-se molho (ô) = caldo (molho de carne), de
nuven(s) + zinhas = nuvenzinhas molho (ó) = feixe (molho de lenha).
funi(s) + zinhos = funizinhos
túnei(s) + zinhos = tuneizinhos Há substantivos que só se usam no singular: o sul, o
norte, o leste, o oeste, a fé, etc.
pai(s) + zinhos = paizinhos
pé(s) + zinhos = pezinhos Outros só no plural: as núpcias, os víveres, os pêsa-
mes, as espadas/os paus (naipes de baralho), as fezes.
pé(s) + zitos = pezitos
Outros, enfim, têm, no plural, sentido diferente do
singular: bem (virtude) e bens (riquezas), honra (probida-
16. Plural dos Nomes Próprios Personativos
de, bom nome) e honras (homenagem, títulos).
Devem-se pluralizar os nomes próprios de pessoas Usamos, às vezes, os substantivos no singular, mas
sempre que a terminação preste-se à flexão. com sentido de plural:
Os Napoleões também são derrotados. Aqui morreu muito negro.
As Raquéis e Esteres. Celebraram o sacrifício divino muitas vezes em cape-
las improvisadas.
17. Plural dos Substantivos Estrangeiros
C) Flexão de Grau do Substantivo
Substantivos ainda não aportuguesados devem ser
escritos como na língua original, acrescentando-se “s” Grau é a propriedade que as palavras têm de expri-
(exceto quando terminam em “s” ou “z”): os shows, os mir as variações de tamanho dos seres. Classifica-se em:
shorts, os jazz. 1. Grau Normal - Indica um ser de tamanho consi-
Substantivos já aportuguesados flexionam-se de derado normal. Por exemplo: casa
acordo com as regras de nossa língua: os clubes, os cho- 2. Grau Aumentativo - Indica o aumento do tama-
pes, os jipes, os esportes, as toaletes, os bibelôs, os garçons, nho do ser. Classifica-se em:
os réquiens. Analítico = o substantivo é acompanhado de um
Observe o exemplo: adjetivo que indica grandeza. Por exemplo: casa grande.
Este jogador faz gols toda vez que joga. Sintético = é acrescido ao substantivo um sufixo in-
O plural correto seria gois (ô), mas não se usa. dicador de aumento. Por exemplo: casarão.

18. Plural com Mudança de Timbre 3. Grau Diminutivo - Indica a diminuição do tama-
nho do ser. Pode ser:
Certos substantivos formam o plural com mudança Analítico = substantivo acompanhado de um adje-
de timbre da vogal tônica (o fechado / o aberto). É um tivo que indica pequenez. Por exemplo: casa pequena.
fato fonético chamado metafonia (plural metafônico). Sintético = é acrescido ao substantivo um sufixo in-
dicador de diminuição. Por exemplo: casinha.
Singular Plural REFERÊNCIAS BIBLIOGRÁFICAS
corpo (ô) corpos (ó) SACCONI, Luiz Antônio. Nossa gramática completa
PORTUGUÊS

Sacconi. 30.ª ed. Rev. São Paulo: Nova Geração, 2010.


esforço esforços Português linguagens: volume 1 / Wiliam Roberto Ce-
fogo fogos reja, Thereza Cochar Magalhães. – 7.ª ed. Reform. – São
Paulo: Saraiva, 2010.
forno fornos

28
CAMPEDELLI, Samira Yousseff. Português – Literatura, 1. Pronomes Pessoais
Produção de Texto & Gramática – Volume único / Samira
Yousseff Campedelli, Jésus Barbosa Souza. – 3.ª edição – São aqueles que substituem os substantivos, indican-
São Paulo: Saraiva, 2002. do diretamente as pessoas do discurso. Quem fala ou
escreve assume os pronomes “eu” ou “nós”; usa-se os
SITE pronomes “tu”, “vós”, “você” ou “vocês” para designar a
http://www.soportugues.com.br/secoes/morf/ quem se dirige, e “ele”, “ela”, “eles” ou “elas” para fazer
morf12.php referência à pessoa ou às pessoas de quem se fala.
Os pronomes pessoais variam de acordo com as fun-
ções que exercem nas orações, podendo ser do caso reto
Pronome
ou do caso oblíquo.

Pronome é a palavra variável que substitui ou acom- A) Pronome Reto


panha um substantivo (nome), qualificando-o de alguma Pronome pessoal do caso reto é aquele que, na sen-
forma. tença, exerce a função de sujeito: Nós lhe ofertamos
O homem julga que é superior à natureza, por isso o flores.
homem destrói a natureza... Os pronomes retos apresentam flexão de número,
Utilizando pronomes, teremos: O homem julga que é gênero (apenas na 3.ª pessoa) e pessoa, sendo essa úl-
superior à natureza, por isso ele a destrói... tima a principal flexão, uma vez que marca a pessoa do
Ficou melhor, sem a repetição desnecessária de ter- discurso. Dessa forma, o quadro dos pronomes retos é
mos (homem e natureza). assim configurado:
Grande parte dos pronomes não possuem significa- 1.ª pessoa do singular: eu
dos fixos, isto é, essas palavras só adquirem significação 2.ª pessoa do singular: tu
dentro de um contexto, o qual nos permite recuperar a 3.ª pessoa do singular: ele, ela
referência exata daquilo que está sendo colocado por 1.ª pessoa do plural: nós
meio dos pronomes no ato da comunicação. Com ex- 2.ª pessoa do plural: vós
ceção dos pronomes interrogativos e indefinidos, os de- 3.ª pessoa do plural: eles, elas
mais pronomes têm por função principal apontar para as
pessoas do discurso ou a elas se relacionar, indicando- Esses pronomes não costumam ser usados como
-lhes sua situação no tempo ou no espaço. Em virtude
complementos verbais na língua-padrão. Frases como
dessa característica, os pronomes apresentam uma for-
“Vi ele na rua”, “Encontrei ela na praça”, “Trouxeram eu
ma específica para cada pessoa do discurso.
até aqui”- comuns na língua oral cotidiana - devem ser
Minha carteira estava vazia quando eu fui assaltada.
evitadas na língua formal escrita ou falada. Na língua for-
[minha/eu: pronomes de 1.ª pessoa = aquele que fala]
mal, devem ser usados os pronomes oblíquos correspon-
Tua carteira estava vazia quando tu foste assaltada?
dentes: “Vi-o na rua”, “Encontrei-a na praça”, “Trouxeram-
[tua/tu: pronomes de 2.ª pessoa = aquele a quem se fala]
-me até aqui”.
A carteira dela estava vazia quando ela foi assaltada.
Frequentemente observamos a omissão do pronome
[dela/ela: pronomes de 3.ª pessoa = aquele de quem
se fala] reto em Língua Portuguesa. Isso se dá porque as próprias
formas verbais marcam, através de suas desinências, as
Em termos morfológicos, os pronomes são palavras pessoas do verbo indicadas pelo pronome reto: Fizemos
variáveis em gênero (masculino ou feminino) e em nú- boa viagem. (Nós)
mero (singular ou plural). Assim, espera-se que a refe-
rência através do pronome seja coerente em termos de B) Pronome Oblíquo
gênero e número (fenômeno da concordância) com o Pronome pessoal do caso oblíquo é aquele que, na
seu objeto, mesmo quando este se apresenta ausente no sentença, exerce a função de complemento verbal
enunciado. (objeto direto ou indireto): Ofertaram-nos flores. (ob-
Fala-se de Roberta. Ele quer participar do desfile da jeto indireto)
nossa escola neste ano.
[nossa: pronome que qualifica “escola” = concordân- Observação:
cia adequada] O pronome oblíquo é uma forma variante do prono-
[neste: pronome que determina “ano” = concordân- me pessoal do caso reto. Essa variação indica a função
cia adequada] diversa que eles desempenham na oração: pronome reto
[ele: pronome que faz referência à “Roberta” = con- marca o sujeito da oração; pronome oblíquo marca o
cordância inadequada] complemento da oração. Os pronomes oblíquos sofrem
variação de acordo com a acentuação tônica que pos-
Existem seis tipos de pronomes: pessoais, possessivos, suem, podendo ser átonos ou tônicos.
demonstrativos, indefinidos, relativos e interrogativos.
PORTUGUÊS

2. Pronome Oblíquo Átono


São chamados átonos os pronomes oblíquos que não
são precedidos de preposição. Possuem acentuação tô-
nica fraca: Ele me deu um presente.

29
Lista dos pronomes oblíquos átonos Não vá sem eu mandar.
1.ª pessoa do singular (eu): me A frase: “Foi fácil para mim resolver aquela questão!”
2.ª pessoa do singular (tu): te está correta, já que “para mim” é complemento de “fá-
3.ª pessoa do singular (ele, ela): o, a, lhe cil”. A ordem direta seria: Resolver aquela questão foi fácil
1.ª pessoa do plural (nós): nos para mim!
2.ª pessoa do plural (vós): vos A combinação da preposição “com” e alguns prono-
3.ª pessoa do plural (eles, elas): os, as, lhes mes originou as formas especiais comigo, contigo, consi-
go, conosco e convosco. Tais pronomes oblíquos tônicos
frequentemente exercem a função de adjunto adverbial
FIQUE ATENTO! de companhia: Ele carregava o documento consigo.
Os pronomes o, os, a, as assumem formas es- A preposição “até” exige as formas oblíquas tônicas:
peciais depois de certas terminações verbais: Ela veio até mim, mas nada falou.
1. Quando o verbo termina em -z, -s ou -r, Mas, se “até” for palavra denotativa (com o sentido de
o pronome assume a forma lo, los, la ou las, inclusão), usaremos as formas retas: Todos foram bem na
ao mesmo tempo que a terminação verbal é prova, até eu! (= inclusive eu)
suprimida. Por exemplo:
fiz + o = fi-lo As formas “conosco” e “convosco” são substituídas
fazeis + o = fazei-lo por “com nós” e “com vós” quando os pronomes pes-
dizer + a = dizê-la soais são reforçados por palavras como outros, mesmos,
próprios, todos, ambos ou algum numeral.
2. Quando o verbo termina em som nasal, o Você terá de viajar com nós todos.
pronome assume as formas no, nos, na, nas. Estávamos com vós outros quando chegaram as más
Por exemplo: notícias.
viram + o: viram-no Ele disse que iria com nós três.
repõe + os = repõe-nos
retém + a: retém-na 3. Pronome Reflexivo
tem + as = tem-nas São pronomes pessoais oblíquos que, embora fun-
cionem como objetos direto ou indireto, referem-se ao
sujeito da oração. Indicam que o sujeito pratica e recebe
B.2 Pronome Oblíquo Tônico a ação expressa pelo verbo.
Os pronomes oblíquos tônicos são sempre precedidos Lista dos pronomes reflexivos:
por preposições, em geral as preposições a, para, de e com. 1.ª pessoa do singular (eu): me, mim = Eu não me
Por esse motivo, os pronomes tônicos exercem a função lembro disso.
de objeto indireto da oração. Possuem acentuação tônica 2.ª pessoa do singular (tu): te, ti = Conhece a ti mesmo.
forte. 3.ª pessoa do singular (ele, ela): se, si, consigo = Gui-
Lista dos pronomes oblíquos tônicos: lherme já se preparou.
1.ª pessoa do singular (eu): mim, comigo Ela deu a si um presente.
2.ª pessoa do singular (tu): ti, contigo Antônio conversou consigo mesmo.
3.ª pessoa do singular (ele, ela): si, consigo, ele, ela
1.ª pessoa do plural (nós): nós, conosco 1.ª pessoa do plural (nós): nos = Lavamo-nos no rio.
2.ª pessoa do plural (vós): vós, convosco 2.ª pessoa do plural (vós): vos = Vós vos beneficiastes
3.ª pessoa do plural (eles, elas): si, consigo, eles, elas com esta conquista.
3.ª pessoa do plural (eles, elas): se, si, consigo = Eles se
Observe que as únicas formas próprias do pronome tô- conheceram. / Elas deram a si um dia de folga.
nico são a primeira pessoa (mim) e segunda pessoa (ti). As
demais repetem a forma do pronome pessoal do caso reto.
#FicaDica
As preposições essenciais introduzem sempre pronomes
O pronome é reflexivo quando se refere à mes-
pessoais do caso oblíquo e nunca pronome do caso reto. Nos
ma pessoa do pronome subjetivo (sujeito): Eu
contextos interlocutivos que exigem o uso da língua formal,
me arrumei e saí.
os pronomes costumam ser usados desta forma:
É pronome recíproco quando indica recipro-
Não há mais nada entre mim e ti.
cidade de ação: Nós nos amamos. / Olhamo-
Não se comprovou qualquer ligação entre ti e ela.
-nos calados.
Não há nenhuma acusação contra mim.
O “se” pode ser usado como palavra expletiva
Não vá sem mim.
ou partícula de realce, sem ser rigorosamente
necessária e sem função sintática: Os explora-
Há construções em que a preposição, apesar de surgir
dores riam-se de suas tentativas. / Será que eles
anteposta a um pronome, serve para introduzir uma oração
PORTUGUÊS

se foram?
cujo verbo está no infinitivo. Nesses casos, o verbo pode ter
sujeito expresso; se esse sujeito for um pronome, deverá
ser do caso reto.
Trouxeram vários vestidos para eu experimentar.

30
C) Pronomes de Tratamento Quando você vier, eu te abraçarei e enrolar-me-ei nos
São pronomes utilizados no tratamento formal, ceri- teus cabelos. (errado)
monioso. Apesar de indicarem nosso interlocutor (por-
tanto, a segunda pessoa), utilizam o verbo na terceira Quando você vier, eu a abraçarei e enrolar-me-ei nos
pessoa. Alguns exemplos: seus cabelos. (correto) = terceira pessoa do singular
Vossa Alteza (V. A.) = príncipes, duques
Vossa Eminência (V. E.ma) = cardeais ou
Vossa Reverendíssima (V. Ver.ma) = sacerdotes e religio-
sos em geral Quando tu vieres, eu te abraçarei e enrolar-me-ei nos
Vossa Excelência (V. Ex.ª) = oficiais de patente superior teus cabelos. (correto) = segunda pessoa do singular
à de coronel, senadores, deputados, embaixadores, profes-
sores de curso superior, ministros de Estado e de Tribunais, 4. Pronomes Possessivos
governadores, secretários de Estado, presidente da Repú-
blica (sempre por extenso) São palavras que, ao indicarem a pessoa gramatical
Vossa Magnificência (V. Mag.ª) = reitores de universida- (possuidor), acrescentam a ela a ideia de posse de algo
des (coisa possuída).
Vossa Majestade (V. M.) = reis, rainhas e imperadores Este caderno é meu. (meu = possuidor: 1.ª pessoa do
Vossa Senhoria (V. S.a) = comerciantes em geral, oficiais singular)
até a patente de coronel, chefes de seção e funcionários de
igual categoria
Vossa Meretíssima (sempre por extenso) = para juízes NÚMERO PESSOA PRONOME
de direito singular primeira meu(s), minha(s)
Vossa Santidade (sempre por extenso) = tratamento ce-
rimonioso singular segunda teu(s), tua(s)
Vossa Onipotência (sempre por extenso) = Deus singular terceira seu(s), sua(s)
Também são pronomes de tratamento o senhor, a se-
plural primeira nosso(s), nossa(s)
nhora e você, vocês. “O senhor” e “a senhora” são emprega-
dos no tratamento cerimonioso; “você” e “vocês”, no trata- plural segunda vosso(s), vossa(s)
mento familiar. Você e vocês são largamente empregados plural terceira seu(s), sua(s)
no português do Brasil; em algumas regiões, a forma tu é
de uso frequente; em outras, pouco empregada. Já a forma Note que:
vós tem uso restrito à linguagem litúrgica, ultraformal ou A forma do possessivo depende da pessoa gramatical
literária.
a que se refere; o gênero e o número concordam com o
objeto possuído: Ele trouxe seu apoio e sua contribuição
Observações:
naquele momento difícil.
1. Vossa Excelência X Sua Excelência: os pronomes de
tratamento que possuem “Vossa(s)” são empregados
Observações:
em relação à pessoa com quem falamos: Espero que
1. A forma “seu” não é um possessivo quando resul-
V. Ex.ª, Senhor Ministro, compareça a este encontro.
tar da alteração fonética da palavra senhor: Muito
2. Emprega-se “Sua (s)” quando se fala a respeito da
pessoa: Todos os membros da C.P.I. afirmaram que obrigado, seu José.
Sua Excelência, o Senhor Presidente da República, agiu 2. Os pronomes possessivos nem sempre indicam
com propriedade. posse. Podem ter outros empregos, como:
3. Os pronomes de tratamento representam uma forma A) indicar afetividade: Não faça isso, minha filha.
indireta de nos dirigirmos aos nossos interlocutores. B) indicar cálculo aproximado: Ele já deve ter seus 40
Ao tratarmos um deputado por Vossa Excelência, por anos.
exemplo, estamos nos endereçando à excelência que C) atribuir valor indefinido ao substantivo: Marisa tem
esse deputado supostamente tem para poder ocupar lá seus defeitos, mas eu gosto muito dela.
o cargo que ocupa. 3. Em frases onde se usam pronomes de tratamento,
4. Embora os pronomes de tratamento dirijam-se à 2.ª o pronome possessivo fica na 3.ª pessoa: Vossa Ex-
pessoa, toda a concordância deve ser feita com a celência trouxe sua mensagem?
3.ª pessoa. Assim, os verbos, os pronomes possessi- 4. Referindo-se a mais de um substantivo, o possessi-
vos e os pronomes oblíquos empregados em relação vo concorda com o mais próximo: Trouxe-me seus
a eles devem ficar na 3.ª pessoa. livros e anotações.
Basta que V. Ex.ª cumpra a terça parte das suas promes- 5. Em algumas construções, os pronomes pessoais
sas, para que seus eleitores lhe fiquem reconhecidos. oblíquos átonos assumem valor de possessivo: Vou
5. Uniformidade de Tratamento: quando escrevemos seguir-lhe os passos. (= Vou seguir seus passos)
ou nos dirigimos a alguém, não é permitido mudar, 6. O adjetivo “respectivo” equivale a “devido, seu, pró-
ao longo do texto, a pessoa do tratamento escolhi- prio”, por isso não se deve usar “seus” ao utilizá-lo,
PORTUGUÊS

da inicialmente. Assim, por exemplo, se começamos para que não ocorra redundância: Coloque tudo
a chamar alguém de “você”, não poderemos usar nos respectivos lugares.
“te” ou “teu”. O uso correto exigirá, ainda, verbo na
terceira pessoa.

31
5. Pronomes Demonstrativos Os pronomes demonstrativos podem ser variáveis ou
invariáveis, observe:
São utilizados para explicitar a posição de certa pa- Variáveis: este(s), esta(s), esse(s), essa(s), aquele(s),
lavra em relação a outras ou ao contexto. Essa relação aquela(s).
pode ser de espaço, de tempo ou em relação ao discurso. Invariáveis: isto, isso, aquilo.
Também aparecem como pronomes demonstrativos:
A) Em relação ao espaço:  o(s), a(s): quando estiverem antecedendo o “que”
Este(s), esta(s) e isto = indicam o que está perto da e puderem ser substituídos por aquele(s), aquela(s),
pessoa que fala: aquilo.
Este material é meu. Não ouvi o que disseste. (Não ouvi aquilo que disseste.)
Essa rua não é a que te indiquei. (não é aquela que te
Esse(s), essa(s) e isso = indicam o que está perto da indiquei.)
pessoa com quem se fala:
Esse material em sua carteira é seu?  mesmo(s), mesma(s), próprio(s), própria(s):
variam em gênero quando têm caráter reforçativo:
Aquele(s), aquela(s) e aquilo = indicam o que está Estas são as mesmas pessoas que o procuraram ontem.
distante tanto da pessoa que fala como da pessoa com Eu mesma refiz os exercícios.
quem se fala: Elas mesmas fizeram isso.
Aquele material não é nosso. Eles próprios cozinharam.
Vejam aquele prédio! Os próprios alunos resolveram o problema.

B) Em relação ao tempo:  semelhante(s): Não tenha semelhante atitude.


Este(s), esta(s) e isto = indicam o tempo presente em  tal, tais: Tal absurdo eu não cometeria.
relação à pessoa que fala: 1. Em frases como: O referido deputado e o Dr. Alcides
Esta manhã farei a prova do concurso! eram amigos íntimos; aquele casado, solteiro este.
(ou então: este solteiro, aquele casado) - este se re-
Esse(s), essa(s) e isso = indicam o tempo passado, po- fere à pessoa mencionada em último lugar; aquele,
rém relativamente próximo à época em que se situa a à mencionada em primeiro lugar.
pessoa que fala: 2. O pronome demonstrativo tal pode ter conotação
Essa noite dormi mal; só pensava no concurso! irônica: A menina foi a tal que ameaçou o professor?
Aquele(s), aquela(s) e aquilo = indicam um afastamen- 3. Pode ocorrer a contração das preposições a, de, em
to no tempo, referido de modo vago ou como tempo com pronome demonstrativo: àquele, àquela, deste,
remoto: desta, disso, nisso, no, etc: Não acreditei no que esta-
Naquele tempo, os professores eram valorizados. va vendo. (no = naquilo)

C) Em relação ao falado ou escrito (ou ao que se 6. Pronomes Indefinidos


falará ou escreverá):
Este(s), esta(s) e isto = empregados quando se quer São palavras que se referem à 3.ª pessoa do discur-
fazer referência a alguma coisa sobre a qual ainda se fa- so, dando-lhe sentido vago (impreciso) ou expressando
lará: quantidade indeterminada.
Serão estes os conteúdos da prova: análise sintática, Alguém entrou no jardim e destruiu as mudas recém-
ortografia, concordância. -plantadas.
Não é difícil perceber que “alguém” indica uma pessoa
Esse(s), essa(s) e isso = utilizados quando se pretende de quem se fala (uma terceira pessoa, portanto) de forma
fazer referência a alguma coisa sobre a qual já se falou: imprecisa, vaga. É uma palavra capaz de indicar um ser
Sua aprovação no concurso, isso é o que mais deseja- humano que seguramente existe, mas cuja identidade é
mos! desconhecida ou não se quer revelar. Classificam-se em:

Este e aquele são empregados quando se quer fazer A) Pronomes Indefinidos Substantivos: assumem o
referência a termos já mencionados; aquele se refere ao lugar do ser ou da quantidade aproximada de seres
termo referido em primeiro lugar e este para o referido na frase. São eles: algo, alguém, fulano, sicrano, bel-
por último: trano, nada, ninguém, outrem, quem, tudo.
Algo o incomoda?
Domingo, no Pacaembu, jogarão Palmeiras e São Pau- Quem avisa amigo é.
lo; este está mais bem colocado que aquele. (= este [São
Paulo], aquele [Palmeiras]) B) Pronomes Indefinidos Adjetivos: qualificam um
ser expresso na frase, conferindo-lhe a noção de
ou quantidade aproximada. São eles: cada, certo(s),
PORTUGUÊS

certa(s).
Domingo, no Pacaembu, jogarão Palmeiras e São Pau- Cada povo tem seus costumes.
lo; aquele está mais bem colocado que este. (= este [São Certas pessoas exercem várias profissões.
Paulo], aquele [Palmeiras])

32
Note que:
Ora são pronomes indefinidos substantivos, ora pro- Observe:
nomes indefinidos adjetivos: Pronomes relativos variáveis = o qual, cujo, quanto, os
algum, alguns, alguma(s), bastante(s) (= muito, mui- quais, cujos, quantos, a qual, cuja, quanta, as quais, cujas,
tos), demais, mais, menos, muito(s), muita(s), nenhum, ne- quantas.
nhuns, nenhuma(s), outro(s), outra(s), pouco(s), pouca(s), Pronomes relativos invariáveis = quem, que, onde.
qualquer, quaisquer, qual, que, quanto(s), quanta(s), tal,
tais, tanto(s), tanta(s), todo(s), toda(s), um, uns, uma(s), Note que:
vários, várias. O pronome “que” é o relativo de mais largo empre-
Menos palavras e mais ações. go, sendo por isso chamado relativo universal. Pode ser
Alguns se contentam pouco. substituído por o qual, a qual, os quais, as quais, quando
seu antecedente for um substantivo.
Os pronomes indefinidos podem ser divididos em va- O trabalho que eu fiz refere-se à corrupção. (= o qual)
riáveis e invariáveis. Observe: A cantora que acabou de se apresentar é péssima. (=
 Variáveis = algum, nenhum, todo, muito, pouco, a qual)
vário, tanto, outro, quanto, alguma, nenhuma, toda, Os trabalhos que eu fiz referem-se à corrupção. (= os
muita, pouca, vária, tanta, outra, quanta, qualquer, quais)
quaisquer*, alguns, nenhuns, todos, muitos, poucos, As cantoras que se apresentaram eram péssimas. (=
vários, tantos, outros, quantos, algumas, nenhumas, as quais)
todas, muitas, poucas, várias, tantas, outras, quan-
tas. O qual, os quais, a qual e as quais são exclusivamente
 Invariáveis = alguém, ninguém, outrem, tudo, pronomes relativos, por isso são utilizados didaticamen-
nada, algo, cada. te para verificar se palavras como “que”, “quem”, “onde”
(que podem ter várias classificações) são pronomes rela-
*Qualquer é composto de qual + quer (do verbo que- tivos. Todos eles são usados com referência à pessoa ou
coisa por motivo de clareza ou depois de determinadas
rer), por isso seu plural é quaisquer (única palavra cujo
preposições: Regressando de São Paulo, visitei o sítio de
plural é feito em seu interior).
minha tia, o qual me deixou encantado. O uso de “que”,
Todo e toda no singular e junto de artigo significa in-
neste caso, geraria ambiguidade. Veja: Regressando de
teiro; sem artigo, equivale a qualquer ou a todas as:
São Paulo, visitei o sítio de minha tia, que me deixou en-
Toda a cidade está enfeitada. (= a cidade inteira)
cantado (quem me deixou encantado: o sítio ou minha
Toda cidade está enfeitada. (= todas as cidades)
tia?).
Trabalho todo o dia. (= o dia inteiro)
Essas são as conclusões sobre as quais pairam muitas
Trabalho todo dia. (= todos os dias)
dúvidas? (com preposições de duas ou mais sílabas utili-
za-se o qual / a qual)
São locuções pronominais indefinidas: cada qual,
cada um, qualquer um, quantos quer (que), quem quer O relativo “que” às vezes equivale a o que, coisa que,
(que), seja quem for, seja qual for, todo aquele (que), tal e se refere a uma oração: Não chegou a ser padre, mas
qual (= certo), tal e qual, tal ou qual, um ou outro, uma deixou de ser poeta, que era a sua vocação natural.
ou outra, etc.
Cada um escolheu o vinho desejado. O pronome “cujo”: exprime posse; não concorda com
o seu antecedente (o ser possuidor), mas com o conse-
7. Pronomes Relativos quente (o ser possuído, com o qual concorda em gêne-
ro e número); não se usa artigo depois deste pronome;
São aqueles que representam nomes já mencionados “cujo” equivale a do qual, da qual, dos quais, das quais.
anteriormente e com os quais se relacionam. Introduzem Existem pessoas cujas ações são nobres.
as orações subordinadas adjetivas. (antecedente) (consequente)
O racismo é um sistema que afirma a superioridade de
um grupo racial sobre outros. Se o verbo exigir preposição, esta virá antes do pro-
(afirma a superioridade de um grupo racial sobre ou- nome: O autor, a cujo livro você se referiu, está aqui! (re-
tros = oração subordinada adjetiva). feriu-se a)

O pronome relativo “que” refere-se à palavra “sis- “Quanto” é pronome relativo quando tem por ante-
tema” e introduz uma oração subordinada. Diz-se que cedente um pronome indefinido: tanto (ou variações) e
a palavra “sistema” é antecedente do pronome relativo tudo:
que.
O antecedente do pronome relativo pode ser o pro- Emprestei tantos quantos foram neces-
nome demonstrativo o, a, os, as. sários.
PORTUGUÊS

Não sei o que você está querendo dizer. (antecedente)


Às vezes, o antecedente do pronome relativo não vem
expresso. Ele fez tudo quanto ha-
Quem casa, quer casa. via falado.

33
(antecedente) sabia se devia ajudar... Ajudar quem? Você (lhe).
O pronome “quem” se refere a pessoas e vem sempre Os pronomes pessoais oblíquos podem ser átonos ou
precedido de preposição. tônicos: os primeiros não são precedidos de preposição,
diferentemente dos segundos, que são sempre precedi-
É um professor a quem mui- dos de preposição.
to devemos. A) Pronome oblíquo átono: Joana me perguntou o
(preposição) que eu estava fazendo.
B) Pronome oblíquo tônico: Joana perguntou para
“Onde”, como pronome relativo, sempre possui ante- mim o que eu estava fazendo.
cedente e só pode ser utilizado na indicação de lugar: A
casa onde morava foi assaltada. REFERÊNCIAS BIBLIOGRÁFICAS
SACCONI, Luiz Antônio. Nossa gramática completa
Na indicação de tempo, deve-se empregar quando ou Sacconi. 30.ª ed. Rev. São Paulo: Nova Geração, 2010.
em que: Sinto saudades da época em que (quando) morá- Português linguagens: volume 2 / Wiliam Roberto Ce-
vamos no exterior. reja, Thereza Cochar Magalhães. – 7.ª ed. Reform. – São
Paulo: Saraiva, 2010.
Podem ser utilizadas como pronomes relativos as pa- Português: novas palavras: literatura, gramática, reda-
lavras: ção / Emília Amaral... [et al.]. – São Paulo: FTD, 2000.
 como (= pelo qual) – desde que precedida das CAMPEDELLI, Samira Yousseff. Português – Literatura,
palavras modo, maneira ou forma: Produção de Texto & Gramática – Volume único / Samira
Não me parece correto o modo como você agiu sema- Yousseff Campedelli, Jésus Barbosa Souza. – 3.ª edição –
na passada. São Paulo: Saraiva, 2002.

 quando (= em que) – desde que tenha como SITE


antecedente um nome que dê ideia de tempo: http://www.soportugues.com.br/secoes/morf/
Bons eram os tempos quando podíamos jogar video- morf42.php
game.
9. Colocação Pronominal
Os pronomes relativos permitem reunir duas orações
numa só frase. Colocação Pronominal trata da correta colocação dos
O futebol é um esporte. / O povo gosta muito deste pronomes oblíquos átonos na frase.
esporte.
= O futebol é um esporte de que o povo gosta muito.
#FicaDica
Numa série de orações adjetivas coordenadas, pode
ocorrer a elipse do relativo “que”: A sala estava cheia de Pronome Oblíquo é aquele que exerce a fun-
gente que conversava, (que) ria, observava. ção de complemento verbal (objeto). Por isso,
memorize:
8. Pronomes Interrogativos OBlíquo = OBjeto!

São usados na formulação de perguntas, sejam elas


diretas ou indiretas. Assim como os pronomes indefini- Embora na linguagem falada a colocação dos prono-
dos, referem-se à 3.ª pessoa do discurso de modo im- mes não seja rigorosamente seguida, algumas normas
preciso. São pronomes interrogativos: que, quem, qual (e devem ser observadas na linguagem escrita.
variações), quanto (e variações).
Com quem andas? Próclise = É a colocação pronominal antes do verbo.
Qual seu nome? A próclise é usada:
Diz-me com quem andas, que te direi quem és.
 Quando o verbo estiver precedido de palavras
O pronome pessoal é do caso reto quando tem fun- que atraem o pronome para antes do verbo. São elas:
ção de sujeito na frase. O pronome pessoal é do caso A) Palavras de sentido negativo: não, nunca, ninguém,
oblíquo quando desempenha função de complemento. jamais, etc.: Não se desespere!
1. Eu não sei essa matéria, mas ele irá me ajudar. B) Advérbios: Agora se negam a depor.
2. Maria foi embora para casa, pois não sabia se devia C) Conjunções subordinativas: Espero que me expli-
lhe ajudar. quem tudo!
Na primeira oração os pronomes pessoais “eu” e “ele” D) Pronomes relativos: Venceu o concurseiro que se
exercem função de sujeito, logo, são pertencentes ao esforçou.
caso reto. Já na segunda oração, o pronome “lhe” exerce E) Pronomes indefinidos: Poucos te deram a oportu-
PORTUGUÊS

função de complemento (objeto), ou seja, caso oblíquo. nidade.


Os pronomes pessoais indicam as pessoas do discur- F) Pronomes demonstrativos: Isso me magoa muito.
so. O pronome oblíquo “lhe”, da segunda oração, aponta
para a segunda pessoa do singular (tu/você): Maria não

34
11. Emprego de o, a, os, as
 Orações iniciadas por palavras interrogativas:
Quem lhe disse isso?  Em verbos terminados em vogal ou ditongo oral,
 Orações iniciadas por palavras exclamativas: os pronomes: o, a, os, as não se alteram.
Quanto se ofendem! Chame-o agora.
 Orações que exprimem desejo (orações optativas): Deixei-a mais tranquila.
Que Deus o ajude.
 A próclise é obrigatória quando se utiliza o pro-  Em verbos terminados em r, s ou z, estas consoan-
nome reto ou sujeito expresso: Eu lhe entregarei o tes finais alteram-se para lo, la, los, las. Exemplos:
material amanhã. / Tu sabes cantar? (Encontrar) Encontrá-lo é o meu maior sonho.
(Fiz) Fi-lo porque não tinha alternativa.
Mesóclise = É a colocação pronominal no meio do
verbo. A mesóclise é usada:  Em verbos terminados em ditongos nasais (am,
em, ão, õe), os pronomes o, a, os, as alteram-se
Quando o verbo estiver no futuro do presente ou fu- para no, na, nos, nas.
turo do pretérito, contanto que esses verbos não estejam Chamem-no agora.
precedidos de palavras que exijam a próclise. Exemplos: Põe-na sobre a mesa.
Realizar-se-á, na próxima semana, um grande evento em
prol da paz no mundo.
Repare que o pronome está “no meio” do verbo “rea-
lizará”: realizar – SE – á. Se houvesse na oração alguma #FicaDica
palavra que justificasse o uso da próclise, esta prevalece-
Dica da Zê!
ria. Veja: Não se realizará...
Próclise – pró lembra pré; pré é prefixo que
Não fossem os meus compromissos, acompanhar-te-ia
significa “antes”! Pronome antes do verbo!
nessa viagem.
Ênclise – “en” lembra, pelo “som”, /Ənd/ (end,
(com presença de palavra que justifique o uso de pró-
em Inglês – que significa “fim, final!). Pronome
clise: Não fossem os meus compromissos, EU te acompa-
depois do verbo!
nharia nessa viagem).
Mesóclise – pronome oblíquo no Meio do
verbo
Ênclise = É a colocação pronominal depois do verbo.
A ênclise é usada quando a próclise e a mesóclise não
forem possíveis:
REFERÊNCIAS BIBLIOGRÁFICAS
 Quando o verbo estiver no imperativo afirmativo:
Quando eu avisar, silenciem-se todos. SACCONI, Luiz Antônio. Nossa gramática completa
 Quando o verbo estiver no infinitivo impessoal: Sacconi. 30.ª ed. Rev. São Paulo: Nova Geração, 2010.
Não era minha intenção machucá-la. Português linguagens: volume 3 / Wiliam Roberto Ce-
 Quando o verbo iniciar a oração. (até porque não reja, Thereza Cochar Magalhães. – 7.ª ed. Reform. – São
se inicia período com pronome oblíquo). Paulo: Saraiva, 2010.
Vou-me embora agora mesmo.
Levanto-me às 6h. SITE
 Quando houver pausa antes do verbo: Se eu passo http://www.portugues.com.br/gramatica/colocacao-
no concurso, mudo-me hoje mesmo! -pronominal-.html
 Quando o verbo estiver no gerúndio: Recusou a
proposta fazendo-se de desentendida. Observação: Não foram encontradas questões
abrangendo tal conteúdo.
10. Colocação pronominal nas locuções verbais
VERBO
 Após verbo no particípio = pronome depois do
verbo auxiliar (e não depois do particípio): Verbo é a palavra que se flexiona em pessoa, número,
Tenho me deliciado com a leitura! tempo e modo. A estes tipos de flexão verbal dá-se o
Eu tenho me deliciado com a leitura! nome de conjugação (por isso também se diz que verbo
Eu me tenho deliciado com a leitura! é a palavra que pode ser conjugada). Pode indicar, entre
 Não convém usar hífen nos tempos compostos e outros processos: ação (amarrar), estado (sou), fenôme-
nas locuções verbais: no (choverá); ocorrência (nascer); desejo (querer).
Vamos nos unir!
Iremos nos manifestar. 1. Estrutura das Formas Verbais
 Quando há um fator para próclise nos tempos
compostos ou locuções verbais: opção pelo uso Do ponto de vista estrutural, o verbo pode apresentar
PORTUGUÊS

do pronome oblíquo “solto” entre os verbos = Não os seguintes elementos:


vamos nos preocupar (e não: “não nos vamos preo- A) Radical: é a parte invariável, que expressa o signi-
cupar”). ficado essencial do verbo. Por exemplo: fal-ei; fal-
-ava; fal-am. (radical fal-)

35
B) Tema: é o radical seguido da vogal temática que
indica a conjugação a que pertence o verbo. Por #FicaDica
exemplo: fala-r. São três as conjugações:
1.ª - Vogal Temática - A - (falar), 2.ª - Vogal Temática Observe que, retirando os radicais, as desi-
- E - (vender), 3.ª - Vogal Temática - I - (partir). nências modo-temporal e número-pessoal
C) Desinência modo-temporal: é o elemento que mantiveram-se idênticas. Tente fazer com
designa o tempo e o modo do verbo. Por exemplo: outro verbo e perceberá que se repetirá o
falávamos (indica o pretérito imperfeito do indicativo) fato (desde que o verbo seja da primeira
/ falasse ( indica o pretérito imperfeito do subjuntivo) conjugação e regular!). Faça com o verbo
D) Desinência número-pessoal: é o elemento que “andar”, por exemplo. Substitua o radical
designa a pessoa do discurso (1.ª, 2.ª ou 3.ª) e o “cant” e coloque o “and” (radical do verbo
número (singular ou plural): andar). Viu? Fácil!
falamos (indica a 1.ª pessoa do plural.) / falavam
(indica a 3.ª pessoa do plural.)
B) Irregulares: são aqueles cuja flexão provoca alterações
no radical ou nas desinências: faço, fiz, farei, fizesse.
FIQUE ATENTO!
O verbo pôr, assim como seus derivados Observação:
(compor, repor, depor), pertencem à 2.ª conju- Alguns verbos sofrem alteração no radical apenas
gação, pois a forma arcaica do verbo pôr era para que seja mantida a sonoridade. É o caso de: corrigir/
poer. A vogal “e”, apesar de haver desapareci- corrijo, fingir/finjo, tocar/toquei, por exemplo. Tais altera-
do do infinitivo, revela-se em algumas formas ções não caracterizam irregularidade, porque o fonema
do verbo: põe, pões, põem, etc. permanece inalterado.

C) Defectivos: são aqueles que não apresentam con-


2. Formas Rizotônicas e Arrizotônicas jugação completa. Os principais são adequar, pre-
caver, computar, reaver, abolir, falir.
Ao combinarmos os conhecimentos sobre a estrutura D) Impessoais: são os verbos que não têm sujeito
dos verbos com o conceito de acentuação tônica, perce- e, normalmente, são usados na terceira pessoa do
bemos com facilidade que nas formas rizotônicas o acen- singular. Os principais verbos impessoais são:
to tônico cai no radical do verbo: opino, aprendam, amo,
por exemplo. Nas formas arrizotônicas, o acento tônico 1. Haver, quando sinônimo de existir, acontecer, reali-
não cai no radical, mas sim na terminação verbal (fora do zar-se ou fazer (em orações temporais).
radical): opinei, aprenderão, amaríamos. Havia muitos candidatos no dia da prova. (Havia =
Existiam)
3. Classificação dos Verbos Houve duas guerras mundiais. (Houve = Acontece-
ram)
Haverá debates hoje. (Haverá = Realizar-se-ão)
Classificam-se em:
Viajei a Madri há muitos anos. (há = faz)
A) Regulares: são aqueles que apresentam o radi-
cal inalterado durante a conjugação e desinências
2. Fazer, ser e estar (quando indicam tempo)
idênticas às de todos os verbos regulares da mes-
Faz invernos rigorosos na Europa.
ma conjugação. Por exemplo: comparemos os ver-
Era primavera quando o conheci.
bos “cantar” e “falar”, conjugados no presente do
Estava frio naquele dia.
Modo Indicativo:
3. Todos os verbos que indicam fenômenos da natu-
canto falo reza são impessoais: chover, ventar, nevar, gear, tro-
vejar, amanhecer, escurecer, etc. Quando, porém,
cantas falas
se constrói, “Amanheci cansado”, usa-se o verbo
canta falas “amanhecer” em sentido figurado. Qualquer verbo
cantamos falamos impessoal, empregado em sentido figurado, dei-
xa de ser impessoal para ser pessoal, ou seja, terá
cantais falais conjugação completa.
cantam falam Amanheci cansado. (Sujeito desinencial: eu)
Choveram candidatos ao cargo. (Sujeito: candidatos)
Fiz quinze anos ontem. (Sujeito desinencial: eu)

4. O verbo passar (seguido de preposição), indicando


tempo: Já passa das seis.
PORTUGUÊS

5. Os verbos bastar e chegar, seguidos da preposição


“de”, indicando suficiência:
Basta de tolices.

36
Chega de promessas.
6. Os verbos estar e ficar em orações como “Está bem, Está muito bem assim, Não fica bem, Fica mal”, sem referência
a sujeito expresso anteriormente (por exemplo: “ele está mal”). Podemos, nesse caso, classificar o sujeito como
hipotético, tornando-se, tais verbos, pessoais.

7. O verbo dar + para da língua popular, equivalente de “ser possível”. Por exemplo:
Não deu para chegar mais cedo.
Dá para me arrumar uma apostila?

E) Unipessoais: são aqueles que, tendo sujeito, conjugam-se apenas nas terceiras pessoas, do singular e do plural.
São unipessoais os verbos constar, convir, ser (= preciso, necessário) e todos os que indicam vozes de animais
(cacarejar, cricrilar, miar, latir, piar).

Os verbos unipessoais podem ser usados como verbos pessoais na linguagem figurada:
Teu irmão amadureceu bastante.
O que é que aquela garota está cacarejando?

Principais verbos unipessoais:

 Cumprir, importar, convir, doer, aprazer, parecer, ser (preciso, necessário):


Cumpre estudarmos bastante. (Sujeito: estudarmos bastante)
Parece que vai chover. (Sujeito: que vai chover)
É preciso que chova. (Sujeito: que chova)

 Fazer e ir, em orações que dão ideia de tempo, seguidos da conjunção que.
Faz dez anos que viajei à Europa. (Sujeito: que viajei à Europa)
Vai para (ou Vai em ou Vai por) dez anos que não a vejo. (Sujeito: que não a vejo)

F) Abundantes: são aqueles que possuem duas ou mais formas equivalentes, geralmente no particípio, em que,
além das formas regulares terminadas em -ado ou -ido, surgem as chamadas formas curtas (particípio irregular).
O particípio regular (terminado em “–do”) é utilizado na voz ativa, ou seja, com os verbos ter e haver; o irregular é
empregado na voz passiva, ou seja, com os verbos ser, ficar e estar. Observe:

Infinitivo Particípio Regular Particípio Irregular


Aceitar Aceitado Aceito
Acender Acendido Aceso
Anexar Anexado Anexo
Benzer Benzido Bento
Corrigir Corrigido Correto
Dispersar Dispersado Disperso
Eleger Elegido Eleito
Envolver Envolvido Envolto
Imprimir Imprimido Impresso
Inserir Inserido Inserto
Limpar Limpado Limpo
Matar Matado Morto
Misturar Misturado Misto
Morrer Morrido Morto
Murchar Murchado Murcho
Pegar Pegado Pego
Romper Rompido Roto
Soltar Soltado Solto
PORTUGUÊS

Suspender Suspendido Suspenso


Tingir Tingido Tinto
Vagar Vagado Vago

37
FIQUE ATENTO!
Estes verbos e seus derivados possuem, apenas, o particípio irregular: abrir/aberto, cobrir/coberto, dizer/
dito, escrever/escrito, pôr/posto, ver/visto, vir/vindo.

G) Anômalos: são aqueles que incluem mais de um radical em sua conjugação. Existem apenas dois: ser (sou, sois,
fui) e ir (fui, ia, vades).

H) Auxiliares: São aqueles que entram na formação dos tempos compostos e das locuções verbais. O verbo prin-
cipal (aquele que exprime a ideia fundamental, mais importante), quando acompanhado de verbo auxiliar, é
expresso numa das formas nominais: infinitivo, gerúndio ou particípio.
Vou espantar todos!
(verbo auxiliar) (verbo principal no infinitivo)

Está chegando a hora!


(verbo auxiliar) (verbo principal no gerúndio)

Observação:
Os verbos auxiliares mais usados são: ser, estar, ter e haver.

4. Conjugação dos Verbos Auxiliares

4.1. SER - Modo Indicativo

Presente Pret.Perfeito Pret. Imp. Pret.mais-que-perf. Fut.do Pres. Fut. Do Pretérito


sou fui era fora serei seria
és foste eras foras serás serias
é foi era fora será seria
somos fomos éramos fôramos seremos seríamos
sois fostes éreis fôreis sereis seríeis
são foram eram foram serão seriam

4.2. SER - Modo Subjuntivo

Presente Pretérito Imperfeito Futuro


que eu seja se eu fosse quando eu for
que tu sejas se tu fosses quando tu fores
que ele seja se ele fosse quando ele for
que nós sejamos se nós fôssemos quando nós formos
que vós sejais se vós fôsseis quando vós fordes
que eles sejam se eles fossem quando eles forem

4.3. SER - Modo Imperativo

Afirmativo Negativo
sê tu não sejas tu
seja você não seja você
sejamos nós não sejamos nós
sede vós não sejais vós
PORTUGUÊS

sejam vocês não sejam vocês

38
4.4. SER - Formas Nominais

Infinitivo Impessoal Infinitivo Pessoal Gerúndio Particípio


ser ser eu sendo sido
seres tu
ser ele
sermos nós
serdes vós
serem eles

4.5. ESTAR - Modo Indicativo



Presente Pret. perf. Pret. Imp. Pret.mais-q-perf. Fut.doPres. Fut.do Preté.
estou estive estava estivera estarei estaria
estás estiveste estavas estiveras estarás estarias
está esteve estava estivera estará estaria
estamos estivemos estávamos estivéramos estaremos estaríamos
estais estivestes estáveis estivéreis estareis estaríeis
estão estiveram estavam estiveram estarão estariam

4.6. ESTAR - Modo Subjuntivo e Imperativo

Presente Pretérito Imperfeito Futuro Afirmativo Negativo


esteja estivesse estiver
estejas estivesses estiveres está estejas
esteja estivesse estiver esteja esteja
estejamos estivéssemos estivermos estejamos estejamos
estejais estivésseis estiverdes estai estejais
estejam estivessem estiverem estejam estejam

4.7. ESTAR - Formas Nominais

Infinitivo Impessoal Infinitivo Pessoal Gerúndio Particípio


estar estar estando estado
estares
estar
estarmos
estardes
estarem

4.8. HAVER - Modo Indicativo

Presente Pret. Perf. Pret. Imp. Pret.Mais-Q-Perf. Fut.do Pres. Fut.doPreté.


hei houve havia houvera haverei haveria
hás houveste havias houveras haverás haverias
há houve havia houvera haverá haveria
PORTUGUÊS

havemos houvemos havíamos houvéramos haveremos haveríamos


haveis houvestes havíeis houvéreis havereis haveríeis
hão houveram haviam houveram haverão haveriam

39
4.9. HAVER - Modo Subjuntivo e Imperativo

Presente Pretérito Imperfeito Futuro Afirmativo Negativo


ja houvesse houver
hajas houvesses houveres há hajas
haja houvesse houver haja haja
hajamos houvéssemos houvermos hajamos hajamos
hajais houvésseis houverdes havei hajais
hajam houvessem houverem hajam hajam

4.10. HAVER - Formas Nominais

Infinitivo Impessoal Infinitivo Pessoal Gerúndio Particípio


haver haver havendo havido
haveres
haver
havermos
haverdes
Haverem

4.11. TER - Modo Indicativo

Presente Pret. Perf. Pret. Imp. Preté.mais-q-perf. Fut. Do Pres. Fut. Do Preté.
tenho tive tinha tivera terei teria
tens tiveste tinhas tiveras terás terias
tem teve tinha tivera terá teria
temos tivemos tínhamos tivéramos teremos teríamos
tendes tivestes tínheis tivéreis tereis teríeis
têm tiveram tinham tiveram terão teriam

4.12. TER - Modo Subjuntivo e Imperativo

Presente Pretérito Imperfeito Futuro Afirmativo Negativo


tenha tivesse tiver
tenhas tivesses tiveres tem tenhas
tenha tivesse tiver tenha tenha
tenhamos tivéssemos tivermos tenhamos tenhamos
Tenhais tivésseis tiverdes tende tenhais
tenham tivessem tiverem tenham tenham

I) Pronominais: São aqueles verbos que se conjugam com os pronomes oblíquos átonos me, te, se, nos, vos, se, na
mesma pessoa do sujeito, expressando reflexibilidade (pronominais acidentais) ou apenas reforçando a ideia já
implícita no próprio sentido do verbo (pronominais essenciais). Veja:
 Essenciais: são aqueles que sempre se conjugam com os pronomes oblíquos me, te, se, nos, vos, se. São poucos:
abster-se, ater-se, apiedar-se, atrever-se, dignar-se, arrepender-se, etc. Nos verbos pronominais essenciais a refle-
xibilidade já está implícita no radical do verbo. Por exemplo: Arrependi-me de ter estado lá.
PORTUGUÊS

A ideia é de que a pessoa representada pelo sujeito (eu) tem um sentimento (arrependimento) que recai sobre ela
mesma, pois não recebe ação transitiva nenhuma vinda do verbo; o pronome oblíquo átono é apenas uma partícula
integrante do verbo, já que, pelo uso, sempre é conjugada com o verbo. Diz-se que o pronome apenas serve de reforço
da ideia reflexiva expressa pelo radical do próprio verbo. Veja uma conjugação pronominal essencial (verbo e respec-
tivos pronomes):

40
Eu me arrependo, Tu te arrependes, Ele se arrepende, Nós nos arrependemos, Vós vos arrependeis, Eles se arrependem.
 Acidentais: são aqueles verbos transitivos diretos em que a ação exercida pelo sujeito recai sobre o objeto re-
presentado por pronome oblíquo da mesma pessoa do sujeito; assim, o sujeito faz uma ação que recai sobre ele
mesmo. Em geral, os verbos transitivos diretos ou transitivos diretos e indiretos podem ser conjugados com os
pronomes mencionados, formando o que se chama voz reflexiva. Por exemplo: A garota se penteava.
A reflexibilidade é acidental, pois a ação reflexiva pode ser exercida também sobre outra pessoa: A garota penteou-
-me.

Por fazerem parte integrante do verbo, os pronomes oblíquos átonos dos verbos pronominais não possuem função
sintática.
Há verbos que também são acompanhados de pronomes oblíquos átonos, mas que não são essencialmente prono-
minais - são os verbos reflexivos. Nos verbos reflexivos, os pronomes, apesar de se encontrarem na pessoa idêntica à
do sujeito, exercem funções sintáticas. Por exemplo:
Eu me feri. = Eu (sujeito) – 1.ª pessoa do singular; me (objeto direto) – 1.ª pessoa do singular.

5. Modos Verbais

Dá-se o nome de modo às várias formas assumidas pelo verbo na expressão de um fato certo, real, verdadeiro.
Existem três modos:
A) Indicativo - indica uma certeza, uma realidade: Eu estudo para o concurso.
B) Subjuntivo - indica uma dúvida, uma possibilidade: Talvez eu estude amanhã.
C) Imperativo - indica uma ordem, um pedido: Estude, colega!

6. Formas Nominais

Além desses três modos, o verbo apresenta ainda formas que podem exercer funções de nomes (substantivo, adje-
tivo, advérbio), sendo por isso denominadas formas nominais. Observe:

A) Infinitivo
A.1 Impessoal: exprime a significação do verbo de modo vago e indefinido, podendo ter valor e função de subs-
tantivo. Por exemplo:
Viver é lutar. (= vida é luta)
É indispensável combater a corrupção. (= combate à)

O infinitivo impessoal pode apresentar-se no presente (forma simples) ou no passado (forma composta). Por exem-
plo:
É preciso ler este livro.
Era preciso ter lido este livro.

A.2 Infinitivo Pessoal: é o infinitivo relacionado às três pessoas do discurso. Na 1.ª e 3.ª pessoas do singular, não
apresenta desinências, assumindo a mesma forma do impessoal; nas demais, flexiona-se da seguinte maneira:
2.ª pessoa do singular: Radical + ES = teres (tu)
1.ª pessoa do plural: Radical + MOS = termos (nós)
2.ª pessoa do plural: Radical + DES = terdes (vós)
3.ª pessoa do plural: Radical + EM = terem (eles)
Foste elogiado por teres alcançado uma boa colocação.

B) Gerúndio: o gerúndio pode funcionar como adjetivo ou advérbio. Por exemplo:


Saindo de casa, encontrei alguns amigos. (função de advérbio)
Água fervendo, pele ardendo. (função de adjetivo)

Na forma simples (1), o gerúndio expressa uma ação em curso; na forma composta (2), uma ação concluída:
Trabalhando (1), aprenderás o valor do dinheiro.
Tendo trabalhado (2), aprendeu o valor do dinheiro.

Quando o gerúndio é vício de linguagem (gerundismo), ou seja, uso exagerado e inadequado do gerúndio:
1. Enquanto você vai ao mercado, vou estar jogando futebol.
PORTUGUÊS

2. – Sim, senhora! Vou estar verificando!


Em 1, a locução “vou estar” + gerúndio é adequada, pois transmite a ideia de uma ação que ocorre no momento da
outra; em 2, essa ideia não ocorre, já que a locução verbal “vou estar verificando” refere-se a um futuro em andamento,
exigindo, no caso, a construção “verificarei” ou “vou verificar”.

41
C) Particípio: quando não é empregado na formação dos tempos compostos, o particípio indica, geralmente, o re-
sultado de uma ação terminada, flexionando-se em gênero, número e grau. Por exemplo: Terminados os exames,
os candidatos saíram.
Quando o particípio exprime somente estado, sem nenhuma relação temporal, assume verdadeiramente a função
de adjetivo. Por exemplo: Ela é a aluna escolhida pela turma.

(Ziraldo)

8. Tempos Verbais

Tomando-se como referência o momento em que se fala, a ação expressa pelo verbo pode ocorrer em diversos
tempos.

A) Tempos do Modo Indicativo


Presente - Expressa um fato atual: Eu estudo neste colégio.
Pretérito Imperfeito - Expressa um fato ocorrido num momento anterior ao atual, mas que não foi completamente
terminado: Ele estudava as lições quando foi interrompido.
Pretérito Perfeito - Expressa um fato ocorrido num momento anterior ao atual e que foi totalmente terminado: Ele
estudou as lições ontem à noite.
Pretérito-mais-que-perfeito - Expressa um fato ocorrido antes de outro fato já terminado: Ele já estudara as lições
quando os amigos chegaram. (forma simples).
Futuro do Presente - Enuncia um fato que deve ocorrer num tempo vindouro com relação ao momento atual: Ele
estudará as lições amanhã.
Futuro do Pretérito - Enuncia um fato que pode ocorrer posteriormente a um determinado fato passado: Se ele
pudesse, estudaria um pouco mais.

B) Tempos do Modo Subjuntivo


Presente - Enuncia um fato que pode ocorrer no momento atual: É conveniente que estudes para o exame.
Pretérito Imperfeito - Expressa um fato passado, mas posterior a outro já ocorrido: Eu esperava que ele vencesse
o jogo.
Futuro do Presente - Enuncia um fato que pode ocorrer num momento futuro em relação ao atual: Quando ele vier
à loja, levará as encomendas.

FIQUE ATENTO!
Há casos em que formas verbais de um determinado tempo podem ser utilizadas para indicar outro.
Em 1500, Pedro Álvares Cabral descobre o Brasil.
descobre = forma do presente indicando passado ( = descobrira/descobriu)

No próximo final de semana, faço a prova!


faço = forma do presente indicando futuro ( = farei)
PORTUGUÊS

42
Tabelas das Conjugações Verbais

1. Modo Indicativo

1.1. Presente do Indicativo

1.ª conjugação 2.ª conjugação 3.ª conjugação Desinência pessoal


CANTAR VENDER PARTIR
cantO vendO partO O
cantaS vendeS parteS S
canta vende parte -
cantaMOS vendeMOS partiMOS MOS
cantaIS vendeIS partIS IS
cantaM vendeM parteM M

1.2. Pretérito Perfeito do Indicativo

1.ª conjugação 2.ª conjugação 3.ª conjugação Desinência pessoal


CANTAR VENDER PARTIR
canteI vendI partI I
cantaSTE vendeSTE partISTE STE
cantoU vendeU partiU U
cantaMOS vendeMOS partiMOS MOS
cantaSTES vendeSTES partISTES STES
cantaRAM vendeRAM partiRAM RAM

1.3. Pretérito mais-que-perfeito

1.ª conjugação 2.ª conjugação 3.ª conjugação Des. temporal Desinência pessoal
1.ª/2.ª e 3.ª conj.
CANTAR VENDER PARTIR
cantaRA vendeRA partiRA RA Ø
cantaRAS vendeRAS partiRAS RA S
cantaRA vendeRA partiRA RA Ø
cantáRAMOS vendêRAMOS partíRAMOS RA MOS
cantáREIS vendêREIS partíREIS RE IS
cantaRAM vendeRAM partiRAM RA M

1.4. Pretérito Imperfeito do Indicativo

1.ª conjugação 2.ª conjugação 3ª. conjugação


CANTAR VENDER PARTIR
cantAVA vendIA partIA
cantAVAS vendIAS partAS
CantAVA vendIA partIA
PORTUGUÊS

cantÁVAMOS vendÍAMOS partÍAMOS


cantÁVEIS vendÍEIS partÍEIS
cantAVAM vendIAM partIAM

43
1.5. Futuro do Presente do Indicativo

1.ª conjugação 2.ª conjugação 3.ª conjugação


CANTAR VENDER PARTIR
cantar ei vender ei partir ei
cantar ás vender ás partir ás
cantar á vender á partir á
cantar emos vender emos partir emos
cantar eis vender eis partir eis
cantar ão vender ão partir ão

1.6. Futuro do Pretérito do Indicativo

1.ª conjugação 2.ª conjugação 3.ª conjugação


CANTAR VENDER PARTIR
cantarIA venderIA partirIA
cantarIAS venderIAS partirIAS
cantarIA venderIA partirIA
cantarÍAMOS venderÍAMOS partirÍAMOS
cantarÍEIS venderÍEIS partirÍEIS
cantarIAM venderIAM partirIAM

1.7. Presente do Subjuntivo

Para se formar o presente do subjuntivo, substitui-se a desinência -o da primeira pessoa do singular do presente do
indicativo pela desinência -E (nos verbos de 1.ª conjugação) ou pela desinência -A (nos verbos de 2.ª e 3.ª conjugação).

1.ª conjug. 2.ª conjug. 3.ª conju. Desinên. pessoal Des. temporal Des.temporal
1.ª conj. 2.ª/3.ª conj.
CANTAR VENDER PARTIR
cantE vendA partA E A Ø
cantES vendAS partAS E A S
cantE vendA partA E A Ø
cantEMOS vendAMOS partAMOS E A MOS
cantEIS vendAIS partAIS E A IS
cantEM vendAM partAM E A M
PORTUGUÊS

44
1.8. Pretérito Imperfeito do Subjuntivo

Para formar o imperfeito do subjuntivo, elimina-se a desinência -STE da 2.ª pessoa do singular do pretérito perfeito,
obtendo-se, assim, o tema desse tempo. Acrescenta-se a esse tema a desinência temporal -SSE mais a desinência de
número e pessoa correspondente.

1.ª conjugação 2.ª conjugação 3.ª conjugação Des. temporal Desin. pessoal
1.ª /2.ª e 3.ª conj.
CANTAR VENDER PARTIR
cantaSSE vendeSSE partiSSE SSE Ø
cantaSSES vendeSSES partiSSES SSE S
cantaSSE vendeSSE partiSSE SSE Ø
cantáSSEMOS vendêSSEMOS partíSSEMOS SSE MOS
cantáSSEIS vendêSSEIS partíSSEIS SSE IS
cantaSSEM vendeSSEM partiSSEM SSE M

1.9. Futuro do Subjuntivo

Para formar o futuro do subjuntivo elimina-se a desinência -STE da 2.ª pessoa do singular do pretérito perfeito,
obtendo-se, assim, o tema desse tempo. Acrescenta-se a esse tema a desinência temporal -R mais a desinência de
número e pessoa correspondente.

1.ª conjugação 2.ª conjugação 3.ª conjugação Des. temporal Desin. pessoal
1.ª /2.ª e 3.ª conj.
CANTAR VENDER PARTIR
cantaR vendeR partiR Ø
cantaRES vendeRES partiRES R ES
cantaR vendeR partiR Ø
cantaRMOS vendeRMOS partiRMOS R MOS
cantaRDES vendeRDES partiRDES R DES
cantaREM vendeREM partiREM R EM

C) Modo Imperativo

1. Imperativo Afirmativo

Para se formar o imperativo afirmativo, toma-se do presente do indicativo a 2.ª pessoa do singular (tu) e a segunda
pessoa do plural (vós) eliminando-se o “S” final. As demais pessoas vêm, sem alteração, do presente do subjuntivo. Veja:

Presente do Indicativo Imperativo Afirmativo Presente do Subjuntivo


Eu canto --- Que eu cante
Tu cantas CantA tu Que tu cantes
Ele canta Cante você Que ele cante
Nós cantamos Cantemos nós Que nós cantemos
Vós cantais CantAI vós Que vós canteis
Eles cantam Cantem vocês Que eles cantem
PORTUGUÊS

45
2. Imperativo Negativo

Para se formar o imperativo negativo, basta antecipar a negação às formas do presente do subjuntivo.

Presente do Subjuntivo Imperativo Negativo

Que eu cante ---


Que tu cantes Não cantes tu
Que ele cante Não cante você
Que nós cantemos Não cantemos nós
Que vós canteis Não canteis vós
Que eles cantem Não cantem eles

 No modo imperativo não faz sentido usar na 3.ª pessoa (singular e plural) as formas ele/eles, pois uma ordem,
pedido ou conselho só se aplicam diretamente à pessoa com quem se fala. Por essa razão, utiliza-se você/vocês.
 O verbo SER, no imperativo, faz excepcionalmente: sê (tu), sede (vós).

3. Infinitivo Pessoal

1.ª conjugação 2.ª conjugação 3.ª conjugação


CANTAR VENDER PARTIR
cantar vender partir
cantarES venderES partirES
cantar vender partir
cantarMOS venderMOS partirMOS
cantarDES venderDES partirDES
cantarEM venderEM partirEM

 O verbo parecer admite duas construções:


Elas parecem gostar de você. (forma uma locução verbal)
Elas parece gostarem de você. (verbo com sujeito oracional, correspondendo à construção: parece gostarem de você).

 O verbo pegar possui dois particípios (regular e irregular):


Elvis tinha pegado minhas apostilas.
Minhas apostilas foram pegas.

REFERÊNCIAS BIBLIOGRÁFICAS
SACCONI, Luiz Antônio. Nossa gramática completa Sacconi. 30.ª ed. Rev. São Paulo: Nova Geração, 2010.
Português linguagens: volume 2 / Wiliam Roberto Cereja, Thereza Cochar Magalhães. – 7.ª ed. Reform. – São Paulo:
Saraiva, 2010.
Português: novas palavras: literatura, gramática, redação / Emília Amaral... [et al.]. – São Paulo: FTD, 2000.

SITE
http://www.soportugues.com.br/secoes/morf/morf54.php

VOZES DO VERBO

Dá-se o nome de voz à maneira como se apresenta a ação expressa pelo verbo em relação ao sujeito, indicando
se este é paciente ou agente da ação. Importante lembrar que voz verbal não é flexão, mas aspecto verbal. São três as
vozes verbais:

A) Ativa = quando o sujeito é agente, isto é, pratica a ação expressa pelo verbo:
PORTUGUÊS

Ele fez o trabalho.


sujeito agente ação objeto (paciente)

46
B) Passiva = quando o sujeito é paciente, recebendo a ação expressa pelo verbo:
O trabalho foi feito por ele.
sujeito paciente ação agente da passiva

C) Reflexiva = quando o sujeito é, ao mesmo tempo, agente e paciente, isto é, pratica e recebe a ação:
O menino feriu-se.

#FicaDica
Não confundir o emprego reflexivo do verbo com a noção de reciprocidade:
Os lutadores feriram-se. (um ao outro)
Nós nos amamos. (um ama o outro)

1. Formação da Voz Passiva

A voz passiva pode ser formada por dois processos: analítico e sintético.
A) Voz Passiva Analítica = Constrói-se da seguinte maneira:
Verbo SER + particípio do verbo principal. Por exemplo:
A escola será pintada pelos alunos. (na ativa teríamos: os alunos pintarão a escola)
O trabalho é feito por ele. (na ativa: ele faz o trabalho)

Observações:
 O agente da passiva geralmente é acompanhado da preposição por, mas pode ocorrer a construção com a pre-
posição de. Por exemplo: A casa ficou cercada de soldados.
 Pode acontecer de o agente da passiva não estar explícito na frase: A exposição será aberta amanhã.
 A variação temporal é indicada pelo verbo auxiliar (SER), pois o particípio é invariável. Observe a transformação
das frases seguintes:

Ele fez o trabalho. (pretérito perfeito do Indicativo)


O trabalho foi feito por ele. (verbo ser no pretérito perfeito do Indicativo, assim como o verbo principal da voz ativa)

Ele faz o trabalho. (presente do indicativo)


O trabalho é feito por ele. (ser no presente do indicativo)

Ele fará o trabalho. (futuro do presente)


O trabalho será feito por ele. (futuro do presente)

 Nas frases com locuções verbais, o verbo SER assume o mesmo tempo e modo do verbo principal da voz ativa.
Observe a transformação da frase seguinte:
O vento ia levando as folhas. (gerúndio)
As folhas iam sendo levadas pelo vento. (gerúndio)

B) Voz Passiva Sintética = A voz passiva sintética - ou pronominal - constrói-se com o verbo na 3.ª pessoa, seguido
do pronome apassivador “se”. Por exemplo:
Abriram-se as inscrições para o concurso.
Destruiu-se o velho prédio da escola.

Observação:
O agente não costuma vir expresso na voz passiva sintética.

1.1 Conversão da Voz Ativa na Voz Passiva

Pode-se mudar a voz ativa na passiva sem alterar substancialmente o sentido da frase.
O concurseiro comprou a apostila. (Voz Ativa)
Sujeito da Ativa objeto Direto
PORTUGUÊS

A apostila foi comprada pelo concurseiro. (Voz Passiva)


Sujeito da Passiva Agente da Passiva

47
Observe que o objeto direto será o sujeito da passiva; 2. (TST – Analista Judiciário – Área Apoio Especiali-
o sujeito da ativa passará a agente da passiva, e o verbo zado – Especialidade Medicina do Trabalho – FCC –
ativo assumirá a forma passiva, conservando o mesmo 2012) Está inadequado o emprego do elemento subli-
tempo. nhado na seguinte frase:
Os mestres têm constantemente aconselhado os alunos.
Os alunos têm sido constantemente aconselhados pelos a) Sou ateu e peço que me deem tratamento similar ao
mestres. que dispenso aos homens religiosos.
b) A intolerância religiosa baseia-se em preconceitos de
Eu o acompanharei. que deveriam desviar-se todos os homens verdadeira-
Ele será acompanhado por mim. mente virtuosos.
Quando o sujeito da voz ativa for indeterminado, não c) A tolerância é uma virtude na qual não podem prescin-
haverá complemento agente na passiva. Por exemplo: dir os que se dizem homens de fé.
Prejudicaram-me. / Fui prejudicado.
d) O ateu desperta a ira dos fanáticos, a despeito de nada
Com os verbos neutros (nascer, viver, morrer, dormir,
fazer que possa injuriá-los ou desrespeitá-los.
acordar, sonhar, etc.) não há voz ativa, passiva ou reflexiva,
e) Respeito os homens de fé, a menos que deixem de
porque o sujeito não pode ser visto como agente, pacien-
fazer o mesmo com aqueles que não a têm.
te ou agente paciente.
Resposta: Letra C.
REFERÊNCIAS BIBLIOGRÁFICAS Corrigindo o inadequado:
SACCONI, Luiz Antônio. Nossa gramática completa Em “a”: Sou ateu e peço que me deem tratamento si-
Sacconi. 30.ª ed. Rev. São Paulo: Nova Geração, 2010. milar ao que dispenso aos homens religiosos.
Português linguagens: volume 2 / Wiliam Roberto Ce- Em “b”: A intolerância religiosa baseia-se em precon-
reja, Thereza Cochar Magalhães. – 7.ª ed. Reform. – São ceitos de que deveriam desviar-se todos os homens
Paulo: Saraiva, 2010. verdadeiramente virtuosos.
Português: novas palavras: literatura, gramática, reda- Em “c”: A tolerância é uma virtude na qual (de que)
ção / Emília Amaral... [et al.]. – São Paulo: FTD, 2000. não podem prescindir os que se dizem homens de fé.
Em “d”: O ateu desperta a ira dos fanáticos, a despeito
SITE de nada fazer que possa injuriá-los ou desrespeitá-los.
http://www.soportugues.com.br/secoes/morf/morf54. Em “e”: Respeito os homens de fé, a menos que dei-
php xem de fazer o mesmo com aqueles que não a têm.

3. (TST – Analista Judiciário – Área Apoio Especiali-


zado – Especialidade Medicina do Trabalho – FCC –
EXERCÍCIOS COMENTADOS 2012)
Transpondo-se para a voz passiva a construção Os ateus
1. (TST – Técnico Judiciário – Área Administrativa – FCC despertariam a ira de qualquer fanático, a forma ver-
– 2012) As vitórias no jogo interior talvez não acrescen- bal obtida será:
tem novos troféus, mas elas trazem recompensas valiosas,
[...] que contribuem de forma significativa para nosso su- a) seria despertada.
cesso posterior, tanto na quadra como fora dela. b) teria sido despertada.
c) despertar-se-á.
Mantêm-se adequados o emprego de tempos e modos
d) fora despertada.
verbais e a correlação entre eles, ao se substituírem os
e) teriam despertado.
elementos sublinhados na frase acima, na ordem dada,
por:
a) tivessem acrescentado − trariam − contribuírem Resposta: Letra A.
b) acrescentassem − têm trazido − contribuírem Os ateus despertariam a ira de qualquer fanático
c) tinham acrescentado − trarão − contribuiriam Fazendo a transposição para a voz passiva, temos: A
d) acrescentariam − trariam− contribuíram ira de qualquer fanático seria despertada pelos ateus.
e) tenham acrescentado − trouxeram − Contribuíram GABARITO OFICIAL: A

Resposta: Letra E. 4. (TST – Técnico Judiciário – Área Administrativa –


Questão que envolve correlação verbal. Realizando as Especialidade Segurança Judiciária – FCC – 2012)
alterações solicitadas, segue como ficariam (em des- ...ela nunca alcançava a musa.
taque): Transpondo-se a frase acima para a voz passiva, a forma
Em “a”: tivessem acrescentado – trariam − contribui- verbal resultante será:
riam
Em “b”: acrescentassem – trariam − contribuiriam a) alcança-se.
Em “c”: tinham acrescentado – trouxeram − contribuí- b) foi alcançada.
PORTUGUÊS

ram c) fora alcançada.


Em “d”: acrescentassem – trariam − contribuíram d) seria alcançada.
Em “e”: tenham acrescentado – trouxeram − Contribuí- e) era alcançada.
ram = correta

48
Resposta: Letra E. 7. (TRT 23.ª REGIÃO-MT – Analista Judiciário – Área
Temos um verbo na voz ativa, então teremos dois Administrativa – FCC – 2016 ) ... para quem Manoel de
na passiva (auxiliar + o verbo da oração da ativa, no Barros era comparável a São Francisco de Assis...
mesmo tempo verbal, forma particípio): A musa nun- O verbo flexionado nos mesmos tempo e modo que o da
ca era alcançada por ela. O verbo “alcançava” está no frase acima está em:
pretérito imperfeito, por isso o auxiliar tem que estar
também (é = presente, foi = pretérito perfeito, era = a) Dizia-se um “vedor de cinema”...
imperfeito, fora = mais que perfeito, será = futuro do b) Porque não seria certo ficar pregando moscas no es-
presente, seria = futuro do pretérito). paço...
c) Na juventude, apaixonou-se por Arthur Rimbaud e
5. (TST – Analista Judiciário – Área Apoio Especiali- Charles Baudelaire.
zado – Especialidade Medicina do Trabalho – FCC – d) Quase meio século separa a estreia de Manoel de Bar-
2012) Aos poucos, contudo, fui chegando à constatação ros na literatura...
de que todo perfil de rede social é um retrato ideal de nós e) ... para depois casá-las...
mesmos.
Mantendo-se a correção e a lógica, sem que outra al- Resposta: Letra A.
teração seja feita na frase, o elemento grifado pode ser “Era” = verbo “ser” no pretérito imperfeito do Indicati-
substituído por: vo. Procuremos nos itens:
a) ademais. Em “a”: Dizia-se = pretérito imperfeito do Indicativo
b) conquanto. Em “b”: Porque não seria = futuro do pretérito do In-
c) porquanto. dicativo
d) entretanto. Em “c”: Na juventude, apaixonou-se = pretérito perfei-
e) apesar. to do Indicativo
Em “d”: Quase meio século separa = presente do Indi-
Resposta: Letra D. cativo
Contudo é uma conjunção adversativa (expressa opo- Em “e”: para depois casá-las = Infinitivo pessoal (casar
sição). A substituição deve utilizar outra de mesma elas)
classificação, para que se mantenha a ideia do perío-
do. A correta é entretanto. 8. (TRT 20.ª REGIÃO-SE – Analista Judiciário – Área
Administrativa – FCC – 2016 ) Aí conheci o escritor e
6. (TST – Analista Judiciário – Área Administrativa – historiador de sua gente, meu saudoso amigo Alcino Al-
FCC – 2012) O verbo indicado entre parênteses deverá ves Costa. E foi dele que ouvi oralmente a história de Zé
flexionar-se no singular para preencher adequadamente de Julião. Considerando-se a norma-padrão da língua,
a lacuna da frase: ao reescrever-se o trecho acima em um único período, o
segmento destacado deverá ser antecedido de vírgula e
a) A nenhuma de nossas escolhas...... (poder) deixar de substituído por
corresponder nossos valores éticos mais rigorosos.
b) Não se...... (poupar) os que governam de refletir sobre a) perante ao qual
o peso de suas mais graves decisões. b) de cujo
c) Aos governantes mais responsáveis não...... (ocorrer) c) o qual
tomar decisões sem medir suas consequências. d) frente à quem
d) A toda decisão tomada precipitadamente...... (cos- e) de quem
tumar) sobrevir consequências imprevistas e injustas.
e) Diante de uma escolha,...... (ganhar) prioridade, reco- Resposta: Letra E.
menda Gramsci, os critérios que levam em conta a dor Voltemos ao trecho: ... meu saudoso amigo Alcino Alves
humana. Costa. E foi dele que ouvi oralmente... = a única alter-
nativa que substitui corretamente o trecho destacado é
Resposta: Letra C. “de quem ouvi oralmente”.
Flexões em destaque e sublinhei os termos que esta-
belecem concordância: 9. (TRT 14.ª REGIÃO-RO e AC – Técnico Judiciário –
Em “a”: A nenhuma de nossas escolhas podem deixar FCC – 2016) “Isto pode despertar a atenção de outras pes-
de corresponder nossos valores éticos mais rigorosos. soas que tenham documentos em casa e se disponham
Em “b”: Não se poupam os que governam de refletir a trazer para a Academia, que é a guardiã desse tipo de
sobre o peso de suas mais graves decisões. acervo, que é muito difícil de ser guardado em casa, pois o
Em “c”: Aos governantes mais responsáveis não ocor- tempo destrói e aqui temos a melhor técnica de conserva-
re tomar decisões sem medir suas consequências. = ção de documentos”, disse Cavalcanti.
Isso não ocorre aos governantes – uma oração exerce O termo sublinhado faz referência a
a função de sujeito (subjetiva)
Em “d”: A toda decisão tomada precipitadamente cos- a) pessoas.
PORTUGUÊS

tumam sobrevir consequências imprevistas e injustas. b) acervo.


Em “e”: Diante de uma escolha, ganham prioridade, c) Academia.
recomenda Gramsci, os critérios que levam em conta d) tempo.
a dor humana. e) casa.

49
Resposta: Letra B. Resposta: Letra C.
Ao trecho: a guardiã desse tipo de acervo, que (o qual) Há um verbo na ativa, então teremos dois na passiva
é muito difícil de ser guardado... (auxiliar + o particípio de “privilegia”) = O Estado e
o mundo são privilegiados pelo modelo ainda domi-
10. (TRT 14.ª REGIÃO-RO e AC – Técnico Judiciário – nante.
FCC – 2016) O marechal organizou o acervo...
A forma verbal está corretamente transposta para a voz 13. (TRT 23.ª REGIÃO-MT – Técnico Judiciário – FCC –
passiva em: 2016 ) Empregam-se todas as formas verbais de acordo
com a norma culta na seguinte frase:
a) estava organizando
b) tinha organizado a) Para que se mantesse sua autenticidade, o documento
c) organizando-se não poderia receber qualquer tipo de retificação.
d) foi organizado b) Os documentos com assinatura digital disporam de
e) está organizado algoritmos de criptografia que os protegeram.
Resposta: Letra D. c) Arquivados eletronicamente, os documentos poderam
Temos: sujeito (o marechal), verbo na ativa (organizou) contar com a proteção de uma assinatura digital.
e objeto (o acervo). Como há um verbo na ativa, ao d) Quem se propor a alterar um documento criptogra-
passarmos para a passiva teremos dois (o auxiliar no fado deve saber que comprometerá sua integridade.
mesmo tempo que o verbo da ativa + o particípio do e) Não é possível fazer as alterações que convierem sem
verbo da voz ativa = organizado). O objeto exercerá comprometer a integridade dos documentos.
a função de sujeito paciente, e o sujeito da ativa será
o agente da passiva (ufa!). A frase ficará: O acervo foi Resposta: Letra E.
organizado pelo marechal. Em “a”: Para que se mantesse (mantivesse) sua auten-
ticidade, o documento não poderia receber qualquer
tipo de retificação.
11. (TRT 20.ª REGIÃO-SE – TÉCNICO JUDICIÁRIO –
Em “b”: Os documentos com assinatura digital dispo-
FCC – 2016) Precisamos de um treinador que nos ajude
ram (dispuseram) de algoritmos de criptografia que os
a comer...
protegeram.
O verbo flexionado nos mesmos tempo e modo que o
Em “c”: Arquivados eletronicamente, os documentos
sublinhado acima está também sublinhado em:
poderam (puderam) contar com a proteção de uma
assinatura digital.
a) [...] assim que conseguissem se virar sem as mães ou
Em “d”: Quem se propor (propuser) a alterar um docu-
as amas...
mento criptografado deve saber que comprometerá
b) Não é por acaso que proliferaram os coaches. sua integridade.
c) [...] país que transformou a infância numa bilionária in- Em “e”: Não é possível fazer as alterações que convie-
dústria de consumo... rem sem comprometer a integridade dos documentos
d) E, mesmo que se esforcem muito [...] = correta
e) Hoje há algo novo nesse cenário.
14. (TRT 21.ª REGIÃO-RN – Técnico Judiciário – FCC
Resposta: Letra D. – 2017) Sessenta anos de história marcam, assim, a traje-
que nos ajude = presente do Subjuntivo tória da utopia no país.
Em “a”: que conseguissem = pretérito do Subjuntivo Transpondo-se a frase acima para a voz passiva, a forma
Em “b”: que proliferaram = pretérito perfeito (e tam- verbal resultante será:
bém mais-que-perfeito) do Indicativo
Em “c”: que transformou = pretérito perfeito do Indi- a) foram marcados.
cativo b) foi marcado.
Em “d”: que se esforcem = presente do Subjuntivo c) são marcados.
Em “e”: há algo novo nesse cenário = presente do In- d) foi marcada.
dicativo e) é marcada.

12. (TRT 23.ª REGIÃO-MT – Técnico Judiciário – FCC Resposta: Letra E.


– 2016) O modelo ainda dominante nas discussões ecoló- Temos um verbo (no tempo presente) na ativa, então
gicas privilegia, em escala, o Estado e o mundo... teremos dois na passiva (auxiliar [no tempo presente]
Transpondo-se a frase acima para a voz passiva, a forma + particípio de “marcam”) = Assim, a trajetória da uto-
verbal resultante será: pia do país é marcada pelos sessenta anos de história.

a) é privilegiado. 15. (Polícia Militar do Estado de São Paulo – Soldado


b) sendo privilegiadas. PM 2.ª Classe – Vunesp – 2017) Considere as seguintes
PORTUGUÊS

c) são privilegiados. frases:


d) foi privilegiado. Primeiro, associe suas memórias com objetos físicos.
e) são privilegiadas. Segundo, não memorize apenas por repetição.
Terceiro, rabisque!

50
Um verbo flexionado no mesmo modo que o dos verbos d) Bem, o fato é que eu era o técnico de som do horário,
empregados nessas frases está em destaque em: precisava “passar” a transmissão lá para a câmara, e o
locutor não chegava para os textos de abertura, pu-
a) [...] o acesso rápido e a quantidade de textos fazem blicidade, chamadas.
com que o cérebro humano não considere útil gravar e) ... estremecíamos quando ele nos chamava para qual-
esses dados [...] quer coisa, fazendo-nos entrar na sua sala imensa, já
b) Na internet, basta um clique para vasculhar um sem- suando frio e atentos às suas finas e cortantes pala-
-número de informações. vras.
c) [...] após discar e fazer a ligação, não precisamos mais
dele... Resposta: Letra C.
d) Pense rápido: qual o número de telefone da casa em Aos itens:
que morou quando era criança? Em “a”: há = presente / acabam = presente / são =
e) É o que mostra também uma pesquisa recente condu- presente
zida pela empresa de segurança digital Kaspersky [...] Em “b”: Voltei = pretérito perfeito / acreditei = preté-
rito perfeito
Resposta: Letra D. Em “c”: deixe / largue / vá / ponha = verbos no modo
Os verbos das frases citadas estão no Modo Imperati- imperativo afirmativo (ordens)
vo (expressam ordem). Vamos aos itens: Em “d”: era = pretérito imperfeito / precisava = preté-
Em “a”: ... o acesso rápido e a quantidade de textos rito imperfeito / chegava = pretérito imperfeito
fazem = presente do Indicativo Em “e”: fazendo-nos = gerúndio / suando = gerúndio
Em “b”: Na internet, basta um clique = presente do
Indicativo 18. (PC-SP – Agente de Polícia – Vunesp – 2013) Em
Em “c”: ... após discar e fazer a ligação, não precisamos – O destino me prestava esse pequeno favor: completa-
= presente do Indicativo va minha identificação com o resto da humanidade, que
Em “d”: Pense rápido: = Imperativo tem sempre para contar uma história de objeto achado;
Em “e”: É o que mostra também uma pesquisa = pre- – o pronome em destaque retoma a seguinte palavra/
sente do Indicativo expressão:

16. (PC-SP – Atendente de Necrotério Policial – Vu- a) o resto da humanidade.


nesp – 2014) Assinale a alternativa em que a palavra em b) esse pequeno favor.
c) minha identificação.
destaque na frase pertence à classe dos adjetivos (pala-
d) O destino.
vra que qualifica um substantivo).
e) completava.
a) Existe grande confusão entre os diversos tipos de eu-
Resposta: Letra A.
tanásia...
Completava minha identificação com o resto da huma-
b)... o médico ou alguém causa ativamente a morte...
nidade, que (a qual) tem sempre para contar uma his-
c) prolonga o processo de morrer procurando distanciar
tória de objeto achado = pronome relativo que retoma
a morte. o resto da humanidade.
d) Ela é proibida por lei no Brasil,...
e) E como seria a verdadeira boa morte? 19. (PC-SP – Agente de Polícia – Vunesp – 2013) Con-
sidere o trecho a seguir.
Resposta: Letra E. É comum que objetos ____________ esquecidos em locais
Em “a”: Existe grande confusão = substantivo públicos. Mas muitos transtornos poderiam ser evitados
Em “b”: o médico ou alguém causa ativamente a mor- se as pessoas __________ a atenção voltada para seus per-
te = pronome tences, conservando-os junto ao corpo.
Em “c”: prolonga o processo de morrer procurando Assinale a alternativa que preenche, correta e respectiva-
distanciar a morte = substantivo mente, as lacunas do texto.
Em “d”: Ela é proibida por lei no Brasil = substantivo
Em “e”: E como seria a verdadeira boa morte? = ad- a) sejam ... mantesse
jetivo b) sejam ... mantém
c) sejam ... mantivessem
17. (PC-SP – Escrivão de Polícia – Vunesp – 2014) As for- d) seja ... mantivessem
mas verbais conjugadas no modo imperativo, expressan- e) seja ... mantêm
do ordem, instrução ou comando, estão destacadas em
Resposta: Letra C.
a) Mas há outros cujas marcas acabam ficando bem ní- Completemos as lacunas e depois busquemos o item
tidas na memória: são aqueles donos de qualidades correspondente. A pegadinha aqui é a conjugação do
incomuns. verbo “manter”, no presente do Subjuntivo (mantiver):
PORTUGUÊS

b) Voltei uns cinquenta minutos depois, cauteloso, e É comum que objetos sejam esquecidos em locais pú-
quase não acreditei no que ouvi. blicos. Mas muitos transtornos poderiam ser evitados se
c) – Ei rapaz, deixe ligado o microfone, largue isso aí, vá as pessoas mantivessem a atenção voltada para seus
pro estúdio e ponha a rádio no ar. pertences, conservando-os junto ao corpo.

51
20. (PC-SP – Atendente de Necrotério Policial – Vu- ARGUMENTAÇÃO
nesp – 2013) Nas frases – Não vou mais à escola!… – e
– Hoje estão na moda os métodos audiovisuais. – as pala- O ato de comunicação não visa apenas transmitir
vras em destaque expressam, correta e respectivamente, uma informação a alguém. Quem se comunica pretende
circunstâncias de criar uma imagem positiva de si mesmo por exemplo, a
de um sujeito educado, ou inteligente, ou culto; quer ser
a) dúvida e modo. aceito, deseja que o que diz seja admitido como verda-
b) dúvida e tempo. deiro. Em síntese, tem a intenção de convencer, ou seja,
c) modo e afirmação. tem o desejo de que o ouvinte creia no que o texto diz e
d) negação e lugar. faça o que ele propõe.
e) negação e tempo. Se essa é a finalidade última de todo ato de comuni-
cação, todo texto contém um componente argumentati-
Resposta: Letra E.
vo. A argumentação é o conjunto de recursos de nature-
“não” – advérbio de negação / “hoje” – advérbio de
za linguística destinados a persuadir a pessoa a quem a
tempo.
comunicação se destina. Está presente em todo tipo de
21. (PC-SP – Escrivão de Polícia – Vunesp – 2013)
Assinale a alternativa que completa respectivamente as texto e visa a promover adesão às teses e aos pontos de
lacunas, em conformidade com a norma-padrão de con- vista defendidos.
jugação verbal. As pessoas costumam pensar que o argumento seja
Há quem acredite que alcançará o sucesso profissional apenas uma prova de verdade ou uma razão indiscutível
quando __________ um diploma de mestrado, mas há para comprovar a veracidade de um fato. O argumento
aqueles que _________ de opinião e procuram investir em é mais que isso: como se disse acima, é um recurso de
cursos profissionalizantes. linguagem utilizado para levar o interlocutor a crer na-
quilo que está sendo dito, a aceitar como verdadeiro o
a) obtiver … divirgem que está sendo transmitido. A argumentação pertence
b) obter … divergem ao domínio da retórica, arte de persuadir as pessoas me-
c) obtesse … devirgem diante o uso de recursos de linguagem.
d) obter … divirgem Para compreender claramente o que é um argumen-
e) obtiver … divergem to, é bom voltar ao que diz Aristóteles, filósofo grego
do século lV a.C., numa obra intitulada “Tópicos: os ar-
Resposta: Letra E. gumentos são úteis quando se tem de escolher entre duas
Há quem acredite que alcançará o sucesso profissio- ou mais coisas”.
nal quando obtiver um diploma de mestrado, mas há Se tivermos de escolher entre uma coisa vantajo-
aqueles que divergem de opinião e procuram investir sa e uma desvantajosa, como a saúde e a doença, não
em cursos profissionalizantes. precisamos argumentar. Suponhamos, no entanto, que
tenhamos de escolher entre duas coisas igualmente
22. (PC-SP – Auxiliar de Necropsia – Vunesp – 2014) vantajosas, a riqueza e a saúde. Nesse caso, precisamos
Considerando que o adjetivo é uma palavra que modifica argumentar sobre qual das duas é mais desejável. O ar-
o substantivo, com ele concordando em gênero e núme- gumento pode então ser definido como qualquer recur-
ro, assinale a alternativa em que a palavra destacada é
so que torna uma coisa mais desejável que outra. Isso
um adjetivo.
significa que ele atua no domínio do preferível. Ele é uti-
lizado para fazer o interlocutor crer que, entre duas te-
a) ... um câncer de boca horroroso, ...
ses, uma é mais provável que a outra, mais possível que
b) Ele tem dezesseis anos...
c) Eu queria que ele morresse logo, ... a outra, mais desejável que a outra, é preferível à outra.
d) ... com a crueldade adicional de dar esperança às fa- O objetivo da argumentação não é demonstrar a
mílias. verdade de um fato, mas levar o ouvinte a admitir como
e) E o inferno não atinge só os terminais. verdadeiro o que o enunciador está propondo.
Há uma diferença entre o raciocínio lógico e a argu-
Resposta: Letra A. mentação. O primeiro opera no domínio do necessário,
Em “a”: um câncer de boca horroroso = adjetivo ou seja, pretende demonstrar que uma conclusão deriva
Em “b”: Ele tem dezesseis anos = numeral necessariamente das premissas propostas, que se deduz
Em “c”: Eu queria que ele morresse logo = advérbio obrigatoriamente dos postulados admitidos. No raciocí-
Em “d”: com a crueldade adicional de dar esperança às nio lógico, as conclusões não dependem de crenças, de
famílias = substantivo uma maneira de ver o mundo, mas apenas do encadea-
Em “e”: E o inferno não atinge só os terminais = subs- mento de premissas e conclusões.
tantivo Por exemplo, um raciocínio lógico é o seguinte enca-
deamento:
PORTUGUÊS

A é igual a B.
A é igual a C.
Então: C é igual a A.

52
Admitidos os dois postulados, a conclusão é, obrigato- Quem disse a frase não fui eu, foi Einstein. Para ele,
riamente, que C é igual a A. uma coisa vem antes da outra: sem imaginação, não há
Outro exemplo: conhecimento. Nunca o inverso.
Todo ruminante é um mamífero. Alex José Periscinoto.
A vaca é um ruminante. In: Folha de S. Paulo. 30 ago.1993, p. 5-2.
Logo, a vaca é um mamífero.
A tese defendida nesse texto é que a imaginação é
Admitidas como verdadeiras as duas premissas, a con- mais importante do que o conhecimento. Para levar o
clusão também será verdadeira. auditório a aderir a ela, o enunciador cita um dos mais
No domínio da argumentação, as coisas são diferen- célebres cientistas do mundo. Se um físico de renome
tes. Nele, a conclusão não é necessária, não é obrigató- mundial disse isso, então as pessoas devem acreditar que
ria. Por isso, devese mostrar que ela é a mais desejável, a é verdade.
mais provável, a mais plausível. Se o Banco do Brasil fizer
uma propaganda dizendose mais confiável do que os con-
2. Argumento de Quantidade
correntes porque existe desde a chegada da família real
portuguesa ao Brasil, ele estará dizendonos que um banco
É aquele que valoriza mais o que é apreciado pelo
com quase dois séculos de existência é sólido e, por isso,
confiável. Embora não haja relação necessária entre a so- maior número de pessoas, o que existe em maior núme-
lidez de uma instituição bancária e sua antiguidade, esta ro, o que tem maior duração, o que tem maior número
tem peso argumentativo na afirmação da confiabilidade de de adeptos etc. O fundamento desse tipo de argumento
um banco. Portanto é provável que se creia que um banco é que mais melhor. A publicidade faz largo uso do argu-
mais antigo seja mais confiável do que outro fundado há mento de quantidade.
dois ou três anos.
Enumerar todos os tipos de argumentos é uma tarefa 3. Argumento do Consenso
quase impossível, tantas são as formas de que nos vale-
mos para fazer as pessoas preferirem uma coisa a outra. É uma variante do argumento de quantidade. Fun-
Por isso, é importante entender bem como eles funcionam. damentase em afirmações que, numa determinada épo-
Já vimos diversas características dos argumentos. É ca, são aceitas como verdadeiras e, portanto, dispensam
preciso acrescentar mais uma: o convencimento do inter- comprovações, a menos que o objetivo do texto seja
locutor, o auditório, que pode ser individual ou coletivo, comprovar alguma delas. Parte da ideia de que o consen-
será tanto mais fácil quanto mais os argumentos estiverem so, mesmo que equivocado, corresponde ao indiscutível,
de acordo com suas crenças, suas expectativas, seus valo- ao verdadeiro e, portanto, é melhor do que aquilo que
res. Não se pode convencer um auditório pertencente a não desfruta dele. Em nossa época, são consensuais, por
uma dada cultura enfatizando coisas que ele abomina. Será exemplo, as afirmações de que o meio ambiente precisa
mais fácil convencêlo valorizando coisas que ele conside- ser protegido e de que as condições de vida são piores
ra positivas. No Brasil, a publicidade da cerveja vem com nos países subdesenvolvidos. Ao confiar no consenso,
frequência associada ao futebol, ao gol, à paixão nacional. porém, correse o risco de passar dos argumentos válidos
Nos Estados Unidos, essa associação certamente não sur- para os lugarescomuns, os preconceitos e as frases ca-
tiria efeito, porque lá o futebol não é valorizado da mesma rentes de qualquer base científica.
forma que no Brasil. O poder persuasivo de um argumento
está vinculado ao que é valorizado ou desvalorizado numa
4. Argumento de Existência
dada cultura.
É aquele que se fundamenta no fato de que é mais
fácil aceitar aquilo que comprovadamente existe do que
Tipos de Argumento
aquilo que é apenas provável, que é apenas possível. A
Já verificamos que qualquer recurso linguístico des- sabedoria popular enuncia o argumento de existência no
tinado a fazer o interlocutor dar preferência à tese do provérbio “Mais vale um pássaro na mão do que dois
enunciador é um argumento. Exemplo: voando”.
1. Argumento de Autoridade Nesse tipo de argumento, incluemse as provas docu-
mentais (fotos, estatísticas, depoimentos, gravações etc.)
É a citação, no texto, de afirmações de pessoas reco- ou provas concretas, que tornam mais aceitável uma afir-
nhecidas pelo auditório como autoridades em certo do- mação genérica. Durante a invasão do Iraque, por exem-
mínio do saber, para servir de apoio aquilo que o enun- plo, os jornais diziam que o exército americano era muito
ciador está propondo. Esse recurso produz dois efeitos mais poderoso do que o iraquiano. Essa afirmação, sem
distintos: revela o conhecimento do produtor do texto a ser acompanhada de provas concretas, poderia ser vista
respeito do assunto de que está tratando; dá ao texto a como propagandística. No entanto, quando documenta-
da pela comparação do número de canhões, de carros de
garantia do autor citado. É preciso, no entanto, não fazer
combate, de navios etc., ganhava credibilidade.
PORTUGUÊS

do texto um amontoado de citações. A citação precisa


ser pertinente e verdadeira. Exemplo:
“A imaginação é mais importante do que o conheci-
mento.”

53
5. Argumento quase lógico falar de um homem público, pode ter a intenção de cri-
ticálo, de ridicularizálo ou, ao contrário, de mostrar sua
É aquele que opera com base nas relações lógicas, grandeza.
como causa e efeito, analogia, implicação, identidade etc. O enunciador cria a orientação argumentativa de seu
Esses raciocínios são chamados quase lógicos porque, di- texto dando destaque a uns fatos e não a outros, omitin-
versamente dos raciocínios lógicos, eles não pretendem do certos episódios e revelando outros, escolhendo de-
estabelecer relações necessárias entre os elementos, mas terminadas palavras e não outra. Veja:
sim instituir relações prováveis, possíveis, plausíveis. Por “O clima da festa era tão pacífico que até sogras e
exemplo, quando se diz “A é igual a B”, “B é igual a C”, noras trocavam abraços afetuosos.”
“então A é igual a C”, estabelecese uma relação de iden- O enunciador aí pretende ressaltar a ideia geral de
tidade lógica. Entretanto, quando se afirma “Amigo de que noras e sogras não se toleram. Não fosse assim, não
amigo meu é meu amigo” não se institui uma identidade teria escolhido esse fato para ilustrar o clima da festa nem
lógica, mas uma identidade provável. teria utilizado o termo até que serve para incluir no argu-
Um texto coerente do ponto de vista lógico é mais mento alguma coisa inesperada.
facilmente aceito do que um texto incoerente. Vários são Além dos defeitos de argumentação mencionados
os defeitos que concorrem para desqualificar o texto do quando tratamos de alguns tipos de argumentação, va-
ponto de vista lógico: fugir do tema proposto, cair em mos citar outros:
contradição, tirar conclusões que não se fundamentam ▪ Uso sem delimitação adequada de palavra de sen-
nos dados apresentados, ilustrar afirmações gerais com tido tão amplo, que serve de argumento para um pon-
fatos inadequados, narrar um fato e dele extrair genera- to de vista e seu contrário. São noções confusas, como
lizações indevidas. paz, que, paradoxalmente, pode ser usada pelo agressor
e pelo agredido. Essas palavras podem ter valor positivo
6. Argumento do Atributo (paz, justiça, honestidade, democracia) ou vir carregadas
de valor negativo (autoritarismo, degradação do meio
ambiente, injustiça, corrupção).
É aquele que considera melhor o que tem proprie-
▪ Uso de afirmações tão amplas, que podem ser der-
dades típicas daquilo que é mais valorizado socialmente,
rubadas por um único contraexemplo. Quando se diz “To-
por exemplo, o mais raro é melhor que o comum, o que
dos os políticos são ladrões”, basta um único exemplo de
é mais refinado é melhor que o que é mais grosseiro etc.
político honesto para destruir o argumento.
Por esse motivo, a publicidade usa, com muita fre-
▪ Emprego de noções científicas sem nenhum rigor,
quência, celebridades recomendando prédios residen-
fora do contexto adequado, sem o significado apropria-
ciais, produtos de beleza, alimentos estéticos etc., com
do, vulgarizandoas e atribuindolhes uma significação
base no fato de que o consumidor tende a associar o subjetiva e grosseira. É o caso, por exemplo, da frase “O
produto anunciado com atributos da celebridade. imperialismo de certas indústrias não permite que outras
Uma variante do argumento de atributo é o argu- cresçam”, em que o termo imperialismo é descabido, uma
mento da competência linguística. A utilização da varian- vez que, a rigor, significa “ação de um Estado visando a
te culta e formal da língua que o produtor do texto co- reduzir outros à sua dependência política e econômica”.
nhece a norma linguística socialmente mais valorizada e,
por conseguinte, deve produzir um texto em que se pode A boa argumentação é aquela que está de acordo
confiar. Nesse sentido é que se diz que o modo de dizer com a situação concreta do texto, que leva em conta os
dá confiabilidade ao que se diz. componentes envolvidos na discussão (o tipo de pessoa
Imaginese que um médico deva falar sobre o estado a quem se dirige a comunicação, o assunto, por exemplo).
de saúde de uma personalidade pública. Ele poderia fa- Convém ainda alertar que não se convence ninguém
zêlo das duas maneiras indicadas abaixo, mas a primeira com manifestações de sinceridade do autor (como eu,
seria infinitamente mais adequada para a persuasão do que não costumo mentir...) ou com declarações de certe-
que a segunda, pois esta produziria certa estranheza e za expressas em fórmulas feitas (como estou certo, creio
não criaria uma imagem de competência do médico: firmemente, é claro, é óbvio, é evidente, afirmo com toda
▪ Para aumentar a confiabilidade do diagnóstico e a certeza etc). Em vez de prometer, em seu texto, since-
levando em conta o caráter invasivo de alguns exames, a ridade e certeza, autenticidade e verdade, o enunciador
equipe médica houve por bem determinar o internamen- deve construir um texto que revele isso. Em outros ter-
to do governador pelo período de três dias, a partir de mos, essas qualidades não se prometem, manifestam-se
hoje, 4 de fevereiro de 2001. na ação.
▪ Para conseguir fazer exames com mais cuidado e A argumentação é a exploração de recursos para fa-
porque alguns deles são barras-pesadas, a gente botou o zer parecer verdadeiro aquilo que se diz num texto e,
governador no hospital por três dias. com isso, levar a pessoa a que texto é endereçado a crer
Como dissemos antes, todo texto tem uma função ar- naquilo que ele diz.
gumentativa, porque ninguém fala para não ser levado Um texto dissertativo tem um assunto ou tema e ex-
a sério, para ser ridicularizado, para ser desmentido: em pressa um ponto de vista, acompanhado de certa funda-
todo ato de comunicação deseja-se influenciar alguém. mentação, que inclui a argumentação, questionamento,
PORTUGUÊS

Por mais neutro que pretenda ser, um texto tem sempre com o objetivo de persuadir. Argumentar é o processo
uma orientação argumentativa. pelo qual se estabelecem relações para chegar à conclu-
A orientação argumentativa é certa direção que o fa- são, com base em premissas. Persuadir é um processo de
lante traça para seu texto. Por exemplo, um jornalista, ao convencimento, por meio da argumentação, no qual se

54
procura convencer os outros, de modo a influenciar seu se pelas proposições mais simples até alcançar, por meio
pensamento e seu comportamento. de deduções, a conclusão final. Para a linha de raciocínio
A persuasão pode ser válida e não válida. Na persua- cartesiana, é fundamental determinar o problema, dividilo
são válida, expõemse com clareza os fundamentos de em partes, ordenar os conceitos, simplificandoos, enume-
uma ideia ou proposição, e o interlocutor pode questio- rar todos os seus elementos e determinar o lugar de cada
nar cada passo do raciocínio empregado na argumen- um no conjunto da dedução.
tação. A persuasão não válida apoiase em argumentos A lógica cartesiana, até os nossos dias, é fundamental
subjetivos, apelos subliminares, chantagens sentimen- para a argumentação dos trabalhos acadêmicos. Descar-
tais, com o emprego de “apelações”, como a inflexão de tes propôs quatro regras básicas que constituem um con-
voz, a mímica e até o choro. junto de reflexos vitais, uma série de movimentos sucessi-
Alguns autores classificam a dissertação em duas mo- vos e contínuos do espírito em busca da verdade:
dalidades, expositiva e argumentativa. Esta exige argu- I – evidência;
mentação, razões a favor e contra uma ideia, ao passo II – divisão ou análise;
que a outra é informativa, apresenta dados sem a inten- III – ordem ou dedução;
ção de convencer. Na verdade, a escolha dos dados le- IV – enumeração.
vantados, a maneira de expôlos no texto já revelam uma
“tomada de posição”, a adoção de um ponto de vista na A enumeração pode apresentar dois tipos de falhas: a
dissertação, ainda que sem a apresentação explícita de omissão e a incompreensão. Qualquer erro na enumera-
argumentos. Desse ponto de vista, a dissertação pode ção pode quebrar o encadeamento das ideias, indispen-
ser definida como discussão, debate, questionamento, o sável para o processo dedutivo.
que implica a liberdade de pensamento, a possibilida- A forma de argumentação mais empregada na re-
de de discordar ou concordar parcialmente. A liberdade dação acadêmica é o silogismo, raciocínio baseado nas
de questionar é fundamental, mas não é suficiente para regras cartesianas, que contém três proposições: duas
organizar um texto dissertativo. É necessária também a premissas, maior e menor, e a conclusão. As três propo-
sições são encadeadas de tal forma, que a conclusão é
exposição dos fundamentos, os motivos, os porquês da
deduzida da maior por intermédio da menor. A premissa
defesa de um ponto de vista.
maior deve ser universal, emprega todo, nenhum, pois al-
Podese dizer que o homem vive em permanente ati-
guns não caracteriza a universalidade.
tude argumentativa. A argumentação está presente em
Há dois métodos fundamentais de raciocínio: a de-
qualquer tipo de discurso, porém, é no texto dissertativo
dução (silogística), que parte do geral para o particular, e
que ela melhor se evidencia.
a indução, que vai do particular para o geral. A expressão
Para discutir um tema, para confrontar argumentos
formal do método dedutivo é o silogismo. A dedução é
e posições, é necessária a capacidade de conhecer ou- o caminho das consequências, baseiase em uma cone-
tros pontos de vista e seus respectivos argumentos. Uma xão descendente (do geral para o particular) que leva à
discussão impõe, muitas vezes, a análise de argumentos conclusão. Segundo esse método, partindose de teorias
opostos, antagônicos. Como sempre, essa capacidade gerais, de verdades universais, podese chegar à previsão
aprendese com a prática. Um bom exercício para apren- ou determinação de fenômenos particulares. O percurso
der a argumentar e contraargumentar consiste em de- do raciocínio vai da causa para o efeito. Exemplo:
senvolver as seguintes habilidades: Todo homem é mortal (premissa maior = geral, universal)
I – argumentação: anotar todos os argumentos a fa- Fulano é homem (premissa menor = particular)
vor de uma ideia ou fato; imaginar um interlocutor que Logo, Fulano é mortal (conclusão)
adote a posição totalmente contrária; A indução percorre o caminho inverso ao da dedu-
II – contraargumentação: imaginar um diálogodebate ção, baseiase em uma conexão ascendente, do particular
e quais os argumentos que essa pessoa imaginária possi- para o geral. Nesse caso, as constatações particulares le-
velmente apresentaria contra a argumentação proposta; vam às leis gerais, ou seja, parte de fatos particulares co-
III – refutação: argumentos e razões contra a argu- nhecidos para os fatos gerais, desconhecidos. O percurso
mentação oposta. do raciocínio se faz do efeito para a causa. Exemplo:
O calor dilata o ferro (particular);
A argumentação tem a finalidade de persuadir, por- O calor dilata o bronze (particular);
tanto, argumentar consiste em estabelecer relações para O calor dilata o cobre (particular);
tirar conclusões válidas, como se procede no método O ferro, o bronze, o cobre são metais;
dialético. O método dialético não envolve apenas ques- Logo, o calor dilata metais (geral, universal).
tões ideológicas, geradoras de polêmicas. Tratase de um Quanto a seus aspectos formais, o silogismo pode
método de investigação da realidade pelo estudo de sua ser válido e verdadeiro; a conclusão será verdadeira se as
ação recíproca, da contradição inerente ao fenômeno em duas premissas também o forem. Se há erro ou equívo-
questão e da mudança dialética que ocorre na natureza co na apreciação dos fatos, pode-se partir de premissas
e na sociedade. verdadeiras para chegar a uma conclusão falsa. Tem-se,
Descartes (15961650), filósofo e pensador francês, desse modo, o sofisma. Uma definição inexata, uma di-
criou o método de raciocínio silogístico, baseado na de- visão incompleta, a ignorância da causa, a falsa analogia
PORTUGUÊS

dução, que parte do simples para o complexo. Para ele, são algumas causas do sofisma. O sofisma pressupõe
verdade e evidência são a mesma coisa, e pelo raciocínio má fé, intenção deliberada de enganar ou levar ao erro;
tornase possível chegar a conclusões verdadeiras, desde quando o sofisma não tem essas intenções propositais,
que o assunto seja pesquisado em partes, começando- costuma-se chamar esse processo de argumentação de

55
paralogismo. Encontra-se um exemplo simples de sofis- ser assim relacionadas:
ma no seguinte diálogo: - Análise: penetrar, decompor, separar, dividir.
Você concorda que possui uma coisa que não per- - Síntese: integrar, recompor, juntar, reunir.
deu?
– Lógico, concordo. A análise tem importância vital no processo de coleta
Você perdeu um brilhante de 40 quilates? de ideias a respeito do tema proposto, de seu desdobra-
– Claro que não! mento e da criação de abordagens possíveis. A síntese
- Então você possui um brilhante de 40 quilates... também é importante na escolha dos elementos que fa-
rão parte do texto.
Exemplos de sofismas: Segundo Garcia (1973, p.300), a análise pode ser
I – Dedução: formal ou informal. A análise formal pode ser científica
Todo professor tem um diploma (geral, universal); ou experimental; é característica das ciências matemáti-
Fulano tem um diploma (particular); cas, físico-naturais e experimentais. A análise informal é
Logo, fulano é professor (geral – conclusão falsa); racional ou total, consiste em “discernir” por vários atos
II – Indução: distintos da atenção os elementos constitutivos de um
O Rio de Janeiro tem uma estátua do Cristo Reden- todo, os diferentes caracteres de um objeto ou fenôme-
tor. (particular); no.
Taubaté (SP) tem uma estátua do Cristo Redentor. A análise decompõe o todo em partes, a classificação
(particular); estabelece as necessárias relações de dependência e hie-
Rio de Janeiro e Taubaté são cidades. rarquia entre as partes. Análise e classificação ligam-se
Logo, toda cidade tem uma estátua do Cristo Reden- intimamente, a ponto de se confundir uma com a outra,
tor. (geral – conclusão falsa). contudo são procedimentos diversos: análise é decom-
posição e classificação é hierarquisação.
Notase que as premissas são verdadeiras, mas a con- Nas ciências naturais, classificam-se os seres, fatos
clusão pode ser falsa. Nem todas as pessoas que têm e fenômenos por suas diferenças e semelhanças; fora
diploma são professores; nem todas as cidades têm uma das ciências naturais, a classificação pode-se efetuar por
estátua do Cristo Redentor. Cometese erro quando se faz meio de um processo mais ou menos arbitrário, em que
generalizações apressadas ou infundadas. A “simples ins- os caracteres comuns e diferenciadores são empregados
peção” é a ausência de análise ou análise superficial dos de modo mais ou menos convencional. A classificação,
fatos, que leva a pronunciamentos subjetivos, baseados no reino animal, em ramos, classes, ordens, subordens,
nos sentimentos não ditados pela razão. gêneros e espécies, é um exemplo de classificação na-
Existem, ainda, outros métodos, subsidiários ou não tural, pelas características comuns e diferenciadoras. A
fundamentais, que contribuem para a descoberta ou classificação dos variados itens integrantes de uma lista
comprovação da verdade: análise, síntese, classificação mais ou menos caótica é artificial.
e definição. Além desses, existem outros métodos parti- Exemplo: aquecedor, automóvel, barbeador, batata,
culares de algumas ciências, que adaptam os processos caminhão, canário, jipe, leite, ônibus, pão, pardal, pintas-
de dedução e indução à natureza de uma realidade par- silgo, queijo, relógio, sabiá, torradeira.
ticular. Podese afirmar que cada ciência tem seu método I – Aves, Canário, Pardal, Pintassilgo, Sabiá;
próprio demonstrativo, comparativo, histórico etc. A aná- II – Alimentos, Batata, Leite, Pão, Queijo;
lise, a síntese, a classificação e a definição são chamadas III – Mecanismos, Aquecedor, Barbeador, Relógio,
métodos sistemáticos, porque pela organização e orde- Torradeira;
nação das ideias visam sistematizar a pesquisa. IV – Veículos, Automóvel, Caminhão, Jipe, Ônibus.
Análise e síntese são dois processos opostos, mas
interligados; a análise parte do todo para as partes, a sín- Os elementos desta lista foram classificados por
tese, das partes para o todo. A análise precede a síntese, ordem alfabética e pelas afinidades comuns entre eles.
porém, de certo modo, uma depende da outra. A análi- Estabelecer critérios de classificação das ideias e argu-
se decompõe o todo em partes, enquanto a síntese re- mentos, pela ordem de importância, é uma habilidade in-
compõe o todo pela reunião das partes. Sabese, porém, dispensável para elaborar o desenvolvimento de uma re-
que o todo não é uma simples justaposição das partes. dação. Tanto faz que a ordem seja crescente, do fato mais
Se alguém reunisse todas as peças de um relógio, não importante para o menos importante, ou decrescente,
significa que reconstruiu o relógio, pois fez apenas um primeiro o menos importante e, no final, o impacto do
amontoado de partes. Só reconstruiria todo se as par- mais importante; é indispensável que haja uma lógica na
tes estivessem organizadas, devidamente combinadas, classificação. A elaboração do plano compreende a clas-
seguida uma ordem de relações necessárias, funcionais, sificação das partes e subdivisões, ou seja, os elementos
então, o relógio estaria reconstruído. do plano devem obedecer a uma hierarquização. (Garcia,
Síntese, portanto, é o processo de reconstrução do 1973, p. 302304.)
todo por meio da integração das partes, reunidas e re- Para a clareza da dissertação, é indispensável que,
lacionadas num conjunto. Toda síntese, por ser uma re- logo na introdução, os termos e conceitos sejam defi-
PORTUGUÊS

construção, pressupõe a análise, que é a decomposição. nidos, pois, para expressar um questionamento, devese,
A análise, no entanto, exige uma decomposição organi- de antemão, expor clara e racionalmente as posições as-
zada, é preciso saber como dividir o todo em partes. As sumidas e os argumentos que as justificam. É muito im-
operações que se realizam na análise e na síntese podem portante deixar claro o campo da discussão e a posição

56
adotada, isto é, esclarecer não só o assunto, mas também VI – deve ter uma estrutura gramatical rígida: sujeito
os pontos de vista sobre ele. (o termo) + cópula (verbo de ligação ser) + predicativo
A definição tem por objetivo a exatidão no emprego (o gênero) + adjuntos (as diferenças).
da linguagem e consiste na enumeração das qualidades
próprias de uma ideia, palavra ou objeto. Definir é clas- As definições dos dicionários de língua são feitas por
sificar o elemento conforme a espécie a que pertence, meio de paráfrases definitórias, ou seja, uma operação
demonstra: a característica que o diferencia dos outros metalinguística que consiste em estabelecer uma relação
elementos dessa mesma espécie. de equivalência entre a palavra e seus significados.
Entre os vários processos de exposição de ideias, a A força do texto dissertativo está em sua fundamen-
definição é um dos mais importantes, sobretudo no âm- tação. Sempre é fundamental procurar um porquê, uma
bito das ciências. A definição científica ou didática é de- razão verdadeira e necessária. A verdade de um ponto
notativa, ou seja, atribui às palavras seu sentido usual ou de vista deve ser demonstrada com argumentos válidos.
consensual, enquanto a conotativa ou metafórica empre- O ponto de vista mais lógico e racional do mundo não
ga palavras de sentido figurado. Segundo a lógica tradi- tem valor, se não estiver acompanhado de uma funda-
cional aristotélica, a definição consta de três elementos: mentação coerente e adequada.
I – o termo a ser definido; Os métodos fundamentais de raciocínio segundo a ló-
II – o gênero ou espécie; gica clássica, que foram abordados anteriormente, auxiliam
III – a diferença específica. o julgamento da validade dos fatos. Às vezes, a argumenta-
ção é clara e pode reconhecerse facilmente seus elementos
O que distingue o termo definido de outros elemen- e suas relações; outras vezes, as premissas e as conclusões
tos da mesma espécie. Veja a classificação dos termos da organizamse de modo livre, misturandose na estrutura
frase a seguir: do argumento. Por isso, é preciso aprender a reconhecer
os elementos que constituem um argumento: premissas/
conclusões. Depois de reconhecer, verificar se tais ele-
mentos são verdadeiros ou falsos; em seguida, avaliar se
o argumento está expresso corretamente; se há coerência
e adequação entre seus elementos, ou se há contradição.
Para isso é que se aprendem os processos de raciocínio
por dedução e por indução. Admitindose que raciocinar é
relacionar, concluise que o argumento é um tipo específico
de relação entre as premissas e a conclusão.

a) Procedimentos Argumentativos
É muito comum formular definições de maneira de- Constituem os procedimentos argumentativos mais
feituosa, por exemplo: Análise é quando a gente decom- empregados para comprovar uma afirmação:
põe o todo em partes. Esse tipo de definição é grama- exemplificação, explicitação, enumeração, compa-
ticalmente incorreto; quando é advérbio de tempo, não ração.
representa o gênero, a espécie, a gente é forma coloquial
não adequada à redação acadêmica. Tão importante é b) Exemplificação
saber formular uma definição, que se recorre a Garcia Procura justificar os pontos de vista por meio de
(1973, p.306), para determinar os “requisitos da definição exemplos, hierarquizar afirmações. São expressões
denotativa”. Para ser exata, a definição deve apresentar os comuns nesse tipo de procedimento: mais im-
seguintes requisitos: portante que, superior a, de maior relevância que.
I – o termo deve realmente pertencer ao gênero ou Empregamse também dados estatísticos, acom-
classe em que está incluído: “mesa é um móvel” (classe panhados de expressões: considerando os dados;
em que ‘mesa’ está realmente incluída) e não “mesa é conforme os dados apresentados. Fazse a exempli-
um instrumento ou ferramenta ou instalação”; ficação, ainda, pela apresentação de causas e con-
II – o gênero deve ser suficientemente amplo para in- sequências, usandose comumente as expressões:
cluir todos os exemplos específicos da coisa definida, e porque, porquanto, pois que, uma vez que, visto que,
suficientemente restritos para que a diferença possa por causa de, em virtude de, em vista de, por motivo
ser percebida sem dificuldade; de.
III – deve ser obrigatoriamente afirmativa: não há, em c) Explicitação
verdade, definição, quando se diz que o “triângulo não O objetivo desse recurso argumentativo é explicar ou
é um prisma”; esclarecer os pontos de vista apresentados. Podese
IV – deve ser recíproca: “O homem é um ser vivo” não alcançar esse objetivo pela definição, pelo teste-
constitui definição exata, porque a recíproca, “Todo ser munho e pela interpretação. Na explicitação por
vivo é um homem” não é verdadeira (o gato é ser vivo definição, empregamse expressões como: quer di-
e não é homem); zer, denominase, chamase, na verdade, isto é, haja
PORTUGUÊS

V – deve ser breve (contida num só período). Quando vista, ou melhor; nos testemunhos são comuns as
a definição, ou o que se pretenda como tal, é muito expressões: conforme, segundo, na opinião de, no
longa (séries de períodos ou de parágrafos), chama-se parecer de, consoante as ideias de, no entender de,
explicação, e também definição expandida; no pensamento de. A explicitação se faz também

57
pela interpretação, em que são comuns as seguin- Fatos não se discutem; discutemse opiniões. As decla-
tes expressões: parece, assim, desse ponto de vista. rações, julgamento, pronunciamentos, apreciações que
expressam opiniões pessoais (não subjetivas) devem ter
d) Enumeração sua validade comprovada, e só os fatos provam. Em re-
Fazse pela apresentação de uma sequência de ele- sumo toda afirmação ou juízo que expresse uma opinião
mentos que comprovam uma opinião, tais como pessoal só terá validade se fundamentada na evidência
a enumeração de pormenores, de fatos, em uma dos fatos, ou seja, se acompanhada de provas, validade
sequência de tempo, em que são frequentes as dos argumentos, porém, pode ser contestada por meio
expressões: primeiro, segundo, por último, antes, da contraargumentação ou refutação. São vários os pro-
depois, ainda, em seguida, então, presentemente, cessos de contraargumentação:
antigamente, depois de, antes de, atualmente, hoje, Refutação pelo absurdo: refutase uma afirmação de-
no passado, sucessivamente, respectivamente. Na monstrando o absurdo da consequência. Exemplo clás-
enumeração de fatos em uma sequência de espa- sico é a contraargumentação do cordeiro, na conhecida
ço, empregamse as seguintes expressões: cá, lá, fábula “O lobo e o cordeiro”;
acolá, ali, aí, além, adiante, perto de, ao redor de, Refutação por exclusão: consiste em propor várias hi-
no Estado tal, na capital, no interior, nas grandes póteses para eliminá-las, apresentandose, então, aquela
cidades, no sul, no leste. que se julga verdadeira;
Desqualificação do argumento: atribuise o argumen-
e) Comparação to à opinião pessoal subjetiva do enunciador, restringin-
Analogia e contraste são as duas maneiras de se es- dose a universalidade da afirmação;
tabelecer a comparação, com a finalidade de com- Ataque ao argumento pelo testemunho de autorida-
provar uma ideia ou opinião. Na analogia, são co- de: consiste em refutar um argumento empregando os
muns as expressões: da mesma forma, tal como, testemunhos de autoridade que contrariam a afirmação
tanto quanto, assim como, igualmente. Para es- apresentada;
tabelecer contraste, empregamse as expressões: Desqualificar dados concretos apresentados: consiste
mais que, menos que, melhor que, pior que. em desautorizar dados reais, demonstrando que o enun-
ciador baseouse em dados corretos, mas tirou conclusões
Entre outros tipos de argumentos empregados para falsas ou inconsequentes. Por exemplo, se na argumen-
aumentar o poder de persuasão de um texto dissertativo tação afirmouse, por meio de dados estatísticos, que “o
encontram-se: controle demográfico produz o desenvolvimento”, afir-
ma-se que a conclusão é inconsequente, pois se baseia
Argumento de autoridade: O saber notório de uma em uma relação de causaefeito difícil de ser comprovada.
autoridade reconhecida em certa área do conhecimen- Para contraargumentar, propõese uma relação inversa:
to dá apoio a uma afirmação. Dessa maneira, procura-se “o desenvolvimento é que gera o controle demográfico”.
trazer para o enunciado a credibilidade da autoridade ci- Apresentamse aqui sugestões possíveis para desen-
tada. Lembre-se que as citações literais no corpo de um volver um tema, que podem ser analisadas e adaptadas
texto constituem argumentos de autoridade. Ao fazer ao desenvolvimento de outros temas. Elegese um tema,
uma citação, o enunciador situa os enunciados nela con- e, em seguida, sugeremse os procedimentos que devem
tidos na linha de raciocínio que ele considera mais ade- ser adotados para a elaboração de um Plano de Redação.
quada para explicar ou justificar um fato ou fenômeno.
Esse tipo de argumento tem mais caráter confirmatório O homem e a máquina: necessidade e riscos da
que comprobatório. evolução tecnológica
Apoio na consensualidade: Certas afirmações dispen- Questionar o tema, transformálo em interrogação,
sam explicação ou comprovação, pois seu conteúdo é responder a interrogação (assumir um ponto de vista);
aceito como válido por consenso, pelo menos em de- dar o porquê da resposta, justificar, criando um argu-
terminado espaço sociocultural. Nesse caso, incluem-se: mento básico;
- A declaração que expressa uma verdade universal (o Imaginar um ponto de vista oposto ao argumento
homem, mortal, aspira à imortalidade); básico e construir uma contra argumentação; pensar a
- A declaração que é evidente por si mesma (caso dos forma de refutação que poderia ser feita ao argumento
postulados e axiomas); básico e tentar desqualificá-la (rever tipos de argumen-
- Quando escapam ao domínio intelectual, ou seja, é tação);
de natureza subjetiva ou sentimental (o amor tem razões Refletir sobre o contexto, ou seja, fazer uma coleta
que a própria razão desconhece); implica apreciação de de ideias que estejam direta ou indiretamente ligadas ao
ordem estética (gosto não se discute); diz respeito à fé tema (as ideias podem ser listadas livremente ou organi-
religiosa, aos dogmas (creio, ainda que parece absurdo). zadas como causa e consequência);
Comprovação pela experiência ou observação: A ver- Analisar as ideias anotadas, sua relação com o tema e
dade de um fato ou afirmação pode ser comprovada por com o argumento básico;
meio de dados concretos, estatísticos ou documentais. Fazer uma seleção das ideias pertinentes, escolhendo
PORTUGUÊS

Comprovação pela fundamentação lógica: A com- as que poderão ser aproveitadas no texto; essas ideias
provação se realiza por meio de argumentos racionais, transformam-se em argumentos auxiliares, que explicam
baseados na lógica: causa/efeito; consequência/causa; e corroboram a ideia do argumento básico;
condição/ocorrência. Fazer um esboço do Plano de Redação, organizando

58
uma sequência na apresentação das ideias selecionadas, de período. Exemplo: Estudei português, matemári-
obedecendo às partes principais da estrutura do texto, ca, constitucional, etc. (e não “etc..”)
que poderia ser mais ou menos a seguinte:  Nos títulos e cabeçalhos é opcional o emprego do
ponto, assim como após o nome do autor de uma
Introdução: citação:
I – função social da ciência e da tecnologia; Haverá eleições em outubro
II – definições de ciência e tecnologia; O culto do vernáculo faz parte do brio cívico. (Napo-
III – indivíduo e sociedade perante o avanço tecno- leão Mendes de Almeida) (ou: Almeida.)
lógico.  Os números que identificam o ano não utilizam
ponto nem devem ter espaço a separá-los, bem como os
Desenvolvimento: números de CEP: 1975, 2014, 2006, 17600-250.
I – apresentação de aspectos positivos e negativos do
desenvolvimento tecnológico; B) Ponto e Vírgula (;)
II – como o desenvolvimento científico-tecnológico
modificou as condições de vida no mundo atual;  Separa várias partes do discurso, que têm a mes-
III – a tecnocracia: oposição entre uma sociedade tec- ma importância: “Os pobres dão pelo pão o traba-
nologicamente desenvolvida e a dependência tecno- lho; os ricos dão pelo pão a fazenda; os de espíritos
lógica dos países subdesenvolvidos; generosos dão pelo pão a vida; os de nenhum espí-
IV – enumerar e discutir os fatores de desenvolvimen- rito dão pelo pão a alma...” (VIEIRA)
to social;  Separa partes de frases que já estão separadas por
V – comparar a vida de hoje com os diversos tipos de vírgulas: Alguns quiseram verão, praia e calor; ou-
vida do passado; apontar semelhanças e diferenças; tros, montanhas, frio e cobertor.
VI – analisar as condições atuais de vida nos grandes  Separa itens de uma enumeração, exposição de
centros urbanos; motivos, decreto de lei, etc.
VII – como se poderia usar a ciência e a tecnologia
Ir ao supermercado;
para humanizar mais a sociedade.
Pegar as crianças na escola;
Conclusão:
Caminhada na praia;
VIII – a tecnologia pode libertar ou escravizar: benefí-
Reunião com amigos.
cios/consequências maléficas;
IX – síntese interpretativa dos argumentos e contra-
C) Dois pontos (:)
-argumentos apresentados.
 Antes de uma citação = Vejamos como Afrânio
Naturalmente esse não é o único, nem o melhor pla- Coutinho trata este assunto:
no de redação: é um dos possíveis.  Antes de um aposto = Três coisas não me agra-
dam: chuva pela manhã, frio à tarde e calor à noite.
 Antes de uma explicação ou esclarecimento: Lá es-
RECONHECIMENTO DE EFEITOS DE SENTIDO tava a deplorável família: triste, cabisbaixa, vivendo
a rotina de sempre.
DECORRENTES DO USO DE PONTUAÇÃO, DA
 Em frases de estilo direto
EXPLORAÇÃO DE RECURSOS ORTOGRÁFICOS, Maria perguntou:
DE CAMPOS SEMÂNTICOS, E DE OUTRAS - Por que você não toma uma decisão?
NOTAÇÕES
D) Ponto de Exclamação (!)
 Usa-se para indicar entonação de surpresa, cóle-
ra, susto, súplica, etc.: Sim! Claro que eu quero me
Os sinais de pontuação são marcações gráficas que casar com você!
servem para compor a coesão e a coerência textual, além  Depois de interjeições ou vocativos
de ressaltar especificidades semânticas e pragmáticas. Ai! Que susto!
Um texto escrito adquire diferentes significados quando João! Há quanto tempo!
pontuado de formas diversificadas. O uso da pontuação
depende, em certos momentos, da intenção do autor do E) Ponto de Interrogação (?)
discurso. Assim, os sinais de pontuação estão diretamen-  Usa-se nas interrogações diretas e indiretas livres.
te relacionados ao contexto e ao interlocutor. “- Então? Que é isso? Desertaram ambos?” (Artur Aze-
vedo)
1. Principais funções dos sinais de pontuação
F) Reticências (...)
A) Ponto (.)  Indica que palavras foram suprimidas: Comprei lá-
 Indica o término do discurso ou de parte dele, en- pis, canetas, cadernos...
cerrando o período.  Indica interrupção violenta da frase: “- Não... quero
PORTUGUÊS

 Usa-se nas abreviaturas: pág. (página), Cia. (Com- dizer... é verdad... Ah!”
panhia). Se a palavra abreviada aparecer em final  Indica interrupções de hesitação ou dúvida: Este
de período, este não receberá outro ponto; neste mal... pega doutor?
caso, o ponto de abreviatura marca, também, o fim  Indica que o sentido vai além do que foi dito: Dei-

59
xa, depois, o coração falar... Temos, ainda, sinais distintivos:
 a barra ( / ) = usada em datas (25/12/2014), sepa-
G) Vírgula (,) ração de siglas (IOF/UPC);
 os colchetes ([ ]) = usados em transcrições feitas
Não se usa vírgula pelo narrador ([vide pág. 5]), usado como primeira
Separando termos que, do ponto de vista sintático, opção aos parênteses, principalmente na matemá-
ligam-se diretamente entre si: tica;
 o asterisco (*) = usado para remeter o leitor a
1. Entre sujeito e predicado: uma nota de rodapé ou no fim do livro, para subs-
Todos os alunos da sala foram advertidos. tituir um nome que não se quer mencionar.
Sujeito predicado
REFERÊNCIAS BIBLIOGRÁFICAS
2. Entre o verbo e seus objetos: Português linguagens: volume 3 / Wiliam Roberto Ce-
O trabalho custou sacrifício aos reja, Thereza Cochar Magalhães. – 7.ª ed. Reform. – São
realizadores. Paulo: Saraiva, 2010.
V.T.D.I. O.D. O.I. SACCONI, Luiz Antônio. Nossa gramática completa
Sacconi. 30.ª ed. Rev. São Paulo: Nova Geração, 2010.
Usa-se a vírgula:
SITE
1. Para marcar intercalação: http://www.infoescola.com/portugues/pontuacao/
A) do adjunto adverbial: O café, em razão da sua http://www.brasilescola.com/gramatica/uso-da-vir-
abundância, vem caindo de preço. gula.htm
B) da conjunção: Os cerrados são secos e áridos. Estão
produzindo, todavia, altas quantidades de alimen-
tos.
C) das expressões explicativas ou corretivas: As indús- EXERCÍCIOS COMENTADOS
trias não querem abrir mão de suas vantagens, isto
é, não querem abrir mão dos lucros altos. 1. (STJ – Conhecimentos Básicos para o Cargo 1 – Ces-
pe – 2018 – adaptada)
2. Para marcar inversão:
A) do adjunto adverbial (colocado no início da ora- Texto CB1A1CCC
ção): Depois das sete horas, todo o comércio está de
portas fechadas. As audiências de segunda a sexta-feira muitas vezes re-
B) dos objetos pleonásticos antepostos ao verbo: Aos velaram o lado mais sórdido da natureza humana. Eram
pesquisadores, não lhes destinaram verba alguma. relatos de sofrimento, dor, angústia que se transporta-
C) do nome de lugar anteposto às datas: Recife, 15 de vam da cadeira das vítimas, testemunhas e réus para mi-
maio de 1982. nha cadeira de juíza. A toga não me blindou daqueles re-
latos sofridos, aflitos. As angústias dos que se sentavam
3. Para separar entre si elementos coordenados à minha frente, por diversas vezes, me escoltaram até
(dispostos em enumeração): minha casa e passaram a ser companheiras de noites de
Era um garoto de 15 anos, alto, magro. insônia. Não havia outra solução a não ser escrever. Era
A ventania levou árvores, e telhados, e pontes, e ani- preciso colocar no papel e compartilhar a dor daquelas
mais. pessoas que, mesmo ao fim do processo e com a senten-
ça prolatada, não me deixavam esquecê-las.
4. Para marcar elipse (omissão) do verbo: Nós que- Foram horas, dias, meses, anos de oitivas de mães, filhas,
remos comer pizza; e vocês, churrasco. esposas, namoradas, companheiras, todas tendo em co-
mum a violência no corpo e na alma sofrida dentro de
5. Para isolar: casa. O lar, que deveria ser o lugar mais seguro para es-
A) o aposto: São Paulo, considerada a metrópole bra- sas mulheres, havia se transformado no pior dos mundos.
sileira, possui um trânsito caótico. Quando finalmente chegavam ao Judiciário e se sen-
B) o vocativo: Ora, Thiago, não diga bobagem.
tavam à minha frente, os relatos se transformavam em
desabafos de uma vida inteira. Era preciso explicar, justi-
Observações:
ficar e muitas vezes se culpar por terem sido agredidas.
Considerando-se que “etc.” é abreviatura da expres-
A culpa por ter sido vítima, a culpa por ter permitido, a
são latina et coetera, que significa “e outras coisas”, seria
culpa por não ter sido boa o suficiente, a culpa por não
dispensável o emprego da vírgula antes dele. Porém, o
ter conseguido manter a família. Sempre a culpa.
acordo ortográfico em vigor no Brasil exige que empre-
Aquelas mulheres chegavam à Justiça buscando uma for-
guemos etc. predecido de vírgula: Falamos de política,
PORTUGUÊS

futebol, lazer, etc. ça externa como se somente nós, juízes, promotores e


As perguntas que denotam surpresa podem ter com- advogados, pudéssemos não apenas cessar aquele ciclo
binados o ponto de interrogação e o de exclamação: de violência, mas também lhes dar voz para reagir àquela
Você falou isso para ela?! violência invisível.

60
Rejane Jungbluth Suxberger. Invisíveis Marias: histórias algo “amplo”.
além das quatro paredes. Brasília: Trampolim, 2018 (com d) uma generalização do termo “direitos”.
adaptações). e) objetivos do “processo de redemocratização”.
O trecho “juízes, promotores e advogados” explica o sen-
tido de “nós”. Resposta: Letra A. Recorramos ao texto (faça isso
SEMPRE durante seu concurso. O texto é a base para
( ) CERTO ( ) ERRADO encontrar as respostas para as questões!): (...) abran-
gendo as três categorias de direitos: civis, políticos e
Resposta: Certo. Ao trecho: (...) Aquelas mulheres che- sociais. Os dois-pontos introduzem a enumeração dos
gavam à Justiça buscando uma força externa como se direitos; apresenta-os.
somente nós, juízes, promotores e advogados, pudésse-
mos não apenas cessar aquele ciclo de violência (...). Os 3. (Aneel – Técnico Administrativo – cespe – 2010) Vão
termos entre vírgulas servem para exemplificar quem surgindo novos sinais do crescente otimismo da indús-
são os “nós” citados pela autora (juízes, promotores, tria com relação ao futuro próximo. Um deles refere-se
advogados). às exportações. “O comércio mundial já está voltando a
se abrir para as empresas”, diz o gerente executivo de
2. (SERES-PE – Agente de Segurança Penitenciária – pesquisas da Confederação Nacional da Indústria (CNI),
Cespe – 2017 – adaptada) Renato da Fonseca, para explicar a melhora das expec-
tativas dos industriais com relação ao mercado externo.
Texto 1A1AAA Quanto ao mercado interno, as expectativas da indústria
não se modificaram. Mas isso não é um mau sinal, pois
Após o processo de redemocratização, com o fim da di- elas já eram francamente otimistas. Há algum tempo, a
tadura militar, em meados da década de 80 do século pesquisa da CNI, realizada mensalmente a partir de 2010,
passado, era de se esperar que a democratização das registra grande otimismo da indústria com relação à de-
instituições tivesse como resultado direto a consolidação manda interna. Trata-se de um sentimento generalizado.
da cidadania — compreendida de modo amplo, abran- Em todos os setores industriais, a expressiva maioria dos
gendo as três categorias de direitos: civis, políticos e entrevistados acredita no aumento das vendas internas.
sociais. Sobressaem, porém, problemas que configuram O Estado de S.Paulo, Editorial, 30/3/2010 (com adapta-
mais desafios para a cidadania brasileira, como a violên- ções).
cia urbana — que ameaça os direitos individuais — e o
desemprego — que ameaça os direitos sociais. O nome próprio “Renato da Fonseca” está entre vírgulas
por tratar-se de um vocativo.
No Brasil, o crime aumentou significantemente a partir
de 1980, impacto do processo de modernização pelo
( ) CERTO ( ) ERRADO
qual o país passou. Isso sugere que o boom do consumo
colocou em circulação bens de alto valor e, consequente-
Resposta: Errado. Recorramos ao texto (lembre-se de
mente, aumentou as oportunidades para o crime, inclusi- fazer a mesma coisa no dia do seu concurso!): (...) diz o
ve porque a maior mobilidade de pessoas torna o espaço gerente executivo de pesquisas da Confederação Nacio-
social mais anônimo, menos supervisionado. nal da Indústria (CNI), Renato da Fonseca, para explicar
Nesse contexto, justiça criminal passa a ser cada vez mais a melhora das expectativas. O termo em destaque não
dissociada de justiça social e reconstrução da sociedade. está exercendo a função de vocativo, já que não é uti-
O objetivo em relação à criminalidade torna-se bem me- lizado para evocar, chamar o interlocutor do diálogo.
nos ambicioso: o controle. A prisão ganha mais impor- Sua função é de aposto – explicar quem é o gerente
tância na modernidade tardia, porque satisfaz uma dupla executivo da CNI.
necessidade dessa nova cultura: castigo e controle do
risco. Essa postura às vezes proporciona controle, porém 4. (Caixa Econômica Federal – Médico do Trabalho –
não segurança, pois o Estado tem o poder limitado de cespe – 2014 – adaptada) A correção gramatical do tre-
manter a ordem por meio da polícia, sendo necessário cho “Entre as bebidas alcoólicas, cervejas e vinhos são as
dividir as tarefas de controle com organizações locais e mais comuns em todo o mundo” seria prejudicada, caso
com a comunidade. se inserisse uma vírgula logo após a palavra “vinhos”.
Jacqueline Carvalho da Silva. Manutenção da ordem pú-
blica e garantia dos direitos individuais: os desafios da ( ) CERTO ( ) ERRADO
polícia em sociedades democráticas. In: Revista Brasilei-
ra de Segurança Pública. São Paulo, ano 5, 8.ª ed., fev. – Resposta: Certo. Não se deve colocar vírgula entre
mar./2011, p. 84-5 (com adaptações). sujeito e predicado, a não ser que se trate de um apos-
to (1), predicativo do sujeito (2), ou algum termo que
No primeiro parágrafo do texto 1A1AAA, os dois-pontos requeira estar separado entre pontuações. Exemplo: O
introduzem Rio de Janeiro, cidade maravilhosa (1), está em festa!
PORTUGUÊS

Os meninos, ansiosos (2), chegaram!


a) uma enumeração das “categorias de direitos”.
b) resultados da “consolidação da cidadania”.
c) um contra-argumento para a ideia de cidadania como

61
Ortografia tamorfose.
 Formas verbais pôr e querer: pôs, pus, quisera,
A ortografia é a parte da Fonologia que trata da cor- quis, quiseste.
reta grafia das palavras. É ela quem ordena qual som  Nomes derivados de verbos com radicais termi-
devem ter as letras do alfabeto. Os vocábulos de uma nados em “d”: aludir - alusão / decidir - decisão /
língua são grafados segundo acordos ortográficos. empreender - empresa / difundir – difusão.
A maneira mais simples, prática e objetiva de apren-  Diminutivos cujos radicais terminam com “s”: Luís
der ortografia é realizar muitos exercícios, ver as palavras, - Luisinho / Rosa - Rosinha / lápis – lapisinho.
familiarizando-se com elas. O conhecimento das regras  Após ditongos: coisa, pausa, pouso, causa.
é necessário, mas não basta, pois há inúmeras exceções  Verbos derivados de nomes cujo radical termina
e, em alguns casos, há necessidade de conhecimento de com “s”: anális(e) + ar - analisar / pesquis(a) + ar
etimologia (origem da palavra). – pesquisar.

1. Regras ortográficas São escritos com Z e não S


 Sufixos “ez” e “eza” das palavras derivadas de
A) O fonema S adjetivo: macio - maciez / rico – riqueza / belo –
São escritas com S e não C/Ç beleza.
 Palavras substantivadas derivadas de verbos com Sufixos “izar” (desde que o radical da palavra de ori-
radicais em nd, rg, rt, pel, corr e sent: pretender gem não termine com s): final - finalizar / concreto
- pretensão / expandir - expansão / ascender - as- – concretizar.
censão / inverter - inversão / aspergir - aspersão /  Consoante de ligação se o radical não terminar
submergir - submersão / divertir - diversão / im- com “s”: pé + inho - pezinho / café + al - cafezal
pelir - impulsivo / compelir - compulsório / repelir Exceção: lápis + inho – lapisinho.
- repulsa / recorrer - recurso / discorrer - discurso /
sentir - sensível / consentir – consensual. C) O fonema j
São escritas com G e não J
São escritos com SS e não C e Ç  Palavras de origem grega ou árabe: tigela, girafa,
 Nomes derivados dos verbos cujos radicais ter- gesso.
minem em gred, ced, prim ou com verbos ter-  Estrangeirismo, cuja letra G é originária: sargento,
minados por tir ou - meter: agredir - agressivo / gim.
imprimir - impressão / admitir - admissão / ceder  Terminações: agem, igem, ugem, ege, oge (com
- cessão / exceder - excesso / percutir - percussão / poucas exceções): imagem, vertigem, penugem,
regredir - regressão / oprimir - opressão / compro- bege, foge.
meter - compromisso / submeter – submissão. Exceção: pajem.
 Quando o prefixo termina com vogal que se junta
com a palavra iniciada por “s”. Exemplos: a + simé-
 Terminações: ágio, égio, ígio, ógio, ugio: sortilégio,
trico - assimétrico / re + surgir – ressurgir.
litígio, relógio, refúgio.
 No pretérito imperfeito simples do subjuntivo.
 Verbos terminados em ger/gir: emergir, eleger, fu-
Exemplos: ficasse, falasse.
gir, mugir.
São escritos com C ou Ç e não S e SS
 Depois da letra “r” com poucas exceções: emergir,
 Vocábulos de origem árabe: cetim, açucena, açú-
surgir.
car.
 Depois da letra “a”, desde que não seja radical ter-
 Vocábulos de origem tupi, africana ou exótica:
minado com j: ágil, agente.
cipó, Juçara, caçula, cachaça, cacique.
 Sufixos aça, aço, ação, çar, ecer, iça, nça, uça,
uçu, uço: barcaça, ricaço, aguçar, empalidecer, car- São escritas com J e não G
niça, caniço, esperança, carapuça, dentuço.  Palavras de origem latinas: jeito, majestade, hoje.
 Nomes derivados do verbo ter: abster - abstenção  Palavras de origem árabe, africana ou exótica:
/ deter - detenção / ater - atenção / reter – retenção. jiboia, manjerona.
 Após ditongos: foice, coice, traição.  Palavras terminadas com aje: ultraje.
 Palavras derivadas de outras terminadas em -te,
to(r): marte - marciano / infrator - infração / ab- D) O fonema ch
sorto – absorção. São escritas com X e não CH
 Palavras de origem tupi, africana ou exótica: aba-
B) O fonema z caxi, xucro.
São escritos com S e não Z  Palavras de origem inglesa e espanhola: xampu,
 Sufixos: ês, esa, esia, e isa, quando o radical é lagartixa.
PORTUGUÊS

substantivo, ou em gentílicos e títulos nobiliárqui-  Depois de ditongo: frouxo, feixe.


cos: freguês, freguesa, freguesia, poetisa, baronesa,  Depois de “en”: enxurrada, enxada, enxoval.
princesa. Exceção: quando a palavra de origem não derive de
 Sufixos gregos: ase, ese, ise e ose: catequese, me- outra iniciada com ch - Cheio - (enchente)

62
ALGUNS USOS ORTOGRÁFICOS ESPECIAIS
São escritas com CH e não X
 Palavras de origem estrangeira: chave, chumbo, 1. Por que / por quê / porquê / porque
chassi, mochila, espadachim, chope, sanduíche, sal-
sicha. POR QUE (separado e sem acento)

E) As letras “e” e “i” É usado em:


 Ditongos nasais são escritos com “e”: mãe, põem. 1. interrogações diretas (longe do ponto de interroga-
Com “i”, só o ditongo interno cãibra. ção) = Por que você não veio ontem?
 Verbos que apresentam infinitivo em -oar, -uar 2. interrogações indiretas, nas quais o “que” equivale
são escritos com “e”: caçoe, perdoe, tumultue. Es- a “qual razão” ou “qual motivo” = Perguntei-lhe por
crevemos com “i”, os verbos com infinitivo em que faltara à aula ontem.
-air, -oer e -uir: trai, dói, possui, contribui. 3. equivalências a “pelo(a) qual” / “pelos(as) quais” =
Ignoro o motivo por que ele se demitiu.

FIQUE ATENTO! POR QUÊ (separado e com acento)


Há palavras que mudam de sentido quan-
do substituímos a grafia “e” pela grafia “i”: Usos:
área (superfície), ária (melodia) / delatar 1. como pronome interrogativo, quando colocado no
(denunciar), dilatar (expandir) / emergir fim da frase (perto do ponto de interrogação) =
(vir à tona), imergir (mergulhar) / peão (de Você faltou. Por quê?
estância, que anda a pé), pião (brinquedo). 2. quando isolado, em uma frase interrogativa = Por
quê?

PORQUE (uma só palavra, sem acento gráfico)


#FicaDica
Usos:
Se o dicionário ainda deixar dúvida quanto 1. como conjunção coordenativa explicativa (equivale
à ortografia de uma palavra, há a possibili- a “pois”, “porquanto”), precedida de pausa na escri-
dade de consultar o Vocabulário Ortográfi- ta (pode ser vírgula, ponto-e-vírgula e até ponto
co da Língua Portuguesa (VOLP), elaborado final) = Compre agora, porque há poucas peças.
pela Academia Brasileira de Letras. É uma 2. como conjunção subordinativa causal, substituível
obra de referência até mesmo para a criação por “pela causa”, “razão de que” = Você perdeu por-
de dicionários, pois traz a grafia atualizada que se antecipou.
das palavras (sem o significado). Na Internet,
o endereço é www.academia.org.br. PORQUÊ (uma só palavra, com acento gráfico)

Usos:
2. Informações importantes 1. como substantivo, com o sentido de “causa”, “ra-
zão” ou “motivo”, admitindo pluralização (porquês). Ge-
Formas variantes são as que admitem grafias ou pro- ralmente é precedido por artigo = Não sei o porquê da
núncias diferentes para palavras com a mesma significa- discussão. É uma pessoa cheia de porquês.
ção: aluguel/aluguer, assobiar/assoviar, catorze/quatorze,
dependurar/pendurar, flecha/frecha, germe/gérmen, in- 2. ONDE / AONDE
farto/enfarte, louro/loiro, percentagem/porcentagem, re-
lampejar/relampear/relampar/relampadar. Onde = empregado com verbos que não expressam a
Os símbolos das unidades de medida são escritos ideia de movimento = Onde você está?
sem ponto, com letra minúscula e sem “s” para indicar
plural, sem espaço entre o algarismo e o símbolo: 2kg, Aonde = equivale a “para onde”. É usado com verbos
20km, 120km/h. que expressam movimento = Aonde você vai?
Exceção para litro (L): 2 L, 150 L.
3. MAU / MAL
Na indicação de horas, minutos e segundos, não
deve haver espaço entre o algarismo e o símbolo: 14h, Mau = é um adjetivo, antônimo de “bom”. Usa-se
22h30min, 14h23’34’’(= quatorze horas, vinte e três mi- como qualificação = O mau tempo passou. / Ele é um
nutos e trinta e quatro segundos). mau elemento.
O símbolo do real antecede o número sem espaço:
Mal = pode ser usado como
PORTUGUÊS

R$1.000,00. No cifrão deve ser utilizada apenas uma bar-


ra vertical ($). 1. conjunção temporal, equivalente a “assim que”,
“logo que”, “quando” = Mal se levantou, já saiu.
2. advérbio de modo (antônimo de “bem”) = Você foi
mal na prova?

63
3. substantivo, podendo estar precedido de artigo ou ção, etc.
pronome = Há males que vêm pra bem! / O mal 9. Na ênclise e mesóclise: amá-lo, deixá-lo, dá-se,
não compensa. abraça-o, lança-o e amá-lo-ei, falar-lhe-ei, etc.
10. Nas formações em que o prefixo tem como se-
REFERÊNCIAS BIBLIOGRÁFICAS gundo termo uma palavra iniciada por “h”: sub-he-
SACCONI, Luiz Antônio. Nossa gramática completa pático, geo-história, neo-helênico, extra-humano,
Sacconi. 30.ª ed. Rev. São Paulo: Nova Geração, 2010. semi-hospitalar, super-homem.
Português linguagens: volume 1 / Wiliam Roberto Ce- 11. Nas formações em que o prefixo ou pseudopre-
reja, Thereza Cochar Magalhães. – 7.ª ed. Reform. – São fixo termina com a mesma vogal do segundo ele-
Paulo: Saraiva, 2010. mento: micro-ondas, eletro-ótica, semi-interno, au-
Português: novas palavras: literatura, gramática, reda- to-observação, etc.
ção / Emília Amaral... [et al.]. – São Paulo: FTD, 2000.
CAMPEDELLI, Samira Yousseff. Português – Literatura, O hífen é suprimido quando para formar outros ter-
Produção de Textos & Gramática. Volume único / Samira mos: reaver, inábil, desumano, lobisomem, reabilitar.
Yousseff, Jésus Barbosa Souza. – 3.ª edição – São Paulo:
Saraiva, 2002.

SITE
#FicaDica
http://www.pciconcursos.com.br/aulas/portugues/or- Lembrete da Zê!
tografia Ao separar palavras na translineação (mu-
dança de linha), caso a última palavra a ser
4. Hífen escrita seja formada por hífen, repita-o na
próxima linha. Exemplo: escreverei anti-in-
O hífen é um sinal diacrítico (que distingue) usado para flamatório e, ao final, coube apenas “anti-”.
ligar os elementos de palavras compostas (como ex-pre- Na próxima linha escreverei: “-inflamatório”
sidente, por exemplo) e para unir pronomes átonos a ver- (hífen em ambas as linhas). Devido à diagra-
bos (ofereceram-me; vê-lo-ei). Serve igualmente para fazer mação, pode ser que a repetição do hífen na
a translineação de palavras, isto é, no fim de uma linha, translineação não ocorra em meus conteú-
separar uma palavra em duas partes (ca-/sa; compa-/nhei-
dos, mas saiba que a regra é esta!
ro).

A) Uso do hífen que continua depois da Reforma


Ortográfica: B) Não se emprega o hífen:
1. Nas formações em que o prefixo ou falso prefi-
1. Em palavras compostas por justaposição que for- xo termina em vogal e o segundo termo inicia-se
mam uma unidade semântica, ou seja, nos termos em “r” ou “s”. Nesse caso, passa-se a duplicar estas
que se unem para formam um novo significado: consoantes: antirreligioso, contrarregra, infrassom,
tio-avô, porto-alegrense, luso-brasileiro, tenente-co- microssistema, minissaia, microrradiografia, etc.
ronel, segunda-feira, conta-gotas, guarda-chuva, ar- 2. Nas constituições em que o prefixo ou pseudopre-
co-íris, primeiro-ministro, azul-escuro. fixo termina em vogal e o segundo termo inicia-se
2. Em palavras compostas por espécies botânicas e com vogal diferente: antiaéreo, extraescolar, coedu-
zoológicas: couve-flor, bem-te-vi, bem-me-quer, cação, autoestrada, autoaprendizagem, hidroelétri-
abóbora-menina, erva-doce, feijão-verde.
co, plurianual, autoescola, infraestrutura, etc.
3. Nos compostos com elementos além, aquém, re-
3. Nas formações, em geral, que contêm os prefixos
cém e sem: além-mar, recém-nascido, sem-número,
“dês” e “in” e o segundo elemento perdeu o “h”
recém-casado.
inicial: desumano, inábil, desabilitar, etc.
4. No geral, as locuções não possuem hífen, mas al-
gumas exceções continuam por já estarem con- 4. Nas formações com o prefixo “co”, mesmo quando
sagradas pelo uso: cor-de-rosa, arco-da-velha, o segundo elemento começar com “o”: coopera-
mais-que-perfeito, pé-de-meia, água-de-colônia, ção, coobrigação, coordenar, coocupante, coautor,
queima-roupa, deus-dará. coedição, coexistir, etc.
5. Nos encadeamentos de vocábulos, como: ponte Rio- 5. Em certas palavras que, com o uso, adquiriram no-
-Niterói, percurso Lisboa-Coimbra-Porto e nas com- ção de composição: pontapé, girassol, paraquedas,
binações históricas ou ocasionais: Áustria-Hungria, paraquedista, etc.
Angola-Brasil, etc. 6. Em alguns compostos com o advérbio “bem”: ben-
6. Nas formações com os prefixos hiper-, inter- e su- feito, benquerer, benquerido, etc.
per- quando associados com outro termo que é
iniciado por “r”: hiper-resistente, inter-racial, super- Os prefixos pós, pré e pró, em suas formas correspon-
-racional, etc. dentes átonas, aglutinam-se com o elemento seguinte,
PORTUGUÊS

7. Nas formações com os prefixos ex-, vice-: ex-diretor, não havendo hífen: pospor, predeterminar, predetermina-
ex-presidente, vice-governador, vice-prefeito. do, pressuposto, propor.
8. Nas formações com os prefixos pós-, pré- e pró-: Escreveremos com hífen: anti-horário, anti-infeccio-
pré-natal, pré-escolar, pró-europeu, pós-gradua- so, auto-observação, contra-ataque, semi-interno, sobre-

64
-humano, super-realista, alto-mar. De acordo com a tonicidade, as palavras são classifi-
Escreveremos sem hífen: pôr do sol, antirreforma, cadas como:
antisséptico, antissocial, contrarreforma, minirrestaurante, Oxítonas – São aquelas cuja sílaba tônica recai sobre a
ultrassom, antiaderente, anteprojeto, anticaspa, antivírus, última sílaba: café – coração – Belém – atum – caju – papel
autoajuda, autoelogio, autoestima, radiotáxi. Paroxítonas – a sílaba tônica recai na penúltima sílaba:
útil – tórax – táxi – leque – sapato – passível
REFERÊNCIA BIBLIOGRÁFICA Proparoxítonas - a sílaba tônica está na antepenúlti-
SACCONI, Luiz Antônio. Nossa gramática completa ma sílaba: lâmpada – câmara – tímpano – médico – ônibus
Sacconi. 30.ª ed. Rev. São Paulo: Nova Geração, 2010.
Há vocábulos que possuem uma sílaba somente: são
SITE os chamados monossílabos. Estes são acentuados quando
http://www.pciconcursos.com.br/aulas/portugues/ tônicos e terminados em “a”, “e” ou “o”: vá – fé – pó - ré.
ortografia
2 Os acentos

A) acento agudo (´) – Colocado sobre as letras “a” e


EXERCÍCIOS COMENTADOS “i”, “u” e “e” do grupo “em” - indica que estas letras repre-
sentam as vogais tônicas de palavras como pá, caí, público.
1. (Polícia Federal – Escrivão de Polícia Federal – Ces- Sobre as letras “e” e “o” indica, além da tonicidade, timbre
pe – 2013 – adaptada) aberto: herói – céu (ditongos abertos).
B) acento circunflexo – (^) Colocado sobre as letras
A fim de solucionar o litígio, atos sucessivos e concatena- “a”, “e” e “o” indica, além da tonicidade, timbre fechado:
dos são praticados pelo escrivão. Entre eles, estão os atos tâmara – Atlântico – pêsames – supôs.
de comunicação, os quais são indispensáveis para que C) acento grave – (`) Indica a fusão da preposição “a”
os sujeitos do processo tomem conhecimento dos atos com artigos e pronomes: à – às – àquelas – àqueles
acontecidos no correr do procedimento e se habilitem a D) trema (¨) – De acordo com a nova regra, foi total-
exercer os direitos que lhes cabem e a suportar os ônus mente abolido das palavras. Há uma exceção: é utilizado
que a lei lhes impõe. em palavras derivadas de nomes próprios estrangeiros:
Disponível em: <http://jus.com.br> (com adaptações). mülleriano (de Müller)
E) til – (~) Indica que as letras “a” e “o” representam
No que se refere ao texto acima, julgue os itens seguin- vogais nasais: oração – melão – órgão – ímã
tes.
Não haveria prejuízo para a correção gramatical do texto 2.1 Regras fundamentais
nem para seu sentido caso o trecho “A fim de solucionar
o litígio” fosse substituído por Afim de dar solução à de- A) Palavras oxítonas: acentuam-se todas as oxítonas
manda e o trecho “tomem conhecimento dos atos acon- terminadas em: “a”, “e”, “o”, “em”, seguidas ou não do plu-
tecidos no correr do procedimento” fosse, por sua vez, ral(s): Pará – café(s) – cipó(s) – Belém.
substituído por conheçam os atos havidos no transcurso Esta regra também é aplicada aos seguintes casos:
do acontecimento. Monossílabos tônicos terminados em “a”, “e”, “o”, se-
guidos ou não de “s”: pá – pé – dó – há
( ) CERTO ( ) ERRADO Formas verbais terminadas em “a”, “e”, “o” tônicos,
seguidas de lo, la, los, las: respeitá-lo, recebê-lo, compô-lo
Resposta: Errado. “A fim” tem o sentido de “com a
intenção de”; já “afim”, “semelhança, afinidade”. Se a B) Paroxítonas: acentuam-se as palavras paroxítonas
primeira substituição fosse feita, o trecho estaria in- terminadas em:
correto gramatical e coerentemente. Portanto, nem há i, is: táxi – lápis – júri
a necessidade de avaliar a segunda substituição. us, um, uns: vírus – álbuns – fórum
l, n, r, x, ps: automóvel – elétron - cadáver – tórax –
Acentuação. fórceps
ã, ãs, ão, ãos: ímã – ímãs – órfão – órgãos
Quanto à acentuação, observamos que algumas pa- ditongo oral, crescente ou decrescente, seguido ou não
lavras têm acento gráfico e outras não; na pronúncia, ora de “s”: água – pônei – mágoa – memória
se dá maior intensidade sonora a uma sílaba, ora a outra.
Por isso, vamos às regras!
#FicaDica
1. Regras básicas
Memorize a palavra LINURXÃO. Repare que
A acentuação tônica está relacionada à intensida- esta palavra apresenta as terminações das
PORTUGUÊS

de com que são pronunciadas as sílabas das palavras. paroxítonas que são acentuadas: L, I N, U
Aquela que se dá de forma mais acentuada, conceitua-se (aqui inclua UM = fórum), R, X, Ã, ÃO. Assim
como sílaba tônica. As demais, como são pronunciadas ficará mais fácil a memorização!
com menos intensidade, são denominadas de átonas.

65
C) Proparoxítona: a palavra é proparoxítona quando 2.4 Regra do Hiato
a sua antepenúltima sílaba é tônica (mais forte). Quanto
à regra de acentuação: todas as proparoxítonas são acen- Quando a vogal do hiato for “i” ou “u” tônicos, segun-
tuadas, independentemente de sua terminação: árvore, da vogal do hiato, acompanhado ou não de “s”, haverá
paralelepípedo, cárcere. acento: saída – faísca – baú – país – Luís
Não se acentuam o “i” e o “u” que formam hiato
2.2 Regras especiais quando seguidos, na mesma sílaba, de l, m, n, r ou z:
Ra-ul, Lu-iz, sa-ir, ju-iz
Os ditongos de pronúncia aberta “ei”, “oi” (ditongos Não se acentuam as letras “i” e “u” dos hiatos se esti-
abertos), que antes eram acentuados, perderam o acento verem seguidas do dígrafo nh:
de acordo com a nova regra, mas desde que estejam em ra-i-nha, ven-to-i-nha.
palavras paroxítonas. Não se acentuam as letras “i” e “u” dos hiatos se vie-
rem precedidas de vogal idêntica: xi-i-ta, pa-ra-cu-u-ba
Não serão mais acentuados “i” e “u” tônicos, forman-
FIQUE ATENTO! do hiato quando vierem depois de ditongo (nas paroxí-
Alerta da Zê! Cuidado: Se os ditongos aber- tonas):
tos estiverem em uma palavra oxítona (he-
rói) ou monossílaba (céu) ainda são acen-
Antes Agora
tuados: dói, escarcéu.
bocaiúva bocaiuva
feiúra feiura
Antes Agora Sauípe Sauipe
assembléia assembleia
O acento pertencente aos encontros “oo” e “ee” foi
idéia ideia abolido:
geléia geleia
jibóia jiboia Antes Agora
apóia (verbo apoiar) apoia crêem creem
paranóico paranoico lêem leem
vôo voo
2.3 Acento Diferencial
enjôo enjoo
Representam os acentos gráficos que, pelas regras de
acentuação, não se justificariam, mas são utilizados para
diferenciar classes gramaticais entre determinadas pala- #FicaDica
vras e/ou tempos verbais. Por exemplo: Memorize a palavra CREDELEVÊ. São os
Pôr (verbo) X por (preposição) / pôde (pretérito perfeito verbos que, no plural, dobram o “e”, mas
do Indicativo do verbo “poder”) X pode (presente do Indica- que não recebem mais acento como antes:
tivo do mesmo verbo). CRER, DAR, LER e VER.
Se analisarmos o “pôr” - pela regra das monossílabas:
terminada em “o” seguida de “r” não deve ser acentuada,
mas nesse caso, devido ao acento diferencial, acentua-se, Repare:
para que saibamos se se trata de um verbo ou preposição. O menino crê em você. / Os meninos creem em você.
Os demais casos de acento diferencial não são mais Elza lê bem! / Todas leem bem!
utilizados: para (verbo), para (preposição), pelo (substanti- Espero que ele dê o recado à sala. / Esperamos que os
vo), pelo (preposição). Seus significados e classes gramati- garotos deem o recado!
cais são definidos pelo contexto. Rubens vê tudo! / Eles veem tudo!
Polícia para o trânsito para que se realize a operação Cuidado! Há o verbo vir: Ele vem à tarde! / Eles vêm
planejada. = o primeiro “para” é verbo; o segundo, conjun- à tarde!
ção (com relação de finalidade). As formas verbais que possuíam o acento tônico na
raiz, com “u” tônico precedido de “g” ou “q” e seguido de
“e” ou “i” não serão mais acentuadas:
#FicaDica
Quando, na frase, der para substituir o “por” Antes Depois
por “colocar”, estaremos trabalhando com
um verbo, portanto: “pôr”; nos demais ca- apazigúe (apaziguar) apazigue
PORTUGUÊS

sos, “por” é preposição: Faço isso por você. averigúe (averiguar) averigue
/ Posso pôr (colocar) meus livros aqui? argúi (arguir) argui

66
Acentuam-se os verbos pertencentes a terceira pessoa do plural de: ele tem – eles têm / ele vem – eles vêm (verbo
vir). A regra prevalece também para os verbos conter, obter, reter, deter, abster: ele contém – eles contêm, ele obtém – eles
obtêm, ele retém – eles retêm, ele convém – eles convêm.

REFERÊNCIAS BIBLIOGRÁFICAS
SACCONI, Luiz Antônio. Nossa gramática completa Sacconi. 30.ª ed. Rev. São Paulo: Nova Geração, 2010.
Português linguagens: volume 1 / Wiliam Roberto Cereja, Thereza Cochar Magalhães. – 7.ª ed. Reform. – São Paulo: Sa-
raiva, 2010.

SITE
http://www.brasilescola.com/gramatica/acentuacao.htm

EXERCÍCIOS COMENTADOS

1. (Polícia Federal – Agente de Polícia Federal – Cespe – 2014) Os termos “série” e “história” acentuam-se em conformi-
dade com a mesma regra ortográfica.

( ) CERTO ( ) ERRADO

Resposta: Certo. “Série” = acentua-se a paroxítona terminada em ditongo / “história” - acentua-se a paroxítona termi-
nada em ditongo
Ambas são acentuadas devido à regra da paroxítona terminada em ditongo.
Observação: nestes casos, admitem-se as separações “sé-ri-e” e “his-tó-ri-as”, o que as tornaria proparoxítonas.

2. (Anatel – Técnico Administrativo – cespe – 2012) Nas palavras “análise” e “mínimos”, o emprego do acento gráfico tem
justificativas gramaticais diferentes.

( ) CERTO ( ) ERRADO

Resposta: Errado. Análise = proparoxítona / mínimos = proparoxítona. Ambas são acentuadas pela mesma regra (an-
tepenúltima sílaba é tônica, “mais forte”).

3. (Ancine – Técnico Administrativo – cespe – 2012) Os vocábulos “indivíduo”, “diária” e “paciência” recebem acento grá-
fico com base na mesma regra de acentuação gráfica.

( ) CERTO ( ) ERRADO

Resposta: Certo. Indivíduo = paroxítona terminada em ditongo; diária = paroxítona terminada em ditongo; paciência =
paroxítona terminada em ditongo. Os três vocábulos são acentuados devido à mesma regra.

4. (Ibama – Técnico Administrativo – cespe – 2012) As palavras “pó”, “só” e “céu” são acentuadas de acordo com a mesma
regra de acentuação gráfica.

( ) CERTO ( ) ERRADO
Resposta: Errado. Pó = monossílaba terminada em “o”; só = monossílaba terminada em “o”; céu = monossílaba
terminada em ditongo aberto “éu”.
PORTUGUÊS

67
IDENTIFICAÇÃO DE DIFERENTES ESTRATÉGIAS QUE CONTRIBUEM PARA A CONTINUIDADE
DO TEXTO (FIGURAS DE LINGUAGEM, PRONOMES RELATIVOS, DEMONSTRATIVOS, ETC);
COMPREENSÃO DE ESTRUTURAS TEMÁTICA E LEXICAL COMPLEXAS.

FIGURA DE LINGUAGEM, PENSAMENTO E CONSTRUÇÃO

Disponível em: <http://www.terapiadapalavra.com.br/figuras-de-linguagem-na-escrita-literaria/> Acesso abr, 2018.

A figura de palavra consiste na substituição de uma palavra por outra, isto é, no emprego figurado, simbólico, seja
por uma relação muito próxima (contiguidade), seja por uma associação, uma comparação, uma similaridade. São
construções que transformam o significado das palavras para tirar delas maior efeito ou para construir uma mensagem
nova.

1. Tipos de Figuras de Linguagem

1.1. Figuras de Som

Aliteração - Consiste na repetição de consoantes como recurso para intensificação do ritmo ou como efeito sonoro
significativo.
Três pratos de trigo para três tigres tristes.
Vozes veladas, veludosas vozes... (Cruz e Sousa)
Quem com ferro fere com ferro será ferido.

Assonância - Consiste na repetição ordenada de sons vocálicos idênticos: “Sou um mulato nato no sentido lato
mulato democrático do litoral.”

Onomatopeia - Ocorre quando se tentam reproduzir na forma de palavras os sons da realidade: Os sinos faziam
blem, blem, blem.

Paranomásia – é o uso de sons semelhantes em palavras próximas: “A fossa, a bossa, a nossa grande dor...” (Carlos Lyra)

1.2. Figuras de Palavras ou de Pensamento

1.2.1. Metáfora

Consiste em utilizar uma palavra ou uma expressão em lugar de outra, sem que haja uma relação real, mas em
virtude da circunstância de que o nosso espírito as associa e percebe entre elas certas semelhanças. É o emprego da
palavra fora de seu sentido normal.
PORTUGUÊS

Observação:
Toda metáfora é uma espécie de comparação implícita, em que o elemento comparativo não aparece.
Seus olhos são como luzes brilhantes.
O exemplo acima mostra uma comparação evidente, através do emprego da palavra como.

68
Observe agora: Seus olhos são luzes brilhantes. 1.2.3. Catacrese
Neste exemplo não há mais uma comparação (note a
ausência da partícula comparativa), e sim símile, ou seja, Trata-se de uma metáfora que, dado seu uso contínuo,
qualidade do que é semelhante. cristalizou-se. A catacrese costuma ocorrer quando, por
Por fim, no exemplo: As luzes brilhantes olhavam-me. falta de um termo específico para designar um conceito,
Há substituição da palavra olhos por luzes brilhantes. toma-se outro “emprestado”. Assim, passamos a empre-
Esta é a verdadeira metáfora. gar algumas palavras fora de seu sentido original. Exem-
plos: “asa da xícara”, “batata da perna”, “maçã do rosto”,
Outros exemplos: “pé da mesa”, “braço da cadeira”, “coroa do abacaxi”.
“Meu pensamento é um rio subterrâneo.” (Fernando
Pessoa) 1.2.4. Perífrase ou Antonomásia
Neste caso, a metáfora é possível na medida em que
o poeta estabelece relações de semelhança entre um rio Trata-se de uma expressão que designa um ser atra-
subterrâneo e seu pensamento (pode estar relacionando vés de alguma de suas características ou atributos, ou de
a fluidez, a profundidade, a inatingibilidade, etc.). um fato que o celebrizou. É a substituição de um nome
por outro ou por uma expressão que facilmente o iden-
Minha alma é uma estrada de terra que leva a lugar tifique:
algum. A Cidade Maravilhosa (= Rio de Janeiro) continua
Uma estrada de terra que leva a lugar algum é, na fra- atraindo visitantes do mundo todo.
se acima, uma metáfora. Por trás do uso dessa expressão A Cidade-Luz (=Paris)
que indica uma alma rústica e abandonada (e angustia- O rei das selvas (=o leão)
damente inútil), há uma comparação subentendida: Mi-
nha alma é tão rústica, abandonada (e inútil) quanto uma
Observação:
estrada de terra que leva a lugar algum.
Quando a perífrase indica uma pessoa, recebe o
A Amazônia é o pulmão do mundo. nome de antonomásia. Exemplos:
Em sua mente povoa só inveja. O Divino Mestre (= Jesus Cristo) passou a vida prati-
cando o bem.
1.2.2. Metonímia (ou sinédoque) O Poeta dos Escravos (= Castro Alves) morreu muito
jovem.
É a substituição de um nome por outro, em virtude de O Poeta da Vila (= Noel Rosa) compôs lindas canções.
existir entre eles algum relacionamento. Tal substituição
pode acontecer dos seguintes modos: 1.2.4. Sinestesia
Autor pela obra: Gosto de ler Machado de Assis. (=
Gosto de ler a obra literária de Machado de Assis). Consiste em mesclar, numa mesma expressão, as sen-
Inventor pelo invento: Édson ilumina o mundo. (= As sações percebidas por diferentes órgãos do sentido. É o
lâmpadas iluminam o mundo). cruzamento de sensações distintas.
Símbolo pelo objeto simbolizado: Não te afastes da Um grito áspero revelava tudo o que sentia. (grito =
cruz. (= Não te afastes da religião). auditivo; áspero = tátil)
Lugar pelo produto do lugar: Fumei um saboroso No silêncio escuro do seu quarto, aguardava os aconte-
Havana. (= Fumei um saboroso charuto). cimentos. (silêncio = auditivo; escuro = visual)
Efeito pela causa: Sócrates bebeu a morte. (= Sócrates Tosse gorda. (sensação auditiva X sensação tátil)
tomou veneno).
Causa pelo efeito: Moro no campo e como do meu tra- 1.2.5. Antítese
balho. (= Moro no campo e como o alimento que produzo).
Continente pelo conteúdo: Bebeu o cálice todo. (= Consiste no emprego de palavras que se opõem
Bebeu todo o líquido que estava no cálice). quanto ao sentido. O contraste que se estabelece serve,
Instrumento pela pessoa que utiliza: Os microfones essencialmente, para dar uma ênfase aos conceitos en-
foram atrás dos jogadores. (= Os repórteres foram atrás volvidos que não se conseguiria com a exposição isolada
dos jogadores).
dos mesmos. Observe os exemplos:
Parte pelo todo: Várias pernas passavam apressada-
“O mito é o nada que é tudo.” (Fernando Pessoa)
mente. (= Várias pessoas passavam apressadamente).
O corpo é grande e a alma é pequena.
Gênero pela espécie: Os mortais pensam e sofrem
nesse mundo. (= Os homens pensam e sofrem nesse mun- “Quando um muro separa, uma ponte une.”
do). Não há gosto sem desgosto.
Singular pelo plural: A mulher foi chamada para ir às
ruas na luta por seus direitos. (= As mulheres foram cha- 1.2.6. Paradoxo ou oximoro
madas, não apenas uma mulher).
Marca pelo produto: Minha filha adora danone. (= É a associação de ideias, além de contrastantes, con-
Minha filha adora o iogurte que é da marca Danone). traditórias. Seria a antítese ao extremo.
PORTUGUÊS

Espécie pelo indivíduo: O homem foi à Lua. (= Alguns Era dor, sim, mas uma dor deliciosa.
astronautas foram à Lua). Ouvimos as vozes do silêncio.
Símbolo pela coisa simbolizada: A balança penderá
para teu lado. (= A justiça ficará do teu lado).

69
1.2.7. Eufemismo 1.3.2. Gradação (ou clímax)

É o emprego de uma expressão mais suave, mais no- Apresentação de ideias por meio de palavras, sinôni-
bre ou menos agressiva, para comunicar alguma coisa mas ou não, em ordem ascendente (clímax) ou descen-
áspera, desagradável ou chocante. dente (anticlímax). Observe este exemplo:
Depois de muito sofrimento, entregou a alma ao Se- Havia o céu, havia a terra, muita gente e mais Joana
nhor. (= morreu) com seus olhos claros e brincalhões...
O prefeito ficou rico por meios ilícitos. (= roubou)
Fernando faltou com a verdade. (= mentiu) O objetivo do narrador é mostrar a expressividade
Faltar à verdade. (= mentir) dos olhos de Joana. Para chegar a este detalhe, ele se
refere ao céu, à terra, às pessoas e, finalmente, a Joana e
1.2.8. Ironia seus olhos. Nota-se que o pensamento foi expresso em
ordem decrescente de intensidade. Outros exemplos:
É sugerir, pela entoação e contexto, o contrário do “Vive só para mim, só para a minha vida, só para meu
que as palavras ou frases expressam, geralmente apre- amor”. (Olavo Bilac)
sentando intenção sarcástica. A ironia deve ser muito “O trigo... nasceu, cresceu, espigou, amadureceu, co-
bem construída para que cumpra a sua finalidade; mal lheu-se.” (Padre Antônio Vieira)
construída, pode passar uma ideia exatamente oposta à
desejada pelo emissor. 1.3.3. Elipse
Como você foi bem na prova! Não tirou nem a nota
mínima. Consiste na omissão de um ou mais termos numa
Parece um anjinho aquele menino, briga com todos oração e que podem ser facilmente identificados, tanto
que estão por perto. por elementos gramaticais presentes na própria oração,
O governador foi sutil como um elefante. quanto pelo contexto.
A catedral da Sé. (a igreja catedral)
1.2.9. Hipérbole Domingo irei ao estádio. (no domingo eu irei ao es-
tádio)
É a expressão intencionalmente exagerada com o in-
tuito de realçar uma ideia. 1.3.4. Zeugma
Faria isso milhões de vezes se fosse preciso.
“Rios te correrão dos olhos, se chorares.” (Olavo Bilac) Zeugma é uma forma de elipse. Ocorre quando é feita
O concurseiro quase morre de tanto estudar! a omissão de um termo já mencionado anteriormente.
Ele gosta de geografia; eu, de português. (eu gosto de
1.2.10. Prosopopeia ou Personificação português)
Na casa dela só havia móveis antigos; na minha, só
É a atribuição de ações ou qualidades de seres ani-
modernos. (só havia móveis)
mados a seres inanimados, ou características humanas a
Ela gosta de natação; eu, de vôlei. (gosto de)
seres não humanos. Observe os exemplos:
As pedras andam vagarosamente.
1.3.5. Silepse
O livro é um mudo que fala, um surdo que ouve, um
cego que guia.
A silepse é a concordância que se faz com o termo
A floresta gesticulava nervosamente diante da serra.
que não está expresso no texto, mas, sim, subentendido.
Chora, violão.
É uma concordância anormal, psicológica, porque se faz
1.3. Figuras de Construção ou de Sintaxe com um termo oculto, facilmente identificado. Há três ti-
pos de silepse: de gênero, número e pessoa.
1.3.1. Apóstrofe
Silepse de Gênero - Os gêneros são masculino e fe-
Consiste na “invocação” de alguém ou de alguma coi- minino. Ocorre a silepse de gênero quando a concordân-
sa personificada, de acordo com o objetivo do discurso, cia se faz com a ideia que o termo comporta. Exemplos:
que pode ser poético, sagrado ou profano. Caracteriza-
-se pelo chamamento do receptor da mensagem, seja ele A) A bonita Porto Velho sofreu mais uma vez com o
imaginário ou não. A introdução da apóstrofe interrompe calor intenso.
a linha de pensamento do discurso, destacando-se assim Neste caso, o adjetivo bonita não está concordando
a entidade a que se dirige e a ideia que se pretende pôr com o termo Porto Velho, que gramaticalmente pertence
em evidência com tal invocação. Realiza-se por meio do ao gênero masculino, mas com a ideia contida no termo
vocativo. Exemplos: (a cidade de Porto Velho).
Moça, que fazes aí parada?
PORTUGUÊS

“Pai Nosso, que estais no céu” B) Vossa Excelência está preocupado.


Deus, ó Deus! Onde estás? O adjetivo preocupado concorda com o sexo da pes-
soa, que nesse caso é masculino, e não com o termo Vos-
sa Excelência.

70
Silepse de Número - Os números são singular e pronome “lo” classificado como objeto direto pleonástico.
plural. A silepse de número ocorre quando o verbo da Outro exemplo:
oração não concorda gramaticalmente com o sujeito da Aos funcionários, não lhes interessam tais medidas.
oração, mas com a ideia que nele está contida. Exemplos: Aos funcionários, lhes = Objeto Indireto
A procissão saiu. Andaram por todas as ruas da cidade
de Salvador. Neste caso, há um pleonasmo do objeto indireto, e o
O povo corria por todos os lados e gritavam muito alto. pronome “lhes” exerce a função de objeto indireto pleo-
nástico.
Note que nos exemplos acima, os verbos andaram e Observação:
gritavam não concordam gramaticalmente com os sujei- O pleonasmo só tem razão de ser quando confere
tos das orações (que se encontram no singular, procissão mais vigor à frase; caso contrário, torna-se um pleonas-
e povo, respectivamente), mas com a ideia que neles está mo vicioso:
contida. Procissão e povo dão a ideia de muita gente, por Vi aquela cena com meus próprios olhos.
isso que os verbos estão no plural. Vamos subir para cima.
Ele desceu pra baixo.
Silepse de Pessoa - Três são as pessoas gramaticais:
eu, tu e ele (as três pessoas do singular); nós, vós, eles 1.3.8. Anáfora
(as três do plural). A silepse de pessoa ocorre quando há
um desvio de concordância. O verbo, mais uma vez, não É a repetição de uma ou mais palavras no início de
concorda com o sujeito da oração, mas sim com a pessoa várias frases, criando, assim, um efeito de reforço e de
que está inscrita no sujeito. Exemplos: coerência. Pela repetição, a palavra ou expressão em cau-
O que não compreendo é como os brasileiros persista- sa é posta em destaque, permitindo ao escritor valorizar
mos em aceitar essa situação. determinado elemento textual. Os termos anafóricos po-
Os agricultores temos orgulho de nosso trabalho. dem muitas vezes ser substituídos por pronomes.
“Dizem que os cariocas somos poucos dados aos jar- Encontrei um amigo ontem. Ele me disse que te co-
dins públicos.” (Machado de Assis) nhecia.
“Tudo cura o tempo, tudo gasta, tudo digere, tudo aca-
Observe que os verbos persistamos, temos e somos ba.” (Padre Vieira)
não concordam gramaticalmente com os seus sujeitos
(brasileiros, agricultores e cariocas, que estão na terceira 1.3.9. Anacoluto
pessoa), mas com a ideia que neles está contida (nós, os
brasileiros, os agricultores e os cariocas). Consiste na mudança da construção sintática no meio
da frase, ficando alguns termos desligados do resto do
1.3.6. Polissíndeto / Assíndeto período. É a quebra da estrutura normal da frase para a
introdução de uma palavra ou expressão sem nenhuma
Para estudarmos as duas figuras de construção é ne- ligação sintática com as demais.
cessário recordar um conceito estudado em sintaxe sobre Esses alunos da escola, não se pode duvidar deles.
período composto. No período composto por coordena- Morrer, todo haveremos de morrer.
ção, podemos ter orações sindéticas ou assindéticas. A Aquele garoto, você não disse que ele chegaria logo?
oração coordenada ligada por uma conjunção (conecti-
vo) é sindética; a oração que não apresenta conectivo é
A expressão “esses alunos da escola”, por exemplo,
assindética. Recordado esse conceito, podemos definir as
deveria exercer a função de sujeito. No entanto, há uma
duas figuras de construção:
interrupção da frase e esta expressão fica à parte, não
A) Polissíndeto - É uma figura caracterizada pela re-
exercendo nenhuma função sintática. O anacoluto tam-
petição enfática dos conectivos. Observe o exem-
bém é chamado de “frase quebrada”, pois corresponde
plo: O menino resmunga, e chora, e grita, e nin-
a uma interrupção na sequência lógica do pensamento.
guém faz nada.
B) Assíndeto - É uma figura caracterizada pela au-
sência, pela omissão das conjunções coordenati- Observação:
vas, resultando no uso de orações coordenadas O anacoluto deve ser usado com finalidade expressi-
assindéticas. Exemplos: va em casos muito especiais. Em geral, evite-o.
Tens casa, tens roupa, tens amor, tens família.
“Vim, vi, venci.” (Júlio César)

1.3.7. Pleonasmo

Consiste na repetição de um termo ou ideia, com as


mesmas palavras ou não. A finalidade do pleonasmo é
realçar a ideia, torná-la mais expressiva.
PORTUGUÊS

O problema da violência, é necessário resolvê-lo logo.


Nesta oração, os termos “o problema da violência”
e “lo” exercem a mesma função sintática: objeto direto.
Assim, temos um pleonasmo do objeto direto, sendo o

71
1.3.10. Hipérbato / Inversão

É a inversão da estrutura frásica, isto é, a inversão da ordem direta dos termos da oração, fazendo com que o sujeito
venha depois do predicado:
Ao ódio venceu o amor. (Na ordem direta seria: O amor venceu ao ódio)
Dos meus problemas cuido eu! (Na ordem direta seria: Eu cuido dos meus problemas)

#FicaDica
O nosso Hino Nacional é um exemplo de
hipérbato, já que, na ordem direta, teríamos:
“As margens plácidas do Ipiranga ouviram o
brado retumbante de um povo heroico”.

EXERCÍCIOS COMENTADOS

1. (DEFENSORIA PÚBLICA DO ESTADO DO RIO DE JANEIRO – TÉCNICO SUPERIOR ESPECIALIZADO EM BIBLIO-


TECONOMIA SUPERIOR – FGV/2014 - adaptada) Ao dizer que os shoppings são “cidades”, faz-se o uso de um tipo
de linguagem figurada denominada

a) metonímia.
b) eufemismo.
c) hipérbole.
d) metáfora.
e) catacrese.

Resposta: Letra D. A metáfora consiste em retirar uma palavra de seu contexto convencional (denotativo) e trans-
portá-la para um novo campo de significação (conotativa), por meio de uma comparação implícita, de uma simila-
ridade existente entre as duas.
(Fonte:http://educacao.uol.com.br/disciplinas/portugues/metafora-figura-de-palavra-variacoes-e-exemplos.htm)

2. (PREFEITURA DE ARCOVERDE/PE - ADMINISTRADOR DE RECURSOS HUMANOS – SUPERIOR - CONPASS/2014)


Identifique a figura de linguagem presente na tira seguinte:

a) metonímia
b) prosopopeia
c) hipérbole
d) eufemismo
e) onomatopeia
PORTUGUÊS

Resposta: Letra D. “Eufemismo = é o emprego de uma expressão mais suave, mais nobre ou menos agressiva, para
comunicar alguma coisa áspera, desagradável ou chocante”. No caso da tirinha, é utilizada a expressão “deram suas
vidas por nós” no lugar de “que morreram por nós”.

72
3. (CASAL/AL - ADMINISTRADOR DE REDE – SUPE- interrogação e reticências) é uma característica da função
RIOR - COPEVE/UFAL/2014) emotiva, pois transmite a subjetividade da mensagem e
Está tão quente que dá para fritar um ovo no asfalto. reforça a entonação emotiva. Essa função é comum em
O dito popular é, na maioria das vezes, uma figura de poemas ou narrativas de teor dramático ou romântico.
linguagem. Entre as 14h30min e às 15h desta terça-feira,
horário do dia em que o calor é mais intenso, a tempera- Função conativa ou apelativa: O objetivo é de in-
tura do asfalto, medida com um termômetro de contato, fluenciar, convencer o receptor de alguma coisa por meio
chegou a 65ºC. Para fritar um ovo, seria preciso que o de uma ordem (uso de vocativos), sugestão, convite ou
local alcançasse aproximadamente 90ºC. apelo (daí o nome da função). Os verbos costumam estar
Disponível em: http://zerohora.clicrbs.com.br. Acesso no imperativo (Compre! Faça!) ou conjugados na 2.ª ou 3.ª
em: 22 jan. 2014. pessoa (Você não pode perder! Ele vai melhorar seu desem-
O texto cita que o dito popular “está tão quente que dá penho!). Esse tipo de função é muito comum em textos
para fritar um ovo no asfalto” expressa uma figura de publicitários, em discursos políticos ou de autoridade.
linguagem. O autor do texto refere-se a qual figura de
linguagem? Função metalinguística: Essa função se refere à me-
a) Eufemismo. talinguagem, que é quando o emissor explica um código
b) Hipérbole. usando o próprio código. Quando um poema fala da pró-
c) Paradoxo. pria ação de se fazer um poema, por exemplo:
d) Metonímia. “Pegue um jornal
e) Hipérbato. Pegue a tesoura.
Escolha no jornal um artigo do tamanho que você dese-
Resposta: Letra B. A expressão é um exagero! Ela ser- ja dar a seu poema.
ve apenas para representar o calor excessivo que está Recorte o artigo.”
fazendo. A figura que é utilizada “mil vezes” (!) para Este trecho da poesia, intitulada “Para fazer um poema
atingir tal objetivo é a hipérbole. dadaísta” utiliza o código (poema) para explicar o próprio
ato de fazer um poema.
REFERÊNCIAS BIBLIOGRÁFICAS Função fática: O objetivo dessa função é estabele-
SACCONI, Luiz Antônio. Nossa gramática completa cer uma relação com o emissor, um contato para verificar
Sacconi. 30.ª ed. Rev. São Paulo: Nova Geração, 2010. se a mensagem está sendo transmitida ou para dilatar a
CEREJA, Wiliam Roberto, MAGALHÃES, Thereza Co- conversa. Quando estamos em um diálogo, por exemplo,
char. Português linguagens: volume 1 – 7.ª ed. Reform. – e dizemos ao nosso receptor “Está entendendo?”, estamos
São Paulo: Saraiva, 2010. utilizando este tipo de função; ou quando atendemos o
CAMPEDELLI, Samira Yousseff. Português – Literatura, celular e dizemos “Oi” ou “Alô”.
Produção de Texto & Gramática – Volume único / Samira
Yousseff Campedelli, Jésus Barbosa Souza. – 3.ª edição – Função poética: O objetivo do emissor é expressar
São Paulo: Saraiva, 2002. seus sentimentos através de textos que podem ser enfa-
tizados por meio das formas das palavras, da sonoridade,
SITES do ritmo, além de elaborar novas possibilidades de com-
Disponível em: <http://www.soportugues.com.br/se- binações dos signos linguísticos. É presente em textos
coes/estil/estil8.php> literários, publicitários e em letras de música.
Disponível em: <http://www.soportugues.com.br/se- Por exemplo: negócio/ego/ócio/cio/0
coes/estil/estil5.php> Na poesia acima “Epitáfio para um banqueiro”, José de
Disponível em: <http://www.soportugues.com.br/se- Paulo Paes faz uma combinação de palavras que passa a
coes/estil/estil2.php> ideia do dia a dia de um banqueiro, de acordo com o poeta

FUNÇÕES DA LINGUAGEM

Função referencial ou denotativa: transmite uma


informação objetiva, expõe dados da realidade de modo
objetivo, não faz comentários, nem avaliação. Geralmente,
o texto se apresenta na terceira pessoa do singular ou plu-
ral, pois transmite impessoalidade. A linguagem é denota-
tiva, ou seja, não há possibilidades de outra interpretação
além da que está exposta. Em alguns textos é mais pre-
dominante essa função, como nos científicos, jornalísticos,
técnicos, didáticos ou em correspondências comerciais.

Função emotiva ou expressiva: o objetivo do emissor


PORTUGUÊS

é transmitir suas emoções e anseios. A realidade é trans-


mitida sob o ponto de vista do emissor, a mensagem é
subjetiva e centrada no emitente e, portanto, apresenta-se
na primeira pessoa. A pontuação (ponto de exclamação,

73
HORA DE PRATICAR!

1. (MAPA – Auditor Fiscal Federal Agropecuário – Médico Veterinário – Superior – ESAF – 2017) Assinale a opção
que apresenta desvio de grafia da palavra.
A acupuntura é uma terapia da medicina tradicional chinesa que favorece a regularização dos processos fisiológicos do
corpo, no sentido de promover ou recuperar o estado natural de saúde e equilíbrio. Pode ser usada preventivamente (1)
para evitar o desenvolvimento de doenças, como terapia curativa no caso de a doença estar instalada ou como método
paliativo (2) em casos de doenças crônicas de difícil tratamento. Tem também uma ação importante na medicina rejenera-
tiva (3) e na reabilitação. O tratamento de acupuntura consiste na introdução de agulhas filiformes no corpo dos animais.
Em geral são deixadas cerca de 15 a 20 minutos. A colocação das agulhas não é dolorosa para os animais e é possível
observar durante os tratamentos diferentes reações fisiológicas (4), indicadoras de que o tratamento está atingindo o
efeito terapêutico (5) desejado.

Disponível: <http://www.veterinariaholistica.net/acupuntura-fitoterapia-e-homeopatia.html/>. Acesso em 28/11/2017.


(Com adaptações)

a) (1)
b) (2)
c) (3)
d) (4)
e) (5)

2. (TRT – 21.ª Região-RN – Técnico Judiciário – Área Administrativa – Médio – FCC – 2017) Respeitando-se as
normas de redação do Manual da Presidência da República, a frase correta é:

a) Solicito a Vossa Senhoria que verifique a possibilidade de implementação de projeto de treinamento de pessoal para
operar os novos equipamentos gráficos a serem instalados em seu setor.
b) Venho perguntar-lhe, por meio desta, sobre a data em que Vossa Excelência pretende nomear vosso representante
na Comissão Organizadora.
c) Digníssimo Senhor: eu venho por esse comunicado, informar, que será organizado seminário, sobre o uso eficiente
de recursos hídricos, em data ainda a ser definida.

d) Haja visto que o projeto anexo contribue para o desenvolvimento do setor em questão, informamos, por meio deste
Ofício, que será amplamente analisado por especialistas.
e) Neste momento, conforme solicitação enviada à Vossa Senhoria anexo, não se deve adotar medidas que possam
com- prometer vossa realização do projeto mencionado.

3. (TRE-MS – Estágio – Jornalismo – TRE-MS – 2014) Analise as assertivas abaixo:

I. O ladrão era de menor.


II. Não há regra sem exceção.
III. É mais saudável usar menas roupa no calor.
IV. O policial foi à delegacia em compania do meliante.
V. Entre eu e você não existe mais nada.

A opção que apresenta vícios de linguagem é:

a) I e III.
b) I, II e IV.
c) II e IV.
d) I, III, IV e V.
e) III, IV e V.

4. (TRE-MS – Estágio – Jornalismo – TRE-MS – 2014) De acordo com a nova ortografia, assinale o item em que todas
PORTUGUÊS

as palavras estão corretas:

a) autoajuda – anti-inflamatório – extrajudicial.


b) supracitado – semi-novo – telesserviço.

74
c) ultrassofisticado – hidro-elétrica – ultra-som.
d) contrarregra – autopista – semi-aberto.
e) contrarrazão – infra-estrutura – coprodutor.

5. (TRE-MS – Estágio – Jornalismo – TRE-MS – 2014) O uso correto do porquê está na opção:

a) Por quê o homem destrói a natureza?


b) Ela chorou por que a humilharam.
c) Você continua implicando comigo porque sou pobre?
d) Ninguém sabe o por quê daquele gesto.
e) Ela me fez isso, porquê?

6. (TJ-PA – Médico Psiquiatra – Superior – VUNESP – 2014)

Assinale a alternativa que completa, correta e respectivamente, as lacunas, de acordo com a norma-padrão da língua
portuguesa, considerando que o termo que preenche a terceira lacuna é empregado para indicar que um evento está
prestes a acontecer

a) anúncio ... A ... Iminente.


b) anuncio ... À ... Iminente.
c) anúncio ... À ... Iminente.
d) anúncio ... A ... Eminente.
e) anuncio ... À ... Eminente.

7. (CEFET-RJ – REVISOR DE TEXTOS – CESGRANRIO – 2014) Observe a grafia das palavras do trecho a seguir.
A macro-história da humanidade mostra que todos encaram os relatos pessoais como uma forma de se manterem vivos.
Desde a idade do domínio do fogo até a era das multicomunicações, os homens tem demonstrado que querem pôr sua
marca no mundo porque se sentem superiores.
A palavra que NÃO está grafada corretamente é

a) macro-história.
b) multicomunicações.
c) tem.
d) pôr.
e) porque.

8. (Liquigás – Profissional Júnior – Ciências Contábeis – cegranrio – 2014) O grupo em que todas as palavras estão
grafadas de acordo com a norma-padrão da Língua Portuguesa é

a) gorjeta, ogeriza, lojista, ferrujem


b) pedágio, ultrage, pagem, angina
PORTUGUÊS

c) refújio, agiota, rigidez, rabujento


d) vigência, jenipapo, fuligem, cafajeste
e) sargeta, jengiva, jiló, lambujem

75
9. (SIMAE – Agente Administrativo – ASSCON-PP – b) Consultaram o juíz acerca da possibilidade de voltar
2014) Assinale a alternativa que apresenta apenas pala- atraz na suspensão do jogador, mas ele foi categórico
vras escritas de forma incorreta. quanto a impossibilidade de rever sua posição.
c) Vossa Excelência leu o documento que será apresen-
a) Cremoso, coragem, cafajeste, realizar; tado em rede nacional daqui a pouco, pela voz de Sua
b) Caixote, encher, análise, poetisa; Excelência, o Senhor Ministro da Educação?
c) Traje, tanger, portuguesa, sacerdotisa; d) A reportagem sobre fascínoras famosos não foi nada
d) Pagem, mujir, vaidozo, enchergar; positiva para o público jovem que estava presente, de
que se desculparam os idealizadores do programa.
10. (Receita Federal – Auditor Fiscal – ESAF – 2014) e) Estudantes e professores são entusiastas de oferecer
Assinale a opção que corresponde a erro gramatical ou aos jovens ingressantes no curso o compartilhamento
de grafia de palavra inserido na transcrição do texto. de projetos, com que serão também autores.

A Receita Federal nem sempre teve esse (1) nome. Secre- 13. (TRE-MS – Estágio – Jornalismo – TRE-MS – 2014)
taria da Receita Federal é apenas a mais recente denomi- A acentuação correta está na alternativa:
nação da Administração Tributária Brasileira nestes cinco
séculos de existência. Sua criação tornou-se (2) necessária a) eu abençôo – eles crêem – ele argúi.
para modernizar a máquina arrecadadora e fiscalizadora, b) platéia – tuiuiu – instrui-los.
bem como para promover uma maior integração entre o c) ponei – geléia – heroico.
Fisco e os Contribuintes, facilitando o cumprimento ex- d) eles têm – ele intervém – ele constrói.
pontâneo (3) das obrigações tributárias e a solução dos e) lingüiça – feiúra – idéia.
eventuais problemas, bem como o acesso às (4) informa-
ções pessoais privativas de interesse de cada cidadão. O 14. (EBSERH – HUCAM-UFES – Advogado – AOCP –
surgimento da Secretaria da Receita Federal representou 2014) A palavra que está acentuada corretamente é:
um significativo avanço na facilitação do cumprimento
das obrigações tributárias, contribuindo para o aumento
a) Históriar.
da arrecadação a partir (5) do final dos anos 60.
b) Memórial.
(Adaptado de <http://www.receita.fazenda.gov.br/srf/
c) Métodico.
historico.htm>. Acesso em: 17 mar. 2014.)
d) Própriedade.
e) Artifício.
a) (1).
b) (2).
c) (3). 15. (prodam-am – assistente – funcab – 2014 – adap-
d) (4). tada) Assinale a opção em que o par de palavras foi
e) (5). acentuado segundo a mesma regra.
11. (Estrada de Ferro Campos do Jordão-SP – Analista
Ferroviário – Oficinas – Elétrica – IDERH – 2014) Leia a) saúde-países
as orações a seguir: b) Etíope-juízes
Minha mãe sempre me aconselha a evitar as _____ compa- c) olímpicas-automóvel
nhias. (mas/más) d) vocês-público
A cauda do vestido da noiva tinha um _________ enorme. e) espetáculo-mensurável
(cumprimento/comprimento)
Precisamos fazer as compras do mês, pois a _________ está 16. (Advocacia Geral da União – Técnico em Contabi-
vazia. (despensa/dispensa). lidade – idecan – 2014) Os vocábulos “cinquentenário”
e “império” são acentuados devido à mesma justificativa.
Completam, correta e respectivamente, as lacunas acima O mesmo ocorre com o par de palavras apresentado em
os expostos na alternativa:
a) prêmio e órbita.
a) mas – cumprimento – despensa. b) rápida e tráfego
b) más – comprimento – despensa. c) satélite e ministério.
c) más – cumprimento – dispensa. d) pública e experiência.
d) mas – comprimento – dispensa. e) sexagenário e próximo.
e) más – comprimento – dispensa.
17. (Rioprevidência – Especialista em Previdência So-
12. (TRT-2ª REGIÃO-SP – Técnico Judiciário - Área Ad- cial – ceperj – 2014) A palavra “conteúdo” recebe acen-
ministrativa – Médio – FCC – 2014) Está redigida com tuação pela mesma razão de:
clareza e em consonância com as regras da gramática
normativa a seguinte frase: a) juízo
b) espírito
PORTUGUÊS

a) Queremos, ou não, ele será designado para dar a pa- c) jornalístico


lavra final sobre a polêmica questão, que, diga-se de d) mínimo
passagem, tem feito muitos exitarem em se pronun- e) disponíveis
ciar.

76
18. (Ministério do Meio Ambiente – icmbio – cespe – 24. (Corpo de Bombeiros Militar-pi – Curso de Forma-
2014) A mesma regra de acentuação gráfica se aplica aos ção de Soldados – uespi – 2014) “O evento promove a
vocábulos “Brasília”, “cenário” e “próprio”. saúde de modo integral.” A regra que justifica o acento
gráfico no termo destacado é a mesma que justifica o
( ) CERTO ( ) ERRADO acento em:

19. (Prefeitura de Balneário Camboriú-sc – Guarda a) “remédio”.


Municipal – fepese – 2014 – adaptada) Assinale a alter- b) “cajú”.
nativa em que todas as palavras são oxítonas. c) “rúbrica”.
d) “fráude”.
a) pé, lá, pasta e) “baú”.
b) mesa, tábua, régua
c) livro, prova, caderno 25. (TJ-BA – Técnico Judiciário – Área Administrativa
d) parabéns, até, televisão – Médio – FGV – 2015)
e) óculos, parâmetros, título Texto 3 – “A Lua Cheia entra em sua fase Crescente no
signo de Gêmeos e vai movimentar tudo o que diz respeito
à sua vida profissional e projetos de carreira. Os próximos
20. (Advocacia Geral da União – Técnico em Comuni- dias serão ótimos para dar andamento a projetos que co-
cação Social – idecan – 2014) Assinale a alternativa em meçaram há alguns dias ou semanas. Os resultados che-
que a acentuação de todas as palavras está de acordo garão rapidamente”.
com a mesma regra da palavra destacada: “Procuradorias
comprovam necessidade de rendimento satisfatório para O texto 3 mostra exemplos de emprego correto do “a”
renovação do FIES”. com acento grave indicativo da crase – “diz respeito à sua
vida profissional”. A frase abaixo em que o emprego do
a) após / pó / paletó acento grave da crase é corretamente empregado é:
b) moído / juízes / caído
c) história / cárie / tênue a) o texto do horóscopo veio escrito à lápis;
d) álibi / ínterim / político b) começaram à chorar assim que leram as previsões;
e) êxito / protótipo / ávido c) o horóscopo dizia à cada leitora o que devia fazer;
d) o leitor estava à procura de seu destino;
21. (Prefeitura de Brusque-sc – Educador Social – fe- e) o astrólogo previa o futuro passo à passo
pese – 2014) Assinale a alternativa em que só palavras
paroxítonas estão apresentadas. 26. (Prefeitura de Sertãozinho-SP – Farmacêutico –
Superior – VUNESP – 2017) O sinal indicativo de crase
a) facilitada, minha, canta, palmeiras está empregado corretamente nas duas ocorrências na
b) maná, papá, sinhá, canção alternativa:
c) cá, pé, a, exílio
d) terra, pontapé, murmúrio, aves a) Muitos indivíduos são propensos à associar, inadverti-
e) saúde, primogênito, computador, devêssemos damente, tristeza à depressão.
b) As pessoas não querem estar à mercê do sofrimento,
22. (Ministério do Desenvolvimento Agrário – Técni- por isso almejam à pílula da felicidade.
co em Agrimensura – funcab – 2014) A alternativa que c) À proporção que a tristeza se intensifica e se prolonga,
apresenta palavra acentuada por regra diferente das de- pode-se, à primeira vista, pensar em depressão.
mais é: d) À rigor, os especialistas não devem receitar remédios
às pessoas antes da realização de exames acurados.
a) dúvidas. e) Em relação à informação da OMS, conclui-se que exis-
b) muitíssimos. tem 121 milhões de pessoas à serem tratadas de de-
c) fábrica. pressão.
d) mínimo.
e) impossível. 27. (TRT – 21.ª Região-RN – Técnico Judiciário – Área
Administrativa – Médio – FCC – 2017) É difícil planejar
23. (prodam-am – Assistente de Hardware – funcab uma cidade e resistir à tentação de formular um projeto
– 2014) Assinale a alternativa em que todas as palavras de sociedade.
foram acentuadas segundo a mesma regra. O sinal indicativo de crase deverá ser mantido caso o ver-
bo sublinhado acima seja substituído por:
a) indivíduos - atraí(-las) - período
b) saíram – veículo - construído a) não acatar.
c) análise – saudável - diálogo b) driblar.
PORTUGUÊS

d) hotéis – critérios - através c) controlar.


e) econômica – após – propósitos d) superar.
e) não sucumbir.

77
28. (TRT – 21.ª Região-RN – Técnico Judiciário – Área e) caso o vocábulo minha fosse empregado imediata-
Administrativa – Médio – FCC – 2017) A frase em que mente antes de “produção”, o uso do sinal indicativo
há uso adequado do sinal indicativo de crase encontra-se de crase seria facultativo.
em:
32. (Sabesp-SP – atendente a clientes – Médio – fcc
a) A tendência de recorrer à adaptações aparece com – 2014 – adaptada) No trecho Refiro-me aos livros que
maior força na Hollywood do século 21. foram escritos e publicados, mas estão – talvez para sem-
b) É curioso constatar a rapidez com que o cinema agre- pre – à espera de serem lidos, o uso do acento de crase
gou à máxima. obedece à mesma regra seguida em:
c) A busca pela segurança leva os estúdios à apostarem
em histórias já testadas e aprovadas. a) Acostumou-se àquela situação, já que não sabia como
d) Tal máxima aplica-se perfeitamente à criação de peças evitá-la.
de teatro. b) Informou à paciente que os remédios haviam surtido
e) Há uma massa de escritores presos à contratos fixos efeito.
em alguns estúdios. c) Vou ficar irritada se você não me deixar assistir à no-
vela.
29. (Prefeitura de Marília-SP – Auxiliar de Escrita – d) Acabou se confundindo, após usar à exaustão a velha
Médio – VUNESP – 2017) Assinale a alternativa em que fórmula.
o sinal indicativo de crase está empregado corretamente. e) Comunique às minhas alunas que as provas estão cor-
rigidas.
a) A voluntária aconselhou a remetente à esquecer o
amor de infância. 33. (TRT-AL – Analista Judiciário – Superior – FCC–
b) O carteiro entregou às voluntárias do Clube de Julieta 2014) ... que acompanham as fronteiras ocidentais chi-
uma nova remessa de cartas. nesas...
O verbo que, no contexto, exige o mesmo tipo de com-
c) O médico ofereceu à um dos remetentes apoio psico-
plemento que o da frase acima está em:
lógico.
d) As integrantes do Clube levaram horas respondendo
a) A Rota da Seda nunca foi uma rota única...
à diversas cartas.
b) Esses caminhos floresceram durante os primórdios da
e) O Clube sugeriu à algumas consulentes que fizessem
Idade Média.
novas amizades.
c) ... viajavam por cordilheiras...
d) ... até cair em desuso, seis séculos atrás.
e) O maquinista empurra a manopla do acelerador.
30. (prefeitura de são Paulo-sp – técnico em saúde –
34. (CASAL-AL – Administrador De Rede – COPEVE –
laboratório – médio – vunesp – 2014) Reescrevendo-se
UFAL – 2014) Na afirmação abaixo, de Padre Vieira,
o segmento frasal – ... incitá-los a reagir e a enfrentar o
“O trigo não picou os espinhos, antes os espinhos o pica-
desconforto, ... –, de acordo com a regência e o acento
ram a ele... Cuidais que o sermão vos picou a vós” o subs-
indicativo da crase, tem-se:
tantivo “espinhos” tem, respectivamente, função sintática
de,
a) ... incitá-los à reação e ao enfrentamento do descon-
forto, ...
a) objeto direto/objeto direto.
b) ... incitá-los a reação e o enfrentamento do descon-
b) sujeito/objeto direto.
forto, ...
c) objeto direto/sujeito.
c) ... incitá-los à reação e à enfrentamento do descon-
d) objeto direto/objeto indireto.
forto, ...
e) sujeito/objeto indireto.
d) ... incitá-los à reação e o enfrentamento do descon-
forto, ...
35. (CASAL-AL – Administrador De Rede – COPEVE –
e) ... incitá-los a reação e à enfrentamento do descon-
UFAL – 2014) No texto, “Arranca o estatuário uma pedra
forto, ..
dessas montanhas, tosca, bruta, dura, informe; e, depois
que desbastou o mais grosso, toma o maço e cinzel na
31. (CONAB – Contabilidade – Superior – IADES –
mão para começar a formar um homem, primeiro mem-
2014 – adaptada) Considerando o trecho “atualizou os
bro a membro e depois feição por feição.”
dados relativos à produção de grãos no Brasil.” e confor-
me a norma-padrão, assinale a alternativa correta. VIEIRA, P. A. In Sermão do Espírito Santo. Acervo da Aca-
demia Brasileira de Letras
a) a crase foi empregada indevidamente no trecho. A oração sublinhada exerce uma função de
b) o autor poderia não ter empregado o sinal indicativo
de crase. a) causalidade.
c) se “produção” estivesse antecedida por essa, o uso do b) conclusão.
PORTUGUÊS

sinal indicativo de crase continuaria obrigatório. c) oposição.


d) se, no lugar de “relativos”, fosse empregado referen- d) concessão.
tes, o uso do sinal indicativo de crase passaria a ser e) finalidade.
facultativo.

78
36. (EBSERH – HUCAM-UFES – Advogado – Superior a) Oração subordinada adjetiva restritiva.
– AOCP – 2014) Em “Se a ‘cura’ fosse cara, apenas uma b) Oração subordinada adjetiva explicativa.
pequena fração da sociedade teria acesso a ela.”, a expres- c) Adjunto adnominal.
são em destaque funciona como: d) Oração subordinada adverbial espacial.

a) objeto direto. 41. (Advocacia-Geral da União – Técnico em Comu-


b) adjunto adnominal. nicação Social – idecan – 2014) Acerca das relações
c) complemento nominal. sintáticas que ocorrem no interior do período a seguir
d) sujeito paciente. “Policiais de Los Angeles tomam facas de criminosos, per-
e) objeto indireto. seguem bêbados na estrada e terminam o dia na delega-
cia fazendo seu relatório.”, é correto afirmar que
37. (EBSERH – HUSM-UFSM-RS – Analista Adminis-
trativo – Jornalismo – Superior – AOCP – 2014) a) “o dia” é sujeito do verbo “terminar”.
“Sinta-se ungido pela sorte de recomeçar. Quando seu fi- b) o sujeito do período, Policiais de Los Angeles, é com-
lho crescer, ele irá entender - mais cedo ou mais tarde -...” posto.
No período acima, a oração destacada: c) “bêbados” e “criminosos” apresentam-se na função de
sujeito.
a) estabelece uma relação temporal com a oração que d) “facas” possui a mesma função sintática que “bêba-
lhe é subsequente. dos” e “relatório”.
b) estabelece uma relação temporal com a oração que a e) “de criminosos”, “na estrada”, “na delegacia” são ter-
antecede. mos que indicam circunstâncias que caracterizam a
c) estabelece uma relação condicional com a oração que ação verbal.
lhe é subsequente.
d) estabelece uma relação condicional com a oração que 42. (TJ-SP – Escrevente Técnico Judiciário – Médio –
a antecede. VUNESP – 2015) Leia o texto, para responder às ques-
e) estabelece uma relação de finalidade com a oração tões.
que lhe é subsequente. O fim do direito é a paz, o meio de que se serve para
consegui-lo é a luta. Enquanto o direito estiver sujeito
às ameaças da injustiça – e isso perdurará enquanto o
mundo for mundo –, ele não poderá prescindir da luta. A
vida do direito é a luta: luta dos povos, dos governos, das
classes sociais, dos indivíduos.
38. (prodam-am – Assistente de Hardware – funcab – Todos os direitos da humanidade foram conquistados
2014) O termo destacado em: “As pessoas estão sempre pela luta; seus princípios mais importantes tiveram de
muito ATAREFADAS.” exerce a seguinte função sintática: enfrentar os ataques daqueles que a ele se opunham;
todo e qualquer direito, seja o direito de um povo, seja
a) objeto direto. o direito do indivíduo, só se afirma por uma disposição
b) objeto indireto. ininterrupta para a luta. O direito não é uma simples
c) adjunto adverbial. ideia, é uma força viva. Por isso a justiça sustenta numa
d) predicativo. das mãos a balança com que pesa o direito, enquanto na
e) adjunto adnominal. outra segura a espada por meio da qual o defende.
A espada sem a balança é a força bruta, a balança sem a
39. (trt-13ª região-pb – Técnico Judiciário – Tecnolo- espada, a impotência do direito. Uma completa a outra,
gia da Informação – Médio – fcc – 2014) Ao mesmo e o verdadeiro estado de direito só pode existir quando
tempo, as elites renunciaram às ambições passadas... a justiça sabe brandir a espada com a mesma habilidade
O verbo que, no contexto, exige o mesmo tipo de com- com que manipula a balança.
plemento que o grifado acima está empregado em: O direito é um trabalho sem tréguas, não só do Poder
Público, mas de toda a população. A vida do direito nos
a) Faltam-nos precedentes históricos para... oferece, num simples relance de olhos, o espetáculo de
b) Nossos contemporâneos vivem sem esse futuro... um esforço e de uma luta incessante, como o despendi-
c) Esse novo espectro comprova a novidade de nossa si- do na produção econômica e espiritual. Qualquer pessoa
tuação... que se veja na contingência de ter de sustentar seu direi-
d) As redes sociais eram atividades de difícil implemen- to participa dessa tarefa de âmbito nacional e contribui
tação... para a realização da ideia do direito. É verdade que nem
e) ... como se imitássemos o padrão de conforto... todos enfrentam o mesmo desafio.
A vida de milhares de indivíduos desenvolve-se tranqui-
40. (Cia de Serviços de Urbanização de Guarapuava- lamente e sem obstáculos dentro dos limites fixados pelo
-pr – Agente de Trânsito – consulplam – 2014) Quanto direito. Se lhes disséssemos que o direito é a luta, não
PORTUGUÊS

à função que desempenha na sintaxe da oração, o trecho nos compreenderiam, pois só veem nele um estado de
em destaque “Tenho uma dor que passa daqui pra lá e de paz e de ordem.
lá pra cá” corresponde a: (Rudolf von Ihering, A luta pelo direito)

79
Assinale a alternativa em que uma das vírgulas foi em- tes, há mais de meio século; porém, sabe-se, que a
pregada para sinalizar a omissão de um verbo, tal como produção cinematográfica sempre bebeu na fonte da
ocorre na passagem – A espada sem a balança é a força literatura.
bruta, a balança sem a espada, a impotência do direito. e) A literatura, sempre serviu de fonte inspiradora do ci-
nema, mas este, declarou-se independente das outras
a) O direito, no sentido objetivo, compreende os princí- artes há mais de meio século − como é sabido.
pios jurídicos manipulados pelo Estado.
b) Todavia, não pretendo entrar em minúcias, pois nunca 45. (Correios – Técnico em Segurança do Trabalho Jú-
chegaria ao fim. nior – Médio – IADES – 2017 – adaptada) Quanto às
c) Do autor exige-se que prove, até o último centavo, o regras de ortografia e de pontuação vigentes, considere
interesse pecuniário. o período “Enquanto lia a carta, as lágrimas rolavam em
d) É que, conforme já ressaltei várias vezes, a essência do seu rosto numa mistura de amor e saudade.” e assinale a
direito está na ação. alternativa correta.
e) A cabeça de Jano tem face dupla: a uns volta uma das
faces, aos demais, a outra. a) O uso da vírgula entre as orações é opcional.
b) A redação “Enquanto lia a carta, as lágrimas rolavam
43. TJ-BA – Técnico Judiciário – Área Administrativa – em seu rosto por que sentia um misto de amor e sauda-
Médio – FGV – 2015 de.” poderia substituir a original.
c) O uso do hífen seria obrigatório, caso o prefixo re fosse
Texto 2 - “A primeira missão tripulada ao espaço profundo acrescentado ao vocábulo “lia”.
desde o programa Apollo, da década 1970, com o objetivo d) Caso a ordem das orações fosse invertida, o uso da
de enviar astronautas a Marte até 2030 está sendo prepa- vírgula entre elas poderia ser dispensado.
rada pela Nasa (agência espacial norte-americana). O pri- e) Assim como o vocábulo “lágrimas”, devem ser acen-
meiro passo para a concretização desse desafio será dado tuados graficamente rúbrica, filântropo e lúcida.
nesta sexta-feira (5), com o lançamento da cápsula Orion,
da base da agência em Cabo Canaveral, na Flórida, nos 46. (TRE-MS – Estágio – Jornalismo – TRE-MS – 2014)
Estados Unidos. O lançamento estava previsto original- Verifique a pontuação nas frases abaixo e marque a as-
mente para esta quinta-feira (4), mas devido a problemas sertiva correta:
técnicos foi reagendado para as 7h05 (10h05 no horário
de Brasília).” a) Céus: Que injustiça.
b) O resultado do placar, não o abateu.
(Ciência, Internet Explorer).
c) O comércio estava fechado; porém, a farmácia estava
em pleno atendimento.
“com o lançamento da cápsula Orion, da base da agência
d) Comam bastantes frutas crianças!
em Cabo Canaveral, na Flórida, nos Estados Unidos.”
e) Comprei abacate, e mamão maduro.
Os termos sublinhados se encarregam da localização do
lançamento da cápsula referida; o critério para essa loca-
47. (SAAE-SP – Fiscal Leiturista – VUNESP – 2014)
lização também foi seguido no seguinte caso: Os protes-
tos contra as cotas raciais ocorreram:

a) em Brasília, Distrito Federal, na região Centro-Oeste;


b) em Porto Alegre, Rio Grande do Sul, região Sul;
c) em Pedrinhas, São Luís, Maranhão;
d) em São Paulo, São Paulo, Brasil;
e) em Goiânia, região Centro-Oeste, Brasil.

44. (TRT – 21.ª Região-RN – Técnico Judiciário – Área


Administrativa – Médio – FCC – 2017) Está plenamente
adequada a pontuação do seguinte período:

a) A produção cinematográfica como é sabido, sempre


bebeu na fonte da literatura, mas o cinema declarou-
-se, independente das outras artes há mais de meio
século.
b) Sabe-se que, a produção cinematográfica sempre con-
siderou a literatura como fonte de inspiração, mas o
cinema declarou-se independente das outras artes, há
mais de meio século.
c) Há mais de meio século, o cinema declarou-se inde- Segundo a norma-padrão da língua portuguesa, a pon-
PORTUGUÊS

pendente das outras artes, embora a produção cine- tuação está correta em:
matográfica tenha sempre considerado a literatura
como fonte de inspiração. a) Hagar disse, que não iria.
d) O cinema declarou-se independente, das outras ar- b) Naquela noite os Stevensens prometeram servir, bifes

80
e lagostas, aos vizinhos. 51. (Emplasa-Sp – Analista Jurídico – Direito – vunesp
c) Chegou, o convite dos Stevensens, bife e lagostas: para – 2014) Segundo a norma-padrão da língua portuguesa,
Hagar e Helga a pontuação está correta em:
d) “Eles são chatos e, nunca param de falar”, disse, Hagar
à Helga. a) Como há suspeita, por parte da família de que João
e) Helga chegou com o recado: fomos convidados, pelos Goulart tenha sido assassinado; a Comissão da Ver-
Stevensens, para jantar bifes e lagostas. dade decidiu reabrir a investigação de sua morte, em
48. (Prefeitura de Paulista-PE – Recepcionista – UPE- maio deste ano, a pedido da viúva e dos filhos.
NET – 2014) Sobre os SINAIS DE PONTUAÇÃO, observe b) Em maio deste ano, a Comissão da Verdade acatou
os itens abaixo: o pedido da família do ex-presidente João Goulart e
reabriu a investigação da morte deste, visto que, para
I. “Calma, gente”. a viúva e para os filhos, Jango pode ter sido assassi-
II. “Que mundo é este que chorar não é “normal”? nado.
III. “Sustentabilidade, paradigma de vida” c) A investigação da morte de João Goulart, foi reaberta,
IV. “Será que precisa de mais licitações? Haja licitações!” em maio deste ano pela Comissão da Verdade, para
V. “E, de repente, aquela rua se tornou um grande lago...” apuração da causa da morte do ex-presidente uma
vez que, para a família, Jango pode ter sido assassi-
Sobre eles, assinale a alternativa CORRETA. nado.
d) A Comissão da Verdade, a pedido da família de João
a) No item I, a vírgula isola um aposto. Goulart, reabriu em maio deste ano a investigação de
b) No item II, a interrogação indica uma mensagem in- sua morte, porque, a hipótese de assassinato não é
terrompida. descartada, pela viúva e filhos.
c) No item III, a vírgula isola termos que explicam o seu e) Como a viúva e os filhos do ex-presidente João Gou-
antecedente. lart, suspeitando que ele possa ter sido assassinado
d) No item IV, os dois sinais de pontuação, a interrogação pediram a reabertura da investigação de sua morte,
e a exclamação, indicam surpresa. à Comissão da Verdade, esta, atendeu o pedido em
e) No item V, as vírgulas poderiam ser substituídas, ape- maio deste ano.
nas, por um ponto e vírgula após o termo “repente”.
52. (Caixa Econômica Federal – Médico do Trabalho –
49. (Prefeitura de Paulista-PE – Recepcionista – UPE- cespe – 2014 – adaptada) A correção gramatical do tre-
NET – 2014 – adaptada) cho “Entre as bebidas alcoólicas, cervejas e vinhos são as
“Já vi gente cansada de amor, de trabalho, de política, de mais comuns em todo o mundo” seria prejudicada, caso
ideais. Jamais conheci alguém sinceramente cansado de se inserisse uma vírgula logo após a palavra “vinhos”.
dinheiro.”
(Millôr Fernandes) ( ) CERTO ( ) ERRADO

Sobre as vírgulas existentes no texto, é CORRETO afirmar 53. (Prefeitura de Arcoverde-PE – Administrador de
que: Recursos Humanos – CONPASS – 2014) Leia o texto a
seguir:
a) são facultativas. “Pagar por esse software não é um luxo, mas uma necessi-
b) isolam apostos. dade”. O uso da vírgula justifica-se porque:
c) separam elementos de mesma função sintática.
d) a terceira é facultativa. a) estabelece a relação entre uma coordenada assindéti-
e) separam orações coordenadas assindéticas. ca e uma conclusiva.
b) separar a oração coordenada “não é um luxo” da ad-
50. (Polícia Militar-SP – Oficial Administrativo – Mé- versativa “mas uma necessidade”, em que o verbo está
dio – vunesp – 2014) A reescrita da frase – Como sem- subentendido.
pre, a resposta depende de como definimos os termos da c) liga a oração principal “Pagar” à coordenada “não é um
pergunta. – está correta, quanto à pontuação, em: luxo, mas uma necessidade”.
d) indica que dois termos da mesma função estão ligados
a) A resposta como sempre, depende de, como defini- por uma conjunção aditiva.
mos os termos da pergunta. e) isola o aposto na segunda oração.
b) A resposta, como sempre, depende de como defini-
mos os termos da pergunta.
c) A resposta como, sempre, depende de como defini-
mos os termos da pergunta.
d) A resposta, como, sempre depende de como defini-
mos os termos da pergunta.
PORTUGUÊS

e) A resposta como sempre, depende de como, defini-


mos os termos da pergunta.

81
54. (TJ-SP – Escrevente Técnico Judiciário – Médio –
VUNESP – 2017)
Há quatro anos, Chris Nagele fez o que muitos executi- GABARITO
vos no setor de tecnologia já tinham feito – ele transferiu
sua equipe para um chamado escritório aberto, sem pa- 1 C
redes e divisórias.
Os funcionários, até então, trabalhavam de casa, mas ele 2 A
queria que todos estivessem juntos, para se conectarem 3 D
e colaborarem mais facilmente. Mas em pouco tempo fi-
cou claro que Nagele tinha cometido um grande erro. 4 A
Todos estavam distraídos, a produtividade caiu, e os nove 5 C
empregados estavam insatisfeitos, sem falar do próprio 6 A
chefe.
Em abril de 2015, quase três anos após a mudança para 7 C
o escritório aberto, Nagele transferiu a empresa para um 8 D
espaço de 900 m² onde hoje todos têm seu próprio es-
paço, com portas e tudo. 9 D
Inúmeras empresas adotaram o conceito de escritório 10 C
aberto – cerca de 70% dos escritórios nos Estados Uni- 11 B
dos são assim – e até onde se sabe poucos retornaram
ao modelo de espaços tradicionais com salas e portas. 12 C
Pesquisas, contudo, mostram que podemos perder até 13 D
15% da produtividade, desenvolver problemas graves
de concentração e até ter o dobro de chances de ficar 14 E
doentes em espaços de trabalho abertos – fatores que 15 A
estão contribuindo para uma reação contra esse tipo de 16 B
organização.
Desde que se mudou para o formato tradicional, Nagele 17 A
já ouviu colegas do setor de tecnologia dizerem sentir 18 CERTO
falta do estilo de trabalho do escritório fechado. “Muita
gente concorda – simplesmente não aguentam o escri- 19 D
tório aberto. Nunca se consegue terminar as coisas e é 20 C
preciso levar mais trabalho para casa”, diz ele.
21 A
É improvável que o conceito de escritório aberto caia em
desuso, mas algumas firmas estão seguindo o exemplo 22 E
de Nagele e voltando aos espaços privados. 23 E
Há uma boa razão que explica por que todos adoram um
espaço com quatro paredes e uma porta: foco. A verdade 24 E
é que não conseguimos cumprir várias tarefas ao mesmo 25 C
tempo, e pequenas distrações podem desviar nosso foco
26 C
por até 20 minutos.
Retemos mais informações quando nos sentamos em um 27 E
local fixo, afirma Sally Augustin, psicóloga ambiental e 28 D
design de interiores.
(Bryan Borzykowski, “Por que escritórios abertos po- 29 B
dem ser ruins para funcionários.” Disponível em:<w- 30 A
ww1.folha.uol.com.br>. Acesso em: 04.04.2017. Adapta-
31 E
do)
32 D
Iniciando-se a frase – Retemos mais informações quando 33 E
nos sentamos em um local fixo... (último parágrafo) – com
o termo Talvez, indicando condição, a sequência que 34 C
apresenta correlação dos verbos destacados de acordo 35 E
com a norma-padrão será:
36 C
a) reteríamos ... sentarmos 37 A
b) retínhamos ... sentássemos 38 D
c) reteremos ... sentávamos
PORTUGUÊS

d) retivemos ... sentaríamos 39 A


e) retivéssemos ... sentássemos 40 A
41 D

82
42 E ANOTAÇÕES
43 A
44 C
________________________________________________
45 D
46 C _________________________________________________
47 E _________________________________________________
48 C _________________________________________________
49 C
_________________________________________________
50 B
51 B _________________________________________________

52 CERTO _________________________________________________
53 C _________________________________________________
54 E
_________________________________________________

_________________________________________________

_________________________________________________
_________________________________________________

_________________________________________________

_________________________________________________

_________________________________________________

_________________________________________________

_________________________________________________

_________________________________________________

_________________________________________________

_________________________________________________

_________________________________________________
_________________________________________________

_________________________________________________

_________________________________________________

_________________________________________________

_________________________________________________

_________________________________________________

_________________________________________________

_________________________________________________

_________________________________________________

_________________________________________________
PORTUGUÊS

_________________________________________________
_________________________________________________

_________________________________________________

83
ANOTAÇÕES

_______________________________________________________________________________________________________

_______________________________________________________________________________________________________

_______________________________________________________________________________________________________

_______________________________________________________________________________________________________

_______________________________________________________________________________________________________

_______________________________________________________________________________________________________

_______________________________________________________________________________________________________

_______________________________________________________________________________________________________

_______________________________________________________________________________________________________

_______________________________________________________________________________________________________

_______________________________________________________________________________________________________

_______________________________________________________________________________________________________

_______________________________________________________________________________________________________

_______________________________________________________________________________________________________

_______________________________________________________________________________________________________

_______________________________________________________________________________________________________

_______________________________________________________________________________________________________

_______________________________________________________________________________________________________

_______________________________________________________________________________________________________

_______________________________________________________________________________________________________

_______________________________________________________________________________________________________

_______________________________________________________________________________________________________

_______________________________________________________________________________________________________

_______________________________________________________________________________________________________

_______________________________________________________________________________________________________

_______________________________________________________________________________________________________

_______________________________________________________________________________________________________

_______________________________________________________________________________________________________

_______________________________________________________________________________________________________

_______________________________________________________________________________________________________

_______________________________________________________________________________________________________
PORTUGUÊS

_______________________________________________________________________________________________________

_______________________________________________________________________________________________________

_______________________________________________________________________________________________________

84
ÍNDICE

MATEMÁTICA
Resolução de problemas envolvendo: Raciocínio lógico; .......................................................................................................................................01
Porcentagem e juros;.............................................................................................................................................................................................................23
Razões e proporções; ...........................................................................................................................................................................................................30
Medidas de tempo; ...............................................................................................................................................................................................................34
Equações de primeiro e segundo grau; ........................................................................................................................................................................39
Sistemas de equações; ........................................................................................................................................................................................................39
Sistema de medidas de tempo; .......................................................................................................................................................................................45
Sistema métrico decimal; ...................................................................................................................................................................................................45
Formas geométricas básicas; ............................................................................................................................................................................................46
Perímetro, área e volume de figuras geométricas; ...................................................................................................................................................46
Regra de três simples e composta. ..................................................................................................................................................................................71
RESOLUÇÃO DE PROBLEMAS ENVOLVENDO: RACIOCÍNIO LÓGICO;

Definição: Todo o conjunto de palavras ou símbolos que exprimem um pensamento de sentido completo.
Nossa professora, bela definição!
Não entendi nada!
Vamos pensar que para ser proposição a frase tem que fazer sentido, mas não só sentido no nosso dia a dia, mas
também no sentido lógico.
Para uma melhor definição dentro da lógica, para ser proposição, temos que conseguir julgar se a frase é verdadeira
ou falsa.

Exemplos:

(A) A Terra é azul.


Conseguimos falar se é verdadeiro ou falso? Então é uma proposição.

(B) >2

Como ≈1,41, então a proposição tem valor lógico falso.


Todas elas exprimem um fato.
Agora, vamos pensar em uma outra frase:
O dobro de 1 é 2?
Sim, correto?
Correto. Mas é uma proposição?
Não! Porque sentenças interrogativas, não podemos declarar se é falso ou verdadeiro.
Bruno, vá estudar.
É uma declaração imperativa, e da mesma forma, não conseguimos definir se é verdadeiro ou falso, portanto, não
é proposição.

Passei!
Ahh isso é muito bom, mas infelizmente, não podemos de qualquer forma definir se é verdadeiro ou falso, porque
é uma sentença exclamativa.
Vamos ver alguns princípios da lógica:
I. Princípio da não Contradição: uma proposição não pode ser verdadeira “e” falsa ao mesmo tempo.
II. Princípio do Terceiro Excluído: toda proposição “ou” é verdadeira “ou” é falsa, isto é, verifica-se sempre um desses
casos e nunca um terceiro caso.

1. Valor Lógico das Proposições

Definição: Chama-se valor lógico de uma proposição a verdade, se a proposição é verdadeira (V), e a falsidade, se
a proposição é falsa (F).

Exemplo
p: Thiago é nutricionista.
V(p)=V essa é a simbologia para indicar que o valor lógico de p é verdadeira, ou
V(p)=F
Basicamente, ao invés de falarmos, é verdadeiro ou falso, devemos falar tem o valor lógico verdadeiro, tem valor
lógico falso.
MATEMÁTICA

1
2. Classificação -Conjunção

Proposição simples: não contém nenhuma outra pro-


posição como parte integrante de si mesma. São geral-
mente designadas pelas letras latinas minúsculas p,q,r,s...
E depois da letra colocamos “:”
Nossa, são muitas formas de se escrever com a con-
Exemplo: junção.
p: Marcelo é engenheiro. Não precisa decorar todos, alguns são mais usuais:
q: Ricardo é estudante. “e”, “mas”, “porém”.
Exemplos
Proposição composta: combinação de duas ou mais p: Vinícius é professor.
proposições. Geralmente designadas pelas letras maiús- q: Camila é médica.
culas P, Q, R, S,...
p∧q: Vinícius é professor e Camila é médica.
Exemplo: p∧q: Vinícius é professor, mas Camila é médica.
P: Marcelo é engenheiro e Ricardo é estudante. p∧q: Vinícius é professor, porém Camila é médica.
Q: Marcelo é engenheiro ou Ricardo é estudante.
Se quisermos indicar quais proposições simples fa- - Disjunção
zem parte da proposição composta:
P(p,q)
Se pensarmos em gramática, teremos uma proposi-
ção composta quando tiver mais de um verbo e proposi- p: Vitor gosta de estudar.
ção simples, quando tiver apenas 1. Mas, lembrando que q: Vitor gosta de trabalhar.
para ser proposição, temos que conseguir definir o valor p∨q: Vitor gosta de estudar ou Vitor gosta de tra-
lógico. balhar.

3. Conectivos - Disjunção Exclusiva


Extensa: Ou...ou...
Agora que vamos entrar no assunto mais interessante Símbolo: ∨
e o que liga as proposições. p: Vitor gosta de estudar.
Antes, estávamos vendo mais a teoria, a partir dos co- q: Vitor gosta de trabalhar
nectivos vem a parte prática. p∨q Ou Vitor gosta de estudar ou Vitor gosta de tra-
balhar.
3.1. Definição
-Condicional
Palavras que se usam para formar novas proposições, Extenso: Se..., então..., É necessário que, Condição ne-
a partir de outras. cessária
Vamos pensar assim: conectivos? Conectam alguma Símbolo: →
coisa?
Sim, vão conectar as proposições, mas cada conectivo Exemplos
terá um nome, vamos ver? p→q: Se chove, então faz frio.
p→q: É suficiente que chova para que faça frio.
-Negação p→q: Chover é condição suficiente para fazer frio.
p→q: É necessário que faça frio para que chova.
p→q: Fazer frio é condição necessária para chover.

-Bicondicional
Exemplo Extenso: se, e somente se, ...
p: Lívia é estudante. Símbolo: ↔
~p: Lívia não é estudante. p: Lucas vai ao cinema.
q: Pedro é loiro. q: Danilo vai ao cinema.
¬q: É falso que Pedro é loiro. p↔q: Lucas vai ao cinema se, e somente se, Danilo vai
r: Érica lê muitos livros. ao cinema.
~r: Não é verdade que Érica lê muitos livros.
s: Cecilia é dentista. Referências
¬s: É mentira que Cecilia é dentista. ALENCAR FILHO, Edgar de – Iniciação a lógica mate-
MATEMÁTICA

mática – São Paulo: Nobel – 2002.

2
Tabela-verdade - Conjunção
Eu comprei bala e chocolate, só vou me contentar se
Com a tabela-verdade, conseguimos definir o valor eu tiver as duas coisas, certo?
lógico de proposições compostas facilmente, analisando Se eu tiver só bala não ficarei feliz, e nem se tiver só
cada coluna. chocolate.
Se tivermos uma proposição p, ela pode ter V(p)=V E muito menos se eu não tiver nenhum dos dois.
ou V(p)=F.
p p q p ∧q
V V V V
F V F F
F V F
Quando temos duas proposições, não basta colocar
só VF, será mais que duas linhas. F F F

-Disjunção
p q Vamos pensar na mesma frase anterior, mas com o
V V conectivo “ou”.
V F Eu comprei bala ou chocolate.
Eu comprei bala e também comprei a chocolate, está
F V certo pois poderia ser um dos dois ou os dois.
F F Se eu comprei só bala, ainda estou certa, da mesma
forma se eu comprei apenas chocolate.
Observe, a primeira proposição ficou VVFF Agora se eu não comprar nenhum dos dois, não dará
E a segunda intercalou VFVF certo.
Vamos raciocinar, com uma proposição temos 2 pos- p q p ∨q
sibilidades, com 2 proposições temos 4, tem que haver
um padrão para se tornar mais fácil! V V V
As possibilidades serão 2n, V F V
F V V
Onde:
n=número de proposições F F F

-Disjunção Exclusiva
p q r Na disjunção exclusiva é diferente, pois OU comprei
V V V chocolate OU comprei bala.
V F V Ou seja, um ou outro, não posso ter os dois ao mes-
mo tempo.
V V F
V F F p q p ∨q
F V V V V F
F F V V F V
F V F F V V
F F F F F F

A primeira proposição, será metade verdadeira e me- -Condicional


tade falsa. Se chove, então faz frio.
A segunda, vamos sempre intercalar VFVFVF. Se choveu e fez frio.
E a terceira VVFFVVFF. Estamos dentro da possibilidade.(V)
Agora, vamos ver a tabela verdade de cada um dos Choveu e não fez frio.
operadores lógicos? Não está dentro do que disse. (F)
Não choveu e fez frio.
-Negação Ahh tudo bem, porque pode fazer frio se não chover,
p ~p certo?(V)
Não choveu, e não fez frio.
V F Ora, se não choveu, não precisa fazer frio. (V)
MATEMÁTICA

F V

Se estamos negando uma coisa, ela terá valor lógico


oposto, faz sentido, não?

3
p q p →q Resposta: Letra C. P: A empresa alegou ter pago suas
obrigações previdenciárias.
V V V Q: apresentou os comprovantes de pagamento.
V F F R: o juiz julgou, pois, procedente a ação movida pelo
ex-empregado.
F V V
Número de linhas: 2³=8
F F V
3.(SERES-PE – AGENTE DE SEGURANÇA PENITENCI-
-Bicondicional ÁRIA – CESPE – 2017) A partir das proposições simples
Ficarei em casa, se e somente se, chover. P: “Sandra foi passear no centro comercial Bom Preço”, Q:
Estou em casa e está chovendo. “As lojas do centro comercial Bom Preço estavam reali-
A ideia era exatamente essa. (V) zando liquidação” e R: “Sandra comprou roupas nas lojas
Estou em casa, mas não está chovendo. do Bom Preço” é possível formar a proposição composta
Você não fez certo, era só pra ficar em casa se cho- S: “Se Sandra foi passear no centro comercial Bom Preço
vesse. (F) e se as lojas desse centro estavam realizando liquidação,
Eu sai e está chovendo. então Sandra comprou roupas nas lojas do Bom Preço ou
Aiaiai não era pra sair se está chovendo (F) Sandra foi passear no centro comercial Bom Preço”. Con-
Não estou em casa e não está chovendo. siderando todas as possibilidades de as proposições P, Q
Sem chuva, você pode sair, ta?(V) e R serem verdadeiras (V) ou falsas (F), é possível cons-
truir a tabela-verdade da proposição S, que está iniciada
p q p ↔q na tabela mostrada a seguir.
V V V
V F F
F V F
F F V

EXERCÍCIOS COMENTADOS

1.(EBSERH – ÁREA MÉDICA – CESPE – 2018) A respei-


to de lógica proposicional, julgue o item que se segue.
Se P, Q e R forem proposições simples e se ~R indicar
a negação da proposição R, então, independentemente
dos valores lógicos V = verdadeiro ou F = falso de P, Q e Completando a tabela, se necessário, assinale a opção
R, a proposição P→Q∨(~R) será sempre V. que mostra, na ordem em que aparecem, os valores ló-
gicos na coluna correspondente à proposição S, de cima
( )CERTO ( )ERRADO para baixo.

Resposta: Errado Se P for verdadeiro, Q falso e R fal- a) V / V / F / F / F / F / F / F.


so, a proposição é falsa. b) V / V / F / V / V / F / F / V.
c) V / V / F / V / F / F / F / V.
2. (TRT 7ª REGIÃO – CONHECIMENTOS BÁSICOS – d) V / V / V / V / V / V / V / V.
CESPE – 2017) e) V / V / V / F / V / V / V / F.

Texto CB1A5AAA – Proposição P Resposta: Letra D


A proposição S é composta por: (p∧q)→(r∨p)
A empresa alegou ter pago suas obrigações previdenciá-
P Q R p∧q r∨p S(p∧q)→(r∨p)
rias, mas não apresentou os comprovantes de pagamen-
to; o juiz julgou, pois, procedente a ação movida pelo V V V V V V
ex-empregado. V V F V V V
A quantidade mínima de linhas necessárias na tabela-
-verdade para representar todas as combinações possí- V F V F V V
veis para os valores lógicos das proposições simples que V F F F V V
compõem a proposição P do texto CB1A5AAA é igual a F V V F V V
MATEMÁTICA

a) 32. F V F F F V
b) 4. F F V F V V
c) 8.
F F F F F V
d) 16.

4
TAUTOLOGIA p∨~(p∧q)

Definição: Chama-se tautologia, toda proposição P Q p∧q ~(p∧q) p∨~(p∧q)


composta que terá a coluna inteira de valor lógico V.
Podemos ter proposições SIMPLES que são falsas e se V V V F V
a coluna da proposição composta for verdadeira é tau- V F F V V
tologia.
F V F V V
Vamos ver alguns exemplos.
F F F V V
A proposição ~(p∧p) é tautologia, pelo Princípio da
não contradição. Está lembrado? Novamente, coluna deu inteira com valor lógico ver-
Princípio da não Contradição: uma proposição não dadeiro, é tautologia.
pode ser verdadeira “e” falsa ao mesmo tempo.
Exemplo 3
Se João é estudante ou não é estudante, então Ma-
P ~p p∧~p ~(p∧∼p)
teus é professor.
V F F V
F V F V P Q ~p p∨~p p∨~p→q
V V F V V
A proposição p∨ ~p é tautológica, pelo princípio do V F F V F
Terceiro Excluído.
Princípio do Terceiro Excluído: toda proposição “ou” F V V V V
é verdadeira “ou” é falsa, isto é, verifica-se sempre um F F V V F
desses casos e nunca um terceiro caso.
Deu pelo menos uma falsa e agora?
P ~p p∨~p Não é tautologia.
V F V Referências
F V V ALENCAR FILHO, Edgar de. Iniciação a lógica mate-
mática. São Paulo: Nobel – 2002.
Esses são os exemplos mais simples, mas normalmen-
te conseguiremos resolver as questões com base na ta-
bela verdade, por isso insisto que a tabela verdade dos
operadores, têm que estar na “ponta da língua”, quase EXERCÍCIO COMENTADO
como a tabuada da matemática.
Veremos outros exemplos. 1.(INSS – ANALISTA DO SEGURO SOCIAL – CESPE –
2016) Com relação a lógica proposicional, julgue o item
Exemplo 1 subsequente.
Vamos pensar nas proposições: Considerando-se as proposições simples “Cláudio pratica
P: João é estudante. esportes” e “Cláudio tem uma alimentação balanceada”,
Q: Mateus é professor. é correto afirmar que a proposição “Cláudio pratica es-
Se João é estudante, então João é estudante ou Ma- portes ou ele não pratica esportes e não tem uma ali-
teus é professor. mentação balanceada” é uma tautologia.

Em simbologia: p→p∨q ( ) CERTO ( ) ERRADO

Resposta: Errado
P Q p∨q p→p∨q
p: Cláudio pratica esportes.
V V V V q: Cláudio tem uma alimentação balanceada.
V F V V (p∨~p)∧~q
F V V V
P ~P Q ~q p∨~P (p∨∼p)∧∼q
F F F V
V F V F V F
A coluna inteira da proposição composta deu verda- V F F V V V
deiro, então é uma tautologia.
F V V F V F
MATEMÁTICA

Exemplo 2 F V F V V V
Com as mesmas proposições anteriores:
João é estudante ou não é verdade que João é estu-
dante e Mateus é professor.

5
EQUIVALÊNCIAS LÓGICAS Exemplo
p: Coelho gosta de cenoura
Diz-se que uma proposição P(p,q,r..) é logicamente q: Coelho é herbívoro.
equivalente ou equivalente a uma proposição Q(p,r,s..) se
as tabelas-verdade dessas duas proposições são IDÊN- p→q: Se coelho gosta de cenoura, então coelho é
TICAS. herbívoro.
Para indicar que são equivalentes, usaremos a seguin- ~p∨q: Coelha não gosta de cenoura ou coelho é her-
te notação: bívoro
P(p,q,r..) ⇔ Q(p,r,s..)
A condicional ~p→~q é equivalente a disjunção
Essa parte de equivalência é um pouco mais chatinha, p∨~q
mas conforme estudamos, vou falando algumas dicas.
p q ~p ~q ~p→~q p∨~q
Regra da Dupla negação
V V F F V V
~~p⇔p V F F V V V
p ~p ~~p F V V F F F
V F V F F V V V V
F V F
Equivalências fundamentais (Propriedades Fun-
São iguais, então ~~p⇔p damentais): a equivalência lógica entre as proposições
goza das propriedades simétrica, reflexiva e transitiva.
Regra de Clavius
1 – Simetria (equivalência por simetria)
~p→p⇔p
a) p ∧ q ⇔ q ∧ p
p ~p ~p→p p q p∧q q∧p
V F V V V V V
F V F V F F F
F V F F
Regra de Absorção F F F F
p→p∧q⇔p→q
b) p ∨ q ⇔ q ∨ p
p q p∨q q∨p
p q p∧q p→p∧q p→q
V V V V
V V V V V
V F V V
V F F F F
F V V V
F V F V V
F F F F
F F F V V
c) p ∨ q ⇔ q ∨ p
Condicional
p q p∨q q ∨ p
Gostaria da sua atenção aqui, pois as condicionais são
as mais pedidas nos concursos. V V F F
V F V V
A condicional p→q e a disjunção ~p∨q, têm tabelas-
F V V V
-verdades idênticas
F F F F
p ~p q p∧q p→q ~p∨q
d) p ↔ q ⇔ q ↔ p
V F V V V V
V F F F F F p q p↔q q↔p
V V V V
MATEMÁTICA

F V V F V V
F V F F V V V F F F
F V F F
F F V V

6
Equivalências notáveis: b) p ∨ (q ∨ r) ⇔ (p ∨ q) ∨ (p ∨ r)

1 - Distribuição (equivalência pela distributiva) p q r q p


p ∨ (q p∨ (p ∨ q) ∨
∨r ∨ r) q ∨r (p ∨ r)
a) p ∧ (q ∨ r) ⇔ (p ∧ q) ∨ (p ∧ r)
V V V V V V V V
p q r q p ∧ (q p∧ p (p ∧ q) ∨ (p V V F V V V V V
∨r ∨ r) q ∧r ∧ r) V F V V V V V V
V V V V V V V V V F F F V V V V
V V F V V V F V F V V V V V V V
V F V V V F V V F V F V V V F V
V F F F F F F F F F V V V F V V
F V V V F F F F F F F F F F F F
F V F V F F F F
3 – Idempotência
F F V V F F F F
F F F F F F F F a) p ⇔ (p ∧ p)

b) p ∨ (q ∧ r) ⇔ (p ∨ q) ∧ (p ∨ r) Para ficar mais fácil o entendimento, vamos fazer duas


colunas com p
p q r q p ∨ (q p∨ p (p ∨ q) ∧
∧r ∧ r) q ∨r (p ∨ r) p p p∧p
V V V V V V V V V V V
V V F F V V V V F F F
V F V F V V V V
b) p ⇔ (p ∨ p)
V F F F V V V V
p p p ∨p
F V V V V V V V
V V V
F V F F F V F F
F F F
F F V F F F V F
F F F F F F F F 4 - Pela contraposição: de uma condicional gera-se
outra condicional equivalente à primeira, apenas inver-
2 - Associação (equivalência pela associativa) tendo-se e negando-se as proposições simples que as
compõem.
a) p ∧ (q ∧ r) ⇔ (p ∧ q) ∧ (p ∧ r) Da mesma forma que vimos na condicional mais aci-
ma, temos outros modos de definir a equivalência da
condicional que são de igual importância.
p q r q p ∧ (q ∧ r) p∧q p (p ∧ q) ∧
∧r ∧r (p ∧ r) 1º caso: (p → q) ⇔ (~q → ~p)
V V V V V V V V
V V F F F V F F p q ~p ~q p→q ~q → ~p
V F V F F F V F V V F F V V
V F F F F F F F V F F V F F
F V V V F F F F F V V F V V
F V F F F F F F F F V V V V
F F V F F F F F
2º caso: (~p → q) ⇔ (~q → p)
F F F F F F F F
p q ~p ~p → q ~q ~q → p
V V F V F V
MATEMÁTICA

V F F V V V
F V V V F V
F F V F V F

7
3º caso: (p → ~q) ⇔ (q → ~p) Proposições Associadas a uma Condicional (se, en-
tão)
p q ~q p → ~q ~p q → ~p
Chama-se proposições associadas a p → q as três pro-
V V F F F F posições condicionadas que contêm p e q:
V F V V F V
– Proposições recíprocas: p → q: q → p
F V F V V V
– Proposição contrária: p → q: ~p → ~q
F F V V V V – Proposição contrapositiva: p → q: ~q → ~p

5 - Pela bicondicional Observe a tabela verdade dessas quatro proposições:

a) (p ↔ q) ⇔ (p → q) ∧ (q → p), por definição


p q ~p ~q p→ q→ ~p → ~q →
q p ~q ~p
p q p↔q p→q q→p (p → q) ∧ (q → p) V V F F V V V V
V V V V V V V F F V F V V F
V F F F V F
F V V F V F F V
F V F V F F
F F V V V V V V
F F V V V V
Observamos ainda que a condicional p → q e a sua
b) (p ↔ q) ⇔ (~q → ~p) ∧ (~p → ~q) recíproca q → p ou a sua contrária ~p → ~q NÃO SÃO
EQUIVALENTES.
p q p↔ ~q ~p ~q → ~p → (~q → ~p) ∧
q ~p ~q (~p → ~q)
V V V F F V V V EXERCÍCIOS COMENTADOS
V F F V F F V F
1. (TRF 1ª REGIÃO – TÉCNICO JUDICIÁRIO – CESPE –
F V F F V V F F
2017) A partir da proposição P: “Quem pode mais, chora
F F V V V V V V menos.”, que corresponde a um ditado popular, julgue o
próximo item.
c) (p ↔ q) ⇔ (p ∧ q) ∨ (~p ∧ ~q) Do ponto de vista da lógica sentencial, a proposição P é
equivalente a “Se pode mais, o indivíduo chora menos”.
p q p p∧ ~p ~q ~p ∧ (p ∧ q) ∨ (~p
↔ ( ) CERTO ( ) ERRADO
q ~q ∧ ~q)
q
Resposta: Certo Uma dica é que normalmente quan-
V V V V F F F V do tem vírgula é condicional, não é regra, mas aconte-
V F F F F V F F ce quando você não acha o conectivo.
F V F F V F F F
2. (PC-PE – PERITO PAPILOSCOPISTA – CESPE –
F F V F V V V V 2016)

6 - Pela exportação-importação Texto CG1A06AAA

[(p ∧ q) → r] ⇔ [p → (q → r)] A Polícia Civil de determinado município prendeu, na


sexta-feira, um jovem de 22 anos de idade suspeito de
p q r p∧q (p ∧ q) → r q→r p → (q → r) ter cometido assassinatos em série. Ele é suspeito de cor-
tar, em três partes, o corpo de outro jovem e de enterrar
V V V V V V V as partes em um matagal, na região interiorana do mu-
V V F V F F F nicípio. Ele é suspeito também de ter cometido outros
dois esquartejamentos, já que foram encontrados vídeos
V F V F V V V
em que ele supostamente aparece executando os crimes
V F F F V V V Assinale a opção que é logicamente equivalente à propo-
F V V F V V V sição “Ele é suspeito também de ter cometido outros dois
MATEMÁTICA

esquartejamentos, já que foram encontrados vídeos em


F V F F V F V que ele supostamente aparece executando os crimes”,
F F V F V V V presente no texto CG1A06AAA.
F F F F V V V

8
a) Se foram encontrados vídeos em que ele supostamente aparece executando os dois esquartejamentos, ele é suspeito
também de ter cometido esses crimes.
b) Ele não é suspeito de outros dois esquartejamentos, já que não foram encontrados vídeos em que ele supostamente
aparece executando os crimes.
c) Se não foram encontrados vídeos em que ele supostamente aparece executando os dois esquartejamentos, ele não
é suspeito desses crimes.
d) Como ele é suspeito de ter cometido também dois esquartejamentos, foram encontrados vídeos em que ele supos-
tamente aparece executando os crimes.
e) Foram encontrados vídeos em que ele supostamente aparece executando os dois esquartejamentos, pois ele é tam-
bém suspeito de ter cometido esses crimes.

Resposta: A A expressão já que=pois


Que se for escrita com a condicional, devemos mudar as proposições de lugar.
Se foram encontrados vídeos em que ele supostamente aparece executando os dois esquartejamentos, ele é suspei-
to também de ter cometido esses crimes.

Referências
ALENCAR FILHO, Edgar de – Iniciação a lógica matemática – São Paulo: Nobel – 2002.
CABRAL, Luiz Cláudio Durão; NUNES, Mauro César de Abreu - Raciocínio lógico passo a passo – Rio de Janeiro:
Elsevier, 2013.

Negação de uma proposição composta

Definição: Quando se nega uma proposição composta primitiva, gera-se outra proposição também composta e
equivalente à negação de sua primitiva.
Ou seja, muitas vezes para os exercícios teremos que saber qual a equivalência da negação para compor uma frase,
por exemplo.

Negação de uma conjunção (Lei de Morgan)

Para negar uma conjunção, basta negar as partes e trocar o conectivo conjunção pelo conectivo disjunção.

~(p ∧ q) ⇔ (~p ∨ ~q)


p q ~p ~q p∧q ~(p ∧ q) ~p ∨ ~q
V V F F V F F
V F F V F V V
F V V F F V V
F F V V F V V

Negação de uma disjunção (Lei de Morgan)

Para negar uma disjunção, basta negar as partes e trocar o conectivo-disjunção pelo conectivo-conjunção.

~(p ∨ q) ⇔ (~p ∧ ~q)


p q ~p ~q p∨q ~(p ∨ q) ~p ∧ ~q
V V F F V F F
V F F V V F F
F V V F V F F
F F V V F V V

Resumindo as negações, quando é conjunção nega as duas e troca por “ou”


Quando for disjunção, nega tudo e troca por “e”.
MATEMÁTICA

9
Negação de uma disjunção exclusiva
~(p ∨ q) ⇔ (p ↔ q)
p q p∨q ~( p∨q) p↔q
V V F V V
V F V F F
F V V F F
F F F V V

Negação de uma condicional


Famoso MANE
Mantém a primeira e nega a segunda.
~(p → q) ⇔ (p ∧ ~q)
p q p→q ~q ~(p → q) p ∧ ~q
V V V F F F
V F F V V V
F V V F F V
F F V V F F

Negação de uma bicondicional


~(p ↔ q) = ~[(p → q) ∧ (q → p)] ⇔ [(p ∧ ~q) ∨ (q ∧ ~p)]

P Q p↔q p→q q→p p → q) ∧ (q → p)] ~[(p → q) ∧ (q → p)] p ∧ ~q q ∧ ~p [(p ∧ ~q) ∨ (q ∧ ~p)]


V V V V V V F F F F
V F F F V F V V F V
F V F V F F V F V V
F F V V V V F F F F

Dupla negação (Teoria da Involução)

a) De uma proposição simples: p ⇔ ~ (~p)

P ~P ~ (~p)
V F V
F V F

b) De uma condicional: Definição: A dupla negação de uma condicional dá-se da seguinte forma: nega-se a 1ª parte
da condicional, troca-se o conectivo-condicional pela disjunção e mantém-se a 2ª parte.
Demonstração: Seja a proposição primitiva: p → q nega-se pela 1ª vez: ~(p → q) ⇔ p ∧ ~q nega-se pela 2ª vez:
~(p ∧ ~q) ⇔ ~p ∨ q
Conclusão: Ao negarmos uma proposição primitiva duas vezes consecutivas, a proposição resultante será equiva-
lente à sua proposição primitiva. Logo, p → q ⇔ ~p ∨ q

EXERCÍCIOS COMENTADOS

1. (TRF 1ª REGIÃO – TÉCNICO JUDICIÁRIO – CESPE – 2017) A partir da proposição P: “Quem pode mais, chora
menos.”, que corresponde a um ditado popular, julgue o próximo item.
A negação da proposição P pode ser expressa por “Quem não pode mais, não chora menos”
MATEMÁTICA

( ) CERTO ( ) ERRADO
Resposta: Errado. Negação de uma condicional: mantém a primeira e nega a segunda

10
2. (PC-PE – PERITO CRIMINAL – CESPE – 2016) Con-
sidere as seguintes proposições para responder a questão.
P1: Se há investigação ou o suspeito é flagrado cometen- EXERCÍCIOS COMENTADOS
do delito, então há punição de criminosos.
P2: Se há punição de criminosos, os níveis de violência 1. (IF-BA – ADMINISTRADOR – FUNRIO – 2016) Ou
não tendem a aumentar. João é culpado ou Antônio é culpado. Se Antônio é ino-
P3: Se os níveis de violência não tendem a aumentar, a cente então Carlos é inocente. João é culpado se e so-
população não faz justiça com as próprias mãos. mente se Pedro é inocente. Ora, Pedro é inocente. Logo,

Assinale a opção que apresenta uma negação correta da a) Pedro e Antônio são inocentes e Carlos e João são cul-
proposição P1. pados.
b) Pedro e Carlos são inocentes e Antônio e João são
a) Se não há punição de criminosos, então não há investi- culpados.
gação ou o suspeito não é flagrado cometendo delito. c) Pedro e João são inocentes e Antônio e Carlos são cul-
b) Há punição de criminosos, mas não há investigação pados.
nem o suspeito é flagrado cometendo delito. d) Antônio e Carlos são inocentes e Pedro e João são
c) Há investigação ou o suspeito é flagrado cometendo culpados.
delito, mas não há punição de criminosos. e) Antônio, Carlos e Pedro são inocentes e João é culpado.
d) Se não há investigação ou o suspeito não é flagrado
cometendo delito, então não há punição de criminosos. Resposta: Letra E.
e) Se não há investigação e o suspeito não é flagrado co- Vamos começar de baixo pra cima.
metendo delito, então não há punição de criminosos.
Ou João é culpado ou Antônio é culpado.
Resposta: Letra C Famoso MANE Se Antônio é inocente então Carlos é inocente.
Mantém a primeira e nega a segunda. João é culpado se e somente se Pedro é inocente.
Há investigação ou o suspeito é flagrado cometendo
Ora, Pedro é inocente.
delito e não há punição de criminosos.
(V)
No caso, a questão ao invés de “e”utilizou mas
Sabendo que Pedro é inocente,
João é culpado se e somente se Pedro é inocente.
ARGUMENTOS
João é culpado, pois a bicondicional só é verdadeira se
ambas forem verdadeiras ou ambas falsas.
Um argumento é um conjunto finito de premissas (pro-
posições), sendo uma delas a consequência das demais.
Tal premissa (proposição), que é o resultado dedutivo ou João é culpado se e somente se Pedro é inocente
consequência lógica das demais, é chamada conclusão. Um (V) (V)
argumento é uma fórmula: P1 ∧ P2 ∧ ... ∧ Pn → Q Ora, Pedro é inocente
OBSERVAÇÃO: A fórmula argumentativa P1 ∧ P2 ∧ (V)
... ∧ Pn → Q, também poderá ser representada pela se- Sabendo que João é culpado, vamos analisar a primei-
guinte forma: ra premissa.
Ou João é culpado ou Antônio é culpado.
Então, Antônio é inocente, pois a disjunção exclusiva
só é verdadeira se apenas uma das proposições for.

Se Antônio é inocente então Carlos é inocente.


Carlos é inocente, pois sendo a primeira verdadeira, a
condicional só será verdadeira se a segunda proposi-
1. Argumentos válidos ção também for.

Um argumento é válido quando a conclusão é verda- Então, temos:


deira (V), sempre que as premissas forem todas verdadei- Pedro é inocente, João é culpado, António é inocente
ras (V). Dizemos, também, que um argumento é válido e Carlos é inocente.
quando a conclusão é uma consequência obrigatória das
verdades de suas premissas. 2. (DPU – AGENTE ADMINISTRATIVO – CESPE –
2016) Considere que as seguintes proposições sejam
Argumentos inválidos verdadeiras.
• Quando chove, Maria não vai ao cinema.
Um argumento é dito inválido (ou falácia, ou ilegítimo • Quando Cláudio fica em casa, Maria vai ao cinema.
MATEMÁTICA

ou mal construído), quando as verdades das premissas • Quando Cláudio sai de casa, não faz frio.
são insuficientes para sustentar a verdade da conclusão. • Quando Fernando está estudando, não chove.
Caso a conclusão seja falsa, decorrente das insuficiências • Durante a noite, faz frio.
geradas pelas verdades de suas premissas, tem-se como
conclusão uma contradição (F).

11
Tendo como referência as proposições apresentadas, jul- Representação de uma proposição quantificada
gue o item subsecutivo.
Se Maria foi ao cinema, então Fernando estava estudando. (∀x)(x ∈ N)(x + 3 > 15)
Quantificador: ∀
( ) CERTO ( ) ERRADO Condição de existência da variável: x ∈ N.
Predicado: x + 3 > 15.
Resposta: Errado
• Durante a noite, faz frio. (∃x)[(x + 1 = 4) ∧ (7 + x = 10)]
V Quantificador: ∃
Condição de existência da variável: não há.
• Quando Cláudio sai de casa, não faz frio. Predicado: “(x + 1 = 4) ∧ (7 + x = 10)”.
F F
Negações de proposições quantificadas ou funcionais
• Quando Cláudio fica em casa, Maria vai ao cinema. Seja uma sentença (∀x)(A(x)).
V V Negação: (∃x)(~A(x))

• Quando chove, Maria não vai ao cinema. Exemplo


F F (∀x)(2x-1=3)
Negação: (∃x)(2x-1≠3)
• Quando Fernando está estudando, não chove. Seja uma sentença (∃x)(Q(x)).
V/F V Negação: (∀x)(~Q(x)).
Portanto, Se Maria foi ao cinema, então Fernando es- (∃x)(2x-1=3)
tava estudando. Negação: (∀x)(2x-1≠3)
Não tem como ser julgado.
1. Definição das proposições
Diagramas Lógicos
Todo A é B.
As questões de Diagramas lógicos envolvem as pro-
O conjunto A está contido no conjunto B, assim todo
posições categóricas (todo, algum, nenhum), cuja solu-
elemento de A também é elemento de B.
ção requer que desenhemos figuras, os chamados dia-
gramas.
Podemos representar de duas maneiras:
Quantificadores são elementos que, quando asso-
ciados às sentenças abertas, permitem que as mesmas
sejam avaliadas como verdadeiras ou falsas, ou seja, pas-
sam a ser qualificadas como sentenças fechadas.

O quantificador universal
O quantificador universal, usado para transformar
sentenças (proposições) abertas em proposições fecha-
das, é indicado pelo símbolo “∀”, que se lê: “qualquer
que seja”, “para todo”, “para cada”.

Exemplo: Quando “todo A é B” é verdadeira, vamos ver como


(∀x)(x + 2 = 6) ficam os valores lógicos das outras?
Lê-se: “Qualquer que seja x, temos que x + 2 = 6”
(falsa). Pensemos nessa frase: Toda criança é linda.
É falso, pois não podemos colocar qualquer x para a
afirmação ser verdadeira. Nenhum A é B é necessariamente falsa.
O quantificador existencial Nenhuma criança é linda, mas eu não acabei de falar
O quantificador existencial é indicado pelo símbolo “∃” que TODA criança é linda? Por isso é falsa.
que se lê: “existe”, “existe pelo menos um” e “existe um”.
Algum A é B é necessariamente verdadeira.
Exemplos: Alguma Criança é linda, sim, se todas são 1, 2, 3...são
(∃x)(x + 5 = 9) lindas.
Lê-se: “Existe um número x, tal que x + 5 = 9” (ver-
dadeira). Algum A não é B necessariamente é falsa, pois A está
MATEMÁTICA

Nesse caso, existe um número, ahh tudo bem... claro contido em B.


que existe algum número que essa afirmação será ver- Alguma criança não é linda, bem como já vimos im-
dadeira. possível, pois todas são.
Ok? Sem maiores problemas, certo?

12
Nenhum A é B. c) Todos os elementos de B estão em A

A e B não terão elementos em comum.

d) O conjunto A é igual ao conjunto B.

Quando “nenhum A é B” é verdadeira, vamos ver


como ficam os valores lógicos das outras?
Frase: Nenhum cachorro é gato. (sim, eu sei. Frase ex-
trema, mas assim é bom para entendermos..hehe)

Todo A é B é necessariamente falsa.


Todo cachorro é gato, faz sentido? Nenhum, não é?
Algum A é B é necessariamente falsa.
Algum cachorro é gato, ainda não faz sentido.

Algum A não é B necessariamente verdadeira.


Algum cachorro não é gato. Ah, sim! Espero que to- Quando “algum A é B” é verdadeira, vamos ver como
dos não sejam, mas se já está dizendo “algum” vou con- ficam os valores lógicos das outras?
cordar. Frase: Algum copo é de vidro.
Nenhum A é B é necessariamente falsa.
Nenhum copo é de vidro.
Algum A é B.
Com frase fica mais fácil né? Porque assim, consegui-
mos ver que é falsa, pois acabei de falar que algum copo
Quer dizer que há pelo menos 1 elemento de A em
é de vidro, ou seja, tenho pelo menos 1 copo de vidro.
comum com o conjunto B
Todo A é B.
Não conseguimos determinar, podendo ser verda-
Temos 4 representações possíveis
deira ou falsa (podemos analisar também os diagramas
mostrados nas figuras a e c).
a) os dois conjuntos possuem uma parte dos elemen- Todo copo é de vidro.
tos em comum. Pode ser que sim, ou não.

Algum A não é B.
Não conseguimos determinar, podendo ser verdadei-
ra ou falsa (contradiz com as figuras b e d)
Algum copo não é de vidro.
Como não sabemos se todos os copos são de vidros,
pode ser verdadeira.

Algum A não é B.
O conjunto A tem pelo menos um elemento que não
pertence ao conjunto B.

b) Todos os elementos de A estão em B. Aqui teremos 3 modos de representar:

a) Os dois conjuntos possuem uma parte dos elemen-


tos em comum.
MATEMÁTICA

13
b) Todos os elementos de B estão em A. 2. (UFES – ASSISTENTE EM ADMINISTRAÇÃO –
UFES – 2017) Em um determinado grupo de pessoas,
• todas as pessoas que praticam futebol também prati-
cam natação,
• algumas pessoas que praticam tênis também praticam
futebol,
• algumas pessoas que praticam tênis não praticam na-
tação.

É CORRETO afirmar que no grupo

a) todas as pessoas que praticam natação também pra-


ticam tênis.
c) Não há elementos em comum entre os dois conjuntos. b) todas as pessoas que praticam futebol também prati-
cam tênis.
c) algumas pessoas que praticam natação não praticam
futebol.
d) algumas pessoas que praticam natação não praticam
tênis.
e) algumas pessoas que praticam tênis não praticam fu-
tebol.

Resposta: Letra E.
Quando “algum A não é B” é verdadeira, vamos ver
como ficam os valores lógicos das outras?
Vamos fazer a frase contrária do exemplo anterior.
Frase: Algum copo não é de vidro.

Nenhum A é B é indeterminada (contradição com as


figuras a e b).
Nenhum copo é de vidro, algum não é, mas não sei se
todos não são de vidro.

Todo A é B é necessariamente falsa.


Todo copo é de vidro, mas eu disse que algum copo
não era.
Algum A é B é indeterminada.
Algum copo é de vidro, não consigo determinar se 3. (SEPOG-RO – TÉCNICO EM TECNOLOGIA DA IN-
tem algum de vidro ou não. FORMAÇÃO E COMUNICAÇÃO – FGV – 2017) Consi-
dere a afirmação:
“Toda pessoa que faz exercícios não tem pressão alta”.
De acordo com essa afirmação é correto concluir que
EXERCÍCIOS COMENTADOS
a) se uma pessoa tem pressão alta então não faz exer-
1. (PC-RS – ESCRIVÃO – FUNDATEC – 2018) Supondo cícios.
a verdade da sentença aberta: Alguns investigados são b) se uma pessoa não faz exercícios então tem pressão
advogados mas nem todos os investigados têm domicí- alta.
lio conhecido. Podemos deduzir a verdade da alternativa: c) se uma pessoa não tem pressão alta então faz exercí-
cios.
a) Todos investigados são advogados e têm domicílio co- d) existem pessoas que fazem exercícios e que têm pres-
nhecido. são alta.
b) Todos investigados são advogados e não têm domicí- e) não existe pessoa que não tenha pressão alta e não
lio conhecido. faça exercícios.
c) Alguns investigados são advogados e têm domicílio
conhecido. Resposta: Letra A Se toda pessoa que faz exercício
d) Alguns investigados são advogados e alguns investi- não tem pressão alta, ora, se a pessoa tem pressão
gados têm domicílio conhecido. alta, então não faz exercício.
MATEMÁTICA

e) Alguns investigados são advogados e alguns investi-


gados não têm domicílio conhecido. Referências
CARVALHO, S. Raciocínio Lógico Simplificado. Série
Resposta: Letra E Nem todos os investigados têm do- Provas e Concursos, 2010.
micilio = Existem investigados que não têm domicilio.

14
Sequências Numéricas

EXERCÍCIO COMENTADO
Definição
O diário do professor é composto pelos nomes de 1. (FCC-2016 – MODIFICADO) Determine o termo ge-
seus alunos e esses nomes obedecem a uma ordem (são
escritos em ordem alfabética). Essa lista de nomes (diá- 1 3 5 7
ral an da sequência numérica , , , , … , an
rio) pode ser considerada uma sequência. Os dias do mês 2 4 6 8
são dispostos no calendário obedecendo a certa ordem
que também é um tipo de sequência. Assim, sequências Resposta:
estão presentes no nosso dia a dia com mais frequência Mediante análise dos termos da sequência, nota-se
que você pode imaginar. que termo geral é

A definição formal de sequência é todo conjunto ou


2n − 1
an =
grupo no qual os seus elementos estão escritos em uma 2n
determinada ordem ou padrão. No estudo da matemáti-
ca estudamos obviamente, as sequências numéricas. 2. (FCC-2016) A sequência numérica 1/2, 3/4, 5/6, 7/8;...é
Ao representarmos uma sequência numérica, deve- ilimitada e criada seguindo o mesmo padrão lógico. A
mos colocar seus elementos entre parênteses. Veja al- diferença entre o 500º e o 50º termos dessa sequência
guns exemplos de sequências numéricas: é igual a:

Ex: (2,4,6,8,10,12,…) - números pares positivos. a) 0,9


Ex: (1,2,3,4,5,6,7,8,9,10,11...) - números naturais. b) 9
Ex: (10,20,30,40,50...) - números múltiplos de 10. c) 0,009
Ex: (10,15,20,30) - múltiplos de 5, maiores que 5 e me- d) 0,09
nores que 35. e) 0,0009
Pelos exemplos, observou-se dois tipos básicos de
sequências: Resposta: Letra C. 2n−1
Sequência finita: Sequência numérica onde a quanti- Utilizando o termo geral dessa sequência an =
dade dos elementos é finita. 2n
, facilmente a500 e a50 são identificados. Substituin-
Sequência infinita: Sequência que seus elementos do para n=500 e n=50, chega-se ao resultado.
seguem ao infinito.
Representação Progressão Aritmética
Em uma sequencia numérica qualquer, o primeiro
termo será representado por uma letra minúscula segui- Definição
do de sua posição na sequência. Assim, o primeiro termo As progressões aritméticas, conhecidas com “PA”, são se-
é representado por , o segundo termo é , o terceiro e quências de números, que seguem um determinado padrão.
assim por diante. Este padrão caracteriza-se pelo termo seguinte da sequência
ser o termo anterior adicionado de um valor fixo, que cha-
#FicaDica maremos de constante da PA, representado pela letra “r”.
Os exemplos a seguir ilustrarão a definição acima:
Na matemática, achar uma expressão que a) S={1,2,3,4,5…} : Esta seqüência é caracterizada por
possa descrever a sequência numérica em sempre somar o valor 1 no termo seguinte, ou seja, tra-
função da posição do termo na mesma torna- ta-se de uma PA com razão . Se , classificaremos com PA
se conveniente e necessário para se usar essa crescente.
teoria. Os exemplos a seguir exemplificam esse b) S={13,11,9,7,5…} : Também podemos ter sequên-
conceito: cias onde ao invés de somar, estaremos subtraindo um
valor fixo. Neste exemplo, o termo seguinte é o termo an-
terior subtraído 2, assim, trata-se de uma PA com razão .
Ex: (1,2,3,4,…)→Essa sequência pode ser descrita Se , classificaremos com PA decrescente.
como sendo: . Ou seja, qualquer termo da sequência é c) S={4,4,4,4,4…} : Além disso, podemos ter uma se-
exatamente o valor de sua posição. quência de valores constantes, nesse caso, é como se
estivéssemos somando 0 ao termos. Assim, se , classifica-
Ex: (5,8,11,14,…)→Essa sequência pode ser descrita remos com PA constante.
como sendo: . Ou seja, qualquer termo da sequência é o
triplo da sua posição somado 2. Termo Geral
Dado esta lógica de formação das progressões arit-
Ex: (0,3,8,15,…)→ Essa sequência pode ser descrita
MATEMÁTICA

méticas, pode-se definir o que chamamos de “expressão


como sendo: . Ou seja, qualquer termo da sequência é o do termo geral”. Trata-se de uma fórmula matemática
quadrado da sua posição subtraído 1. que relaciona dois termos de uma PA com a razão r:
Essa expressão de an é definida como expressão do
termo geral da sequência. an = ap + n − p � r , com n ∈ ℕ∗

15
Onde an e ap são termos quaisquer da PA. Essa ex- Assim, com , a1 = 2, a5 = 10 e n = 5 , podemos
pressão geral pode ser utilizada de 2 formas:
calcular a soma:
a) Sabemos um termo e a razão e queremos encon-
trar outro termo. 2 + 10 � 5 12 � 5 60
Ex: O primeiro termo da PA igual a 7 e a razão é 3, qual Sn = = = = 30
2 2 2
é o quinto termo?
Temos então a1 = 7 e r = 3 e queremos achar a5.
Ou seja, não precisamos saber nem a razão da PA
Substituindo na fórmula do termo geral, temos que p = para acharmos a soma.
1 e n = 5. Assim:
Propriedades
an = ap + n − p � r
a5 = a1 + 5 − 1 � r
As progressões aritméticas possuem algumas pro-
priedades interessantes que podem ser exploradas em
a5 = 7 + 4 � 3
provas de concursos:
a5 = 19
P1: Para três termos consecutivos de uma PA, o termo
Ou seja, o quinto termo desta PA é 19. médio é a média aritmética dos outros dois termos. Essa
propriedade é fácil de verificar com o exemplo: Vamos
b) Sabemos dois termos quaisquer e queremos obter considerar três termos consecutivos de uma PA sendo
a razão da PA. an−1, an e an+1 .Podemos afirmar a partir da fórmula
Ex: O terceiro termo da PA é 2 e o sexto é -1, qual será do termo geral que:
a razão da PA?
an = an−1 + r
Temos então a3 = 2 e a6 = −1 e queremos achar r. an = an+1 – r
Substituindo na fórmula do termo geral, temos que p = Somando as duas expressões:
3 e n = 6. Assim:

an = ap + n − p � r 2an = an−1 + r + an +1 − r
a6 = a3 + 6 − 3 � r 2an = an−1 + an + 1
−1 = 2 + 3 � r
3r = −3 O que leva a:
r = −1
an−1 + an + 1
an =
Ou seja, a razão desta PA é -1.
2

Soma dos termos P2: Termos equidistantes dos Extremos. Numa se-
Outro ponto importante de uma progressão arit- quência finita, dizemos que dois termos são equidistan-
mética é a soma dos termos. Considerando uma PA tes dos extremos se a quantidade de termos que prece-
que queremos saber a soma dos 5 primeiros termos: derem o primeiro deles for igual à quantidade de termos
S={2,4,6,8,10…}. Como são poucos termos e sabemos to- que sucederem ao outro termo. Assim, na sucessão:
dos eles, podemos simplesmente somá-los: 2+4+6+8+10
= 30. Agora, considere que você saiba apenas o primei- (a1 , a2 , a3, a4 , . . . , ap , . . . , ak , . . . , an 3 , an 2 , an 1 , an ),
ro e o quinto termo, ou seja: a1 = 2 e a5 = 10 e . − − −

Como você calcularia a soma dos 5 termos?


O jeito que você aprendeu até agora seria obter os Temos:
outros termos e a razão a partir da expressão do termo
geral, porém você teria que fazer muitas contas para che-
a2 e an 1 são termos equidistantes dos extremos;
gar ao resultado, gastando tempo. Como a PA segue um −
padrão, foi possível deduzir uma expressão que dependa a3 e an 2 são termos equidistantes dos extremos;

apenas do primeiro termo e do último: a4 e an 3 são termos equidistantes dos extremos.

a1 + an � n
Nota-se que sempre que dois termos são equidis-
MATEMÁTICA

Sn =
2 tantes dos extremos, a soma dos seus índices é igual ao
valor de n + 1. Assim sendo, podemos generalizar que,
se os termos e são equidistantes dos extremos, então:
p + k = n+1

16
a15 = 10 + (15 – 1). 7
FIQUE ATENTO! a15 = 10 + 14 . 7
a15 = 10 + 98
Com as considerações anteriores, temos
que numa PA com termos, a soma de dois ter- a15 = 108
mos equidistantes dos extremos é constante e
igual a soma do primeiro termo com o último O 15° termo da progressão é 108.
termo.
2. (CONED-2016) Em uma PA com 12 termos, a soma
dos três primeiros é 12 e a soma dos dois últimos é 65. A
Ex: Sejam, numa PA de termos, ap e ak termos equi- razão dessa PA é um número:
distantes dos extremos, teremos, então: a) Múltiplo de 5
b) Primo
ap = a1 + (p – 1) � r ⇒ ap = a1 + p � r – r c) Com 3 divisores positivos
ak = a1 + (k – 1) � r ⇒ ak = a1 + k � r – r d) Igual a média geométrica entre 9 e 4
e) Igual a 4!

Somando as expressões: Resposta: Letra B.


Aplicando a fórmula do termo geral nas duas conside-
rações do enunciado, chega-se a razão igual a 3, que
ap + ak = a1 + p � r – r + a1 + k � r – r é um número primo
ap + ak = a1 + a1 + (p + k – 1 – 1) � r
Progressão Geométrica (PG)

Considerando que p + k = n + 1 , ficamos com: Definição


As progressões geométricas, conhecidas com “PG”,
ap + ak = a1 + a1 + (n + 1 – 1) � r são sequências de números, como as PA, mas seu padrão
está relacionado com a operação de multiplicação e di-
ap + ak = a1 + a1 + (n – 1) � r visão. Ou seja, o termo seguinte de uma PG é composto
ap + ak = a1 + an pelo termo anterior multiplicado por uma razão cons-
tante, que será chamada de “q”.
Os exemplos a seguir ilustrarão melhor essas definições:

a) S={2,4,8,16,32…}: Esta sequência é caracterizada por


EXERCÍCIOS COMENTADOS sempre multiplicar o termo anterior por uma razão cons-
tante, q = 2. Como os termos subseqüentes são maiores,
1. Em relação à progressão aritmética (10, 17, 24, …), de-
temos uma PG crescente (caracterizada por q>0)
termine:
1 1 1
b) S = {9,3,1, 3 , 9 , 27 … }: Esta sequência é ca-
a) o termo geral dessa PA;
b) o seu 15° termo;
racterizada por sempre multiplicar o termo anterior por
1
Resposta: uma razão constante q = , ou seja, estar sendo divi-
3
dida sempre por 3. Assim, como os termos subseqüentes
a) Para encontrar o termo geral da progressão aritmé-
tica, devemos, primeiramente, determinar a razão r: são menores, temos uma PG decrescente (caracterizada
r = a2 – a1
r = 17 – 10 por a1 > 0 e 0 < q < 1 ).
r=7
c) S={-1,-2,-4,-8 ,-16,…}: Esta sequência é caracteri-
zada por sempre multiplicar o termo anterior por uma
A razão é 7, e o primeiro termo da progressão (a1) é 10.
constante q=-2. Assim, como os termos subseqüentes
Através da fórmula do termo geral da PA, temos:
são menores, temos outro caso de PG decrescente (ca-
an = a1 + (n – 1). r
racterizada por a1 < 0 e q > 1 ).
an = 10 + (n – 1). 7
d) S={1,-4,16,-64,256…}: Esta sequência mostra al-
Portanto, o termo geral da progressão é dado por an =
ternância de sinal entre os termos. A razão neste caso é
10 + (n – 1). 7.
q=-4 e quando isto ocorre, definimos como PG alterna-
MATEMÁTICA

da. (caracterizada por q < 0 ).


b) Como já encontramos a fórmula do termo geral,
vamos utilizá-la para encontrar o 15° termo. Tendo em e) S={5,5,5,5,5,5,…}: Esta sequência possui termos
vista que n = 15, temos então: constantes e é caracterizada por ter uma razão q=1. Nes-
an = 10 + (n – 1). 7 te caso, é definido o que chamamos de PG constante.

17
Soma finita dos termos
FIQUE ATENTO! Seguindo o mesmo princípio da PA, temos na PG a so-
Atenção as definições de PG decrescen- matória dos “n” primeiros termos também. Uma fórmula
te e PG alternada, muitos alunos se confundem foi deduzida e está apresentada a seguir:
e dizem que PG decrescente ocorre quando ,
em uma analogia a PA. a1 � qn − 1
Sn =
q−1
Termo Geral
O ponto interessante desta fórmula é que ela depende
Dado esta lógica de formação das progressões geo- apenas da razão e do primeiro termo, sem a necessidade
métricas, podemos também definir a “expressão do ter- de obter o termo . Caso você tenha qualquer outro termo
mo geral”. Trata-se de uma fórmula matemática que rela- e a razão q, você obtém primeiramente o primeiro termo
ciona dois termos de uma PG com a razão q: com a fórmula do termo geral e depois obtém a soma. O
exemplo a seguir ilustra isso:
Exemplo: Calcule a soma dos quatro primeiros temos
an = ap � qn−p , n ∈ ℕ∗ , q ∈ ℝ
de uma PG, com q = 3 e a2 = 12

Onde an e ap são termos quaisquer da PG. Essa ex- Resolução: Para aplicar a fórmula da soma, é necessá-
pressão geral pode ser utilizada de 2 formas: rio obter o primeiro termo da PG. Usando o termo geral
(com n=2 e p=1):
a) Sabemos um termo e a razão e queremos encon-
trar outro termo. Exemplo: O primeiro termo da PG igual
a 5 e a razão é 2, qual é o quarto termo? an = ap � qn−p
a2 = a1 � q2−1
Resolução: Temos então a1 = 5 e q = 2 e quere-
12 = a1 � 31
mos achar a4 . Substituindo na fórmula do termo geral,
a1 = 4
temos que p = 1 e n = 4. Assim:

an = ap � qn−p Com o primeiro termo obtido, podemos encontrar a


somatória (com n=4):
a4 = a1 � q4−1
a4 = 5 � 23 a1 qn − 1
Sn =
a4 = 5 � 8 = 40 q−1

Ou seja, o quarto termo desta PG é 40. a1 q4 − 1


S4 =
q−1
b) Sabemos dois termos quaisquer e queremos obter a
razão da PG. Exemplo: O segundo termo da PG é 3 e o quar- 4 34 − 1
to é 1/3, qual será a razão da PG, sabendo que q < 0 ? S4 =
3−1
1
Resolução: Temos então a2 = 3 e a4 = 3 e que-
4 � 343 − 1
remos achar q. Substituindo na fórmula do termo geral, S4 = = 2 � 342 = 684
2
temos que p = 2 e n = 4. Assim:
Assim, a soma dos primeiros quatro termos desta PG
an = ap � qn−p é igual a 684.
a4 = a2 � q4−2
Soma da PG infinita
1
= 3 � q2
3 Além da soma dos “n” primeiros termos, as progres-
1 sões geométricas possuem uma particularidade. Para PG
q2 = com , ou seja, para PG decrescentes ou alternadas, pode-
9 mos definir a “soma da PG infinita”. Em outras palavras,
se tivermos uma PG com infinitos termos com q < 1 ,
MATEMÁTICA

podemos obter a somar todos eles e obter um valor finito.


A fórmula da PG infinita é apresentada a seguir:
a1
S∞ =
Como q < 0 , temos que a razão dessa PG é -1/3. 1−q

18
Lógica sentencial (ou proposicional)
#FicaDica As sequências lógicas aparecem com frequências nas
A fórmula é bem simples e como na fórmula da provas de concurso. São vários tipos: números, letras,
soma dos “n” primeiros termos, temos depen- figuras, baralhos, dominós e como é um assunto mui-
dência apenas do primeiro termo e da razão. to abrangente, e pode ser pedido de qualquer forma, o
que ajudará nos estudos serão as práticas de exercícios
e algumas dicas que darei. Em cada exemplo, darei al-
Exemplo: Calcule a soma infinita da seguinte PG: gumas dicas para toda vez que você visualizar esse tipo
de questão já ajude a analisar que tipo será. Vamos lá?
1 1 1 1 1
S = {1, , , , , ,…�
2 4 8 16 32 1. Sequência de Números

Resolução: Como se trata de uma PG decrescente Pode ser feita por soma, subtração, divisão, multipli-
1 cação.
com a1 = 1 e q = , ela atende aos requisitos da soma Mas lembre-se, se estamos falando de SEQUÊNCIA,
2
infinita: Substituindo na fórmula: ela vai seguir um padrão, basta você achar esse padrão,
alguns serão mais difíceis, outro beeem fácil e não se as-
a1 suste se achar rápido, não terá uma “PEGADINHA”, será
S∞ = isso e ponto.
1−q
Vamos ver alguns tipos de sequências:
1 -Progressão Aritmética
S∞ =
1 2 5 8 11
1−2

2. Progressão aritmética sempre terá a mesma ra-


1 zão.
S∞ = =2
1
2 No nosso exemplo, a razão é 3, pois para cada núme-
ro seguinte, temos que somar 3.
Ou seja, a soma dos termos desta PG infinita vale 2.
-Progressão Geométrica
9 18 36 72
EXERCÍCIO COMENTADO
3. E agora para essa nova sequência?
1. (FUNAI – CONHECIMENTOS GERAIS – Se somarmos 9, não teremos uma sequência, então
ESAF/2016) O limite da série infinita S de razão 1/3, não é soma.
1 1 é: O próximo que tentamos é a multiplicação,9x2=18
S = 9 + 3 + 1 + + + ⋯) 18x2=36
3 9
36x2=72
a) 13,444... Opa, deu certo?
b) 13,5 Progressão geométrica de razão 2.
c) 13,666...
d) 13,6 -Incremento em Progressão
e) 14 1 2 4 7

Resposta: Letra B. a1 Observe que estamos somando 1 a mais para cada


Aplicando a fórmula da PG infinita S∞ = , che- número.
1−q
ga-se na resposta. 1=1=2
2+2=4
2. Determine o valor do sexto termo da seguinte pro-
4+3=7
gressão geométrica (1, 2, 4, 8, ...).
-Série de Fibonacci
Resposta: a6 = 32
1 1 2 3 5 8 13
Nota-se que a razão da progressão geométrica é igual
a 2. Então, tem-se que: Cada termo é igual à soma dos dois anteriores.
MATEMÁTICA

an = ap � qn−p -Números Primos


a6 = a1 � q6−1 2 3 5 7 11 13 17
Naturais que possuem apenas dois divisores naturais.
a6 = 1 � 25 = 32

19
-Quadrados Perfeitos Resposta: Letra A.
1 4 9 16 25 36 49 Primeiro tentamos número de sílabas ou letras.
Números naturais cujas raízes são naturais. Letras já não deu certo.
Galo=4
Exemplo 1 Pato=4
Carneiro=8
(UFPB – ADMINISTRADOR – IDECAN/2016) Consi- Cobra=4
dere a sequência numérica a seguir: Jacaré=6
3, 6, 3, 3, 2, 5/3, 11/9. . . Não tem um padrão
Sabendo-se que essa sequência obedece uma regra de Número de sílabas
formação a partir do terceiro termo, então o denomina- Está dividido em 2 e 3 e sem padrões
dor do próximo termo da sequência é: Começadas com as letras dos meses?não...
Difícil...
a) 9. São animais, então:
b) 11. Galo e pato são aves
c) 26. Cobra e jacaré são répteis
d) 27. O carneiro é mamífero, se estão aos pares, devemos
procurar outro mamífero que no caso é o boi
Resposta: Letra D. Resposta: A.
Quando há uma sequência que não parece progres-
são aritmética ou geométrica, devemos “apelar” para 2. (IBGE - TÉCNICO EM INFORMAÇÕES GEOGRÁ-
soma os dois anteriores, soma 1, e assim por diante. FICAS E ESTATÍSTICAS – FGV/2016) Considere a se-
No caso se somarmos os dois primeiros para dar o quência infinita
terceiro: 3+6=9 IBGEGBIBGEGBIBGEG...
Para dar 3, devemos dividir por 3: 9/3=3 A 2016ª e a 2017ª letras dessa sequência são, respecti-
Vamos ver se ficará certo com o restante vamente:
6+3=9
9/3=3 a) BG;
3+2=5 b) GE;
5/3 c) EG;
Opa...parece que deu certo d) GB;
e) BI.
Então:
Resposta: Letra E.
É uma sequência com 6
Cada letra equivale a sequência
I=1
B=2
G=3
E=4
G=5
3. Sequência de Letras B=0
2016/6=336 resta 0
Sobre a sequência de Letras, fica um pouco mais difí- 2017/6=336 resta 1
cil de falar, pois podem ser de vários tipos.
Às vezes temos que substituir por números, outras Portanto, 2016 será a letra B, pois resta 0, será equiva-
analisar o padrão de como aparecem. Vamos ver uns lente a última letra
exemplos? E 2017 será a letra I, pois resta 1 e é igual a primeira
letra.
Exemplos
4. Sequência de Figuras
1. (AGERIO – ANALISTA DE DESENVOLVIMENTO –
FDC/2015) Considerando a sequência de vocábulos: Do mesmo modo que a sequência de letras, é um
galo - pato - carneiro - X - cobra – jacaré tema abrangente, pois a banca pode pedir a figura que
A alternativa lógica que substitui X é: convém.

a) boi
MATEMÁTICA

b) siri
c) sapo
d) besouro
e) gaivota

20
Exemplo Resposta: Letra C.
Os termos tem uma sequência começando por 2²+1
1. (FACEPE – Assistente em Gestão de Ciência e Tec- Portanto, para sabermos o 12º termo, fazemos
nologia – UPENET/2015) Assinale a alternativa que con- 213+1=8193
tém a próxima figura da sequência.
02. (DESENBAHIA – Técnico Escriturário - INSTITUTO
AOCP/2017) Uma máquina foi programada para distri-
buir senhas para atendimento em uma agência bancária
alternando algarismos e letras do alfabeto latino, no qual
estão inclusas as letras K, W e Y, sendo a primeira senha
(A) o número 2, a segunda a letra A, e sucessivamente na
seguinte forma: (2; A; 5; B; 8; C; ...). Com base nas infor-
mações mencionadas, é correto afirmar que a 51ª e a 52ª
senhas, respectivamente, são:

a) 69 e Z.
(B) b) 90 e Y.
c) T e 88.
d) 77 e Z.
e) Y e 100.
(C) Resposta: Letra D
A 51ª senha segue a sequência ímpar que são: (2, 5, 8,...)
51/2=25 e somamos 1, para saber qual posição ocu-
pará na sequência. Portanto será a 26
A26=a1+25r
A26=2+25⋅3
(D) A26=2+75=77

A 52ª senha ocupará a posição 26 também, mas na


sequência par, ou seja, a 26ª letra do alfabeto que é
a letra Z.
(E) 03. (TJ/RS - TÉCNICO JUDICIÁRIO –
FAURGS/2017) Na figura abaixo, encontram-se repre-
sentadas três etapas da construção de uma sequência
elaborada a partir de um triângulo equilátero.

Resposta: Letra B.
Primeiro risco vai na parte de baixo, depois do lado
E depois 2 riscos e assim por diante.
Então nossa figura terá que ter 3 riscos, mas a B ou D?
É a B, pois o risco de cima, tem que ser o maior de
todos.
Na etapa 1, marcam-se os pontos médios dos lados do
triângulo equilátero e retira-se o triângulo com vértices
nesses pontos médios, obtendo-se os triângulos pre-
EXERCÍCIO COMENTADO
tos. Na etapa 2, marcam-se os pontos médios dos lados
dos triângulos pretos obtidos na etapa 1 e retiram-se os
01. ( TRE/RJ - TÉCNICO JUDICIÁRIO - OPERAÇÃO triângulos com vértices nesses pontos médios, obtendo-
DE COMPUTADORES – CONSULPLAN/2017) Os ter- -se um novo conjunto de triângulos pretos. A etapa 3 e
mos de uma determinada sequência foram sucessiva- as seguintes mantêm esse padrão de construção.
mente obtidos seguindo um determinado padrão: Mantido o padrão de construção acima descrito, o nú-
(5, 9, 17, 33, 65, 129...) mero de triângulos pretos existentes na etapa 7 é
O décimo segundo termo da sequência anterior é um
número a) 729.
MATEMÁTICA

b) 1.024.
a) menor que 8.000. c) 2.187.
b) maior que 10.000. d) 4.096.
c) compreendido entre 8.100 e 9.000. e) 6.561.
d) compreendido entre 9.000 e 10.000.

21
Resposta: Letra C 07. (IF/PE – Técnico em Eletrotécnica – IFPE/0217) Consi-
dere a seguinte sequência de figuras formadas por círculos:
É uma PG de razão 3 e o a1 também é 3.

Continuando a sequência de maneira a manter o mesmo


04. (SESAU/RO – ENFERMEIRO – FUNRIO/2017) Ob- padrão geométrico, o número de círculos da Figura 18 é:
serve a sequência: 43, 46, 50, 55, 61, ...
O próximo termo é o: a) 334.
b) 314.
a) 65. c) 342.
b) 66. d) 324.
c) 67. e) 316.
d) 68.
e) 69. Resposta: Letra D.
Figura 1:1
Resposta: Letra D. Figura 2:4
Observe que de 43 para 46 são 3 Figura 3:9
50-46=4 Figura 4:16
55-50=5 O número de círculos é o quadrado da posição
61-55=6 Figura 18: 18²=324
Portanto, o próximo será somando 7
08. (CODEBA – GUARDA PORTUÁRIO – FGV/2016)
61+7=68
Para passar o tempo, um candidato do concurso escre-
veu a sigla CODEBA por sucessivas vezes, uma após a
05. (TRT 24ª REGIÃO – ANALISTA JUDICIÁRIO –
outra, formando a sequência:
FCC/2017) Na sequência 1A3E; 5I7O; 9U11A; 13E15I;
C O D E B A C O D E B A C O D E B A C O D ...
17O19U; 21A23E; . . ., o 12° termo é formado por algaris-
A 500ª letra que esse candidato escreveu foi:
mos e pelas letras
a) O
a) EI. b) D
b) UA. c) E
c) OA. d) B
d) IO. e) A
e) AE.
Resposta: Letra A.
Resposta: Letra D. É uma sequência com 6 letras:
A partir do 5º termo começa a repetir as letras, por- 500/6=83 e resta 2
tanto: C=1
12/5=2 e resta 2 O=2
Assim, será igual ao segundo termo, IO. D=3
E=4
06. (EBSERH – ASSISTENTE ADMINISTRATIVO – B=5
IBFC/2017) Considerando a sequência de figuras @, % A=0
, &, # , @, %, &, #,..., podemos dizer que a figura que Como restaram 2, então será igual a O.
estará na 117ª posição será:
09. (MPE/SP – OFICIAL DE PROMOTORIA I – VU-
a) @ NESP/2016) A sequência ((3, 5); (3, 3, 3); (5; 5); (3, 3,
b) % 5); ...) tem como termos sequências contendo apenas
c) & os números 3 ou 5. Dentro da lógica de formação da
d) # sequência, cada termo, que também é uma sequência,
e) $ deve ter o menor número de elementos possível. Dessa
forma, o número de elementos contidos no décimo oi-
Resposta: Letra A. tavo termo é igual a:
117/4=29 e resta 1
MATEMÁTICA

Portanto, é igual a figura 1 @ a) 5.


b) 4.
c) 6.
d) 7.
e) 8.

22
Resposta: Letra A. A porcentagem nada mais é do que uma razão, que
Vamos somar os números: representa uma “parte” e um “todo” a qual referimos como
3+5=8 100%. Assim, de uma maneira geral, temos que:
3+3+3=9 𝑝
5+5=10 𝐴= .𝑉
3+3+5=11 100

Observe que Onde A, é a parte, p é o valor da porcentagem e V é


os termos formam uma PA de razão 1. o todo (100%). Assim, os problemas básicos de porcenta-
a18=? gem se resumem a três tipos:
a18=a1+17r
a18=8+17
a18=25 Cálculo da parte (Conheço p e V e quero achar A):
Para dar 25, com o menor número de elementos possí- Para calcularmos uma porcentagem de um valor V, basta
veis, devemos ter (5,5,5,5,5)
multiplicarmos a fração correspondente, ou seja, 𝑝 por
100
10. (CODAR – RECEPCIONISTA – EXATUS/2016) A se- V. Assim:
quência numérica (99; 103; 96; 100; 93; 97; ...) possui deter-
minada lógica em sua formação. O número corresponden- 𝑝
te ao décimo elemento dessa sequência é: P% de V =A= 100 .V

a) 91
b) 88. Ex. 23% de 240 = 23 .240 = 55,2
c) 87 100
d) 84
Ex. Em uma pesquisa de mercado, constatou-se que
Resposta: Letra A. 67% de uma amostra assistem a certo programa de TV.
A princípio, queremos ver a sequência com os termos Se a população é de 56.000 habitantes, quantas pessoas
seguidos mesmo, o que seria: assistem ao tal programa?
99+4=103
103-7=96 Aqui, queremos saber a “parte” da população que as-
96+4=100 siste ao programa de TV, como temos a porcentagem e o
100-7=93 total, basta realizarmos a multiplicação:
Alternando essa sequência, mas se conseguirmos vi-
sualizar uma outra maneira, ficará mais fácil.
67
Observe que os termos ímpares (a1,a3,a5...)formam 67% de 56000=A= 56000=37520
uma PA de razão r=-3 100
Os termos pares (a2,a4,a6..) formam uma PA de razão
também r=-3 Resp. 37 520 pessoas.
Como a10 é par, devemos tomar como base a sequên-
cia par, mas para isso, vamos lembrar que se estamos
tratando apenas dela, a10=a5 Cálculo da porcentagem (conheço A e V e quero
Pois, devemos transformar o a2 em a1 e assim por dian- achar p): Utilizaremos a mesma relação para achar o valor
te. de p e apenas precisamos rearranjar a mesma:
A5=a1+4r
A5=103-12 𝑝 𝐴
𝐴= . 𝑉 → 𝑝 = . 100
A5=31 100 𝑉

PORCENTAGEM E JUROS; Ex. Um time de basquete venceu 10 de seus 16 jogos.


Qual foi sua porcentagem de vitórias?
Neste caso, o exercício quer saber qual a porcentagem
A definição de porcentagem passa pelo seu próprio de vitórias que esse time obteve, assim:
nome, pois é uma fração de denominador centesimal, ou
seja, é uma fração de denominador 100. Representamos
porcentagem pelo% e lê-se: “por cento”. 𝐴 10
MATEMÁTICA

𝑝= . 100 = . 100 = 62,5%


𝑉 16
50
Deste modo, a fração ou qualquer uma equivalen-
100
te a ela é uma porcentagem que podemos representar por Resp: O time venceu 62,5% de seus jogos.
50%.

23
Ex. Em uma prova de concurso, o candidato acertou Fatorando:
48 de 80 questões. Se para ser aprovado é necessário
acertar 55% das questões, o candidato foi ou não foi
p
aprovado? VA = ( 1 + ) .V
100
Para sabermos se o candidato passou, é necessário
calcular sua porcentagem de acertos: p ) será definido como fator de au-
Em que (1 +
𝐴 48 100
𝑝= . 100 = . 100 = 60% > 55% mento, que pode estar representado tanto na forma de
𝑉 80
fração ou decimal.
Logo, o candidato foi aprovado.

Desconto Percentual: Consideremos um valor inicial


Calculo do todo (conheço p e A e quero achar V): V que deve sofrer um desconto de p% de seu valor. Cha-
No terceiro caso, temos interesse em achar o total (Nosso memos de VD o valor após o desconto.
100%) e para isso basta rearranjar a equação novamente:

p
VD = V – .V
𝑝 𝐴 𝐴 100
𝐴= . 𝑉 → 𝑝 = . 100 → 𝑉 = . 100
100 𝑉 𝑝
Fatorando:
Ex. Um atirador tem taxa de acerto de 75% de seus
tiros ao alvo. Se em um treinamento ele acertou 15 tiros,
quantos tiros ele deu no total? p
VD = (1 – ) .V
100
Neste caso, o problema gostaria de saber quanto vale
o “todo”, assim: p
Em que (1 – ) será definido como fator de descon-
100
𝐴 15 to, que pode estar representado tanto na forma de fração
𝑉= . 100 = . 100 = 0,2.100 = 20 𝑡𝑖𝑟𝑜𝑠
𝑝 75 ou decimal.

Forma Decimal: Outra forma de representação de


porcentagens é através de números decimais, pois to- Ex. Uma empresa admite um funcionário no mês de
dos eles pertencem à mesma classe de números, que são janeiro sabendo que, já em março, ele terá 40% de au-
os números racionais. Assim, para cada porcentagem, há mento. Se a empresa deseja que o salário desse funcio-
um numero decimal equivalente. Por exemplo, 35% na nário, a partir de março, seja R$ 3 500,00, com que salário
forma decimal seriam representados por 0,35. A conver- deve admiti-lo?
são é muito simples: basta fazer a divisão por 100 que
está representada na forma de fração: Neste caso, o problema deu o valor de e gostaria de
saber o valor de V, assim:
75
75% = = 0,75 p
100
VA = ( 1 + ).V
100

Aumento e desconto percentual 40


3500 = ( 1 + ).V
100

Outra classe de problemas bem comuns sobre por-


centagem está relacionada ao aumento e a redução per- 3500 =(1+0,4).V
centual de um determinado valor. Usaremos as defini-
ções apresentadas anteriormente para mostrar a teoria
envolvida 3500 =1,4.V

3500
Aumento Percentual: Consideremos um valor inicial V= =2500
1,4
V que deve sofrer um aumento de de seu valor. Chame-
MATEMÁTICA

mos de VA o valor após o aumento. Assim:


Resp. R$ 2 500,00
p
VA = V + .V
100

24
Ex. Uma loja entra em liquidação e pretende abaixar Assim, para cada aumento, temos um fator corres-
em 20% o valor de seus produtos. Se o preço de um deles pondente e basta ir multiplicando os fatores para chegar
é de R$ 250,00, qual será seu preço na liquidação? ao resultado final.
Aqui, basta calcular o valor de VD : No caso de desconto, temos o mesmo caso, sendo V
um valor inicial, vamos considerar que ele irá sofrer dois
descontos sucessivos de p1% e p2%.
p
VD = (1 – ) .V
100
Sendo V1 o valor após o primeiro desconto, temos:
20
VD = (1 – ) .250,00
100
𝑝1
V1 = V.(1 – )
VD = (1 –0,2) .250,00 100
VD = (0,8) .250,00
Sendo V2 o valor após o segundo desconto, ou seja,
VD = 200,00 após já ter descontado uma vez, temos que:
Resp. R$ 200,00
𝑝2
V2 = V_1 .(1 – )
100
FIQUE ATENTO!
Como temos também uma expressão para , basta
Em alguns problemas de porcentagem são substituir:
necessários cálculos sucessivos de aumentos
ou descontos percentuais. Nesses casos é ne-
cessário ter atenção ao problema, pois erros 𝑝1 𝑝2
V2 = V .(1 – ) .(1 – )
costumeiros ocorrem quando se calcula a por- 100 100
centagens do valor inicial para obter todos os
valores finais com descontos ou aumentos. Na
verdade, esse cálculo só pode ser feito quando Além disso, essa formulação também funciona para
o problema diz que TODOS os descontos ou aumentos e descontos em sequência, bastando apenas
aumentos são dados a uma porcentagem do a identificação dos seus fatores multiplicativos. Sendo V
valor inicial. Mas em geral, os cálculos são fei- um valor inicial, vamos considerar que ele irá sofrer um
tos como mostrado no texto a seguir. aumento de p1% e, sucessivamente, um desconto de p2%.

Sendo V1 o valor após o aumento, temos:


Aumentos e Descontos Sucessivos: Consideremos
um valor inicial V, e vamos considerar que ele irá sofrer
dois aumentos sucessivos de p1% e p2%. Sendo V1 o valor 𝑝1
V1 = V .(1+ )
após o primeiro aumento, temos: 100

𝑝1 Sendo V2 o valor após o desconto, temos que:


V1 = V .(1 + )
100
𝑝2
V2 = V_1 .(1 – )
Sendo V2 o valor após o segundo aumento, ou seja, 100
após já ter aumentado uma vez, temos que:
Como temos uma expressão para , basta substituir:
𝑝
V2 = V1 .(1 + 2 )
100 𝑝1 𝑝2
V2 = V .(1+ ) .(1 – )
100 100
Como temos também uma expressão para V1, basta
substituir:
Ex. Um produto sofreu um aumento de 20% e depois
sofreu uma redução de 20%. Isso significa que ele voltará
MATEMÁTICA

𝑝1 𝑝2 ao seu valor original.


V2 = V .(1 + ) .(1 + )
100 100
( ) Certo ( ) Errado

25
Este problema clássico tem como finalidade concei- Juros Simples
tuar esta parte de aumento e redução percentual e evitar
o erro do leitor ao achar que aumentando p% e dimi- Toda vez que falamos em juros estamos nos referin-
nuindo p%, volta-se ao valor original. Se usarmos o que do a uma quantia em dinheiro que deve ser paga por
aprendemos, temos que: um devedor, pela utilização de dinheiro de um credor
(aquele que empresta).
𝑝1 𝑝2
V2 = V . 1+ . 1– 1. Nomenclatura
100 100
𝐴𝑢𝑚𝑒𝑛𝑡𝑜 𝑟𝑒𝑑𝑢çã𝑜
a) Os juros são representados pela letra J.
b) O dinheiro que se deposita ou se empresta chama-
20 20 mos de capital e é representado pela letra C.
V2 = V .(1+ ) .(1 – )
100 100 c) O tempo de depósito ou de empréstimo é repre-
sentado pela letra t.
d) A taxa de juros é a razão centesimal que incide so-
V2 = V .(1+0,2) .(1 – 0,2 )
bre um capital durante certo tempo. É representado pela
letra i e utilizada para calcular juros.
V2 = V .(1,2) .(0,8) Chamamos de simples os juros que são somados ao
capital inicial no final da aplicação.
96
V2 = 0,96.V= V=96% de V
100
FIQUE ATENTO!
Devemos sempre relacionar taxa e tempo
Ou seja, o valor final corresponde a 96% de V e não
numa mesma unidade:
100%, assim, eles não são iguais, portanto deve-se assi-
Taxa anual --------------------- tempo em anos
nalar a opção ERRADO
Taxa mensal-------------------- tempo em meses
Taxa diária---------------------- tempo em dias

EXERCÍCIOS COMENTADOS
Exemplo: Uma pessoa empresta a outra, a juros sim-
ples, a quantia de R$ 3000,00, pelo prazo de 4 meses, à
1. (UNESP) Suponhamos que, para uma dada eleição, taxa de 2% ao mês. Quanto deverá ser pago de juros?
uma cidade tivesse 18.500 eleitores inscritos. Suponha-
mos ainda que, para essa eleição, no caso de se verificar Resolução:
um índice de abstenções de 6% entre os homens e de 9%
entre as mulheres, o número de votantes do sexo mas- - Capital aplicado (C): R$ 3.000,00
culino será exatamente igual ao número de votantes do - Tempo de aplicação (t): 4 meses
sexo feminino. Determine o número de eleitores de cada - Taxa (i): 2% ou 0,02 a.m. (= ao mês)
sexo.
Fazendo o cálculo, mês a mês:
Resposta: Denotamos o número de eleitores do sexo
No final do 1º período (1 mês), os juros serão: 0,02 R$
femininos de F e de votantes masculinos de M. Pelo
3.000,00 = R$ 60,00
enunciado do exercícios, F+M = 18500. Além disso, o
No final do 2º período (2 meses), os juros serão: R$
índice de abstenções entre os homens foi de 6% e
60,00 + R$ 60,00 = R$ 120,00
de 9% entre as mulheres, ou seja, 94% dos homens
No final do 3º período (3 meses), os juros serão: R$
e 91% das mulheres compareceram a votação, onde
120,00 + R$ 60,00 = R$ 180,00
94%M = 91%F ou 0,94M = 0,91F. Assim, para deter-
No final do 4º período (4 meses), os juros serão: R$
minar o número de eleitores de cada sexo temos os
180,00 + R$ 60,00 = R$ 240,00
seguinte sistema para resolver:
Para evitar essa sequência de cálculos toda vez que
F + M = 18500 vamos calcular os juros simples, existe uma fórmula que

0,94M = 0,91F quantifica o total de juros simples do período, e ela está
apresentada abaixo:
0,91
Da segunda equação, temos que M = F . Agora,
0,94 J=C ∙ i ∙ t
MATEMÁTICA

substituindo M na primeira equação do sistema en-

contra-se F = 9400 e por fim determina-se M = 9100.

26
Além disso, quando quisermos saber o total que será pago de um empréstimo, ou o quanto se resgatará do inves-
timento, o qual definimos como Montante (M), basta somar o capital com os juros, usando o conceito fundamental da
matemática financeira:

M=C+J

Ou

M=C(1+i . t)

EXERCÍCIOS COMENTADOS
1.(ENEM 2015) Um casal realiza um financiamento imobiliário de R$ 180 000,00, a ser pago em 360 prestações men-
sais, com taxa de juros efetiva de 1% ao mês. A primeira prestação é paga um mês após a liberação dos recursos e o
valor da prestação mensal é de R$ 500,00 mais juro de 1% sobre o saldo devedor (valor devido antes do pagamento).
Observe que, a cada pagamento, o saldo devedor se reduz em R$ 500,00 e considere que não há prestação em atraso.

Efetuando o pagamento dessa forma, o valor, em reais, a ser pago ao banco na décima prestação é de

a) 2 075,00.
b) 2 093,00.
c) 2 138,00.
d) 2 255,00.
e) 2 300,00.

Resposta: Letra d. Temos que, na décima prestação, o valor devido é de 175 500. Calculando os juros, temos 1% de
175500 = 1755. Logo, na décima prestação o valor será de 1755 + 500 = 2255.

2.(NUCEPE 2009) Um investidor possui R$ 80.000,00. Ele aplica 30% desse dinheiro em um investimento que rende
juros simples a uma taxa de 3% a.m., durante 2 meses, e aplica o restante em investimento que rende 2% a.m., durante
2 meses também. Ao fim desse período, esse investidor possui:
A) R$ 83.680,00
B) R$ 84.000,00
C) R$ 84.320,00
D) R$ 84.400,00
E) R$ 88.000,00

Resposta: Letra a. Temos neste problema um capital sendo investido em duas etapas. Vamos realizar os cálculos
separadamente:

1º investimento
30% de R$ 80.000,00 = R$ 24.000,00 valor a ser investido a uma taxa i = 3% a.m., durante um período t = 2 meses.
Lembrando que i = 3% = 0,03.
Cálculo dos juros J, onde J=C∙i∙t:
J = 24000∙ (0,03) ∙2 = 1440.
Juros do 1º investimento = R$ 1440,00.

2º investimento
R$ 80.000,00 – R$ 24.000,00 = R$ 56.000,00 valor a ser investido a uma taxa i = 2% a.m., durante um período t = 2
meses.
J = 56000∙ (0,02) ∙ 2 = 2240.
Juros do 2º investimento = R$ 2.240,00.
Portanto, o montante final será de
R$ 80.00,00 + R$ 1.440,00 + R$ 2.240,00 = R$ 83.680,00.
MATEMÁTICA

27
Juros Compostos

O capital inicial (principal) pode crescer como já sabemos, devido aos juros. Basicamente, há duas modalidades de
como se calcular os juros:
Juros simples - ao longo do tempo, somente o principal rende juros.
Juros compostos - após cada período, os juros são incorporados ao principal e passam, por sua vez, a render juros.
Também conhecido como “juros sobre juros”.

Vamos ilustrar a diferença entre os crescimentos de um capital através juros simples e juros compostos, com um
exemplo: Suponha que $100,00 são empregados a uma taxa de 10% a.a. (ao ano) Teremos:

Capital = 100 Juros Simples Juros Compostos


N° de Anos Montante Simples Montante Composto
1 100 + 0,1 ∙ 100 = 110 100,00 + 0,1 ∙ (100,00) = 110,00
2 110 + 0,1 ∙ 100 = 120 110,00 + 0,1 ∙ (110,00) = 121,00
3 120 + 0,1 ∙ 100 = 130 121,00 + 0,1 ∙ (121,00) = 133,10
4 130 + 0,1 ∙ 100 = 140 133,10 + 0,1 ∙ (133,10) = 146,41
5 140 + 0,1 ∙ 100 = 150 146,41 + 0,1 ∙ (146,41) = 161,05

Observe que o crescimento do principal segundo juros simples é LINEAR enquanto que o crescimento segundo
juros compostos é EXPONENCIAL, e, portanto tem um crescimento muito mais “rápido”. Isto poderia ser ilustrado gra-
ficamente da seguinte forma:

Na prática, as empresas, órgãos governamentais e investidores particulares costumam reinvestir as quantias geradas
pelas aplicações financeiras, o que justifica o emprego mais comum de juros compostos na Economia. Na verdade, o
uso de juros simples não se justifica em estudos econômicos.

Fórmula para o cálculo de Juros compostos

Considere o capital inicial (principal P) $1000,00 aplicado a uma taxa mensal de juros compostos (i) de 10% (i = 10%
a.m.). Vamos calcular os montantes (capital + juros), mês a mês:
Após o 1º mês, teremos: M1 = 1000 ∙ 1,1 = 1100 = 1000(1 + 0,1)
Após o 2º mês, teremos: M2 = 1100 ∙1,1 = 1210 = 1000(1 + 0,1)2
Após o 3º mês, teremos: M3 = 1210 ∙ 1,1 = 1331 = 1000(1 + 0,1)3
.....................................................................................................
Após o nº (enésimo) mês, sendo S o montante, teremos evidentemente: M = 1000(1 + 0,1)n
De uma forma genérica, teremos para um capital C, aplicado a uma taxa de juros compostos i durante o período n :
MATEMÁTICA

M = C(1 + i)n

Onde M = montante, C = Capital, i = taxa de juros e n = número de períodos que o principal C foi aplicado.

28
#FicaDica
Na fórmula acima, as unidades de tempo referentes à taxa de juros (i) e do período (n), tem de ser neces-
sariamente iguais. Este é um detalhe importantíssimo, que não pode ser esquecido! Assim, por exemplo,
se a taxa for 2% ao mês e o período 3 anos, deveremos considerar 2% ao mês durante 3 ∙ 12=36 meses.
Exemplo: Calcule o montante de uma aplicação financeira de R$ 2000,00 aplicada a juros compostos de
2% ao mês durante 2 meses:

Resolução:

M = C∙(1 + i)n→M = 2000∙(1 +0,02)2→M = 2000∙(1,02)2=R$ 2080,80

Com aplicação da fórmula, obtém-se o montante. Agora, se quisermos os juros? Como se calcula os juros desta
aplicação sendo que agora não temos uma fórmula para J como nos juros simples? Para resolver isso, basta relembrar
o conceito fundamental:

M=C+J→J=M-C

Como calculamos o montante e temos o capital:

J=M-C→2080,80-2000,00=R$ 80,80

Esse exemplo é a aplicação básica de juros compostos.

FIQUE ATENTO!
Alguns concursos podem complicar um pouco as questões, deixando como incógnita o período da
operação “n”.

Exemplo: Em quanto tempo devo deixar R$ 3000,00 em uma aplicação para que renda um montante de R$ 3376,53
a uma taxa de 3% ao mês.

Resolução: Neste caso, precisamos saber n, vamos isolá-lo na fórmula do montante:

n M n M n
M=C 1+i → = 1+i → log = log 1 + i
C C

M
log = n � log 1 + i →
C

A fórmula envolve logaritmos e você tem dois caminhos: Memorize ou sempre lembre da dedução a partir da fór-
mula do montante. Substituindo os valores:
MATEMÁTICA

29
EXERCÍCIOS COMENTADOS

1. (NOVA 2017) Calcule o montante de um empréstimo a juros compostos de R$ 10000,00 a uma taxa de 0,5% a.m
durante 6 meses. Dado: 1,0056 = 1,0304

a) R$ 10303,77
b) R$ 10090,90
c) R$ 13030,77
d) R$ 13250,80
e) NDA

Resposta: Letra a. M = C(1 + i)^n→M = 10000∙(1 +0,005)^6→M = 10000∙(1,005)^6=R$ 10303,77

RAZÕES E PROPORÇÕES;

Razão

Quando se utiliza a matemática na resolução de problemas, os números precisam ser relacionados para se obter
uma resposta. Uma das maneiras de se relacionar os números é através da razão. Sejam dois números reais a e b, com
𝑎
b ≠ 0,define-se razão entre a e b (nessa ordem) o quociente a ÷ b, ou .
𝑏

A razão basicamente é uma fração, e como sabem, frações são números racionais. Entretanto, a leitura deste número
é diferente, justamente para diferenciarmos quando estamos falando de fração ou de razão.

3
a) Quando temos o número 5 e estamos tratando de fração, lê-se: “três quintos”.

3
b) Quando temos o número 5
e estamos tratando de razão, lê-se: “3 para 5”.

Além disso, a nomenclatura dos termos também é diferente:

O número 3 é numerador

3
a) Na fração 5

O número 5 é denominador

O número 3 é antecedente

3
b) Na razão
5

O número 5 é consequente
MATEMÁTICA

30
20 2
Ex. A razão entre 20 e 50 é = 5 já a razão entre 50
50 5
50 Razão entre grandezas de espécies diferentes: É
e 20 é = . Ou seja, deve-se sempre indicar o antece- possível também relacionar espécies diferentes e isto
20 2
dente e o consequente para sabermos qual a ordem de está normalmente relacionado a unidades utilizadas na
montarmos a razão. física:

Ex.Numa classe de 36 alunos há 15 rapazes e 21 mo- Ex. Considere um carro que às 9 horas passa pelo qui-
ças. A razão entre o número de rapazes e o número de lômetro 30 de uma estrada e, às 11 horas, pelo quilô-
metro 170. Qual a razão entre a distância percorrida e o
moças é 15
, se simplificarmos, temos que a fração equi- tempo gasto no translado?
21
5 Para montarmos a razão, precisamos obter as infor-
valente 7 , o que significa que para “cada 5 rapazes há 7 mações:
moças”. Por outro lado, a razão entre o número de rapa-
Distância percorrida: 170 km – 30 km = 140 km
zes e o total de alunos é dada por 15
=
5
, o que equivale
36 12 Tempo gasto: 11h – 9h = 2h
a dizer que “de cada 12 alunos na classe, 5 são rapazes”. Calculamos a razão entre a distância percorrida e o
tempo gasto para isso:
Razão entre grandezas de mesma espécie: A razão
entre duas grandezas de mesma espécie é o quociente 140 𝑘𝑚 70
𝑣= = = 70 𝑘 𝑚 ⁄ℎ
dos números que expressam as medidas dessas grande- 2ℎ 1
zas numa mesma unidade.
Ex. Um automóvel necessita percorrer uma estrada de Como são duas espécies diferentes, a razão entre elas
360 km. Se ele já percorreu 240 km, qual a razão entre a será uma espécie totalmente diferente das outras duas.
distância percorrida em relação ao total?
Como os dois números são da mesma espécie (distân-
cia) e estão na mesma unidade (km), basta fazer a razão: #FicaDica

240 𝑘𝑚 2
A razão entre uma distância e uma medida
𝑟= = de tempo é chamada de velocidade.
360 𝑘𝑚 3

No caso de mesma espécie, porém em unidades di-


ferentes, deve-se escolher uma das unidades e converter Ex. A Região Sudeste (Espírito Santo, Minas Gerais,
a outra. Rio de Janeiro e São Paulo) tem uma área aproximada de
Ex. Uma maratona possui aproximadamente 42 km de 927 286 km2 e uma população de 66 288 000 habitantes,
extensão. Um corredor percorreu 36000 metros. Qual a aproximadamente, segundo estimativas projetadas pelo
razão entre o que falta para percorrer em relação à ex- Instituto Brasileiro de Geografia e Estatística (IBGE) para o
tensão da prova? ano de 1995. Qual a razão entre o número de habitantes
Veja que agora estamos tentando relacionar metros e a área total?
com quilômetros. Para isso, deve-se converter uma das Dividindo-se o número de habitantes pela área, obte-
unidades, vamos utilizar “km”: remos o número de habitantes por km2 (hab./km2):
36000 m=36 km
66288000 ℎ𝑎𝑏 ℎ𝑎𝑏
Como é pedida a razão entre o que falta em relação 𝑑= = 71,5
ao total, temos que: 927286 𝑘𝑚² 𝑘𝑚2

42 𝑘𝑚 − 36 𝑘𝑚 6 𝑘𝑚 1
𝑟=
42 𝑘𝑚
= =
42 𝑘𝑚 7 #FicaDica

Ex. Uma sala tem 8 m de comprimento. Esse compri- A razão entre o número de habitantes e a
mento é representado num desenho por 20 cm. Qual é a área deste local é denominada densidade
razão entre o comprimento representado no desenho e demográfica.
o comprimento real?
Convertendo o comprimento real para cm, temos que: Ex. Um carro percorreu, na cidade, 83,76 km com 8 L
20 𝑐𝑚 1 de gasolina. Dividindo-se o número de quilômetros per-
𝑒= =
800 𝑐𝑚 40 corridos pelo número de litros de combustível consumi-
dos, teremos o número de quilômetros que esse carro
percorre com um litro de gasolina:
#FicaDica
MATEMÁTICA

A razão entre um comprimento no desenho 83,76 𝑘𝑚 𝑘𝑚


e o correspondente comprimento real, cha- 𝑐= = 10,47
8𝑙 𝑙
ma-se escala

31
Outro jeito de ver a proporção: Já vimos que uma
#FicaDica proporção é verdadeira quando realizamos a multiplica-
ção em cruz e encontramos o mesmo valor nos dois pro-
A razão entre a distância percorrida em rela-
dutos. Outra maneira de verificar a proporção é verificar
ção a uma quantidade de combustível é de-
se a duas razões que estão sendo igualadas são frações
finida como “consumo médio”
equivalentes. Lembra deste conceito?

Proporção
FIQUE ATENTO!
A definição de proporção é muito simples, pois se tra- Uma fração é equivalente a outra quando
podemos multiplicar (ou dividir) o nume-
ta apenas da igualdade de razões. rador e o denominador da fração por um
3 6 mesmo número, chegando ao numerador e
Na proporção = (lê-se: “3 está para 5 assim denominador da outra fração.
5 10
como 6 está para 10”).
4 12
Ex. e são frações equivalentes, pois:
Observemos que o produto 3 x 10=30 é igual ao pro- 3 9
duto 5 x 6=30, o que caracteriza a propriedade funda-
mental das proporções
4x=12 →x=3
3x=9 →x=3
#FicaDica
4
Ou seja, o numerador e o denominador de quan-
Se multiplicarmos em cruz (ou em x), tere- 3
mos que os produtos entre o numeradores do multiplicados pelo mesmo número (3), chega ao nu-
e os denominadores da outra razão serão
iguais. merador e denominador da outra fração, logo, elas são
equivalentes e consequentemente, proporcionais.

2 6 Agora vamos apresentar algumas propriedades da


Ex. Na igualdade = , temos 2 x 9=3 x 6=18, logo, proporção:
3 9
temos uma proporção. a) Soma dos termos: Quando duas razões são pro-
porcionais, podemos criar outra proporção somando os
Ex. Na bula de um remédio pediátrico recomenda-se numeradores com os denominadores e dividindo pelos
a seguinte dosagem: 7 gotas para cada 3 kg do “peso” da numeradores (ou denominadores) das razões originais:
criança. Se uma criança tem 15 kg, qual será a dosagem
correta? 5 10 5 + 2 10 + 4 7 14
= → = → =
Como temos que seguir a receita, temos que atender 2 4 5 10 5 10
a proporção, assim, chamaremos de x a quantidade de
gotas a serem ministradas: ou

5 10 5 + 2 10 + 4 7 14
7 𝑔𝑜𝑡𝑎𝑠 𝑥 𝑔𝑜𝑡𝑎𝑠 = → = → =
= 2 4 2 4 2 4
3 𝑘𝑔 15 𝑘𝑔
b) Diferença dos termos: Analogamente a soma,
Logo, para atendermos a proporção, precisaremos temos também que se realizarmos a diferença entre os
encontrar qual o número que atenderá a proporção. Mul- termos, também chegaremos em outras proporções:
tiplicando em cruz, temos que:
4 8 4−3 8−6 1 2
3x=105 = → = → =
3 6 4 8 4 8

105 ou
𝑥=
3 4 8 4−3 8−6 1 2
= → = → =
MATEMÁTICA

3 6 3 6 3 6
x=35 gotas

Ou seja, para uma criança de 30 kg, deve-se ministrar


35 gotas do remédio, atendendo a proporção.

32
c) Soma dos antecedentes e consequentes: A soma 3.(CELESC – Assistente Administrativo – FEPESE/2016)
dos antecedentes está para a soma dos consequentes as- Dois amigos decidem fazer um investimento conjunto
sim como cada antecedente está para o seu consequente: por um prazo determinado. Um investe R$ 9.000 e o ou-
tro R$ 16.000. Ao final do prazo estipulado obtêm um
lucro de R$ 2.222 e decidem dividir o lucro de maneira
12 3 12 + 3 15 12 3
= → = = = proporcional ao investimento inicial de cada um. Portan-
8 2 8+2 10 8 2 to o amigo que investiu a menor quantia obtém com o
investimento um lucro:

d) Diferença dos antecedentes e consequentes: a) Maior que R$ 810,00


A soma dos antecedentes está para a soma dos conse- b) Maior que R$ 805,00 e menor que R$ 810,00
quentes assim como cada antecedente está para o seu c) Maior que R$ 800,00 e menor que R$ 805,00
consequente: d) Maior que R$ 795,00 e menor que R$ 800,00
12 3 12 − 3 9 12 3 e) Menor que R$ 795,00
= → = = =
8 2 8−2 6 8 2 Resposta : Letra D.
Ambos aplicaram R$ 9000,00+R$ 16000,00=R$
25000,00 e o lucro de R$ 2222,00 foi sobre este valor.
FIQUE ATENTO! Assim, constrói-se uma proporção entre o valor apli-
cado (neste caso, R$ 9000,00 , pois o exercício quer o
Usamos razão para fazer comparação entre
lucro de quem aplicou menos) e seu respectivo lucro:
duas grandezas. Assim, quando dividimos
uma grandeza pela outra estamos compa-
rando a primeira com a segunda. Enquanto 9000 25000
proporção é a igualdade entre duas razões. = → 25x = 19998 → x = R$ 799,92
x 2222

EXERCÍCIOS COMENTADOS

1. O estado de Tocantins ocupa uma área aproximada de


278.500 km². De acordo com o Censo/2000 o Tocantins
tinha uma população de aproximadamente 1.156.000 ha-
bitantes. Qual é a densidade demográfica do estado de
Tocantins?

Resposta :
A densidade demográfica é definida como a razão en-
tre o número de habitantes e a área ocupada:

1 156 000 hab.


d= = 4,15 ha b⁄k m²
278 500 km²

2. Se a área de um retângulo (A1 ) mede 300 cm² e a


área de um outro retângulo (A2 ) mede 100 cm², qual é o
valor da razão entre as áreas (A1 ) e (A2 ) ?

Resposta : Ao fazermos a razão das áreas, temos:

A1 300
= =3
A2 100

Então, isso significa que a área do retângulo 1 é 3 ve-


zes maior que a área do retângulo 2.
MATEMÁTICA

33
MEDIDAS DE TEMPO;

O sistema de medidas e unidades existe para quantificar dimensões. Como a variação das mesmas pode ser gigan-
tesca, existem conversões entre unidades para melhor leitura.

Medidas de Comprimento

A unidade principal (utilizada no sistema internacional de medidas) de comprimento é o metro. Para medir dimen-
sões muito maiores ou muito menores que essa referência, surgiram seis unidades adicionais:

km hm dam m dm cm mm
(kilômetro) (hectômetro) (decâmetro) (metro) (decímetro) (centímetro) (milímetro)

A conversão de unidades de comprimento segue potências de 10. Para saber o quanto se deve multiplicar (ou dividir),
utiliza-se a regra do , onde c é o número de casas que se andou na tabela acima. Adicionalmente, se você andou para a
direita, o número deverá ser multiplicado, se andou para a esquerda, será dividido. As figuras a seguir exemplificam as
conversões:

Ex: Conversão de 2,3 metros para centímetros


MATEMÁTICA

34
Ex: Conversão de 125 000 mm para decâmetro:

Medidas de Área (Superfície)

As medidas de área seguem as mesmas referências que as medidas de comprimento. A unidade principal é o metro
quadrado e as outras seis unidades são apresentadas a seguir:

km² hm² dam² m² dm² cm² mm²


(kilômetro (hectômetro (decâmetro (metro qua- (decímetro (centímetro (milímetro
quadrado) quadrado) quadrado) drado) quadrado) quadrado) quadrado)

A conversão de unidades segue com potências de 10. A diferença agora é que ao invés da regra de , utiliza-se a re-
gra de , ou seja, o número de casas que se andou deve ser multiplicado por 2. A definição se multiplica ou divide segue
a mesma regra: Andou para a direita, multiplica, andou para a esquerda, divide. Sigam os exemplos:

Ex: Conversão de 2 km² para m²

MATEMÁTICA

35
Ex: Conversão de 20 mm² para cm²

Medidas de Volume(Capacidade)

As medidas de volume seguem as mesmas referências que as medidas de comprimento. A unidade principal é o
metro cúbico e as outras seis unidades são apresentadas a seguir:

km³ hm³ dam³ m³ dm³ cm³ mm³


(kilômetro (hectômetro- (decâmetro- (metro- (decímetro- (centímetro- (milímetro-
cúbico) cúbico) cúbico) cúbico) cúbico) cúbico) cúbico)

A conversão de unidades segue com potências de 10. A diferença agora é que ao invés da regra de , utiliza-se a re-
gra de , ou seja, o número de casas que se andou deve ser multiplicado por 3. A definição se multiplica ou divide segue
a mesma regra: Andou para a direita, multiplica, andou para a esquerda, divide. Sigam os exemplos:

Ex: Conversão de 3,7 m³ para cm³


MATEMÁTICA

36
Ex: Conversão de 50000 dm³ para m³

kL hL dam³ m³ dm³ cm³ mm³


(quilolitro) (hectolitro) (decalitro) (litro) (decilitro) (centilitro) (mililitro)

Para essa tabela, o roteiro para converter unidades de medidas é o mesmo utilizado para as medidas anteriores. A
diferença é que para cada unidade à direita multiplica-se por 10 e para cada unidade à esquerda divide-se por 10 (igual
para unidades de comprimento).

Medidas de Massa

As medidas de massa segue a base 10, como as medidas de comprimento. A unidade principal é o grama (g) e suas
seis unidades complementares estão apresentadas a seguir:

kg hg dag g dg cg mg
(kilograma) (hectograma) (decagrama) (grama) (decígrama) (centígrama) (milígrama)

Os passos para conversão de unidades segue o mesmo das medidas de comprimento. Utiliza-se a regra do , multi-
plicando se caminha para a direita e divide quando caminha para a esquerda.

FIQUE ATENTO!
Outras unidades importantes:
• Massa: A tonelada, sendo que 1 tonelada vale 1000 kg.
• Volume : O litro (l) que vale 1 decímetro cúbico (dm³) e o mililitro, que vale 1 cm³.
• Área: O hectare (ha) que vale 1 hectômetro quadrado (ou 10000 m²) e o alqueire, (varia de região para
região e normalmente a conversão desejada é dada na prova).

Medidas de Tempo

Desse grupo, o sistema hora – minuto – segundo, que mede intervalos de tempo, é o mais conhecido.
2h = 2 ∙ 60min = 120 min = 120 ∙ 60s = 7 200s
Para passar de uma unidade para a menor seguinte, multiplica-se por 60.
0,3h não indica 30 minutos nem 3 minutos; como 1 décimo de hora corresponde a 6 minutos, conclui-se que 0,3h =
MATEMÁTICA

18min.
Para medir ângulos, também temos um sistema não decimal. Nesse caso, a unidade básica é o grau. Na astronomia,
na cartografia e na navegação são necessárias medidas inferiores a 1º. Temos, então:
1 grau equivale a 60 minutos (1º = 60’)
1 minuto equivale a 60 segundos (1’ = 60”)

37
Os minutos e os segundos dos ângulos não são, é 4. Passe 5.200 gramas para quilogramas.
claro, os mesmos do sistema hora – minuto – segundo.
Há uma coincidência de nomes, mas até os símbolos que Resposta: 5,2 kg. Para passarmos 5.200 gra-
os indicam são diferentes: mas para quilogramas, devemos dividir (por-
1h32min24s é um intervalo de tempo ou um instante que na tabela grama está à direita de quilogra-
do dia. ma) 5.200 por 10 três vezes, pois para passarmos
1º 32’ 24” é a medida de um ângulo. de gramas para quilogramas saltamos três níveis à
esquerda.
Primeiro passamos de grama para decagrama, depois
#FicaDica de decagrama para hectograma e finalmente de hec-
tograma para quilograma:
Por motivos óbvios, cálculos no sistema hora 5200g:10:10:10 ⟹ 5,2Kg
– minuto – segundo são similares a cálculos Isto equivale a passar a vírgula três casas para a esquer-
no sistema grau – minuto – segundo, da.
embora esses sistemas correspondam a Portanto, 5.200 g são iguais a 5,2 kg.
grandezas distintas.
5. Converta 2,5 metros em centímetros.

Resposta: 250 cm. Para convertermos 2,5 me-


EXERCÍCIOS COMENTADOS tros em centímetros, devemos multiplicar (por-
que na tabela metro está à esquerda de centíme-
1. Raquel saiu de casa às 13h 45min, caminhando até o tro) 2,5 por 10 duas vezes, pois para passarmos
curso de inglês que fica a 15 minutos de sua casa, e che- de metros para centímetros saltamos dois níveis à
gou na hora da aula cuja duração é de uma hora e meia. direita.
A que horas terminará a aula de inglês? Primeiro passamos de metros para decímetros e de-
pois de decímetros para centímetros:
a) 14h 2,5m∙10∙10 ⟹ 250cm
b) 14h 30min Isto equivale a passar a vírgula duas casas para a di-
c) 15h 15min reita.
d) 15h 30min Logo, 2,5 m é igual a 250 cm.
e) 15h 45min

Resposta: Letra D. Basta somarmos todos os valores


mencionados no enunciado do teste, ou seja:
13h 45min + 15 min + 1h 30 min = 15h 30min
Logo, a questão correta é a letra D.

2. 348 mm3 equivalem a quantos decilitros?

Resposta: “0, 00348 dl”. Como 1 cm3 equivale a 1 ml,


é melhor dividirmos 348 mm3 por mil, para obtermos
o seu equivalente em centímetros cúbicos: 0,348 cm3.
Logo 348 mm3 equivalem a 0, 348 ml, já que cm3 e ml se
equivalem.
Neste ponto já convertemos de uma unidade de me-
dida de volume, para uma unidade de medida de ca-
pacidade.
Falta-nos passarmos de mililitros para decilitros, quan-
do então passaremos dois níveis à esquerda. Dividire-
mos então por 10 duas vezes:
0,348ml:10:100,00348dl
Logo, 348 mm³ equivalem a 0, 00348 dl.

3. Passe 50 dm2 para hectômetros quadrados.

Resposta: 0, 00005 hm². Para passarmos de decíme-


tros quadrados para hectômetros quadrados, passare-
MATEMÁTICA

mos três níveis à esquerda.


Dividiremos então por 100 três vezes:
50dm2:100:100:100 ⟹ 0,00005hm2
Isto equivale a passar a vírgula seis casas para a esquerda.
Portanto, 50 dm² é igual a 0, 00005 hm².

38
EQUAÇÕES DE PRIMEIRO E SEGUNDO GRAU; SISTEMAS DE EQUAÇÕES;

EQUAÇÃO DO 1º GRAU

Uma equação é uma igualdade na qual uma ou mais variáveis, conhecidas por incógnitas, são desconhecidas. Resol-
ver uma equação significa encontrar o valor das incógnitas. Equações do primeiro grau são equações onde há somente
uma incógnita a ser encontrada e seu expoente é igual a 1. A forma geral de uma equação do primeiro grau é: ax+b=0

Onde e são números reais.

O “lado esquerdo” da equação é denominado 1º membro enquanto o “lado direito” é denominado 2º membro.

#Fica Dica
Para resolver uma equação do primeiro grau, costuma-se concentrar todos os termos que contenham
incógnitas no 1º membro e todos os termos que contenham somente números no 2º membro.

Há diversas formas de equações do primeiro grau e a seguir serão apresentados alguns deles. Antes, há uma lista
de “regras” para a solução de equações do primeiro grau:

Regra 1 – Eliminar os parênteses


Regra 2 – Igualar os denominadores de todos os termos caso haja frações
Regra 3 – Transferir todos os termos que contenham incógnitas para o 1º membro
Regra 4 – Transferir todos os termos que contenham somente números para o 2º membro
Regra 5 – Simplificar as expressões em ambos os membros
Regra 5 – Isolar a incógnita no 1º membro

Exemplo: Resolva a equação 5x-4=2x+8

As regras 1 e 2 não se aplicam pois não há parênteses, nem frações. Aplicando a regra 3, transfere-se o termo “” para
o 1º membro. Para fazer isso, basta colocá-lo no 1º membro com o sinal trocado: 5x-4-2x=8

Aplicando a regra 4, transfere-se o termo “” para o 2º membro. Para fazer isso, basta colocá-lo no 1º membro com
o sinal trocado: 5x-2x=8+4

Aplicando a regra 5, simplifica-se as expressões em ambos os membros. Simplificar significa “juntar” todos os ter-
mos com incógnitas em um único termo no 1º membro e fazer o mesmo com todos os temos que contenham somente
números no 2º membro: 3x=12

Aplicando a regra 6, isola-se a incógnita no 1º membro. Para isso, divide-se ambos os lados da equação por 3, fa-
zendo com que no 1º membro reste apenas :

3x 12 12
= →x= → x = 4 → S = {4�
3 3 3

2x
Exemplo: Resolva a equação 3
+2 x− 4 = x +1

Aplicando a regra 1, eliminam-se os parênteses. Para isso, aplica-se a distributiva no termo com parênteses:

2x
MATEMÁTICA

+ 2x − 8 = x + 1
3

39
Aplicando a regra 2, igualam-se os denominadores de to- Ex:
dos os termos. Nessa equação, o denominador comum é “3”: Dada a equação: 2x2 – 7x + 4 = 1 – x^2, vamos escre-
vê-la na forma normal ou reduzida.
2x 6x 24 3x 3 2x2 – 7x + 4 – 1 + x2 = 0
+ − = + 2x2 + x2 – 7x + 4 – 1 = 0
3 3 3 3 3 3x2 – 7x + 3 = 0

Como há o mesmo denominador em todos os termos, RESOLUÇÃO DE EQUAÇÕES DO 2º GRAU: FÓR-


eles podem ser “cortados”: 2x+6x-24=3x+3 MULA DE BHÁSKARA

Aplicando a regra 3, transfere-se o termo “” para o 1º Para encontras as soluções de equações do segundo
membro: 2x+6x-3x-24=3 grau, é necessário conhecer seu discriminante, represen-
tado pela letra grega Δ (delta).
Aplicando a regra 4, transfere-se o termo “” para o 2º
membro: 2x+6x-3x=24+3 Δ=b2 - 4∙a∙c

Aplicando a regra 5, simplificam-se as expressões em


ambos os membros: 5x=27 FIQUE ATENTO!
O discriminante fornece importantes
Por fim, aplicando a regra 6, isola-se a incógnita no 1º informações de uma equação do 2ª grau:
27 27 Se Δ>0→ A equação possui duas raízes reais
membro: x = →S = e distintas
5 5 Se Δ=0→ A equação possui duas raízes reais
e idênticas
Se Δ<0→ A equação não possui raízes reais

EQUAÇÃO DO 2º GRAU
Equações do segundo grau são equações nas quais o
maior expoente de é igual a 2. Sua forma geral é expres- A solução é dada pela Fórmula de Bháskara:
sa por: ax2+bx+c=0 −b± Δ válida para os casos onde Δ>0 ou Δ=0.
x= ,
2a
Onde a,b e c e são números reais e a≠0. Os números a, Observações importantes:
b e c são chamados coeficientes da equação: • Para utilizar a Fórmula de Bháskara a equação
- a é sempre o coeficiente do termo em X2 . deve estar obrigatoriamente no formato ax2+bx+c=0.
- b é sempre o coeficiente do termo em X. Caso não esteja, é necessário colocar a equação nesse
- c é sempre o coeficiente ou termo independente. formato para, em seguida, aplicar a fórmula!
• Quando b=0 diz-se que as raízes das equações
EQUAÇÃO COMPLETA E INCOMPLETA:
são simétricas.
- Quando b ≠ 0 e c ≠ 0, a equação do 2º grau se diz
As regras para solução de uma equação do 2º grau
completa.
são as seguintes:
Exemplos:
Regra 1 – Identificar os números e
5x2 – 8x + 3 = 0 é uma equação completa (a = 5,b
Regra 2 – Calcular o discriminante
= – 8,c = 3).
y2 + 12y + 20 = 0 é uma equação completa (a = 1,b = Regra 3 – Caso o discriminante não seja negativo, uti-
12,c = 20. lizar a Fórmula de Bháskara

Quando b = 0 ou c = 0 ou b = c = 0, a equação do 2º Exemplo: Resolva a equação


grau se diz incompleta. Aplicando a regra 1, identifica-se: a=1, b=-1 e c=-6
Exemplos : Aplicando a regra 2, calcula-se o discriminante:
x2 – 81 = 0 é uma equação incompleta (a = 1,b = 0 e Δ=b^2-4∙a∙c=(-1)2-4∙1∙(-6)=1+24=25
c = – 81).
10t2 +2t = 0 é uma equação incompleta (a = 10,b = Como o discriminante não é negativo, aplica-se a re-
2 e c = 0). gra 3, que consiste em utilizar a fórmula de Bháskara:
5y2 = 0 é uma equação incompleta (a = 5,b = 0 e c = 0).
1 +5 6
Todas essas equações estão escritas na forma ax2 + bx −b ± Δ − −1 ± 25 1 ± 5 = =3
x= = = = 2 2
+ c = 0 , que é denominada forma normal ou forma redu- 2a 2 ×1 2 1 − 5 −4
zida de uma equação do 2º grau com uma incógnita. = = −2
MATEMÁTICA

2 2
Há, porém, algumas equações do 2º grau que não es-
tão escritas na forma ax2 + bx + c = 0 ; por meio de trans-
formações convenientes, em que aplicamos o princípio
aditivo para reduzi-las a essa forma.

40
Assim, S={-2,3} (50 · 7) + (5 · x) = 510
350 + 5x = 510
Exemplo: Resolva a equação x2-4x+4=0 5x = 510 – 350
Aplicando a regra 1, identifica-se: a=1, b=-4 e c=4 5x = 160
Aplicando a regra 2, calcula-se o discriminante: Δ=b- x = 32
2
-4∙a∙c=(-4)2-4∙1∙4=16-16=0
Portanto, cada um pagou o valor total de R$ 32,00.
Como o discriminante não é negativo, aplica-se a re-
gra 3, que consiste em utilizar a fórmula de Bháskara: 2. (ENEM 2013) A temperatura T de um forno (em graus
centígrados) é reduzida por um sistema a partir do ins-
−b ± Δ − −4 ± 0 4 ± 0 4 tante de seu desligamento (t = 0) e varia de acordo com a
x= = = = =2 2
expressão T t = − t4 + 400 com t em minutos. Por motivos
2a 2×1 2 2
de segurança, a trava do forno só é liberada para abertu-
Assim, S={2} ra quando o forno atinge a temperatura de 39 ºC. Qual o
Note que, como o discriminante é nulo, a equação tempo mínimo de espera, em minutos, após se desligar o
possui duas raízes reais e idênticas iguais a 2. forno, para que a porta possa ser aberta?

Exemplo: Resolva a equação x2+2x+3=0 a) 19,0.


Aplicando a regra 1, identifica-se: a=1, b=2 e c=3 b) 19,8.
Aplicando a regra 2, calcula-se o discriminante: Δ=b- c) 20,0.
2
-4∙a∙c=(2)2-4∙1∙3=4-12=-8 d) 38,0.
e) 39,0.
Como o discriminante é negativo, a equação não pos-
sui raízes reais. Resolução: Letra d. De acordo com o enunciado, te-
t2
Assim, S=∅ (solução vazia). mos a equação T t = − + 400 e a informação
4
de que o forno só pode ser aberto quando T = 39.
Vamos igualar a equação a esse valor:

EXERCÍCIOS COMENTADOS t2
39 = − + 400
4
1. (ENEM 2009) Um grupo de 50 pessoas fez um orça-
mento inicial para organizar uma festa, que seria dividido Novamente, para facilitar os cálculos, multiplicaremos
entre elas em cotas iguais. Verificou-se ao final que, para toda a equação por 4:
arcar com todas as despesas, faltavam R$ 510,00, e que
5 novas pessoas haviam ingressado no grupo. No acerto – t² + 1600 = 156
foi decidido que a despesa total seria dividida em partes – t² = – 1444
iguais pelas 55 pessoas. Quem não havia ainda contribuí-
do pagaria a sua parte, e cada uma das 50 pessoas do Multiplicando toda a equação por (– 1), teremos a se-
grupo inicial deveria contribuir com mais R$ 7,00. guinte equação do 2° grau incompleta:
De acordo com essas informações, qual foi o valor da t² = 1444
cota calculada no acerto final para cada uma das 55 pes- 𝑡 = 1444
soas? t = ± 38

a) R$ 14,00. Como estamos procurando um valor para o “tempo”,


b) R$ 17,00. podemos desconsiderar a resposta negativa. Portan-
c) R$ 22,00. to, t = 38 minutos.
d) R$ 32,00.
e) R$ 57,00. Inequação do 1˚ Grau

Resolução: Letra d. De acordo com o enunciado da Inequação é toda sentença aberta expressa por uma
questão, 50 pessoas já haviam pagado sua parte da desigualdade. Ao invés do sinal de igualdade ( = ) rela-
despesa total, por isso não consideraremos o valor to- cionando duas expressões matemáticas, teremos os si-
tal para elas, apenas o valor de R$ 7,00 adicional, que nais de maior ( > ), menor ( < ), maior ou igual ( ≥ ) ou
deverá ser multiplicado por 50 pessoas. Além desse menor ou igual ( ≤ ). Abaixo seguem alguns exemplos:
pessoal, outros cinco juntaram-se ao grupo e preci- x+1>0
sam pagar sua parte, um valor que não conhecemos 2x-3≤5
e, portanto, podemos identificar como x. Somando-se -3x+7<2x+3
MATEMÁTICA

o valor que essas pessoas pagarão ao valor acrescen- x-2≤4x-1


tado ao restante do grupo, teremos um recolhimento
de R$ 510,00. Podemos então montar uma equação As inequações acima são do primeiro grau pois o ex-
do 1° grau: poente da variável x é igual a 1.

41
b) x < 5, sendo U = ℤ
FIQUE ATENTO!
Todo número inteiro menor que 5 satisfaz a desigual-
O método de resolução das inequações de dade. Logo, V = {..., –2, –1, 0, 1, 2, 3, 4}.
primeiro grau é o mesmo do método de
equações. c) x < 5, sendo U = ℚ

Todo número racional menor que 5 é solução da ine-


PROPRIEDADES DA DESIGUALDADE quação dada. Como não é possível representar os infi-
nitos números racionais menores que 5 nomeando seus
a) Propriedade Aditiva: elementos, nós o faremos por meio da propriedade que
caracteriza seus elementos. Assim:
Mesmo sentido
V = {X ∈ Q|x<5 }
Exemplo: Se 8 > 3, então 8 + 2 > 3 + 2, isto é: 10 > 5 RESOLUÇÃO DE INEQUAÇÕES DO 1º GRAU

Somamos +2 aos dois membros da desigualdade Como mencionado, a resolução de inequações ocorre
de forma similar à de equações. A diferença está na re-
presentação do conjunto solução. Para mostrarmos essa
b) Propriedade Multiplicativa: diferença, vamos resolver os dois primeiros exemplos
apresentados sem U = ℝ
Mesmo sentido
x+1>0

Exemplo: Se 8 > 3, então 8∙ 2 > 3 ∙ 2, isto é: 16 > 6. Passando o +1 para o lado direito: x>-1

Ou seja, o conjunto solução desta inequação serão to-


Multiplicamos os dois membros por 2 dos os valores de x pertencente ao domínio (neste caso,
vamos utilizar todo o conjunto dos números reais) maio-
Mudou de sentido res que -1, assim a solução fica: S = x ∈ ℝ ⁄ x > −1

Uma outra maneira é a representação entre colchetes,


que fica da seguinte forma: S = −1, +∞
Exemplo: Se 8 > 3, então 8 ∙ (–2) < 3 ∙ (–2), isto é: –16 < –6
Vamos explicar essa representação. A representação
dos colchetes para fora indica que o número de dentro
Multiplicamos os dois membros por –2
não pertence a solução e também representa quando há
a presença do infinito (seja “mais” ou “menos” infinito).
FIQUE ATENTO! Assim, -1 não está na solução mas é o limite inferior da
Uma desigualdade não muda de sentido mesma.
quando adicionamos ou subtraímos um Resolvendo agora o segundo exemplo, temos que:
mesmo número aos seus dois membros, nem 2x − 3 ≤ 5
quando multiplicamos ou dividimos seus dois
membros por um mesmo número positivo ou
negativo. 2x ≤ 5 + 3

2x ≤ 8
CONJUNTO UNIVERSO
8
Toda inequação (assim como toda equação) deve ser x≤
2
resolvida em um conjunto universo dado. O conjunto
universo () corresponde ao conjunto de todos os valores
possíveis para a varíavel (ou outra incógnita). x≤4

Vejamos, através do exemplo, a resolução de inequa- Colocando na solução tradicional:


ções do 1º grau.
MATEMÁTICA

S = x∈ ℝ ⁄ x ≤ 4
a) x < 5, sendo U = ℕ
Já a solução em colchetes fica:
Os números naturais que tornam a desigualdade ver-
dadeira são: 0, 1, 2, 3 ou 4. Então V = {0, 1, 2, 3, 4}. S = −∞, 4

42
Ou seja, neste caso, o colchete é fechado no lado do ax2+bx+c >0
número 4 pois ele é o limite superior (todos os valores ax2+bx+c<0
da solução devem ser maiores ou iguais a ele) e neste ax2+bx+c ≥0
caso, ele também é solução (devido a ser maior ou igual ax2+bx+c ≤0
e não somente maior), já o limite inferior vai para menos
infinito e permanece com colchete aberto. FORMAS DA INEQUAÇÃO DO 2º GRAU

Vamos analisar novamente os seis casos possíveis de


combinações de valores de “a” e Δ e verificar o que ocor-
EXERCÍCIOS COMENTADOS re com o sinal de “y”:
a) a>0 e Δ>0: Observe neste tipo de gráfico que
1.(FGV 2010). O conjunto de todas as soluções reais da os valores de y menores que x1 e maiores que x_]2 pos-
inequação 2x + 1 < 3x + 2 é suem y positivo e entre x_1 e x2, y é negativo. Assim, uma
representação na reta real indicará o seguinte:
a) ]-∞, -1[
b) ]-∞, 1[
c) ]-1, +∞[
d) ]1, +∞[
e) ]-1, 1[
ou

Resposta: Letra c. Resolvendo a inequação do primei-


ro grau:
2x + 1 < 3x + 2
2x – 3x < 2 – 1
-x < 1
x > -1
FIQUE ATENTO!
A solução da inequação é o conjunto de números reais A diferença das duas representações está se a
maiores que -1. inequação será apenas maior ou menor (“bo-
Logo, S = ]-1,+∞[. linha vazia” nas raízes, ou seja, elas não entra-
rão no conjunto solução) ou maior ou igual ou
2.(CESGRANRIO 2012) Qual é o menor valor inteiro que menor ou igual (“bolinha cheia” nas raízes, elas
satisfaz a desigualdade apresentada a seguir? entrarão no conjunto solução).
9x + 2(3x – 4) > 11x – 14

a) -2 Exemplo:
b) -1 Resolva a seguinte inequação: x2-3x+2<0
c) 0 Resolução: As raízes dessa função de segundo grau são
d) 1 1 e 2 (Verifique!). Com isso, temos que Δ>0 e pela expres-
e) 2 são, temos que a>0. Colocando na reta real, temos que

Resposta: Letra b. Resolvendo a inequação do pri-


meiro grau:
9x + 2(3x – 4) > 11x – 14
9x + 6x – 8 > 11x – 14
15x – 8 > 11x – 14 Como desejamos valores menores que zero, as raízes
15x – 11x > – 14 + 8 não entram no conjunto solução (“Bolinha vazia”). Assim,
4x > – 6 os valores possíveis que atendem a inequação são os
x > -6/4 valores entre 1 e 2. Pela representação dos colchetes,
x > -3/2 temos que: S = 1,2

Como -3/2 = -1,5, o menor valor inteiro que satisfaz a


inequação é -1. Ou por outra representação: S = x ∈ ℝ 1 < 𝑥 < 2 }

Inequações do 2˚ Grau b) a<0 e Δ>0: Observe neste tipo de gráfico que


MATEMÁTICA

os valores de y menores que x1 e maiores que x2 pos-


As inequações de segundo grau seguem exatamente suem y negativo e entre x1 e x2, y é positivo. Assim, uma
o que foi visto nos diversos tipos de posição que a pa- representação na reta real indicará o seguinte:
rábola pode ter no plano cartesiano. Basicamente uma
inequação de segundo grau possui a seguinte forma:

43
ou
ou

A única diferença é se a raiz entrará ou não no con-


A diferença das duas representações está se a ine- junto solução.
quação será apenas maior ou menor (“bolinha vazia” nas
raízes, ou seja, elas não entrarão no conjunto solução) ou e) a>0 e Δ<0: Os dois últimos tipos serão os mais
maior ou igual ou menor ou igual (“bolinha cheia” nas simples. Como a parábola não toca no eixo x, apenas o
raízes, elas entrarão no conjunto solução) sinal de “a” determina todo o sinal da parábola, assim,
para “a” positivo temos a parábola toda positiva:
Exemplo:
Resolva a seguinte inequação: -x2+5x-4≤0
Resolução: Se você resolver a função do segundo
grau, encontrará como raízes, os números 1 e 4, ou seja,
Δ>0. Como a<0, temos a seguinte representação na reta
real: f) a<0 e Δ<0: Como a parábola não toca no eixo x,
apenas o sinal de “a” determina todo o sinal da parábola,
assim, para “a” negativo temos a parábola toda negativa:

Como desejamos valores menores ou iguais a zero,


temos como solução, todos os valores abaixo de 1 e to- FIQUE ATENTO!
dos os valores acima de 4. Assim, na representação por Em resumo, para resolver uma inequação,
colchetes: S = −∞, 1 ∪ 4, +∞ inicialmente calculamos os zeros da equação
de segundo grau, ou seja, os pontos em que
Ou por outra representação: S = x ∈ ℝ x ≤ 1ou x ≥ 4} o gráfico toca o eixo x. Em seguida, sabendo
qual a concavidade da parábola e os zeros da
c) a>0 e Δ=0: Nos casos onde Δ=0, há apenas um equação, o gráfico pode ser esboçado e a
toque da parábola no eixo x. Assim, o sinal de “a” deter- análise do sinal pode ser feita.
mina se os valores da parábola serão positivos ou ne-
gativos. No caso de “a” positivo, temos toda a parábola
positiva:

ou

A única diferença é se a raiz entrará ou não no con-


junto solução.

d) a<0 e Δ=0: Nos casos onde Δ=0, há apenas um


MATEMÁTICA

toque da parábola no eixo x. Assim, o sinal de “a” deter-


mina se os valores da parábola serão positivos ou nega-
tivos. No caso de “a” negativo, temos toda a parábola
negativa:

44
EXERCÍCIO COMENTADO

1.(CESGRANRIO) O conjunto-solução da inequação 9 – x² > 0 é:

a) – 3 > x > 3
b) – 3 < x < 3
c) x = 3
d) x < 3
e) x > 3

Resposta: Letra b. Sabemos que a função f(x) = 9 – x² é uma função quadrática, onde a=-1, b=0 e c=9.
Podemos concluir que o gráfico de f é uma parábola com a concavidade para baixo, pois a<0.
Vamos agora descobrir as raízes da função f resolvendo a equação:
9 – x² = 0
x² = 9
x = ± √9
x=±3
Daí, o conjunto solução da equação é S = {-3, 3}.
O gráfico de f com as informações obtidas é apresentado a seguir.

Logo, como queremos os valores de x onde f(x) > 0, temos que o conjunto solução é: S = {x ∈ R | − 3 < 𝑥 < 3 }

SISTEMA DE MEDIDAS DE TEMPO; SISTEMA MÉTRICO DECIMAL;

“Prezado Candidato, o tópico acima foi abordado no decorrer da matéria”


MATEMÁTICA

45
FORMAS GEOMÉTRICAS BÁSICAS; PERÍMETRO, ÁREA E VOLUME DE FIGURAS GEOMÉTRICAS;

INTRODUÇÃO A GEOMETRIA PLANA

1. Ponto, Reta e Plano

A definição dos entes primitivos ponto, reta e plano é quase impossível, o que se sabe muito bem e aqui será o
mais importante é sua representação geométrica e espacial.

1.1. Representação, (notação)

→ Pontos serão representados por letras latinas maiúsculas; ex: A, B, C,…


→ Retas serão representados por letras latinas minúsculas; ex: a, b, c,…
→ Planos serão representados por letras gregas minúsculas; ex: β,∞,α,...

1.2. Representação gráfica

Postulados primitivos da geometria, qualquer postulado ou axioma é aceito sem que seja necessária a prova, con-
tanto que não exista a contraprova.
- Numa reta bem como fora dela há infinitos pontos distintos.
- Dois pontos determinam uma única reta (uma e somente uma reta).

- Pontos colineares pertencem à mesma reta.

- Três pontos determinam um único plano.


MATEMÁTICA

46
Uma reta é perpendicular a um plano no espaço , se
- Se uma reta contém dois pontos de um plano, esta ela intersecta o plano em um ponto P e todo segmento
reta está contida neste plano. de reta contido no plano que tem P como uma de suas
extremidades é perpendicular à reta.

Uma reta r é paralela a um plano no espaço , se existe


- Duas retas são concorrentes se tiverem apenas um uma reta s inteiramente contida no plano que é paralela
ponto em comum. à reta dada.
Seja P um ponto localizado fora de um plano. A dis-
tância do ponto ao plano é a medida do segmento de
reta perpendicular ao plano em que uma extremidade é
o ponto P e a outra extremidade é o ponto que é a inter-
seção entre o plano e o segmento.
Se o ponto P estiver no plano, a distância é nula.

Observe que . Sendo que H está contido na reta r e


na reta s.
Um plano é um subconjunto do espaço de tal modo
que quaisquer dois pontos desse conjunto podem ser li-
gados por um segmento de reta inteiramente contida no Planos concorrentes no espaço são planos cuja inter-
conjunto. seção é uma reta.
Planos paralelos no espaço são planos que não tem
Um plano no espaço pode ser determinado por qual- interseção.
quer uma das situações: Quando dois planos são concorrentes, dizemos que
- Três pontos não colineares (não pertencentes à tais planos formam um diedro e o ângulo formado en-
mesma reta); tre estes dois planos é denominado ângulo diedral. Para
- Um ponto e uma reta que não contem o ponto; obter este ângulo diedral, basta tomar o ângulo forma-
- Um ponto e um segmento de reta que não contem do por quaisquer duas retas perpendiculares aos planos
o ponto; concorrentes.
- Duas retas paralelas que não se sobrepõe;
- Dois segmentos de reta paralelos que não se so-
brepõe;
- Duas retas concorrentes;
- Dois segmentos de reta concorrentes.
Duas retas (segmentos de reta) no espaço podem
ser: paralelas, concorrentes ou reversas.
Duas retas são ditas reversas quando uma não tem
interseção com a outra e elas não são paralelas. Pode-se Planos normais são aqueles cujo ângulo diedral é um
pensar de uma reta r desenhada no chão de uma casa e ângulo reto (90°).
uma reta s desenhada no teto dessa mesma casa.
Razão entre Segmentos de Reta

Segmento de reta é o conjunto de todos os pontos


de uma reta que estão limitados por dois pontos que são
as extremidades do segmento, sendo um deles o ponto
inicial e o outro o ponto final. Denotamos um segmento
por duas letras como, por exemplo, AB, sendo A o início
e B o final do segmento.
MATEMÁTICA

Ex: AB é um segmento de reta que denotamos por AB.

47
Segmentos Proporcionais Teorema de Tales: Um feixe de retas paralelas de-
termina sobre duas transversais quaisquer, segmentos
Proporção é a igualdade entre duas razões equiva- proporcionais. A figura abaixo representa uma situação
lentes. De forma semelhante aos que já estudamos com onde aparece um feixe de três retas paralelas cortadas
números racionais, é possível estabelecer a proporcio- por duas retas transversais.
nalidade entre segmentos de reta, através das medidas
desse segmentos.
Vamos considerar primeiramente um caso particular
com quatro segmentos de reta com suas medidas apre-
sentadas na tabela a seguir:

m(AB) = 2cm m(PQ) =4 cm


m(CD) = 3cm m(RS) = 6cm

A razão entre os segmentos e e a razão entre os


segmentos e , são dadas por frações equivalentes, isto
é: ; PQ/RS = 4/6 e como , segue a existência de uma pro-
porção entre esses quatro segmentos de reta. Isto nos Identificamos na sequência algumas proporções:
conduz à definição de segmentos proporcionais.
Diremos que quatro segmentos de reta, , , e , nesta Ex: Consideremos a figura ao lado com um feixe de
ordem, são proporcionais se: . retas paralelas, sendo as medidas dos segmentos indica-
Os segmentos e são os segmentos extremos e os das em centímetros.
segmentos e são os segmentos meios.
A proporcionalidade acima é garantida pelo fato que
existe uma proporção entre os números reais que repre-
sentam as medidas dos segmentos:

Feixe de Retas Paralelas

Um conjunto de três ou mais retas paralelas num pla-


no é chamado feixe de retas paralelas. A reta que inter-
cepta as retas do feixe é chamada de reta transversal. As
retas a, b, c e d que aparecem no desenho anexado, for-
mam um feixe de retas paralelas enquanto que as retas s
e t são retas transversais.

Assim:

B C⁄A B = E F⁄D E
A B⁄D E = B C⁄E F
D E⁄A B = E F⁄B C

#FicaDica
Uma proporção entre segmentos pode
ser formulada de várias maneiras. Se um
dos segmentos do feixe de paralelas for
desconhecido, a sua dimensão pode ser
determinada com o uso de razões proporcionais.
MATEMÁTICA

48
Ângulos Ângulo Inscrito: É o ângulo cujo vértice pertence a
uma circunferência e seus lados são secantes a ela.
Ângulo: Do latim - angulu (canto, esquina), do grego
- gonas; reunião de duas semi-retas de mesma origem
não colineares.

Ângulo Obtuso: É o ângulo cuja medida é maior do


que 90º.

Ângulo Agudo: É o ângulo, cuja medida é menor do


que 90º.

Ângulo Raso:

- É o ângulo cuja medida é 180º;


- É aquele, cujos lados são semi-retas opostas.
Ângulo Central

a) Da circunferência: é o ângulo cujo vértice é o centro


da circunferência;
b) Do polígono: é o ângulo, cujo vértice é o centro do
polígono regular e cujos lados passam por vértices
consecutivos do polígono. Ângulo Reto:

- É o ângulo cuja medida é 90º;


- É aquele cujos lados se apóiam em retas perpendi-
culares.

Ângulo Circunscrito: É o ângulo, cujo vértice não


pertence à circunferência e os lados são tangentes à ela.

Ângulos Complementares: Dois ângulos são com-


plementares se a soma das suas medidas é 900.
MATEMÁTICA

49
Ângulos Congruentes: São ângulos que possuem a Ângulos colaterais internos: O termo colateral sig-
mesma medida. nifica “mesmo lado” e sua propriedade é que a soma des-
tes ângulos será sempre 180°

Ângulos Opostos pelo Vértice: Dois ângulos são


opostos pelo vértice se os lados de um são as respectivas
semi-retas opostas aos lados do outro.

Assim a soma dos ângulos 4 e 5 é 180° e a soma dos


Ângulos Suplementares: Dois ângulos são ditos su- ângulos 3 e 6 também será 180°
plementares se a soma das suas medidas de dois ângulos
é 180º. Ângulos colaterais externos: O termo colateral sig-
nifica “mesmo lado” e sua propriedade é que a soma des-
tes ângulos será sempre 180°

Grau: (º): Do latim - gradu; dividindo a circunferência


em 360 partes iguais, cada arco unitário que corresponde
a 1/360 da circunferência denominamos de grau.

Ângulos formados por duas retas paralelas com


uma transversal

Lembre-se: Retas paralelas são retas que estão no


mesmo plano e não possuem ponto em comum.

Vamos observar a figura abaixo:

Assim a soma dos ângulos 2 e 7 é 180° e a soma dos


ângulos 1 e 8 também será 180°
MATEMÁTICA

Todos esses ângulos possuem relações entre si, e elas


estão descritas a seguir:

50
Ângulos alternos internos: O termo alterno significa Ângulos correspondentes: São ângulos que ocupam
lados diferentes e sua propriedade é que eles sempre se- uma mesma posição na reta transversal, um na região
rão congruentes interna e o outro na região externa.

Assim, o ângulo 1 é igual ao ângulo 5, o ângulo 2 é


igual ao ângulo 6, o ângulo 3 é igual ao ângulo 7 e o
ângulo 4 é igual ao ângulo 8.
Assim, o ângulo 4 é igual ao ângulo 6 e o ângulo 3 é
igual ao ângulo 5 FIQUE ATENTO!
Há cinco classificações distintas para os
Ângulos alternos externos: O termo alterno signifi-
ângulos formados por duas retas paralelas
ca lados diferentes e sua propriedade é que eles sempre
que intersectam uma transversal. Então,
serão congruentes
procure visualizar bem as imagens para
associá-las a cada classificação existente.

EXERCÍCIOS COMENTADOS
1. (CS-UFG-2016) Considere que a figura abaixo re-
presenta um relógio analógico cujos ponteiros das ho-
ras (menor) e dos minutos (maior) indicam 3 h e 40 min.
Nestas condições, a medida do menor ângulo, em graus,
formado pelos ponteiros deste relógio, é:

a) 120°
b) 126°
c) 135°
d) 150°
Assim, o ângulo 1 é igual ao ângulo 7 e o ângulo 2 é
MATEMÁTICA

igual ao ângulo 8 Resposta: Letra B.


Considerando que cada hora equivale a um ângulo de
30° (360/12 = 30) e que a cada 15 min o ponteiro da
hora percorre 7,5°. Assim, as 3h e 40 min indica um
ângulo de aproximadamente 126°.

51
2. Na imagem a seguir, as retas u, r e s são paralelas e cortadas por uma reta transversal. Determine o valor dos ângu-
los x e y.

Resposta: x = 50° e y = 130°


Facilmente observamos que os ângulos x e 50° são opostos pelo vértice, logo, x = 50°. Podemos constatar também
que y e 50° são suplementares, ou seja:

50° + y = 180°
y = 180° – 50°
y = 130°
Portanto,os ângulos procurados são y = 130° e x = 50°.

POLÍGONOS

Um polígono é uma figura geométrica plana limitada por uma linha poligonal fechada. A palavra “polígono” advém
do grego e quer dizer muitos (poly) e ângulos (gon).

Linhas poligonais e polígonos

Linha poligonal é uma sucessão de segmentos consecutivos e não-colineares, dois a dois. Classificam-se em:

Linha poligonal fechada simples:

Linha poligonal fechada não-simples:

Linha poligonal aberta simples:


MATEMÁTICA

Linha poligonal aberta não-simples:

52
FIQUE ATENTO!
Polígono é uma linha fechada simples. Um polígono divide o plano em que se encontra em duas regiões
(a interior e a exterior), sem pontos comuns.

Elementos de um polígono

Um polígono possui os seguintes elementos:

Lados: Cada um dos segmentos de reta que une vértices consecutivos: AB, BC, CD, DE, EA.

Vértices: Ponto de encontro de dois lados consecutivos: A, B, C, D, E

Diagonais: Segmentos que unem dois vértices não consecutivos: AC, AD, BD, BE, CE

Ângulos internos: Ângulos formados por dois lados consecutivos: a� , b� , c� , d


� , e� .

Ângulos externos: Ângulos formados por um lado e pelo prolongamento do lado a ele consecutivo: a�1 , b1 , c1, d1 , e1

Classificação dos polígonos quanto ao número de lados

Nome Número de lados Nome Número de lados


triângulo 3 quadrilátero 4
pentágono 5 hexágono 6
heptágono 7 octógono 8
eneágono 9 decágono 10
hendecágono 11 dodecágono 12
tridecágono 13 tetradecágono 14
pentadecágono 15 hexadecágono 16
heptadecágono 17 octodecágono 18
eneadecágono 19 icoságono 20

A classificação dos polígonos pode ser ilustrada pela seguinte árvore:


MATEMÁTICA

53
Um polígono é denominado simples se ele for des- A soma das medidas dos ângulos centrais de um po-
crito por uma fronteira simples e que não se cruza (daí lígono regular de n lados (Sc ) é igual a 360º.
divide o plano em uma região interna e externa), caso o
contrário é denominado complexo. A medida do ângulo central de um polígono regular
Um polígono simples é denominado convexo se não de n lados () é dada por:
tiver nenhum ângulo interno cuja medida é maior que 360°
180°, caso o contrário é denominado côncavo. ac =
n
Um polígono convexo é denominado circunscrito a
uma circunferência ou polígono circunscrito se todos os
vértices pertencerem a uma mesma circunferência. Polígonos regulares
Um polígono inscritível é denominado regular se to-
dos os seus lados e todos os seus ângulos forem con- Os polígonos regulares são aqueles que possuem to-
gruentes. dos os lados congruentes e todos os ângulos congruen-
tes. Todas as propriedades anteriores são válidas para os
Alguns polígonos regulares: polígonos regulares, a diferença é que todos os valores
a) triângulo equilátero são distribuídos uniformemente, ou seja, todos os ân-
b) quadrado gulos terão o mesmo valor e todas as medidas terão o
c) pentágono regular mesmo valor.
d) hexágono regular

Propriedades dos polígonos #FicaDica


Polígonos regulares são formas de polígonos
De cada vértice de um polígono de n lados, saem mais estudadas e cobradas em questões de
dv = n – 3 concursos.

O número de diagonais de um polígono é dado por:


n n−3
d=
2 EXERCÍCIOS COMENTADOS
Onde n é o número de lados do polígono. 1. (PREF. DE POÁ-SP – ENGENHEIRO DE SEGURAN-
ÇA DE TRABALHO – VUNESP/2015) A figura ilustra
A soma das medidas dos ângulos internos de um po- um octógono regular de lado cm.
lígono de n lados (Si) é dada por:

Si = n − 2 � 180°

A soma das medidas dos ângulos externos de um po-


lígono de n lados (Se) é igual a:

360°
Se =
n

Em um polígono convexo de n lados, o número de


triângulos formados por diagonais que saem de cada
vértice é dado por n - 2. Sendo a altura do trapézio ABCD igual a 1 cm, a área do
triângulo retângulo ADE vale, em cm²
A medida do ângulo interno de um polígono regular
de n lados (ai ) é dada por: a) 5
b) 4
n − 2 � 180° c) 5
ai =
n d) 2 + 1
e) 2
A medida do ângulo externo de um polígono regular
MATEMÁTICA

de n lados (ae ) é dada por:


360°
ae =
n

54
Resposta: Letra D.

Como a altura do trapézio mede 1 cm, temos um trian-


gulo isósceles de hipotenusa AB, assim, o segmento
AD = 2 + 2 . Assim, a área de ADE é:

A fórmula para calcular a soma dos ângulos internos


2+2 2 2 2 2
A= = + =1+ 2 de um polígono é: S = (n – 2) · 180
2 2 2
*n é o número de lados do polígono. No caso desse
2. (UNIFESP - 2003) Pentágonos regulares congruentes exercício:
podem ser conectados lado a lado, formando uma es-
trela de cinco pontas, conforme destacado na figura a S = (5 – 2) · 180
seguir S = 3 · 180
S = 540

Dividindo a soma dos ângulos internos por 5, pois um


pentágono possui cinco ângulos internos, encontrare-
mos 108° como medida de cada ângulo interno.
Observe na imagem anterior que a soma de três ângu-
los internos do pentágono com o ângulo θ tem como
resultado 360°.

108 + 108 + 108 + θ = 360


324 + θ = 360
θ = 360 – 324
θ = 36°
Nessas condições, o ângulo θ mede:
Quadriláteros, Circunferência e Círculo
a) 108°.
b) 72°. Quadriláteros
c) 54°.
d) 36°. São figuras que possuem quatro lados dentre os
e) 18°. quais temos os seguintes subgrupos:

Resposta: Letra D. Paralelogramo


Na ponta da estrela onde está destacado o ângulo θ,
temos o encontro de três ângulos internos de pen-
tágonos regulares. Para descobrir a medida de cada
um desses ângulos, basta calcular a soma dos ângulos
internos do pentágono e dividir por 5.
MATEMÁTICA

55
Características: Losango
Possuem lados paralelos, dois a dois, ou seja:
AB // DC e AD // BC .
Além de paralelos, os lados paralelos possuem a mes-
ma medida, ou seja: AB = DC e AD = BC

A altura é medida em relação a distância entre os seg-


mentos paralelos, ou seja: BG: altura = h

A base é justamente a medida dos lados que se me-


diu a altura: AD: base = b

A área é calculada como o produto da base pela al-


tura: Área= b∙h

O perímetro é calculado como a soma das medidas Características:


Possuem lados paralelos, dois a dois, ou seja:
de todos os quatro lados: AB + BC + CD + DA
AB // DC e AD // BC
Retângulo
Possuem os quatro lados com medidas iguais:
AB = DC = AD = BC

No losango, definem-se diagonais como a distância


entre vértices opostos, assim:

BD: diagonal maior = D e AC: diagonal menor = d

A área é calculada a partir das diagonais e não dos


D�d
lados: Área = 2

O perímetro é calculado como a soma das medidas


Características:
de todos os quatro lados: AB + BC + CD + DA
Possuem lados paralelos, dois a dois, ou seja:
Quadrado
AB // DC e AD // BC

Além de paralelos, os lados paralelos possuem a mes-


ma medida, ou seja: AB = DC e AD = BC

Diferentemente do paralelogramo, todos os ângulos


do retângulo medem 90°: A
�=B � = C� = D
� = 90°

No retângulo, um par de lados paralelos


será a base e o outro será a altura, no desenho:
AB: altura = h e AD: base = b

A área é calculada como o produto da base pela al-


tura: Área= b∙h
Características:
O perímetro é calculado como a soma das medidas Possuem lados paralelos, dois a dois, ou seja:
de todos os quatro lados: AB // DC e AD // BC

Perímetro = AB + BC + CD + DA = 2b + 2h Possuem os quatro lados com medidas iguais:


MATEMÁTICA

AB = DC = AD = BC

Diferentemente do losango, todos os ângulos do


quadrado medem 90°: A
�=B
� = C� = D
� = 90°

56
Seguindo a lógica do retângulo, temos o valor da base Já um círculo é definido como um conjunto de pontos
e da altura iguais neste caso: BC: lado = L e AB: lado = cuja distância de O é menor ou igual a R.

A área é calculada de maneira simples: Área = L2

O perímetro é calculado como a soma


das medidas de todos os quatro lados:
Perímetro = AB + BC + CD + DA = 4L

Trapézio

Características:

A medida relevante da circunferência é o raio (R) que


é a distância de qualquer ponto da circunferência em re-
lação ao centro C.

A área é calculada em função do raio: Área = πR2

Características: O perímetro, também chamado de comprimento da


circunferência, é calculado em função do raio também:
Possuirá apenas um par de lados paralelos que serão Perímetro = 2πR
chamados de bases maior e menor:
Setor Circular
AD// BC, AB: base maior = B e CD: base menor = b
Um Setor Circular é uma região de um círculo com-
A altura será definida como a distância entre as bases: preendida entre dois segmentos de reta que se iniciam
no centro e vão até a circunferência.
BG: altura = h

A área é calculada em função das bases e da altura: #FicaDica


B+b Em termos práticos, um setor circular é um
Área = �h
2 “pedaço” de um círculo.

O perímetro é calculado como a soma das medidas


de todos os quatro lados: AB + BC + CD + DA

Circunferência e Círculo

Uma circunferência é definida como o conjunto de


pontos cuja distância de um ponto, denominado de cen-
tro, O é igual a R, definido como raio.

Características:

O ângulo α é definido como ângulo central


απR2
MATEMÁTICA

Área do Setor Circular (para α em graus): A =


360

αR2
Área do Setor Circular (para α em radianos): A =
2

57
Segmento Circular Reta Exterior: Reta e circunferência não possuem
pontos em comum (dOP > R)
Um Segmento Circular é uma região de um círculo
compreendida entre um segmento que liga os pontos de
cruzamento dos segmentos de reta com a circunferência,
ao qual definimos como corda AB e a circunferência.

Reta Secante: Reta e circunferência possuem dois


pontos em comum (dOP < R)

Características:

A Área do Setor Circular (para α em radianos):


R2
A= α − sen α
2
3. Posições Relativas entre Retas e Circunferências

Dado uma circunferência de raio R e uma reta ‘r’ cuja


distância ao centro da circunferência é ‘d’, temos as se-
guintes posições relativas:

Reta Tangente: Reta e circunferência possuem ape-


nas um ponto em comum (dOP = R)

EXERCÍCIOS COMENTADOS

1.(SEEDUC-RJ – Professor – CEPERJ/2015) O quadrado


MNPQ abaixo tem lado igual a 12cm. Considere que as
curvas MQ e QP representem semicircunferências de diâ-
metros respectivamente iguais aos segmentos MQ e QP.

A área sombreada, em cm2, corresponde a:

a) 30
MATEMÁTICA

b) 36
c) 3 46π − 2
d) 6(36π − 1)
e) 2(6π − 1)

58
Resposta: Letra B. Altura: É um segmento de reta traçada a partir de
Aplicando a fórmula do segmento circular, encontra- um vértice de forma a encontrar o lado oposto ao vértice
-se a área de intersecção dos dois círculos. Subtraindo formando um ângulo reto. BH é uma altura do triângulo.
esse valor da área do semicírculo, chega-se ao resul-
tado.

2. A figura abaixo é um losango. Determine o valor


de x e y, a medida da diagonal AC , da diagonal BD e
o perímetro do triângulo BMC.

Mediana: É o segmento que une um vértice ao ponto


médio do lado oposto. BM é uma mediana.

Resposta:
Aplicando as relações geométricas referentes ao lo- Bissetriz: É a semi-reta que divide um ângulo em
sango, tem-se: duas partes iguais. O ângulo B está dividido ao meio e
neste caso Ê = Ô.
x = 15
y = 20
AC = 20 + 20 = 40
BD = 15 + 15 = 30
BMC = 15 + 20 + 25 = 60

TRIÂNGULOS E TEOREMA DE PITÁGORAS


Ângulo Interno: É formado por dois lados do triângu-
Definição lo. Todo triângulo possui três ângulos internos.

Triângulo é um polígono de três lados. É o polígono Ângulo Externo: É formado por um dos lados do triân-
que possui o menor número de lados. Talvez seja o polí- gulo e pelo prolongamento do lado adjacente (ao lado).
gono mais importante que existe. Todo triângulo possui
alguns elementos e os principais são: vértices, lados, ân-
gulos, alturas, medianas e bissetrizes.
Apresentaremos agora alguns objetos com detalhes
sobre os mesmos.

a) Vértices: A,B,C.
b) Lados: AB,BC e AC.
MATEMÁTICA

c) Ângulos internos: a, b e c.

59
Classificação dos triângulos quanto ao número de Triângulo Retângulo: Possui um ângulo interno reto
lados (90 graus). Atenção a esse tipo de triângulo pois ele é
muito cobrado!
Triângulo Equilátero: Os três lados têm medidas
iguais. .

m(AB) = m(BC) = m(CA)

Medidas dos Ângulos de um Triângulo

Ângulos Internos: Consideremos o triângulo ABC. Po-


deremos identificar com as letras a, b e c as medidas dos
Triângulo Isósceles: Dois lados têm medidas iguais. ângulos internos desse triângulo.
m(AB) = m(AC).
#FicaDica
Em alguns locais escrevemos as letras maiús-
culas, acompanhadas de acento () para re-
presentar os ângulos.

Triângulo Escaleno: Todos os três lados têm medidas


diferentes.

Seguindo a regra dos polígonos, a soma dos ângu-


los internos de qualquer triângulo é sempre igual a 180
2.1. Classificação dos triângulos quanto às medi- graus, isto é: a + b + c = 180°
das dos ângulos
Ex: Considerando o triângulo abai-
Triângulo Acutângulo: Todos os ângulos internos
são agudos, isto é, as medidas dos ângulos são menores xo, podemos achar o valor de x, escrevendo:
do que 90º. 70º + 60º + x = 180º e dessa forma, obtemos
x = 180º − 70º − 60º = 50º

Triângulo Obtusângulo: Um ângulo interno é obtuso,


isto é, possui um ângulo com medida maior do que 90º.

Ângulos Externos: Consideremos o triângulo ABC.


Como observamos no desenho, as letras minúsculas
representam os ângulos internos e as respectivas letras
maiúsculas os ângulos externos.
MATEMÁTICA

60
Todo ângulo externo de um triângulo é igual à soma Casos de Congruência de Triângulos
dos dois ângulos internos não adjacentes a esse ângulo
LLL (Lado, Lado, Lado): Os três lados são conheci-
externo. Assim: A = b + c, B = a + c, C = a + b dos. Dois triângulos são congruentes quando têm, res-
pectivamente, os três lados congruentes. Observe que os
Ex: No triângulo desenhado, podemos achar elementos congruentes têm a mesma marca.
a medida do ângulo externo x, escrevendo:
x = 50º + 80º = 130°.

LAL (Lado, Ângulo, Lado): Dados dois lados e um


ângulo. Dois triângulos são congruentes quando têm
dois lados congruentes e os ângulos formados por eles
Congruência de Triângulos também são congruentes.
Duas figuras planas são congruentes quando têm a
mesma forma e as mesmas dimensões, isto é, o mesmo
tamanho. Para escrever que dois triângulos ABC e DEF
são congruentes, usaremos a notação: ABC ~ DEF
Para os triângulos das figuras abaixo, existe a congruên-
cia entre os lados, tal que: AB ~ RS, BC ~ ST, CA ~ T e
entre os ângulos: ALA (Ângulo, Lado, Ângulo): Dados dois ângulos e
um lado. Dois triângulos são congruentes quando têm
um lado e dois ângulos adjacentes a esse lado, respecti-
vamente, congruentes.

Se o triângulo ABC é congruente ao triângulo RST,


�~R
escrevemos: A � , B� ~ S� , C� ~ �T
LAAo (Lado, Ângulo, Ângulo oposto): Conhecido
um lado, um ângulo e um ângulo oposto ao lado. Dois
triângulos são congruentes quando têm um lado, um ân-
FIQUE ATENTO! gulo, um ângulo adjacente e um ângulo oposto a esse
Dois triângulos são congruentes, se os lado respectivamente congruente.
seus elementos correspondentes são
ordenadamente congruentes, isto é, os três
lados e os três ângulos de cada triângulo
têm respectivamente as mesmas medidas.
Deste modo, para verificar se um triângulo
é congruente a outro, não é necessário
saber a medida de todos os seis elementos,
basta conhecerem três elementos, entre os
quais esteja presente pelo menos um lado.
Para facilitar o estudo, indicaremos os lados
correspondentes congruentes marcados
MATEMÁTICA

com símbolos gráficos iguais.

61
Semelhança de Triângulos

Duas figuras são semelhantes quando têm a mesma forma, mas não necessariamente o mesmo tamanho. Se duas
figuras R e S são semelhantes, denotamos: .

Ex: As ampliações e as reduções fotográficas são figuras semelhantes. Para os triângulos:

Os três ângulos são respectivamente congruentes, isto é: A~R, B~S, C~T

Casos de Semelhança de Triângulos

Dois ângulos congruentes: Se dois triângulos tem dois ângulos correspondentes congruentes, então os triângulos
são semelhantes.

Se A~D e C~F então: ABC =


� DEF

Dois lados proporcionais: Se dois triângulos tem dois lados correspondentes proporcionais e os ângulos formados
por esses lados também são congruentes, então os triângulos são semelhantes.

Como m(AB) ⁄ m(EF) = m(BC) ⁄ m(FG) = 2 ,

então ABC =
� EFG
Ex: Na figura abaixo, observamos que um triângulo pode ser “rodado” sobre o outro para gerar dois triângulos se-
melhantes e o valor de x será igual a 8.
MATEMÁTICA

62
Três lados proporcionais: Se dois triângulos têm os três lados correspondentes proporcionais, então os triângulos
são semelhantes.

Teorema de Pitágoras

Dizem que Pitágoras, filósofo e matemático grego que viveu na cidade de Samos no século VI a. C., teve a intuição
do seu famoso teorema observando um mosaico como o da ilustração a seguir.

Observando o quadro, podemos estabelecer a seguinte tabela:

Triângulo Triângulo Triângulo


ABC A`B`C` A``B``C``
Área do quadrado construído sobre a hipotenusa 4 8 16
Área do quadrado construído sobre um cateto 2 4 8
Área do quadrado construído sobre o outro cateto 2 4 9

Como 4 = 2 + 2 � 8 = 4 + 4 � 16 = 8 + 8 , Pitágoras observou que a área do quadrado construído sobre


a hipotenusa é igual à soma das áreas dos quadrados construídos sobre os catetos.
A descoberta feita por Pitágoras estava restrita a um triângulo particular: o triângulo retângulo isósceles. Estudos
realizados posteriormente permitiram provar que a relação métrica descoberta por Pitágoras era válida para todos os
triângulos retângulos. Os lados do triângulo retângulo são identificados a partir a figura a seguir:

Onde os catetos são os segmentos que formam o ângulo de 90° e a hipotenusa é o lado oposto a esse ângulo. Cha-
mando de “a” e “b” as medidas dos catetos e “c” a medida da hipotenusa, define-se um dos teoremas mais conhecidos
MATEMÁTICA

da matemática, o Teorema de Pitágoras:


c 2 = a2 + b2

Onde a soma das medidas dos quadrados dos catetos é igual ao quadrado da hipotenusa.

63
Teorema de Pitágoras no quadrado

Aplicando o teorema de Pitágoras, podemos estabe-


lecer uma relação importante entre a medida d da diago-
nal e a medida l do lado de um quadrado.

3l2
h² =
4

l 3
h=
2
d= medida da diagonal
l= medida do lado

Aplicando o teorema de Pitágoras no triângulo retân-


gulo ABC, temos: EXERCÍCIOS COMENTADOS

d² = l² + l² 1.(TJ-SP – ESCREVENTE JUDICIÁRIO – VU-


NESP/2017) A figura seguinte, cujas dimensões estão
d = √2l²
indicadas em metros, mostra as regiões R e R , e ,
d=l 2 1 2
ambas com formato de triângulos retângulos, situadas
em uma praça e destinadas a atividades de recreação in-
Teorema de Pitágoras no triângulo equilátero fantil para faixas etárias distintas.

Aplicando o teorema de Pitágoras, podemos estabe-


lecer uma relação importante entre a medida h da altura
e a medida l do lado de um triângulo equilátero.

Se a área de R eé R
1
54 ,m², então o perímetro
2
e R 2 , é, em
R1de
metros, igual a:

a) 54
b) 48
c) 36
d) 40
l= medida do lado e) 42
h= medida da altura
Resposta: Letra B.
No triângulo equilátero, a altura e a mediana coin- Esse problema se resolve tanto por semelhança de
cidem. Logo, é ponto médio do lado BC. No triângulo triângulos, quanto pela área de . Em ambos os casos,
retângulo AHC, é ângulo reto. De acordo com o teorema encontraremos x = 12 m. Após isso, pelo teorema de
de Pitágoras, podemos escrever: R1 e R 2 , , que
Pitágoras, achamos a hipotenusa do triângulo
MATEMÁTICA

será 20 m. Assim, o perímetro será 12+16+20 = 48 m.

64
2. (PM SP 2014 – VUNESP). Duas estacas de madei- Lei dos Cossenos
ra, perpendiculares ao solo e de alturas diferentes, estão
distantes uma da outra, 1,5 m. Será colocada entre elas A lei dos cossenos é considerada uma generalização do
uma outra estaca de 1,7 m de comprimento, que ficará teorema de Pitágoras, onde para qualquer triângulo, conse-
apoiada nos pontos A e B, conforme mostra a figura. guimos relacionar seus lados com a subtração de um termo
que possui o ângulo oposto do lado de referência.


a2 = b2 + c 2 − 2 � b � c � cos A
2 2 2
b = a + c − 2 � a � c � cos B �
A diferença entre a altura da maior estaca e a altura da
menor estaca, nessa ordem, em cm, é: c = a + b − 2 � a � b � cos C�
2 2 2

a) 95.
b) 75. FIQUE ATENTO!
c) 85.
Há três formas distintas de utilizar a Lei dos
d) 80.
Cossenos. Quando for utilizá-la, tenha cui-
e) 90.
dado ao expressar os termos conhecidos e
a incógnita em uma das três equações pro-
Resposta: Letra D.
postas. Note que o termo à esquerda do si-
Note que x é exatamente a diferença que queremos, e
nal de igual é o lado oposto ao ângulo que
podemos calculá-lo através do Teorema de Pitágoras:
deve aparecer na equação.
1,72 = 1,52 + x 2
2,89 = 2,25 + x 2
x 2 = 2,89 – 2,25
x² = 0,64x = 0,8 m ou 80 cm EXERCÍCIOS COMENTADOS

LEI DOS SENOS E LEI DOS COSSENOS 1. Calcule a medida de x:

Lei dos Senos

A Lei dos senos relaciona os senos dos ângulos de


um triângulo qualquer (não precisa necessariamente ser
retângulo) com os seus respectivos lados opostos. Além
disso, há uma relação direta com o raio da circunferência
circunscrita neste triângulo:

Resposta: Aplicando a lei dos senos, lembrando que


temos que aplicar ao ângulo oposto ao lado que ire-
mos usar. Assim, o lado de medida 100 possui o ângu-
� como oposto, e ele mede 30°, dado as medidas
lo A
dos outros ângulos, assim:

x 100
=
sen 45° sen 30°
MATEMÁTICA

x 100
=
2/2 1/2
a b c
= = = 2R
� sen B
sen A � sen C�
x = 100 2

65
2.Calcule a medida de x:

Resposta: Aplicando a lei dos senos, lembrando que ela se relaciona com a circunferência circunscrita ao triângulo:

x
= 2R
sen 60°

x
=2 3
3/2

x=3

GEOMETRIA ESPACIAL

1. Poliedros

Poliedros são sólidos compostos por faces, arestas e vértices. As faces de um poliedro são polígonos. Quando as
faces do poliedro são polígonos regulares e todas as faces possuem o mesmo número de arestas, temos um poliedro
regular. Há 5 tipos de poliedros regulares, a saber:

Tetraedro: poliedro de quatro faces


Hexaedro: poliedro de seis faces (cubo)
Octaedro: poliedro de oito faces
Dodecaedro: poliedro de doze faces
Icosaedro: poliedro de vinte faces

Já poliedros não regulares são sólidos cujas faces ou são polígonos não regulares ou não possuem o mesmo nú-
mero de arestas. Os exemplos mais usuais são pirâmides (com exceção do tetraedro) e prismas (com exceção do cubo).

Relação de Euler: relação entre o número de arestas (A), faces (F) e vértices (V) de um poliedro convexo. É dada por:

V− A+F = 2
MATEMÁTICA

66
2. Prismas

Prisma é um sólido geométrico delimitado por faces planas, no qual as bases se situam em planos paralelos. Quanto
à inclinação das arestas laterais, os prismas podem ser retos ou oblíquos.

2.1. Prisma reto

As arestas laterais têm o mesmo comprimento.


As arestas laterais são perpendiculares ao plano da base.
As faces laterais são retangulares.

2.2. Prisma oblíquo

As arestas laterais têm o mesmo comprimento.


As arestas laterais são oblíquas (formam um ângulo diferente de um ângulo reto) ao plano da base.
As faces laterais não são retangulares.

Bases: regiões poligonais congruentes

Altura: distância entre as bases

Arestas laterais paralelas: mesmas


medidas

Faces laterais: paralelogramos

Prisma reto Aspectos comuns Prisma oblíquo

Prismas regulares: prismas que possuem como base, polígonos regulares (todos os lados iguais).

Sendo AB , a área da base, ou seja, a área do polígono correspondente e AL , a área lateral, caracterizada pela
soma das áreas dos retângulos formados entre as duas bases, temos que:

Área total: AT = AL + 2 � AB

Volume: V = AB . h , onde h é a altura, caracterizada pela distância entre as duas bases


MATEMÁTICA

67
3. Cilindros 5. Cones

São sólidos parecidos com prismas, que apresentam Os cones são sólidos possuem uma única base (cír-
bases circulares e também podem ser retos ou oblíquos. culo). A distância do vértice à circunferência (contorno
da base) é chamada de geratriz (g) e a distância entre o
vértice e o centro do círculo é a altura do cone (h).

Sendo R o raio da base, a altura do cilindro, temos


que: Geratriz: g 2 = R2 + h2

Área da base: AB = πR2 Área da Base: AB = πR2

Área lateral: AL = 2πRh Área Lateral: AL = πgR

Área total: AT = AL + 2AB Área Total: AT = AL + AB


AB � h
Volume: V = AB � h Volume: V =
3

4. Pirâmides 6. Esfera

As pirâmides possuem somente uma base e as faces A esfera é o conjunto de pontos nos quais a distância
laterais são triângulos. A distância do vértice de encontro em relação a um centro é menor ou igual ao raio da es-
dos triângulos com a base é o que determina a altura da fera R. A esfera é popularmente conhecida como “bola”
pirâmide (h). pois seu formato é o mesmo de uma bola de futebol, por
exemplo.

2
Área da Superfície Esférica: A = 4πR

4πR3
Volume: V =
3
Sendo a área da base, determinada pelo polígono
que forma a base, a área lateral, determinada pela soma
das áreas dos triângulos laterais, temos que:
MATEMÁTICA

Área total: AT = AL + AB

AB � h
Volume: V =
3

68
Resposta: Letra A.
#FicaDica
1L = 1dm3 = 1000 cm³
Na área total dos prismas e cilindros, mul-
tiplicamos a área da base por 2 pois temos Volume da lata(cilindro):
duas bases formando o sólido. Já no caso
das pirâmides e dos cones isto não ocorre, VC = πR2h → 3,14 � 42 � h = 1000 → h ≅ 20 cm
pois há apenas uma base em ambos.
Obs: Como o diâmetro é igual a 8cm o raio é igual a
4cm.

3. (PREF. ITAPEMA-SC – TÉCNICO CONTÁBIL – MS


EXERCÍCIO COMENTADO CONCURSOS/2016) O volume de um cone circular reto,
1. (TJ-SP – ESCREVENTE TÉCNICO JUDICIÁRIO – cuja altura é 39 cm, é 30% maior do que o volume de
VUNESP/2017) As figuras seguintes mostram os blocos um cilindro circular reto. Sabendo que o raio da base do
de madeira A, B e C, sendo A e B de formato cúbico e cone é o triplo do raio da base do cilindro, a altura do
C com formato de paralelepípedo reto retângulo, cujos cilindro é:
respectivos volumes, em cm³, são representados por VA,
VB e VC. a) 9 cm
b) 30 cm
c) 60 cm
d) 90 cm

Resposta: Letra D.

Volume do cone: VC

Volume do cilindro: Vcil

Do enunciado: VC = 1,3. Vcil (30% maior).


Se VA + VB = 1/2 VC , então a medida da altura do bloco C,
indicada por h na figura, é, em centímetros, igual a: 4. (CÂMARA DE ARACRUZ-ES – AGENTE ADMINIS-
TRATIVO E LEGISLATIVO – IDECAN/2016) João pos-
a) 15,5 sui cinco esferas as quais, quando colocadas em certa
b) 11 ordem, seus volumes formam uma progressão aritméti-
c) 12,5 ca. Sabendo que a diferença do volume da maior esfera
d) 14 para a menor é 32 cm³ e que o volume da segunda maior
e) 16 esfera é 86,5 cm³, então o diâmetro da menor esfera é:
(Considere: π = 3)
Resposta : Letra C.
VA = 53 = 125 cm³ a) 2 cm
b) 2,5 cm
VB = 103 = 1000 cm³
c) 4,25 cm
VC
Logo: = VA + VB = 125 + 1000 d) 5 cm
2

VC Resposta: Letra D.
→ = 1125 → VC = 2250 cm³
2 Sendo a P.A. (V1 , V2 , V3, V4 , V5), (, o enunciado fornece:

Portanto, VC = 18 � 10 � h = 2250 → h =
2250
= 12,5 cm Do termo geral da P.A., sabe-se que
180
V5 = V1 + 5 − 1 � r = V1 + 4r
2. (PEDAGOGO – IF/2016) Uma lata de óleo de soja de onde r é a razão da P.A.
1 litro, com formato cilíndrico, possui 8 cm de diâmetro
interno. Assim, a sua altura é de aproximadamente: (Con- Assim, V1 + 4r − V1 = 32 → 4r = 32 → r = 8
sidere π = 3,14 )
Como
MATEMÁTICA

a) 20 cm
b) 25 cm V4 = V1 + 3r
c) 201 cm → V1 + 3 � 8 = 86,5
d) 200 cm → V1 + 24 = 86,5
e) 24 cm → V1 = 62,5 cm

69
Como Ampliada: a escala ampliada é aquela na qual o de-
senho é maior do que a realidade. Figuras obtidas com
4 3 auxílio de microscópios, por exemplo, estão em escala
V= πR ampliada.
3
4
→ � 3 � R3 = 62,5 Cálculo de Escala
3
62,5
→ R3 = A escala (E) pode ser expressa como:
4
→ R3 = 15,625 𝑑
𝐸=
→ R = 2,5 cm 𝐷

Como o exercício pediu o diâmetro, D = 2.2,5 = 5 cm onde d é a distância medida no desenho (mapa) e D é
a distância real do objeto (local que o mapa representa).
ESCALAS Assim é possível calcular quaisquer distâncias medidas
no desenho.
Em linhas gerais, escala é a relação matemática en-
tre a distância medida em um mapa (ou desenho, planta,
etc.) e a dimensão real do objeto (local) representado por FIQUE ATENTO!
esse mapa (ou desenho, planta, etc.). Quando se observa
Não se esqueça de trabalhar sempre com as
um mapa e lê-se que ele foi feito em escala 1:500 cm,
mesmas unidades de medida!
significa que 1 cm medido no mapa equivale a 500 cm
na realidade.

Tipos de Escala
Considerando a forma de apresentação, há dois tipos EXERCÍCIOS COMENTADOS
de escala, a saber:
1. (NOVA CONCURSOS - 2018) Considere o mapa a se-
Gráfica: a escala gráfica é aquela na qual a distância
guir:
a ser medida no mapa e sua equivalência são apresenta-
das por unidade. Geralmente estão na parte inferior do
mapa, como no exemplo abaixo:

Fonte: brasilescola.uol.com.br/geografia/escalas.htm

Medindo com uma régua a distância entre 0 e 50 me-


tros, por exemplo, significa que a medida dessa distância
no mapa equivale a 50 metros na realidade.

Numérica: a escala numérica é apresentada como


uma relação e estabelece diretamente qual é a relação
entre distâncias no mapa e real, sem a necessidade de
medições com régua como na escala gráfica. Um exem-
plo de escala numérica: 1:50.000
Isso significa que 1 cm no mapa equivale a 50.000 cm
na realidade. Fonte: GIRARDI, G. ROSA, J.V. 1998 (Adaptação)
Considerando o tamanho da representação de deter-
minado mapa ou desenho, a escala pode ser classificada Determine, em quilômetros, a distância entre as cidades
de três formas: do Rio de Janeiro e Vitória, e de Belo Horizonte a Vitória.

Natural: a escala natural é aquela na qual o tamanho Resposta: 385 km e 346,5 km. Começando pela dis-
MATEMÁTICA

do desenho coincide com o tamanho do objeto real. tância entre Rio de Janeiro e Vitória.
Pela definição de escala:
Reduzida: a escala reduzida é aquela na qual o de- 𝑑 1 5
senho é menor do que a realidade. É a escala na qual a 𝐸= → = → 𝐷 = 5 ∙ 7.700.000 = 38.500.00 𝑐𝑚
𝐷 7.700.000 𝐷
maioria dos mapas é feita.

70
Logo, a distância em quilômetros é igual a: 385 km Na coluna em que aparece a variável x (“litros de ál-
Entre Belo Horizonte e Vitória. cool”), vamos colocar uma flecha:
Pela definição de escala:
𝑑 1 4
𝐸= → = → 𝐷 = 4,5 ∙ 7.700.000 = 34.650.00 𝑐𝑚
𝐷 7.700.000 𝐷

Logo, a distância em quilômetros é igual a: 346,5 km


Observe que, se duplicarmos a distância, o consumo
2. (NOVA CONCURSOS - 2018) Em uma cidade duas de álcool também duplica. Então, as grandezas distância
atrações turísticas distam 4 km. Sabe-se que no mapa e litros de álcool são diretamente proporcionais. No
dessa cidade, esses pontos estão distantes 20 cm um do esquema que estamos montando, indicamos esse fato
outro. Qual é a escala do mapa? colocando uma flecha na coluna “distância” no mesmo
sentido da flecha da coluna “litros de álcool”:
Resposta: 1:20000 Antes de utilizar a definição de es-
cala é importante que ambas as distâncias estejam na
mesma medida. Assim, é necessário passar 4 km para
cm: 4 km=400000 cm.

Pela definição de escala:


𝑑 20 1
𝐸= →𝐸= = → 𝐸 = 1: 20000
𝐷 400000 20000 Armando a proporção pela orientação das flechas,
temos:
3. (NOVA CONCURSOS - 2018) Qual será a distância
entre dois pontos em um mapa sabendo que a escala do
mapa é de 1:200 000 e a distância real entre eles é de 8
km?

Resposta: 4 cm. Antes de utilizar a definição de escala


é importante que ambas as distâncias estejam na mes-
ma medida. Assim, é necessário passar 8 km para cm:
8 km=800000 cm.
Resposta: O carro gastaria 17,5 L de álcool.
Pela definição de escala:
𝑑 1 𝑑 1 𝑑
𝐸= → = → = → 𝑑 = 4𝑐𝑚 #FicaDica
𝐷 200000 800000 2 8
Procure manter essa linha de raciocínio nos di-
versos problemas que envolvem regra de três
REGRA DE TRÊS SIMPLES E COMPOSTA. simples ! Identifique as variáveis, verifique qual
é a relação de proporcionalidade e siga este
exemplo !
Regra de Três Simples

Os problemas que envolvem duas grandezas direta- Ex: Viajando de automóvel, à velocidade de 60 km/h,
mente ou inversamente proporcionais podem ser resol- eu gastaria 4 h para fazer certo percurso. Aumentando
vidos através de um processo prático, chamado regra de a velocidade para 80 km/h, em quanto tempo farei esse
três simples. percurso?
Ex: Um carro faz 180 km com 15L de álcool. Quantos Solução: Indicando por x o número de horas e colo-
litros de álcool esse carro gastaria para percorrer 210 km? cando as grandezas de mesma espécie em uma mesma
coluna e as grandezas de espécies diferentes que se cor-
Solução: respondem em uma mesma linha, temos:
O problema envolve duas grandezas: distância e litros
de álcool. Velocidade (km/h) Tempo (h)
Indiquemos por x o número de litros de álcool a ser 60 4
consumido. 80 x
Coloquemos as grandezas de mesma espécie em uma
mesma coluna e as grandezas de espécies diferentes que Na coluna em que aparece a variável x (“tempo”), va-
MATEMÁTICA

se correspondem em uma mesma linha: mos colocar uma flecha:

Distância (km) Litros de álcool Velocidade (km/h) Tempo (h)


180 15 60 4
210 x 80 x

71
Observe que, se duplicarmos a velocidade, o tempo 2. Para se construir um muro de 17m² são necessários 3
fica reduzido à metade. Isso significa que as grandezas trabalhadores. Quantos trabalhadores serão necessários
velocidade e tempo são inversamente proporcionais. para construir um muro de 51m²?
No nosso esquema, esse fato é indicado colocando-se na
coluna “velocidade” uma flecha em sentido contrário ao Resposta: 9 trabalhadores.
da flecha da coluna “tempo”:
As grandezas (área e trabalhadores) são diretamente
proporcionais. Assim, a regra de três é direta:

Área N Trabalha-
dores
17 3
51 x
Na montagem da proporção devemos seguir o senti-
do das flechas. Assim, temos: 17 � x = 51 � 3 → x = 9 trabalhadores

Regra de Três Composta

O processo usado para resolver problemas que en-


volvem mais de duas grandezas, diretamente ou inver-
samente proporcionais, é chamado regra de três com-
posta.
Ex: Em 4 dias 8 máquinas produziram 160 peças. Em
quanto tempo 6 máquinas iguais às primeiras produzi-
Resposta: Farei esse percurso em 3 h. riam 300 dessas peças?
Solução: Indiquemos o número de dias por x. Co-
loquemos as grandezas de mesma espécie em uma só
EXERCÍCIOS COMENTADOS coluna e as grandezas de espécies diferentes que se
correspondem em uma mesma linha. Na coluna em que
aparece a variável x (“dias”), coloquemos uma flecha:
1. (CBTU – ASSISTENTE OPERACIONAL – FU-
MARC/2016) Dona Geralda comprou 4 m de tecido im-
portado a R$ 12,00 o metro linear. No entanto, o metro
linear do lojista media 2 cm a mais. A quantia que o lojis-
ta deixou de ganhar com a venda do tecido foi:
Comparemos cada grandeza com aquela em que está o x.
a) R$ 0,69
b) R$ 0,96 As grandezas peças e dias são diretamente propor-
c) R$ 1,08 cionais. No nosso esquema isso será indicado colocan-
d) R$ 1,20 do-se na coluna “peças” uma flecha no mesmo sentido
da flecha da coluna “dias”:
Resposta: Letra B.
As grandezas (comprimento e preço) são diretamente
proporcionais. Assim, a regra de três é direta:

Metros Preço
1 12
0,02 x
As grandezas máquinas e dias são inversamente pro-
1 � x = 0,02 � 12 → x = R$ 0,24 porcionais (duplicando o número de máquinas, o número
de dias fica reduzido à metade). No nosso esquema isso
Note que foi necessário passar 2 cm para metros, para será indicado colocando-se na coluna (máquinas) uma
que as unidades de comprimento fiquei iguais. Assim, flecha no sentido contrário ao da flecha da coluna “dias”:
cada 2 cm custaram R$ 0,24 para o vendedor. Como
ele vendeu 4 m de tecido, esses 2 cm não foram con-
MATEMÁTICA

siderados quatro vezes. Assim, ele deixou de ganhar

72
Agora vamos montar a proporção, igualando a razão • Digitadores e horas são inversamente proporcio-
nais, pois se o número de horas trabalhadas au-
que contém o x, que é 4 , com o produto das outras ra- menta, então são necessários menos digitadores
x
para o serviço e, portanto, a quantidade de digi-
zões, obtidas segundo a orientação das flechas 6 160
� : tadores diminui.
8 300

A regra de três fica:

40 2400 12 3
= � �
x 5616 18 2,5
40 86400
→ =
x 252720
→ 86500x = 10108800
→ x = 117 digitadores
Resposta: Em 10 dias.
2. Em uma fábrica de brinquedos, 8 homens montam 20
FIQUE ATENTO! carrinhos em 5 dias. Quantos carrinhos serão montados
por 4 homens em 16 dias?
Repare que a regra de três composta, embo-
ra tenha formulação próxima à regra de três
Resposta:
simples, é conceitualmente distinta devido à
presença de mais de duas grandezas propor-
cionais. Homens Carrinhos Dias
8 20 5
4 x 16

EXERCÍCIOS COMENTADOS Observe que, aumentando o número de homens, a


produção de carrinhos aumenta. Portanto a relação
1. (SEDUC-SP - ANALISTA DE TECNOLOGIA DA IN- é diretamente proporcional (não precisamos inverter
FORMAÇÃO – VUNESP/2014) Quarenta digitadores a razão).
preenchem 2 400 formulários de 12 linhas, em 2,5 ho- Aumentando o número de dias, a produção de carri-
ras. Para preencher 5 616 formulários de 18 linhas, em 3 nhos aumenta. Portanto a relação também é direta-
horas, e admitindo-se que o ritmo de trabalho dos digi- mente proporcional (não precisamos inverter a razão).
tadores seja o mesmo, o número de digitadores neces- Devemos igualar a razão que contém o termo x com o
sários será produto das outras razões.
Montando a proporção e resolvendo a equação, temos:
a) 105
20 8 5
b) 117 = �
c) 123 π 4 16
d) 131
e) 149 Logo, serão montados 32 carrinhos.

Resposta: Letra B.
A tabela com os dados do enunciado fica:

Digitadores Formulários Linhas Horas


40 2400 12 2,5
x 5616 18 3

Comparando-se as grandezas duas a duas, nota-se


que:
• Digitadores e formulários são diretamente propor-
cionais, pois se o número de digitadores aumenta,
a quantidade de formulários que pode ser digitada
MATEMÁTICA

também aumenta.
• Digitadores e linhas são diretamente proporcio-
nais, pois se a quantidade de digitadores aumenta,
o número de linhas que pode ser digitado também
aumenta.

73
6. (Pref. Teresina – PI) Roberto trabalha 6 horas por dia
de expediente em um escritório. Para conseguir um dia
extra de folga, ele fez um acordo com seu chefe de que
HORA DE PRATICAR! trabalharia 20 minutos a mais por dia de expediente pelo
número de dias necessários para compensar as horas de
um dia do seu trabalho. O número de dias de expediente
que Roberto teve que trabalhar a mais para conseguir
1.(SAAE de Aimorés – MG) Em uma festa de aniversário, seu dia de folga foi igual a Parte superior do formulário
cada pessoa ingere em média 5 copos de 250 ml de refri-
gerante. Suponha que em uma determinada festa, havia a) 16
20 pessoas presentes. Quantos refrigerantes de 2 litros o b) 15
organizador da festa deveria comprar para alimentar as c) 18
20 pessoas? d) 13
e) 12
a) 12
b) 13 7.(ITAIPU BINACIONAL) O valor da expressão:
c) 15 1 + 1 + 1 + 1𝑥7 + 1 + 1𝑥0 + 1 − 1 é
d) 25
a) 0
2. Analise as afirmativas a seguir e assinale a alternativa b) 11
CORRETA: c) 12
I) 3 𝑥 4 ∶ 2 = 6 d) 29
II) 3 + 4 𝑥 2 = 14 e) 32
III) O resto da divisão de 18 por 5 é 3
8. Qual a diferença prevista entre as temperaturas no
a) I somente Piauí e no Rio Grande do Sul, num determinado dia, se-
b) I e II somente gundo as informações? Tempo no Brasil: Instável a enso-
c) I e III somente larado no Sul. Mínima prevista -3º no Rio Grande do Sul.
d) I, II e III Máxima prevista 37° no Piauí.
3. (Pref. de Timon – MA) O problema de divisão 648 : 2 a) 34
é equivalente à: b) 36
c) 38
a) 600: 2 𝑥 40: 2 𝑥 8: 2 d) 40
b) 6: 2 + 4: 2 + 8: 2 e) 42
c) 600: 2 − 40: 2 − 8: 2
d) 600: 2 + 40: 2 + 8: 2 9. Qual é o produto de três números inteiros consecuti-
e) 6: 2𝑥4: 2𝑥8: 2 vos em que o maior deles é –10?
4. (Pref. de São José do Cerrito – SC) Qual o valor da a) -1320
expressão: 34 + 14.4⁄2 − 4 ? b) -1440
c) +1320
a) 58 d) +1440
b) -31 e) nda
c) 92
d) -96 10. Três números inteiros são consecutivos e o menor
deles é +99. Determine o produto desses três números.
5. (IF-ES) Um caminhão tem uma capacidade máxima de
700 kg de carga. Saulo precisa transportar 35 sacos de ci- a) 999.000
mento de 50 kg cada um. Utilizando-se desse caminhão, b) 999.111
o número mínimo de viagens que serão necessárias para c) 999.900
realizar o transporte de toda a carga é de: d) 999.999
e) 1.000.000
a) 4
b) 5
c) 2
d) 6
e) 3
MATEMÁTICA

74
11. Adicionando –846 a um número inteiro e multiplican- 16. (IF-SE – TÉCNICO DE TECNOLOGIA DA INFOR-
do a soma por –3, obtém-se +324. Que número é esse? MAÇÃO - FDC-2014) João, nascido entre 1980 e 1994,
irá completar, em 2014, x anos de vida. Sabe-se que x
a) 726 é divisível pelo produto dos seus algarismos. Em 2020,
b) 738 João completará a seguinte idade:
c) 744
d) 752 a) 32
e) 770 b) 30
c) 28
12. Numa adição com duas parcelas, se somarmos 8 à d) 26
primeira parcela, e subtrairmos 5 da segunda parcela, o
que ocorrerá com o total? 17. (PREF. ITATINGA-PE – ASSISTENTE ADMINIS-
TRATIVO – IDHTEC/2016) O número 102 + 101 + 100 é
a) -2 a representação de que número?
b) -1
c) +1 a) 100
d) +2 b) 101
e) +3 c) 010
d) 111
13. (Prefeitura de Chapecó – Engenheiro de Trânsito e) 110
– IOBV/2016) A alternativa cujo valor não é divisor de
18.414 é: 18. (TRF-SP – TÉCNICO JUDICIÁRIO – FCC/2014) O
resultado da expressão numérica 53 : 51 × 54 : 5 × 55 : 5 :
a) 27 56 - 5 é igual a :
b) 31
c) 37 a) 120.
d) 22
1
b)
5
14. Verifique se os números abaixo são divisíveis por 4.
c) 55.
a) 23418 d) 25.
b) 65000 e) 620.
c) 38036
d) 24004 19. (FEI-SP) O valor da expressão B = 5 . 108 . 4 . 10-3 é:
e) 58617
a) 206
15. (ALGÁS – ASSISTENTE DE PROCESSOS ORGANI- b) 2 . 106
ZACIONAIS – COPEVE/2014) c) 2 . 109
d) 20 . 10-4
Critério de divisibilidade por 11
20. (PREF. GUARULHOS-SP –ASSISTENTE DE GES-
Esse critério é semelhante ao critério de divisibilidade por TÃO ESCOLAR – VUNESP/2016) Para iniciar uma visita
9. Um número é divisível por 11 quando a soma alter- monitorada a um museu, 96 alunos do 8º ano e 84 alunos
nada dos seus algarismos é divisível por 11. Por soma do 9º ano de certa escola foram divididos em grupos,
alternada queremos dizer que somamos e subtraímos al- todos com o mesmo número de alunos, sendo esse nú-
garismos alternadamente (539  5 - 3 + 9 = 11). mero o maior possível, de modo que cada grupo tivesse
Disponível em:<http://educacao.globo.com> . Acesso somente alunos de um único ano e que não restasse ne-
em: 07 maio 2014. nhum aluno fora de um grupo. Nessas condições, é cor-
reto afirmar que o número total de grupos formados foi
Se A e B são algarismos do sistema decimal de numera-
ção e o número 109AB é múltiplo de 11, então a) 8
b) 12
a) B = A c) 13
b) A+B=1 d) 15
c) B-A=1 e) 18
d) A-B=10
e) A+B=-10
MATEMÁTICA

75
21. (PREF. ITATINGA-PE – ASSISTENTE ADMINIS- 2.5 (PREF. TERRA DE AREIA-RS – AGENTE ADMI-
TRATIVO – IDHTEC/2016) Um ciclista consegue fazer NISTRATIVO – OBJETIVA/2016) Três funcionários
um percurso em 12 min, enquanto outro faz o mesmo (Fernando, Gabriel e Henrique) de determinada empresa
percurso 15 min. Considerando que o percurso é circular deverão dividir o valor de R$ 950,00 entre eles, de forma
e que os ciclistas partem ao mesmo tempo do mesmo diretamente proporcional aos dias trabalhos em certo
local, após quanto tempo eles se encontrarão? mês. Sabendo-se que Fernando trabalhou 10 dias, Ga-
briel, 12, e Henrique, 16, analisar os itens abaixo:
a) 15 min I - Fernando deverá receber R$ 260,00.
b) 30 min II - Gabriel deverá receber R$ 300,00.
c) 1 hora III - Henrique deverá receber R$ 410,00.
d) 1,5 horas Está(ão) CORRETO(S):
e) 2 horas
a) II
22. (PREF. SANTA TERIZINHA DO PROGRESSO-SC b) I e II
– PROFESSOR DE MATEMÁTICA – CURSIVA/2018) c) I e III
Acerca dos números primos, analise. d) II e III
I- O número 11 é um número primo; e) Todos os itens
II- O número 71 não é um número primo;
III- Os números 20 e 21 são primos entre si. 26. (TRT- 15ª REGIÃO SP– ANALISTA JUDICIÁRIO –
FCC/2018) André, Bruno, Carla e Daniela eram sócios em
Dos itens acima: um negócio, sendo a participação de cada um, respecti-
vamente, 10%, 20%, 20% e 50%. Bruno faleceu e, por não
a) Apenas o item I está correto. ter herdeiros naturais, estipulara, em testamento, que sua
b) Apenas os itens I e II estão corretos. parte no negócio deveria ser distribuída entre seus só-
c) Apenas os itens I e III estão corretos. cios, de modo que as razões entre as participações dos
d) Todos os itens estão corretos. três permanecessem inalteradas. Assim, após a partilha, a
nova participação de André no negócio deve ser igual a:
23. (SAMAE DE CAXIAS DO SUL –RS – OPERADOR
DE ESTAÇÃO DE TRATAMENTO DE ÁGUA E ESGOTO a) 20%.
– OBJETIVA/2017) Marcar C para as afirmativas Certas, b) 8%
E para as Erradas e, após, assinalar a alternativa que apre- c) 12,5%
senta a sequência CORRETA: d) 15%
(---) Pertencem ao conjunto dos números naturais ímpa- e) 10,5%
res os números ímpares negativos e os positivos.
(---) O número 72 é divisível por 2, 3, 4, 6, 8 e 9 27. (PREF. GUARULHOS-SP – AUXILIAR ADMINIS-
(---) A decomposição do número 256 em fatores primos TRATIVO – VUNESP/2018) Um terreno retangular tem
é 27 35 m de largura e 1750 m2 de área. A razão entre a largu-
(---) Considerando-se os números 84 e 96, é correto afir- ra e o comprimento desse terreno é
mar que o máximo divisor comum é igual a 12.
a) 0,8.
a) E - E - C - C. b) 0,7.
b) E - C - C - E. c) 0,6.
c) C - E - E - E. d) 0,5.
d) E - C - E - C. e) 0,4.
e) C - E - C - C.

24. (PREF. GUARULHOS-SP – AGENTE ESCOLAR


– VUNESP/2016) No ano de 2014, três em cada cinco
estudantes, na faixa etária dos 18 aos 24 anos, estavam
cursando o ensino superior, segundo dados do Instituto
Brasileiro de Geografia e Estatística. Supondo-se que na-
quele ano 2,4 milhões de estudantes, naquela faixa etária,
não estivesse cursando aquele nível de ensino, o número
dos que cursariam o ensino superior, em milhões, seria:

a) 3,0
b) 3,2
MATEMÁTICA

c) 3,4
d) 3,6
e) 4,0

76
Leia o texto, para responder a Questão a seguir: 28. (UTPR 2018) O preço de cada peça é definido pro-
porcionalmente à área de cada uma em relação à unida-
Uma loja vende peças de MDF (mistura de fibras de de padrão. Por exemplo, a área da peça B é metade da
madeira prensada) retangulares para artesãos. A unidade área da unidade padrão, desse modo o preço da peça B
padrão mede 22 cm de comprimento por 15 cm de lar- é metade do preço da unidade padrão, ou seja, R$ 12,00.
gura e custa R$ 24,00. Assim, as peças A, C e D custam respectivamente:

a) R$ 12,00; R$ 12,00; R$ 4,00


b) R$ 12,00; R$ 6,00; R$ 6,00
c) R$ 6,00; R$ 4,00; R$ 4,00
d) R$ 12,00; R$ 4,00; R$ 6,00
e) R$ 12,00; R$ 6,00; R$ 4,00

Fonte: http://voltarelliprudente.com.br/ o-que-e-m-


df-cru/ 29. Dividindo-se 660 em partes inversamente proporcio-
O catálogo desta loja disponibiliza peças com outras nais aos números 1/2, 1/3 e 1/6 obtém-se que números?
medidas cortadas a partir da unidade padrão. Observe
que ele está com informações incompletas em relação a a) 30, 10, 5.
área e preço das peças. b) 30, 20, 10.
c) 40, 30, 20.
d) 20, 10, 5

30. Certo concreto é obtido misturando-se uma parte de


cimento, dois de areis e quatro de pedra. Qual será (em
m³) a quantidade de areia a ser empregada, se o volume
a ser concretado é 378 m³?

a) 108m3
b) 100m3
c) 80m3
e) 60m3

31. A herança de R$ 30.000,00 deve ser repartida entre


Antonio, Bento e Carlos. Cada um deve receber em partes
diretamente proporcionais a 3, 5 e 6, respectivamente, e
inversamente proporcionais às idades de cada um. Sa-
bendo-se que Antonio tem 12 anos, Bento tem 15 anos
e Carlos 24 anos, qual será a parte recebida por Bento?

a) R$ 12.000,00.
b) R$ 14.000,00.
b) R$ 8.000,00.
c) R$ 24.000,00.

32. (SAAE Aimorés- MG – Ajudante – MÁXIMA/2016)


Misturam-se 30 litros de álcool com 20 litros de gasolina.
A porcentagem de gasolina na mistura é igual a:

a) 40%
b) 20%
c) 30%
d) 10%
MATEMÁTICA

77
33. (PREF. PIRAÚBA-MG – OFICIAL DE SERVIÇO PÚ- 36. (EBSERH – TÉCNICO EM ENFERMAGEM –
BLICO – MS CONCURSOS/2017) Certo estabelecimen- IBFC/2017) Paulo gastou 40% de 3/5 de seu salário e
to de ensino possui em seu quadro de estudantes alunos ainda lhe restou R$ 570,00. Nessas condições o salário
de várias idades. A quantidade de alunos matriculados de Paulo é igual a:
neste estabelecimento é de 1300. Sabendo que deste to-
tal 20% são alunos maiores de idade, podemos concluir a) R$ 2375,00
que a quantidade de alunos menores de idade que estão b) R$ 750,00
matriculados é: c) R$ 1240,00
d) R$ 1050,00
a) 160 e) R$ 875,00
b) 1040
c) 1100 37. (PREF. TANGUÁ-RJ – TÉCNICO E ENFERMAGEM
d) 1300 – MS CONCURSOS/2017) Raoni comprou um fogão
com 25% de desconto, pagando por ele R$ 330,00. Qual
34. (PREF. JACUNDÁ-PA – AUXILIAR ADMINISTRA- era o preço do fogão sem o desconto?
TIVO – INAZ/2016) Das 300 dúzias de bananas que seu
José foi vender na feira, no 1° dia, ele vendeu 50% ao a) R$ 355,00
preço de R$ 3,00 cada dúzia; no 2° dia ele vendeu 30% da b) R$ 412,50
quantidade que sobrou ao preço de R$ 2,00; e no 3° dia c) R$ 440,00
ele vendeu 20% do que restou da venda dos dias ante- d) R$ 460,00
riores ao preço de R$ 1,00. Quanto seu José apurou com
as vendas das bananas nos três dias? 38. (EBSERH – ADVOGADO – IBFC/2016) Ao comprar
um produto, José obteve um desconto de 12% (doze
a) R$ 700,00 por cento) por ter pagado à vista e pagou o valor de R$
b) R$ 540,00 105,60 (cento e cinco reais e sessenta centavos). Nessas
c) R$ 111,00 condições, o valor do produto, sem desconto, é igual a:
d) R$ 450,00
e) R$ 561,00 a) R$ 118,27
b) R$ 125,00
35. (COLÉGIO PEDRO II – PROFESSOR – 2016) Com c) R$ 120,00
a criação de leis trabalhistas, houve muitos avanços em d) R$ 130,00
relação aos direitos dos trabalhadores. Entretanto, ainda e) R$ 115,00
há muitas barreiras. Atualmente, a renda das mulheres
corresponde, aproximadamente, a três quartos da renda 39. (PREF. ITAPEMA-SC – AGENTE MUNICIPAL DE
dos homens. Considerando os dados apresentados, qual TRÂNSITO – MS CONCURSOS/2016) Segundo da-
a diferença aproximada, em termos percentuais, entre a dos do IBGE, a população de Itapema (SC) em 2010 era
renda do homem e a da mulher? de, aproximadamente, 45.800 habitantes. Já atualmente,
essa população é de, aproximadamente, 59.000 habitan-
a) 75% tes. O aumento percentual dessa população no período
b) 60% de 2010 a 2016 foi de:
c) 34%
d) 25% a) 22,4%
b) 28,8%
c) 71,2%
d) 77,6%

40. (EBSERH – ADVOGADO – IBFC/2016) Joana gas-


tou 60% de 50% de 80% do valor que possuía. Portanto,
a porcentagem que representa o que restou para Joana
do valor que possuía é:

a) 76%
b) 24%
c) 32%
d) 68%
e) 82%
MATEMÁTICA

78
41. (TRT 11ª REGIÃO – ANALISTA JUDICIÁRIO – FCC/2015) Em 2015 as vendas de uma empresa foram 60% su-
periores as de 2014. Em 2016 as vendas foram 40% inferiores as de 2015. A expectativa para 2017 é de que as vendas
sejam 10% inferiores as de 2014. Se for confirmada essa expectativa, de 2016 para 2017 as vendas da empresa vão.

a) diminuir em 6,25%
b) aumentar em 4%
c) diminuir em 4%
d) diminuir em 4,75%
e) diminuir em 5,5%

42. (SAMAE CAXIAS DO SUL –RS –AJUSTADOR DE HIDRÔMETROS – OBJETIVA/2017) Em certa turma de
matemática do Ensino Fundamental, o professor dividiu igualmente os 34 alunos em dois grupos (A e B) para que
participassem de certa competição de matemática envolvendo frações. Para cada resposta correta dada pelo grupo,
este ganhava 10 pontos e, para cada resposta incorreta, o grupo transferia 5 dos seus pontos para a equipe adversária.
Considerando-se que os grupos A e B iniciaram a competição com 20 pontos cada, e as questões foram as seguintes,
assinalar a alternativa CORRETA:

a) grupo B ficou com 25 pontos a mais do que o grupo A.


b) grupo A ficou com 10 pontos a mais do que o grupo B.
c) grupo B ganhou ao todo 30 pontos e perdeu 5.
d) grupo A ganhou ao todo 20 pontos e perdeu 10.
e) Os dois grupos terminaram a competição com a mesma pontuação, 30 pontos cada.

43. (UFGO) Uma fração equivalente a 3/4 cujo denominador é um múltiplo dos números 3 e 4 é:

a) 6/8
b) 9/12
c) 15/24
d) 12/16
MATEMÁTICA

79
44. (COLÉGIO PEDRO II – PROFESSOR – 2018) O nú- 48. (PREF. SUZANO-SP – GUARDA CIVIL MUNICI-
mero decimal que representa a quantidade de crianças e PAL – VUNESP/2018) Para imprimir um lote de panfle-
jovens envolvidos em atividades não agrícolas no Brasil, tos, uma gráfica utiliza apenas uma máquina, trabalhan-
segundo o PNAD 2015, é: do 5 horas por dia durante 3 dias. O número de horas
diárias que essa máquina teria que trabalhar para impri-
a) 68/10 mir esse mesmo lote em 2 dias seria
b) 0,68
c) 6,8 a) 8,0.
d) 68/100 b) 7,5.
c) 7,0.
45. Em seu testamento, uma mulher decide dividir seu d) 6,5.
patrimônio entre seus quatro filhos. Tal divisão foi feita e) 6,0.
da seguinte forma:
49. (VUNESP – CÂMARA MUNICIPAL DE SÃO CAR-
• João receberá 1/5; LOS – AGENTE DE COPA – 2013) Com uma lata de leite
• Camila receberá 15%; condensado, é possível se fazer 30 brigadeiros. Sabendo
• Ana receberá R$ 16.000,00; que o preço de cada lata é de 4 reais, e para uma come-
• Carlos receberá 25%. moração serão necessários 450 brigadeiros, o total gasto,
A fração que representa a parte do patrimônio recebida em reais, para fazer esses brigadeiros, será de
por Ana é:
a) 45
a) 2/4. b) 53
b) 3/5. c) 60
c) 2/5. d) 70.
d) 1/4. 50. (VUNESP – CÂMARA MUNICIPAL DE SÃO CAR-
e) 3/4. LOS – RECEPCIONISTA – 2013) Num posto de gaso-
lina, foi pedido ao frentista que enchesse o tanque de
46. Bela é uma leitora voraz. Ela comprou uma cópia combustível. Foram colocados 20,6 litros de gasolina, pe-
do best seller «A Beleza da Matemática». No primeiro dia, los quais custou R$ 44,29. Se fossem colocados 38 litros
Bela leu 1/5 das páginas mais 12 páginas, e no segundo de gasolina, o valor a ser pago seria de
dia, ela leu 1/4 das páginas restantes mais 15 páginas.
No terceiro dia, ela leu 1/3 das páginas restantes mais 18 a) R$ 37,41.
páginas. Então, Bela percebeu que restavam apenas 62 b) R$ 79,80.
páginas para ler, o que ela fez no dia seguinte. Então, o c) R$ 81,70.
livro lido por Bela possuía o seguinte número de páginas: d) R$ 85,30.
e) R$ 88,50.
a) 120.
b) 180. 51. (VUNESP - CÂMARA MUNICIPAL DE SÃO CAR-
c) 240. LOS – RECEPCIONISTA – 2013) Lendo 30 páginas por
d) 300. dia de um livro, gastarei 6 dias para ler esse livro. Se eu ler
20 páginas por dia desse mesmo livro, gastarei
47. (EMAP – CARGOS DE NÍVEL MËDIO – CES-
PE/2018) Os operadores dos guindastes do Porto de a) 9 dias.
Itaqui são todos igualmente eficientes. Em um único dia, b) 8 dias.
seis desses operadores, cada um deles trabalhando du- c) 6 dias.
rante 8 horas, carregam 12 navios. d) 5 dias.
Com referência a esses operadores, julgue o item seguin- e) 4 dias.
te.
52. (VUNESP – PROCON – AUXILIAR DE MANUTEN-
Para carregar 18 navios em um único dia, seis desses ÇÃO – 2013) Um supermercado fez a seguinte oferta
operadores deverão trabalhar durante mais de 13 horas. “3/4 de quilograma de carne moída por apenas R$ 4,50
‘’. Uma pessoa aproveitou a oferta e comprou 3 quilogra-
( ) CERTO ( ) ERRADO mas de carne moída. Essa pessoa pagou pelos 3 quilo-
gramas de carne

R$ 18,00.
R$ 18,50.
MATEMÁTICA

R$ 19,00.
R$ 19,50.
R$ 20,00.

80
53. (VUNESP – TJM – SP – AGENTE DE SEGURAN- 57. (CISMARPA – AUXILIAR ADMINISTRATIVO –
ÇA JUDICIÁRIA – 2013) Se certa máquina trabalhar IPEFAE/2015) Em um restaurante, 4 cozinheiros fazem
seis horas por dia, de forma constante e sem parar, ela 120 pratos em 5 dias. Para atender uma demanda maior
produzira n peças em seis dias. Para produzir quantidade de pessoas, o gerente desse estabelecimento contratou
igual das mesmas peças em quatro dias, essa máquina mais 2 cozinheiros. Quantos pratos serão feitos em 8 dias
deverá trabalhar diariamente, nas mesmas condições, um de funcionamento do restaurante?
número de horas igual a
a) 288
a) 12. b) 294
b) 10. c) 296
c) 9. d) 302
d) 8.
58. (CRO-SP – ASSISTENTE ADMINISTRATIVO –
54. (VUNESP – AUXILIAR AGROPECUÁRIO – 2014) VUNESP/2015) Cinco máquinas, todas de igual eficiên-
O refeitório de uma fábrica prepara suco para servir no cia, funcionando 8 horas por dia, produzem 600 peças
almoço. Com 5 litros de suco é possível encher comple- por dia. O número de peças que serão produzidas por 12
tamente 20 copos de 250 ml. Em um certo dia, foram dessas máquinas, funcionando 10 horas por dia, durante
servidas 90 refeições e acompanhando cada uma delas, 5 dias, será igual a
1 copo com 250 ml de suco. O número, mínimo, de litros
de suco necessário para o almoço, desse dia, foi a) 1800.
b) 3600.
a) 21,5. c) 5400.
b) 22. d) 7200.
c) 22,5. e) 9000.
d) 23.
e) 23,5. 59. (PREF. PORTO ALEGRE-RS – FMP CONCUR-
55. (PREF. TERESINA-PI – PROFESSOR – NUCE- SOS/2012) A construção de uma casa é realizada em 10
PE/2016) Sabendo que o comprimento do muro Parque dias por 30 operários trabalhando 8 horas por dia. O nú-
Zoobotânico é de aproximadamente 1,7 km e sua altura mero de operários necessários para construir uma casa
é de 1,7 m, um artista plástico pintou uma área corres- em 8 dias trabalhando 6 horas por dia é
pondente a 34 m² do muro em 8 horas trabalhadas em
um único dia. Trabalhando no mesmo ritmo e nas mes- a) 18.
mas condições, para pintar este muro, o pintor levará b) 24.
c) 32.
a) 83 dias. d) 38.
b) 84 dias. e) 50.
c) 85 dias.
d) 86 dias. 60.(VUNESP – PMESP – CURSO DE FORMAÇÃO DE
e) 87 dias. OFICIAIS – 2014) A tabela, com dados relativos à cidade
de São Paulo, compara o número de veículos de frota, o
56. (SES-PR – TÉCNICO DE ENFERMAGEM – número de radares e o valor total, em reais, arrecada-
UFPR/2009) Uma indústria metalúrgica consegue pro- do com multas de trânsito, relativos aos anos de 2004 e
duzir 24.000 peças de determinado tipo em 4 dias, tra- 2013:
balhando com seis máquinas idênticas, que funcionam 8
horas por dia em ritmo idêntico de produção. Quantos Ano Frota Radares Arrecadação
dias serão necessários para que essa indústria consiga
produzir 18.000 peças, trabalhando apenas com 4 dessas 2004 5,8 milhões 260 328 milhões
máquinas, no mesmo ritmo de produção, todas elas fun- 2013 7,5 milhões 601 850 milhões
cionando 12 horas por dia?
Se o número de radares e o valor da arrecadação ti-
a) 3. vessem crescido de forma diretamente proporcional ao
b) 4. crescimento da frota de veículos no período considera-
c) 5. do, então em 2013 a quantidade de radares e o valor
d) 6. aproximado da arrecadação, em milhões de reais (des-
e) 8. considerando-se correções monetárias), seriam, respec-
tivamente,
MATEMÁTICA

a) 336 e 424.
b) 336 e 426.
c) 334 e 428.
d) 334 e 430.
e) 330 e 432.

81
43 B
GABARITO 44 C
45 B
1 B 46 C
2 C 47 ERRADO
3 D 48 B
4 A 49 C
5 E 50 C
6 C 51 A
7 B 52 A
8 D 53 C
9 A 54 C
10 C 55 C
11 B 56 A
12 E 57 A
13 C 58 E
14 B 59 E
15 C 60 A
16 B
17 D
18 A
19 B
20 D
21 C
22 C
23 D
24 D
25 A
26 C
27 B
28 E
29 A
30 B
31 A
32 A
33 B
34 E
35 D
36 B
37 C
38 C
39 B
MATEMÁTICA

40 A
41 A
42 C

82
ÍNDICE

CONHECIMENTOS GERAIS
São José dos Pinhais: fundação, colonização, emancipação política; fatos e aspectos históricos, características geográficas, cul-
turais, econômicos e étnicos. Atualidades políticas, econômicas e sociais do Brasil e do Paraná..........................................................01
SÃO JOSÉ DOS PINHAIS: FUNDAÇÃO, COLONIZAÇÃO, EMANCIPAÇÃO POLÍTICA; FATOS E
ASPECTOS HISTÓRICOS, CARACTERÍSTICAS GEOGRÁFICAS, CULTURAIS, ECONÔMICOS E
ÉTNICOS. ATUALIDADES POLÍTICAS, ECONÔMICAS E SOCIAIS DO BRASIL E PARANÁ.

Fundação

O processo de fundação de São José dos Pinhais, deve ser analisada em uma perspectiva bastante ampla, avaliando
o território em paralelo com a colonização do estado do Paraná e também do Brasil. No início do processo de coloniza-
ção, as tribos indígenas que habitavam a região pertencente ao município, não faziam uso de fronteiras entre as terras,
o único fator de divisão era quando encontrava as tribos.
Dentro das tribos existentes na região, destacam-se os tupi-guarani, sendo formados pelos Tupiniquins e Carijós,
ocupando a região litorânea e de serras, e os Botocudos, agrupamento indígena da família dos Jê, que acabaram por
ocupar mais as regiões de planalto. Os portugueses ao começarem o processo de ocupação do território, atribuíram
aos Tuipi-Guaranis um caráter de maior evolução, pois mesmo vivendo em uma característica primitiva fazendo uso
de Pedra Polida, já não eram totalmente nômades, começando a fazer uso da agricultura. Já os Botocudos, eram mais
hostis e tiveram grandes enfrentamentos com os colonizadores nos séculos XVI e XVII.

#FicaDica
A formação tribal que compõe a região que vai originar São José dos Pinhais, são recorrentes em várias
bancas do país, então destine um tempo para entender quem formou inicialmente esse território.

No advento do século XVI e XVII, foram registrados os primeiros centros populacionais oficiais, que visavam encon-
trar ouro na região. Nessa busca por metais às margens do rio Arraial, começou a instalação de igrejas, e com isso, os
primeiros colonizadores portugueses começaram a efetivar a sua ocupação formando na região uma Vila, ainda não
emancipada.
Após um período de grande estagnação na região, nos entornos do rio Arraial, em meados do XIX, a província
conseguiu sua independência política perante a Curitiba no ano de 1853, porém foi a partir de 1878 que o fluxo de
imigrantes Europeus passou a entrar massivamente no território, instaurando assim, o início do processo de ocupação
efetiva na imigração europeia no município.

FIQUE ATENTO!
A origem da ocupação dessa região, remonta a busca por metais preciosos, logo, entenda que os Bandei-
rantes participaram desse processo de entrada interiorana.

EXERCÍCIOS COMENTADOS

1. No processo de colonização do estado do Paraná, encontra-se a formação de São José dos Pinhais, a colonização
desse território foi realizada por Europeus, porém alguns troncos linguísticos já ocupavam esses territórios. Qual das
alternativas abaixo, compreende a verdadeira ocupação desse povoamento.

a) Tupi-Guarani e Jê
b) Apurinã e Tupi-Guarani
c) Ticuna e Jê
d) Apurinã e Ticuna
CONHECIMENTOS GERAIS

Resposta: Letra A. Apenas o tronco linguístico Jê e Tupi-Guarani ocuparam efetivamente o território onde se forma
São José dos Pinhais. As tribos Apurinã e Ticuna, são características da porção Norte do Brasil e não do Sul do país.

1
2. “A formação efetiva do município de São José dos Pi-
nhais, está intimamente vinculado busca por metais às
margens do rio Arraial, e partir disso, começou a instala- EXERCÍCIO COMENTADO
ção de igrejas, iniciando o processo colonizador.” A cita-
ção acima de certa ou errada? 1. Qual foi o fator decisivo para o aumento do fluxo mi-
gratório para os entornos de São José dos Pinhais. Assi-
( ) CERTO ( ) ERRADO nale a alternativa correta.

Resposta: Certo. Realmente o processo de ocupação a) A grande mescla cultural e aceitação da diversidade.
da região, ocorre por viés da busca por metais às mar- b) Forte ase agrária da cidade de São José.
gens do rio Arraial, sendo assim, o item acima correto. c) Estímulo do governo do Estado do Paraná.
d) Criação da CIC trouxe ondas migratórias definitivas
para cidade.
Colonização d) A manutenção do perfil interiorano, atraindo mais ido-
sos para a região.
O processo de efetivação da colonização imigrante
no município de São José dos Pinhais, foram realizados Resposta: Letra D. O fator primordial para a migra-
pelos Europeus, mas devemos lembrar que, eles não ex- ção para a região de São José dos pinhais, refere-se
cluíram os nativos, na verdade, os índios foram absorvi- a edificação CIC, pois depois dessa criação, a migra-
dos pelos colonizadores, e também, os escravos africanos ção se tornou intensa, até porque, São José está muito
foram inseridos na formação cultural, social, econômica e próxima da capital do estado, e esses avanços atraem
religiosa do município. diversos públicos para a cidade.

#FicaDica Emancipação Política


A miscigenação aplicada no processo de A cidade de São José dos Pinhais é um dos poucos
formação da colonização de São José dos municípios criados antes da Proclamação da República no
Pinhais, não segue a mesma lógico até seu Brasil, aliás, se tornou um Município meses antes do Paraná
final, pois com a intensificação da imigra- deixar de pertencer a São Paulo e se tornar uma Província.
ção europeia, os grupos menos favorecidos
acabaram perdendo um pouco de espaço
na região e no fomento cultural. #FicaDica
Província, era o termo usado na época do
Após o município estar edificado, começa um proces- Regime Imperial no Brasil para designar
so de fortalecimento na cidade, pois sempre existe um Estado, sendo assim, entender essa termo-
processo de amadurecimento para que a região se tor- logia possibilita que você não cometa um
ne efetivamente atrativa, no contexto de São José dos erro de interpretação nas questões no dia
Pinhais não foi diferente, que mesmo emancipado po- da prova.
liticamente em 1853, viu o município começar a crescer
efetivamente após a segunda metade do século XX. A
partir nesse impulso, começa a chegar na região mais de-
senvolvimento, e isso, trouxe mais pessoas para a região. O processo de emancipação política de São José dos
Quando foi criado a CIC – Cidade Industrial de Curiti- Pinhais, ocorreu no dia 08 de janeiro de 1853, e neste ano
ba, o município de São José dos Pinhais começou a rece- completou 166 anos de sua efetiva emancipação, devido
ber mais ondas de ocupação, pois com a proximidade da ao seu desenvolvimento sócio-político. Desde o século
capital, diversas pessoas do interior do Estado do Paraná, XVIII, a Câmara de Curitiba possui entre os seus edis (ve-
e também, de outros estados, começaram a migrar para readores) representantes políticos de São José dos Pi-
o município, mudando o cenário da cidade, tirando ela nhais. Contudo, eles queriam mesmo é que a freguesia
de um perfil interiorano e transformando-a, em uma ci- passasse a ser independente de Curitiba. Isto daria mais
dade com crescente expansão. autonomia política para a localidade, bem como mais
poder aos seus representantes políticos.
Essa disputa por poder ficou clara na história da insta-
CONHECIMENTOS GERAIS

FIQUE ATENTO! lação da Câmara de Vereadores de São José dos Pinhais,


O fluxo migratório para a região que com- quando das desavenças políticas entre os líderes locais
põe São José dos Pinhais e Curitiba, estão da época, resultando inclusive em tiros e mortes em fren-
intimamente ligados com CIC, afinal, ela te à Matriz da cidade.
abriu as portas da região para os avanços e Essa organização política, fez então do município de
capitais industrializados. São José dos Pinhais um importante centro político no
Estado do Paraná e no Brasil, pois sua formação política
e social seguiu uma ótica de absorção de públicos múlti-
plos padrões linguísticos, várias etnias e culturas.

2
entre 200m e 1.300m. Desta forma, São José dos Pinhais
está localizada entre o Primeiro Planalto Paranaense e a
EXERCÍCIO COMENTADO Serra do Mar.
Em critérios geológicos, os terrenos do município são
1. São José dos Pinhais foi um dos poucos municípios de origem quaternário-pleistocênicas, quaternário-ho-
criado antes da proclamação da República, além disso, locênicas, arqueano-proterozoicas, proterozoico-cam-
ele consegue sua emancipação, antes mesmo do estado brianas, paleozoico-cambrianas e mesozoico-jurássico-
do Paraná se tornar uma província. -cretáceas. Já os tipos de solo existentes no município
são: gleissolo melânico, organossolo mésico, latossolo
( ) CERTO ( ) ERRADO bruno, argissolo vermelho-amarelo, latossolo vermelho,
cambissolo háplico, afloramentos de rocha e latossolo
Resposta: Certo. Realmente, o município de São José vermelho-amarelo.
dos Pinhais é um ponto fora da curva dentro do pro- O município é totalmente cortado por ribeirões e
cesso brasileiro e paranaense, afinal, se torna muni- córregos, mas seu principal acidente geográfico é o rio
cípio antes da Republica e da sua própria capital ser Iguaçu, cuja sua nascente se verifica no município vizi-
emancipada de São Paulo e se torna uma Província. nho de Piraquara e lhe serve de limite com o município
de Curitiba. Os principais rios afluentes do rio Iguaçu em
território são-joseense são os rios: Imbaú da Prata, Gua-
Fatos e aspectos históricos ratubinha, Arraial dos Quatis, Castelhanos, Capivari e São
João correm da Serra do mar para o Oceano Atlântico, e
O processo de criação do município de São José dos o ribeirão da Onça é o único tributário do rio da Várzea.
Pinhais foi efetivado pela Lei n°10, da então província de No processo de ocupação do Brasil em 1500, o terri-
São Paulo, no dia 16 de julho de 1852, na qual defini- tório correspondente ao município era coberto por for-
ria que a sede do município seria chamada de Villa de mações vegetais originais, sendo elas: estepe gramíneo
São José dos Pinhais, porém, o estabelecimento político lenhosa, floresta ombrófila mista e floresta ombrófila
efetivo, ocorreu apenas em janeiro de 1853, quando foi densa. Já fazendo referência a base econômica, a argila
instalado a Câmara dos Vereadores. constitui a principal riqueza natural do município, sendo
Essa emancipação política e administrativa de São fonte direta de lucro.
José dos Pinhais, fez dela uma importante base econô-
mica para o estado do Paraná e para o Brasil, pois ela
passou a receber muitos eixos de desenvolvimento. Esse #FicaDica
desenvolvimento aprece mais no aspecto econômico,
afinal, o parque industrial e montagens no segmento As primeiras bases econômicas do território
automobilístico foi muito forte na região, gerando carga que formam São José dos Pinhais, estão vin-
tributária e muitos empregos. culadas as riquezas de ordem natural, sen-
do elas: a vasta vegetação e a argila. Em es-
pecial, quando analisar a ARGILA, lembre-se
FIQUE ATENTO! que ela foi uma importante fonte de lucro
É preciso diferenciar a criação do muni- nos primeiros fomentos colonizadores, pos-
cípio de sua efetiva emancipação, pois o sibilitando a formação de grandes cerâmi-
município foi criado em 1852, mas a efeti- cas que proporcionaram avanços a região.
va emancipação, ocorreu apenas em 1853,
com a instalação da Câmara dos Vereado-
res, estabelecendo um poder organizado e
independente.
EXERCÍCIOS COMENTADOS

Características Geográficas 1. “O município de São José dos Pinhais não é cortado


por nenhum rio, fazendo do município uma cidade que
O município de São José dos Pinhais está localiza- não utiliza os rios para exatamente.” Afirmação é correta
do na Região Metropolitana de Curitiba, fazendo limi- ou errada?
tes com Pinhais e Piraquara ao Norte, Tijucas do Sul ao
Sul, Morretes a Guanabara a Leste, Fazenda Rio Grande ( ) CERTO ( ) ERRADO
CONHECIMENTOS GERAIS

e Mandirituba a oeste. A sede do poder municipal está


instalada nas seguintes coordenadas: 25º32’06” de lati- Resposta: Errado. O item acima está completamen-
tude sul e 49º12’21” de longitude oeste do meridiano de te ERRADO, afinal, o município é totalmente cortado
Greenwich, ligando à capital estadual pela BR-376 em um por ribeirões e córregos, possibilitando uma forte pro-
percurso de 7km. O território total do município com- dução agrícola, fomento esse que é importantíssimo
preende uma área de 931,73 km². para o CEASA de Curitiba.
A altitude é de 906 metros, na sede municipal, sen-
do o relevo formado pelo talvegue do Iguaçu a oeste
e pela Serra do Mar a leste, com altitudes que oscilam

3
2. Seguindo a ótima da formação geográfica de São José Sendo assim, tomando como base a pesquisa de
dos Pinhais, identifique o item que não corresponde a mestrado de Myriam Sbravati, São José dos Pinhais, 1776-
sua formação inicial. 1852 – uma paróquia paranaense em estudo (1980), em
1776 a população escrava negra era de 23,1% (270 in-
a) Estepe gramíneo lenhosa divíduos). Um ano depois de São José se tornar Municí-
b) Floresta ombrófila mista pio, em 1854, a população escrava era de 7,9%, ou seja,
c) Mangues Vermelhos 365 pessoas. Mais tarde, poucos anos antes da liberta-
d) Floresta ombrófila densa ção definitiva dos escravos pela princesa Isabel, São José
possuía, em 1863, 769 escravos, representando 10,18%.
Resposta: Letra C. Os Mangues Vermelhos não são Seguinte a ótica da autora acima, a escravidão existiu sim
típicos da região Sul do país, sendo assim, ele é consi- em São José dos Pinhais, e como reflexo disso, a cultura
derado um ecossistema costeiro de transição entre os Africana foi inserida nesse município paranaense.
ambientes terrestre e marinho. Característico de regi- Mas lembre-se, as atividades executadas aqui, eram
ões tropicais e subtropicais, está sujeito ao regime das bem diferentes dos senhores de engenho de açúcar do
marés, dominado por espécies vegetais típicas, às quais Nordeste, Rio de Janeiro e São Paulo, contudo, sendo es-
se associam a outros componentes vegetais e animais. cravos, precisariam fazer todo tipo de trabalho pesado,
como desmatar florestas, cortar lenha, fazer serviços do-
mésticos, fazer roçadas e plantações, etc.
Culturais

Na formação cultura de São José dos Pinhais, é pre- FIQUE ATENTO!


ciso lembrar do forte processo de miscigenação exis- No processo colonizador, podemos obser-
tente na formação do povoado, sendo assim, podemos var várias culturas formando o território e
observar uma grande presença da cultura europeia, uma as características de sua população, mas ao
influência de alguns aspectos africanos e algumas carac- longo dessa edificação, as culturas de viés
terísticas culturais referentes aos nativos que ocuparam o EUROPEU, foram mais privilegiadas e exal-
território em seu processo inicial. Porém, ainda não existe tadas, deixando as contribuições negras e
pesquisas massivas com relação a forte existência cultural índias um pouco afastadas desse fomento
das populações que estão fora do eixo europeu. Muitos cultural.
escritores, dentre eles o famoso Romário Martins, chega-
ram mesmo a defender um “branqueamento” da história
do Estado, esse evento foi comum no Brasil, e não foi di-
ferente em São José dos Pinhais. Uma grande injustiça a Adentrando agora nos eventos culturais que irão
quem teve que vir milhares de léguas da África até aqui e marcar a cidade, podemos destacar os seguintes eventos:
servir de escravo para os mais diversos trabalhos forçados,  Carnaval de Bonecos: esse evento acontece sem-
e ainda, não ter o reconhecimento deste fato, tendo inclu- pre uma semana antes do Carnaval, sendo formado
sive sua contribuição cultural deixada de lado. por um tradicional desfile de bonecos. Os bonecos
utilizados nessa festividade, também acabam sen-
do usados em Antonina na semana do Carnaval,
demarcando uma grande tradição cultural do mu-
nicípio.
 Festa da Cidade e Festa do Pinhão: a festa do
Pinhão entra para o roteiro paranaense por ser um
grande evento musical, trazendo nomes de reno-
me nacional como Tchê Garotos, Raimundos, Os
Serranos e outros grupos musicais de grande re-
putação nacional, isso tudo, com preço popular em
seus ingressos, mostrando uma grande mistura de
ritmos musicais em uma mesma festividade.
 Festa da Colheita, da Murici, do Morango: são
grandes festas que agitam a região das colônias
com muita comida e tradição das colônias euro-
peias que por lá vivem, sendo assim, é a festivi-
dade com maior tradição europeia que acontece
CONHECIMENTOS GERAIS

em São José dos Pinhais, o que mostra o quanto


eles ainda são influenciados pelo seu processo de
povoamento.

A mão-de-obra escrava negra, de origem africana, tam-


bém foi utilizada e explorada em SJP
[Imagem: Alberto Henschel]

4
#FicaDica FIQUE ATENTO!
Na formação econômica contemporânea
O movimento cultura mais evidente do mu-
de São José dos Pinhais, temos que nos
nicípio, acaba sendo eventos de vinculações
atentar para sua importância enquanto eixo
locais, criando uma cultura mais regionalis-
comercial, centro de produção industrial no
ta e focada no processo de aculturação da
segmento automobilístico, a indústrias de
cidade. Todos os eventos destacados acima,
cosméticos e no setor de alimentação. Essa
priorizam o fortalecimento de uma cultura
formação, deixa o polo industrial do muni-
local.
cípio muito completo e atraente a muitos
mercados nacionais e internacionais.

EXERCÍCIO COMENTADO O modelo agrícola da cidade também recebe desta-


que, sendo o município de São José dos Pinhais o maior
1. Em São José dos Pinhais, nunca houve uso de mão de produtor de OLERÍCOLAS da região, fornecendo princi-
obra escrava, sendo assim, os africanos não fizeram parte palmente mercadorias para o CESA de Curitiba.
da colonização do município. Segundo os dados do último sendo do IBGE, a eco-
nomia são joseense está situada como SEXTA economia
( ) CERTO ( ) ERRADO dentro do Estado do Paraná e a SEGUNDA na sua micror-
região, mostrando a forte força econômica do município
Resposta: Errado. O item acima está INCORRETO, frente ao processo de desenvolvimento do seu estado.
pois houve sim o uso de mão de obra escrava em São Com relação ao salário médio da população, segundo o
José dos Pinhais, chegando a atingir 10,18%. DA PO- senso do IBGE, o salário médio mensal era de 3.4 salários
mínimos. A proporção de pessoas ocupadas em relação
PULAÇÃO às vésperas da Lei Áurea, sendo assim, eles
à população total era de 33.7%. Na comparação com os
participaram sim desse processo colonizador.
outros municípios do estado, ocupava as posições 4 de
399 e 32 de 399, respectivamente. Já na comparação com
cidades do país todo, ficava na posição 81 de 5570 e 317
Econômicos
de 5570, respectivamente. Considerando domicílios com
rendimentos mensais de até meio salário mínimo por pes-
A cidade de São José dos Pinhais possui uma eco-
soa, tinha 28.2% da população nessas condições, o que o
nomia influenciada pela presença do Aeroporto Interna- colocava na posição 338 de 399 dentre as cidades do esta-
cional Afonso Pena, e da instalação de grandes fábricas do e na posição 5001 de 5570 dentre as cidades do Brasil.
de autopeças, que foram edificadas no mesmo período
em que as montadoras de automóveis se estabeleceram
#FicaDica
na região, bem como suas fornecedoras. Ocupa a ter-
ceira colocação em nível Brasil quando se refere a polo A produção econômica é extremamente
automotivo, abrigando as seguintes montadoras: Audi, eficiente, no qual, também ganha destaque
Nissan, Renault e Volkswagen. na produção agrícola, sendo a principal for-
Mas não é apenas da linha de montagem que vive necedora de alimentos para abastecimento
o município, a cidade também é famosa por ser sede de do CEASA de Curitiba, logo, São José dos
uma das mais importantes redes de cosméticos do país, Pinhais passa por vários processos econô-
O Boticário, e além destas, tem em suas delimitações micos com uma grande competência.
territoriais a empresa Nutrimental, que atua no setor de
alimentos.
A base econômica de São José dos Pinhais passa tam-
bém pelo comércio, sendo ele, autônomo em relação à
capital. O centro da cidade, está apenas a alguns quilô- EXERCÍCIO COMENTADO
metros do grande polo comercial municipal, contando
com grandes supermercados, estrutura de shopping cen- 1. A edificação econômica do município de São José
ter muito grande, no qual, pode se encontrar uma gran- dos Pinhais segue um layout de obter uma amplitude
de variedade de lojas. Esse centro comercial do municí- em suas áreas de atuação, sendo assim, ele atuou em
pio fica situada nas imediações da rua XV de Novembro.
CONHECIMENTOS GERAIS

diversos segmentos econômicos de maneira simultânea.


Senda essa vertente, identifique o item no qual, São José
dos Pinhais não atua.

a) Produção olerícola
b) Montadoras de automóveis
c) Extração de borracha
d) Produção de cosméticos

5
Resposta: Letra C. Dentro dos itens apresentados, a
única área na qual São José não atua, é na extração de #FicaDica
borracha, nas demais áreas ele tem uma forte estrutu-
ra produtiva. São joseense irão receber várias ondas mi-
gratórias europeias, e foi por esse motivo,
que existe nos arredores do município essa
Étnicos concepção de branqueamento no imaginá-
rio social, mas não se esqueça, não foram
São José dos Pinhais tem sua origem vinculada a ele- apenas os Europeus que ocuparam o muni-
mentos religiosos e geográficos, pois homenageia o seu cípio e seus entornos.
santo padroeiro, São José, e também, destacam a forma-
ção de sua vegetação, que possui grandes extensões de
pinheiros, cobrindo todo o território municipal. A episte- Atualidades políticas, econômicas e sociais do Bra
mologia da palavra José, tem sua origem da civilização sil (janeiro/2019)
hebraica, e significa “YOSEF”, significando “QUE DEUS
MULTIPLIQUE”, posteriormente, foi latinizado e aos pou- Novo presidente da República
cos ficou “JOSEPH”. São José na história, era um carpin-
teiro em Nazaré, e desempenhou o papel de pai de Jesus, Em janeiro teve início o mandato do 38° presidente
por ser carpinteiro, é também, o padroeiro de todos que do Brasil, Jair Messias Bolsonaro executará seu mandado
trabalham com madeira. entre 2019 e 2022. A eleição do candidato, marca uma
Essa vinculação religiosa e geográfica do município, evolução no processo de fazer política no Brasil, pois
mostra o quanto a influência europeia por viés dos colo- consegue sua eleição por viés das redes sociais e não das
nizadores e dos jesuítas foi efetiva na região, desta for- mídias televisas como de costume, além disso, ele se tor-
ma, existe um forte panorama católico na cidade. nou um ícone político do antipetismo e da luta contra a
corrupção. Em seu discurso presidencial, fez uma fala em
torno de 10 minutos e reafirmou sua missão enquanto
FIQUE ATENTO! presidente, que era livrar o país da corrupção e submis-
A mistura de elementos geográficos e re- são ideológica.
ligiosos que originam o município é um
conteúdo bastante recorrente por algumas Rompimento da barragem de Brumadinho
bancas, logo precisa compreender que São
José tem apelo católico no fomento coloni- Uma barragem da mina localizada no Córrego do Fei-
zador e Pinhais é oriundo da grande quan- jão, de extração realizada pela mineradora Vale, localiza-
tidade de pinheiros existente na região. da em Minas Gerais, na cidade de Brumadinho se rom-
peu no final de janeiro. A onda de rejeitos de minério de
ferro atingiu a sede administrativa da empresa e chegou
No processo formado da contemplação étnica são até a comunidade da Vila Ferteco, deixo mortos, feridos
joseense, podemos destacar os seguintes grupos: índios e muitos desaparecidos.
(os primeiros habitantes da região e do território do mu-
nicípio), os portugueses (colonizadores que ocuparam o O caso Damares Alves
Estado do Paraná e desenvolveram São José dos Pinhais),
posteriormente, ocorre a entrada dos escravos (esses de Pouco tempo depois de Jair Bolsonaro tomar posse
origem Africana, tendo uma grande representatividade de seu cargo, ele selecionou os seus respectivos minis-
populacional na época), e por fim, ocorre a chegada dos tros para atuar na nova gestão administrativa do Brasil.
imigrantes europeus, sobretudo poloneses, italianos e Dentro dos novos ministros, encontra-se Damares Alves,
ucranianos (esses agrupamentos da Europa, acabam por que foi empossada como ministra da Mulher, Família e
fortalecer uma visão de região com padrões étnicos de Direitos Humanos, a então ministra, casou muita polêmi-
“BRANQUEAMENTO POPULACIONAL”, passando a exal- ca com um vídeo dela que passou a circular nas redes, no
tar mais a cultura europeia, do que as demais existentes qual, ela define que meninos veste azul e meninas vestem
no munícipio). rosa. A declaração causou revolta e mobilizou as redes
O crescimento populacional é extremamente visível, sociais de pensamentos contrários ao da ministra. Inclu-
segundo os sensos do IBGE no de 2010 tinha uma popu- sive muitas celebridades tomaram partido repudiando a
lação estimada em 264.210 habitantes, no ano de 2015 já afirmação. Damares é educadora, advogada e evangélica,
e em seu discurso de posse também usou os dizeres: “o
CONHECIMENTOS GERAIS

possuía uma população estimada em 297.895 e em 2019,


sua população estimada está com 322.235 habitantes. Estado é laico, mas esta ministra é terrivelmente cristã”.
Desta forma, os dados em destaque mostram o quanto o
município vem crescente em índices populacionais, esse Pose de armas em debate
crescimento, está intimamente ligada à sua edificação
econômica, e ao fato, de muitos moradores residirem em O novo presidente do Brasil, assinou no mês de janei-
São José dos Pinhais e realizarem o movimento pendular ro o polêmico projeto para flexibilização a posse e co-
até a capital para trabalhar. mercialização de armas de fogo no Brasil, essa era uma
de suas promessas na plataforma política que levou o

6
mesmo a eleição. A questão em debate, refere-se ao di-
reito à posse de armas, ou seja, uma autorização para
manter a arma de fogo em casa ou em seu local de tra- HORA DE PRATICAR!
balho, desde que, o dono da arma seja o responsável
pelo estabelecimento. O que muita gente confunde é 1. A formação do nome são José dos Pinhais, tem sua
com relação ao direito ao porte, neste caso, as regras são origem nos aspectos religiosos e geográficos. Afirmação
bem mais rigorosas e não foram abortadas no referido é considerada correta?
decreto.
( ) CERTO ( ) ERRADO
Jean Wyllys e a desistência do seu mandato
2. Qual foi o ano de fundação do município de São José
O deputado federal Jean Wyllys, do PSOL-RJ, que as- dos Pinhais com sua sede administrativa?
sumia em 2019 seu terceiro mandato consecutivo com
mais de 24.000 votos, informou estava abrindo mão do a) 1857
mandado para construir sua vida fora do Brasil. O receio b) 1851
do deputado era ter um fim trágico como da Marielle c) 1853
Franco, então mediante as ameaças apresentadas por e) 1854
ele, optou por sair do país e garantir sua segurança. O
Ministério da justiça e Segurança Pública lamentou o 3. Dentro das ondas de imigrantes que formaram o mu-
ocorrido e a opção do deputado federal, e afirmou que a nicípio de São José dos Pinhais, imigrantes de diversas
Polícia federal abriu inquéritos para investigar as amea- partes da Europa vieram para cá. Assinale o grupo de
ças sofridas pelo parlamentar. imigrantes que não veio para os entornos do município.

Flávio Bolsonaro em evidência a) Poloneses


b) Italianos
O então deputado estadual há quatro mandados se- c) Russos
guidos, que recentemente foi eleito senador e é filho do d) Ucranianos
atual presidente da República, se tornou o pivô de uma e) Portugueses
série de movimentações financeiras suspeitas ocorridas
em 2017. O fato mencionou, destravou um gatilho de 4. No ato da criação do município de São José dos Pi-
pesquisas e procura por demais irregularidades sobre nhais, foi assinado uma lei que efetivava o processo de
ele e sua família, ele na ocasião está sendo acusado de emancipação do município. Seguindo essa perspectiva,
lavagem de dinheiro e ocultação de bens, dando muitas qual foi o número da Lei que promoveu o processo de
evidencias para o motorista da família, conhecido como autonomia ao município?
Fabrício Queiroz.
a) Lei n°10
b) Lei n° 9
c) Lei n° 8
d) Lei n° 7
e) Lei n° 6

GABARITO

1 CERTO
2 C
3 C
4 A
CONHECIMENTOS GERAIS

7
ANOTAÇÕES

_______________________________________________________________________________________________________

_______________________________________________________________________________________________________

_______________________________________________________________________________________________________

_______________________________________________________________________________________________________

_______________________________________________________________________________________________________

_______________________________________________________________________________________________________

_______________________________________________________________________________________________________

_______________________________________________________________________________________________________

_______________________________________________________________________________________________________

_______________________________________________________________________________________________________

_______________________________________________________________________________________________________

_______________________________________________________________________________________________________

_______________________________________________________________________________________________________

_______________________________________________________________________________________________________

_______________________________________________________________________________________________________

_______________________________________________________________________________________________________

_______________________________________________________________________________________________________

_______________________________________________________________________________________________________

_______________________________________________________________________________________________________

_______________________________________________________________________________________________________

_______________________________________________________________________________________________________

_______________________________________________________________________________________________________

_______________________________________________________________________________________________________

_______________________________________________________________________________________________________

_______________________________________________________________________________________________________

_______________________________________________________________________________________________________

_______________________________________________________________________________________________________

_______________________________________________________________________________________________________

_______________________________________________________________________________________________________
CONHECIMENTOS GERAIS

_______________________________________________________________________________________________________

_______________________________________________________________________________________________________

_______________________________________________________________________________________________________

_______________________________________________________________________________________________________

_______________________________________________________________________________________________________

8
ÍNDICE

NOÇÕES DE INFORMÁTICA
Correio Eletrônico: conceitos; Gerenciador de e-mail (Outlook Express); ........................................................................................................01
BrOffice Documento Texto (Writer): Atalhos e barra de ferramentas; Modos de seleção de texto; Formatação de texto; Formatação
de parágrafos; Alinhamento; ..............................................................................................................................................................................................03
BrOffice Planilha (Calc): Atalhos e barra de ferramentas; Formatação de dados; Seleção de células; Atributos de caracteres .....23
Na página Configuração Automática de Conta, insira o
CORREIO ELETRÔNICO: CONCEITOS; seu nome, endereço de email e senha e escolha Avançar.
GERENCIADOR DE E-MAIL (OUTLOOK Observação : Se você receber uma mensagem de erro
EXPRESS); após escolher Avançar, verifique novamente seu endere-
ço de email e senha. Se ambos estiverem corretos, esco-
lha Configuração manual ou tipos de servidor adicionais..
OUTLOOK Escolha Concluir.

O Microsoft Outlook é um software de automação de A configuração automática não funcionou


escritório e cliente de correio electrónico que faz parte Se a instalação não tiver sido concluída, o Outlook
do pack Microsoft Office. Mas não é apenas um geren- pode solicitar que você tente novamente usando uma
ciador de e-mails, o Outlook possui funcionalidades de conexão não criptografada com o servidor de email. Se
grupos de discussão com base em padrões avançados de isso não funcionar, escolha Configuração manual ou ti-
internet, além de possuir calendário integral e gerencia- pos de servidor adicionais.
mento de tarefas e de contatos. Além disso, possui ca- Se você atualizar para o Outlook 2016 a partir de uma
pacidades de colaboração em tempo de execução e em versão anterior e receber mensagens de erro informando
tempo de criação robustos e integrados. que não é possível fazer logon ou iniciar o Outlook, é
porque o serviço Descoberta Automática do Exchange
Configurando uma conta1 não foi configurado ou não está funcionando correta-
Em muitos casos, o Outlook pode configurar a sua mente.
conta para você apenas com um endereço de email e
uma senha. Ao iniciar o Outlook pela primeira vez, o As- Para configurar uma conta manualmente
sistente Automático de Conta é iniciado. Escolha Configuração manual ou tipos de servidor
adicionais > Avançar.
Para configurar uma conta automaticamente
Abra o Outlook e, quando o Assistente Automático de
Conta for aberto, escolha Avançar.
Observação : Se o Assistente não abrir ou se você
quiser adicionar uma outra conta de email, na barra de
ferramentas, escolha a guia Arquivo.

Selecione o tipo de conta desejado e escolha Avançar.


Preencha as seguintes informações:
Seu Nome, Endereço de Email, Tipo de Conta, Servi-
Na página Contas de Email, escolha Avançar > Adi- dor de Entrada de Emails, Servidor de Saída de Emails,
cionar Conta. Nome de Usuário e Senha.
Escolha Testar Configurações da Conta para verificar
as informações inseridas.
Observação : Se o teste falhar, escolha Mais Configu-
rações. O administrador poderá solicitar que você faça
outras alterações, incluindo inserir portas específicas
para o servidor de entrada (POP3 ou IMAP) ou o servidor
de saída (SMTP).
Escolha Avançar > Concluir.
NOÇÕES DE INFORMÁTICA

1 Fonte: Ajuda do MSOutlook

1
Excluir uma conta de email Criar uma mensagem de email
Para excluir uma conta de email Clique em Novo Email, ou pressione Ctrl + N.
No painel direito da guia Arquivo, selecione Configu-
rações de Conta > Configurações de Conta.

Se várias contas de email configuradas no Microsoft


Outlook, o botão de aparece e a conta que enviará a
mensagem é mostrada. Para alterar a conta, clique em
de.
Na caixa Assunto, digite o assunto da mensagem.
Insira os endereços de email dos destinatários ou os
nomes na caixa Para, Cc ou Cco. Separe vários destinatá-
rios com um ponto e vírgula.
Para selecionar os nomes dos destinatários em uma
lista no Catálogo de Endereços, clique em Para, Cc ou
Cco e clique nos nomes desejados.
Não vejo a caixa Cco. Como posso ativá-lo?
Para exibir a caixa Cco para esta e todas as mensagens
futuras, clique em Opções e, no grupo Mostrar Campos,
clique em Cco.
Clique em Anexar arquivo para adicionar um anexo.
Na lista de contas de email, selecione a conta que de- Ou clique em Anexar Item para anexar itens do Outlook,
seja excluir e escolha Remover. como mensagens de email, tarefas, contatos ou itens de
calendário.

#FicaDica
Se você não gostar a fonte ou o estilo
do seu email, você pode alterar sua aparência.
Também é uma boa ideia Verificar a ortografia
em sua mensagem antes de enviar.

Após terminar de redigir sua mensagem, clique em Enviar.


Obs.: Microsoft Exchange Server é uma aplicação ser- Observação : Se você não consegue encontrar o bo-
vidora de e-mails de propriedade da Microsoft Corp e tão Enviar, talvez você precise configurar uma conta de
que pode ser instalado somente em plataformas da fa- email.
mília Windows Server.
Pesquisar email
NOÇÕES DE INFORMÁTICA

Da Caixa de Entrada, ou de qualquer outra pasta de


email, localize a caixa Pesquisar na parte superior de suas
mensagens.
Para encontrar uma palavra que você sabe que está
em uma mensagem ou uma mensagem de uma pessoa
em particular, digite a palavra ou o nome da pessoa na
caixa Pesquisar. Mensagens que contenham a palavra ou
o nome que você especificou serão exibidas com o texto
de pesquisa destacado nos resultados.

2
Restrinja os resultados de pesquisa Ctrl + Shift + M: criar nova mensagem
No grupo Escopo na faixa de opções, escolha onde Se você vai mandar um e-mail para alguém, use o ata-
você deseja pesquisar: Todas as Caixas de Correio, Caixa lho Ctrl + Shift + M para criar uma nova mensagem.
de Correio Atual, Pasta Atual, Subpasta ou Todos os Itens
do Outlook. Resposta: D
No grupo Refinar na faixa de opções, escolha se você
está procurando pela pessoa que enviou a mensagem ou 02. (Prefeitura de Matozinhos - MG - Assistente Ad-
pelo assunto. ministrativo – Médio – FUMARC – 2016) - Adaptado.
Você pode filtrar ainda mais os resultados da pesqui- Na janela de edição de mensagens do Microsoft Outlook,
sa ao selecionar: versão português do Office 2010, uma mensagem pode
Com Anexos – para localizar somente emails com ser marcada com “Alta Prioridade” na guia “Marcas” pelo
anexos ícone:
Categorizado – para localizar emails que tenham sido
atribuídos a uma categoria específica
Esta Semana – para pesquisar quando o email foi re- (A)
cebido. Há vários períodos de tempo para escolher (Hoje,
Ontem, Mês Passado, etc.)
Enviado para – para localizar emails enviados a você, (B)
não enviados diretamente a você ou enviados por outro
destinatário
Sinalizado – para localizar emails sinalizados por você (C)
apenas
Importante – para localizar somente emails rotulados
como importantes
(D)

EXERCÍCIO COMENTADO
Na guia Mensagem, no grupo Marcas, escolha Alta
Prioridade ou Baixa Prioridade. O ícone escolhido será
01. (COPASA - Analista de Saneamento - Administra-
realçado para indicar que está ativo.
dor – Superior – FUMARC – 2018). Analise as seguintes
afirmativas sobre as opções disponíveis na guia “MEN-
Resposta: A
SAGEM” quando uma mensagem enviada pelo Microsoft
Outlook, versão português do Office 2013, estiver aberta
na tela do computador: BROFFICE DOCUMENTO TEXTO (WRITER):
I – Para “Encaminhar” a mensagem a outros destinatá- ATALHOS E BARRA DE FERRAMENTAS;
rios, basta acionar o atalho de teclado “Ctrl + Shift + R”. MODOS DE SELEÇÃO DE TEXTO;
II – Para encaminhar a mensagem como um anexo de
FORMATAÇÃO DE TEXTO; FORMATAÇÃO DE
uma nova mensagem, basta selecionar a opção “Mais”
PARÁGRAFOS; ALINHAMENTO;
do grupo “Responder” e, em seguida, selecionar a opção
“Encaminhar como Anexo”.
III – Para cancelar uma mensagem enviada, basta acionar
a opção “Ações” do grupo “Mover” e, em seguida, sele- LIBREOFFICE WRITER
cionar a opção “Cancelar Mensagem Enviada...”.
Estão CORRETAS as afirmativas: O Writer é um aplicativo de processamento de texto
(A) I e II, apenas. que lhe permite criar documentos, como cartas, currí-
(B) I e III, apenas. culos, livros ou formulários online. Alternativa gratuita e
(C) I, II e III. open source ao tradicional pacote Microsoft Office. Sur-
(D) II e III, apenas. gido a partir de um fork do OpenOffice, o LibreOffice traz
soluções completas para edição de texto, criação de pla-
Ctrl + Shift + R: responder a todos nilhas, apresentações de slides, desenhos, base de dados
Se a ideia é responder a todos (destinatário e pessoas e ainda fórmulas matemáticas.
copiadas no e-mail), use o atalho Ctrl + Shift + R. O Writer do LibreOffice suporta os seguintes for-
NOÇÕES DE INFORMÁTICA

matos: ODT, OTT, SXW, STW, DOC (Word), DOCX (Word


Ctrl + F: encaminhar uma mensagem 2007), RTF, SDW, VOR, TXT, HTML, PDB, XML, PSW e UOT.
Para encaminhar rapidamente um e-mail recebido, use
o atalho Ctrl + F. Formato Open Document
No Writer o formato padrão dos arquivos passou de
Ctrl + Shift + B: abrir a agenda de endereços sdw (formato do antigo StarWriter) para odt (Open docu-
Se você não se lembra dos dados de algum contato, ment text), que dota os arquivos de uma estrutura XML,
pode começar abrindo a agenda de endereços do Ou- permitindo uma maior interoperabilidade entre as várias
tlook. Para isso, use o atalho Ctrl + Shift + B. aplicações.

3
Aliás, este foi um dos principais motivos pelos quais a Microsoft tem mantido o monopólio nas aplicações de escri-
tório: a compatibilidade ou a falta dela.
É importante notar que versões mais antigas do OpenOffice já abriam e gravavam arquivos com a terminação doc.
Entretanto, não havia compatibilidade de 100% e os documentos perdiam algumas formatações.
A proposta da Sun, da IBM e de outras empresas foi normalizar os tipos de documento, num formato conhecido
por todos, o odt.

Layout

O Writer aparece sob a forma de uma janela genérica de documento em branco, a tela de edição, que é composta
por vários elementos:
Barra de Título: Apresenta o nome do arquivo e o nome do programa que está sendo usado nesse momento. Usan-
do-se os 3 botões no canto superior direito pode-se minimizar, maximizar / restaurar ou fechar a janela do programa.
Barra de Menus: Apresenta os menus suspensos onde estão as listas de todos os comandos e funções disponíveis
do programa.
Barra de Formatação: Apresenta os atalhos que dão forma e cor aos textos e objetos.
Área para trabalho: Local para digitação de texto e inserção de imagens e sons.
Barra de Status: Apresenta o número de páginas / total de páginas, o valor percentual do Zoom
e a função inserir / sobrescrever está na parte inferior e central da tela.
Réguas: Permite efetuar medições e configurar tabulações e recuos.
NOÇÕES DE INFORMÁTICA

4
Barra de menus Para abrir mais de um documento ao mesmo tempo,
cada um em sua própria janela, pressione a tecla Ctrl ao
Arquivo clicar nos arquivos e, em seguida, clique em Abrir.
Esses comandos se aplicam ao documento atual, abre Para classificar os arquivos, clique em um cabeçalho
um novo documento ou fecha o aplicativo. de coluna. Para inverter a ordem de classificação, clique
novamente.
Para excluir um arquivo, clique com o botão direito
do mouse sobre ele e, em seguida, escolha Excluir.
Para renomear um arquivo, dê um clique com o bo-
tão direito do mouse sobre ele e, em seguida, escolha
Renomear.
Nome do arquivo: Insira um nome de arquivo ou um
caminho para o arquivo. Você também pode inserir um
URL que começa com o nome de protocolo ftp, http,
ou https.
Caso deseje, utilize caracteres curinga na caixa Nome
do arquivo para filtrar a lista de arquivos exibida.
Por exemplo, para listar todos os arquivos de texto
em uma pasta, insira o caractere asterisco (*) com a ex-
tensão de arquivo de texto (*.txt) e, em seguida, clique
em Abrir. Utilize o caractere curinga ponto de interro-
gação (?) para representar qualquer caractere, como em
(??3*.txt), o que só exibe arquivos de texto com um ‘3’
como terceiro caractere no nome do arquivo.
O LibreOffice possui uma função autocompletar que
se ativa sozinha em alguns textos e caixas de listagem.
Por exemplo, entre ~/a no campo da URL e a função
autocompletar exibe o primeiro arquivo ou o primeiro
diretório encontrado no seu diretório de usuário que co-
meça com a letra “a”.
Utilize a seta para baixo para rolar para outros arqui-
vos e diretórios. Utilize a seta para a direita para exibir
-Novo:
também um subdiretório existente no campo da URL.
Cria um novo documento do LibreOffice.
Escolha Arquivo - Novo A função autocompletar rápida está disponível se você
Ícone Novo na Barra de ferramentas (o ícone mostra pressionar a tecla End após inserir parte da URL. Uma
o tipo do novo documento) vez encontrado o documento ou diretório desejado,
pressione Enter.
Versão: Se houver várias versões do arquivo selecio-
Novo nado, selecione a versão que deseja abrir. Você pode sal-
var e organizar várias versões de um documento, esco-
Tecla Ctrl+N lhendo Arquivo - Versões. As versões de um documento
Para criar um documento a partir de um modelo, es- são abertas em modo somente leitura.
colha Novo - Modelos. Tipo de arquivo: Selecione o tipo de arquivo que de-
Um modelo é um arquivo que contém os elementos seja abrir ou selecione Todos os arquivos(*) para exibir
de design para um documento, incluindo estilos de for- uma lista de todos os arquivos na pasta.
matação, planos de fundo, quadros, figuras, campos, la- Abrir: Abre o(s) documento(s) selecionado(s).
yout de página e texto. Inserir: Se você tiver aberto a caixa de diálogo es-
colhendo Inserir - Arquivo, o botão Abrir será rotulado
- Abrir Inserir. Insere no documento atual, na posição do cursor,
Abre ou importa um arquivo. o arquivo selecionado.
Escolha Arquivo - Abrir Somente leitura: Abre o arquivo no modo somente
Ctrl+O leitura.
NOÇÕES DE INFORMÁTICA

Na Barra de ferramentas, clique em Abrir documentos com modelos: O LibreOffice re-


conhece modelos localizados em qualquer uma das se-
Abrir arquivo guintes pastas:
• Pasta de modelos compartilhados
Locais: Mostra os locais favoritos. Por exemplo, os • Pasta de modelos do usuário em Documents
atalhos das pastas locais ou remotas. and Settingsno diretório inicial do usuário
Área de exibição: Mostra os arquivos e pastas existen- • Todas as pastas de modelos definidas em Ferra-
tes em que você está. Para abrir um arquivo, selecione-o mentas - Opções - LibreOffice - Caminhos
e clique em Abrir.

5
Ao utilizar Arquivo - Modelo - Salvar como modelo Salvar como: Salva o documento atual em outro local
para salvar um modelo, o modelo será armazenado na ou com um nome de arquivo ou tipo de arquivo diferen-
sua pasta de modelos do usuário. Ao abrir um documen- te. Escolha Arquivo - Salvar como
to baseado neste modelo, o documento será verificado
para detectar uma mudança do modelo, como descrito
abaixo. O modelo é associado com o documento e pode
ser chamado de “modelo vinculado”.
Ao utilizar Arquivo - Salvar como e selecionar um fil-
tro de modelo para salvar um modelo em qualquer outra
pasta que não esteja na lista, então os documentos ba-
seados nesse modelo não serão verificados.
Ao abrir um documento criado a partir de um “mode-
lo vinculado” (definido acima), O LibreOffice verifica se o
modelo foi modificado desde a última vez que foi aberto.
Se o modelo tiver sido alterado, uma caixa de diálogo
aparecerá para você poder selecionar os estilos que de-
vem ser aplicados ao documento.
Para aplicar os novos estilos do modelo ao documen-
to, clique em Sim.
Para manter os estilos que estão sendo usados no do-
cumento, clique em Não.
Se um documento tiver sido criado por meio de um Nome do arquivo: Insira um nome de arquivo ou um
modelo que não possa ser encontrado, será mostrada caminho para o arquivo. Você também pode inserir um URL
uma caixa de diálogo perguntando como proceder na • Tipo de arquivo: Selecione o formato de arquivo
próxima vez em que o documento for aberto. para o documento que você está salvando. Na área de
Para quebrar o vínculo entre o documento e o mo- exibição, serão exibidos somente os documentos com
esse tipo de arquivo.
delo que está faltando, clique em Não; caso contrário, o
• Salvar com senha: Protege o arquivo com uma
LibreOffice procurará o modelo na próxima vez que você
senha que deve ser digitada para que o usuário possa
abrir o documento.
abrir o arquivo.
- Documentos recentes
Salvar tudo: Salva todos os documentos do LibreOffi-
Lista os arquivos abertos mais recentemente. Para
ce que foram modificados.
abrir um arquivo da lista, clique no nome dele.
- Recarregar
- Assistentes Substitui o documento atual pela última versão salva.
Guia você na criação de cartas comerciais e pessoais, Todos as alterações efetuadas após o último salva-
fax, agendas, apresentações, etc. mento serão perdidas.
Escolha Arquivo - Recarregar
- Fechar
Fecha o documento atual sem sair do programa. - Versões
Escolha Arquivo - Fechar Salva e organiza várias versões do documento atual
O comando Fechar fecha todas as janelas abertas do no mesmo arquivo. Você também pode abrir, excluir e
documento atual. comparar versões anteriores.
Se foram efetuadas alterações no documento atual, Escolha Arquivo - Versões
você será perguntado se deseja salvar as lterações. Novas versões - Define as opções para salvar uma
Ao fechar a última janela de documento aberta, apa- nova versão do documento.
recerá a Tela inicial. Salvar nova versão - Salva o estado atual do docu-
mento como nova versão. Caso deseje, antes de salvar
- Salvar a nova versão, insira também comentários na caixa de
Salva o documento atual. diálogo Inserir comentário da versão.
Escolha Arquivo - Salvar Inserir comentário da versão - Insira um comentário
Ctrl+S aqui quando estiver salvando uma nova versão. Se você
Na Barra de ferramentas ou de tabela de dados, cli- tiver clicado em Mostrar para abrir esta caixa de diálogo,
NOÇÕES DE INFORMÁTICA

que em não poderá editar o comentário.


Salvar sempre uma versão ao fechar - Se você tiver
feito alterações no documento, o LibreOffice salvará au-
Salvar
tomaticamente uma nova versão quando você o fechar.
Se salvar o documento manualmente, e não alterar o
documento após salvar, não será criada uma nova versão.
Versões existentes - Lista as versões existentes do do-
cumento atual, a data e a hora em que elas foram criadas,
o autor e os comentários associados.

6
- Exportar Editar
Salva o documento atual com outro nome e formato
em um local a especificar.
Escolha Arquivo – Exportar
- Exportar como PDF
Salva o arquivo atual no formato Portable Document
Format (PDF) versão 1.4. Um arquivo PDF pode ser visto
e impresso em qualquer plataforma com a formatação
original intacta, desde que haja um software compatível
instalado.
Escolha Arquivo - Exportar como PDF

- Visualizar impressão
Exibe uma visualização da página impressa ou fecha
a visualização.
Menu Arquivo - Visualizar impressão
Utilize os ícones na barra Visualização de impressão
para folhear as páginas do documento ou para imprimir
o documento.
Você também pode pressionar as teclas Page Up e
Page Down para folhear as páginas.
Obs: Não é possível editar seu documento enquanto
estiver na visualização de impressão.

- Imprimir
Imprime o documento atual, a seleção ou as páginas
que você especificar. Você também pode definir as op-
ções de impressão para o documento atual. Tais opções
variam de acordo com a impressora e com o sistema
operacional utilizado.
Escolha Arquivo - Imprimir
Ctrl+P
Na Barra de ferramentas, clique em

- Configuração da impressora Este menu contém comandos para editar o conteúdo


Selecione a impressora padrão para o documento do documento atual.
atual.
Escolha Arquivo - Imprimir - Configurações da im- - Desfazer
pressora Desfaz o último comando ou a última entrada digi-
Fecha todos os programas do LibreOffice e pede para tada. Para selecionar o comando que deseja desfazer,
salvar as modificações. clique na seta ao lado do ícone Desfazer na barra de fer-
- Sair ramentas Padrão.
Escolha Arquivo - Sair - Refazer
Ctrl+Q Reverte a ação do último comando Desfazer. Para se-
lecionar a etapa Desfazer que deseja reverter, clique na
seta ao lado do ícone Refazer na barra de ferramentas
Padrão.

- Repetir
Repete o último comando. Esse comando está dispo-
nível no Writer e no Calc.

- Recortar
NOÇÕES DE INFORMÁTICA

Remove e copia a seleção para a área de transferência.

- Copiar
Copia a seleção para a área de transferência.
Escolha Editar - Copiar
Ctrl+C

7
- Colar - Campos
Insere o conteúdo da área de transferência no local do Abre um caixa de diálogo na qual você pode edi-
cursor, e substitui qualquer texto ou objeto selecionado. tar as propriedades de um campo. Clique antes de um
Escolha Editar - Colar campo e selecione este comando. Na caixa de diálogo,
Ctrl+V você pode usar as setas para ir para o próximo campo ou
voltar para o anterior.
- Colar especial
Insere o conteúdo da área de transferência no ar- - Notas de rodapé
quivo atual em um formato que você pode especificar. Edita a âncora de nota de rodapé ou de nota de fim
Escolha Editar - Colar especial selecionada. Clique na frente da nota de rodapé ou da
nota de fim e, em seguida, escolha este comando.
- Selecionar texto
Você pode ativar um cursor de seleção em um texto - Entrada de índice
somente leitura ou na Ajuda. Escolha Editar - Selecionar Edita a entrada de índice selecionada. Clique antes da
texto ou abra o menu de contexto de um documento entrada de índice ou na própria entrada e, em seguida,
somente leitura e escolha Selecionar texto. O cursor de escolha este comando.
seleção não fica intermitente.
Use o ícone Editar arquivo para ativar ou desativar o - Entrada bibliográfica
modo de edição. Edita a entrada bibliográfica selecionada.

- Modo de seleção - Hiperlink


Escolha o modo de seleção do submenu: modo de Abre uma caixa de diálogo que permite que você crie
seleção normal, ou modo de seleção por bloco. e edite hiperlinks.

- Selecionar tudo - Vínculos


Seleciona todo o conteúdo do arquivo, quadro ou Permite a edição das propriedades de cada vínculo no
objeto de texto atual. documento atual, incluindo o caminho para o arquivo de
Escolha Editar - Selecionar tudo origem. Este comando não estará disponível se o docu-
Ctrl+A mento atual não contiver vínculos para outros arquivos.

- Alterações - Plug-in
Lista os comandos que estão disponíveis para ras- Permite a edição de plug-ins no seu arquivo. Esco-
trear as alterações em seu arquivo. lha este comando para ativar ou desativar este recurso.
Quando ativado, aparecerá uma marca de seleção ao
- Comparar documento lado do comando, e você verá comandos para editar o
Compara o documento atual com um documento plug-in em seu menu de contexto. Quando desativado,
que você seleciona. você verá comandos para controlar o plug-in no menu
de contexto.
- Localizar e substituir
Procura ou substitui textos ou formatos no docu- - Mapa de imagem
mento atual. Permite que você anexe URLs a áreas específicas, de-
Escolha Editar - Localizar e substituir nominadas pontos de acesso, em uma figura ou em um
Ctrl+H grupo de figuras. Um Mapa de imagem é um grupo com
Na Barra de ferramentas, clique em um ou mais pontos de acesso.

- Autotexto - Objeto
Cria, edita ou insere Autotexto. Você pode arma- Permite editar um objeto selecionado no arquivo, in-
zenar texto formatado, texto com figuras, tabelas e serido com o comando Inserir - Objeto.
campos como Autotexto. Para inserir Autotexto rapida-
mente, digite o atalho do Autotexto no documento e
pressione F3.
Escolha Editar – Autotexto
Ctrl+F3
NOÇÕES DE INFORMÁTICA

- Trocar banco de dados


Altera a fonte de dados do documento atual. Para
exibir corretamente o conteúdo dos campos inseridos,
o banco de dados que foi substituído deve conter no-
mes de campos idênticos.

8
Exibir - Limites do texto
Este menu contém comandos para controlar a exibi- Mostra ou oculta os limites da área imprimível da pá-
ção do documento na tela. gina. As linhas de limite não são impressas.

- Sombreamentos de campos
Mostra ou oculta os sombreamentos de campos no
documento, incluindo espaços incondicionais, hifens
personalizados, índices e notas de rodapé.

- Nomes de campos
Alterna a exibição entre o nome e o conteúdo do
campo. A presença de uma marca de seleção indica que
os nomes dos campos são exibidos e a ausência dessa
marca indica que o conteúdo é exibido. O conteúdo de
alguns campos não pode ser exibido.

- Caracteres não-imprimíveis
Mostra os caracteres não imprimíveis no texto, como
marcas de parágrafo, quebras de linha, paradas de tabu-
lação e espaços.

- Parágrafos ocultos
Mostra ou oculta parágrafos ocultos. Esta opção afeta
somente a exibição de parágrafos ocultos. Ela não afeta a
impressão desses parágrafos.

- Fontes de dados
Lista os bancos de dados registrados para o LibreOffi-
ce e permite que você gerencie o conteúdo deles.

- Navegador
Mostra ou oculta o Navegador. Você pode usá-lo para
acessar rapidamente diferentes partes do documento e
- Layout de impressão para inserir elementos do documento atual ou de outros
Exibe a forma que terá o documento quando este for documentos abertos, bem como para organizar docu-
impresso. mentos mestre. Para editar um item do Navegador, clique
com o botão direito do mouse no item e, em seguida,
- Layout da Web escolha um comando do menu de contexto. Se preferir,
Exibe o documento como seria visualizado em um nave- você pode encaixar o Navegador na borda do espaço de
gador da Web. Esse recurso é útil ao criar documentos HTML. trabalho.

- Código-fonte HTML - Tela inteira


Exibe o código fonte do documento HTML atual. Para Exibe ou oculta os menus e as barras de ferramentas
exibir o código fonte HTML de um novo documento, é no Writer ou no Calc. Para sair do modo de tela inteira,
necessário primeiro salvar o novo documento como um clique no botão Ativar/Desativar tela inteira.
documento HTML.
- Zoom
- Barra de status Reduz ou amplia a exibição de tela do LibreOffice.
Mostra ou oculta a barra de status na borda inferior
da janela.

- Status do método de entrada


Mostra ou oculta a janela de status do IME (Input Me-
NOÇÕES DE INFORMÁTICA

thod Engine).

- Régua
Mostra ou oculta a régua horizontal, que é utilizada
para ajustar as margens da página, paradas de tabulação,
recuos, bordas, células de tabela e para dispor objetos na
página. Para mostrar a régua vertical, escolha Ferramen-
tas - Opções - LibreOffice Writer - Exibir, e selecione a
caixa Régua vertical na área Réguas.

9
Inserir - Hiperlink
O menu Inserir contém os comandos necessários para Abre uma caixa de diálogo que permite que você crie
inserir novos elementos no seu documento. Isso inclui e edite hiperlinks.
seções, notas de rodapé, anotações, caracteres especiais,
figuras e objetos de outros aplicativos. - Cabeçalho
Adiciona ou remove um cabeçalho do estilo de pági-
na que você selecionar no submenu. O cabeçalho é adi-
cionado a todas as páginas que usam o mesmo estilo
de página. Em um novo documento, é listado apenas o
estilo de página “Padrão”. Outros estilos de páginas serão
adicionados à lista depois que você aplicá-los ao docu-
mento.

- Rodapé
Adiciona ou remove um rodapé do estilo de página
selecionado no submenu. O rodapé é adicionado a todas
as páginas que usam o mesmo estilo. Em um novo do-
cumento, somente o estilo de página “Padrão” é listado.
Outros estilos serão adicionados à lista depois que forem
aplicados ao documento.

- Nota de rodapé / Nota de fim


Insere uma nota de rodapé ou uma nota de fim no
documento. A âncora para a nota será inserida na posi-
ção atual do cursor. Você pode escolher entre numeração
automática ou um símbolo personalizado.

- Legenda
Adiciona uma legenda numerada à figura, tabela,
quadro, quadro de texto ou objeto de desenho selecio-
nado. Você também pode acessar este comando clicando
com o botão direito do mouse no item ao qual deseja
adicionar a legenda.

- Marcador
Insere um indicador na posição do cursor. Use o Na-
vegador para saltar rapidamente para a posição indicada
em outra hora. em um documento HTML, os indicadores
- Quebra manual são convertidos em âncoras para você navegar através
Insere uma quebra manual de linha, de coluna ou de de hyperlinks.
página na posição atual em que se encontra o cursor.
- Referência
- Campos Esta é a posição em que você insere as referências ou
Insere um campo na posição atual do cursor. O sub- os campos referidos no documento atual. As referências
menu lista os tipos de campos mais comuns. Para exibir são os campos referidos no mesmo documento ou em
todos os campos disponíveis, escolha Outro. subdocumentos de um documento mestre.

- Caractere especial - Anotação


Insere caracteres especiais a partir das fontes instaladas. Insere uma anotação.

- Marca de formatação - Script


Abe um submenu para inserir marcas especiais de Insere um script na posição atual do cursor em um
formatação. Ative o CTL para mais comandos. documento HTML ou de texto.
NOÇÕES DE INFORMÁTICA

- Seção - Índices e índices gerais


Insere uma seção de texto no mesmo local em que o Abre um menu para inserir entradas de índice e inserir
cursor está posicionado no documento. Também é possí- índices e tabelas.
vel selecionar um bloco de texto e, em seguida, escolher
esse comando para criar uma seção. Use as seções para - Envelope
inserir blocos de texto de outros documentos, para apli- Cria um envelope. Nas três guias, você pode especi-
car layouts de colunas personalizados ou para proteger ficar o destinatário e o remetente, a posição e o formato
ou ocultar os blocos de texto quando uma condição for de ambos os endereços, o tamanho e a orientação do
atendida. envelope.

10
- Quadro - Limpar formatação direta
Insere um quadro que você pode usar para criar um Remove a formatação direta e a formatação por esti-
layout com uma ou mais colunas de texto e objetos. los de caracteres da seleção.

- Tabela - Caractere
Insere uma tabela no documento. Você também pode Muda a fonte e a formatação de fonte dos caracteres
clicar na seta, arrastar o mouse para selecionar o número selecionados.
de linhas e colunas a serem incluídas na tabela e, em se-
guida, clicar na última célula. - Parágrafo
Modifica o formato do parágrafo atual, por exemplo,
- Figura alinhamento e recuo.
Selecione a origem da figura que deseja inserir.
- Marcadores e numeração
- Objeto de desenho Adiciona marcadores ou numeração ao parágrafo
Insere um objeto no documento. Para vídeo e áudio atual e permite que você edite o formato da numeração
utilize Inserir - Multimídia - Áudio ou vídeo. ou dos marcadores.

- Quadro flutuante - Página


Insere um quadro flutuante no documento atual. Especifique os estilos de formatação e o layout do es-
Quadros flutuantes são utilizados em documentos HTML tilo de página atual, incluindo margens da página, cabe-
para exibir conteúdo de outro arquivo. çalhos, rodapés e o plano de fundo da página.

- Áudio ou vídeo - Alterar caixa


Insere um arquivo de vídeo ou áudio no documento. Altera a caixa dos caracteres selecionados. Se o cursor
estiver no meio de uma palavra e não houver texto sele-
- Arquivo cionado, então a palavra será a seleção.
Insere um arquivo de texto na posição atual do cursor.
- Guia fonético asiático
Formatar Permite que você adicione comentários sobre carac-
Contém comandos para formatar o layout e o conteú- teres asiáticos para serem usados como manual de pro-
do de seu documento. núncia.

- Colunas
Especifica o número de colunas e o layout de coluna
para um estilo de página, quadro ou seção.

- Seções
Altera as propriedades das seções definidas no docu-
mento. Para inserir uma seção, selecione o texto ou cli-
que no documento e, em seguida, escolha Inserir - Seção.

- Estilos e formatação
Use a janela Estilos e formatação para aplicar, criar,
editar, adicionar e remover estilos de formatação. Clique
duas vezes para aplicar o estilo.

- Autocorreção
Formata automaticamente o arquivo de acordo com
as opções definidas em Ferramentas - Opções da auto-
correção.

- Ancorar
Define as opções de ancoramento para o objeto se-
NOÇÕES DE INFORMÁTICA

lecionado.

- Quebra Automática
Define as opções de quebra automática de texto para
figuras, objetos e quadros.

- Alinhar (objetos)
Alinha os objetos selecionados em relação a outro.

11
- Alinhamento (objetos de texto) Linhas
Define as opções de alinhamento para a seleção atual. Insere linhas.

- Dispor - Excluir
Altera a ordem de empilhamento do(s) objeto(s) se- Tabela
lecionado(s). Exclui a tabela atual.

- Inverter Colunas
Inverte o objeto selecionado, horizontalmente ou Exclui as colunas selecionadas.
verticalmente.
Linhas
- Agrupar Exclui as linhas selecionadas.
Agrupa os objetos selecionados de forma que pos-
sam ser movidos ou formatados como um único objeto. Selecionar
Tabela
- Objeto Seleciona a tabela atual.
Abre um submenu para editar propriedades do obje-
to selecionado. Coluna
Seleciona a coluna atual.
- Quadro
Insere um quadro que você pode usar para criar um Linha
layout com uma ou mais colunas de texto e objetos. Seleciona a linha atual.

- Imagem Célula
Formata o tamanho, a posição e outras propriedades Seleciona a célula atual.
da figura selecionada.
- Mesclar células
Tabela Combina o conteúdo das células selecionadas da ta-
bela em uma única célula.

- Dividir células
Divide a célula ou grupo de células horizontalmente
ou verticalmente em um número especificado de células.

- Mesclar tabela
Combina duas tabelas consecutivas em uma única ta-
bela. As tabelas devem estar lado a lado, e não separadas
por um parágrafo vazio.

- Dividir tabela
Divide a tabela atual em duas tabelas separadas na
posição do cursor. Você também pode clicar com o botão
direito do mouse em uma célula da tabela para acessar
este comando.

- Autoformatação de tabela
Aplica automaticamente formatos à tabela atual, in-
cluindo fontes, sombreamento e bordas.

Autoajustar
Largura da coluna
Abre a caixa de diálogo Largura da coluna para alterar
a largura de uma coluna.
NOÇÕES DE INFORMÁTICA

- Largura de coluna ideal


- Inserir Ajusta automaticamente a largura das colunas para
Tabela coincidir com o conteúdo das células. A alteração da lar-
Insere uma nova tabela. gura de uma coluna não afeta a largura das outras co-
lunas na tabela. A largura da tabela não pode exceder a
Colunas largura da página.
Insere colunas.

12
- Distribuir colunas uniformemente Ferramentas
Ajusta a largura das colunas selecionadas para a lar- Contém ferramentas de verificação ortográfica, uma
gura da coluna mais larga da seleção. A largura total da galeria de objetos artísticos que podem ser adicionados
tabela não pode exceder a largura da página. ao documento, bem como ferramentas para configurar
menus e definir preferências do programa.
Altura da linha
Abre a caixa de diálogo Altura da linha para alterar a
altura de uma linha.

- Altura de linha ideal


Ajusta automaticamente a altura das linhas para que
corresponda ao conteúdo das células. Esta é a definição
padrão para novas tabelas.

- Distribuir linhas uniformemente


Ajusta a altura das linhas selecionadas para a altura
da linha mais alta na seleção.

Permitir quebra de linha em páginas e colunas


Permite uma quebra de página na linha atual.

Repetir linhas de cabeçalho


Repete os cabeçalhos das tabelas nas páginas sub-
sequentes quando a tabela se estende por uma ou mais
páginas.

Converter

Texto em tabela
Abre uma caixa de diálogo para poder converter em
tabela o texto selecionado.

Tabela para texto


Abre uma caixa de diálogo para converter a tabela - Verificação ortográfica
atual em texto. Verifica a ortografia manualmente.

Classificar - Idioma
Classifica alfabeticamente ou numericamente os pa- Abre um submenu para escolher comandos específi-
rágrafos ou linhas de tabela selecionados. Você pode cos do idioma.
definir até três chaves de classificação bem como com-
binar chaves alfabéticas com numéricas. - Contagem de palavras
Conta as palavras e caracteres, com ou sem espaços,
Fórmula na seleção atual, e em todo o documento. A contagem é
Abre a Barra de fórmulas para inserir ou editar uma mantida atualizada enquanto digita, ou altera a seleção.
fórmula.
- Numeração da estrutura de tópicos
Formato numérico Especifica o formato de número e a hierarquia para a
Abre uma caixa de diálogo para especificar o forma- numeração de capítulos no documento atual.
to de números na tabela.
- Numeração de linhas
Limites da tabela Adiciona ou remove e formata números de linha no
Mostra ou oculta os limites em torno das células da documento atual. Para desativar a numeração de linhas
tabela. Os limites só são visíveis na tela e não são im- em um parágrafo, clique no parágrafo, escolha Forma-
NOÇÕES DE INFORMÁTICA

pressos. tar - Parágrafo, clique na guia Numeração e, em seguida,


desmarque a caixa de seleção Incluir este parágrafo na
Propriedades da tabela numeração de linhas
Especifica as propriedades da tabela selecionada,
como, por exemplo, nome, alinhamento, espaçamento, - Notas de rodapé
largura da coluna, bordas e plano de fundo. Especifica as configurações de exibição de notas de
rodapé e notas de fim.

13
- Galeria Janela
Abre a Galeria, onde você poderá selecionar figuras e
sons para inserir em seu documento.

- Banco de dados bibliográfico


Insira, exclua, edite e organize arquivos no banco de
dados bibliográfico.

- Assistente de Mala Direta


Inicia o Assistente de Mala Direta para criar cartas- Contém comandos para manipulação e exibição de
-modelo ou enviar mensagens de e-mail a vários desti- janelas de documentos.
natários.
- Nova janela
- Classificar Abre uma nova janela que exibe os conteúdos da
Classifica alfabeticamente ou numericamente os pa- janela atual. Você pode agora ver diferentes partes do
rágrafos ou linhas de tabela selecionados. Você pode de- mesmo documento ao mesmo tempo.
finir até três chaves de classificação bem como combinar
chaves alfabéticas com numéricas. - Fechar a janela
Fecha a janela atual. Escolha Janela - Fechar janela,
- Calcular ou pressione Ctrl+F4. Na visualização de impressão do
Calcula a fórmula selecionada e copia o resultado LibreOffice Writer e Calc, você pode fechar a janela ao
para a área de transferência. clicar no botão Fechar visualização.

- Atualizar - Lista de documentos


Atualiza os itens do documento atual com conteúdo Lista os documentos abertos no momento atual. Se-
dinâmico, como campos e índices. lecione o nome de um documento na lista para alternar
para esse documento.
- Player de mídia
Abre a janela do Player de mídia, para poder visuali- Ajuda
zar arquivos de vídeo e áudio e inseri-los no documento O menu da Ajuda permite iniciar e controlar o sistema
atual. de Ajuda do LibreOffice.

- Macros
Permite gravar, organizar e editar macros.

- Gerenciador de extensão
O Gerenciador de extensão adiciona, remove, desati-
va, ativa e atualiza extensões do LibreOffice.

- Filtros XML
Abre a caixa de diálogo Configurações do filtro XML,
onde você pode criar, editar, excluir e testar filtros para
importar e exportar arquivos XML.

- Opções da Autocorreção Barras de ferramentas


Configura as opções para substituir texto automatica-
mente ao digitar. Barra de objetos de texto
Contém comandos de formatação para o texto de
- Personalizar um objeto de desenho. A barra Objetos de texto aparece
Personaliza menus, teclas de atalho, barras de fer- quando você faz um duplo clique dentro de um objeto
ramentas e atribuições de macros do LibreOffice para de desenho.
eventos.
- Nome da fonte
NOÇÕES DE INFORMÁTICA

- Opções Permite que você selecione um nome de fonte na lista


Este comando abre uma caixa de diálogo para confi- ou digite um nome de fonte diretamente.
guração personalizada do programa. Você pode inserir várias fontes, separadas por pon-
to-e-vírgulas. O LibreOffice usará cada fonte nomeada
em sucessão se as fontes anteriores não estiverem dis-
poníveis.

Nome da fonte

14
- Tamanho da fonte - Justificar
Permite que você escolha entre diferentes tamanhos Alinha os parágrafos selecionados às margens es-
de fonte na lista ou que digite um tamanho manualmente. querda e direita da página. Se preferir, você pode espe-
cificar as opções de alinhamento para a última linha de
- Negrito um parágrafo, escolhendo Formatar - Parágrafo - Alinha-
Aplica o formato negrito ao texto selecionado. Se o mento.
cursor estiver sobre uma palavra, ela ficará toda em ne-
grito. Se a seleção ou a palavra já estiver em negrito, a
Justificado
formatação será removida.
- Suporte a idiomas asiáticos
Negrito Esses comandos só podem ser acessados após ativar
o suporte para idiomas asiáticos em Ferramentas - Op-
- Itálico ções - Configurações de idioma - Idiomas.
Aplica o formato itálico ao texto selecionado. Se o
cursor estiver sobre uma palavra, ela ficará toda em itáli- - Texto escrito da esquerda para a direita
co. Se a seleção ou palavra já estiver em itálico, a forma- Especifica a direção horizontal do texto.
tação será removida.
Direção do texto da esquerda para a direita
Itálico
- Texto escrito de cima para baixo
- Sublinhado Especifica a direção vertical do texto.
Sublinha o texto selecionado ou remove o sublinhado
do texto selecionado.
Texto escrito de cima para baixo

Sublinhado
- Selecionar tudo
Seleciona todo o conteúdo do arquivo, quadro ou ob-
- Sobrescrito jeto de texto atual.
Reduz o tamanho da fonte do texto selecionado e le-
vanta o texto acima da linha de base.
Selecionar tudo

Sobrescrito - Caractere
Muda a fonte e a formatação de fonte dos caracteres
selecionados.
- Subscrito
Reduz o tamanho da fonte do texto selecionado e po-
Caractere
siciona o texto abaixo da linha de base.
- Parágrafo
Subscrito Aqui você pode definir recuos, espaçamento, alinha-
mento e espaçamento de linha para o parágrafo selecio-
- Esquerda nado.
Alinha o parágrafo selecionado em relação à margem
esquerda da página.
Parágrafo

Alinhar à esquerda Adicionando e editando texto

Você pode adicionar texto ao documento das seguin-


- Centralizar tes maneiras:
Centraliza na página os parágrafos selecionados. • Digitando texto com o teclado
NOÇÕES DE INFORMÁTICA

• Copiando e colando texto de outro documento


• Importando texto de outro arquivo

Digitando texto
- Direita
Alinha os parágrafos selecionados em relação à mar- A maneira mais fácil de inserir texto no documento é
gem direita da página. digitar o texto. Ao digitá-lo, a ferramenta AutoCorreção
corrige automaticamente possíveis erros de ortografia
Alinhar à direita comuns, como “catra” em vez de “carta”.

15
Por padrão, a ferramenta Completar palavras coleta palavras longas enquanto você digita. Ao começar a digitar
novamente a mesma palavra, o
LibreOffice.org completa automaticamente a palavra. Para aceitar a palavra, pressione Enter ou continue digitando.

Dica – Para desativar as ferramentas de completar e substituir automaticamente procure na ajuda on-line os seguin-
tes termos:
• Função de AutoCorreção
• Função de AutoEntrada
• Completar palavras
• Reconhecimento de números
• Função de AutoFormatação

Selecionando texto
Você pode selecionar texto com o mouse ou com o teclado.

Selecionando texto com o mouse


• Para selecionar um trecho de texto, clique no início do trecho, mantenha pressionado o botão esquerdo do mouse
e arraste o mouse até o fim do texto.
Pode também clicar na frente do trecho, mover o mouse até o fim do texto, manter pressionada a tecla Shift e clicar
novamente.
• Para selecionar uma frase inteira, clique três vezes em qualquer lugar na frase.
• Para selecionar uma única palavra, clique duas vezes em qualquer lugar na palavra.
• Para acrescentar mais de um trecho a uma seleção, selecione o trecho, mantenha pressionada a tecla Ctrl e sele-
cione outro trecho de texto.

Selecionando texto com o teclado


• Para selecionar o documento inteiro, pressione Ctrl+A.
• Para selecionar uma única palavra em um dos lados do cursor, mantenha pressionadas as teclas Ctrl+Shift e pres-
sione a seta para a esquerda <- ou a seta para a direita ->.
NOÇÕES DE INFORMÁTICA

• Para selecionar um único caractere em um dos lados do cursor, mantenha pressionada a tecla Shift e pressione a
seta para a esquerda <- ou a seta para a direita ->. Para selecionar mais de um caractere, mantenha pressionada a tecla
Shift enquanto pressiona a tecla de direção.
• Para selecionar o texto restante na linha à esquerda do cursor, mantenha pressionada a tecla Shift e pressione a
tecla Home.
• Para selecionar o texto restante na linha à direita do cursor, mantenha pressionada a tecla Shift e pressione a tecla End.

16
Copiando, colando e excluindo texto - Para localizar e substituir texto
Você pode copiar texto de um lugar para outro no
mesmo documento ou de um documento para outro.

- Para copiar e colar texto


Etapas
1. Selecione o texto que deseja copiar e siga um des-
tes procedimentos:
• Escolha Editar – Copiar.
• Pressione Ctrl+C.
• Clique no ícone Copiar na barra Padrão.
• Clique com o botão direito do mouse no texto sele-
cionado e escolha Copiar.
O texto continua na área de transferência até você
copiar outra seleção de texto ou item.
2. Clique ou mova o cursor para onde deseja colar o
texto. Siga um destes procedimentos:
• Escolha Editar – Colar.
• Pressione Ctrl+V.
• Clique no ícone Colar na barra Padrão. Etapas
• Clique com o botão direito do mouse onde deseja 1. Escolha Editar – Localizar e substituir.
colar o texto e escolha Colar. Abre-se a caixa de diálogo Localizar e substituir.
2. Na caixa Pesquisar por, digite o texto que você de-
- Para excluir texto seja localizar no documento.
Etapas Pode selecionar também a palavra ou a frase que de-
1. Selecione o texto que deseja excluir. seja procurar no documento de texto e escolher Editar
2. Siga um destes procedimentos: - Localizar e substituir. O texto selecionado é inserido au-
• Escolha Editar - Recortar ou pressione Ctrl+X. tomaticamente na caixa Procurar.
O texto é excluído do documento e adicionado à área 3. Na caixa Substituir por, insira a palavra ou a frase
de transferência, para você colar o texto onde pretender. de substituição.
• Pressione a tecla Delete ou Backspace. 4. Clique em Localizar para iniciar a procura.
Observação – Você pode usar essas teclas para tam- 5. Quando o Writer localizar a primeira ocorrência da
bém excluir caracteres individuais. palavra ou frase, siga um destes procedimentos:
Se desejar desfazer uma exclusão, escolha Editar - • Para substituir a ocorrência do texto encontrada
Desfazer ou pressione Ctrl+Z. pela que você inseriu na caixa Substituir por, clique em
Substituir.
-Para inserir um documento de texto • Para substituir todas as ocorrências do texto encon-
Você pode inserir o conteúdo de qualquer documen- tradas pela que você inseriu na caixa Substituir por, clique
to de texto no documento do Writer, desde que o forma- em Substituir tudo.
to do arquivo seja conhecido pelo LibreOffice.org. • Para ignorar o texto encontrado e continuar a pro-
Etapas cura, clique em
1. Clique no documento do Writer onde deseja inserir Localizar próxima.
o texto. 6. Clique em Fechar quando concluir a procura.
2. Escolha Inserir – Arquivo.
3. Localize o arquivo que deseja inserir e clique em Verificando ortografia
Inserir.
Localizando e substituindo texto O Writer pode verificar possíveis erros ortográficos
Você pode usar o recurso Localizar e substituir no Li- enquanto você digita ou em um documento inteiro.
breOffice.org Writer para procurar e substituir palavras
em um documento de texto. - Para verificar ortografia enquanto digita
NOÇÕES DE INFORMÁTICA

O Writer pode avisar sobre possíveis erros de orto-


grafia enquanto você digita.
Para ativar e desativar esse recurso, clique no ícone
AutoOrtografia e gramatica na barra de Ferramentas.
Quando esse recurso está ativado, uma linha vermelha
ondulada marca possíveis erros ortográficos.

17
1. Clique com o botão direito do mouse em uma palavra com um sublinhado ondulado em vermelho.
2. Siga um destes procedimentos:
• Escolha uma das palavras de substituição sugeridas no alto do menu de contexto.
A palavra incorreta é substituída pela palavra que você escolher.
• Escolha uma das palavras de substituição no submenu AutoCorreção.
A palavra incorreta é substituída pela palavra que você escolher.
As duas palavras são acrescentadas automaticamente à lista de substituição da ferramenta AutoCorreção. Na próxi-
ma vez que cometer o mesmo erro ortográfico, o Writer fará a correção ortográfica automaticamente.
• Escolha Ortografia e gramatica para abrir a caixa de diálogo Ortografia e gramatica.
• Para adicionar a palavra a um dos dicionários, escolha Adicionar e clique no nome do dicionário.
Observação – O número de entradas em um dicionário definido pelo usuário é limitado, mas você pode criar quan-
tos dicionários definidos pelo usuário forem necessários.

- Para verificar a ortografia em um documento inteiro


Se não deseja verificar a ortografia enquanto digita, você pode usar a ferramenta Ortografia e gramatica para corri-
gir erros manualmente. A ferramenta Ortografia e gramatica começa a partir da posição atual do cursor ou a partir do
início do texto selecionado.
Etapas
1. Clique no documento ou selecione o texto que deseja corrigir.
2. Escolha Ferramentas - Ortografia e gramatica.
3. Quando um possível erro de ortografia é localizado, a caixa de diálogo Ortografia e gramatica sugere uma cor-
reção.
NOÇÕES DE INFORMÁTICA

18
4. Siga um destes procedimentos:
• Para aceitar uma correção, clique em Corrigir.
• Substitua a palavra incorreta na caixa no alto pela palavra correta e clique em Alterar.
• Para ignorar a palavra atual uma vez e continuar a Ortografia e gramatica, clique em Ignorar uma vez.
• Para ignorar a palavra atual no documento inteiro e continuar a Ortografia e gramatica, clique em Ignorar sempre.

Formatando texto
O Writer permite-lhe formatar o texto manualmente ou ao usar estilos. Com os dois métodos, você controla tama-
nho, tipo de fonte, cor, alinhamento e espaçamento do texto. A principal diferença é que a formatação manual aplica-se
apenas ao texto selecionado, enquanto a formatação de estilo aplica-se toda vez que o estilo é usado no documento.

Formatando texto manualmente

Para uma formatação simples, como alterar o tamanho e a cor do texto, use os ícones na barra Formatação. Se de-
sejar, pode também usar os comandos de menu no menu Formato, assim como teclas de atalho.

Selecione o texto que deseja alterar e siga um destes procedimentos:


• Para alterar o tipo de fonte usado, selecione uma fonte diferente na caixa Nome da fonte.
• Para alterar o tamanho do texto, selecione um tamanho na caixa Tamanho da fonte.
• Para alterar o tipo de letra do texto, clique no ícone Negrito, Itálico ou Sublinhado.
Pode também usar as seguintes teclas de atalho: Ctrl+B para negrito, Ctrl+I para itálico ou Ctrl+U para sublinhado.
Para restaurar o tipo de letra padrão, selecione o texto novamente e clique no mesmo ícone, ou pressione as mesmas
teclas de atalho.
• Para alterar o alinhamento do texto, clique no ícone Alinhar à esquerda, Centralizar, Alinhar à direita ou Justificado.
• Para adicionar ou remover marcadores ou números de uma lista, clique no ícone Ativar/desativar numeração ou
Ativar/desativar marcadores.
• Para alterar um recuo do texto, use os ícones de recuo.
• Para alterar a cor do texto, clique no ícone Cor da fonte.
• Para alterar a cor do plano de fundo do texto, clique no ícone Cor do plano de fundo ou no ícone Realce.
Dica – Consulte a ajuda on-line para obter informação sobre a diferença desses dois ícones.

Formatando texto com estilos


No Writer, a formatação padrão de caracteres, parágrafos, páginas, quadros e listas é feita com estilos. Um estilo é
um conjunto de opções de formatação, como tipo e tamanho da fonte. Um estilo define o aspecto geral do texto, assim
como o layout de um documento.
Você pode selecionar alguns estilos comuns, e todos os estilos aplicados, a partir da lista drop-down Aplicar estilo
na barra Formatar.

NOÇÕES DE INFORMÁTICA

19
Uma maneira fácil de aplicar um estilo de formatação é Cabeçalho e rodapé
com a janela Estilos e formatação. Para abrir a janela Esti- Cabeçalhos e rodapés são áreas nas margens superior
los e formatação, escolha Formatar – Estilos e formatação. e inferior das páginas para adiciona textos ou figuras. Os
cabeçalhos e rodapés são adicionados ao estilo de pá-
gina atual. Todas as páginas que usarem o mesmo estilo
receberão automaticamente o cabeçalho ou rodapé adi-
cionado. É possível inserir Campos, tais como números
de páginas e títulos de capítulos, nos cabeçalhos e roda-
pés de um documento de texto.
Obs.: O estilo de página para a página atual será exi-
bido na Barra de status.
Para adicionar um cabeçalho a uma página, escolha
Inserir - Cabeçalho e, em seguida, no submenu, selecione
o estilo de página para a página atual.
Para adicionar um rodapé a uma página, escolha In-
serir - Rodapé e, em seguida, selecione o estilo de página
para a página atual no submenu.
Você também pode escolher Formatar - Página, clicar
na guia Cabeçalho ou Rodapé, e selecionar Ativar cabe-
çalho ou Ativar rodapé. Desmarque a caixa de seleção
Mesmo conteúdo esquerda/direita se desejar definir ca-
beçalhos e rodapés diferentes para páginas pares e ím-
pares.
Para utilizar diferentes cabeçalhos e rodapés docu-
mento, adicione-os a diferentes estilos de páginas e, em
seguida, aplique os estilos às páginas nas deseja exibir os
Usando tabelas em documentos do Writer cabeçalhos ou rodapés.
Você pode usar tabelas para apresentar e organizar
informações importantes em linhas e colunas, para as in- Marcadores e numeração
formações serem lidas com facilidade. A interseção de Para adicionar marcadores
uma linha e uma coluna é chamada de célula. Selecione o(s) parágrafo(s) ao(s) qual(is) deseja adi-
cionar marcadores.
- Para adicionar uma tabela a um documento do Writer Na barra de Formatação, clique no ícone Ativar/desa-
Etapas tivar marcadores.
1. Escolha Tabela - Inserir – Tabela.
2. Na área Tamanho, digite o número de linhas e co-
lunas para a tabela.

Para remover marcadores, selecione os parágrafos


com marcadores e, em seguida, clique no ícone Ativar/
Desativar marcadores na barra Formatação.
Para formatar marcadores
Para alterar a formatação da lista com marcadores,
escolha Formatar - Marcadores e numeração.
NOÇÕES DE INFORMÁTICA

3. (Opcional) Para usar um layout de tabela predefini-


do, clique em AutoFormatar, selecione o formato deseja-
do e clique em OK.

20
Por exemplo, para mudar o símbolo do marcador, Utilizar diferentes estilos de número de página
clique na guia Opções , clique no botão de seleção (...) Você precisa que algumas páginas apresentem o esti-
perto de Caractere, e selecione um caractere especial. lo em numerais romanos e o restante das páginas, outro
Pode-se também clicar na guia Figura, e clicar num estilo estilo.
de símbolo na área Seleção. No Writer, serão necessários vários estilos de página.
O primeiro estilo de página apresenta um rodapé com
Números de página um campo de número de página formatado em nume-
No Writer, o número da página é um campo que pode rais romanos. O estilo de página seguinte apresenta um
ser inserido no texto. rodapé com um campo de número de página formatado
em outro estilo.
Para inserir números de página Ambos estilos de página devem estar separados por
Escolha Inserir - Campos - Número da página para uma quebra de página. No Writer, é possível inserir que-
inserir um número de página na posição atual do cursor. bras de página automática ou manualmente.
Obs.: Se, em vez do número, aparecer o texto “Núme- Uma quebra de página automática aparece no final
ro da página”, escolha Exibir - Nome de campos. de uma página quando o estilo de página possui um
No entanto, esses campos mudarão de posição quan- “próximo estilo” diferente.
do um texto for adicionado ou removido. Assim, é me- Por exemplo, o estilo de página da “Primeira página”
lhor inserir o campo de número da página em um cabe- apresenta “Padrão” como próximo estilo. Para compro-
çalho ou rodapé que se encontram na mesma posição e var essa possibilidade, pressione F11 para abrir a janela
se repetem em todas as páginas. Estilos e formatação, clique no ícone Estilos de página e
Escolha Inserir - Cabeçalho - (nome do estilo de pá- clique com o botão direito do mouse na entrada Primeira
gina) ou Inserir - Rodapé - (nome do estilo de rodapé) página. Escolha Modificar no menu de contexto. Na guia
para adicionar um cabeçalho ou um rodapé em todas as Organizador, pode ser visto o “próximo estilo”.
páginas com o estilo de página atual. A quebra de página manual pode ser aplicada com ou
sem alterações dos estilos de página.
Para iniciar com um número de página definido Se pressionar Ctrl+Enter, será aplicada a quebra de
Agora você que ter um pouco mais de controle sobre página sem alterações nos estilos de página.
o número da página. Você está editando um documento Se escolher Inserir - Quebra manual, a quebra de pá-
de texto que deve começar com o número de página 12. gina pode ser inserida com ou sem alterações no estilo
ou com alterações no número da página.
Clique no primeiro parágrafo do documento. Dependendo do documento, é melhor: utilizar a que-
Escolha Formatar - Parágrafo - Fluxo de texto. bra de página inserida manualmente entre os estilos de
Em Quebras, ative Inserir. Ative Com estilo de página para página, ou utilizar uma mudança automática. Se for ne-
poder definir o novo Número da página. Clique em OK. cessária apenas uma página de título com estilo diferen-
te, o método automático pode ser utilizado:

Para aplicar um estilo de página diferente na pri-


meira página
Clique na primeira página do documento.
Escolha Formatar - Estilos e formatação.
Na janela Estilos e formatação, clique no ícone Estilos
de página.
Clique duas vezes no estilo “Primeira página”.
A página de título apresentará o estilo “Primeira pági-
na” e as páginas seguintes apresentarão automaticamen-
te o estilo “Padrão”.

#FicaDica
Você pode agora por exemplo, inserir um rodapé
somente para o estilo de página “Padrão”, ou
inserir rodapés em ambos estilos de página, mas
O novo número da página é um atributo do primeiro com os campos de número de página formatados
NOÇÕES DE INFORMÁTICA

parágrafo da página. de modo diferente.

Para formatar o estilo dos números de página


Você deseja que os números da página sejam em nu- Para aplicar uma alteração de estilo de página inseri-
merais romanos: i, ii, iii, iv e assim por diante. da manualmente
Clique duas vezes imediatamente na frente do campo Clique no início do primeiro parágrafo da página
de número da página. A caixa de diálogo Editar campos onde será aplicado um estilo de página diferente.
se abrirá. Escolha Inserir - Quebra manual. A caixa de diálogo
Selecione um formato de número e clique em OK. Editar quebra se abrirá.

21
Na caixa de listagem Estilo, selecione um estilo de Ctrl + tecla mais - Calcula o texto selecionado e copia
página. Você pode definir um novo número de página o resultado para a área de transferência.
também. Clique em OK. Ctrl+Hífen(-) - Hifens personalizados; hifenização de-
O estilo de página selecionado será usado a partir do finida pelo usuário.
parágrafo atual até a próxima quebra de página com es- Ctrl+Shift+sinal de menos (-) - Traço incondicional
tilo. Você pode precisar criar primeiro um novo estilo de (não utilizado na hifenização)
página. Ctrl+sinal de multiplicação * (somente no teclado nu-
mérico) - Executar campo de macro
Teclas de atalho do LibreOffice Writer Ctrl+Shift+Espaço - Espaços incondicionais. Esses
Você pode utilizar teclas de atalho para executar ra- espaços não serão usados para hifenização nem serão
pidamente tarefas comuns no LibreOffice. Esta seção re- expandidos se o texto estiver justificado.
laciona as teclas de atalho padrão do LibreOffice Writer. Shift+Enter - Quebra de linha sem mudança de pa-
rágrafo
Teclas de função para o LibreOffice Writer Ctrl+Enter - Quebra manual de página
Teclas de atalho - Efeito Ctrl+Shift+Enter - Quebra de coluna em textos com
F2 - Barra de fórmulas várias colunas
Ctrl+F2 - Insere campos Alt+Enter - Insere um novo parágrafo sem numeração
F3 - Completa o autotexto numa lista. Não funciona se o cursor estiver no fim da
Ctrl+F3 - Edita o autotexto lista.
F4 - Abre a exibição da fonte de dados Alt+Enter - Insere um novo parágrafo antes ou depois
Shift+F4 - Seleciona o próximo quadro de uma seção ou antes de uma tabela.
F5 - Ativar/Desativar o Navegador Seta para a esquerda - Move o cursor para a esquerda
Ctrl+Shift+F5 - Ativar Navegador, vai para número da Shift+Seta para a esquerda - Move o cursor para a
página esquerda com seleção
F7 - Verificação ortográfica Ctrl+Seta para a esquerda - Vai para o início da pa-
Ctrl+F7 - Dicionário de sinônimos lavra
Ctrl+Shift+Seta para a esquerda - Seleciona à
F8 - Modo de extensão
esquerda, uma palavra de cada vez
Ctrl+F8 - Ativar/Desativar sombreamentos de campos
Seta para a direita - Move o cursor para a direita
Shift+F8 - Modo de seleção adicional
Shift+Seta para a direita - Move o cursor para a direi-
Ctrl+Shift+F8 - Modo de seleção por bloco
ta com seleção
F9 - Atualiza os campos
Ctrl+Seta para a direita - Vá para o início da próxima
Ctrl+F9 - Mostra os campos
palavra
Shift+F9 - Calcula a tabela Ctrl+Shift+Seta para a direita - Seleciona à direita,
Ctrl+Shift+F9 - Atualiza os campos e as listas de en- uma palavra de cada vez
trada Seta para cima - Move o cursor uma linha acima
Ctrl+F10 - Ativar/Desativar caracteres não imprimíveis Shift+Seta para cima - Seleciona linhas de baixo para
F11 - Ativar/Desativar janela Estilos e formatação cima
Shift+F11 - Cria um estilo Ctrl+Seta para cima - Move o cursor para o começo
Ctrl+F11 - Define o foco para a caixa Aplicar estilos do parágrafo anterior
Ctrl+Shift+F11 - Atualiza o estilo CtrlShift+Seta para cima - Seleciona até o começo do
F12 - Ativar numeração parágrafo. Ao repetir, estende a seleção até o início do
Ctrl+F12 - Insere ou edita a tabela parágrafo anterior
Shift+F12 - Ativa marcadores Seta para baixo - Move o cursor uma linha para baixo
Ctrl+Shift+F12 - Desativa Numeração / Marcadores Shift+Seta para baixo - Seleciona linhas de cima para
Teclas de atalho para o LibreOffice Writer baixo
Teclas de atalho - Efeito Ctrl+Seta para baixo - Move o cursor para o final do
Ctrl+A - Selecionar tudo parágrafo.
Ctrl+J - Justificar CtrlShift+Seta para baixo - Seleciona até o fim do pa-
Ctrl+D - Sublinhado duplo rágrafo. Ao repetir, estende a seleção até o fim do próxi-
Ctrl+E - Centralizado mo parágrafo
Ctrl+H - Localizar e substituir Home - Vai até o início da linha
Ctrl+Shift+P - Sobrescrito Home+Shift - Vai e seleciona até o início de uma linha
Ctrl+L - Alinhar à esquerda End - Vai até o fim da linha
NOÇÕES DE INFORMÁTICA

Ctrl+R - Alinhar à direita End+Shift - Vai e seleciona até o fim da linha


Ctrl+Shift+B - Subscrito Ctrl+Home - Vai para o início do documento
Ctrl+Y - Refaz a última ação Ctrl+Home+Shift - Vai e seleciona o texto até o início
Ctrl+0 (zero) - Aplica o estilo de parágrafo Padrão do documento
Ctrl+1 - Aplica o estilo de parágrafo Título 1 Ctrl+End - Vai para o fim do documento
Ctrl+2 - Aplica o estilo de parágrafo Título 2 Ctrl+End+Shift - Vai e seleciona o texto até o fim do
Ctrl+3 - Aplica o estilo de parágrafo Título 3 documento
Ctrl+4 - Aplica o estilo de parágrafo Título 4 Ctrl+PageUp - Alterna o cursor entre o texto e o ca-
Ctrl+5 - Aplica o estilo de parágrafo Título 5 beçalho

22
Ctrl+PageDown - Alterna o cursor entre o texto e o
rodapé BROFFICE PLANILHA (CALC): ATALHOS E
Insert - Ativa / Desativa modo de inserção BARRA DE FERRAMENTAS; FORMATAÇÃO DE
PageUp - Move uma página da tela para cima DADOS; SELEÇÃO DE CÉLULAS; ATRIBUTOS
Shift+PageUp - Move uma página da tela para cima DE CARACTERES
com seleção
PageDown - Move uma página da tela para baixo
Shift+PageDown - Move uma página da tela para bai-
xo com seleção O LibreOffice Calc é um programa de planilha que
Ctrl+Del - Exclui o texto até o fim da palavra você pode usar para organizar e manipular dados que
Ctrl+Backspace - Exclui o texto até o início da palavra contêm texto, números, valores de data e tempo, e mais,
Em uma lista: exclui um parágrafo vazio na frente do por exemplo para o orçamento doméstico.
parágrafo atual O Calc nos permite:
Ctrl+Del+Shift - Exclui o texto até o fim da frase • Aplicar fórmulas e funções a dados numéricos e
Ctrl+Shift+Backspace - Exclui o texto até o início da efetuar cálculos
frase • Aplicação de uma muitas formatações, como
Ctrl+Tab - Próxima sugestão com Completar palavra tipo, tamanho e coloração das fontes, impressão em co-
automaticamente lunas, alinhamento automático etc ...,
Ctrl+Shift+Tab - Utiliza a sugestão anterior com Com- • Utilização de figuras, gráficos e símbolos,
pletar palavra automaticamente • Movimentação e duplicação dos dados e fórmu-
Ctrl+Alt+Shift+V - Cola o conteúdo da área de trans- las dentro das planilhas ou para outras planilhas,
ferência como texto sem formatação. • Armazenamento de textos em arquivos, o que
Ctrl + clique duplo ou Ctrl + Shift + F10 - Utilize esta permite usá-los ou modificá-los no futuro.
combinação para encaixar ou desencaixar rapidamente
a janela do Navegador, a janela Estilos e Formatação ou Fundamentos da planilha
outras janelas Uma planilha (“sheet”) é uma grande tabela, já prepa-
rada para efetuar cálculos, operações matemáticas, pro-
jeções, análise de tendências, gráficos ou qualquer tipo
EXERCÍCIO COMENTADO de operação que envolva números.
Cada planilha se compõe de colunas e linhas, cuja in-
01. (Câmara de Pará de Minas - MG - Auxiliar de Adminis- tersecção delimita as células:
tração – Fundamental – FUMARC – 2018). Fonte padrão Colunas: Estão dispostas na posição vertical e são
de um novo documento do OpenOffice Writer 4.1.3, ver- identificadas da esquerda para a direita, começando com
são português, logo após a instalação: A até Z. Depois de Z, são utilizadas 2 letras: AA até AZ,
que são seguidas por BA até BZ, e assim por diante, até a
(A) Arial. última (IV), num total de 256 colunas.
(B) Calibri. Linhas: Estão dispostas na posição horizontal e são
(C) Times New Roman.
numeradas. Portanto, a intersecção entre linhas e colunas
(D) Verdana.
gera milhões de células disponíveis.
Cada planilha possui 1.048.576 linhas e as colunas vão
O padrão do Writer é a fonte Times New Roman.
até AMJ.
Resposta: C
Valores: Um valor pode representar um dado numéri-
co ou textual entrado pelo usuário ou pode ser resultado
02. (Câmara de Pará de Minas - MG - Assistente Admi-
de uma fórmula ou função.
nistrativo – Médio – FUMARC – 2016). São opções dispo-
Fórmulas: A fórmula é uma expressão matemática
níveis no menu “Ferramentas” do LibreOffice Writer 5.2,
versão português, EXCETO: dada ao computador (o usuário tem que montar a fór-
mula) para calcular um resultado, é a parte inteligente da
(A) Calcular planilha; sem as fórmulas a planilha seria um amontoado
(B) Estilos e formatação de textos e números.
(C) Macros Funções: São fórmulas pré-definidas para pouparem
(D) Ortografia e gramática... tempo e trabalho na criação de uma equação.
Uma célula é identificada por uma referência que
NOÇÕES DE INFORMÁTICA

Todos os itens se encontra no menu Ferramentas menos consiste na letra da coluna a célula seguida do número
o Estilos e Formatação, este se encontra no menu Estilos da linha da célula. Por exemplo, a referência de uma célu-
e na opçao Estilo e Formatação ou pela tecla F11 la na interseção da coluna A e da linha 2 é A2. Além disso,
Resposta: B a referência do intervalo de células nas colunas de A a C
e linhas 1 a 5 é A1:C5.
Endereço ou Referência: Cada planilha é formada
por linhas numeradas e por colunas ordenadas alfabeti-
camente, que se cruzam delimitando as células. Quando
se clica sobre uma delas, seleciona-se a célula.

23
Célula: corresponde à unidade básica da planilha, ou seja, cada retângulo da área de edição.
Célula Ativa: É a célula onde os dados serão digitados, ou seja, onde está o cursor no instante da entrada de dados.
Dá-se o nome Endereço ou Referência ao conjunto das coordenadas que uma célula ocupa em uma planilha. Por
exemplo, a intersecção entre a coluna B e a linha 3 é exclusiva da casela B3, portanto é a sua referência ou endereço.
A figura abaixo mostra a célula B3 ativa (ou atual, ou selecionada), ou seja, o cursor está na intersecção da linha 3
com a coluna B. (Notar que tanto a linha 3 como a coluna B destacam-se em alto relevo).

A célula ativa ou célula atual é a que está clicada, ou seja, é aquela na qual serão digitadas os dados nesse momento.
Apenas uma célula pode ficar ativa de cada vez e a seleção é representada pelas bordas da célula que ficam negri-
tadas.
Para mudar a posição da célula ativa pode-se usar o mouse ou as teclas de seta do teclado.

FIQUE ATENTO!
Você pode também incluir o nome do arquivo e o nome da folha em uma referência a uma célula ou a um
intervalo de células. Pode atribuir um nome a uma célula ou intervalo de células, para usar o nome em vez de
uma referência à coluna/número.

Layout
NOÇÕES DE INFORMÁTICA

Barra de título
A barra de título, localizada no alto da tela, mostra o nome da planilha atual. Quando a planilha for recém criada,
seu nome é Sem título X, onde X é um número. Quando a planilha é salva pela primeira vez, você é solicitado a dar um
nome a sua escolha.

24
Barra de Menus
A Barra de Menus é composta por 9 menus, que são: Arquivo, Editar, Exibir, Inserir, Formatar, Tabela, Ferramentas, Ja-
nela e Ajuda. Onde encontra-se todos os comandos do Calc.
Menu Arquivo: Este menu contém os comandos para Criar uma planilha, salva-la, abrir os documentos já salvos e os
mais recentes e por fim fechar a planilha.
Menu Editar: Responsável pelos comando de copiar, mover e excluir os dados, desfazer e refazer uma ação além de
localizar e substituir dados.
Menu Exibir: Esse menu é utilizado para estruturar a área de trabalho. A partir de suas opções, podem ser exibidas ou
ocultadas as barras do LibreOffice.Calc. Ainda podem ser trabalhadas a aparência e o tamanho da tela de trabalho.
Menu Inserir: É responsável pela inserção de elementos como linhas, colunas, comentários nas células, figuras, gráfi-
cos, fórmulas músicas, vídeos entre outros.
Menu Formatar: É responsável pela formatação de fontes e números, alinhamento dados nas células inserção de bor-
das nas células e alteração de cores tanto de fonte quanto de plano de fundo.
Menu Ferramentas: Possui comandos para proteger o documento impedindo que alterações sejam feitas, personaliza
menus e as teclas de atalho, além de possuir recurso Galeria, onde podemos selecionar figuras e sons do próprio LibreOf-
fice para inserir no documento.
Menu Dados: Seleciona um intervalo e Classifica as linhas selecionadas de acordo com as condições especificas além
de outras funções.
Menu Janela: É utilizado para Abrir uma nova janela, Fecha a janela atual, Dividir a janela atual e listar todas as janelas
do documento.
Menu Ajuda: Abre a página principal da Ajuda de OpenOffice.org.br para o aplicativo atual. Você pode percorrer as
páginas da Ajuda e procurar pelos termos do índice ou outro texto.
Você pode personalizar a Barra de menu conforme as suas necessidades, para isso, vá em Ferramentas → Personali-
zar... e vá na guia Menu.

Barra de ferramentas
Três barras de ferramentas estão localizadas abaixo da Barra de menus, por padrão: A Barra de ferramentas padrão, a
Barra de ferramentas de formatação, e a Barra de fórmulas.
Na Barra de ferramentas padrão estão várias opções tais como, gráficos, impressão, ajuda, salvar, outros.
Na Barra de ferramentas de formatação existem opções para alinhamento, numeração, recuo, cor da fonte e o outros.

Barra de fórmulas

Apresentando a barra de fórmulas.


À direita da Caixa de nome estão os botões do Assistente de Funções, de Soma, e de Função.
Clicando no botão do Assistente de Funções abre-se uma caixa de diálogo onde pode-se pesquisar em uma lista de
funções disponíveis em várias categorias como data e hora, matemática, financeira, dados, texto e outras. Isso pode ser
muito útil porque também mostra como as funções são formatadas.
Clicando no botão Soma insere-se uma fórmula na célula selecionada que soma os valores numéricos das células
acima dela. Se não houver números acima da célula selecionada, a soma será feita pelos valores das células à esquerda.
Clicando no botão Função insere-se um sinal de igual (=) na célula selecionada e na Linha de Entrada de dados, ati-
vando a célula para aceitar fórmulas.
Quando você digita novos dados numa célula, os botões de Soma e de Função mudam para os botões Cancelar e Aceitar.
O conteúdo da célula selecionada (dados, fórmula, ou função) são exibidos na Linha de Entrada de Dados, que é um
lembrete da Barra de Fórmulas. Você pode editar o seu conteúdo na própria Linha de Entrada de Dados. Para editá-la, cli-
que na Linha de Entrada de Dados e digite suas alterações. Para editar dentro da célula selecionada, clique duas vezes nela.
NOÇÕES DE INFORMÁTICA

Células individuais
A seção principal da tela exibe as células na forma de uma tabela, onde cada célula fica na interseção de uma coluna
com uma linha. É nela que colocaremos valores, referências e formatos.
No alto de cada coluna, e à esquerda de cada linha, há uma célula cinza, contendo letras (colunas) e números (linhas).
Esses são os cabeçalhos das colunas e linhas. As colunas começam em A e seguem para a direita, e as linhas começam
em 1 e seguem para baixo.
Os cabeçalhos das colunas e linhas formam a referência da célula que aparece na Caixa de Nome na Barra de Fór-
mulas. Você pode desligar esses cabeçalhos em Exibir → Cabeçalhos de Linhas e Colunas.

25
Abas de folhas
Abaixo da tabela com as células estão as abas das folhas. Essas abas permitem que você acesse cada folha da pla-
nilha individualmente, com a folha visível (ativa) estando na cor branca. Você pode escolher cores diferentes para cada
folha.
Clicando em outra aba de folha exibe-se outra folha e sua aba fica branca. Você também pode selecionar várias
folhas de uma só vez, pressionando a tecla Ctrl ao mesmo tempo que clica nas abas.

Barra de estado
Na parte inferior da janela do Calc está a barra de estado, que mostra informações sobre a planilha e maneiras con-
venientes de alterar algumas das suas funcionalidades. A maioria dos campos é semelhante aos outros componentes
do LibreOffice.
Partes esquerda e direita da barra de estado, respectivamente:

Criando uma planilha


Para criar uma nova planilha a partir de qualquer programa do Libreoffice, escolha Arquivo - Novo – Planilha.
Movendo-se em uma folha
Você pode usar o mouse ou o teclado para mover-se em uma folha do Calc ou para selecionar itens na folha. Se
selecionou um intervalo de células, o cursor permanece no intervalo ao mover o cursor.
- Para mover-se em uma folha com o mouse
Etapa
Use a barra de rolagem horizontal ou vertical para mover para os lados ou para cima e para baixo em uma folha.
• Clique na seta na barra de rolagem horizontal ou vertical.
• Clique no espaço vazio na barra de rolagem.
• Arraste a barra na barra de rolagem.
Dica – Para mover o cursor para uma célula específica, clique na célula.

- Para mover-se em uma folha com o teclado


Etapa
Use as seguintes teclas e combinações de teclas para mover-se em uma folha:
• Para mover uma célula para baixo em uma coluna, pressione a tecla de seta para baixo ou Enter.
• Para mover uma célula para cima em uma coluna, pressione a tecla de seta para cima.
• Para mover uma célula para a direita, pressione a tecla de seta para a direita ou Tab.
• Para mover uma célula para a esquerda, pressione a tecla de seta para a esquerda.
Dica – Para mover para a última célula que contém dados em uma coluna ou linha, mantenha pressionada a tecla
Ctrl enquanto pressiona uma tecla de direção.

Selecionando células em uma folha


Você pode usar o mouse ou o teclado para selecionar células em uma folha do Calc.
NOÇÕES DE INFORMÁTICA

• Para selecionar um intervalo de células com o mouse, clique em uma célula e arraste o mouse para outra célula.
• Para selecionar um intervalo de células com o teclado, certifique-se de que o cursor esteja em uma célula, mante-
nha pressionada a tecla Shift e pressione uma tecla de direção.

26
Dica – Para copiar sem alterar os valores em uma sé-
rie, pressione a tecla Ctrl enquanto arrasta.

Digitando ou colando dados Editando e excluindo o conteúdo de células


A maneira mais simples de adicionar dados a uma fo- Você pode editar o conteúdo de uma célula ou inter-
lha é digitar, ou copiar e colar dados de outra folha do valo de células em uma folha.
Calc ou de outro programa. - Para editar o conteúdo de células em uma folha
- Para digitar ou colar dados em uma planilha Etapas
Etapas 1. Clique em uma célula ou selecione um intervalo de
1. Clique na célula à qual deseja adicionar dados. células.
2. Digite os dados. Dica – Para selecionar um intervalo de células, clique
Se desejar colar dados da área de transferência na cé- em uma célula. Em seguida arraste o mouse até cobrir o
lula, escolha Editar - Colar. intervalo que deseja selecionar.
3. Pressione Enter. Para selecionar uma linha ou coluna inteira, clique no
Você pode também pressionar uma tecla de direção rótulo da linha ou coluna.
para inserir os dados e mover a próxima célula na direção 2. Para editar o conteúdo de uma única célula, cli-
da seta. que duas vezes na célula, faça as alterações necessárias e
pressione Return.
Observação – Pode também clicar na célula, digitar as
#FicaDica alterações na caixa de Linha de entrada da barra Fórmula
Para digitar texto em mais de uma linha em uma e clicar no ícone verde da marca de seleção.
célula, pressione Ctrl+Return no fim de cada li- No entanto, não pode inserir quebras de linha na cai-
nha e, quando concluir, pressione Return. xa de Linha de entrada.
3. Para excluir o conteúdo da célula ou do intervalo de
células, pressione a tecla Backspace ou Delete.
- Para inserir rapidamente datas e números consecu- a. Na caixa de diálogo Excluir conteúdo, selecione as
tivos opções que deseja.
O Calc oferece um recurso de preenchimento para b. Clique em OK.
você inserir rapidamente uma série sucessiva de dados,
como datas, dias, meses e números. O conteúdo de cada Formatando planilhas
célula sucessiva na série é incrementado por um. 1 é in- O Calc permite-lhe formatar a folha manualmente ou
crementado para 2, segunda-feira é incrementada para ao usar estilos. A diferença principal é que a formatação
terceira-feira, e assim por diante. manual aplica-se apenas às células selecionadas. A for-
Etapas matação de estilo aplica-se toda vez que o estilo é usado
1. Clique em uma célula e digite o primeiro item da no documento de planilha.
série, por exemplo, segunda-feira. Pressione Return.
2. Clique na célula novamente para ver a alça de
preenchimento — a caixa preta pequena no canto direito
NOÇÕES DE INFORMÁTICA

inferior da célula.
3. Arraste a alça de preenchimento até realçar o inter-
valo de células no qual deseja inserir a série.
4. Solte o botão do mouse.
Os itens consecutivos na série são adicionados auto-
maticamente às células realçadas.

27
Usando AutoFormatação
A maneira mais fácil de formatar um intervalo de células é usar o recurso AutoFormatação do Calc.

- Para aplicar formatação automática a um intervalo de células


Etapas
1. Selecione o intervalo de células que deseja formatar.
Selecione ao menos um intervalo de células 3 x 3.
2. Escolha Formatar - AutoFormatar.
Abre-se a caixa de diálogo AutoFormatação.
3. Na lista de formatos, clique no formato que deseja usar e clique em OK.

Formatando células manualmente


Para aplicar formatação simples ao conteúdo de uma célula, como alterar o tamanho do texto, use os ícones na
barra Formatar objeto.
- Para formatar células com a barra Formatar objeto
A barra Formatar objetos permite-lhe aplicar formatos rapidamente a células individuais ou intervalos de células.
Etapas
1. Selecione a célula ou o intervalo de células que deseja formatar.
2. Na barra Formatar objeto, clique no ícone que corresponde à formatação que deseja aplicar.
Observação – Pode também selecionar uma opção das caixas Nome da fonte ou Tamanho da fonte.

- Para aplicar formatação manual com a caixa de diálogo Formatar células


Se precisar de mais opções de formatação do que a barra Objeto fornece, use a caixa de diálogo Formatar células.
NOÇÕES DE INFORMÁTICA

28
Etapas - Para aplicar formatação com a janela Estilos e for-
1. Selecione a célula ou o intervalo de células que de- matação
seja formatar e escolha Formatar - Células. Etapas
Abre-se a caixa de diálogo Formatar células. 1. Escolha Formatar - Estilos e formatação.
2. Clique em uma das guias e escolha as opções de 2. Para alterar a formatação de células, clique em uma
formatação. célula ou selecione um intervalo de células.
a. Clique no ícone Estilos de célula na parte superior
Guia Números da janela Estilos e formatação.
Altera a formatação de números nas células, como a b. Clique duas vezes em um estilo na lista.
alteração do número de casas decimais exibidas 3. Para alterar o layout de uma folha, clique em qual-
quer lugar na folha.
Guia Fonte a. Clique no ícone Estilos de página na parte superior
Altera a fonte, o tamanho da fonte e o tipo de letra da janela Estilos e formatação.
usado na célula b. Clique duas vezes em um estilo na lista.

Guia Efeitos de fonte Usando fórmulas e funções


Altera a cor da fonte e os efeitos de sublinhado, ta- Você pode inserir fórmulas em uma planilha para efe-
chado ou alto-relevo do texto tuar cálculos.
Se a fórmula contiver referências a células, o resulta-
Guia Alinhamento do será atualizado automaticamente toda vez que você
Altera o alinhamento do texto e a orientação do texto alterar o conteúdo das células. Você pode também usar
no interior das células uma das várias fórmulas ou funções pré-definidas que o
Calc oferece para efetuar cálculos.
Guia Bordas
Altera as opções de bordas das células Criando fórmulas
Uma fórmula começa com um sinal de igual (=) e
Guia Plano de fundo pode conter valores, referências a células, operadores,
Altera o preenchimento do plano de fundo das cé- funções e constantes.
lulas
- Para criar uma fórmula
Guia Proteção de célula Etapas
Protege o conteúdo das células no interior de folhas 1. Clique na célula à qual deseja exibir o resultado da
protegidas. fórmula.
3. Clique em OK. 2. Digite = e, a seguir, digite a fórmula.
Por exemplo, se desejar adicionar o conteúdo da cé-
Formatando células e folhas com estilos lula A1 ao conteúdo da célula A2, digite =A1+A2 em ou-
No Calc, a formatação padrão de células e folhas fa- tra célula.
z-se com estilos. Um estilo é um conjunto de opções de 3. Pressione Return.
formatação que define o aspecto do conteúdo da célula,
assim como o layout de uma folha. Quando você altera a
formatação de um estilo, as alterações são aplicadas toda
vez que o estilo é usado na planilha.

NOÇÕES DE INFORMÁTICA

29
Usando operadores
Você pode usar os seguintes operadores nas fórmulas:

Exemplo de Fórmulas do Calc

=A1+15
Exibe o resultado de adicionar 15 ao conteúdo da células A1
=A1*20%
Exibe 20 por cento do conteúdo da célula A1
=A1*A2
Exibe o resultado da multiplicação do conteúdo das células A1 e A2

Usando parênteses

O Calc segue a ordem de operações ao calcular uma fórmula. Multiplicação e divisão são feitas antes de adi-
ção e subtração, independentemente de onde esses operadores aparecem na fórmula. Por exemplo, para a fórmula
=2+5+5*2, o Calc retorna o valor de 17 e não de 24.

Editando uma fórmula


Uma célula que contém uma fórmula exibe apenas o resultado da fórmula. A fórmula é exibida na caixa de Linha
de entrada.
NOÇÕES DE INFORMÁTICA

- Para editar uma fórmula


Etapas
1. Clique em uma célula que contém uma fórmula.
A fórmula é exibida na caixa de Linha de entrada da barra Fórmula.

30
* Você também pode editar uma célula pressionado F2 ou dando um clique duplo na célula
2. Clique na caixa de Linha de entrada e efetue as alterações.
Para excluir parte da fórmula, pressione a tecla Delete ou Backspace.
3. Pressione Return ou clique no ícone na barra Fórmula para confirmar as alterações.
Para rejeitar as alterações feitas, pressione Esc ou clique no ícone na barra Fórmula.

Usando funções
O Calc é fornecido com várias fórmulas e funções predefinidas. Por exemplo, em vez de digitar =A2+A3+A4+A5,
você pode digitar =SUM(A2:A5) .

- Para usar uma função


Etapas
1. Clique na célula à qual deseja adicionar uma função.
2. Escolha Inserir – Função.
Abre-se a caixa de diálogo Assistente de função.
3. Na caixa Categoria, selecione a categoria que contém o tipo de função que você deseja usar.
4. Na lista Funções, clique na função que deseja usar.
5. Clique em Próximo.
6. Insira os valores necessários ou clique nas células que contêm os valores que você deseja.
7. Clique em OK.
NOÇÕES DE INFORMÁTICA

31
Usando gráficos
Gráficos podem ajudar a visualizar padrões e tendências nos dados numéricos.
O LibreOffice fornece vários estilos de gráfico que você pode usar para representar os números.
Observação – Gráficos não se restringem a planilhas. Você pode também inserir um gráfico ao escolher Inserir - Ob-
jeto - Gráfico nos outros programas do LibreOffice.

- Para criar um gráfico


Etapas
1. Selecione as células, inclusive os títulos, que contêm dados para o gráfico.
2. Escolha Inserir – Gráfico.
Abre-se a caixa de diálogo Assistente de Graficos. O intervalo de célula selecionado é exibido na caixa Intervalo.
Observação – Se desejar especificar um intervalo de célula diferente para os dados, clique no botão Encolher ao
lado da caixa de texto Intervalo e selecione as células. Clique no botão Encolher novamente quando concluir.
3. Clique em Próximo.
4. Na caixa Escolher tipo de gráfico, clique no tipo de gráfico que deseja criar.
5. Clique em Próximo.
6. Na caixa Escolher variante, clique na variante que deseja usar.
7. Clique em Próximo.
8. Na caixa Título do gráfico, digite o nome do gráfico.
9. Clique em Concluir.
NOÇÕES DE INFORMÁTICA

Editando gráficos
Depois de criar um gráfico, poderá voltar e alterar, mover, redimensionar ou excluir o gráfico.
- Para redimensionar, mover ou excluir um gráfico
Etapa
Clique no gráfico e siga um destes procedimentos:
• Para redimensionar o gráfico, mova o ponteiro do mouse sobre uma das alças, pressione o botão do mouse e
arraste o mouse.

32
O Calc exibe uma linha pontilhada do novo tamanho do gráfico enquanto você arrasta.
• Para mover o gráfico, mova o ponteiro do mouse para dentro do gráfico, pressione o botão do mouse e arraste o
mouse para um novo lugar.
• Para excluir o gráfico, pressione a tecla Delete.

- Para alterar a aparência de um gráfico


Você pode usar os ícones na barra de ferramentas Padrão do gráfico para alterar a aparência do gráfico.
Etapas
1. Clique duas vezes em um gráfico para exibir a barra de ferramentas Padrão do gráfico.
A barra de ferramentas aparece ao lado da barra padrão do Calc ou Writer.

2. Use os ícones na barra de ferramentas para alterar as propriedades do gráfico.


3. Para modificar outras opções do gráfico você pode dar um clique duplo sobre o respectivo elemento ou acessar
as opções através do menu Inserir e Formatar.

Navegando dentro das planilhas


O Calc oferece várias maneiras para navegar dentro de uma planilha de uma célula para outra, e de uma folha para
outra. Você pode utilizar a maneira que preferir.

Indo para uma célula específica


Utilizando o mouse: posicione o ponteiro do mouse sobre a célula e clique.
Utilizando uma referência de célula: clique no pequeno triângulo preto invertido na Caixa de nome. A referência
da célula selecionada ficará destacada. Digite a referência da célula que desejar e pressione a tecla Enter. Ou, clique na
Caixa de nome, pressione a tecla backspace para apagar a referência da célula selecionada. Digite a referência de célula
que desejar e pressione Enter.
Utilizando o Navegador: para abrir o Navegador, clique nesse ícone na Barra de ferramentas padrão, ou pres-
sione a tecla F5, ou clique em Exibir → Navegador na Barra de menu, ou clique duas vezes no Número Sequencial das
Folhas na Barra de estado. Digite a referência da célula nos dois campos na parte superior, identificados como Coluna
e Linha, e pressione Enter.
Você pode embutir o Navegador em qualquer lado da janela principal do Calc, ou deixá-lo flutuando. (Para embutir
ou fazer flutuar o navegador, pressione e segure a tecla Ctrl e clique duas vezes em uma área vazia perto dos ícones
dentro da caixa de diálogo do Navegador.)

NOÇÕES DE INFORMÁTICA

Figura 5: Apresentando o navegador.

O Navegador exibe listas de todos os objetos em um documento, agrupados em categorias. Se um indicador (sinal
de mais (+) ou seta) aparece próximo a uma categoria, pelo menos um objeto daquele tipo existe. Para abrir uma cate-
goria e visualizar a lista de itens, clique no indicador.
Para esconder a lista de categorias e exibir apenas os ícones, clique no ícone de Conteúdo. Clique neste ícone
de novo para exibir a lista.

33
Movendo-se de uma célula para outra Inserindo colunas e linhas
Em uma planilha, normalmente, uma célula possui Você pode inserir colunas e linhas individualmente ou
uma borda preta. Essa borda preta indica onde o foco em grupos.
está. Se um grupo de células estiver selecionado, elas são Coluna ou linha única
destacadas com a cor azul, enquanto a célula que possui Utilizando o menu Inserir:
o foco terá uma borda preta. Selecione a célula, coluna ou linha onde você quer
Utilizando o mouse: para mover o foco utilizando o inserir a nova coluna ou linha.
mouse, simplesmente coloque o ponteiro dele sobre a Clique em Inserir → Colunas ou Inserir → Linhas.
célula que deseja e clique com o botão esquerdo. Isso Utilizando o mouse:
muda o foco para a nova célula. Esse método é mais útil Selecione a célula, coluna ou linha onde você quer
quando duas células estão distantes uma da outra. inserir a nova coluna ou linha.
Utilizando as teclas de Tabulação e Enter Clique com o botão direito do mouse no cabeçalho
Pressionando Enter ou Shift+Enter move-se o foco da coluna ou da linha.
para baixo ou para cima, respectivamente. Clique em Inserir Linhas ou Inserir Colunas.
Pressionando Tab ou Shift+Tab move-se o foco para a
esquerda ou para a direita, respectivamente. Múltiplas colunas ou linhas
Utilizando as teclas de seta Você pode inserir várias colunas ou linhas de uma só
Pressionando as teclas de seta do teclado move-se o vez, ao invés de inseri-las uma por uma.
foco na direção das teclas. Selecione o número de colunas ou de linhas pressio-
Utilizando as teclas Home, End, Page Up e Page Down nando e segurando o botão esquerdo do mouse na pri-
A tecla Home move o foco para o início de uma linha. meira e arraste o número necessário de identificadores.
A tecla End move o foco para a ultima célula à direita Proceda da mesma forma, como fosse inserir uma
que contenha dados. única linha ou coluna, descrito acima.
A tecla Page Down move uma tela completa para bai-
xo e a tecla Page Up move uma tela completa para cima.
Combinações da tecla Ctrl e da tecla Alt com as te-
clas Home, End, Page Down, Page Up, e as teclas de seta
movem o foco da célula selecionada de outras maneiras.

Localização e substituição de dados


Este recurso é muito útil quando há a necessidade de
serem localizados e substituídos dados em planilhas.
Para localizar e substituir escolha o menu Editar→Lo-
calizar e substituir ou pressione Ctrl + H; Você pode so-
mente localizar, ou localizar e substituir;

Inserindo 3 linhas abaixo da linha 1.

Apagando colunas e linhas


Colunas e linhas podem ser apagadas individualmen-
te ou em grupos.
Coluna ou linha única
Uma única coluna ou linha pode ser apagada utilizan-
do-se o mouse:
Selecione a coluna ou linha a ser apagada.
Clique com o botão direito do mouse no identificador
da coluna ou linha.
Selecione Excluir Colunas ou Excluir Linhas no menu
de contexto.

Múltiplas colunas e linhas


NOÇÕES DE INFORMÁTICA

Você pode apagar várias colunas ou linhas de uma


vez ao invés de apagá-las uma por uma.
Selecione as colunas que deseja apagar, pressionando
Trabalhando com colunas e linhas o botão esquerdo do mouse na primeira e arraste o nú-
mero necessário de identificadores.
Proceda como fosse apagar uma única coluna ou li-
nha acima.

34
Ctrl+Page Up / Move uma planilha para a esquerda.
Na visualização de impressão: Move para a página de
impressão anterior.
Ctrl+Page Down / Move uma planilha para a direita.
Na visualização de impressão: Move para a página de
impressão seguinte.
Alt+Page Up / Move uma tela para a esquerda.
Alt+Page Down / Move uma página de tela para a direita.
Shift+Ctrl+Page Up / Adiciona a planilha anterior à
seleção de planilhas atual. Se todas as planilhas de um
documento de planilha forem selecionadas, esta combi-
nação de teclas de atalho somente selecionará a planilha
anterior. Torna atual a planilha anterior.
Shift+Ctrl+Page Down / Adiciona a próxima planilha
à seleção de planilhas atual. Se todas as planilhas de um
Excluir 3 linhas abaixo da linha 1. documento de planilha forem selecionadas, esta combi-
nação de teclas de atalho somente selecionará a próxima
Teclas de atalho para planilhas planilha. Torna atual a próxima planilha.
Ctrl+ * onde (*) é o sinal de multiplicação no teclado
Navegar em planilhas numérico
Teclas de atalho/Efeito Seleciona o intervalo de dados que contém o cursor.
Ctrl+Home / Move o cursor para a primeira célula Um intervalo é um intervalo de células contíguas que
na planilha (A1). contém dados e é delimitado por linhas e colunas vazias.
Ctrl+End / Move o cursor para a última célula que Ctrl+ / onde (/) é o sinal de divisão no teclado nu-
contém dados na planilha. mérico
Home / Move o cursor para a primeira célula da linha Seleciona o intervalo de fórmulas de matriz que con-
atual. tém o cursor.
End / Move o cursor para a última célula da linha atual. Ctrl+tecla de adição / Insere células (como no menu
Shift+Home / Seleciona todas as células desde a atual Inserir - Células)
até a primeira célula da linha. Ctrl+tecla de subtração / Exclui células (tal como no
Shift+End / Seleciona todas as células desde a atual menu Editar - Excluir células)
até a última célula da linha. Enter ( num intervalo selecionado) / Move o cursor
Shift+Page Up / Seleciona as células desde a atual até uma célula para baixo no intervalo selecionado. Para es-
uma página acima na coluna ou extende a seleção exis- pecificar a direção do movimento do cursor, selecione
tente uma página para cima. Ferramentas - Opções - LibreOffice Calc - Geral.
Shift+Page Down / Seleciona as células desde a atual Ctrl+ ` / Exibe ou oculta as fórmulas em vez dos valo-
até uma página abaixo na coluna ou estende a seleção res em todas as células.
existente uma página para baixo. A tecla ` está ao lado da tecla “1” na maioria dos tecla-
Ctrl+Seta para a esquerda / Move o cursor para o dos em Inglês. Se seu teclado não possui essa tecla, você
canto esquerdo do intervalo de dados atual. Se a coluna pode atribuir uma outra tecla: Selecione Ferramentas -
à esquerda da célula que contém o cursor estiver vazia, o Personalizar, clique na guia Teclado. Selecione a categoria
cursor se moverá para a esquerda da próxima coluna que “Exibir” e a função “Exibir fórmula”.
contenha dados.
Ctrl+Seta para a direita / Move o cursor para o canto Teclas de função utilizadas em planilhas
direito do intervalo de dados atual. Se a coluna à direita Teclas de atalho / Efeito
da célula que contém o cursor estiver vazia, o cursor se Ctrl+F1 / Exibe a anotação anexada na célula atual
moverá para a direita da próxima coluna que contenha F2 / Troca para o modo de edição e coloca o cursor no
dados. final do conteúdo da célula atual. Pressione novamente
Ctrl+Seta para cima / Move o cursor para o canto para sair do modo de edição.
superior do intervalo de dados atual. Se a linha acima Se o cursor estiver em uma caixa de entrada de uma
da célula que contém o cursor estiver vazia, o cursor se caixa de diálogo que possui o botão Encolher, a caixa de
moverá para cima da próxima linha que contenha dados. diálogo ficará oculta e a caixa de entrada permanecerá
Ctrl+Seta para cima / Move o cursor para o canto in- visível. Pressione F2 novamente para mostrar a caixa de
NOÇÕES DE INFORMÁTICA

ferior do intervalo de dados atual. Se a linha abaixo da diálogo inteira.


célula que contém o cursor estiver vazia, o cursor se mo- Ctrl+F2 / Abre o Assistente de funções.
verá para baixo da próxima linha que contenha dados. Shift+Ctrl+F2 / Move o cursor para a Linha de entrada
Ctrl+Shift+Seta / Seleciona todas as células conten- onde você pode inserir uma fórmula para a célula atual.
do dados da célula atual até o fim do intervalo contínuo Ctrl+F3 / Abre a caixa de diálogo Definir nomes.
das células de dados, na direção da seta pressionada. Um F4 / Mostra ou oculta o Explorador de Banco de dados.
intervalo de células retangular será selecionado se esse Shift+F4 / Reorganiza as referências relativas ou ab-
grupo de teclas for usado para selecionar linhas e colu- solutas (por exemplo, A1, $A$1, $A1, A$1) no campo de
nas ao mesmo tempo. entrada.

35
F5 / Mostra ou oculta o Navegador.
Shift+F5 / Rastreia dependentes. EXERCÍCIO COMENTADO
Shift+F7 / Rastreia precedentes.
Shift+Ctrl+F5 / Move o cursor da Linha de entrada
para a caixa Área da planilha. 01. (Câmara de Lagoa da Prata - MG - Assistente
F7 / Verifica a ortografia na planilha atual. Administrativo – Médio – FUMARC – 2016) - Adaptado.
Ctrl+F7 / Abre o Dicionário de sinônimos se a célula São opções disponíveis no menu “Inserir” do LibreOffice
atual contiver texto. Calc, versão português, EXCETO:
F8 / Ativa ou desativa o modo de seleção adicional. (A) Tabela dinâmica...
Nesse modo, você pode usar as teclas de seta para esten- (B) Linhas...
der a seleção. Você também pode clicar em outra célula (C) Gráfico...
para estender a seleção. (D) Figura...
Ctrl+F8 / Realça células que contém valores.
F9 / Recalcula as fórmulas modificadas na planilha Na nova versão o Calc ganhou um novo menu que é o
atual. ‘’ MENU PLANILHA’’, esse menu agora é responsável por
Ctrl+Shift+F9 / Recalcula todas as fórmulas em todas tratar especificamente das particularidades da planilha,
as planilhas. por ex : inserir coluna, linhas, células, quebra de página,
Ctrl+F9 / Atualiza o gráfico selecionado. exclui linhas, colunas, células, inserir planilha, renome-
F11 Abre a janela Estilos e formatação para você ar planilhas, preencher células, dentre outras funções.
aplicar um estilo de formatação ao conteúdo da célula Resposta: B
ou à planilha atual.
Shift+F11 / Cria um modelo de documento. 02. (Câmara de Lagoa da Prata - MG - Assistente
Shift+Ctrl+F11 / Atualiza os modelos. Administrativo – Médio – FUMARC – 2016) - Adaptado.
F12 / Agrupa o intervalo de dados selecionado. Função do LibreOffice Calc, versão português, que
Ctrl+F12 / Desagrupa o intervalo de dados selecio- retorna a soma de todos os argumentos:
nado. (A) ADICAO
Alt+Seta para baixo / Aumenta a altura da linha atual (B) SOMAR
(somente no Modo de compatibilidade legada do Ope- (C) SOMAS
nOffice.org). (D) SOMA
Alt+Seta para cima / Diminui a altura da linha atual
(somente no Modo de compatibilidade legada do Ope- A função SOMA, uma das funções de matemática e
nOffice.org). trigonometria, adiciona valores. É possível adicionar
Alt+Seta para a direita / Aumenta a largura da coluna valores individuais, referências de célula ou intervalos,
atual. ou uma mistura dos três.
Alt+Seta para a esquerda / Diminui a largura da co- Por exemplo:
luna atual. =SOMA(A2:A10)
Alt+Shift+Tecla de seta / Otimiza a largura da coluna Resposta: D
ou o tamanho da linha com base na célula atual.

Formatar células com as teclas de atalho


Os formatos de célula a seguir podem ser aplicados
com o teclado:
Teclas de atalho / Efeito
Ctrl+1 (não use o teclado numérico) / Abre a caixa de
diálogo Formatar células
Ctrl+Shift+1 (não use o teclado numérico) / Duas ca-
sas decimais, separador de milhar
Ctrl+Shift+2 (não use o teclado numérico) / Formato
exponencial padrão
Ctrl+Shift+3 (não use o teclado numérico) / Formato
de data padrão
Ctrl+Shift+4 (não use o teclado numérico) / Formato
monetário padrão
NOÇÕES DE INFORMÁTICA

Ctrl+Shift+5 (não use o teclado numérico) / Formato


de porcentagem padrão (duas casas decimais)
Ctrl+Shift+6 (não use o teclado numérico) / Formato
padrão

36
04. (Câmara de Lagoa da Prata - MG - Assistente Ad-
HORA DE PRATICAR ministrativo – Médio – FUMARC – 2016)
Analise as seguintes afirmativas sobre os atalhos de te-
clado do LibreOffice Writer 5.2, versão português:
I – “Ctrl+N” aplica a formatação “Negrito” ao texto sele-
01. (Câmara de Pará de Minas - MG - Auxiliar de Ad- cionado.
ministração – Fundamental – FUMARC – 2018) II – “Ctrl+U” aplica a formatação “Sublinhado” ao texto
Texto formatado no Microsoft Word, versão português selecionado.
do Office 2013, que utiliza as opções e os efeitos de for- III – “Ctrl+I” aplica a formatação “Itálico” ao texto sele-
matação sublinhado, itálico, tachado e versalete: cionado.

(A) Estão CORRETAS as afirmativas:


(A) I e II, apenas.
(B) (B) I e III, apenas.
(C) II e III, apenas.
(C) (D) I, II e III.

(D) 05. (Prefeitura de Matozinhos - MG - Assistente Ad-


ministrativo – Médio – FUMARC – 2016)
02. (COPASA - Analista de Saneamento - Administra- São exemplos de gráfico de “Dispersão” do Microsoft Ex-
dor – Superior – FUMARC – 2018) cel, versão português do Office 2010, EXCETO:
Analise as seguintes afirmativas sobre as opções dispo-
níveis na guia “MENSAGEM” quando uma mensagem
enviada pelo Microsoft Outlook, versão português do
(A)
Office 2013, estiver aberta na tela do computador:
I – Para “Encaminhar” a mensagem a outros destinatá-
rios, basta acionar o atalho de teclado “Ctrl + Shift + R”.
II – Para encaminhar a mensagem como um anexo de
(B)
uma nova mensagem, basta selecionar a opção “Mais”
do grupo “Responder” e, em seguida, selecionar a opção
“Encaminhar como Anexo”.
III – Para cancelar uma mensagem enviada, basta acionar (C)
a opção “Ações” do grupo “Mover” e, em seguida, sele-
cionar a opção “Cancelar Mensagem Enviada...”.

Estão CORRETAS as afirmativas: (D)


(A) I e II, apenas.
(B) I e III, apenas.
(C) I, II e III. 06. (Prefeitura de Matozinhos - MG - Advogado – Su-
(D) II e III, apenas. perior – FUMARC – 2016) - Adaptado
Função do Microsoft Excel, versão português do Office,
03. (Câmara de Lagoa da Prata - MG - Assistente Ad- que verifica se uma condição foi satisfeita e retorna um
ministrativo – Médio – FUMARC – 2016) valor se for verdadeiro ou retorna outro valor se for falso:
Opção da barra de ferramentas “Padrão” do LibreOffice (A) CONT.SE
Writer 5.2, versão português, que permite inserir uma (B) SE
nota de rodapé: (C) SEERRO
(D) SOMASE

07. (Câmara de Lagoa da Prata - MG - Assistente Ad-


(A) ministrativo – Médio – FUMARC – 2016) - adaptado
Em relação às opções disponíveis na barra de ferramen-
tas “Padrão” do LibreOffice Calc, versão português, cor-
(B) relacione as colunas a seguir:
NOÇÕES DE INFORMÁTICA

(C)

(D)

37
Está CORRETA a seguinte sequência de respostas: 11. (Prefeitura de Matozinhos - MG - Advogado – Su-
perior – FUMARC – 2016)
(A) III, I, IV, II. A opção “Dispositivos e Impressoras” pode ser encon-
(B) IV, I, II, III. trada no “Painel de Controle” do Microsoft Windows 7,
(C) IV, I, III, II. versão português, dentro da categoria:
(D) IV, III, II, I.
(A) Aparência e Personalização.
08. (Câmara de Pará de Minas - MG - Auxiliar de Ad- (B) Hardware e Sons.
ministração – Fundamental – FUMARC – 2018) - adap- (C) Rede e Internet.
tado (D) Sistema e Segurança. (A) Contas de Usuário e Segu-
Atalho de teclado do OpenOffice Calc, versão português, rança Familiar
que abre a janela “Formatar células” para formatação de
Números, Fonte, Borda, dentre outras opções, nas células 12. (Prefeitura de Matozinhos - MG - Assistente Ad-
selecionadas: ministrativo – Médio – FUMARC – 2016)
A possibilidade de alteração do fuso horário pode ser
(A) Ctrl+1. feita no “Painel de Controle” do Microsoft Windows 7,
(B) Ctrl+2. versão português, dentro da categoria:
(C) Ctrl+D.
(D) Ctrl+F. (A) Aparência e Personalização.
(B) Facilidade de Acesso.
09. (Prefeitura de Matozinhos - MG - Advogado – Su- (C) Relógio, Idioma e Região.
perior – FUMARC – 2016) (D) Sistema e Segurança.
Analise as seguintes afirmativas sobre as opções disponí-
veis na guia “Página Inicial” do Microsoft Outlook, versão
português do Office 2010:
GABARITO
I– Permite mover ou copiar um item.

1 A
2 D
II – Permite responder ao remetente.
3 D
4 C
5 A
III – Permite encaminhar o item.
6 B
7 C
Estão CORRETAS as afirmativas:
8 A
(A) I, II e III. 9 D
(B) I e II, apenas. 10 C
(C) I e III, apenas.
(D) II e III, apenas. 11 B
12 C
10. (SEBRAE-NACIONAL - Agente de Inovação – Su-
perior – FUMARC – 2013)
Sobre tecnologias e aplicações da internet, todas as afir-
mativas estão corretas, EXCETO:

(A) Google Chrome, Internet Explorer, Opera e Safari são


exemplos de navegadores para acesso à Internet.
(B) O Facebook é uma rede social em que usuários criam
perfis com fotos e listas de interesses pessoais e trocam
NOÇÕES DE INFORMÁTICA

mensagens públicas ou privadas entre si.


(C) Cloud Computing ou Computação em Nuvem foi criada
com o objetivo exclusivo de armazenar dados de usuários
ou empresas em discos virtuais espalhados pelo mundo.
(D) Um dos principais objetivos do HTML5 é facilitar a
manipulação de elementos, possibilitando que o desen-
volvedor seja capaz de modificar características de obje-
tos de uma forma não intrusiva e transparente ao usuário
final.

38
ÍNDICE

LEGISLAÇÃO ESPECÍFICA
Lei Orgânica do Município de São José dos Pinhais; ...............................................................................................................................................01
Constituição da República Federativa do Brasil; Dos direitos e garantias fundamentais - Capítulo I (dos direitos e deveres indivi-
duais e coletivos); Da organização do Estado; Da defesa do Estado e das Instituições Democráticas - Capítulo III (da segurança
pública); .....................................................................................................................................................................................................................................14
Declaração Universal dos Direitos Humanos de 1948; ............................................................................................................................................37
Lei Ordinária nº 10.741, de 1º de outubro de 2003. (Estatuto do Idoso); Dos direitos fundamentais; Das medidas de proteção;
Dos crimes; ...............................................................................................................................................................................................................................47
Lei Ordinária nº 10.826, de 22 de dezembro de 2003. (Estatuto do Desarmamento); ..............................................................................56
Decreto nº 5.123, de 1º de Julho de 2004; ..................................................................................................................................................................63
Portaria Interministerial 4.226, de 31 de Dezembro de 2010; .............................................................................................................................13
Lei nº 11.340, de 7 de Agosto de 2006 (Lei Maria da Penha); .............................................................................................................................74
Lei Ordinária nº 8.069, de 13 de julho de 1990 (Estatuto da Criança e do Adolescente); Dos direitos fundamentais; Da prática do
ato infracional; .........................................................................................................................................................................................................................83
Lei 9.503, de 23 de setembro de 1997. (Código de Trânsito Brasileiro); Disposições preliminares; Do sistema nacional de trânsito;
Das normas gerais de circulação e conduta; Da habilitação; Dos crimes de trânsito............................................................................... 138
Da competência do município
LEI ORGÂNICA DO MUNICÍPIO DE SÃO
JOSÉ DOS PINHAIS/PR. Competência privativa
Ao Município compete prover a tudo quanto diga res-
peito ao seu peculiar interesse e ao bem-estar de sua
população, cabendo-lhe, privativamente, dentre ou-
Do Município tras, as seguintes atribuições:
O Município de São José dos Pinhais é uma pessoa
I - legislar sobre assuntos de interesse local;
jurídica de direito público interno, com autonomia po-
II - suplementar a legislação federal e a estadual, no
lítica, administrativa e financeira, regida pela sua Lei
que couber;
Orgânica, a qual foi votada e aprovada por sua Câmara
Municipal. III - elaborar o Plano Diretor de Desenvolvimento In-
Os poderes Legislativos e Executivos compõem o go- tegrado;
verno do município, sendo harmônicos e independen- IV - criar, organizar e suprimir Distritos, observada a
tes entre si. Não obstante, são símbolos do município legislação estadual e esta Lei Orgânica;
o hino e o brasão, os quais representam sua cultura. V - manter, com a cooperação técnica e financeira da
Por fim, são bens do município as coisas móveis e imó- União e do Estado, programas de educação em todos
veis, direitos e ações de qualquer título que a pertençam. os níveis, observadas as prescrições das Constituições
Federal e Estadual;
Da divisão administrativa do município VI - elaborar o orçamento anual e plurianual de inves-
De acordo com o artigo 5º da Lei, “O Município po- timentos;
derá dividir-se, para fins administrativos, em distritos VII - instituir e arrecadar tributos, bem como aplicar as
a serem criados, organizados, suprimidos ou fundidos suas rendas;
por lei após consulta plebiscitária à população direta- VIII - fixar, fiscalizar e cobrar tarifas ou preços públicos;
mente interessada, observada a legislação estadual e IX - dispor sobre organização, administração e execu-
o atendimento aos requisitos estabelecidos no Art. 6º ção dos serviços locais;
desta Lei Orgânica”. X - dispor sobre administração e utilização dos bens
São requisitos para a criação do Distrito: públicos;
I - população, eleitorado e arrecadação não inferiores XI - organizar o quadro e estabelecer o regime jurídico
à quinta parte exigida para a criação do Município; único dos servidores públicos;
II - existência, na povoação-sede, de pelo menos cin- XII - organizar e prestar, diretamente, ou sob regime
qüenta moradias, escola pública de ensino fundamen- jurídico de concessão ou permissão, os serviços locais;
tal completa, posto de saúde e posto policial, em áre- XIII - planejar o uso e a ocupação do solo em seu ter-
as adequadas para a existência destes equipamentos ritório, especialmente em sua zona urbana;
comunitários. XIV - estabelecer normas de edificação, de loteamen-
A comprovação de atendimento às exigências enume-
to, de arruamento e de zoneamento urbano e rural,
radas acima far-se-á mediante:
bem como as limitações urbanísticas convenientes à
a) declaração, emitida pela Fundação Instituto Brasi-
ordenação do seu território, observada a Lei Federal;
leiro de Geografia e Estatística, de estimativa de po-
pulação; XV - conceder e renovar licença para localização e
b) certidão, emitida pelo Tribunal Regional Eleitoral, funcionamento de estabelecimentos industriais, co-
certificando o número de eleitores; merciais prestadoras de serviços e quaisquer outras;
c) certidão, emitida pelo agente municipal de estatísti- XVI - cassar a licença que houver concedido ao esta-
ca ou pela repartição fiscal do Município, certificando belecimento que se tornar prejudicial à saúde, à higie-
o número de moradias; ne, ao sossego, à segurança, aos bons costumes e ao
d) certidão do órgão fazendário estadual e do mu- meio ambiente, fazendo cessar a atividade ou deter-
nicipal, certificando a arrecadação na respectiva área minando o fechamento do estabelecimento;
territorial; XVII - estabelecer servidões administrativas necessá-
e) certidão emitida pela Prefeitura ou pelas Secretarias rias à realização de seus serviços, inclusive à dos seus
de Educação, de Saúde e de Segurança Pública do Es- concessionários;
tado, certificando a existência da escola pública e dos XVIII - regular a disposição, o traçado e as demais con-
postos de saúde e policial na povoação sede. dições dos bens públicos de uso comum;
Na fixação das divisas distritais serão observadas as XIX - regulamentar a utilização dos logradouros públi-
seguintes normas: cos e, especialmente no perímetro urbano, determinar
I - evitar-se-ão, tanto quanto possível, formas assimé- o itinerário e os pontos de parada dos transportes co-
LEGISLAÇÃO ESPECÍFICA

tricas, estrangulamentos e alongamentos exagerados; letivos;


II - dar-se-á preferência, para a delimitação, às linhas XX - fixar os locais de estabelecimento de táxis e de-
naturais, facilmente identificáveis; mais veículos;
III - na inexistência de linhas naturais, utilizar-se-á linha XXI - conceder, permitir ou autorizar os serviços de
reta, cujos extremos, pontos naturais ou não, sejam fa- transporte coletivo e de táxis, fixando as respectivas
cilmente identificáveis e tenham condições de fixidez; tarifas;
IV - é vedada a interrupção de continuidade territorial XXII - fixar e sinalizar as zonas de silêncio e de trânsito
do Município ou Distrito de origem. e tráfego em condições especiais;

1
XXIII - disciplinar os serviços de carga e descarga e fixar Por fim, a lei estabelecerá as diretrizes e normas para
a tonelagem máxima permitida a veículos que circulem a exploração de serviços funerários deste Município, de
em vias públicas municipais; que trata o inciso XXVIII disposto acima, ficando vedada,
XXIV - tornar obrigatória a utilização de terminais rodo- a quaisquer títulos, as exclusividades.
viários;
XXV - sinalizar as vias urbanas e as estradas municipais, Da competência comum
bem como regulamentar e fiscalizar sua utilização; É da competência administrativa comum do Município,
XXVI - prover sobre a limpeza das vias e logradouros pú- da União e do Estado, observada a lei complementar fe-
blicos, remoção e destino de lixo domiciliar e de outros deral, o exercício das seguintes medidas:
resíduos de qualquer natureza; I - zelar pela guarda da Constituição, das leis e das insti-
XXVII - ordenar as atividades urbanas, fixando condi- tuições democráticas e conservar o patrimônio público;
ções e horários para funcionamento de estabelecimen- II - cuidar da saúde e assistência pública, da proteção e
tos industriais, comerciais e de serviços, observadas as garantia das pessoas portadoras de deficiências;
normas federais pertinentes; III - proteger os documentos, as obras e outros bens de
XXVIII - dispor sobre os serviços funerários e de cemi- valor histórico, artístico e cultural, os monumentos, as
térios; paisagens naturais notáveis e os sítios arqueológicos;
XXIX - regulamentar, licenciar, permitir, autorizar e fis- IV - impedir a evasão, a destruição e a descaracterização
calizar a afixação de cartazes e anúncios, bem como a de obras de arte e de outros bens de valor histórico, ar-
utilização de quaisquer outros meios de publicidade e tístico e cultural;
propaganda, nos locais sujeitos ao poder de polícia mu- V - proporcionar os meios de acesso à cultura, à educa-
nicipal; ção e à ciência;
XXX - prestar assistência nas emergências médico-hos- VI - a proteção do meio ambiente, a garantia de qualida-
pitalares de pronto socorro, por seu próprios serviços ou de de vida e o combate à poluição;
mediante convênio com instituição especializada; VII - preservar as florestas, a fauna e a flora;
XXXI - organizar e manter os serviços de fiscalização VIII - fomentar a produção agropecuária e organizar o
necessários ao exercício de seu poder de política admi- abastecimento alimentar;
nistrativa; IX - promover programas de construção de moradias e a
XXXII - fiscalizar, nos locais de venda, peso, medidas e melhoria das condições habitacionais e de saneamento
condições sanitárias dos gêneros alimentícios; básico;
XXXIII - dispor sobre o depósito de venda de animais e X - combater as causas de pobreza e os fatores de mar-
mercadorias apreendidas em decorrência de transgres- ginalização, promovendo a integração social dos setores
são da legislação Municipal; desfavorecidos;
XXXIV - dispor sobre registro, vacinação e captura de XI - registrar, acompanhar e fiscalizar as concessões de
animais, com a finalidade precípua de erradicar as mo- direitos de pesquisa e exploração de recursos hídricos e
léstias de que possam ser portadores ou transmissores; minerais em seus territórios;
XXXV - estabelecer e impor penalidade por infração de XII - estabelecer e implantar política de educação para a
suas leis e regulamentos; segurança do trânsito.
XXXVI - promover os seguintes serviços:
a) mercados, feiras e matadouros; Da competência suplementar
b) construção e conservação de estradas e caminhos Ao Município compete suplementar a legislação federal
municipais; e a estadual no que couber e naquilo que diz respeito ao
c) transportes coletivos estritamente municipais; seu peculiar interesse, visando a realidade local.
d) iluminação pública;
XXXVII - regulamentar o serviço de carros de aluguel, Das vedações
inclusive o uso de taxímetro; Ao Município é vedado:
XXXVIII - assegurar a expedição de certidões requeridas I - estabelecer cultos religiosos ou igrejas, subvencioná-
às repartições administrativas municipais, para defesa -los, embaraçar-lhes o funcionamento ou manter com
de direitos e esclarecimento de situações, estabelecen- eles ou seus representantes relações de dependência ou
do os prazos de atendimento; aliança, ressalvada, na forma da lei, a colaboração de in-
As normas de loteamento e arruamento a que se refere teresse público;
o inciso XIV disposto acima deverão exigir reserva de II - recusar fé aos documentos públicos;
áreas destinadas a: III - criar distinções entre brasileiros ou preferências entre si;
a) zonas verdes e demais logradouros públicos; IV - subvencionar ou auxiliar, de qualquer modo, com
b) vias de tráfego e de passagem de canalizações públi- recursos pertencentes aos cofres públicos, quer pela im-
cas, de esgotos e de águas pluviais nos fundos dos vales; prensa, rádio, televisão, serviço de alto falante ou qual-
LEGISLAÇÃO ESPECÍFICA

c) passagem de canalizações públicas de esgotos e de quer outro meio de comunicação, propaganda político-
águas pluviais com largura mínima de dois metros nos -partidária ou fins estranhos à administração;
fundos de lotes, cujo desnível seja superior a um metro V - manter a publicidade de atos, programas, obras, ser-
da frente ao fundo. viços e campanhas de órgãos públicos que não tenham
Não obstante, a lei complementar de criação da guar- caráter educativo, informativo ou de orientação social,
da municipal estabelecerá a organização e competência assim como a publicidade da qual constar nomes, sím-
dessa força auxiliar na proteção dos bens, serviços e ins- bolos ou imagens que caracterizem promoção pessoal
talações municipais. de autoridades ou servidores públicos;

2
VI - outorgar isenções e anistias, ou permitir a remissão de dívidas, sem interesse público justificado, sob pena de nu-
lidade do ato;
VII - exigir ou aumentar tributos sem lei que o estabeleça;
VIII - instituir tratamento desigual entre contribuintes que se encontrem em situação equivalente, proibida qualquer
distinção em razão de ocupação profissional ou função por eles exercida, independentemente da denominação jurídica
dos rendimentos, títulos ou direitos;
IX - estabelecer diferença tributária entre bens e serviços, de qualquer natureza, em razão de sua procedência ou des-
tinos;
X - cobrar tributos:
a) em relação a fatos geradores ocorridos antes do início da vigência da lei que os houver instituído ou aumentado;
b) no mesmo exercício financeiro em que haja sido publicada a lei que os instituiu ou aumentou;
XI - utilizar tributos com efeito de confisco;
XII - estabelecer limitações ao tráfego de pessoas ou bens, por meio de tributos, ressalvada a cobrança de pedágio pela
utilização de vias conservadas pelo Poder Público;
XIII - instituir impostos sobre:
a) patrimônio, renda ou serviços da União, do Estado e de outros Municípios;
b) templos de qualquer culto;
c) patrimônio, renda ou serviços dos partidos políticos, inclusive suas fundações, das entidades sindicais dos trabalha-
dores, das instituições de educação e de assistência social, sem fins lucrativos, atendidos os requisitos da Lei Federal;
d) livros, jornais, periódicos e o papel destinado a sua impressão.
XIV - contrair empréstimo externo sem prévia autorização do Senado Federal.

FIQUE ATENTO!
IMPORTANTE:
A vedação do inciso XIII, “a”, é extensiva às autarquias e às fundações instituídas e mantidas pelo Poder
Público, no que se refere ao patrimônio, à renda e aos serviços, vinculados às suas finalidades essenciais
ou à delas decorrentes;
As vedações do inciso XIII, “a”, e do parágrafo anterior não se aplicam ao patrimônio, à renda e aos serviços
relacionados com exploração de atividades econômicas regidas pelas normas aplicáveis e empreendimentos
privados, ou em que haja contraprestação ou pagamento de preços ou tarifas pelo usuário, nem exonera o
promitente comprador da obrigação de pagar imposto relativamente ao bem imóvel;
As vedações expressas no inciso XIII, alíneas “b” e “c”, compreendem somente o patrimônio, a renda e os
serviços relacionados com as finalidades essenciais das entidades nelas mencionadas;
As vedações expressas nos incisos VII e XIII, serão regulamentadas em lei complementar federal.

Da organização dos poderes


Do poder legislativo

O poder legislativo é exercido pela Câmara Municipal, com legislatura a cada quatro anos. É composta por 21 vereado-
res, exercendo papel de representantes do povo.
São condições de elegibilidade para o mandato de Vereador, na forma da Lei Federal:

I - a nacionalidade brasileira;
II - o pleno exercício dos direitos políticos;
III - o alistamento eleitoral;
IV - o domicílio eleitoral na circunscrição;
V - a filiação partidária;
VI - a idade mínima de dezoito anos;

#FicaDica
A alteração do número de Vereadores poderá ser efetuada pela Câmara Municipal, por meio de Decreto
LEGISLAÇÃO ESPECÍFICA

Legislativo, em havendo variação no número de habitantes do Município, conforme contagem populacional


oficialmente divulgada pelo Instituto Brasileiro de Geografia e Estatística - IBGE, e observados os limites
estabelecidos no artigo 29, inciso IV, da Constituição Federal. (Redação dada pela Emenda à Lei Orgânica
nº 8/2008)

3
De acordo com o artigo 15 da Lei Orgânica: realizar atos que gerem qualquer espécie de despesas
A Câmara Municipal reunir-se-á, na sede do Municí- para a Câmara Municipal nesse período. (Nova redação
pio, de 02 de fevereiro a 17 de julho e de 1º de agosto acrescida pela Emenda à Lei Orgânica nº 6/2006)
a 22 de dezembro. (Redação dada pela Emenda à Lei Art. 22 O mandato da Mesa Diretiva será de dois anos,
Orgânica nº 5/2006) permitida a reeleição de seus membros, para o mesmo
§ 1º As reuniões marcadas para essas datas serão cargo, na eleição imediatamente subseqüente. (Reda-
transferidas para o primeiro dia útil subseqüente, ção dada pela Emenda à Lei Orgânica nº 4/1998)
quando recaírem sem sábados, domingos e feriados. Art. 23 A Mesa da Câmara se compõe do Presidente,
§ 2º A Câmara se reunirá em sessões ordinárias, extra- Vice-Presidente, do Primeiro Secretário e Segundo Se-
ordinárias ou solenes, conforme dispuser o seu Regi- cretário, os quais se substituirão nessa ordem.
mento Interno. § 1º Na constituição da Mesa é assegurada, tanto quan-
§ 3º A convocação extraordinária da Câmara Municipal to possível, a representação proporcional dos partidos
far-se-á: ou dos blocos parlamentares que participem da Casa.
I - pelo Prefeito, pelo Presidente da Câmara ou a re- § 2º Na ausência dos Membros da Mesa o Vereador
querimento da maioria dos membros da Casa, em mais votado assumirá a Presidência.
caso de urgência ou interesse público relevante; § 3º Qualquer componente da Mesa poderá ser des-
II - pelo Presidente da Câmara para o compromisso e tituído da mesma, pelo voto de dois terços (2/3) dos
a posse do Prefeito e do Vice-Prefeito. membros da Câmara, quando faltoso, omisso e inefi-
§ 4º Na sessão legislativa extraordinária, a Câmara Mu- ciente no desempenho de suas atribuições regimen-
nicipal somente deliberará sobre a matéria para a qual tais, elegendo-se outro Vereador para a complemen-
foi convocada. tação do mandato.
Por fim, consta ressaltar que as deliberações da Câ- Art. 24 A Câmara terá comissões permanentes e espe-
mara serão tomadas por maioria dos votos, presente ciais.
a maioria dos seus membros, salvo disposições em § 1º Às comissões permanentes em razão da matéria
contrário constantes na Constituição Federal e nesta de sua competência, cabe:
Lei Orgânica. Além disso, as sessões serão públicas, I - discutir e votar projeto de lei que dispensar na forma
salvo deliberação em contrário, de dois terços (2/3) do Regimento Interno, a competência do Plenário, sal-
dos Vereadores, adotada em razão de motivo relevan- vo se houver recurso de um quinto (1/5) dos membros
te; e as sessões somente poderão ser abertas com a da Casa;
presença de no mínimo, um terço (1/3) dos membros II - realizar audiências públicas com entidades da so-
da Câmara. ciedade civil;
Para as demais questões acerca do poder legislativo III - convocar os Secretários Municipais ou Diretores
do município, vejamos o que diz a Lei Orgânica: equivalentes, para prestar informações sobre assuntos
inerentes a suas atribuições;
DO FUNCIONAMENTO DA CÂMARA IV - receber petições, reclamações, representações ou
queixas de qualquer pessoa contra atos ou omissão
Art. 21 A Câmara Municipal reunir-se-á em sessão pre- das autoridades ou entidades públicas;
paratória, a partir de 1º de janeiro do primeiro ano da V - solicitar depoimento de qualquer autoridade ou
legislatura, para posse de seus membros. cidadão;
§ 1º Sob a Presidência do Vereador mais votado nas VI - exercer, no âmbito de sua competência, a fiscaliza-
últimas eleições municipais, dentre os presentes, os ção dos atos do Executivo e da Administração Indireta;
Vereadores prestarão os seguintes compromissos: § 2º As comissões especiais, criadas por deliberação do
“Prometo cumprir a Constituição Federal, a Constitui- Plenário, serão destinadas ao estudo de assuntos es-
ção Estadual e a Lei Orgânica Municipal, observar as pecíficos e à representação da Câmara em congressos,
leis, desempenhar o mandato que me foi confiado e solenidades ou outros atos públicos.
trabalhar pelo progresso do Município e bem estar de § 3º Na formação das Comissões, assegurar-se-á, tan-
seu povo.” to quanto possível, a representação proporcional dos
§ 2º Prestado o compromisso pelo Presidente, o Secre- Partidos ou dos blocos parlamentares que participem
tário que for designado para esse fim, fará a chamada da Câmara.
nominal de cada Vereador, que declarará: § 4º As comissões parlamentares de inquérito, que te-
“Assim o prometo.” rão poderes de investigação próprios das autoridades
§ 3º O Vereador que não tomar posse na sessão previs- judiciais, além de outros previstos no Regimento Inter-
ta neste artigo deverá fazê-lo no prazo de 15 (quinze) no da Casa, serão criadas pela Câmara Municipal, me-
dias, salvo motivo justo aceito pela Câmara Municipal. diante requerimento de um terço dos seus membros,
LEGISLAÇÃO ESPECÍFICA

§ 4º A eleição da Mesa da Câmara, para o segundo para a apuração de fato determinado e por prazo cer-
biênio, far-se-á na última sessão ordinária do primeiro to, sendo suas conclusões, se for o caso, encaminhadas
biênio de cada legislatura, considerando-se automa- ao Ministério Público, para que promova a responsabi-
ticamente empossados os eleitos. (Redação acrescida lidade civil ou criminal dos infratores.
pela Emenda à Lei Orgânica nº 6/2006) Art. 25 A indicação dos Líderes será feita em documen-
§ 5º A responsabilidade administrativa da gestão fiscal to subscrito pelos membros das representações parti-
até o dia 31 de dezembro do ano respectivo é de com- dárias, à Mesa, nas vinte e quatro horas que se segui-
petência da Mesa anterior, não podendo a Mesa eleita rem à instalação do primeiro período legislativo anual.

4
Parágrafo Único - Os Líderes indicarão os respectivos III - interpretar e fazer cumprir o Regimento Interno;
Vice-Líderes, dando conhecimento à Mesa da Câmara IV - promulgar as resoluções e decretos legislativos;
desse designação. V - promulgar as leis com sanção tácita ou cujo veto
Art. 26 Além de outras atribuições previstas no Regi- tenha sido rejeitado pelo Plenário, desde que não
mento Interno, os Líderes indicarão os representantes aceita esta decisão, em tempo hábil, pelo Prefeito;
partidários nas Comissões da Câmara. VI - fazer publicar os atos da Mesa, as resoluções, de-
Parágrafo Único - Ausente ou impedido o Líder, suas cretos legislativos e as leis que vier a promulgar;
atribuições serão exercidas pelo Vice-Líder. VII - autorizar as despesas da Câmara;
Art. 27 Câmara Municipal, observado o disposto nesta VIII - representar por decisão da Câmara, sobre a in-
Lei Orgânica, compete elaborar seu Regimento Interno, constitucionalidade da lei ou ato municipal;
dispondo sobre sua organização, polícia e provimento IX - solicitar, por decisão da maioria absoluta da Câ-
de cargos dos seus serviços e, especialmente, sobre: mara, a intervenção no Município nos casos admitidos
I - sua instalação e funcionamento; pela Constituição Federal e pela Constituição Estadual;
II - posse de seus membros; X - manter a ordem no recinto da Câmara, podendo
III - eleição da Mesa, sua composição e suas atribuições; solicitar a força necessária para esse fim;
IV - número de reuniões mensais; XI - encaminhar, para parecer prévio, a prestação de
V - comissões; contas do Município ao Tribunal de Contas do Estado.
VI - sessões;
VII - deliberações; DAS ATRIBUIÇÕES DA CÂMARA MUNICIPAL
VIII - todo e qualquer assunto de sua administração in-
terna. Art. 33 Compete à Câmara, com a sanção do Prefei-
Art. 28 Por deliberação da maioria dos seus membros, to, dispor sobre todas as matérias de competência do
a Câmara poderá convocar Secretário Municipal ou Di- Município e, especialmente:
retor equivalente para, pessoalmente, prestar informa- I - instituir e arrecadar os tributos de sua competência,
ções acerca de assuntos previamente estabelecidos. bem como aplicar suas rendas;
Parágrafo Único - A falta de comparecimento do Secre- II - autorizar isenções e anistias fiscais e a remissão de
tário ou Diretor equivalente, sem justificativa razoável, dívidas;
será considerada desacato à Câmara e, se o Secretário ou III - votar o orçamento anual e plurianual de investi-
Diretor equivalente for Vereador licenciado, o não com- mentos, bem como autorizar a abertura de créditos
parecimento nas condições mencionadas caracterizará suplementares e especiais;
procedimento incompatível com a dignidade da Câmara, IV - deliberar sobre a obtenção e concessão de em-
para instauração do respectivo processo, na forma da lei préstimos e operações de crédito, bem como a forma
federal, e conseqüente cassação de mandato. e meios de pagamento;
Art. 29 O Secretário Municipal ou Diretor equivalente, a V - autorizar a concessão de auxílios e subvenções;
seu pedido, poderá comparecer perante o Plenário ou VI - autorizar a concessão de serviços públicos;
qualquer Comissão da Câmara, para expor assunto e VII - autorizar a concessão de direito real de uso de
discutir projeto de lei ou qualquer outro ato normativo bens municipais;
relacionado com o seu serviço administrativo. VIII - autorizar a concessão administrativa de uso de
Art. 30 A Mesa da Câmara poderá encaminhar pedidos bens municipais;
escritos de informações aos Secretários Municipais ou IX - autorizar a alienação de bens imóveis;
Diretores equivalentes, importando crimes de respon- X - autorizar a aquisição de bens imóveis, salvo quan-
sabilidade e recusa ou o não atendimento no prazo de do se tratar de doação sem encargo;
quinze dias, bem como a prestação de informação falsa. XI - criar, transformar e extinguir cargos, empregos e
Art. 31 Mesa, dentre outras atribuições, compete: funções públicas e fixar os respectivos vencimentos,
I - tomar todas as medidas necessárias à regularidade inclusive os dos servidores da Câmara;
dos trabalhos legislativos; XII - criar, estruturar e conferir atribuições a Secretários
II - propor projetos que criem ou extingam cargos nos ou Diretores equivalentes e órgãos da administração
serviços da Câmara e fixem os respectivos vencimentos; pública;
III - apresentar projetos de lei dispondo sobre abertu- XIII - aprovar o Plano Diretor de Desenvolvimento In-
ra de créditos suplementares ou especiais, através do tegrado;
aproveitamento total ou parcial das consignações orça- XIV - autorizar convênios com entidades públicas ou
mentárias da Câmara; particulares e consórcios com outros Municípios;
IV - promulgar a Lei Orgânica e suas emendas; XV - delimitar o perímetro urbano;
V - representar, junto ao Executivo, sobre necessidades XVI - autorizar a alteração da denominação de pró-
de economia interna; prios, vias e logradouros públicos;
LEGISLAÇÃO ESPECÍFICA

VI - contratar, na forma da lei, por tempo determinado, XVII - estabelecer normas urbanísticas, particularmen-
para atender necessidade temporária de excepcional te as relativas a zoneamento e loteamento;
interesse público; Art. 34 Compete privativamente à Câmara Municipal
Art. 32 Dentre outras atribuições, compete ao Presi- exercer as seguintes atribuições, dentre outras:
dente da Câmara: I - eleger sua Mesa Diretiva;
I - representar a Câmara em juízo e fora dele; II - elaborar o Regimento Interno;
II - dirigir, executar e disciplinar os trabalhos legislati- III - organizar os serviços administrativos internos e
vos e administrativos da Câmara; prover os cargos respectivos;

5
IV - propor a criação ou a extinção de cargos dos ser- Art. 36 vedado ao Vereador:
viços administrativos internos e a fixação dos respec- I - desde a expedição do diploma:
tivos vencimentos; a) firmar ou manter acordo com o Município, com suas
V - conceder licença ao Prefeito, ao Vice-Prefeito e aos autarquias, fundações, empresas públicas, sociedades
Vereadores; de economia mista ou com suas empresas conces-
VI - autorizar o Prefeito a ausentar-se do Município por sionárias de serviço público, salvo quando o contrato
mais de 15 (quinze) dias, por necessidade de serviço; obedecer as cláusulas uniformes;
VII - tomar e julgar as contas do Prefeito, deliberando b) aceitar cargos, emprego ou função, no âmbito da
sobre o parecer do Tribunal de Contas do Estado; administração pública direta e indireta municipal, sal-
VIII - decretar a perda do mandato do Prefeito e dos vo mediante aprovação em concurso público e obser-
Vereadores, nos casos indicados na Constituição Fe- vado o disposto no Art. 82, I, IV e V desta Lei Orgânica.
deral, nesta Lei Orgânica e na Legislação Federal apli- II - desde a posse:
cável; a) ocupar cargo, função ou emprego na administração
IX - autorizar a realização de empréstimo, operação pública direta ou indireta do Município, de que seja
ou acordo externo de qualquer natureza, de interesse exonerável “ad nutum”, salvo o cargo de Secretário
do Município; Municipal ou Diretor equivalente, desde que se licen-
X - proceder à tomada de contas do Prefeito, através cie do exercício do mandato;
de Comissão especial, quando não apresentadas à Câ- b) exercer outro cargo eletivo federal, estadual ou mu-
mara, dentro de 60 (sessenta) dias, após a abertura da nicipal;
sessão legislativa; c) ser proprietário, controlador ou diretor de empresa
XI - aprovar convênio, acordo ou qualquer outro ins- que goze de favor decorrente de contrato com pessoa
trumento celebrado pelo Município com a União, o Es- jurídica de direito público do Município, ou nela exer-
tado, outra pessoa jurídica de direito público interno cer função remunerada;
ou entidades assistenciais culturais; d) patrocinar causa junto ao Município em que seja
XII - estabelecer e mudar temporariamente o local de interessada qualquer das entidades a que se refere a
suas reuniões;
alínea “a” do inciso I;
XIII - convocar o Prefeito, Secretários do Município ou
Art. 37 Perderá o mandato o Vereador:
Diretores equivalentes para prestar esclarecimentos,
I - que infringir qualquer das proibições estabelecidas
aprazando dia e hora para o comparecimento;
no artigo anterior;
XIV - deliberar sobre o adiamento e a suspensão de
II - cujo procedimento for declarado incompatível com
suas reuniões;
o decoro parlamentar ou atentatório às instituições vi-
XV - criar comissão parlamentar de inquérito sobre
gentes;
fato determinado e prazo certo, mediante requeri-
III - que utilizar do mandato para a prática de atos de
mento de um terço (1/3) de seus membros;
XVI - conceder título de cidadão honorário ou con- corrupção ou de improbidade administrativa;
ferir homenagem a pessoas que reconhecidamente IV - que deixar de comparecer, em cada sessão legis-
tenham prestado relevantes serviços ao Município ou lativa anual, à terça parte das sessões ordinárias da
nele se destacado pela atuação exemplar na vida pú- Câmara, salvo doença comprovada, licença ou missão
blica e particular mediante proposta pelo voto de dois autorizada pela edilidade;
terços (2/3) dos membros da Câmara; V - que fixar residência fora do Município;
XVII - solicitar a intervenção do Estado no Município; VI - que perder ou tiver suspensos os direitos políticos.
XVIII - julgar o Prefeito, o Vice-Prefeito e os Vereado- § 1º Além de outros casos definidos no Regimento
res, nos casos previstos em Lei Federal; Interno da Câmara Municipal, considerar-se-á incom-
XIX - fiscalizar e controlar os atos do Poder Executivo, patível com o decoro parlamentar o abuso das prer-
incluídos os da administração indireta; rogativas asseguradas ao Vereador ou a percepção de
XX - fixar, observado o que dispõem os Arts. 37, XI, vantagens ilícitas ou imorais;
150, II, 153, III e 153, § 2º, I, da Constituição Federal, § 2º Nos casos dos incisos I e II a perda do mandato
o subsídio dos vereadores em cada legislatura para será declarada pela Câmara por voto secreto e maio-
a subseqüente, sobre a qual incidirá o imposto sobre ria absoluta, mediante representação da Mesa ou de
renda e proventos de qualquer natureza; (Redação Partido Político representado na Câmara, assegurada
dada pela Emenda à Lei Orgânica nº 11/2012) ampla defesa.
XXI - fixar, observado o que dispõem os Arts. 37, XI, § 3º Nos casos previstos nos incisos III e VI, a perda
150, II, 153, III e 153, § 2º, I, da Constituição Federal, será declarada pela Mesa da Câmara, de ofício ou me-
em cada legislatura para a subseqüente, o subsídio do diante representação de qualquer de seus membros
LEGISLAÇÃO ESPECÍFICA

Prefeito e do Vice-Prefeito. (Redação dada pela Emen- ou de Partido Político representado na Casa, assegu-
da à Lei Orgânica nº 11/2012) rada ampla defesa.
Art. 38 O Vereador poderá licenciar-se:
DOS VEREADORES I - por motivo de doença;
II - para tratar, sem remuneração, de interesse particu-
Art. 35 Os Vereadores são invioláveis no exercício do lar, desde que o afastamento não ultrapasse 120 (cen-
mandato, e na circunscrição do Município, por suas to e vinte) dias por sessão legislativa. (Redação dada
opiniões, palavras e votos. pela Emenda à Lei Orgânica nº 16/2017)

6
III - para desempenhar missões temporárias, de cará- § 1º A proposta será votada em dois turnos com in-
ter cultural ou de interesse do Município; terstício mínimo de dez dias, e aprovada por dois ter-
§ 1º Não perderá o mandato, considerando-se au- ços dos membros da Câmara Municipal.
tomaticamente licenciado, o Vereador investido no § 2º A emenda à Lei Orgânica Municipal será promul-
Cargo de Secretário Municipal ou Diretor equivalente, gada pela Mesa da Câmara com o respectivo número
conforme previsto no Art. 36, inciso II, alínea “a” desta de ordem.
Lei Orgânica. § 3º A Lei Orgânica não poderá ser emendada na vi-
§ 2º Revogado. (Revogado pela Emenda à Lei Orgânica gência de estado de sítio ou de intervenção no Mu-
nº 11/2012) nicípio.
§ 3º Revogado. (Revogado pela Emenda à Lei Orgânica Art. 44 A iniciativa das leis cabe a qualquer Vereador,
nº 11/2012) ao Prefeito e ao eleitorado que a exercerá sob a forma
§ 4º A licença para tratar de interesse particular não de moção articulada, subscrita, no mínimo, por cinco
será inferior a 30 (trinta) dias, e o Vereador não poderá por cento do total do número de eleitores do Muni-
reassumir o exercício do mandato antes do término cípio.
da licença. Art. 45 As leis complementares somente serão apro-
§ 5º Independentemente de requerimento, considerar- vadas se obtiverem maioria absoluta dos votos dos
-se-á como licença o não comparecimento às reuniões membros da Câmara Municipal, observados os demais
de Vereador, privado temporariamente de sua liberda- termos de votação das leis ordinárias.
de, em virtude de processo criminal em curso. Parágrafo Único - Serão leis complementares, dentre
§ 6º Na hipótese do § 1º o Vereador poderá optar pela outras previstas nesta Lei Orgânica:
remuneração do mandato. I - Código Tributário do Município;
Art. 39 Dar-se-á a convocação do Suplente de Verea- II - Código de Obras;
dor nos casos de vaga ou de licença. III - Plano Diretor de Desenvolvimento Integrado;
§ 1º O Suplente convocado deverá tomar posse no IV - Código de Postura;
prazo de 15(quinze) dias, contados da data da convo- V - Lei instituidora de regime jurídico único dos servi-
cação, salvo justo motivo aceito pela Câmara, quando dores municipais;
se prorrogará o prazo. VI - Lei Orgânica instituidora da Guarda Municipal;
§ 2º Enquanto a vaga a que se refere o parágrafo an- VII - Lei de criação de cargos, funções ou empregos
terior não for preenchida, calcular-se-á o quorum em públicos;
função dos Vereadores remanescentes. Art. 46 São de iniciativa exclusiva do Prefeito as leis
Art. 40 O subsídio máximo dos Vereadores será cor- que disponham sobre:
respondente ao fixado pela Constituição Federal, no I - criação, transformação ou extinção de cargos, fun-
seu art.29, VI, e a respectiva alínea referente à popula- ções ou empregos públicos na Administração Direta e
ção oficial do Município. autárquica ou aumento de sua remuneração;
§ 1º Os Vereadores e o Presidente da Câmara perce- II - servidores públicos, seu regime jurídico, provimen-
berão o décimo terceiro subsídio, correspondente ao to de cargos, estabilidade e aposentadoria;
subsídio do mês de dezembro. III - criação, estruturação e atribuições das secretarias
§ 2º A Resolução que fixar o subsídio deverá prever o ou departamentos equivalentes e órgãos da Adminis-
valor a ser descontado nos casos de faltas dos Verea- tração Pública;
dores às sessões ordinárias da Câmara Municipal. (Re- IV - matéria orçamentária, e a que autoriza a abertu-
dação dada pela Emenda à Lei Orgânica nº 11/2012) ra de créditos suplementares ou especiais. (Redação
Art. 41 O subsídio do Presidente da Câmara e do Pri- dada pela Emenda à Lei Orgânica nº 10/2009)
meiro Secretário poderá ser superior, no máximo, a Parágrafo Único - Não será admitido aumento da des-
50% (cinquenta por cento) e 20% (vinte por cento), pesa nos projetos de iniciativa exclusiva do Prefeito
respectivamente, do fixado para o Vereador. (Redação Municipal, ressalvado o disposto no Artigo 166, §§ 3º
dada pela Emenda à Lei Orgânica nº 11/2012) e 4º, da Constituição Federal.
Art. 47 É da competência exclusiva da Mesa da Câmara
DO PROCESSO LEGISLATIVO a iniciativa das Leis que disponham sobre:
I - autorização para abertura de créditos suplementa-
Art. 42 O processo legislativo municipal compreende res ou especiais, através do aproveitamento total ou
a elaboração de: parcial das consignações Orçamentárias da Câmara;
I - emendas à Lei Orgânica Municipal; II - organização dos serviços administrativos da Câma-
II - leis complementares; ra, criação, transformação ou extinção de seus cargos,
III - leis ordinárias; empregos ou funções e fixação da respectiva remu-
LEGISLAÇÃO ESPECÍFICA

IV - leis delegadas; neração.


V - resoluções, e Parágrafo Único - Nos projetos de competência exclu-
VI - decretos legislativos. siva da Mesa da Câmara, não serão admitidas emen-
Art. 43 A Lei Orgânica Municipal poderá ser emendada das que aumentem a despesa prevista, ressalvada a
mediante proposta: fixação da remuneração dos servidores da Câmara, se
I - de um terço, no mínimo, dos membros da Câmara proposta pela maioria dos Vereadores.
Municipal; Art. 48 O Prefeito poderá solicitar urgência para apre-
II - do Prefeito Municipal. ciação de projetos de sua iniciativa.

7
§ 1º Somente será considerado motivo de urgência a Art. 52 A matéria constante de projeto de lei rejeitada
discussão da matéria cujo adiamento torne inútil a de- somente poderá constituir objeto de novo projeto, na
liberação ou importe em grave prejuízo à coletividade. mesma sessão legislativa, mediante proposta da maio-
§ 2º Solicitada a urgência, a Câmara deverá se mani- ria absoluta dos membros da Câmara.
festar em até 45 (quarenta e cinco) dias sobre a propo-
sição, contando da data em que for feita a solicitação. DA FISCALIZAÇÃO CONTÁBIL, FINANCEIRA E
§ 3º Esgotado o prazo previsto no parágrafo anterior ORÇAMENTÁRIA
sem deliberação pela Câmara, será a proposição incluí-
da na Ordem do Dia, sobrestando-se às demais propo- Art. 53 A fiscalização contábil, financeira e orçamentá-
sições, para que se ultime a votação. ria do Município será exercida pela Câmara Municipal,
§ 4º O prazo do § 2º não corre no período de recesso mediante controle externo e pelos sistemas de contro-
da Câmara nem se aplica aos projetos de lei comple- le interno do Executivo, instituídos em lei.
mentar. § 1º O controle externo da Câmara será exercido com
Art. 49 Aprovado o projeto de lei será este enviado ao o auxílio do Tribunal de Contas do Estado e compreen-
Prefeito, que, aquiescendo, o sancionará. derá a apreciação das Contas do Prefeito e da Mesa da
§ 1º O Prefeito considerando o projeto, no todo ou em Câmara, o acompanhamento das atividades financei-
parte, inconstitucional ou contrário ao interesse público ras e orçamentárias do Município, o desempenho das
vetá-lo-á total ou parcialmente, no prazo de 15 (quinze) funções de auditoria financeira e orçamentária, bem
dias úteis, contados da data do recebimento. como o julgamento das contas dos administradores e
§ 2º O veto parcial somente abrangerá texto integral de demais responsáveis por bens e valores públicos.
artigo, de parágrafo, de inciso ou de alínea. § 2º As contas do Prefeito e da Câmara Municipal,
§ 3º Decorrido o prazo do parágrafo 1º, o silêncio do prestadas anualmente, serão julgadas pela Câmara
Prefeito importará em sanção tácita. dentro de 60 (sessenta) dias após o recebimento do
§ 4º Usando o Prefeito o direito do veto, no prazo legal, parecer prévio do Tribunal de Contas, considerando-se
será ele apreciado dentro de 30 (trinta) dias, a contar julgadas nos termos das conclusões desse parecer, se
do seu recebimento, em uma só discussão e votação, não houver deliberação dentro desse prazo.
considerando-se mantido o veto que não obtiver o § 3º Somente por decisão de dois terços dos membros
da Câmara Municipal deixará de prevalecer o parecer
voto contrário da maioria absoluta. Se o veto não for
emitido pelo Tribunal de Contas do Estado.
apreciado nesse prazo, considerar-se-á mantido pela
I - Rejeitadas as contas, serão estas imediatamente re-
Câmara.(Redação dada pela Emenda à Lei Orgânica nº
metidas ao Ministério Público para os fins de direito.
6/2006)
§ 4º As contas relativas à aplicação dos recursos trans-
§ 5º Rejeitado o veto, será o projeto enviado ao Prefeito
feridos pela União e Estado serão prestadas na forma
para a promulgação.
da legislação federal e estadual em vigor, podendo o
§ 6º Esgotado sem deliberação o prazo estabelecido no
Município, suplementar essas contas, sem prejuízo de
§ 4º, o veto será colocado na Ordem do Dia da sessão
sua inclusão na prestação anual de contas.
imediata, sobrestadas às demais proposições, até a sua Art. 54 O Executivo manterá sistema de controle inter-
votação final, ressalvadas as matérias de que trata o Art. no, a fim de:
48 desta Lei Orgânica. I - criar condições indispensáveis para assegurar eficá-
§ 7º A não promulgação da lei no prazo de quarenta e cia ao controle externo e regularidade à realização da
oito horas pelo Prefeito, nos casos dos §§ 3º e 5º, criará receita e despesa;
para o Presidente da Câmara a obrigação de fazê-lo em II - acompanhar as execuções de programas de traba-
igual prazo. lho e do orçamento;
Art. 50 As leis delegadas serão elaboradas pelo Prefeito, III - avaliar os resultados alcançados pelos administra-
que deverá solicitar a delegação à Câmara Municipal. dores;
§ 1º Os atos de competência privativa da Câmara, a ma- IV - verificar a execução dos contratos.
téria reservada à lei complementar e os planos pluria- Art. 55 As contas do Município ficarão durante ses-
nuais e orçamentos não serão objetos de delegação. senta dias, anualmente, à disposição de qualquer
§ 2º A delegação ao Prefeito será efetuada sob a forma contribuinte, para exame e apreciação, o qual poderá
de decreto legislativo, que especificará o seu conteúdo questionar-lhes a legitimidade, nos termos da lei.
e os termos de seu exercício.
§ 3º O decreto legislativo poderá determinar a aprecia- DO PODER EXECUTIVO
ção do projeto pela Câmara que fará em votação única,
vedada a apresentação de emenda. Tal como acerca do poder legislativo, a leitura da lei
LEGISLAÇÃO ESPECÍFICA

Art. 51 Os projetos de resoluções disporão sobre ma- seca se faz fundamental para entendimento dos as-
térias de interesse interno da Câmara e os projetos de suntos acerca do poder executivo do município.
decreto legislativo sobre os demais casos de sua com-
petência privativa. DO PREFEITO E DO VICE-PREFEITO
Parágrafo Único - Nos casos de projeto de resolução e
de projeto de decreto legislativo, considerar-se-á en- O Poder Executivo Municipal é exercido pelo Prefei-
cerrada com a votação final a elaboração da norma jurí- to, auxiliado pelos Secretários Municipais ou Diretores
dica, que será promulgada pelo Presidente da Câmara. equivalentes.

8
Parágrafo Único - Aplica-se, para o Prefeito e Vice-Pre- § 1º O subsídio do Vice-Prefeito corresponderá a, no
feito, a elegibilidade ao mandato de Vereador, dispos- máximo, 50% (cinqüenta por cento) do fixado para o
ta nesta Lei Orgânica e, idade mínima de 21 (vinte e Prefeito Municipal.
um) anos. § 2º O subsídio do Prefeito e Vice-Prefeito, serão atu-
A eleição do Prefeito e do Vice-Prefeito realizar-se-á alizados anualmente, sempre à mesma época e índi-
simultaneamente, nos termos estabelecidos no Art. ces da atualização aplicada aos vencimentos aplicados
29, incisos I e II da Constituição Federal. aos Servidores Públicos do Município.
Parágrafo Único - A eleição do Prefeito importará a do § 3º O Prefeito e Vice-Prefeito perceberão o décimo
Vice-Prefeito com ele registrado. terceiro subsídio, correspondente ao subsídio do mês
O Prefeito e Vice-Prefeito tomarão posse no dia 1º de de dezembro. (Redação dada pela Emenda à Lei Orgâ-
janeiro do ano subseqüente à eleição, em sessão da nica nº 11/2012)
Câmara Municipal, prestando o compromisso de man- § 4º Revogado. (Revogado pela Emenda à Lei Orgânica
ter, defender e cumprir a Lei Orgânica, observar as leis nº 11/2012)
da União, do Estado e do Município, promover o bem
geral dos munícipes e exercer o cargo sob a inspiração DAS ATRIBUIÇÕES DO PREFEITO
da democracia, da legitimidade e da legalidade.
Parágrafo Único - Decorridos dez dias da data fixa- Ao Prefeito, como chefe da administração, compete
da para a posse, o Prefeito ou Vice-Prefeito, salvo por dar cumprimento às deliberações da Câmara, dirigir,
motivo de força maior, não tiver assumido o cargo, fiscalizar e defender os interesses do Município, bem
este será declarado vago. como adotar de acordo com a lei, todas as medidas
Substituirá o Prefeito, no caso de impedimento e suce- administrativas de utilidade pública, sem exceder as
der-lhe-á, no de vacância, o Vice-Prefeito. verbas orçamentárias.
§ 1º O Vice-Prefeito não poderá se recusar a substituir Compete ao Prefeito, entre outras, as seguintes atri-
o Prefeito, sob pena da extinção do mandato. buições:
§ 2º O Vice-Prefeito, além de outras atribuições que
I - a iniciativa das leis, na forma e casos previstos nesta
lhe forem conferidas por lei, auxiliará o Prefeito, sem-
Lei Orgânica;
pre que por ele for convocado para missões especiais,
II - representar o Município em juízo e fora dele;
Em caso de impedimento do Prefeito e do Vice-Pre-
III - sancionar, promulgar e fazer publicar as leis apro-
feito, ou vacância do cargo, assumirá a administração
vadas pela Câmara e expedir os regulamentos para
municipal o Presidente da Câmara.
sua fiel execução;
Parágrafo Único - O Presidente da Câmara recusando-
IV - vetar, no todo ou em parte, os projetos de lei apro-
-se, por qualquer motivo, a assumir o cargo de Prefei-
to, renunciará, incontinente, à função de dirigente do vados pela Câmara;
Legislativo, ensejando assim a eleição de outro mem- V - decretar, nos termos da lei, a desapropriação por
bro para ocupar como Presidente da Câmara, a chefia necessidade ou utilidade pública, ou ainda por inte-
do Poder Executivo. resse social;
Art. 61 Verificando-se a vacância do cargo de Prefeito VI - expedir decretos, portarias e outros atos adminis-
e inexistindo Vice-Prefeito, observar-se-á o seguinte: trativos;
I - ocorrendo a vacância nos três primeiros anos de VII - permitir ou autorizar o uso de bens municipais,
mandato, dar-se-á eleição noventa dias após a sua por terceiros;
abertura, cabendo aos eleitos completar o período VIII - permitir ou autorizar a execução de serviços pú-
dos seus antecessores; blicos, por terceiros;
II - ocorrendo a vacância no último ano do manda- IX - prover os cargos públicos e expedir os demais atos
to, assumirá o Presidente da Câmara para completar referentes à situação funcional dos servidores;
o período. X - enviar à Câmara os projetos de lei relativos ao or-
O mandato do Prefeito é de quatro anos, vedada a çamento anual e ao plurianual do Município.
reeleição para o período subseqüente. XI - encaminhar à Câmara, até 15 de abril, a prestação
O Prefeito e o Vice-Prefeito, quando no exercício do de contas, bem como os balanços do exercício findo;
cargo, não poderão, sem licença da Câmara Munici- XII - encaminhar aos órgãos competentes os planos
pal, ausentar-se do Município por período superior a de aplicação e as prestações de contas exigidas em lei;
15 (quinze) dias, sob pena de perda do cargo ou de XIII - fazer publicar os atos oficiais, no prazo de 30
mandato. (trinta) dias;
Parágrafo Único - O Prefeito regularmente licenciado XIV - prestar à Câmara, dentro de 15 (quinze) dias, as
terá direito a perceber o subsídio, quando: (Redação informações solicitadas pela mesma, salvo prorroga-
dada pela Emenda à Lei Orgânica nº 11/2012) ção, a seu pedido, por prazo determinado e com a
LEGISLAÇÃO ESPECÍFICA

I - impossibilidade de exercer o cargo, por motivo de concordância da Câmara em face de complexidade da


doença devidamente comprovada; matéria ou dificuldade de obtenção, nas respectivas
II - a serviço ou em missão de representação do Mu- fontes, dos dados pleiteados;
nicípio. XV - prover os serviços e obras da administração pública;
O subsídio do Prefeito Municipal e Vice-Prefeito, será XVI - superintender a arrecadação dos tributos bem
fixado por Lei de iniciativa da Câmara Municipal, em como a guarda e aplicação da receita, autorizando as
parcela única, obedecido, em qualquer caso, o dispos- despesas e pagamentos dentro das disponibilidades
to no art. 37, X e XI, da Constituição Federal. orçamentárias ou dos créditos votados pela Câmara;

9
XVII - colocar à disposição da Câmara, dentro de 10 As incompatibilidades declaradas e observadas nes-
(dez) dias de sua requisição, as quantias que devam ta Lei Orgânica, estende-se no que forem aplicáveis,
ser despendidas de uma só vez; ao Prefeito e aos Secretários Municipais ou Diretores
XVIII - aplicar multas previstas em leis e contratos; equivalentes.
XIX - resolver sobre os requerimentos, reclamações ou São crimes de responsabilidade do Prefeito os previs-
representações que lhe forem dirigidas; tos em lei federal.
XX - oficializar, obedecidas as normas urbanísticas Parágrafo Único - O Prefeito será julgado pela prática
aplicáveis, as vias e logradouros públicos, mediante de crime de responsabilidade, perante o Tribunal de
denominação aprovada pela Câmara; Justiça do Estado.
XXI - convocar extraordinariamente a Câmara em caso São infrações político-administrativas do Prefeito as
de urgência ou interesse público relevante; previstas em lei federal.
XXII - aprovar os projetos de edificação, os planos de Parágrafo Único - O Prefeito será julgado pela prática
loteamento, arruamento e zoneamento urbano ou de infrações político-administrativas, perante a Câma-
para fins urbanos; ra Municipal.
XXIII - apresentar, anualmente, à Câmara, relatório cir- Será declarado vago, pela Câmara Municipal, o cargo
cunstanciado sobre o estado das obras e dos serviços do Prefeito, quando:
municipais, bem assim o programa da administração I - ocorrer falecimento, renúncia ou condenação por
para o ano seguinte; crime funcional ou eleitoral;
XXIV - organizar os serviços internos das repartições II - deixar de tomar posse, sem motivo justo, aceito
criadas por lei, sem exceder as verbas para tal desti- pela Câmara, dentro do prazo de 10 (dez) dias;
nadas; III - infringir as normas dos artigos 36 e 63 desta Lei
XXV - contrair empréstimos e realizar operações de Orgânica;
crédito, mediante prévia autorização da Câmara; IV - perder ou tiver suspensos os direitos políticos.
XXVI - providenciar sobre a administração dos bens do
município e sua alienação, na forma da lei; DOS AUXILIARES DIRETOS DO PREFEITO
XXVII - organizar e dirigir, nos termos da lei, os servi-
ços relativos às terras do Município; São auxiliares do Prefeito:
XXVIII - desenvolver o sistema viário do Município; I - Os Secretários Municipais ou Diretores equivalen-
XXIX - conceder auxílios, prêmios e subvenções nos tes;
limites das respectivas verbas orçamentárias e do pla- II - os Subprefeitos
no de distribuição, prévia e anualmente aprovado pela Parágrafo Único - Os cargos são de livre nomeação e
Câmara; demissão do Prefeito.
XXX - providenciar sobre o incremento do ensino por
todos os setores do Município; A Lei Municipal estabelecerá as atribuições dos auxi-
XXXI - estabelecer a divisão administrativa do Municí- liares diretos do Prefeito, definindo-lhes a competên-
pio, de acordo com a lei; cia, deveres e responsabilidades.
XXXII - solicitar o auxílio das autoridades policiais do São condições essenciais para a investidura no cargo
Estado para garantia do cumprimento de seus atos; de Secretário ou Diretor equivalente:
XXXIII - solicitar, obrigatoriamente, autorização à Câ- I – ser brasileiro;
mara para ausentar-se do Município por tempo supe- II – estar no exercício dos direitos políticos;
rior a 15 (quinze) dias; III – ser maior de 21 (vinte e um) anos.
XXXIV - adotar providências para conservação e salva- Parágrafo único. A escolha do Secretário ou Diretor
guarda do patrimônio municipal; equivalente deverá recair, preferencialmente, em pes-
XXXV - publicar até 30 (trinta) dias após o encerra- soas de notória capacidade profissional, respectiva ao
mento de cada bimestre, relatório resumido da execu- cargo a que irá ocupar. (Redação alterada conforme: TJ
ção orçamentária. PR - Ação direta de inconstitucionalidade nº 11683813
Art. 67 O Prefeito poderá delegar, por decreto, a seus TR)
auxiliares, as funções administrativas previstas nos in-
cisos IX, XV, XXII, e XXIV do artigo anterior. Além das atribuições fixadas em lei, compete aos Se-
cretários Municipais ou Diretores equivalentes:
DA PERDA E EXTINÇÃO DO MANDATO I - subscrever atos e regulamentos referentes aos seus
órgãos;
É vedado ao Prefeito assumir outro cargo ou função II - expedir instruções para a boa execução e cumpri-
na administração pública direta ou indireta, ressalvada mento das leis, decretos e regulamentos;
LEGISLAÇÃO ESPECÍFICA

a posse em virtude de concurso público e observado III - apresentar ao Prefeito, relatório anual dos serviços
o disposto na Constituição Federal, Constituição Esta- realizados por suas repartições;
dual e nesta Lei Orgânica. IV - comparecer à Câmara Municipal, sempre que con-
§ 1º É igualmente vedado ao Prefeito, desempenhar fun- vocados pela mesma, para prestação de esclarecimen-
ção de administração em qualquer empresa privada. tos oficiais.
§ 2º A infringência ao disposto neste artigo e em seu § Parágrafo Único - A infringência ao inciso IV deste ar-
1º, importará em perda de mandato. tigo, sem justificação, importa em crime de responsa-
bilidade.

10
Os Secretários Municipais ou Diretores equivalentes X - a revisão geral da remuneração dos servidores pú-
são solidariamente responsáveis com o Prefeito pelos blicos, far-se-á sempre na mesma data;
atos que assinarem, ordenarem ou praticarem. XI - a lei fixará o limite máximo e a relação de valores
A competência do Subprefeitos limitar-se-á ao Distrito entre a maior e a menor remuneração dos servidores
para o qual foi nomeado. públicos, observado como limite máximo o valor do
Parágrafo Único - Aos Subprefeitos, como delegados subsídio do Prefeito Municipal; (Redação dada pela
do Executivo, compete: Emenda à Lei Orgânica nº 11/2012)
I - cumprir e fazer cumprir, de acordo com as instru- XII - os vencimentos dos cargos do Poder Legislativo não
ções recebidas, as leis, resoluções, regulamentos e de- poderão ser superior aos pagos pelo Poder Executivo;
mais atos do Prefeito e da Câmara; XIII - é vedada a vinculação ou equiparação de venci-
II - fiscalizar os serviços distritais; mentos, para efeito de remuneração de pessoal do ser-
III - atender às reclamações das partes e encaminhá- viço público, ressalvado o disposto no inciso anterior e
-las ao Prefeito, quando se tratar de matéria estranha no Art. 84, § 1º, desta Lei Orgânica.
às suas atribuições ou quando lhes for favorável a de- XIV - os acréscimos pecuniários percebidos por servi-
cisão proferida; dor público, não serão computados nem acumulados,
IV - indicar ao Prefeito as providências necessárias ao para fins de concessão de acréscimos ulteriores, sob o
Distrito; mesmo título ou idêntico fundamento;
V - prestar contas ao Prefeito mensalmente, ou quan- XV - os vencimentos dos servidores públicos são irre-
do lhe forem solicitadas. dutíveis e a remuneração observará o que dispõem os
O Subprefeito, em caso de licença ou impedimento, artigos 37, XI e XII, 150, II, 153, III e 153, § 2º, I, da Cons-
será substituído por pessoa à livre escolha do Prefeito. tituição Federal;
Os auxiliares diretos do Prefeito farão declaração de XVI - é vedada a acumulação remunerada de cargos
bens no ato da posse e no término do exercício do públicos, exceto quando houver compatibilidade de
cargo. horários:
a) a dois cargos de professor;
DA ADMINISTRAÇÃO PÚBLICA b) a de um cargo de professor com outro técnico ou
científico;
c) a de dois cargos privativos de médico;
A Administração Pública direta e indireta do Município,
XVII - a proibição de acumular, estende-se a empregos
obedecerá aos princípios de legalidade, impessoalida-
e funções, e, abrange autarquias, empresas públicas,
de, moralidade, publicidade e também ao seguinte:
sociedade de economia mista e fundações mantidas
I - os cargos, empregos e funções públicas são acessí-
pelo Poder Público Municipal;
veis aos brasileiros que preencham os requisitos esta-
XVIII - a administração fazendária e seus servidores fis-
belecidos em lei;
cais, terão dentro de suas áreas de competência e juris-
II - a investidura em cargo ou emprego público de- dição, precedência sobre os demais setores administra-
pende de aprovação prévia em concurso público de tivos, na forma da lei;
provas ou de provas e títulos, ressalvadas as nome- XIX - somente por lei específica, poderão ser criadas
ações para cargo em comissão declarado em lei, de empresas públicas, sociedade de economia mista, au-
livre nomeação e exoneração; tarquia ou fundação pública;
III - o prazo de validade do concurso público será de XX - depende de autorização legislativa, em cada caso,
até dois anos, prorrogável uma vez por igual período; a criação de subsidiárias das entidades mencionadas no
IV - durante o prazo improrrogável previsto no edital inciso anterior, assim como a participação de qualquer
de convocação, o candidato aprovado em concurso delas em empresa privada;
público de provas ou de provas e títulos, será convo- XXI - ressalvados os casos especificados na legislação, as
cado com prioridade sobre novos concursados para obras, serviços, compras e alienação pública que assegu-
assumir cargos ou empregos na carreira; re igualdade de condições a todos os concorrentes, com
V - os cargos em comissão e as funções de confian- cláusulas que estabeleçam obrigações de pagamento,
ça serão exercidos preferencialmente por servidores mantidas as condições efetivas de proposta, nos termos
ocupantes de cargo de carreira técnica ou profissional, da lei, exigindo-se a qualificação técnico- econômica in-
nos casos e condições previstos em lei; dispensável à garantia do cumprimento das obrigações.
VI - é garantido ao Servidor Público Municipal Civil o Ressalta-se que ao servidor público com exercício de
direito à livre associação sindical, bem como aos elei- mandato eletivo aplicam-se as seguintes disposições:
tos presidentes da instituição, a liberação do horário I - tratando-se de mandato eletivo federal, ou estadual,
integral de trabalho, para dedicar-se àquela função, ficará afastado de seu cargo, emprego ou função;
durante toda a sua gestão; II - investido no mandato de Prefeito, será afastado do
LEGISLAÇÃO ESPECÍFICA

VII - o direito de greve será exercido nos termos e nos cargo, emprego ou função, sendo-lhe facultado optar
limites definidos em lei complementar federal; pela remuneração;
VIII - a lei reservará percentual dos cargos e empregos III - investido no mandato de Vereador, havendo com-
públicos para as pessoas portadoras de deficiências e patibilidade de horários, perceberá as vantagens de
definirá os critérios de sua admissão; seu cargo, emprego ou função, sem prejuízo do sub-
IX - a lei estabelecerá os casos de contratação por sídio do cargo eletivo, e, não havendo compatibilida-
tempo determinado para atender a necessidade tem- de, será aplicada a norma do inciso anterior; (Redação
porária e de excepcional interesse público; dada pela Emenda à Lei Orgânica nº 11/2012)

11
IV - em qualquer caso que exija o afastamento para XVI - Proibição de diferença de vencimentos, de exer-
o exercício de mandato eletivo, seu tempo de serviço cício de funções e de critérios de admissão por motivo
será contado para todos os efeitos legais, exceto para do sexo, idade, cor ou estado civil; (Redação acrescida
promoção por merecimento; pela Emenda à Lei Orgânica nº 9/2009)
V - para efeito de benefício previdenciário, no caso de XVII - Adicionais por tempo de serviço, na forma que a
afastamento, os valores serão determinados como se lei estabelecer; (Redação acrescida pela Emenda à Lei
no exercício estivesse. Orgânica nº 9/2009)
XVIII - Licença prêmio, licença sem vencimento, licença
SERVIDORES PÚBLICOS para tratamento de saúde e licença por motivo de do-
enças de pessoa de família, na forma da lei; (Redação
De acordo com o artigo 84 da Lei, são direitos dos acrescida pela Emenda à Lei Orgânica nº 9/2009)
servidores públicos, entre outros: (Redação acrescida XIX - Assistência e previdência sociais, extensivas aos
pela Emenda à Lei Orgânica nº 9/2009) dependentes e ao cônjuge. (Redação acrescida pela
I - Vencimentos ou proventos não inferiores ao salário Emenda à Lei Orgânica nº 9/2009)
mínimo; (Redação acrescida pela Emenda à Lei Orgâ- Não obstante, quanto à aposentadoria, tem-se que o
nica nº 9/2009) servidor será aposentado:
II - Irredutibilidade dos vencimentos; (Redação acres- I - por invalidez permanente, sendo os proventos e be-
cida pela Emenda à Lei Orgânica nº 9/2009) nefícios integrais, quando decorrentes de acidente em
III - Garantia de vencimento nunca inferior ao sa- serviço, moléstia profissional ou doença grave, conta-
lário mínimo para os que percebem remuneração giosa ou incurável, especificadas em lei e proporcionais
variável;(Redação acrescida pela Emenda à Lei Orgâ- nos demais casos;
nica nº 9/2009) II - compulsoriamente, aos setenta anos de idade, com
IV - Décimo terceiro vencimentos com base na remu- proventos proporcionais ao tempo de serviço;
neração integral ou no valor da aposentadoria; (Reda- III - voluntariamente:
ção acrescida pela Emenda à Lei Orgânica nº 9/2009) a) aos trinta e cinco ano de serviço, se homem, e aos
V - Remuneração do trabalho noturno superior a do trinta anos, se mulher, com proventos integrais;
diurno; (Redação acrescida pela Emenda à Lei Orgâni- b) aos trinta anos de efetivo exercício em funções de
ca nº 9/2009) magistério, se professor, e vinte e cinco anos, se pro-
VI - Salário família para os dependentes, na forma da fessora, com proventos integrais;
lei; (Redação acrescida pela Emenda à Lei Orgânica nº c) aos trinta anos de serviço, se homem, e aos vinte e
9/2009) cinco anos, se mulher, com proventos proporcionais a
VII - duração de jornada de trabalho normal não su- esse tempo;
perior a oito horas diárias e quarenta horas semanais, d) aos sessenta e cinco anos de idade, se homem, e aos
facultadas a compensação de horário, escalas e redu- sessenta anos, se mulher, com proventos proporcionais
ção de jornada; (Redação dada pela Emenda à Lei Or- ao tempo de serviço;
gânica nº 17/2019) Por fim, esclarecemos que são estáveis, após dois anos
VIII - Repouso semanal remunerado; (Redação acresci- de efetivo e comprovado exercício, os servidores no-
da pela Emenda à Lei Orgânica nº 9/2009) meados em virtude de concurso público.
IX - Remuneração do serviço extraordinário su- O servidor público estável só perderá o cargo em vir-
perior, no mínimo, em cinquenta por cento a do tude de sentença judicial transitada em julgado ou
normal;(Redação acrescida pela Emenda à Lei Orgâ- mediante processo administrativo em que lhe seja as-
nica nº 9/2009) segurada ampla defesa.
X - Gozo de férias anuais remuneradas, pelo menos, Invalidada por sentença judicial a demissão do servidor
com um terço a mais que a remuneração normal, ve- estável, será ele reintegrado e o eventual ocupante da
dada a contagem em dobro; (Redação acrescida pela vaga, reconduzido ao cargo de origem, sem direito a
Emenda à Lei Orgânica nº 9/2009) indenização, aproveitado em outro cargo ou posto em
XI - Licença à gestante, sem prejuízo do emprego e disponibilidade.
dos vencimentos, e com duração de 180 (cento e oi- Extinto o cargo ou declarada a sua desnecessidade, o
tenta) dias consecutivos, mediante inspeção médica, servidor estável ficará em disponibilidade remunerada,
nos termos da lei; (Redação acrescida pela Emenda à até seu adequado aproveitamento em outro cargo.
Lei Orgânica nº 9/2009)
XII - Licença paternidade nos termos fixados em Dos tributos
lei; (Redação acrescida pela Emenda à Lei Orgânica nº São de competência do Município, os impostos sobre:
9/2009) I - propriedade predial e territorial urbana;
XIII - Proteção do trabalho da mulher, nos termos da II - transmissão inter-vivos, a qualquer título, por ato
LEGISLAÇÃO ESPECÍFICA

lei; (Redação acrescida pela Emenda à Lei Orgânica nº oneroso de bens imóveis, por natureza ou acessão
9/2009) física e de direitos reais sobre imóveis, exceto os de
XIV - Redução dos riscos inerentes ao trabalho por garantia, bem como cessão de direito à sua aquisição;
meio de normas de saúde, higiene e segurança; (Reda- III - vendas a varejo de combustíveis líquidos e gasoli-
ção acrescida pela Emenda à Lei Orgânica nº 9/2009) na, exceto óleo diesel;
XV - Adicional de remuneração para atividades peno- IV - serviços de qualquer natureza, não compreendidos
sas, insalubres ou perigosas, na forma da lei; (Redação na competência do Estado, definidos na lei comple-
acrescida pela Emenda à Lei Orgânica nº 9/2009) menta prevista no artigo 146 da Constituição Federal.

12
DOS PRINCÍPIOS GERAIS DA ATIVIDADE A sociedade possui o direito a segurança, no entan-
ECONÔMICA to, deve cooperar para que a paz social e a ordem se
mantenha. Neste diapasão, é dever do Estado buscar
O Município, na sua circunscrição territorial e dentro meios para concretizar a segurança através do poder
de sua competência constitucional, assegura a todos, de polícia.
dentro dos princípios da ordem econômica, fundada A atividade policial é carreira de Estado imprescindível
na valorização do trabalho humano e na livre iniciativa, à manutenção da normalidade democrática, sendo im-
existência digna, observados os seguintes princípios: possível sua complementação ou substituição pela ati-
I - autonomia municipal; vidade privada. A carreira policial é o braço armado do
II - propriedade privada; Estado, responsável pela garantia da segurança interna,
III - função social da propriedade; ordem pública e paz social (...) [ARE 654.432, rel. p/ o
IV - livre concorrência; ac. min. Alexandre de Moraes, j. 5-4-2017, P, DJE de
V - defesa do consumidor; 11-6-2018, Tema 541.]
VI - defesa do meio ambiente; O Estado delega a função de exercer a segurança públi-
VII - redução das desigualdades regionais e sociais; ca através dos órgãos: 1) Polícia Federal, 2) Rodoviária
VIII - busca de pleno emprego; Federal, 3) Ferroviária Federal, 4) Policia Civil e 5) Milita-
IX - tratamento favorecido para cooperativas e empre- res e Bombeiros.
sas brasileiras de pequeno porte e microempresas; Importante lembrar que, através da Ação Direito de In-
constitucionalidade (ADI) nº 236-8-RJ, os órgãos des-
critos acima são os únicos responsáveis para realizar a
PORTARIA INTERMINISTERIAL Nº 4.226/2010 função de proteção, não podendo ser delegado a mais
nenhum órgão, ente ou pessoa física ou jurídica.
O item I “Policia Federal” é um órgão organizado e
mantido pela União, o qual subordina-se ao Ministério
A presente portaria, de forma muito simples e eficaz, da Justiça.
prevê diretrizes para o uso de força pelos agentes de As principais funções da Polícia Federal é investigar e
segurança pública, compostos pelo Departamento de apurar as infrações penais que atinjam exclusivamente
Polícia Federal, pelo Departamento de Polícia Rodovi- a União, suas entidades autárquicas ou empresas públi-
ária Federal, pelo Departamento Penitenciário Nacio- cas, até mesmo investigações de outras funções (desde
nal e pela Força Nacional de Segurança Pública. que previstas). Ainda assim, a polícia federal é respon-
Conforme a Constituição Federal, a segurança pública sável por exercer as funções de polícia marítima, aero-
é obrigação do Estado, sendo todos os indivíduos de- portuária e de fronteiras, bem como, exercer, com exclu-
tentores de direitos e responsabilidades para preser- sividade, as funções de polícia judiciária da União.
var a ordem pública, bem como da incolumidade das No tocante a Polícia Rodoviária Federal, o mesmo tem o
pessoas e do patrimônio. dever de realizar patrulhamento ostensivo das rodovias
O conceito jurídico de ordem pública não se confunde federais, para fiscalizar o tráfego nas rodovias e evitar
com incolumidade das pessoas e do patrimônio (art. crimes de trânsito. Ainda assim, o policial rodoviário fe-
144 da CF/1988). Sem embargo, ordem pública se deral também é responsável pelo controle das frontei-
constitui em bem jurídico que pode resultar mais ou ras do país. E também, é responsável por mais algumas
menos fragilizado pelo modo personalizado com que funções descritas no art.20 do CTB.
se dá a concreta violação da integridade das pessoas Já a Polícia Ferroviária Federal, destina-se também a
ou do patrimônio de terceiros, tanto quanto da saúde realizar patrulhamento ostensivo, porém, nas ferrovias,
pública (nas hipóteses de tráfico de entorpecentes e e é responsável pela fiscalização, repressão de atos de
drogas afins). Daí sua categorização jurídico-positiva, vandalismo e crimes, e prevenção de acidentes e em
não como descrição do delito nem cominação de toda a malha ferroviária do país.
pena, porém como pressuposto de prisão cautelar; ou Os Policiais Civis, são policiais capacitados por apurar
seja, como imperiosa necessidade de acautelar o meio infrações penais, realizar Boletins de Ocorrência, inves-
social contra fatores de perturbação que já se loca- tigar algum crime, bem como a função de polícia judi-
lizam na gravidade incomum da execução de certos ciária.
crimes. Não da incomum gravidade abstrata deste ou Por fim, o Policial Militar é responsável, principalmente,
daquele crime, mas da incomum gravidade na perpe- pela ordem pública e paz social através da segurança.
tração em si do crime, levando à consistente ilação de Já o Corpo de Bombeiro, além das atribuições definidas
que, solto, o agente reincidirá no delito. Donde o vín- em lei, incumbe a execução de atividades de defesa civil.
culo operacional entre necessidade de preservação da
LEGISLAÇÃO ESPECÍFICA

ordem pública e acautelamento do meio social. Logo,


conceito de ordem pública que se desvincula do con- FIQUE ATENTO!
ceito de incolumidade das pessoas e do patrimônio As polícias militares e corpos de bombeiros
alheio (assim como da violação à saúde pública), mas militares, forças auxiliares e reserva do
que se enlaça umbilicalmente à noção de acautela- Exército, subordinam-se, juntamente com as
mento do meio social. polícias civis, aos Governadores dos Estados,
[HC 101.300, rel. min. Ayres Britto, j. 5-10-2010, 2ª do Distrito Federal e dos Territórios.
T, DJE 18-11-2010.]

13
*Os municípios podem constituir guardas munici- A constituição, por sua vez, é o documento que
pais para auxílio na segurança pública. alicerça os fundamentos do Estado para a qual ela foi
delineada. Também é possível utilizar outros sinônimos
Vide: https://www.conjur.com.br/dl/integra-portaria- como constituir, delimitar, organizar; enfim, a Constitui-
-ministerial.pdf ção tem essa finalidade: organizar e estruturar o Estado.
Portanto, podemos definir constituição como um
conglomerado de normas de caráter fundamental e su-
premo, escritas ou alicerçadas nos costumes, responsá-
EXERCÍCIOS COMENTADOS veis pela criação, estruturação e organização do Estado
– uma espécie de estatuto do poder.
1- (PC-MA – ESCRIVÃO DE POLÍCIA – CESPE – 2018) O título II da Constituição Federal é intitulado “Direitos
A segurança pública é uma forma de serviço público de e Garantias fundamentais”, gênero que abrange as seguin-
natureza tes espécies de direitos fundamentais: direitos individuais
a) geral. e coletivos (art. 5º, CF), direitos sociais (genericamente
b) administrativa.
previstos no art. 6º, CF), direitos da nacionalidade (artigos
c) descentralizada.
12 e 13, CF) e direitos políticos (artigos 14 a 17, CF).
d) não exclusiva.
Em termos comparativos à clássica divisão tridi-
e) individual.
mensional dos direitos humanos, os direitos individuais
A segurança pública é de natureza pública geral e in- (maior parte do artigo 5º, CF), os direitos da nacionali-
delegável, sendo prestada para toda coletividade. dade e os direitos políticos se encaixam na primeira di-
GABARITO OFICIAL: A mensão (direitos civis e políticos); os direitos sociais se
enquadram na segunda dimensão (direitos econômicos,
2- O processo legislativo municipal não compreende a sociais e culturais) e os direitos coletivos na terceira di-
elaboração de: mensão. Contudo, a enumeração de direitos humanos
a) emendas à Lei Orgânica Municipal; na Constituição vai além dos direitos que expressamente
b) leis complementares; constam no título II do texto constitucional.
c) leis ordinárias;
d) leis delegadas; Características principais:
e) emendas à constituição estadual
a) Historicidade: os direitos fundamentais possuem
As emendas as constituições estaduais somente po- antecedentes históricos relevantes e, através dos
derão ser feitas pelas assembléias de cada estado, se- tempos, adquirem novas perspectivas. Nesta ca-
guindo as regras estabelecidas pela constituição esta- racterística se enquadra a noção de dimensões de
dual competente. direitos.
GABARITO: E b) Universalidade: os direitos fundamentais perten-
cem a todos, tanto que apesar da expressão res-
tritiva do caput do artigo 5º aos brasileiros e es-
CONSTITUIÇÃO DA REPÚBLICA trangeiros residentes no país tem se entendido pela
FEDERATIVA DO BRASIL; DOS DIREITOS E extensão destes direitos, na perspectiva de preva-
GARANTIAS FUNDAMENTAIS - CAPÍTULO lência dos direitos humanos.
I (DOS DIREITOS E DEVERES INDIVIDUAIS c) Inalienabilidade: os direitos fundamentais não
E COLETIVOS); DA ORGANIZAÇÃO DO possuem conteúdo econômico-patrimonial, logo,
ESTADO; DA DEFESA DO ESTADO E DAS são intransferíveis, inegociáveis e indisponíveis, es-
INSTITUIÇÕES DEMOCRÁTICAS - CAPÍTULO tando fora do comércio, o que evidencia uma limi-
III (DA SEGURANÇA PÚBLICA); tação do princípio da autonomia privada.
d) Irrenunciabilidade: direitos fundamentais não po-
dem ser renunciados pelo seu titular devido à fun-
A disciplina de direito constitucional é talvez a mais damentalidade material destes direitos para a dig-
importante de todo o ordenamento jurídico, em especial nidade da pessoa humana.
do brasileiro, posto que todas as demais normas devem e) Inviolabilidade: direitos fundamentais não podem
estar de acordo com a Constituição Federal. deixar de ser observados por disposições infra-
Segundo Nathália Masson, “Direito Constitucional constitucionais ou por atos das autoridades públi-
é um dos ramos do Direito Público, a matriz que fun- cas, sob pena de nulidades.
LEGISLAÇÃO ESPECÍFICA

damenta e orienta todo o ordenamento jurídico. Surgiu f) Indivisibilidade: os direitos fundamentais com-
com os ideais liberais atentando-se, a princípio, para a põem um único conjunto de direitos porque não
organização estrutural do Estado, o exercício e transmis- podem ser analisados de maneira isolada, separa-
são do poder e a enumeração de direitos e garantias da.
fundamentais dos indivíduos. Atualmente, preocupa-se g) Imprescritibilidade: os direitos fundamentais não
não somente com a limitação do poder estatal na esfera se perdem com o tempo, não prescrevem, uma vez
particular, mas também com a finalidade das ações esta- que são sempre exercíveis e exercidos, não deixan-
tais e a ordem social, democrática e política”. do de existir pela falta de uso (prescrição).

14
h) Relatividade: os direitos fundamentais não podem Somente alguns direitos não são estendidos a todas as
ser utilizados como um escudo para práticas ilícitas pessoas. A exemplo, o direito de intentar ação popular exi-
ou como argumento para afastamento ou diminui- ge a condição de cidadão, que só é possuída por nacionais
ção da responsabilidade por atos ilícitos, assim es- titulares de direitos políticos.
tes direitos não são ilimitados e encontram seus li-
mites nos demais direitos igualmente consagrados 2) Relação direitos-deveres
como humanos. O capítulo em estudo é denominado “direitos e garan-
tias deveres e coletivos”, remetendo à necessária relação
Vale destacar que a Constituição vai além da prote- direitos-deveres entre os titulares dos direitos fundamen-
ção dos direitos e estabelece garantias em prol da pre- tais. Acima de tudo, o que se deve ter em vista é a premis-
servação destes, bem como remédios constitucionais a sa reconhecida nos direitos fundamentais de que não há
serem utilizados caso estes direitos e garantias não sejam direito que seja absoluto, correspondendo-se para cada
preservados. Neste sentido, dividem-se em direitos e ga- direito um dever. Logo, o exercício de direitos fundamen-
rantias as previsões do artigo 5º: os direitos são as dis- tais é limitado pelo igual direito de mesmo exercício por
parte de outrem, não sendo nunca absolutos, mas sempre
posições declaratórias e as garantias são as disposições
relativos.
assecuratórias.
Explica Canotilho2 quanto aos direitos fundamentais: “a
O legislador muitas vezes reúne no mesmo dispositivo
ideia de deveres fundamentais é suscetível de ser enten-
o direito e a garantia, como no caso do artigo 5º, IX: “é dida como o ‘outro lado’ dos direitos fundamentais. Como
livre a expressão da atividade intelectual, artística, científi- ao titular de um direito fundamental corresponde um de-
ca e de comunicação, independentemente de censura ou ver por parte de um outro titular, poder-se-ia dizer que o
licença” – o direito é o de liberdade de expressão e a ga- particular está vinculado aos direitos fundamentais como
rantia é a vedação de censura ou exigência de licença. Em destinatário de um dever fundamental. Neste sentido, um
outros casos, o legislador traz o direito num dispositivo e direito fundamental, enquanto protegido, pressuporia um
a garantia em outro: a liberdade de locomoção, direito, é dever correspondente”. Com efeito, a um direito funda-
colocada no artigo 5º, XV, ao passo que o dever de rela- mental conferido à pessoa corresponde o dever de respei-
xamento da prisão ilegal de ofício pelo juiz, garantia, se to ao arcabouço de direitos conferidos às outras pessoas.
encontra no artigo 5º, LXV1.
Em caso de ineficácia da garantia, implicando em vio- 3) Direitos e garantias em espécie
lação de direito, cabe a utilização dos remédios consti- Preconiza o artigo 5º da Constituição Federal em seu
tucionais. caput:
Atenção para o fato de o constituinte chamar os re- Artigo 5º, caput, CF. Todos são iguais perante a lei, sem
médios constitucionais de garantias, e todas as suas fór- distinção de qualquer natureza, garantindo-se aos bra-
mulas de direitos e garantias propriamente ditas apenas sileiros e aos estrangeiros residentes no País a inviola-
de direitos. bilidade do direito à vida, à liberdade, à igualdade, à
segurança e à propriedade, nos termos seguintes [...].
Direitos e deveres individuais e coletivos
O caput do artigo 5º, que pode ser considerado um
O capítulo I do título II é intitulado “direitos e deve- dos principais (senão o principal) artigos da Constituição
res individuais e coletivos”. Da própria nomenclatura do Federal, consagra o princípio da igualdade e delimita as
capítulo já se extrai que a proteção vai além dos direitos cinco esferas de direitos individuais e coletivos que mere-
do indivíduo e também abrange direitos da coletivida- cem proteção, isto é, vida, liberdade, igualdade, segurança
de. A maior parte dos direitos enumerados no artigo 5º e propriedade. Os incisos deste artigos delimitam vários
do texto constitucional é de direitos individuais, mas são direitos e garantias que se enquadram em alguma destas
incluídos alguns direitos coletivos e mesmo remédios esferas de proteção, podendo se falar em duas esferas es-
constitucionais próprios para a tutela destes direitos co- pecíficas que ganham também destaque no texto consti-
letivos (ex.: mandado de segurança coletivo). tucional, quais sejam, direitos de acesso à justiça e direitos
constitucionais-penais.
1) Brasileiros e estrangeiros
O caput do artigo 5º aparenta restringir a proteção - Direito à igualdade
conferida pelo dispositivo a algumas pessoas, notada-
mente, “aos brasileiros e aos estrangeiros residentes no Abrangência
País”. No entanto, tal restrição é apenas aparente e tem Observa-se, pelo teor do caput do artigo 5º, CF, que o
sido interpretada no sentido de que os direitos estarão constituinte afirmou por duas vezes o princípio da igual-
LEGISLAÇÃO ESPECÍFICA

protegidos com relação a todas as pessoas nos limites da dade:


soberania do país. Artigo 5º, caput, CF. Todos são iguais perante a lei, sem
Em razão disso, por exemplo, um estrangeiro pode distinção de qualquer natureza, garantindo-se aos bra-
ingressar com habeas corpus ou mandado de seguran- sileiros e aos estrangeiros residentes no País a inviola-
ça, ou então intentar ação reivindicatória com relação a bilidade do direito à vida, à liberdade, à igualdade, à
imóvel seu localizado no Brasil (ainda que não resida no segurança e à propriedade, nos termos seguintes [...].
país).
1 FARIA, Cássio Juvenal. Notas pessoais tomadas em teleconferên- 2 CANOTILHO, José Joaquim Gomes. Direito constitucional e teoria da
cia. constituição. 2. ed. Coimbra: Almedina, 1998, p. 479.

15
Não obstante, reforça este princípio em seu primeiro dívidas históricas, como uma compensação aos negros
inciso: por tê-los feito escravos, p. ex.); representam o ideal de
Artigo 5º, I, CF. Homens e mulheres são iguais em justiça distributiva (a preocupação, aqui, é com o presen-
direitos e obrigações, nos termos desta Constituição. te. Busca-se uma concretização do princípio da igualdade
material); bem como promovem a diversidade.
Este inciso é especificamente voltado à necessidade Neste sentido, as discriminações legais asseguram a
de igualdade de gênero, afirmando que não deve ha- verdadeira igualdade, por exemplo, com as ações afir-
ver nenhuma distinção sexo feminino e o masculino, de mativas, a proteção especial ao trabalho da mulher e do
modo que o homem e a mulher possuem os mesmos menor, as garantias aos portadores de deficiência, entre
direitos e obrigações. outras medidas que atribuam a pessoas com diferentes
Entretanto, o princípio da isonomia abrange muito condições, iguais possibilidades, protegendo e respei-
mais do que a igualdade de gêneros, envolve uma pers- tando suas diferenças3. Tem predominado em doutrina
pectiva mais ampla. e jurisprudência, inclusive no Supremo Tribunal Federal,
O direito à igualdade é um dos direitos norteadores que as ações afirmativas são válidas.
de interpretação de qualquer sistema jurídico. O primeiro
enfoque que foi dado a este direito foi o de direito civil, - Direito à vida
enquadrando-o na primeira dimensão, no sentido de que Abrangência
a todas as pessoas deveriam ser garantidos os mesmos O caput do artigo 5º da Constituição assegura a prote-
direitos e deveres. Trata-se de um aspecto relacionado à ção do direito à vida. A vida humana é o centro gravitacio-
igualdade enquanto liberdade, tirando o homem do ar- nal em torno do qual orbitam todos os direitos da pessoa
bítrio dos demais por meio da equiparação. Basicamente, humana, possuindo reflexos jurídicos, políticos, econômi-
estaria se falando na igualdade perante a lei. cos, morais e religiosos. Daí existir uma dificuldade em
No entanto, com o passar dos tempos, se percebeu conceituar o vocábulo vida. Logo, tudo aquilo que uma
que não bastava igualar todos os homens em direitos e pessoa possui deixa de ter valor ou sentido se ela perde
a vida. Sendo assim, a vida é o bem principal de qualquer
deveres para torná-los iguais, pois nem todos possuem
pessoa, é o primeiro valor moral inerente a todos os seres
as mesmas condições de exercer estes direitos e deveres.
humanos4.
Logo, não é suficiente garantir um direito à igualdade
No tópico do direito à vida tem-se tanto o direito de
formal, mas é preciso buscar progressivamente a igual-
nascer/permanecer vivo, o que envolve questões como
dade material. No sentido de igualdade material que
pena de morte, eutanásia, pesquisas com células-tronco
aparece o direito à igualdade num segundo momento,
e aborto; quanto o direito de viver com dignidade, o
pretendendo-se do Estado, tanto no momento de legis-
que engloba o respeito à integridade física, psíquica e
lar quanto no de aplicar e executar a lei, uma postura de
moral, incluindo neste aspecto a vedação da tortura, bem
promoção de políticas governamentais voltadas a gru-
pos vulneráveis. como a garantia de recursos que permitam viver a vida
Assim, o direito à igualdade possui dois sentidos no- com dignidade.
táveis: o de igualdade perante a lei, referindo-se à apli- Embora o direito à vida seja em si pouco delimitado
cação uniforme da lei a todas as pessoas que vivem em nos incisos que seguem o caput do artigo 5º, trata-se de
sociedade; e o de igualdade material, correspondendo um dos direitos mais discutidos em termos jurispruden-
à necessidade de discriminações positivas com relação ciais e sociológicos. É no direito à vida que se encaixam
a grupos vulneráveis da sociedade, em contraponto à polêmicas discussões como: aborto de anencéfalo, pes-
igualdade formal. quisa com células tronco, pena de morte, eutanásia, etc.

Ações afirmativas Vedação à tortura


Neste sentido, desponta a temática das ações afir- De forma expressa no texto constitucional destaca-se
mativas,que são políticas públicas ou programas priva- a vedação da tortura, corolário do direito à vida, confor-
dos criados temporariamente e desenvolvidos com a me previsão no inciso III do artigo 5º:
finalidade de reduzir as desigualdades decorrentes de
discriminações ou de uma hipossuficiência econômica Artigo 5º, III, CF. Ninguém será submetido a tortura
ou física, por meio da concessão de algum tipo de van- nem a tratamento desumano ou degradante.
tagem compensatória de tais condições.
Quem é contra as ações afirmativas argumenta que, A tortura é um dos piores meios de tratamento desu-
em uma sociedade pluralista, a condição de membro de mano, expressamente vedada em âmbito internacional,
um grupo específico não pode ser usada como critério como visto no tópico anterior. No Brasil, além da disci-
de inclusão ou exclusão de benefícios. Ademais, afirma- plina constitucional, a Lei nº 9.455, de 7 de abril de 1997
LEGISLAÇÃO ESPECÍFICA

-se que elas desprivilegiam o critério republicano do define os crimes de tortura e dá outras providências, des-
mérito (segundo o qual o indivíduo deve alcançar de- tacando-se o artigo 1º:
terminado cargo público pela sua capacidade e esforço,
3 SANFELICE, Patrícia de Mello. Comentários aos artigos I e II. In:
e não por pertencer a determinada categoria); fomenta- BALERA, Wagner (Coord.). Comentários à Declaração Universal dos
riam o racismo e o ódio; bem como ferem o princípio da Direitos do Homem. Brasília: Fortium, 2008, p. 08.
isonomia por causar uma discriminação reversa. 4 BARRETO, Ana Carolina Rossi; IBRAHIM, Fábio Zambitte. Comen-
Por outro lado, quem é favorável às ações afirmati- tários aos Artigos III e IV. In: BALERA, Wagner (Coord.). Comentários
vas defende que elas representam o ideal de justiça com- à Declaração Universal dos Direitos do Homem. Brasília: Fortium,
pensatória (o objetivo é compensar injustiças passadas, 2008, p. 15.

16
Art. 1º Constitui crime de tortura: Liberdade de pensamento e de expressão
I - constranger alguém com emprego de violência ou O artigo 5º, IV, CF prevê:
grave ameaça, causando-lhe sofrimento físico ou men-
tal: Artigo 5º, IV, CF. É livre a manifestação do pensa-
a) com o fim de obter informação, declaração ou con- mento, sendo vedado o anonimato.
fissão da vítima ou de terceira pessoa;
b) para provocar ação ou omissão de natureza crimi- Consolida-se a afirmação simultânea da liberdade de
nosa; pensamento e da liberdade de expressão.
c) em razão de discriminação racial ou religiosa; Em primeiro plano tem-se a liberdade de pensamen-
II - submeter alguém, sob sua guarda, poder ou auto- to. Afinal, “o ser humano, através dos processos internos
ridade, com emprego de violência ou grave ameaça, de reflexão, formula juízos de valor. Estes exteriorizam
a intenso sofrimento físico ou mental, como forma de nada mais do que a opinião de seu emitente. Assim, a re-
aplicar castigo pessoal ou medida de caráter preven- gra constitucional, ao consagrar a livre manifestação do
tivo. pensamento, imprime a existência jurídica ao chamado
Pena - reclusão, de dois a oito anos. direito de opinião”5. Em outras palavras, primeiro existe o
§ 1º Na mesma pena incorre quem submete pessoa direito de ter uma opinião, depois o de expressá-la.
presa ou sujeita a medida de segurança a sofrimento No mais, surge como corolário do direito à liberdade
físico ou mental, por intermédio da prática de ato não de pensamento e de expressão o direito à escusa por con-
previsto em lei ou não resultante de medida legal. vicção filosófica ou política:
§ 2º Aquele que se omite em face dessas condutas,
quando tinha o dever de evitá-las ou apurá-las, incor- Artigo 5º, VIII, CF. Ninguém será privado de direitos
re na pena de detenção de um a quatro anos. por motivo de crença religiosa ou de convicção filosó-
§ 3º Se resulta lesão corporal de natureza grave ou fica ou política, salvo se as invocar para eximir-se de
gravíssima, a pena é de reclusão de quatro a dez obrigação legal a todos imposta e recusar-se a cum-
anos; se resulta morte, a reclusão é de oito a dezes-
prir prestação alternativa, fixada em lei.
seis anos.
Trata-se de instrumento para a consecução do direito
§ 4º Aumenta-se a pena de um sexto até um terço:
I - se o crime é cometido por agente público; assegurado na Constituição Federal – não basta permitir
que se pense diferente, é preciso respeitar tal posiciona-
II – se o crime é cometido contra criança, gestante, mento.
portador de deficiência, adolescente ou maior de 60 Com efeito, este direito de liberdade de expressão é
(sessenta) anos; limitado. Um destes limites é o anonimato, que consiste
III - se o crime é cometido mediante sequestro. na garantia de atribuir a cada manifestação uma autoria
§ 5º A condenação acarretará a perda do cargo, fun- certa e determinada, permitindo eventuais responsabili-
ção ou emprego público e a interdição para seu exer- zações por manifestações que contrariem a lei.
cício pelo dobro do prazo da pena aplicada. Tem-se, ainda, a seguinte previsão no artigo 5º, IX, CF:
§ 6º O crime de tortura é inafiançável e insuscetível de
graça ou anistia. Artigo 5º, IX, CF. É livre a expressão da atividade
§ 7º O condenado por crime previsto nesta Lei, salvo a intelectual, artística, científica e de comunicação,
hipótese do § 2º, iniciará o cumprimento da pena em independentemente de censura ou licença.
regime fechado.
Consolida-se outra perspectiva da liberdade de ex-
- Direito à liberdade pressão, referente de forma específica a atividades inte-
O caput do artigo 5º da Constituição assegura a pro-
lectuais, artísticas, científicas e de comunicação. Dispen-
teção do direito à liberdade, delimitada em alguns incisos
sa-se, com relação a estas, a exigência de licença para a
que o seguem.
manifestação do pensamento, bem como veda-se a cen-
Liberdade e legalidade sura prévia.
Prevê o artigo 5º, II, CF: A respeito da censura prévia, tem-se não cabe impedir
a divulgação e o acesso a informações como modo de
Artigo 5º, II, CF. Ninguém será obrigado a fazer ou controle do poder. A censura somente é cabível quando
deixar de fazer alguma coisa senão em virtude de lei. necessária ao interesse público numa ordem democrática,
por exemplo, censurar a publicação de um conteúdo de
O princípio da legalidade se encontra delimitado neste exploração sexual infanto-juvenil é adequado.
inciso, prevendo que nenhuma pessoa será obrigada a fazer O direito à resposta (artigo 5º, V, CF) e o direito à inde-
LEGISLAÇÃO ESPECÍFICA

ou deixar de fazer alguma coisa a não ser que a lei assim nização (artigo 5º, X, CF) funcionam como a contrapartida
determine. Assim, salvo situações previstas em lei, a pessoa para aquele que teve algum direito seu violado (notada-
tem liberdade para agir como considerar conveniente. mente inerentes à privacidade ou à personalidade) em
Portanto, o princípio da legalidade possui estrita rela- decorrência dos excessos no exercício da liberdade de
ção com o princípio da liberdade, posto que, a priori, tudo expressão.
à pessoa é lícito. Somente é vedado o que a lei expressa-
mente estabelecer como proibido. A pessoa pode fazer
tudo o que quiser, como regra, ou seja, agir de qualquer 5 ARAÚJO, Luiz Alberto David; NUNES JÚNIOR, Vidal Serrano. Curso
maneira que a lei não proíba. de direito constitucional. 10. ed. São Paulo: Saraiva, 2006.

17
Liberdade de crença/religiosa Trata-se da liberdade de informação, consistente na
Dispõe o artigo 5º, VI, CF: liberdade de procurar e receber informações e ideias por
quaisquer meios, independente de fronteiras, sem inter-
Artigo 5º, VI, CF. É inviolável a liberdade de consci- ferência.
ência e de crença, sendo assegurado o livre exercí- A liberdade de informação tem um caráter passivo, ao
cio dos cultos religiosos e garantida, na forma da passo que a liberdade de expressão tem uma caracterís-
lei, a proteção aos locais de culto e a suas liturgias. tica ativa, de forma que juntas formam os aspectos ativo
Cada pessoa tem liberdade para professar a sua fé e passivo da exteriorização da liberdade de pensamento:
como bem entender dentro dos limites da lei. Não há uma não basta poder manifestar o seu próprio pensamento, é
crença ou religião que seja proibida, garantindo-se que a preciso que ele seja ouvido e, para tanto, há necessidade
profissão desta fé possa se realizar em locais próprios. de se garantir o acesso ao pensamento manifestado para
Nota-se que a liberdade de religião engloba 3 tipos a sociedade.
distintos, porém intrinsecamente relacionados de liberda- Por sua vez, o acesso à informação envolve o direi-
des: a liberdade de crença; a liberdade de culto; e a liber- to de todos obterem informações claras, precisas e ver-
dade de organização religiosa. dadeiras a respeito de fatos que sejam de seu interesse,
Consoante o magistério de José Afonso da Silva6, entra notadamente pelos meios de comunicação imparciais e
na liberdade de crença a liberdade de escolha da religião, não monopolizados (artigo 220, CF). No entanto, nem
a liberdade de aderir a qualquer seita religiosa, a liberda- sempre é possível que a imprensa divulgue com quem
de (ou o direito) de mudar de religião, além da liberdade obteve a informação divulgada, sem o que a segurança
de não aderir a religião alguma, assim como a liberdade desta poderia ficar prejudicada e a informação inevitavel-
de descrença, a liberdade de ser ateu e de exprimir o ag- mente não chegaria ao público.
nosticismo, apenas excluída a liberdade de embaraçar o Especificadamente quanto à liberdade de informação
livre exercício de qualquer religião, de qualquer crença. no âmbito do Poder Público, merecem destaque algumas
A liberdade de culto consiste na liberdade de orar e de previsões.
praticar os atos próprios das manifestações exteriores em Primeiramente, prevê o artigo 5º, XXXIII, CF:
casa ou em público, bem como a de recebimento de con- Artigo 5º, XXXIII, CF. Todos têm direito a receber dos
tribuições para tanto. Por fim, a liberdade de organização órgãos públicos informações de seu interesse parti-
religiosa refere-se à possibilidade de estabelecimento e cular, ou de interesse coletivo ou geral, que serão pres-
organização de igrejas e suas relações com o Estado. tadas no prazo da lei, sob pena de responsabilidade,
Como decorrência do direito à liberdade religiosa, as- ressalvadas aquelas cujo sigilo seja imprescindível à
segurando o seu exercício, destaca-se o artigo 5º, VII, CF: segurança da sociedade e do Estado.
Artigo 5º, VII, CF. É assegurada, nos termos da lei, a
prestação de assistência religiosa nas entidades ci- A respeito, a Lei nº 12.527, de 18 de novembro de
vis e militares de internação coletiva. 2011 regula o acesso a informações previsto no inciso
XXXIII do art. 5º, CF, também conhecida como Lei do
O dispositivo refere-se não só aos estabelecimentos Acesso à Informação.
prisionais civis e militares, mas também a hospitais. Não obstante, estabelece o artigo 5º, XXXIV, CF:
Ainda, surge como corolário do direito à liberdade re- Artigo 5º, XXXIV, CF. São a todos assegurados, inde-
ligiosa o direito à escusa por convicção religiosa: pendentemente do pagamento de taxas:
Artigo 5º, VIII, CF. Ninguém será privado de direitos a) o direito de petição aos Poderes Públicos em defe-
por motivo de crença religiosa ou de convicção filo- sa de direitos ou contra ilegalidade ou abuso de poder;
sófica ou política, salvo se as invocar para eximir-se b) a obtenção de certidões em repartições públicas,
de obrigação legal a todos imposta e recusar-se a para defesa de direitos e esclarecimento de situações
cumprir prestação alternativa, fixada em lei. de interesse pessoal.

Sempre que a lei impõe uma obrigação a todos, por Quanto ao direito de petição, de maneira prática,
exemplo, a todos os homens maiores de 18 anos o alis- cumpre observar que o direito de petição deve resultar
tamento militar, não cabe se escusar, a não ser que tenha em uma manifestação do Estado, normalmente dirimin-
fundado motivo em crença religiosa ou convicção filosó- do (resolvendo) uma questão proposta, em um verda-
fica/política, caso em que será obrigado a cumprir uma deiro exercício contínuo de delimitação dos direitos e
prestação alternativa, isto é, uma outra atividade que não obrigações que regulam a vida social e, desta maneira,
contrarie tais preceitos. quando “dificulta a apreciação de um pedido que um ci-
dadão quer apresentar” (muitas vezes, embaraçando-lhe
Liberdade de informação o acesso à Justiça); “demora para responder aos pedi-
LEGISLAÇÃO ESPECÍFICA

O direito de acesso à informação também se liga a dos formulados” (administrativa e, principalmente, judi-
uma dimensão do direito à liberdade. Neste sentido, pre- cialmente) ou “impõe restrições e/ou condições para a
vê o artigo 5º, XIV, CF: formulação de petição”, traz a chamada insegurança ju-
Artigo 5º, XIV, CF. É assegurado a todos o acesso à rídica, que traz desesperança e faz proliferar as desigual-
informação e resguardado o sigilo da fonte, quando dades e as injustiças.
necessário ao exercício profissional. Dentro do espectro do direito de petição se insere,
por exemplo, o direito de solicitar esclarecimentos, de
6 SILVA, José Afonso da. Curso de direito constitucional positivo.
solicitar cópias reprográficas e certidões, bem como de
25. ed. São Paulo: Malheiros, 2006.

18
ofertar denúncias de irregularidades. Contudo, o cons- O livre exercício profissional é garantido, respeitados
tituinte, talvez na intenção de deixar clara a obrigação os limites legais. Por exemplo, não pode exercer a profis-
dos Poderes Públicos em fornecer certidões, trouxe a são de advogado aquele que não se formou em Direito e
letra b) do inciso, o que gera confusões conceituais no não foi aprovado no Exame da Ordem dos Advogados do
sentido do direito de obter certidões ser dissociado do Brasil; não pode exercer a medicina aquele que não fez
direito de petição. faculdade de medicina reconhecida pelo MEC e obteve o
Por fim, relevante destacar a previsão do artigo 5º, LX, CF: cadastro no Conselho Regional de Medicina.
Artigo 5º, LX, CF. A lei só poderá restringir a publici-
dade dos atos processuais quando a defesa da inti- Liberdade de reunião
midade ou o interesse social o exigirem. Sobre a liberdade de reunião, prevê o artigo 5º, XVI, CF:
Artigo 5º, XVI, CF. Todos podem reunir-se pacificamen-
Logo,o processo, em regra, não será sigiloso. Apenas te, sem armas, em locais abertos ao público, indepen-
o será quando a intimidade merecer preservação (ex: dentemente de autorização, desde que não frustrem
processo criminal de estupro ou causas de família em outra reunião anteriormente convocada para o mesmo
geral) ou quando o interesse social exigir (ex: investiga- local, sendo apenas exigido prévio aviso à autoridade
ções que possam ser comprometidas pela publicidade). competente.
A publicidade é instrumento para a efetivação da liber-
dade de informação. Pessoas podem ir às ruas para reunirem-se com de-
mais na defesa de uma causa, apenas possuindo o dever
de informar tal reunião. Tal dever remonta-se a questões
Liberdade de locomoção
de segurança coletiva. Imagine uma grande reunião de
Outra faceta do direito à liberdade encontra-se no pessoas por uma causa, a exemplo da Parada Gay, que
artigo 5º, XV, CF: chega a aglomerar milhões de pessoas em algumas capi-
Artigo 5º, XV, CF. É livre a locomoção no território tais: seria absurdo tolerar tal tipo de reunião sem o prévio
nacional em tempo de paz, podendo qualquer pes- aviso do poder público para que ele organize o policia-
soa, nos termos da lei, nele entrar, permanecer ou mento e a assistência médica, evitando algazarras e so-
dele sair com seus bens. correndo pessoas que tenham algum mal-estar no local.
Outro limite é o uso de armas, totalmente vedado, assim
A liberdade de locomoção é um aspecto básico do como de substâncias ilícitas (Ex: embora a Marcha da Ma-
direito à liberdade, permitindo à pessoa ir e vir em todo conha tenha sido autorizada pelo Supremo Tribunal Fede-
o território do país em tempos de paz (em tempos de ral, vedou-se que nela tal substância ilícita fosse utilizada).
guerra é possível limitar tal liberdade em prol da se-
gurança). A liberdade de sair do país não significa que Liberdade de associação
existe um direito de ingressar em qualquer outro país, No que tange à liberdade de reunião, traz o artigo 5º,
pois caberá à ele, no exercício de sua soberania, contro- XVII, CF:
lar tal entrada. Classicamente, a prisão é a forma de res-
trição da liberdade. Neste sentido, uma pessoa somen- Artigo 5º, XVII, CF. É plena a liberdade de associação
para fins lícitos, vedada a de caráter paramilitar.
te poderá ser presa nos casos autorizados pela própria
Constituição Federal. A despeito da normativa específica
A liberdade de associação difere-se da de reunião por
de natureza penal, reforça-se a impossibilidade de se sua perenidade, isto é, enquanto a liberdade de reunião
restringir a liberdade de locomoção pela prisão civil por é exercida de forma sazonal, eventual, a liberdade de as-
dívida. sociação implica na formação de um grupo organizado
Prevê o artigo 5º, LXVII, CF: que se mantém por um período de tempo considerável,
Artigo 5º, LXVII, CF. Não haverá prisão civil por dí- dotado de estrutura e organização próprias.
vida, salvo a do responsável pelo inadimplemento Por exemplo, o PCC e o Comando vermelho são as-
voluntário e inescusável de obrigação alimentícia e a sociações ilícitas e de caráter paramilitar, pois possuem
do depositário infiel. armas e o ideal de realizar sua própria justiça paralela-
mente à estatal.
Nos termos da Súmula Vinculante nº 25 do Supremo O texto constitucional se estende na regulamentação
Tribunal Federal, “é ilícita a prisão civil de depositário da liberdade de associação.
infiel, qualquer que seja a modalidade do depósito”. Por O artigo 5º, XVIII, CF, preconiza:
isso, a única exceção à regra da prisão por dívida do Artigo 5º, XVIII, CF. A criação de associações e, na
ordenamento é a que se refere à obrigação alimentícia. forma da lei, a de cooperativas independem de au-
LEGISLAÇÃO ESPECÍFICA

torização, sendo vedada a interferência estatal em seu


Liberdade de trabalho funcionamento.
O direito à liberdade também é mencionado no ar-
Neste sentido, associações são organizações resultan-
tigo 5º, XIII, CF:
tes da reunião legal entre duas ou mais pessoas, com ou
Artigo 5º, XIII, CF. É livre o exercício de qualquer
sem personalidade jurídica, para a realização de um obje-
trabalho, ofício ou profissão, atendidas as qualifi- tivo comum; já cooperativas são uma forma específica de
cações profissionais que a lei estabelecer. associação, pois visam a obtenção de vantagens comuns
em suas atividades econômicas.

19
Ainda, tem-se o artigo 5º, XIX, CF: “O direito à honra distancia-se levemente dos dois
Artigo 5º, XIX, CF. As associações só poderão ser anteriores, podendo referir-se ao juízo positivo que a
compulsoriamente dissolvidas ou ter suas atividades pessoa tem de si (honra subjetiva) e ao juízo positivo que
suspensas por decisão judicial, exigindo-se, no pri- dela fazem os outros (honra objetiva), conferindo-lhe
meiro caso, o trânsito em julgado. respeitabilidade no meio social. O direito à imagem tam-
bém possui duas conotações, podendo ser entendido
O primeiro caso é o de dissolução compulsória, ou em sentido objetivo, com relação à reprodução gráfica
seja, a associação deixará de existir para sempre. Obvia- da pessoa, por meio de fotografias, filmagens, desenhos,
mente, é preciso o trânsito em julgado da decisão judicial ou em sentido subjetivo, significando o conjunto de qua-
que assim determine, pois antes disso sempre há possibi- lidades cultivadas pela pessoa e reconhecidas como suas
lidade de reverter a decisão e permitir que a associação pelo grupo social”8.
continue em funcionamento. Contudo, a decisão judicial
pode suspender atividades até que o trânsito em julgado Inviolabilidade de domicílio e sigilo de correspon-
ocorra, ou seja, no curso de um processo judicial. dência
Em destaque, a legitimidade representativa da associa- Correlatos ao direito à privacidade, aparecem a invio-
ção quanto aos seus filiados, conforme artigo 5º, XXI, CF: labilidade do domicílio e o sigilo das correspondências e
Artigo 5º, XXI, CF. As entidades associativas, quando comunicações.
expressamente autorizadas, têm legitimidade para re-
presentar seus filiados judicial ou extrajudicialmente. Neste sentido, o artigo 5º, XI, CF prevê:
Artigo 5º, XI, CF. A casa é asilo inviolável do indiví-
Trata-se de caso de legitimidade processual extraor- duo, ninguém nela podendo penetrar sem consenti-
mento do morador, salvo em caso de flagrante delito
dinária, pela qual um ente vai a juízo defender interesse
ou desastre, ou para prestar socorro, ou, durante o dia,
de outra(s) pessoa(s) porque a lei assim autoriza.
por determinação judicial.
A liberdade de associação envolve não somente o
direito de criar associações e de fazer parte delas, mas
O domicílio é inviolável, razão pela qual ninguém
também o de não associar-se e o de deixar a associação, pode nele entrar sem o consentimento do morador, a
conforme artigo 5º, XX, CF: não ser EM QUALQUER HORÁRIO no caso de flagrante
Artigo 5º, XX, CF. Ninguém poderá ser compelido a delito (o morador foi flagrado na prática de crime e fugiu
associar-se ou a permanecer associado. para seu domicílio) ou desastre (incêndio, enchente, ter-
remoto...) ou para prestar socorro (morador teve ataque
- Direitos à privacidade e à personalidade do coração, está sufocado, desmaiado...), e SOMENTE
DURANTE O DIA por determinação judicial.
Abrangência Quanto ao sigilo de correspondência e das comunica-
Prevê o artigo 5º, X, CF: ções, prevê o artigo 5º, XII, CF:
Artigo 5º, X, CF. São invioláveis a intimidade, a vida
privada, a honra e a imagem das pessoas, assegu- Artigo 5º, XII, CF. É inviolável o sigilo da correspon-
rado o direito a indenização pelo dano material ou dência e das comunicações telegráficas, de dados e
moral decorrente de sua violação. das comunicações telefônicas, salvo, no último caso,
por ordem judicial, nas hipóteses e na forma que a
O legislador opta por trazer correlacionados no lei estabelecer para fins de investigação criminal ou
mesmo dispositivo legal os direitos à privacidade e à instrução processual penal.
personalidade.
Reforçando a conexão entre a privacidade e a intimi- O sigilo de correspondência e das comunicações está
dade, ao abordar a proteção da vida privada – que, em melhor regulamentado na Lei nº 9.296, de 1996.
resumo, é a privacidade da vida pessoal no âmbito do
domicílio e de círculos de amigos –, Silva7 entende que
#FicaDica
“o segredo da vida privada é condição de expansão da
personalidade”, mas não caracteriza os direitos de per- A doutrina vem aplicando um conceito
sonalidade em si. amplo de casa, considerando casa qualquer
A união da intimidade e da vida privada forma a pri- lugar onde o indivíduo possuía residencia
vacidade, sendo que a primeira se localiza em esfera mais habitual. Exemplo, casa de veraneio, barco,
estrita. É possível ilustrar a vida social como se fosse um restaurante, hotel.
grande círculo no qual há um menor, o da vida privada, e
LEGISLAÇÃO ESPECÍFICA

dentro deste um ainda mais restrito e impenetrável, o da


intimidade. Com efeito, pela “Teoria das Esferas” (ou “Teo- Remédios Constitucionais
ria dos Círculos Concêntricos”), importada do direito ale- Os remédios constitucionais ou remédios jurídicos
mão, quanto mais próxima do indivíduo, maior a prote- são os meios pelos quais os individuos podem provocar a
ção a ser conferida à esfera (as esferas são representadas intervenção das autoridades competentes, com o intuito
pela intimidade, pela vida privada, e pela publicidade). de sanar abuso de poder e irregularidades.
7 SILVA, José Afonso da. Curso de direito constitucional positivo. 8 MOTTA, Sylvio; BARCHET, Gustavo. Curso de direito constitucio-
25. ed. São Paulo: Malheiros, 2006. nal. Rio de Janeiro: Elsevier, 2007.

20
São eles: Pressupostos, legitimidade e competência
O habeas data está regulado na Lei Federal n.º
1. Habbeas Corpus (art. 5º, LXVII da CF) 9.507/97, que estabelece os seguintes pressupostos para
o seu cabimento:
Conceito e cabimento a) a recusa da autoridade administrativa em disponi-
O habeas corpus é o remédio constitucional destina- bilizar ao impetrante o acesso às suas informações
do a assegurar o direito fundamental à liberdade de lo- pessoais e
comoção. É cabível sempre que houver lesão ou ameaça b) o caráter pessoal dos dados e das informações
de lesão à liberdade de locomoção por ato ilegal ou abu- a ser obtidos e/ou corrigidos pelo impetrante. A
sivo, nos termos do art. 5º, LXVIII, da CF: “conceder-se- legitimidade ativa pertence à pessoa física ou ju-
-á habeas corpus sempre que alguém sofrer ou se achar rídica titular do direito personalíssimo ao conhe-
ameaçado de sofrer violência ou coação em sua liberda- cimento e à retificação de seus dados, devendo o
de de locomoção, por ilegalidade ou abuso de poder;”. polo passivo da ação ser integrado pelas entidades
públicas ou privadas detentoras dos registros ou
Legitimidade ativa bancos de dados de caráter público que conte-
Podem impetrar o habeas corpus quaisquer pessoas nham as informações pessoais do impetrante. A
físicas ou jurídicas, inclusive o Ministério Público, em seu competência será definida em função da natureza
favor ou de terceiros. É o que estabelece o art. 654 do da autoridade impetrada.
CPP: “O habeas corpus poderá ser impetrado por qual-
quer pessoa, em seu favor ou de outrem, bem como pelo Gratuidade
Ministério Público.” O habeas data é gratuito, conforme o art. 5º, LXXVII,
da CF: “são gratuitas as ações de habeas corpus e habeas
Partes data, e, na forma da lei, os atos necessários ao exercício
O paciente é a pessoa cuja liberdade de locomoção da cidadania.”.
esteja sendo tutelada no habeas corpus. A autoridade
coatora é a responsável pela prática do ato ilegal ou abu- 3. Mandado de Segurança (art. 5º, LXIX e LXX da CF)
sivo que viola ou põe em risco a liberdade de locomoção
do paciente. A competência para julgamento do habeas Mandado de segurança individual
corpus será definida em função da autoridade coatora ré. O mandado de segurança é o remédio constitucional
previsto no art. 5º, LXIX, da CF, que estabelece: “conceder-
Gratuidade -se-á mandado de segurança para proteger direito líqui-
O habeas corpus é gratuito, conforme o art. 5º, LXXVII, do e certo, não amparado por habeas corpus ou habeas
da CF: “são gratuitas as ações de habeas corpus e habeas data, quando o responsável pela ilegalidade ou abuso de
data, e, na forma da lei, os atos necessários ao exercício poder for autoridade pública ou agente de pessoa jurídi-
da cidadania.”. Além disso, o habeas corpus não precisa ca no exercício de atribuições do Poder Público;”.
ser subscrito por advogado (art. 654 do CPP).
Conceito e cabimento
2. Habeas Data (art. 5º LXXVIII da CF)m O mandado de segurança é a ação constitucional
mandamental destinada à proteção de um direito líquido
Conceito e finalidade e certo ameaçado ou violado por ato ilegal ou abusivo
O habeas data é a ação constitucional prevista no art. de autoridade pública ou de agente de pessoa jurídica
5º, LXXII, da CF, que estabelece: no exercício de atribuições delegadas do Poder Público,
LXXII – conceder-se-á habeas data: quando não caiba habeas corpus ou habeas data. Líquido
a) para assegurar o conhecimento de informações re- e certo é o direito comprovável de plano, cuja demons-
lativas à pessoa do impetrante, constantes de registros tração não depende de dilação probatória. O mandado
ou bancos de dados de entidades governamentais ou de segurança está regulado na Lei Federal n.º 12.016/09,
de caráter público; que, em seu art. 5º, prevê os casos de não cabimento
b) para a retificação de dados, quando não se prefira desta ação:
fazê-lo por processo sigiloso, judicial ou administra- Art. 5º Não se concederá mandado de segurança
tivo; quando se tratar:
Este remédio constitucional destina-se, portanto, à I - de ato do qual caiba recurso administrativo com
proteção de dois direitos fundamentais: efeito suspensivo, independentemente de caução;
(i) o direito de acesso aos dados pessoais constantes II - de decisão judicial da qual caiba recurso com efeito
de registros ou bancos de dados de entidades gover- suspensivo;
III - de decisão judicial transitada em julgado.
LEGISLAÇÃO ESPECÍFICA

namentais ou de caráter público, e


(ii) o direito de retificação destas informações. O ha-
beas data assume, portanto, tríplice finalidade: Partes
a) a de garantir o direito de acesso aos registros de A legitimidade ativa para impetração do mandado de
informações relativas ao impetrante; segurança individual pertence ao titular do direito líquido
b) a de assegurar o direito de retificação destas in- e certo violado ou ameaçado (pessoa física ou jurídica),
formações e denominado impetrante. O polo passivo será integrado
c) a de garantir o direito de complementação das in- pela autoridade coatora, isto é, a que praticou o ato coa-
formações pessoais do impetrante. tor ou deu a ordem para a sua prática. Ato coator é o

21
que ameaça ou viola o direito líquido e certo tutelado gulamentadora torne inviável o exercício dos direitos e
no mandado de segurança. Além da autoridade coatora, liberdades constitucionais e das prerrogativas inerentes
a petição inicial deverá indicar a pessoa jurídica que esta à nacionalidade, à soberania e à cidadania;”. Trata-se de
integra, à qual se acha vinculada ou da qual exerce atri- remédio constitucional destinado ao controle difuso das
buições (art. 6º da Lei n. 12.016/09). omissões inconstitucionais, que se caracterizam pela au-
sência total ou parcial de norma para regular um dispositivo
Mandado de segurança coletivo constitucional de eficácia limitada (que é o que depende de
O mandado de segurança é o remédio constitucional regulamentação para produzir efeitos jurídicos). Assim, se a
previsto no art. 5º, LXX, da CF, que estabelece: falta total ou parcial de norma regulamentadora da Cons-
LXX - o mandado de segurança coletivo pode ser im- tituição inviabilizar o exercício de um direito, liberdade ou
petrado por: prerrogativa constitucional, caberá mandado de injunção. A
a) partido político com representação no Congresso Lei Federal n. 13.300/16 regula o processo e o julgamento
Nacional; dos mandados de injunção individual e coletivo.
b) organização sindical, entidade de classe ou asso-
ciação legalmente constituída e em funcionamento Partes
há pelo menos um ano, em defesa dos interesses de O mandado de injunção individual pode ser proposto
seus membros ou associados; (...). pelo titular dos direitos, das liberdades ou das prerroga-
tivas constitucionais (pessoa física ou jurídica) cujo exer-
Conceito e cabimento cício está sendo frustrado pela omissão legislativa, isto
O mandado de segurança coletivo foi introduzido é, pela ausência de norma regulamentadora (art. 3º da
pela Constituição de 1988. É o remédio constitucional Lei n. 13.300/16). O mandado de injunção coletivo pode
destinado à proteção de direitos coletivos ou individuais ser proposto pelo Ministério Público; por partido políti-
homogêneos, líquidos e certos, não amparados por ha- co com representação no Congresso Nacional; por orga-
beas corpus ou habeas data, se a violação ou ameaça se nização sindical, entidade de classe ou associação legal-
der por ato ilegal ou abusivo de autoridade pública ou mente constituída e em funcionamento há pelo menos
de agente de pessoa jurídica no exercício de atribuições 1 (um) ano ou pela Defensoria Pública (art. 12 da Lei n.
do Poder Público. Coletivo é o direito indivisível compar- 13.300/16). O autor da ação é denominado impetrante.
tilhado por um número determinado de pessoas que, no O polo passivo será integrado pelo Poder, órgão ou au-
seu conjunto, formam grupos, classes ou categorias de toridade competente pela edição da norma regulamen-
pessoas, ligadas entre si ou com a parte contrária por tadora faltante, denominado impetrado (art. 3º da Lei n.
uma relação jurídica básica (art. 21, Parágrafo único, I, 13.300/16). A competência para julgamento da ação será
da Lei n. 12.016/09). Individual homogêneo é o direito definida em função do impetrado.
de origem comum e da atividade ou situação específi-
ca da totalidade ou de parte dos associados ou mem- Efeitos da decisão
bros do impetrante (art. 21, Parágrafo único, II, da Lei n. Conforme o art. 8º da Lei n. 13.300/16, a sentença que
12.016/09). reconhecer a mora legislativa (falta da norma regulamen-
tadora) e julgar procedente a injunção, determinará prazo
Partes razoável para que o impetrado edite a norma faltante e
A legitimidade para impetração do mandado de se- estabelecerá as condições para o exercício dos direitos re-
gurança coletivo está taxativamente prevista art. 5º, LXX, clamados ou para a promoção de ação própria para exer-
da CF, pertencendo a qualquer partido político com re- cê-los, caso não seja suprida a mora legislativa no prazo
presentação no Congresso Nacional e às organizações determinado.
sindicais, entidades de classe ou associações legalmente
constituídas e em funcionamento há pelo menos um ano, 5. Ação Popular (art. 5º LXXIII, da CF)
em defesa dos interesses de seus membros ou associa-
dos. O polo passivo será integrado pela autoridade coa- 1. Conceito
tora, que deu a ordem ou praticou o ato coator, devendo A ação popular é a medida judicial que pode ser pro-
a petição inicial indicar a pessoa jurídica que esta integra, posta por qualquer cidadão buscando a anulação de um
à qual se acha vinculada ou da qual exerce atribuições ato do Poder Público que seja lesivo ao patrimônio pú-
(art. 6º da Lei n. 12.016/09). blico, à moralidade administrativa, ao meio ambiente ou
ao patrimônio histórico e cultural, nos termos do art. 5º,
Prazo para impetração LXXIII, da CF:
O prazo para impetração do mandado de segurança LXXIII – qualquer cidadão é parte legítima para propor
(individual ou coletivo) é de 120 dias, contados da ciên- ação popular que vise a anular ato lesivo ao patrimô-
cia, pelo interessado, do ato impugnado (art. 23 da Lei n.
LEGISLAÇÃO ESPECÍFICA

nio público ou de entidade de que o Estado participe,


12.016/09 e Súmula 632 do STF). à moralidade administrativa, ao meio ambiente e ao
patrimônio histórico e cultural, ficando o autor, salvo
4. Mandado de Injução ( art. 5º, LXXI, da CF) comprovada má-fé, isento de custas judiciais e do ônus
da sucumbência; (...)
Conceito e cabimento A ação popular está regulada na Lei Federal n. 4.717/65.
O mandado de injunção é a ação constitucional previs- Por meio dela, qualquer do povo poderá requerer ao Po-
ta no art. 5º, LXXI, da CF, que estabelece: “conceder-se-á der Judiciário a anulação de atos ilegais e lesivos ao pa-
mandado de injunção sempre que a falta de norma re- trimônio público e social.

22
2. Legitimidade ativa Direito à indenização e direito de resposta
Somente o cidadão pode propor a ação popular. Se- Com vistas à proteção do direito à privacidade, do di-
gundo o art. 1º, §3º, da Lei Federal n. 4.717/65, a prova da reito à personalidade e do direito à imagem, asseguram-
cidadania, para ingresso em juízo, será feita com o título -se dois instrumentos, o direito à indenização e o direito
eleitoral, ou documento que a ele corresponda. Cidadão, de resposta, conforme as necessidades do caso concreto.
deste modo, é o eleitor, ou seja, a pessoal natural que Com efeito, prevê o artigo 5º, V, CF:
esteja no gozo de seus direitos políticos ativos (capaci- Artigo 5º, V, CF. É assegurado o direito de respos-
dade eleitoral ativa; capacidade para votar). Assim, o Mi- ta, proporcional ao agravo, além da indenização por
nistério Público, a Defensoria Pública ou qualquer outra dano material, moral ou à imagem.
pessoa jurídica não podem propor a ação popular, já que
a legitimidade ativa desta ação é exclusiva do cidadão. É, “A manifestação do pensamento é livre e garantida
aliás, o que estabelece a Súmula n.º 365 do STF: “Pessoa em nível constitucional, não aludindo a censura prévia
jurídica não tem legitimidade para propor ação popular.”. em diversões e espetáculos públicos. Os abusos porven-
Caso o autor popular abandone o processo, o Ministério tura ocorridos no exercício indevido da manifestação do
Público poderá promover o andamento da ação (art. 9º pensamento são passíveis de exame e apreciação pelo
da Lei Federal n.º 4.717/65), o que não significa que ele Poder Judiciário com a consequente responsabilidade
possa ingressar com ela. civil e penal de seus autores, decorrentes inclusive de pu-
blicações injuriosas na imprensa, que deve exercer vigi-
3. Características lância e controle da matéria que divulga”9.
A ação popular é uma medida judicial de caráter des- O direito de resposta é o direito que uma pessoa
constitutivo, pois tem por objetivo principal a anulação tem de se defender de críticas públicas no mesmo
de um ato praticado pelo Poder Público que seja lesivo meio em que foram publicadas garantida exatamente
ao patrimônio público, à moralidade administrativa, ao a mesma repercussão. Mesmo quando for garantido
patrimônio histórico e cultural ou ao meio ambiente. O o direito de resposta não é possível reverter plena-
autor popular é, em regra, isento de custas judiciais, ou mente os danos causados pela manifestação ilícita de
pensamento, razão pela qual a pessoa inda fará jus à
seja, não precisa pagar quaisquer custas para poder en-
indenização.
trar com a ação, salvo se comprovada a sua má-fé, nos
A manifestação ilícita do pensamento geralmente
termos do art. 5º, LXXIII, da CF.
causa um dano, ou seja, um prejuízo sofrido pelo agente,
que pode ser individual ou coletivo, moral ou material,
econômico e não econômico.
#FicaDica Dano material é aquele que atinge o patrimônio
São gratuitas as ações de Habeas Corpus (material ou imaterial) da vítima, podendo ser mensura-
e de Habeas Data. o Habeas Corpus não do financeiramente e indenizado.
necessita de advogado para ser impetrado, “Dano moral direto consiste na lesão a um interesse
já o Habeas Data necessita. que visa a satisfação ou gozo de um bem jurídico extra-
patrimonial contido nos direitos da personalidade (como
a vida, a integridade corporal, a liberdade, a honra, o
decoro, a intimidade, os sentimentos afetivos, a própria
Personalidade jurídica e gratuidade de registro imagem) ou nos atributos da pessoa (como o nome, a
Quando se fala em reconhecimento como pessoa pe- capacidade, o estado de família)”10.
rante a lei desdobra-se uma esfera bastante específica Já o dano à imagem é delimitado no artigo 20 do
dos direitos de personalidade, consistente na persona- Código Civil:
lidade jurídica. Basicamente, consiste no direito de ser Artigo 20, CC. Salvo se autorizadas, ou se necessá-
reconhecido como pessoa perante a lei. rias à administração da justiça ou à manutenção da
Para ser visto como pessoa perante a lei mostra-se ordem pública, a divulgação de escritos, a transmis-
necessário o registro. Por ser instrumento que serve são da palavra, ou a publicação, a exposição ou a
como pressuposto ao exercício de direitos fundamentais, utilização da imagem de uma pessoa poderão ser
assegura-se a sua gratuidade aos que não tiverem condi- proibidas, a seu requerimento e sem prejuízo da in-
ção de com ele arcar. denização que couber, se lhe atingirem a honra, a
Aborda o artigo 5º, LXXVI, CF: boa fama ou a respeitabilidade, ou se se destinarem
Artigo 5º, LXXVI, CF. São gratuitos para os reconhe- a fins comerciais.
cidamente pobres, na forma da lei: a) o registro civil
de nascimento; b) a certidão de óbito. - Direito à segurança
O caput do artigo 5º da Constituição assegura a pro-
LEGISLAÇÃO ESPECÍFICA

O reconhecimento do marco inicial e do marco final teção do direito à segurança. Na qualidade de direito in-
da personalidade jurídica pelo registro é direito indivi- dividual liga-se à segurança do indivíduo como um todo,
dual, não dependendo de condições financeiras. Eviden- desde sua integridade física e mental, até a própria segu-
te, seria absurdo cobrar de uma pessoa sem condições a rança jurídica.
elaboração de documentos para que ela seja reconheci-
da como viva ou morta, o que apenas incentivaria a indi- 9 BONAVIDES, Paulo. Curso de direito constitucional. 26. ed. São
Paulo: Malheiros, 2011.
gência dos menos favorecidos.
10 ZANNONI, Eduardo. El daño en la responsabilidad civil. Buenos
Aires: Astrea, 1982.

23
No sentido aqui estudado, o direito à segurança pes- Com efeito, a proteção da propriedade privada está
soal é o direito de viver sem medo, protegido pela soli- limitada ao atendimento de sua função social, sendo
dariedade e liberto de agressões, logo, é uma maneira de este o requisito que a correlaciona com a proteção da
garantir o direito à vida. dignidade da pessoa humana. A propriedade de bens e
Nesta linha, para Silva11, “efetivamente, esse conjun- valores em geral é um direito assegurado na Constituição
to de direitos aparelha situações, proibições, limitações Federal e, como todos os outros, se encontra limitado
e procedimentos destinados a assegurar o exercício e o pelos demais princípios conforme melhor se atenda à
gozo de algum direito individual fundamental (intimida- dignidade do ser humano.
de, liberdade pessoal ou a incolumidade física ou moral)”. A Constituição Federal delimita o que se entende por
Especificamente no que tange à segurança jurídica, função social:
tem-se o disposto no artigo 5º, XXXVI, CF:
Artigo 5º, XXXVI, CF. A lei não prejudicará o direito ad- Art. 182, caput, CF. A política de desenvolvimento
quirido, o ato jurídico perfeito e a coisa julgada. urbano, executada pelo Poder Público municipal,
conforme diretrizes gerais fixadas em lei, tem por
Pelo inciso restam estabelecidos limites à retroativi- objetivo ordenar o pleno desenvolvimento das fun-
dade da lei. ções sociais da cidade e garantir o bem-estar de seus
Define o artigo 6º da Lei de Introdução às Normas do habitantes.
Direito Brasileiro:
Artigo 6º, LINDB. A Lei em vigor terá efeito imediato Artigo 182, § 1º, CF. O plano diretor, aprovado pela
e geral, respeitados o ato jurídico perfeito, o direito Câmara Municipal, obrigatório para cidades com mais
adquirido e a coisa julgada. de vinte mil habitantes, é o instrumento básico da
§ 1º Reputa-se ato jurídico perfeito o já consumado política de desenvolvimento e de expansão urbana.
segundo a lei vigente ao tempo em que se efetuou.
§ 2º Consideram-se adquiridos assim os direitos que Artigo 182, § 2º, CF. A propriedade urbana cumpre
o seu titular, ou alguém por ele, possa exercer, como sua função social quando atende às exigências funda-
aqueles cujo começo do exercício tenha termo pré- mentais de ordenação da cidade expressas no plano
-fixo, ou condição pré-estabelecida inalterável, a ar-
diretor13.
bítrio de outrem.
§ 3º Chama-se coisa julgada ou caso julgado a deci-
Artigo 186, CF. A função social é cumprida quando a
são judicial de que já não caiba recurso.
propriedade rural atende, simultaneamente, segundo
critérios e graus de exigência estabelecidos em lei, aos
- Direito à propriedade
seguintes requisitos:
O caput do artigo 5º da Constituição assegura a pro-
I - aproveitamento racional e adequado;
teção do direito à propriedade, tanto material quanto
II - utilização adequada dos recursos naturais dispo-
intelectual, delimitada em alguns incisos que o seguem.
níveis e preservação do meio ambiente;
Função social da propriedade material III - observância das disposições que regulam as rela-
O artigo 5º, XXII, CF estabelece: ções de trabalho;
Artigo 5º, XXII, CF. É garantido o direito de proprie- IV - exploração que favoreça o bem-estar dos pro-
dade. prietários e dos trabalhadores.

A seguir, no inciso XXIII do artigo 5º, CF estabelece o Desapropriação


principal fator limitador deste direito: No caso de desrespeito à função social da proprieda-
Artigo 5º, XXIII, CF. A propriedade atenderá a sua de cabe até mesmo desapropriação do bem, de modo
função social. que pode-se depreender do texto constitucional duas
possibilidades de desapropriação: por desrespeito à fun-
A propriedade, segundo Silva12, “[...] não pode mais ção social e por necessidade ou utilidade pública.
ser considerada como um direito individual nem como A Constituição Federal prevê a possibilidade de desa-
instituição do direito privado. [...] embora prevista entre propriação por desatendimento à função social:
os direitos individuais, ela não mais poderá ser conside- Artigo 182, § 4º, CF. É facultado ao Poder Público
rada puro direito individual, relativizando-se seu con- municipal, mediante lei específica para área incluída
ceito e significado, especialmente porque os princípios no plano diretor, exigir, nos termos da lei federal, do
da ordem econômica são preordenados à vista da rea- proprietário do solo urbano não edificado, subuti-
lização de seu fim: assegurar a todos existência digna, lizado ou não utilizado, que promova seu adequado
LEGISLAÇÃO ESPECÍFICA

conforme os ditames da justiça social. Se é assim, então aproveitamento, sob pena, sucessivamente, de:
a propriedade privada, que, ademais, tem que atender a I - parcelamento ou edificação compulsórios;
sua função social, fica vinculada à consecução daquele II - imposto sobre a propriedade predial e territorial
princípio”. urbana progressivo no tempo;

11 SILVA, José Afonso da. Curso de direito constitucional positivo... 13 Instrumento básico de um processo de planejamento municipal
Op. Cit., p. 437. para a implantação da política de desenvolvimento urbano, nor-
12 SILVA, José Afonso da. Curso de direito constitucional positivo. teando a ação dos agentes públicos e privados (Lei n. 10.257/2001
25. ed. São Paulo: Malheiros, 2006. - Estatuto da cidade).

24
III - desapropriação com pagamento mediante títulos e) a criação e melhoramento de centros de popula-
da dívida pública de emissão previamente aprovada ção, seu abastecimento regular de meios de subsis-
pelo Senado Federal, com prazo de resgate de até dez tência;
anos, em parcelas anuais, iguais e sucessivas, assegu- f) o aproveitamento industrial das minas e das jazi-
rados o valor real da indenização e os juros legais14. das minerais, das águas e da energia hidráulica;
g) a assistência pública, as obras de higiene e deco-
Artigo 184, CF. Compete à União desapropriar por ração, casas de saúde, clínicas, estações de clima e
interesse social, para fins de reforma agrária, o imóvel fontes medicinais;
rural que não esteja cumprindo sua função social, h) a exploração ou a conservação dos serviços pú-
mediante prévia e justa indenização em títulos da blicos;
dívida agrária, com cláusula de preservação do valor i) a abertura, conservação e melhoramento de vias
real, resgatáveis no prazo de até vinte anos, a partir ou logradouros públicos; a execução de planos de
do segundo ano de sua emissão, e cuja utilização será urbani
definida em lei15. zação; o parcelamento do solo, com ou sem edifica-
ção, para sua melhor utilização econômica, higiênica
Artigo 184, § 1º, CF. As benfeitorias úteis e necessárias ou estética; a construção ou ampliação de distritos
serão indenizadas em dinheiro. industriais;
j) o funcionamento dos meios de transporte coletivo;
No que tange à desapropriação por necessidade ou k) a preservação e conservação dos monumentos
utilidade pública, prevê o artigo 5º, XXIV, CF: históricos e artísticos, isolados ou integrados em
Artigo 5º, XXIV, CF. A lei estabelecerá o procedimen- conjuntos urbanos ou rurais, bem como as medidas
to para desapropriação por necessidade ou utilidade necessárias a manter-lhes e realçar-lhes os aspectos
pública, ou por interesse social, mediante justa e pré- mais valiosos ou característicos e, ainda, a proteção
via indenização em dinheiro, ressal3vados os casos de paisagens e locais particularmente dotados pela
previstos nesta Constituição. natureza;
l) a preservação e a conservação adequada de arqui-
Ainda, prevê o artigo 182, § 3º, CF: vos, documentos e outros bens moveis de valor his-
Artigo 182, §3º, CF. As desapropriações de imóveis tórico ou artístico;
urbanos serão feitas com prévia e justa indenização m) a construção de edifícios públicos, monumentos
em dinheiro. comemorativos e cemitérios;
n) a criação de estádios, aeródromos ou campos de
Tem-se, ainda o artigo 184, §§ 2º e 3º, CF: pouso para aeronaves;
Artigo 184, §2º, CF. O decreto que declarar o imóvel o) a reedição ou divulgação de obra ou invento de
como de interesse social, para fins de reforma agrária, natureza científica, artística ou literária;
autoriza a União a propor a ação de desapropriação. p) os demais casos previstos por leis especiais.

Artigo 184, §3º, CF. Cabe à lei complementar esta- Um grande problema que faz com que processos que
belecer procedimento contraditório especial, de rito tenham a desapropriação por objeto se estendam é a
sumário, para o processo judicial de desapropriação. indevida valorização do imóvel pelo Poder Público, que
geralmente pretende pagar valor muito abaixo do devi-
A desapropriação por utilidade ou necessidade pú- do, necessitando o Judiciário intervir em prol da correta
blica deve se dar mediante prévia e justa indenização em avaliação.
dinheiro. O Decreto-lei nº 3.365/1941 a disciplina, delimi- Outra questão reside na chamada tredestinação, pela
tando o procedimento e conceituando utilidade pública, qual há a destinação de um bem expropriado (desapro-
em seu artigo 5º: priação) a finalidade diversa da que se planejou inicial-
Artigo 5º, Decreto-lei n. 3.365/1941. Consideram-se mente. A tredestinação pode ser lícita ou ilícita. Será ilí-
casos de utilidade pública: cita quando resultante de desvio do propósito original;
a) a segurança nacional; e será lícita quando a Administração Pública dê ao bem
b) a defesa do Estado; finalidade diversa, porém preservando a razão do inte-
c) o socorro público em caso de calamidade; resse público.
d) a salubridade pública;
14 Nota-se que antes de se promover a desapropriação de imó- Política agrária e reforma agrária
vel urbano por desatendimento à função social é necessário tomar Enquanto desdobramento do direito à propriedade
duas providências, sucessivas: primeiro, o parcelamento ou edifica- imóvel e da função social desta propriedade, tem-se ain-
LEGISLAÇÃO ESPECÍFICA

ção compulsórios; depois, o estabelecimento de imposto sobre a


da o artigo 5º, XXVI, CF:
propriedade predial e territorial urbana progressivo no tempo. Se
ambas medidas restarem ineficazes, parte-se para a desapropriação
por desatendimento à função social. Artigo 5º, XXVI, CF. A pequena propriedade rural,
15 A desapropriação em decorrência do desatendimento da função assim definida em lei, desde que trabalhada pela fa-
social é indenizada, mas não da mesma maneira que a desapro- mília, não será objeto de penhora para pagamento de
priação por necessidade ou utilidade pública, já que na primeira há débitos decorrentes de sua atividade produtiva, dis-
violação do ordenamento constitucional pelo proprietário, mas na pondo a lei sobre os meios de financiar o seu desen-
segunda não. Por isso, indeniza-se em títulos da dívida agrária, que
volvimento.
na prática não são tão valorizados quanto o dinheiro.

25
Assim, se uma pessoa é mais humilde e tem uma pe- Os que forem favorecidos pela reforma agrária (ho-
quena propriedade será assegurado que permaneça com mens, mulheres, ambos, qualquer estado civil) não po-
ela e a torne mais produtiva. derão negociar seus títulos pelo prazo de 10 anos (artigo
A preservação da pequena propriedade em detri- 189, CF).
mento dos grandes latifúndios improdutivos é uma das Consta, ainda, que “a lei regulará e limitará a aquisi-
diretrizes-guias da regulamentação da política agrária ção ou o arrendamento de propriedade rural por pessoa
brasileira, que tem como principal escopo a realização da física ou jurídica estrangeira e estabelecerá os casos que
reforma agrária. dependerão de autorização do Congresso Nacional” (ar-
Parte da questão financeira atinente à reforma agrária tigo 190, CF).
se encontra prevista no artigo 184, §§ 4º e 5º, CF:
Artigo 184, §4º, CF. O orçamento fixará anualmente o Usucapião
volume total de títulos da dívida agrária, assim como Usucapião é o modo originário de aquisição da pro-
o montante de recursos para atender ao programa priedade que decorre da posse prolongada por um lon-
de reforma agrária no exercício. go tempo, preenchidos outros requisitos legais. Em ou-
Artigo 184, §5º, CF. São isentas de impostos federais, tras palavras, usucapião é uma situação em que alguém
estaduais e municipais as operações de transferência tem a posse de um bem por um tempo longo, sem ser
de imóveis desapropriados para fins de reforma agrária. incomodado, a ponto de se tornar proprietário.
A Constituição regulamenta o acesso à propriedade
Como a finalidade da reforma agrária é transformar mediante posse prolongada no tempo – usucapião – em
terras improdutivas e grandes propriedades em atinen- casos específicos, denominados usucapião especial urba-
tes à função social, alguns imóveis rurais não podem ser na e usucapião especial rural.
abrangidos pela reforma agrária: O artigo 183 da Constituição regulamenta a usuca-
Art. 185, CF. São insuscetíveis de desapropriação para pião especial urbana:
fins de reforma agrária: Art. 183, CF. Aquele que possuir como sua área urbana
I - a pequena e média propriedade rural, assim defi- de até duzentos e cinquenta metros quadrados, por
nida em lei, desde que seu proprietário não possua cinco anos, ininterruptamente e sem oposição, utili-
outra; zando-a para sua moradia ou de sua família, adqui-
II - a propriedade produtiva. rir-lhe-á o domínio, desde que não seja proprietário de
Parágrafo único. A lei garantirá tratamento especial à outro imóvel urbano ou rural.
propriedade produtiva e fixará normas para o cum- § 1º O título de domínio e a concessão de uso serão
primento dos requisitos relativos a sua função social. conferidos ao homem ou à mulher, ou a ambos, in-
dependentemente do estado civil.
Sobre as diretrizes da política agrícola, prevê o artigo 187: § 2º Esse direito não será reconhecido ao mesmo
Art. 187, CF. A política agrícola será planejada e exe- possuidor mais de uma vez.
cutada na forma da lei, com a participação efetiva do § 3º Os imóveis públicos não serão adquiridos por
setor de produção, envolvendo produtores e trabalha- usucapião.
dores rurais, bem como dos setores de comercializa-
ção, de armazenamento e de transportes, levando em Além dos requisitos gerais (animus e posse que seja
conta, especialmente: pública, pacífica, ininterrupta e contínua), são exigidos os
I - os instrumentos creditícios e fiscais; seguintes requisitos específicos:
II - os preços compatíveis com os custos de produção e a) Área urbana – há controvérsia. Pela teoria da loca-
a garantia de comercialização; lização, área urbana é a que está dentro do perí-
III - o incentivo à pesquisa e à tecnologia; metro urbano. Pela teoria da destinação, mais im-
IV - a assistência técnica e extensão rural; portante que a localização é a sua utilização. Ex.:
V - o seguro agrícola; se tem fins agrícolas/pecuários e estiver dentro
VI - o cooperativismo; do perímetro urbana, o imóvel é rural. Para fins de
VII - a eletrificação rural e irrigação; usucapião a maioria diz que prevalece a teoria da
VIII - a habitação para o trabalhador rural. localização.
§ 1º Incluem-se no planejamento agrícola as ativi- b) Imóveis até 250 m² – Pode dentro de uma posse
dades agroindustriais, agropecuárias, pesqueiras e maior isolar área de 250m² e ingressar com a ação?
florestais. A jurisprudência é pacífica que a posse desde o iní-
§ 2º Serão compatibilizadas as ações de política agrí- cio deve ficar restrita a 250m². Predomina também
cola e de reforma agrária. que o terreno deve ter 250m², não a área construí-
da (a área de um sobrado, por exemplo, pode ser
As terras devolutas e públicas serão destinadas con-
LEGISLAÇÃO ESPECÍFICA

maior que a de um terreno).


forme a política agrícola e o plano nacional de reforma c) 5 anos – houve controvérsia porque a Constitui-
agrária (artigo 188, caput, CF). Neste sentido, “a alienação ção Federal de 1988 que criou esta modalidade.
ou a concessão, a qualquer título, de terras públicas com E se antes de 05 de outubro de 1988 uma pessoa
área superior a dois mil e quinhentos hectares a pessoa tivesse há 4 anos dentro do limite da usucapião
física ou jurídica, ainda que por interposta pessoa, depen- urbana? Predominou que só corria o prazo a partir
derá de prévia aprovação do Congresso Nacional”, salvo da criação do instituto, não só porque antes não
no caso de alienações ou concessões de terras públicas existia e o prazo não podia correr, como também
para fins de reforma agrária (artigo 188, §§ 1º e 2º, CF). não se poderia prejudicar o proprietário.

26
d) Moradia sua ou de sua família – não basta ter posse, Artigo 5º, XXX, CF. É garantido o direito de herança;
é preciso que a pessoa more, sozinha ou com sua Artigo 5º, XXXI, CF.A sucessão de bens de estrangeiros
família, ao longo de todo o prazo (não só no início situados no País será regulada pela lei brasileira em be-
ou no final). Logo, não cabe acessio temporis por nefício do cônjuge ou dos filhos brasileiros, sempre que
cessão da posse. não lhes seja mais favorável a lei pessoal do de cujus.
e) Nenhum outro imóvel, nem urbano, nem rural, no
Brasil. O usucapiente não prova isso, apenas alega. O direito à herança envolve o direito de receber – seja
Se alguém não quiser a usucapião, prova o contrá- devido a uma previsão legal, seja por testamento – bens
rio. Este requisito é verificado no momento em que de uma pessoa que faleceu. Assim, o patrimônio passa
completa 5 anos. para outra pessoa, conforme a vontade do falecido e/ou
a lei determine. A Constituição estabelece uma disciplina
Em relação à previsão da usucapião especial rural, des- específica para bens de estrangeiros situados no Brasil,
taca-se o artigo 191, CF: assegurando que eles sejam repassados ao cônjuge e fi-
Art. 191, CF. Aquele que, não sendo proprietário de lhos brasileiros nos termos da lei mais benéfica (do Brasil
imóvel rural ou urbano, possua como seu, por cinco ou do país estrangeiro).
anos ininterruptos, sem oposição, área de terra, em
zona rural, não superior a cinquenta hectares, tornan- Direito do consumidor
do-a produtiva por seu trabalho ou de sua família, Nos termos do artigo 5º, XXXII, CF:
tendo nela sua moradia, adquirir-lhe-á a propriedade. Artigo 5º, XXXII, CF. O Estado promoverá, na forma da
Parágrafo único. Os imóveis públicos não serão adqui- lei, a defesa do consumidor.
ridos por usucapião.
O direito do consumidor liga-se ao direito à proprie-
Além dos requisitos gerais (animus e posse que seja dade a partir do momento em que garante à pessoa que
pública, pacífica, ininterrupta e contínua), são exigidos os irá adquirir bens e serviços que estes sejam entregues e
seguintes requisitos específicos: prestados da forma adequada, impedindo que o forne-
a) Imóvel rural cedor se enriqueça ilicitamente, se aproveite de maneira
b) 50 hectares, no máximo – há também legislação que indevida da posição menos favorável e de vulnerabilida-
de técnica do consumidor.
estabelece um limite mínimo, o módulo rural (Esta-
O Direito do Consumidor pode ser considerado um
tuto da Terra). É possível usucapir áreas menores que
ramo recente do Direito. No Brasil, a legislação que o re-
o módulo rural? Tem prevalecido o entendimento
gulamentou foi promulgada nos anos 90, qual seja a Lei
de que pode, mas é assunto muito controverso.
nº 8.078, de 11 de setembro de 1990, conforme deter-
c) 5 anos – pode ser considerado o prazo antes 05 de
minado pela Constituição Federal de 1988, que também
outubro de 1988 (Constituição Federal)? Depende.
estabeleceu no artigo 48 do Ato das Disposições Consti-
Se a área é de até 25 hectares sim, pois já havia tal tucionais Transitórias:
possibilidade antes da CF/88. Se área for maior (en- Artigo 48, ADCT. O Congresso Nacional, dentro de
tre 25 ha e 50 ha) não. cento e vinte dias da promulgação da Constituição,
d) Moradia sua ou de sua família – a pessoa deve mo- elaborará código de defesa do consumidor.
rar na área rural.
e) Nenhum outro imóvel. A elaboração do Código de Defesa do Consumidor
f) O usucapiente, com seu trabalho, deve ter tornado foi um grande passo para a proteção da pessoa nas re-
a área produtiva. Por isso, é chamado de usucapião lações de consumo que estabeleça, respeitando-se a
“pro labore”. Dependerá do caso concreto. condição de hipossuficiente técnico daquele que adquire
um bem ou faz uso de determinado serviço, enquanto
Uso temporário consumidor.
No mais, estabelece-se uma terceira limitação ao di-
reito de propriedade que não possui o caráter definitivo Propriedade intelectual
da desapropriação, mas é temporária, conforme artigo 5º, Além da propriedade material, o constituinte protege
XXV, CF: também a propriedade intelectual, notadamente no arti-
Artigo 5º, XXV, CF. No caso de iminente perigo públi- go 5º, XXVII, XXVIII e XXIX, CF:
co, a autoridade competente poderá usar de proprie- Artigo 5º, XXVII, CF. Aos autores pertence o direito ex-
dade particular, assegurada ao proprietário indeniza- clusivo de utilização, publicação ou reprodução de
ção ulterior, se houver dano. suas obras, transmissível aos herdeiros pelo tempo
que a lei fixar;
Se uma pessoa tem uma propriedade, numa situação
LEGISLAÇÃO ESPECÍFICA

de perigo, o poder público pode se utilizar dela (ex: mon- Artigo 5º, XXVIII, CF. São assegurados, nos termos da lei:
tar uma base para capturar um fugitivo), pois o interesse a) a proteção às participações individuais em obras
da coletividade é maior que o do indivíduo proprietário. coletivas e à reprodução da imagem e voz huma-
nas, inclusive nas atividades desportivas;
Direito sucessório b) o direito de fiscalização do aproveitamento eco-
O direito sucessório aparece como uma faceta do di- nômico das obras que criarem ou de que participa-
reito à propriedade, encontrando disciplina constitucio- rem aos criadores, aos intérpretes e às respectivas re-
nal no artigo 5º, XXX e XXXI, CF: presentações sindicais e associativas;

27
Artigo 5º, XXIX, CF. A lei assegurará aos autores de in- onda anterior, restando parcialmente implementada (visto
ventos industriais privilégio temporário para sua utili- que até hoje enfrentam-se obstáculos ao pleno atendimen-
zação, bem como proteção às criações industriais, à pro- to em todas as ondas).
priedade das marcas, aos nomes de empresas e a outros Primeiro, Cappelletti e Garth17 entendem que surgiu
signos distintivos, tendo em vista o interesse social e o uma onda de concessão de assistência judiciária aos po-
desenvolvimento tecnológico e econômico do País. bres, partindo-se da prestação sem interesse de remu-
neração por parte dos advogados e, ao final, levando à
Assim, a propriedade possui uma vertente intelectual criação de um aparato estrutural para a prestação da as-
que deve ser respeitada, tanto sob o aspecto moral quanto sistência pelo Estado.
sob o patrimonial. No âmbito infraconstitucional brasileiro, Em segundo lugar, no entender de Cappelletti e
a Lei nº 9.610, de 19 de fevereiro de 1998, regulamenta os Garth18, veio a onda de superação do problema na re-
direitos autorais, isto é, “os direitos de autor e os que lhes presentação dos interesses difusos, saindo da concepção
são conexos”. tradicional de processo como algo restrito a apenas duas
O artigo 7° do referido diploma considera como obras partes individualizadas e ocasionando o surgimento de
intelectuais que merecem a proteção do direito do autor novas instituições, como o Ministério Público.
Finalmente, Cappelletti e Garth19 apontam uma ter-
os textos de obras de natureza literária, artística ou científi-
ceira onda consistente no surgimento de uma concepção
ca; as conferências, sermões e obras semelhantes; as obras
mais ampla de acesso à justiça, considerando o conjunto
cinematográficas e televisivas; as composições musicais;
de instituições, mecanismos, pessoas e procedimentos
fotografias; ilustrações; programas de computador; coletâ-
utilizados: “[...] esse enfoque encoraja a exploração de
neas e enciclopédias; entre outras. uma ampla variedade de reformas, incluindo alterações
Os direitos morais do autor, que são imprescritíveis, ina- nas formas de procedimento, mudanças na estrutura dos
lienáveis e irrenunciáveis, envolvem, basicamente, o direito tribunais ou a criação de novos tribunais, o uso de pes-
de reivindicar a autoria da obra, ter seu nome divulgado na soas leigas ou paraprofissionais, tanto como juízes quan-
utilização desta, assegurar a integridade desta ou modifi- to como defensores, modificações no direito substantivo
cá-la e retirá-la de circulação se esta passar a afrontar sua destinadas a evitar litígios ou facilitar sua solução e a uti-
honra ou imagem. lização de mecanismos privados ou informais de solução
Já os direitos patrimoniais do autor, nos termos dos ar- dos litígios. Esse enfoque, em suma, não receia inovações
tigos 41 a 44 da Lei nº 9.610/98, prescrevem em 70 anos radicais e compreensivas, que vão muito além da esfera
contados do primeiro ano seguinte à sua morte ou do de representação judicial”.
falecimento do último coautor, ou contados do primeiro Assim, dentro da noção de acesso à justiça, diversos
ano seguinte à divulgação da obra se esta for de nature- aspectos podem ser destacados: de um lado, deve criar-
za audiovisual ou fotográfica. Estes, por sua vez, abrangem, -se o Poder Judiciário e se disponibilizar meios para que
basicamente, o direito de dispor sobre a reprodução, edi- todas as pessoas possam buscá-lo; de outro lado, não
ção, adaptação, tradução, utilização, inclusão em bases de basta garantir meios de acesso se estes forem insuficien-
dados ou qualquer outra modalidade de utilização; sendo tes, já que para que exista o verdadeiro acesso à justiça é
que estas modalidades de utilização podem se dar a título necessário que se aplique o direito material de maneira
oneroso ou gratuito. justa e célere.
“Os direitos autorais, também conhecidos como co- Relacionando-se à primeira onda de acesso à justiça,
pyright (direito de cópia), são considerados bens móveis, prevê a Constituição em seu artigo 5º, XXXV:
podendo ser alienados, doados, cedidos ou locados. Ressal- Artigo 5º, XXXV, CF. A lei não excluirá da apreciação
te-se que a permissão a terceiros de utilização de criações do Poder Judiciário lesão ou ameaça a direito.
artísticas é direito do autor. [...] A proteção constitucional
O princípio da inafastabilidade da jurisdição é o prin-
abrange o plágio e a contrafação. Enquanto que o primei-
cípio de Direito Processual Público subjetivo, também
ro caracteriza-se pela difusão de obra criada ou produzida
cunhado como Princípio da Ação, em que a Constituição
por terceiros, como se fosse própria, a segunda configura a
garante a necessária tutela estatal aos conflitos ocorren-
reprodução de obra alheia sem a necessária permissão do
tes na vida em sociedade. Sempre que uma controvérsia
autor”16. for levada ao Poder Judiciário, preenchidos os requisitos
de admissibilidade, ela será resolvida, independente-
- Direitos de acesso à justiça mente de haver ou não previsão específica a respeito na
A formação de um conceito sistemático de acesso à jus- legislação.
tiça se dá com a teoria de Cappelletti e Garth, que aponta- Também se liga à primeira onda de acesso à justiça,
ram três ondas de acesso, isto é, três posicionamentos bási- no que tange à abertura do Judiciário mesmo aos menos
cos para a realização efetiva de tal acesso. Tais ondas foram favorecidos economicamente, o artigo 5º, LXXIV, CF:
LEGISLAÇÃO ESPECÍFICA

percebidas paulatinamente com a evolução do Direito mo- Artigo 5º, LXXIV, CF. O Estado prestará assistência
derno conforme implementadas as bases da onda anterior, jurídica integral e gratuita aos que comprovarem
quer dizer, ficou evidente aos autores a emergência de uma insuficiência de recursos.
nova onda quando superada a afirmação das premissas da
17 CAPPELLETTI, Mauro; GARTH, Bryant. Acesso à Justiça. Tradução
16 MORAES, Alexandre de. Direitos humanos fundamentais: teoria Ellen Grace Northfleet. Porto Alegre: Sérgio Antônio Fabris Editor,
geral, comentários aos artigos 1º a 5º da Constituição da Repúbli- 1998, p. 31-32.
ca Federativa do Brasil, doutrina e jurisprudência. São Paulo: Atlas, 18 Ibid., p. 49-52
1997. 19 Ibid., p. 67-73

28
O constituinte, ciente de que não basta garantir o Sigilo das votações envolve a realização de votações
acesso ao Poder Judiciário, sendo também necessária a secretas, preservando a liberdade de voto dos que com-
efetividade processual, incluiu pela Emenda Constitucio- põem o conselho que irá julgar o ato praticado.
nal nº 45/2004 o inciso LXXVIII ao artigo 5º da Consti- A decisão tomada pelo conselho é soberana. Contu-
tuição: do, a soberania dos veredictos veda a alteração das deci-
Artigo 5º, LXXVIII, CF. A todos, no âmbito judicial e sões dos jurados, não a recorribilidade dos julgamentos
administrativo, são assegurados a razoável duração do Tribunal do Júri para que seja procedido novo julga-
do processo e os meios que garantam a celeridade mento uma vez cassada a decisão recorrida, haja vista
de sua tramitação. preservar o ordenamento jurídico pelo princípio do du-
plo grau de jurisdição.
Com o tempo se percebeu que não bastava garantir Por fim, a competência para julgamento é dos crimes
o acesso à justiça se este não fosse célere e eficaz. Não dolosos (em que há intenção ou ao menos se assume o
significa que se deve acelerar o processo em detrimento risco de produção do resultado) contra a vida, que são:
de direitos e garantias assegurados em lei, mas sim que homicídio, aborto, induzimento, instigação ou auxílio a
é preciso proporcionar um trâmite que dure nem mais e suicídio e infanticídio. Sua competência não é absoluta e
nem menos que o necessário para a efetiva realização da é mitigada, por vezes, pela própria Constituição (artigos
justiça no caso concreto. 29, X /102, I, b) e c) / 105, I, a) / 108, I).
- Direitos constitucionais-penais Anterioridade e irretroatividade da lei
O artigo 5º, XXXIX, CF preconiza:
Juiz natural e vedação ao juízo ou tribunal de ex- Artigo5º, XXXIX, CF. Não há crime sem lei anterior
ceção que o defina, nem pena sem prévia cominação legal.
Quando o artigo 5º, LIII, CF menciona: É a consagração da regra do nullum crimen nulla
Artigo 5º, LIII, CF. Ninguém será processado nem sen- poena sine praevia lege. Simultaneamente, se assegura o
tenciado senão pela autoridade competente”, con-
princípio da legalidade (ou reserva legal), na medida em
solida o princípio do juiz natural que assegura a toda
que não há crime sem lei que o defina, nem pena sem
pessoa o direito de conhecer previamente daquele que
prévia cominação legal, e o princípio da anterioridade,
a julgará no processo em que seja parte, revestindo tal
posto que não há crime sem lei anterior que o defina.
juízo em jurisdição competente para a matéria especí-
fica do caso antes mesmo do fato ocorrer.
Ainda no que tange ao princípio da anterioridade,
tem-se o artigo 5º, XL, CF:
Por sua vez, um desdobramento deste princípio en- Artigo 5º, XL, CF. A lei penal não retroagirá, salvo
contra-se no artigo 5º, XXXVII, CF: para beneficiar o réu.
Artigo 5º, XXXVII, CF. Não haverá juízo ou tribunal
de exceção. O dispositivo consolida outra faceta do princípio da
anterioridade: se, por um lado, é necessário que a lei te-
Juízo ou Tribunal de Exceção é aquele especialmente nha definido um fato como crime e dado certo tratamen-
criado para uma situação pretérita, bem como não reco- to penal a este fato (ex.: pena de detenção ou reclusão,
nhecido como legítimo pela Constituição do país. tempo de pena, etc.) antes que ele ocorra; por outro lado,
se vier uma lei posterior ao fato que o exclua do rol de
Tribunal do júri crimes ou que confira tratamento mais benéfico (dimi-
A respeito da competência do Tribunal do júri, prevê nuindo a pena ou alterando o regime de cumprimento,
o artigo 5º, XXXVIII, CF: notadamente), ela será aplicada. Restam consagrados
Artigo 5º, XXXVIII. É reconhecida a instituição do júri, tanto o princípio da irretroatividade da lei penal in pejus
com a organização que lhe der a lei, assegurados: quanto o da retroatividade da lei penal mais benéfica.
a) a plenitude de defesa;
b) o sigilo das votações; Menções específicas a crimes
c) a soberania dos veredictos; O artigo 5º, XLI, CF estabelece:
d) a competência para o julgamento dos crimes do- Artigo 5º, XLI, CF. A lei punirá qualquer discriminação
losos contra a vida. atentatória dos direitos e liberdades fundamentais.

O Tribunal do Júri é formado por pessoas do povo, Sendo assim confere fórmula genérica que remete
que julgam os seus pares. Entende-se ser direito fun- ao princípio da igualdade numa concepção ampla, razão
LEGISLAÇÃO ESPECÍFICA

damental o de ser julgado por seus iguais, membros da pela qual práticas discriminatórias não podem ser acei-
sociedade e não magistrados, no caso de determinados tas. No entanto, o constituinte entendeu por bem prever
crimes que por sua natureza possuem fortes fatores de tratamento específico a certas práticas criminosas.
influência emocional. Neste sentido, prevê o artigo 5º, XLII, CF:
Plenitude da defesa envolve tanto a autodefesa quan- Artigo 5º, XLII, CF. A prática do racismo constitui cri-
to a defesa técnica e deve ser mais ampla que a denomi- me inafiançável e imprescritível, sujeito à pena de
nada ampla defesa assegurada em todos os procedimen- reclusão, nos termos da lei.
tos judiciais e administrativos.

29
A Lei nº 7.716, de 5 de janeiro de 1989 define os crimes invés de restringir-se ao malfeitor, alcançaria inocentes.
resultantes de preconceito de raça ou de cor. Contra eles Contudo, se uma pessoa deixou patrimônio e faleceu, este
não cabe fiança (pagamento de valor para deixar a prisão patrimônio responderá pelas repercussões financeiras do
provisória) e não se aplica o instituto da prescrição (perda ilícito.
de pretensão de se processar/punir uma pessoa pelo de-
curso do tempo). Individualização da pena
Não obstante, preconiza ao artigo 5º, XLIII, CF: A individualização da pena tem por finalidade concreti-
Artigo 5º, XLIII, CF. A lei considerará crimes inafian- zar o princípio de que a responsabilização penal é sempre
çáveis e insuscetíveis de graça ou anistia a prática pessoal, devendo assim ser aplicada conforme as peculia-
da tortura, o tráfico ilícito de entorpecentes e drogas ridades do agente.
afins, o terrorismo e os definidos como crimes hedion- A primeira menção à individualização da pena se en-
dos, por eles respondendo os mandantes, os executores contra no artigo 5º, XLVI, CF:
e os que, podendo evitá-los, se omitirem. Artigo 5º, XLVI, CF. A lei regulará a individualização
da pena e adotará, entre outras, as seguintes:
Anistia, graça e indulto diferenciam-se nos seguintes a) privação ou restrição da liberdade;
termos: a anistia exclui o crime, rescinde a condenação e b) perda de bens;
extingue totalmente a punibilidade, a graça e o indulto c) multa;
apenas extinguem a punibilidade, podendo ser parciais; a d) prestação social alternativa;
anistia, em regra, atinge crimes políticos, a graça e o in- e) suspensão ou interdição de direitos.
dulto, crimes comuns; a anistia pode ser concedida pelo
Poder Legislativo, a graça e o indulto são de competência Pelo princípio da individualização da pena, a pena deve
exclusiva do Presidente da República; a anistia pode ser ser individualizada nos planos legislativo, judiciário e exe-
concedida antes da sentença final ou depois da condena- cutório, evitando-se a padronização a sanção penal. A in-
ção irrecorrível, a graça e o indulto pressupõem o trânsito dividualização da pena significa adaptar a pena ao conde-
em julgado da sentença condenatória; graça e o indulto nado, consideradas as características do agente e do delito.
apenas extinguem a punibilidade, persistindo os efeitos A pena privativa de liberdade é aquela que restringe,
do crime, apagados na anistia; graça é em regra individual com maior ou menor intensidade, a liberdade do condena-
e solicitada, enquanto o indulto é coletivo e espontâneo. do, consistente em permanecer em algum estabelecimen-
Não cabe graça, anistia ou indulto (pode-se considerar to prisional, por um determinado tempo.
que o artigo o abrange, pela doutrina majoritária) contra A pena de multa ou patrimonial opera uma diminuição
crimes de tortura, tráfico, terrorismo (TTT) e hediondos do patrimônio do indivíduo delituoso.
(previstos na Lei nº 8.072 de 25 de julho de 1990). Além A prestação social alternativa corresponde às penas
disso, são crimes que não aceitam fiança. restritivas de direitos, autônomas e substitutivas das penas
Ainda, prevê o artigo 5º, XLIV, CF: privativas de liberdade, estabelecidas no artigo 44 do Có-
Artigo 5º, XLIV, CF. Constitui crime inafiançável e digo Penal.
imprescritível a ação de grupos armados, civis ou Por seu turno, a individualização da pena deve também
militares, contra a ordem constitucional e o Estado se fazer presente na fase de sua execução, conforme se
Democrático. depreende do artigo 5º, XLVIII, CF:
Artigo 5º, XLVIII, CF. A pena será cumprida em estabe-
Por fim, dispõe a CF sobre a possibilidade de extradi- lecimentos distintos, de acordo com a natureza do
ção de brasileiro naturalizado caso esteja envolvido com delito, a idade e o sexo do apenado.
tráfico ilícito de entorpecentes:
Artigo 5º, LI, CF. Nenhum brasileiro será extraditado, A distinção do estabelecimento conforme a natureza
salvo o naturalizado, em caso de crime comum, pra- do delito visa impedir que a prisão se torne uma faculdade
ticado antes da naturalização, ou de comprovado en- do crime. Infelizmente, o Estado não possui aparato sufi-
volvimento em tráfico ilícito de entorpecentes e ciente para cumprir tal diretiva, diferenciando, no máximo,
drogas afins, na forma da lei. o nível de segurança das prisões. Quanto à idade, desta-
cam-se as Fundações Casas, para cumprimento de medida
Personalidade da pena por menores infratores. Quanto ao sexo, prisões costumam
A personalidade da pena encontra respaldo no artigo ser exclusivamente para homens ou para mulheres.
5º, XLV, CF: Também se denota o respeito à individualização da
Artigo 5º, XLV, CF. Nenhuma pena passará da pes- pena nesta faceta pelo artigo 5º, L, CF:
soa do condenado, podendo a obrigação de reparar o Artigo 5º, L, CF. Às presidiárias serão asseguradas con-
dano e a decretação do perdimento de bens ser, nos ter- dições para que possam permanecer com seus filhos
LEGISLAÇÃO ESPECÍFICA

mos da lei, estendidas aos sucessores e contra eles exe- durante o período de amamentação.
cutadas, até o limite do valor do patrimônio transferido.
Preserva-se a individualização da pena porque é toma-
O princípio da personalidade encerra o comando de da a condição peculiar da presa que possui filho no perío-
o crime ser imputado somente ao seu autor, que é, por do de amamentação, mas também se preserva a dignidade
seu turno, a única pessoa passível de sofrer a sanção. Seria da criança, não a afastando do seio materno de maneira
flagrante a injustiça se fosse possível alguém responder precária e impedindo a formação de vínculo pela ama-
pelos atos ilícitos de outrem: caso contrário, a reação, ao mentação.

30
Vedação de determinadas penas Se uma pessoa possui identificação civil, não há por-
O constituinte viu por bem proibir algumas espécies que fazer identificação criminal, colhendo digitais, fotos,
de penas, consoante ao artigo 5º, XLVII, CF: etc. Pensa-se que seria uma situação constrangedora
Artigo 5º, XLVII, CF. não haverá penas: desnecessária ao suspeito, sendo assim, violaria a inte-
a) de morte, salvo em caso de guerra declarada, nos gridade moral.
termos do art. 84, XIX;
b) de caráter perpétuo; Devido processo legal, contraditório e ampla defesa
c) de trabalhos forçados; Estabelece o artigo 5º, LIV, CF:
d) de banimento; Artigo 5º, LIV, CF. Ninguém será privado da liberdade
e) cruéis. ou de seus bens sem o devido processo legal.

Em resumo, o inciso consolida o princípio da humani- Pelo princípio do devido processo legal a legislação
dade, pelo qual o “poder punitivo estatal não pode apli- deve ser respeitada quando o Estado pretender punir al-
car sanções que atinjam a dignidade da pessoa humana guém judicialmente. Logo, o procedimento deve ser livre
ou que lesionem a constituição físico-psíquica dos con- de vícios e seguir estritamente a legislação vigente, sob
denados”20 . pena de nulidade processual.
Quanto à questão da pena de morte, percebe-se que Surgem como corolário do devido processo legal o
o constituinte não estabeleceu uma total vedação, au- contraditório e a ampla defesa, pois somente um pro-
torizando-a nos casos de guerra declarada. Obviamente, cedimento que os garanta estará livre dos vícios. Neste
deve-se respeitar o princípio da anterioridade da lei, ou sentido, o artigo 5º, LV, CF:
seja, a legislação deve prever a pena de morte ao fato Artigo 5º, LV, CF. Aos litigantes, em processo judicial
antes dele ser praticado. No ordenamento brasileiro, este ou administrativo, e aos acusados em geral são as-
papel é cumprido pelo Código Penal Militar (Decreto-Lei segurados o contraditório e ampla defesa, com os
nº 1.001/1969), que prevê a pena de morte a ser execu- meios e recursos a ela inerentes.
tada por fuzilamento nos casos tipificados em seu Livro
II, que aborda os crimes militares em tempo de guerra. O devido processo legal possui a faceta formal, pela
Por sua vez, estão absolutamente vedadas em quais- qual se deve seguir o adequado procedimento na apli-
quer circunstâncias as penas de caráter perpétuo, de tra- cação da lei e, sendo assim, respeitar o contraditório e a
balhos forçados, de banimento e cruéis. ampla defesa. Não obstante, o devido processo legal tem
No que tange aos trabalhos forçados, vale destacar sua faceta material que consiste na tomada de decisões
que o trabalho obrigatório não é considerado um trata- justas, que respeitem os parâmetros da razoabilidade e
mento contrário à dignidade do recluso, embora o tra- da proporcionalidade.
balho forçado o seja. O trabalho é obrigatório, dentro
das condições do apenado, não podendo ser cruel ou Vedação de provas ilícitas
menosprezar a capacidade física e intelectual do conde- Conforme o artigo 5º, LVI, CF:
nado; como o trabalho não existe independente da edu- Artigo 5º, LVI, CF. São inadmissíveis, no processo, as
cação, cabe incentivar o aperfeiçoamento pessoal; até provas obtidas por meios ilícitos.
mesmo porque o trabalho deve se aproximar da realida-
de do mundo externo, será remunerado; além disso, con- Provas ilícitas, por força da nova redação dada ao ar-
dições de dignidade e segurança do trabalhador, como tigo 157 do CPP, são as obtidas em violação a normas
descanso semanal e equipamentos de proteção, deverão constitucionais ou legai, ou seja, prova ilícita é a que vio-
ser respeitados. la regra de direito material, constitucional ou legal, no
momento da sua obtenção. São vedadas porque não se
Respeito à integridade do preso pode aceitar o descumprimento do ordenamento para
Prevê o artigo 5º, XLIX, CF: fazê-lo cumprir: seria paradoxal.
Artigo 5º, XLIX, CF. É assegurado aos presos o respei-
to à integridade física e moral. Presunção de inocência
Prevê a Constituição no artigo 5º, LVII:
Obviamente, o desrespeito à integridade física e mo- Artigo 5º, LVII, CF. Ninguém será considerado culpado
ral do preso é uma violação do princípio da dignidade da até o trânsito em julgado de sentença penal condena-
pessoa humana. tória.
Dois tipos de tratamentos que violam esta integrida-
de estão mencionados no próprio artigo 5º da Constitui- Consolida-se o princípio da presunção de inocência,
ção Federal. Em primeiro lugar, tem-se a vedação da tor- pelo qual uma pessoa não é culpada até que, em de-
LEGISLAÇÃO ESPECÍFICA

tura e de tratamentos desumanos e degradantes (artigo finitivo, o Judiciário assim decida, respeitados todos os
5º, III, CF), o que vale na execução da pena. princípios e garantias constitucionais.
No mais, prevê o artigo 5º, LVIII, CF:
Artigo 5º, LVIII, CF.O civilmente identificado não será Ação penal privada subsidiária da pública
submetido a identificação criminal, salvo nas hipó- Nos termos do artigo 5º, LIX, CF:
teses previstas em lei. Artigo 5º, LIX, CF. Será admitida ação privada nos
crimes de ação pública, se esta não for intentada no
20 BITENCOURT, Cezar Roberto. Tratado de direito penal. 16. ed. São
prazo legal.
Paulo: Saraiva, 2011. v. 1.

31
A chamada ação penal privada subsidiária da públi- Mesmo que a pessoa seja presa em flagrante, devido
ca encontra respaldo constitucional, assegurando que a ao princípio da presunção de inocência, entende-se que
omissão do poder público na atividade de persecução ela não deve ser mantida presa quando não preencher
criminal não será ignorada, fornecendo-se instrumento os requisitos legais para prisão preventiva ou temporária.
para que o interessado a proponha.
Indenização por erro judiciário
Prisão e liberdade A disciplina sobre direitos decorrentes do erro judi-
O constituinte confere espaço bastante extenso no ciário encontra-se no artigo 5º, LXXV, CF:
artigo 5º em relação ao tratamento da prisão, notada- Artigo 5º, LXXV, CF. O Estado indenizará o condena-
mente por se tratar de ato que vai contra o direito à liber- do por erro judiciário, assim como o que ficar preso
dade. Obviamente, a prisão não é vedada em todos os além do tempo fixado na sentença.
casos, porque práticas atentatórias a direitos fundamen-
tais implicam na tipificação penal, autorizando a restrição Trata-se do erro em que incorre um juiz na apreciação
da liberdade daquele que assim agiu. e julgamento de um processo criminal, resultando em
No inciso LXI do artigo 5º, CF, prevê-se: condenação de alguém inocente. Neste caso, o Estado
Artigo 5º, LXI, CF. Ninguém será preso senão em fla- indenizará. Ele também indenizará uma pessoa que ficar
grante delito ou por ordem escrita e fundamen- presa além do tempo que foi condenada a cumprir.
tada de autoridade judiciária competente, salvo nos
casos de transgressão militar ou crime propriamente 4) Direitos fundamentais implícitos
militar, definidos em lei. Nos termos do § 2º do artigo 5º da Constituição
Federal:
Logo, a prisão somente se dará em caso de flagrante Artigo 5º, §2º, CF. Os direitos e garantias expressos
delito (necessariamente antes do trânsito em julgado), ou nesta Constituição não excluem outros decorrentes
em caráter temporário, provisório ou definitivo (as duas do regime e dos princípios por ela adotados, ou dos
primeiras independente do trânsito em julgado, preen-
tratados internacionais em que a República Federati-
chidos requisitos legais e a última pela irreversibilidade
va do Brasil seja parte.
da condenação).
Aborda-se no artigo 5º, LXII o dever de comunicação
Daí se depreende que os direitos ou garantias podem
ao juiz e à família ou pessoa indicada pelo preso:
estar expressos ou implícitos no texto constitucional.
Artigo 5º, LXII, CF. A prisão de qualquer pessoa e o
Sendo assim, o rol enumerado nos incisos do artigo 5º é
local onde se encontre serão comunicados imediata-
mente ao juiz competente e à família do preso ou à apenas exemplificativo, não taxativo.
pessoa por ele indicada.
5) Tratados internacionais incorporados ao orde-
Não obstante, o preso deverá ser informado de todos namento interno
os seus direitos, inclusive o direito ao silêncio, podendo Estabelece o artigo 5º, § 2º, CF que os direitos e ga-
entrar em contato com sua família e com um advogado, rantias podem decorrer, dentre outras fontes, dos “tra-
conforme artigo 5º, LXIII, CF: tados internacionais em que a República Federativa do
Artigo 5º, LXIII, CF. O preso será informado de seus Brasil seja parte”.
direitos, entre os quais o de permanecer calado, Para o tratado internacional ingressar no ordenamen-
sendo-lhe assegurada a assistência da família e de to jurídico brasileiro deve ser observado um procedimen-
advogado. to complexo, que exige o cumprimento de quatro fases: a
negociação (bilateral ou multilateral, com posterior assi-
Estabelece-se no artigo 5º, LXIV, CF: natura do Presidente da República), submissão do trata-
Artigo 5º, LXIV, CF. O preso tem direito à identifica- do assinado ao Congresso Nacional (que dará referendo
ção dos responsáveis por sua prisão ou por seu inter- por meio do decreto legislativo), ratificação do tratado
rogatório policial. (confirmação da obrigação perante a comunidade inter-
nacional) e a promulgação e publicação do tratado pelo
Por isso mesmo, o auto de prisão em flagrante e a Poder Executivo21. Notadamente, quando o constituinte
ata do depoimento do interrogatório são assinados pelas menciona os tratados internacionais no §2º do artigo 5º
autoridades envolvidas nas práticas destes atos procedi- refere-se àqueles que tenham por fulcro ampliar o rol de
mentais. direitos do artigo 5º, ou seja, tratado internacional de di-
Ainda, a legislação estabelece inúmeros requisitos reitos humanos.
para que a prisão seja validada, sem os quais cabe rela- O §1° e o §2° do artigo 5° existiam de maneira ori-
xamento, tanto que assim prevê o artigo 5º, LXV, CF: ginária na Constituição Federal, conferindo o caráter de
LEGISLAÇÃO ESPECÍFICA

Artigo 5º, LXV, CF. A prisão ilegal será imediatamen- primazia dos direitos humanos, desde logo consagran-
te relaxada pela autoridade judiciária. do o princípio da primazia dos direitos humanos, como
reconhecido pela doutrina e jurisprudência majoritários
Desta forma, como decorrência lógica, tem-se a pre- na época. “O princípio da primazia dos direitos humanos
visão do artigo 5º, LXVI, CF: nas relações internacionais implica em que o Brasil deve
Artigo 5º, LXVI, CF. Ninguém será levado à prisão ou incorporar os tratados quanto ao tema ao ordenamento
nela mantido, quando a lei admitir a liberdade pro-
visória, com ou sem fiança. 21 VICENTE SOBRINHO, Benedito. Direitos Fundamentais e Prisão
Civil. Porto Alegre: Sérgio Antonio Fabris Editor, 2008.

32
interno brasileiro e respeitá-los. Implica, também em que as normas voltadas à proteção da dignidade em caráter uni-
versal devem ser aplicadas no Brasil em caráter prioritário em relação a outras normas”22.
Regra geral, os tratados internacionais comuns ingressam com força de lei ordinária no ordenamento jurídico brasi-
leiro porque somente existe previsão constitucional quanto à possibilidade da equiparação às emendas constitucionais
se o tratado abranger matéria de direitos humanos. Antes da emenda alterou o quadro quanto aos tratados de direitos
humanos, era o que acontecia, mas isso não significa que tais direitos eram menos importantes devido ao princípio da
primazia e ao reconhecimento dos direitos implícitos.
Por seu turno, com o advento da Emenda Constitucional nº 45/04 se introduziu o §3º ao artigo 5º da Constituição
Federal, de modo que os tratados internacionais de direitos humanos foram equiparados às emendas constitucionais,
desde que houvesse a aprovação do tratado em cada Casa do Congresso Nacional e obtivesse a votação em dois turnos
e com três quintos dos votos dos respectivos membros:
Art. 5º, § 3º, CF. Os tratados e convenções internacionais sobre direitos humanos que forem aprovados, em cada
Casa do Congresso Nacional, em dois turnos, por três quintos dos votos dos respectivos membros, serão equivalentes às
emendas constitucionais.

Logo, a partir da alteração constitucional, os tratados de direitos humanos que ingressarem no ordenamento jurí-
dico brasileiro, versando sobre matéria de direitos humanos, irão passar por um processo de aprovação semelhante ao
da emenda constitucional.
Contudo, há posicionamentos conflituosos quanto à possibilidade de considerar como hierarquicamente consti-
tucional os tratados internacionais de direitos humanos que ingressaram no ordenamento jurídico brasileiro anterior-
mente ao advento da referida emenda. Tal discussão se deu com relação à prisão civil do depositário infiel, prevista
como legal na Constituição e ilegal no Pacto de São José da Costa Rica (tratado de direitos humanos aprovado antes
da EC nº 45/04), sendo que o Supremo Tribunal Federal firmou o entendimento pela supralegalidade do tratado de
direitos humanos anterior à Emenda (estaria numa posição que paralisaria a eficácia da lei infraconstitucional, mas não
revogaria a Constituição no ponto controverso).

6) Tribunal Penal Internacional


Preconiza o artigo 5º, CF em seu § 4º:
Artigo 5º, §4º, CF. O Brasil se submete à jurisdição de Tribunal Penal Internacional a cuja criação tenha manifestado
adesão.

O Estatuto de Roma do Tribunal Penal Internacional foi promulgado no Brasil pelo Decreto nº 4.388 de 25 de se-
tembro de 2002. Ele contém 128 artigos e foi elaborado em Roma, no dia 17 de julho de 1998, regendo a competência
e o funcionamento deste Tribunal voltado às pessoas responsáveis por crimes de maior gravidade com repercussão
internacional (artigo 1º, ETPI).
“Ao contrário da Corte Internacional de Justiça, cuja jurisdição é restrita a Estados, ao Tribunal Penal Internacional
compete o processo e julgamento de violações contra indivíduos; e, distintamente dos Tribunais de crimes de guerra
da Iugoslávia e de Ruanda, criados para analisarem crimes cometidos durante esses conflitos, sua jurisdição não está
restrita a uma situação específica”23.
Resume Mello24: “a Conferência das Nações Unidas sobre a criação de uma Corte Criminal Internacional, reunida
em Roma, em 1998, aprovou a referida Corte. Ela é permanente. Tem sede em Haia. A corte tem personalidade interna-
cional. Ela julga: a) crime de genocídio; b) crime contra a humanidade; c) crime de guerra; d) crime de agressão. Para o
crime de genocídio usa a definição da convenção de 1948. Como crimes contra a humanidade são citados: assassinato,
escravidão, prisão violando as normas internacionais, violação tortura, apartheid, escravidão sexual, prostituição for-
çada, esterilização, etc. São crimes de guerra: homicídio internacional, destruição de bens não justificada pela guerra,
deportação, forçar um prisioneiro a servir nas forças inimigas, etc.”.
LEGISLAÇÃO ESPECÍFICA

22 PORTELA, Paulo Henrique Gonçalves. Direito Internacional Público e Privado. Salvador: JusPodivm, 2009.
23 NEVES, Gustavo Bregalda. Direito Internacional Público & Direito Internacional Privado. 3. ed. São Paulo: Atlas, 2009.
24 MELLO, Celso D. de Albuquerque. Curso de Direito Internacional Público. 14. ed. São Paulo: Saraiva, 2000.

33
#FicaDica
Fundamentos (art. 1º)
Soberania
Cidadania
Dignidade da pessoa humana
Valores sociais do trabalho e da livre iniciativa
Pluralismo Político
Este método mnemônico ajuda muito na hora da prova.
Objetivos (art. 3º - são verbos)
Construir uma sociedade livre, justa e solidária
Garantir o desenvolvimento nacional
Erradicar a pobreza e marginalização e reduzir as desigualdades sociais e regionais
Promover o bem de todos, sem preconceito de origem, raça, sexo, cor, idade e quaiquer outras formas
de descriminação.

Organização político administrativa do Estado.


Conforme o art. 18 da CF, a organização político-administrativa da República Federativa do Brasil compreende a
União, os Estados, o Distrito Federal e os Municípios, sendo que todos possuem sua autonomia, tendo Brasília como
Capital Federal.
Dalmo Dallari define o estado como uma ordem jurídica soberana que tem por finalidade o bem do povo situado
em um determinado território. Isto é, dentro desta frase o Dalmo trouxe os principais elementos que compõe o Estado,
que são: soberania, finalidade, povo e território.
A estrutura e organização do Estado podem ser analisados sob três aspectos, conforme Pedro Lenza, p. 499:
1) Forma de governo: República ou Monarquia;
2) Sistema de Governo: Presidencialismo ou Parlamentarismo;
3) Forma de Estado: Estado unitário ou Federação.

Em 1889, surgiu a Federação do Brasil, juntamente com a forma de governo (republicana). A forma de governo
republicana seria realizar através do regime representativo em 1891.
Desta forma, o Brasil consagrou o seguinte:
1) Forma de Estado: Federação.
2) Entes componentes do Estado brasileiro: União, Estado, Distrito Federal e Municípios.
3) Características do Estado brasileiro: Estado Democrático de Direito.
4) Sistema de Governo: Presidencialista.
5) Forma de Governo: Republicana.

O idioma oficial do país é a língua portuguesa e os símbolos da República Federativa do Brasil são: bandeira, hino,
armas e o selo nacional, sendo que o Distrito Federal, Estados e os Municípios poderão ter seus próprios símbolos,
conforme o art. 13 §1º e §2º da CF.
Conforme tutela o art. 19 da CF, existe vedações constitucional para que os Estados, Distrito Federal, Munícipios e
a União não possam:
- estabelecer cultos religiosos ou igrejas, subvencioná-los, embaraçar-lhes o funcionamento ou manter com eles ou
seus representantes relações de dependência ou aliança, ressalvada, na forma da lei, a colaboração de interesse público;
- recusar fé aos documentos públicos;
- criar distinções entre brasileiros ou preferências entre si.

União Federal
A República Federativa do Brasil é composta pela União, Estados Membros, Distrito Federal e os Municípios.
A União possui bens próprios os quais estão descritos no art. 20 da CF, como por exemplo: mar territorial, os terre-
nos de marinha e seus acrescidos, as ilhas fluviais e lacustres nas zonas limítrofes com outros países; as praias marítimas;
as ilhas oceânicas e as costeiras, excluídas, destas, as que contenham a sede de Municípios, exceto aquelas áreas afe-
LEGISLAÇÃO ESPECÍFICA

tadas ao serviço público e a unidade ambiental federal, e as referidas no art. 26, II, os potenciais de energia hidráulica,
as terras tradicionalmente ocupadas pelos índios, dentre outros.

34
FIQUE ATENTO!
É assegurada, nos termos da lei, aos Estados, ao Distrito Federal e aos Municípios, bem como a órgãos
da administração direta da União, participação no resultado da exploração de petróleo ou gás natural,
de recursos hídricos para fins de geração de energia elétrica e de outros recursos minerais no respectivo
território, plataforma continental, mar territorial ou zona econômica exclusiva, ou compensação financeira
por essa exploração.
No tocante a área de atuação da União, a mesma possui competência não legislativa, ou seja, ela atua no
campo politico-administrativo, como por exemplo:
- manter relações com Estados estrangeiros e participar de organizações internacionais;
- declarar a guerra e celebrar a paz;
- assegurar a defesa nacional;
- permitir, nos casos previstos em lei complementar, que forças estrangeiras transitem pelo território
nacional ou nele permaneçam temporariamente;
- emitir moeda;
- elaborar e executar planos nacionais e regionais de ordenação do território e de desenvolvimento
econômico e social;
- explorar, diretamente ou mediante autorização, concessão ou permissão, os serviços de telecomunicações,
nos termos da lei, que disporá sobre a organização dos serviços, a criação de um órgão regulador e outros
aspectos institucionais;
- organizar e manter o Poder Judiciário, o Ministério Público do Distrito Federal e dos Territórios e a
Defensoria Pública dos Territórios, DENTRE OUTROS.

Os itens elencados acima, são de competência exclusiva da União. Já os itens do art. 23 da CF, são de competência
cumulativa (comuns) entre a União, Estados, Distrito Federal e Munícipios, como por exemplo: zelar pela guarda da
Constituição, das leis e das instituições democráticas e conservar o patrimônio público, cuidar da saúde e assistência
pública, da proteção e garantia das pessoas portadoras de deficiência, proteger os documentos, as obras e outros bens
de valor histórico, artístico e cultural, os monumentos, as paisagens naturais notáveis e os sítios arqueológicos, pro-
porcionar os meios de acesso à cultura, à educação, à ciência, à tecnologia, à pesquisa e à inovação, proteger o meio
ambiente e combater a poluição em qualquer de suas formas e etc.

#FicaDica
Conforme o art.24 da CF, compete à União, aos Estados e ao Distrito Federal legislar concorrentemente
sobre: direito tributário, financeiro, penitenciário, econômico e urbanístico; orçamento; juntas comerciais;
custas dos serviços forenses; produção e consumo; Florestas, proteção ao patrimônio histórico, cultural,
artístico, turístico e paisagístico; responsabilidade por dano ao meio ambiente, ao consumidor, a bens
e direitos de valor artístico e etc; educação, cultura, ensino, desporto, ciência, tecnologia, pesquisa,
desenvolvimento e inovação, dentre outros.

Estados – Membros
Os Estados membros são a materialização da descentralização do poder político. Esses Estados são autônomos e
devido a isso, possuem a capacidade de auto-organização, autogoverno, autoadministração e autolegislação.
Por se tratarem de Estados autônomos, a Constituição Federal delegou a competência da estruturação de seus
poderes para eles mesmos, sem que haja qualquer interferência federal ou subordinação ao poder central: o legislativo
(art. 27 da CF), executivo (art. 28 da CF) e o judiciário (art. 125 da CF). (MASSON, 2016, p. 552).
Em especial ao poder legislativo, em âmbito estadual, podemos dizer que ele é unicameral (conforme art. 27da CF),
sendo o poder representado pela Assembleia Legislativa. O sistema eleitoral para a casa é o sistema proporcional, isto
é, os deputados são eleitos para um mandato de 4 anos, sendo que o número de Deputados estaduais corresponderá
ao triplo da representação do Estado da Câmara dos Deputados, e atingido o número de trinta e seis, será acrescido de
tantos quantos forem os Deputados Federais acima de 12. (MASSON, 2016, p. 552).
LEGISLAÇÃO ESPECÍFICA

Por fim, a eleição do governador e vice, é pelo sistema majoritário absoluto, sendo que a posse ocorrerá no dia 1º
de Janeiro do ano subsequente (art. 28, CF).

Municípios
Conforme dispõe o art. 29 da CF, os municípios de organizam através de Lei Orgânica, votada sempre em dois
turnos, com o interstício mínimo de 10 dias, e aprovada por 2/3 dos membros da respectiva Câmara Municipal, que a
promulgará. Ao elaborar sua lei, o município deverá observar os princípios abordados na Constituição, bem como, pela
Constituição Estadual, conforme o art. 11, parágrafo único do ADCT.

35
Os municípios possuem o autogoverno de eleger o § 4º Às polícias civis, dirigidas por delegados de polí-
poder executivo (seu prefeito), bem como, o poder legis- cia de carreira, incumbem, ressalvada a competência
lativo da cidade (os vereadores). da União, as funções de polícia judiciária e a apura-
ção de infrações penais, exceto as militares.
§ 5º Às polícias militares cabem a polícia ostensiva e
a preservação da ordem pública; aos corpos de bom-
EXERCÍCIO COMENTADO beiros militares, além das atribuições definidas em
lei, incumbe a execução de atividades de defesa civil.
1. (POLÍCIA FEDERAL – DELEGADO – CESPE – 2018) § 6º As polícias militares e corpos de bombeiros milita-
Acerca da disciplina constitucional da segurança pública, res, forças auxiliares e reserva do Exército, subordinam-
do Poder Judiciário, do MP e das atribuições da PF, julgue -se, juntamente com as polícias civis, aos Governadores
o seguinte item. dos Estados, do Distrito Federal e dos Territórios.
É concorrente a competência da União e dos estados § 7º A lei disciplinará a organização e o funcionamen-
para legislar sobre a organização, os direitos e os deveres to dos órgãos responsáveis pela segurança pública,
das polícias civis dos estados. de maneira a garantir a eficiência de suas atividades.
§ 8º Os Municípios poderão constituir guardas muni-
( ) Certo ( ) Errado cipais destinadas à proteção de seus bens, serviços e
instalações, conforme dispuser a lei.
Resposta: Certo
Conforme o Art. 24, CF. Compete à União, aos Estados
e ao Distrito Federal legislar concorrentemente sobre: #FicaDica
XVI - organização, garantias, direitos e deveres das A Segurança Pública é dever do Estado, e
polícias civis. direito e responsabilidade de todos.
Título V
Da Defesa do Estado e das Instituições Democráticas
Capítulo III
Da Segurança Pública EXERCÍCIOS COMENTADOS
Art. 144. A segurança pública, dever do Estado, di-
1. (TJ/MG - Juiz - FUNDEP/2014) Sobre o conceito de
reito e responsabilidade de todos, é exercida para
Constituição, assinale a alternativa CORRETA.
a preservação da ordem pública e da incolumidade
das pessoas e do patrimônio, através dos seguintes
a) É o estatuto que regula as relações entre Estados so-
órgãos:
beranos.
I -polícia federal;
b) É o conjunto de normas que regula os direitos e deve-
II -polícia rodoviária federal; res de um povo.
III -polícia ferroviária federal; c) É a lei fundamental e suprema de um Estado, que con-
IV -polícias civis; tém normas referentes à estruturação, à formação dos
V -polícias militares e corpos de bombeiros militares. poderes públicos, direitos, garantias e deveres dos ci-
§ 1º A polícia federal, instituída por lei como órgão dadãos.
permanente, estruturado em carreira, destina-se a: d) É a norma maior de um Estado, que regula os direitos
I -apurar infrações penais contra a ordem política e deveres de um povo nas suas relações.
e social ou em detrimento de bens, serviços e inte-
resses da União ou de suas entidades autárquicas e Resposta: Letra C. Constituição é muito mais do
empresas públicas, assim como outras infrações cuja que um documento escrito que fica no ápice do or-
prática tenha repercussão interestadual ou interna- denamento jurídico nacional estabelecendo normas
cional e exija repressão uniforme, segundo se dispu- de limitação e organização do Estado, mas tem um
ser em lei; significado intrínseco sociológico, político, cultural e
II -prevenir e reprimir o tráfico ilícito de entorpecen- econômico. Independente do conceito, percebe-se
tes e drogas afins, o contrabando e o descaminho, que o foco é a organização do Estado e a limitação
sem prejuízo da ação fazendária e de outros órgãos de seu poder.
públicos nas respectivas áreas de competência;
III -exercer as funções de polícia marítima, aérea e de 2. (TJ/MG - Juiz - FUNDEP/2014) Dentre as formas de
fronteiras; classificação das Constituições, uma delas é quanto à ori-
LEGISLAÇÃO ESPECÍFICA

IV -exercer, com exclusividade, as funções de polícia gem.


judiciária da União. Em relação às características de uma Constituição quanto
§ 2º A polícia rodoviária federal, órgão permanente, à sua origem, assinale a alternativa CORRETA.
estruturado em carreira, destina-se, na forma da lei,
ao patrulhamento ostensivo das rodovias federais. a) Dogmáticas ou históricas.
§ 3º A polícia ferroviária federal, órgão permanente, b) Materiais ou formais.
estruturado em carreira, destina-se, na forma da lei, c) Analíticas ou sintéticas.
ao patrulhamento ostensivo das ferrovias federais. d) Promulgadas ou outorgadas.

36
Resposta: Letra D. Quanto à origem, a Constituição
pode ser outorgada, quando imposta unilateralmente DECLARAÇÃO UNIVERSAL DOS DIREITOS
pelo agente revolucionário, ou promulgada, quando é HUMANOS DE 1948;
votada, sendo também conhecida como democrática
ou popular.
DECLARAÇÃO UNIVERSAL DOS DIREITOS
3. (TJ/MG - Juiz - FUNDEP/2014) Sobre a supremacia HUMANOS
da Constituição da República, assinale a alternativa COR-
RETA. Adotada e proclamada pela Resolução n° 217 A (III) da
Assembleia Geral das Nações Unidas em 10 de dezembro
a) A supremacia está no fato de o controle da consti- de 1948
tucionalidade das leis só ser exercido pelo Supremo
Tribunal Federal. Preâmbulo
b) A supremacia está na obrigatoriedade de submissão O preâmbulo é um elemento comum em textos cons-
das leis aos princípios que norteiam o Estado por ela titucionais. Em relação ao preâmbulo constitucional, Jor-
instituído. ge Miranda25 define: “[...] proclamação mais ou menos
c) A supremacia está no fato de a interpretação da cons- solene, mais ou menos significante, anteposta ao arti-
tituição não depender da observância dos princípios culado constitucional, não é componente necessário de
que a norteiam. qualquer Constituição, mas tão somente um elemento
d) A supremacia está no fato de que os princípios e fun- natural de Constituições feitas em momentos de ruptura
damentos da constituição se resumam na declaração histórica ou de grande transformação político-social”. Do
de soberania. conceito do autor é possível extrair elementos para de-
finir o que representam os preâmbulos em documentos
Resposta: Letra B. A Constituição Federal e os demais internacionais: proclamação dotada de certa solenidade
atos normativos que compõem o denominado bloco e significância que antecede o texto do documento inter-
de constitucionalidade, notadamente, emendas cons- nacional e, embora não seja um elemento necessário a
titucionais e tratados internacionais de direitos huma- ele, merece ser considerada porque reflete o contexto de
nos aprovados com quórum especial após a Emenda ruptura histórica e de transformação político-social que
Constitucional nº 45/2004, estão no topo do ordena- levou à elaboração do documento como um todo. No
mento jurídico. Sendo assim, todos os atos abaixo de- caso da Declaração de 1948 ficam evidentes os antece-
les devem guardar uma estrita compatibilidade, sob dentes históricos inerentes às Guerras Mundiais.
pena de serem inconstitucionais. Por isso, estes atos Considerando que o reconhecimento da dignidade ine-
que estão abaixo na pirâmide, se sujeitam a controle rente a todos os membros da família humana e de seus
de constitucionalidade. direitos iguais e inalienáveis é o fundamento da liberdade,
da justiça e da paz no mundo,
4. (PC/PI - Delegado de Polícia – UESPI/2014) Entre os O princípio da dignidade da pessoa humana, pelo
chamados sentidos doutrinariamente atribuídos à Cons- qual todos os seres humanos são dotados da mesma
tituição, existe um que realiza a distinção entre Consti- dignidade e para que ela seja preservada é preciso que
tuição e lei constitucional. Assinale a alternativa que o os direitos inerentes à pessoa humana sejam garantidos,
contempla. já aparece no preâmbulo constitucional, sendo guia de
todo documento.
a) Sentido político Denota-se, ainda, a característica da inalienabilidade
b) Sentido sociológico. dos direitos humanos, pela qual os direitos humanos não
c) Sentido jurídico. possuem conteúdo econômico-patrimonial, logo, são in-
d) Sentido culturalista. transferíveis, inegociáveis e indisponíveis, estando fora
e) Sentido simbólico. do comércio, o que evidencia uma limitação do princípio
da autonomia privada.
Resposta: Letra A. Carl Schmitt propõe que o con- Considerando que o desprezo e o desrespeito pelos di-
ceito de Constituição não está na Constituição em si, reitos humanos resultaram em atos bárbaros que ultraja-
mas nas decisões políticas tomadas antes de sua ela- ram a consciência da Humanidade e que o advento de um
boração. Sendo assim, o conceito de Constituição será mundo em que os homens gozem de liberdade de palavra,
estruturado por fatores como o regime de governo e de crença e da liberdade de viverem a salvo do temor e da
a forma de Estado vigentes no momento de elabora- necessidade foi proclamado como a mais alta aspiração
ção da lei maior. A Constituição é o produto de uma
LEGISLAÇÃO ESPECÍFICA

do homem comum,
decisão política e variará conforme o modelo político A humanidade nunca irá esquecer das imagens vistas
à época de sua elaboração. quando da abertura dos campos de concentração na-
zistas, nos quais os cadáveres esqueléticos do que não
eram considerados seres humanos perante aquele regi-
me político se amontoavam. Aquelas pessoas não eram
consideradas iguais às demais por possuírem alguma ca-
25 MIRANDA, Jorge (Coord.). Estudos sobre a constituição.
Lisboa: Petrony, 1978.

37
racterística, crença ou aparência que o Estado não apoia- O primeiro artigo da Declaração é altamente repre-
va. Daí a importância de se atentar para os antecedentes sentativo, trazendo diversos conceitos chaves de todo o
históricos e compreender a igualdade de todos os ho- documento:
mens, independentemente de qualquer fator. a) Princípios da universalidade, presente na palavra to-
Considerando essencial que os direitos humanos sejam dos, que se repete no documento inteiro, pelo qual
protegidos pelo Estado de Direito, para que o homem não os direitos humanos pertencem a todos e por isso
seja compelido, como último recurso, à rebelião contra ti- se encontram ligados a um sistema global (ONU), o
rania e a opressão, que impede o retrocesso.
Por todo o mundo se espalharam, notadamente du- Na primeira parte do artigo estatui-se que não basta a
rante a Segunda Guerra Mundial, regimes totalitários al- igualdade formal perante a lei, mas é preciso realizar esta
tamente opressivos, não só por parte das Potências do igualdade de forma a ser possível que todo homem atinja
Eixo (Alemanha, Itália, Japão), mas também no lado dos um grau satisfatório de dignidade. Neste sentido, as dis-
Aliados (Rússia e o regime de Stálin). criminações legais asseguram a verdadeira igualdade, por
Considerando essencial promover o desenvolvimento exemplo, com as ações afirmativas, a proteção especial ao
de relações amistosas entre as nações, trabalho da mulher e do menor, as garantias aos porta-
Depois de duas grandes guerras a humanidade con- dores de deficiência, entre outras medidas que atribuam
seguiu perceber o quanto era prejudicial não manter re- a pessoas com diferentes condições, iguais possibilidades,
lações amistosas entre as nações, de forma que o ideal protegendo e respeitando suas diferenças.26
de paz ganhou uma nova força. b) Princípio da dignidade da pessoa humana: a dignida-
Considerando que os povos das Nações Unidas rea- de é um atributo da pessoa humana, segundo o qual
firmaram, na Carta, sua fé nos direitos humanos funda- ela merece todo o respeito por parte dos Estados
mentais, na dignidade e no valor da pessoa humana e na e dos demais indivíduos, independentemente de
igualdade de direitos dos homens e das mulheres, e que qualquer fator como aparência, religião, sexualidade,
decidiram promover o progresso social e melhores condi- condição financeira. Todo ser humano é digno e, por
ções de vida em uma liberdade mais ampla, isso, possui direitos que visam garantir tal dignidade.
c) Dimensões de direitos humanos: tradicionalmente,
Considerando que os Estados-Membros se comprome-
os direitos humanos dividem-se em três dimensões,
teram a desenvolver, em cooperação com as Nações Uni-
cada qual representativa de um momento histórico
das, o respeito universal aos direitos humanos e liberdades
no qual se evidenciou a necessidade de garantir di-
fundamentais e a observância desses direitos e liberdades,
reitos de certa categoria. A primeira dimensão, pre-
Considerando que uma compreensão comum desses
sente na expressão livres, refere-se aos direitos civis
direitos e liberdades é da mais alta importância para o
e políticos, os quais garantem a liberdade do ho-
pleno cumprimento desse compromisso,
mem no sentido de não ingerência estatal e de par-
Todos os países que fazem parte da Organização das ticipação nas decisões políticas, evidenciados histo-
Nações Unidas, tanto os 51 membros fundadores quanto ricamente com as Revoluções Americana e Francesa.
os que ingressaram posteriormente (basicamente, todos A segunda dimensão, presente na expressão iguais,
demais países do mundo), totalizando 193, assumiram o refere-se aos direitos econômicos, sociais e culturais,
compromisso de cumprir a Carta da ONU, documento os quais garantem a igualdade material entre os ci-
que a fundou e que traz os princípios condutores da ação dadãos exigindo prestações positivas estatais nesta
da organização. direção, por exemplo, assegurando direitos traba-
lhistas e de saúde, possuindo como antecedente
A Assembleia Geral proclama histórico a Revolução Industrial. A terceira dimensão,
A presente Declaração Universal dos Diretos Humanos presente na expressão fraternidade, refere-se ao ne-
como o ideal comum a ser atingido por todos os povos e cessário olhar sobre o mundo como um lugar de to-
todas as nações, com o objetivo de que cada indivíduo e dos, no qual cada qual deve reconhecer no outro seu
cada órgão da sociedade, tendo sempre em mente esta semelhante, digno de direitos, olhar este que tam-
Declaração, se esforce, através do ensino e da educação, bém se lança para as gerações futuras, por exemplo,
por promover o respeito a esses direitos e liberdades, e, com a preservação do meio ambiente e a garantia
pela adoção de medidas progressivas de caráter nacional da paz social, sendo o marco histórico justamente
e internacional, por assegurar o seu reconhecimento e a as Guerras Mundiais.27 Assim, desde logo a Decla-
sua observância universais e efetivos, tanto entre os povos ração estabelece seus parâmetros fundamentais,
dos próprios Estados-Membros, quanto entre os povos dos com esteio na Declaração dos Direitos do Homem e
territórios sob sua jurisdição. do Cidadão de 1789 e na Constituição Francesa de
A Assembleia Geral é o principal órgão deliberativo 1791, quais sejam igualdade, liberdade e fraternida-
das Nações Unidas, no qual há representatividade de to- de. Embora os direitos de 1ª, 2ª e 3ª dimensão, que
LEGISLAÇÃO ESPECÍFICA

dos os membros e por onde passam inúmeros tratados se baseiam nesta tríade, tenham surgido de forma
internacionais. paulatina, devem ser considerados em conjunto
proporcionando a plena realização do homem28.
Artigo I 26 BALERA, Wagner (Coord.). Comentários à
Todas as pessoas nascem livres e iguais em digni- Declaração Universal dos Direitos do Homem. Brasília: Fortium,
dade e direitos. São dotadas de razão e consciência e 2008.
devem agir em relação umas às outras com espírito de 27 BOBBIO, Norberto. A era dos direitos.
Tradução Celso Lafer. 9. ed. Rio de Janeiro: Elsevier, 2004.
fraternidade.
28 BALERA, Wagner (Coord.). Comentários à

38
Na primeira parte do artigo estatui-se que não basta primeira esfera, enquadram-se questões como pena de
a igualdade formal perante a lei, mas é preciso realizar morte, aborto, pesquisas com células-tronco, eutanásia,
esta igualdade de forma a ser possível que todo homem entre outras polêmicas. Na segunda esfera, notam-se
atinja um grau satisfatório de dignidade. desdobramentos como a proibição de tratamentos in-
Neste sentido, as discriminações legais asseguram a dignos, a exemplo da tortura, dos trabalhos forçados, etc.
verdadeira igualdade, por exemplo, com as ações afir- A vida humana é o centro gravitacional no qual or-
mativas, a proteção especial ao trabalho da mulher e do bitam todos os direitos da pessoa humana, possuindo
menor, as garantias aos portadores de deficiência, entre reflexos jurídicos, políticos, econômicos, morais e religio-
outras medidas que atribuam a pessoas com diferentes sos. Daí existir uma dificuldade em conceituar o vocábulo
condições, iguais possibilidades, protegendo e respei- vida. Logo, tudo aquilo que uma pessoa possui deixa de
tando suas diferenças. ter valor ou sentido se ela perde a vida. Sendo assim, a
vida é o bem principal de qualquer pessoa, é o primeiro
Artigo II valor moral de todos os seres humanos. Trata-se de um
Toda pessoa tem capacidade para gozar os direitos e as direito que pode ser visto em 4 aspectos, quais sejam: a)
liberdades estabelecidos nesta Declaração, sem distinção direito de nascer; b) direito de permanecer vivo; c) direito
de qualquer espécie, seja de raça, cor, sexo, língua, religião, de ter uma vida digna quanto à subsistência e; d) direito
opinião política ou de outra natureza, origem nacional ou de não ser privado da vida através da pena de morte32.
social, riqueza, nascimento, ou qualquer outra condição. Por sua vez, o direito à liberdade é posto como con-
Reforça-se o princípio da igualdade, bem como o da sectário do direito à vida, pois ela depende da liberdade
dignidade da pessoa humana, de forma que todos se- para o desenvolvimento intelectual e moral. Assim, “[...]
res humanos são iguais independentemente de qualquer liberdade é assim a faculdade de escolher o próprio ca-
condição, possuindo os mesmos direitos visando a pre- minho, sendo um valor inerente à dignidade do ser, uma
servação de sua dignidade. vez que decorre da inteligência e da volição, duas carac-
O dispositivo traz um aspecto da igualdade que im- terísticas da pessoa humana”33.
pede a distinção entre pessoas pela condição do país ou O direito à segurança pessoal é o direito de viver sem
medo, protegido pela solidariedade e liberto de agres-
território a que pertença, o que é importante sob o as-
sões, logo, é uma maneira de garantir o direito à vida34.
pecto de proteção dos refugiados, prisioneiros de guerra,
pessoas perseguidas politicamente, nacionais de Estados
Artigo IV
que não cumpram os preceitos das Nações Unidas. Não
Ninguém será mantido em escravidão ou servidão, a
obstante, a discriminação não é proibida apenas quan-
escravidão e o tráfico de escravos serão proibidos em todas
to a indivíduos, mas também quanto a grupos humanos,
as suas formas.
sejam formados por classe social, etnia ou opinião em
“O trabalho escravo não se confunde com o trabalho
comum29.“A Declaração reconhece a capacidade de gozo servil. A escravidão é a propriedade plena de um homem
indistinto dos direitos e liberdades assegurados a todos sobre o outro. Consiste na utilização, em proveito pró-
os homens, e não apenas a alguns setores ou atores so- prio, do trabalho alheio. Os escravos eram considerados
ciais. Garantir a capacidade de gozo, no entanto, não é seres humanos sem personalidade, mérito ou valor. A
suficiente para que este realmente se efetive. É funda- servidão, por seu turno, é uma alienação relativa da li-
mental aos ordenamentos jurídicos próprios dos Estados berdade de trabalho através de um pacto de prestação
viabilizar os meios idôneos a proporcionar tal gozo, a fim de serviços ou de uma ligação absoluta do trabalhador
de que se perfectibilize, faticamente, esta garantia. Isto à terra, já que a servidão era uma instituição típica das
se dá não somente com a igualdade material diante da sociedades feudais. A servidão, representava a espinha
lei, mas também, e principalmente, através do reconhe- dorsal do feudalismo. O servo pagava ao senhor feudal
cimento e respeito das desigualdades naturais entre os uma taxa altíssima pela utilização do solo, que superava
homens, as quais devem ser resguardadas pela ordem a metade da colheita”35.
jurídica, pois é somente assim que será possível propiciar A abolição da escravidão foi uma luta histórica em
a aludida capacidade de gozo a todos”30. todo o globo. Seria totalmente incoerente quanto aos
princípios da liberdade, da igualdade e da dignidade se
Artigo III admitir que um ser humano pudesse ser submetido ao
Toda pessoa tem direito à vida, à liberdade e à segu- outro, ser tratado como coisa. O ser humano não possui
rança pessoal. valor financeiro e nem serve ao domínio de outro, razão
Segundo Lenza31, “abrange tanto o direito de não ser pela qual a escravidão não pode ser aceita.
morto, privado da vida, portanto, direito de continuar
32 BALERA, Wagner (Coord.). Comentários à
vivo, como também o direito de ter uma vida digna”. Na Declaração Universal dos Direitos do Homem. Brasília: Fortium,
LEGISLAÇÃO ESPECÍFICA

Declaração Universal dos Direitos do Homem. Brasília: Fortium, 2008.


2008 33 BALERA, Wagner (Coord.). Comentários à
29 BALERA, Wagner (Coord.). Comentários Declaração Universal dos Direitos do Homem. Brasília: Fortium,
à Declaração Universal dos Direitos do Homem. Brasília: 2008.
Fortium, 2008. 34 BALERA, Wagner (Coord.). Comentários à
30 BALERA, Wagner (Coord.). Comentários à Declaração Universal dos Direitos do Homem. Brasília: Fortium,
Declaração Universal dos Direitos do Homem. Brasília: Fortium, 2008.
2008. 35 BALERA, Wagner (Coord.). Comentários à
31 LENZA, Pedro. Curso de direito constitucional Declaração Universal dos Direitos do Homem. Brasília: Fortium,
esquematizado. 15. ed. São Paulo: Saraiva, 2011. 2008.

39
Artigo V Artigo VII
Ninguém será submetido à tortura, nem a tratamento Todos são iguais perante a lei e têm direito, sem
ou castigo cruel, desumano ou degradante. qualquer distinção, a igual proteção da lei. Todos têm di-
Tortura é a imposição de dor física ou psicológica reito a igual proteção contra qualquer discriminação que
por crueldade, intimidação, punição, para obtenção de viole a presente Declaração e contra qualquer incitamento a
uma confissão, informação ou simplesmente por prazer tal discriminação.
da pessoa que tortura. A tortura é uma espécie de tra- Um dos desdobramentos do princípio da igualdade
tamento ou castigo cruel, desumano ou degradante. A refere-se à igualdade perante à lei. Toda lei é dotada de
Convenção das Nações Unidas contra a Tortura e Outros caráter genérico e abstrato que evidencia não aplicar-se a
Tratamentos ou Penas Cruéis, Desumanos ou Degradan- uma pessoa determinada, mas sim a todas as pessoas que
tes (Resolução n° 39/46 da Assembleia Geral das Nações venham a se encontrar na situação por ela descrita. Não sig-
Unidas) foi estabelecida em 10 de dezembro de 1984 e nifica que a legislação não possa estabelecer, em abstrato,
ratificada pelo Brasil em 28 de setembro de 1989. Em regras especiais para um grupo de pessoas desfavorecido
destaque, o artigo 1 da referida Convenção: socialmente, direcionando ações afirmativas, por exemplo,
Artigo 1º, Convenção da ONU contra Tortura e Ou- aos deficientes, às mulheres, aos pobres - no entanto, todas
tros Tratamentos ou Penas Cruéis estas ações devem respeitar a proporcionalidade e a razoa-
1. Para os fins da presente Convenção, o termo “tor- bilidade (princípio da igualdade material).
tura” designa qualquer ato pelo qual dores ou so-
frimentos agudos, físicos ou mentais, são infligidos Artigo VIII
intencionalmente a uma pessoa a fim de obter, dela Toda pessoa tem direito a receber dos tributos nacionais
ou de uma terceira pessoa, informações ou confis- competentes remédio efetivo para os atos que violem os
sões; de castigá-la por ato que ela ou uma terceira direitos fundamentais que lhe sejam reconhecidos pela
pessoa tenha cometido ou seja suspeita de ter co- constituição ou pela lei.
metido; de intimidar ou coagir esta pessoa ou outras Não basta afirmar direitos, é preciso conferir meios
pessoas; ou por qualquer motivo baseado em discri- para garanti-los. Ciente disto, a Declaração traz aos Esta-
minação de qualquer natureza; quando tais dores ou dos-partes o dever de estabelecer em suas legislações in-
sofrimentos são infligidos por um funcionário públi-
ternas instrumentos para proteção dos direitos humanos.
co ou outra pessoa no exercício de funções públicas,
Geralmente, nos textos constitucionais são estabelecidos os
ou por sua instigação, ou com o seu consentimento
direitos fundamentais e os instrumentos para protegê-los,
ou aquiescência. Não se considerará como tortura as
por exemplo, o habeas corpus serve à proteção do direito à
dores ou sofrimentos que sejam consequência unica-
liberdade de locomoção.
mente de sanções legítimas, ou que sejam inerentes
a tais sanções ou delas decorram.
Artigo IX
2. O presente Artigo não será interpretado de ma-
neira a restringir qualquer instrumento internacional Ninguém será arbitrariamente preso, detido ou exilado.
ou legislação nacional que contenha ou possa conter Prisão e detenção são formas de impedir que a pes-
dispositivos de alcance mais amplo. soa saia de um estabelecimento sob tutela estatal, privan-
do-a de sua liberdade de locomoção. Exílio é a expulsão
Artigo VI ou mudança forçada de uma pessoa do país, sendo assim
Toda pessoa tem o direito de ser, em todos os lugares, também uma forma de privar a pessoa de sua liberdade de
reconhecida como pessoa perante a lei. locomoção em um determinado território. Nenhuma destas
“Afinal, se o Direito existe em função da pessoa hu- práticas é permitida de forma arbitrária, ou seja, sem o res-
mana, será ela sempre sujeito de direitos e de obriga- peito aos requisitos previstos em lei.
ções. Negar-lhe a personalidade, a aptidão para exercer Não significa que em alguns casos não seja aceita a pri-
direitos e contrair obrigações, equivale a não reconhecer vação de liberdade, notadamente quando o indivíduo tiver
sua própria existência. [...] O reconhecimento da perso- praticado um ato que comprometa a segurança ou outro
nalidade jurídica é imprescindível à plena realização da direito fundamental de outra pessoa.
pessoa humana. Trata-se de garantir a cada um, em to-
dos os lugares, a possibilidade de desenvolvimento livre Artigo X
e isonômico”36. Toda pessoa tem direito, em plena igualdade, a uma au-
O sistema de proteção de direitos humanos estabe- diência justa e pública por parte de um tribunal inde-
lecido no âmbito da Organização das Nações Unidas é pendente e imparcial, para decidir de seus direitos e deveres
global, razão pela qual não cabe o seu desrespeito em ou do fundamento de qualquer acusação criminal contra ele.
qualquer localidade do mundo. Por isso, um estrangeiro “De acordo com a ordem que promana do preceito
que visite outro país não pode ter seus direitos humanos acima reproduzido, as pessoas têm a faculdade de exigir
LEGISLAÇÃO ESPECÍFICA

violados, independentemente da Constituição daquele um pronunciamento do Poder Judiciário, acerca de seus


país nada prever a respeito dos direitos dos estrangeiros. direitos e deveres postos em litígio ou do fundamento de
A pessoa humana não perde tal caráter apenas por sair acusação criminal, realizado sob o amparo dos princípios
do território de seu país. Em outras palavras, denota-se da isonomia, do devido processo legal, da publicidade dos
uma das facetas do princípio da universalidade. atos processuais, da ampla defesa e do contraditório e da
imparcialidade do juiz”37.
36 BALERA, Wagner (Coord.). Comentários à
Declaração Universal dos Direitos do Homem. Brasília: Fortium, 37 BALERA, Wagner (Coord.). Comentários à
2008. Declaração Universal dos Direitos do Homem. Brasília: Fortium,

40
Em outras palavras não é possível juízo ou tribunal de Artigo XII
exceção, ou seja, um juízo especialmente delegado para Ninguém será sujeito a interferências na sua vida
o julgamento do caso daquela pessoa. O juízo deve ser privada, na sua família, no seu lar ou na sua corres-
escolhido imparcialmente, de acordo com as regras de pondência, nem a ataques à sua honra e reputação.
organização judiciária que valem para todos. Não obs- Toda pessoa tem direito à proteção da lei contra tais inter-
tante, o juízo deve ser independente, isto é, poder julgar ferências ou ataques.
independentemente de pressões externas para que o jul- A proteção aos direitos à privacidade e à personali-
gamento se dê num ou noutro sentido. O juízo também dade se enquadra na primeira dimensão de direitos fun-
deve ser imparcial, não possuindo amizade ou inimizade damentais no que tange à proteção à liberdade. Enfim,
em graus relevantes para com o acusado. Afinal, o direito o exercício da liberdade lega-se também às limitações a
à liberdade é consagrado e para que alguém possa ser este exercício: de que adianta ser plenamente livre se a
privado dela por uma condenação criminal é preciso que liberdade de um interfere na liberdade - e nos direitos
esta se dê dentro dos trâmites legais, sem violar direitos inerentes a esta liberdade - do outro.
humanos do acusado. “O direito à intimidade representa relevante manifes-
tação dos direitos da personalidade e qualifica-se como
Artigo XI expressiva prerrogativa de ordem jurídica que consiste em
1. Toda pessoa acusada de um ato delituoso tem o reconhecer, em favor da pessoa, a existência de um espa-
direito de ser presumida inocente até que a sua ço indevassável destinado a protegê-la contra indevidas
culpabilidade tenha sido provada de acordo interferências de terceiros na esfera de sua vida privada”39.
com a lei, em julgamento público no qual lhe te- Reforçando a conexão entre a privacidade e a intimida-
nham sido asseguradas todas as garantias necessá- de, ao abordar a proteção da vida privada - que, em resumo,
rias à sua defesa. é a privacidade da vida pessoal no âmbito do domicílio e
O princípio da presunção de inocência ou não culpa- de círculos de amigos -, Silva40 entende que “o segredo da
bilidade liga-se ao direito à liberdade. Antes que ocor- vida privada é condição de expansão da personalidade”, mas
ra a condenação criminal transitada em julgado, isto é, não caracteriza os direitos de personalidade em si. “O direito
processada até o último recurso interposto pelo acusado, à honra distancia-se levemente dos dois anteriores, poden-
este deve ser tido como inocente. Durante o processo do referir-se ao juízo positivo que a pessoa tem de si (hon-
penal, o acusado terá direito ao contraditório e à ampla ra subjetiva) e ao juízo positivo que dela fazem os outros
defesa, bem como aos meios e recursos inerentes a estas (honra objetiva), conferindo-lhe respeitabilidade no meio
garantias, e caso seja condenado ao final poderá ser con- social. O direito à imagem também possui duas conotações,
siderado culpado. A razão é que o estado de inocência é podendo ser entendido em sentido objetivo, com relação
inerente ao ser humano até que ele viole direito alheio, à reprodução gráfica da pessoa, por meio de fotografias, fil-
caso em que merecerá sanção. magens, desenhos, ou em sentido subjetivo, significando o
“Através desse princípio verifica-se a necessidade de conjunto de qualidades cultivadas pela pessoa e reconheci-
o Estado comprovar a culpabilidade do indivíduo presu- das como suas pelo grupo social”41.
mido inocente. Está diretamente relacionado à questão O artigo também abrange a proteção ao domicílio,
da prova no processo penal que deve ser validamente local no qual a pessoa deseja manter sua privacidade e
produzida para ao final do processo conduzir a culpabi- pode desenvolver sua personalidade; e à correspondên-
lidade do indivíduo admitindo-se a aplicação das penas cia, enviada ao seu lar unicamente para sua leitura e não
previamente cominadas. Entretanto, a presunção de ino- de terceiros, preservando-se sua privacidade.
cência não afasta a possibilidade de medidas cautelares
como as prisões provisórias, busca e apreensão, quebra Artigo XIII
de sigilo como medidas de caráter excepcional cujos re- 1. Toda pessoa tem direito à liberdade de locomoção
quisitos autorizadores devem estar previstos em lei”38. e residência dentro das fronteiras de cada Estado.
2. Ninguém poderá ser culpado por qualquer ação ou Não há limitações ao direito de locomoção dentro do
omissão que, no momento, não constituíam deli- próprio Estado, nem ao direito de residir. Vale lembrar que
to perante o direito nacional ou internacional. a legislação interna pode estabelecer casos em que tal di-
Tampouco será imposta pena mais forte do que reito seja relativizado, por exemplo, obrigando um funcio-
aquela que, no momento da prática, era aplicá- nário público a residir no município em que está sediado
vel ao ato delituoso. ou impedindo o ingresso numa área de interesse estatal.
Evidencia-se o princípio da irretroatividade da lei São exceções à liberdade de locomoção: decisão judicial
penal in pejus (para piorar a situação do acusado) pelo que imponha pena privativa de liberdade ou limitação da li-
qual uma lei penal elaborada posteriormente não pode berdade, normas administrativas de controle de vias e veícu-
se aplicar a atos praticados no passado - nem para um los, limitações para estrangeiros em certas regiões ou áreas
LEGISLAÇÃO ESPECÍFICA

ato que não era considerado crime passar a ser, nem para de segurança nacional e qualquer situação em que o direito
que a pena de um ato que era considerado crime seja à liberdade deva ceder aos interesses públicos42.
aumentada. Evidencia não só o respeito à liberdade, mas 39 MOTTA, Sylvio; BARCHET, Gustavo. Curso
também - e principalmente - à segurança jurídica. de direito constitucional. Rio de Janeiro: Elsevier, 2007.
40 SILVA, José Afonso da. Curso de direito
2008. constitucional positivo. 25. ed. São Paulo: Malheiros, 2006.
38 BALERA, Wagner (Coord.). Comentários à 41 MOTTA, Sylvio; BARCHET, Gustavo. Curso
Declaração Universal dos Direitos do Homem. Brasília: Fortium, de direito constitucional. Rio de Janeiro: Elsevier, 2007.
2008. 42 BALERA, Wagner (Coord.). Comentários à

41
2. Toda pessoa tem o direito de deixar qualquer país, padrões da migração se tornaram cada vez mais com-
inclusive o próprio, e a este regressar. plexos nos tempos modernos, envolvendo não apenas
A nacionalidade é um direito humano, assim como refugiados, mas também milhões de migrantes econô-
a liberdade de locomoção. Destaca-se que o artigo não micos. Mas refugiados e migrantes, mesmo que viajem
menciona o direito de entrar em qualquer país, mas sim da mesma forma com frequência, são fundamentalmente
o de deixá-lo. distintos, e por esta razão são tratados de maneira muito
diferente perante o direito internacional moderno.
Artigo XIV Migrantes, especialmente migrantes econômicos, de-
1.Toda pessoa, vítima de perseguição, tem o direito de cidem deslocar-se para melhorar as perspectivas para si
procurar e de gozar asilo em outros países. mesmos e para suas famílias. Já os refugiados necessitam
2. Este direito não pode ser invocado em caso de per- deslocar-se para salvar suas vidas ou preservar sua liber-
seguição legitimamente motivada por crimes de dade. Eles não possuem proteção de seu próprio Estado
direito comum ou por atos contrários aos propósitos e de fato muitas vezes é seu próprio governo que amea-
e princípios das Nações Unidas. ça persegui-los. Se outros países não os aceitarem em
O direito de asilo serve para proteger uma pessoa seus territórios, e não os auxiliarem uma vez acolhidos,
perseguida por suas opiniões políticas, situação racial, poderão estar condenando estas pessoas à morte ou à
convicções religiosas ou outro motivo político em seu uma vida insuportável nas sombras, sem sustento e sem
país de origem, permitindo que ela requeira perante a direitos”43.
autoridade de outro Estado proteção. Claro, não se pro- As Nações Unidas44 descrevem sua participação no
tege aquele que praticou um crime comum em seu país histórico do direito dos refugiados no mundo:
e fugiu para outro, caso em que deverá ser extraditado “Desde a sua criação, a Organização das Nações Uni-
para responder pelo crime praticado. das tem dedicado os seus esforços à proteção dos re-
O direito dos refugiados é o que envolve a garantia fugiados no mundo. Em 1951, data em que foi criado o
de asilo fora do território do qual é nacional por algum Alto Comissariado das Nações Unidas para os Refugia-
dos motivos especificados em normas de direitos hu- dos (ACNUR), havia um milhão de refugiados sob a sua
manos, notadamente, perseguição por razões de raça, responsabilidade. Hoje este número aumentou para 17,5
religião, nacionalidade, pertença a um grupo social de- milhões, para além dos 2,5 milhões assistidos pelo Orga-
terminado ou convicções políticas. Diversos documentos nismo das Nações Unidas das Obras Públicas e Socorro
internacionais disciplinam a matéria, a exemplo da De- aos Refugiados da Palestina, no Próximo Oriente (ANUA-
claração Universal de 1948, Convenção de 1951 relativa TP), e ainda mais de 25 milhões de pessoas deslocadas
ao Estatuto dos Refugiados, Quarta Convenção de Ge- internamente. Em 1951, a maioria dos refugiados eram
nebra Relativa à Proteção das Pessoas Civis em Tempo Europeus. Hoje, a maior parte é proveniente da África e
de Guerra de 1949, Convenção relativa ao Estatuto dos da Ásia. Atualmente, os movimentos de refugiados assu-
Apátridas de 1954, Convenção sobre a Redução da Apa- mem cada vez mais a forma de êxodos maciços, diferen-
tridia de 1961 e Declaração das Nações Unidas sobre a temente das fugas individuais do passado. Hoje, oitenta
Concessão de Asilo Territorial de 1967. Não obstante, a por cento dos refugiados são mulheres e crianças. Tam-
constituição brasileira adota a concessão de asilo político bém as causas dos êxodos se multiplicaram, incluindo
como um de seus princípios nas relações internacionais agora as catástrofes naturais ou ecológicas e a extrema
(art. 4º, X, CF). pobreza. Daí que muitos dos atuais refugiados não se en-
“A prática de conceder asilo em terras estrangeiras a quadrem na definição da Convenção relativa ao Estatuto
pessoas que estão fugindo de perseguição é uma das dos Refugiados. Esta Convenção refere-se a vítimas de
características mais antigas da civilização. Referências perseguição por razões de raça, religião, nacionalidade,
a essa prática foram encontradas em textos escritos há pertença a um grupo social determinado ou convicções
3.500 anos, durante o florescimento dos antigos grandes políticas. [...]
impérios do Oriente Médio, como o Hitita, Babilônico, Existe uma relação evidente entre o problema dos re-
Assírio e Egípcio antigo. fugiados e a questão dos direitos humanos. As violações
Mais de três milênios depois, a proteção de refugia- dos direitos humanos constituem não só uma das princi-
dos foi estabelecida como missão principal da agência pais causas dos êxodos maciços, mas afastam também a
de refugiados da ONU, que foi constituída para assistir, opção do repatriamento voluntário enquanto se verifica-
entre outros, os refugiados que esperavam para retornar rem. As violações dos direitos das minorias e os conflitos
aos seus países de origem no final da II Guerra Mundial. étnicos encontram-se cada vez mais na origem quer dos
A Convenção de Refugiados de 1951, que estabele- êxodos maciços, quer das deslocações internas. [...]
ceu o ACNUR, determina que um refugiado é alguém Na sua segunda sessão, no final de 1946, a Assem-
que ‘temendo ser perseguida por motivos de raça, re- bleia Geral criou a Organização Internacional para os
Refugiados (OIR), que assumiu as funções da Agência
LEGISLAÇÃO ESPECÍFICA

ligião, nacionalidade, grupo social ou opiniões políticas,


se encontra fora do país de sua nacionalidade e que não das Nações Unidas para a Assistência e a Reabilitação
pode ou, em virtude desse temor, não quer valer-se da (ANUAR). Foi investida no mandato temporário de regis-
proteção desse país’. trar, proteger, instalar e repatriar refugiados. [...] Cedo se
Desde então, o ACNUR tem oferecido proteção e 43 http://www.acnur.org/t3/portugues/a-
assistência para dezenas de milhões de refugiados, en- quem-ajudamos/refugiados/
contrando soluções duradouras para muitos deles. Os 44 ORGANIZAÇÃO DAS NAÇÕES UNIDAS -
ONU. Direitos Humanos e Refugiados. Ficha normativa nº 20.
Declaração Universal dos Direitos do Homem. Brasília: Fortium, Disponível em: <http://www.gddc.pt/direitos-humanos/Ficha_
2008. Informativa_20.pdf >. Acesso em: 13 jun. 2013.

42
tornou evidente que a responsabilidade pelos refugiados Artigo XVII
merecia um maior esforço da comunidade internacional, 1. Toda pessoa tem direito à propriedade, só ou em
a desenvolver sob os auspícios da própria Organização sociedade com outros.
das Nações Unidas. Assim, muito antes de terminar o 2. Ninguém será arbitrariamente privado de sua
mandato da OIR, iniciaram-se as discussões sobre a cria- propriedade.
ção de uma organização que lhe pudesse suceder. “Toda pessoa [...] tem direito à propriedade, podendo
O Alto Comissariado das Nações Unidas para os Refu- o ordenamento jurídico estabelecer suas modalidades de
giados (ACNUR) Na sua Resolução 319 A (IV) de 3 de De- aquisição, perda, uso e limites. O direito de propriedade,
zembro de 1949, a Assembleia Geral decidiu criar o Alto constitucionalmente assegurado, garante que dela nin-
Comissariado das Nações Unidas para os Refugiados. O guém poderá ser privado arbitrariamente [...]”46. O direito
Alto Comissariado foi instituído em 1 de Janeiro de 1951, à propriedade se insere na primeira dimensão de direitos
como órgão subsidiário da Assembleia Geral, com um humanos, garantindo que cada qual tenha bens materiais
mandato inicial de três anos. Desde então, o mandato do justamente adquiridos, respeitada a função social.
ACNUR tem sido renovado por períodos sucessivos de
cinco anos [...]”. Artigo XVIII
Toda pessoa tem direito à liberdade de pensamento,
Artigo XV consciência e religião; este direito inclui a liberdade de
1. Toda pessoa tem direito a uma nacionalidade. mudar de religião ou crença e a liberdade de manifestar
2. Ninguém será arbitrariamente privado de sua na- essa religião ou crença, pelo ensino, pela prática, pelo cul-
cionalidade, nem do direito de mudar de naciona- to e pela observância, isolada ou coletivamente, em públi-
lidade. co ou em particular.
Nacionalidade é o vínculo jurídico-político que liga Silva47 aponta que a liberdade de pensamento, que
um indivíduo a determinado Estado, fazendo com que também pode ser chamada de liberdade de opinião, é
ele passe a integrar o povo daquele Estado, desfrutando considerada pela doutrina como a liberdade primária, eis
assim de direitos e obrigações. Não é aceita a figura do que é ponto de partida de todas as outras, e deve ser en-
apátrida ou heimatlos, o indivíduo que não possui ne- tendida como a liberdade da pessoa adotar determinada
nhuma nacionalidade. atitude intelectual ou não, de tomar a opinião pública
É possível mudar de nacionalidade nas situações pre- que crê verdadeira. Tal opinião pública se refere a diver-
vistas em lei, naturalizando-se como nacional de outro sos aspectos, entre eles religião e crença.
Estado que não aquele do qual originalmente era nacio- A liberdade de religião atrela-se à liberdade de cons-
nal. Geralmente, a permanência no território do pais por ciência e à liberdade de pensamento, mas o inverso não
um longo período de tempo dá direito à naturalização, ocorre, porque é possível existir liberdade de pensamen-
abrindo mão da nacionalidade anterior para incorporar to e consciência desvinculada de cunho religioso. Aliás, a
a nova. liberdade de consciência também concretiza a liberdade
de ter ou não ter religião, ter ou não ter opinião político-
Artigo XVI -partidária ou qualquer outra manifestação positiva ou
1. Os homens e mulheres de maior idade, sem qual- negativa da consciência48.
quer restrição de raça, nacionalidade ou religião, No que tange à exteriorização da liberdade de reli-
têm o direito de contrair matrimônio e fundar gião, ou seja, à liberdade de expressão religiosa, não é
uma família. Gozam de iguais direitos em relação devida nenhuma perseguição, assim como é garantido o
ao casamento, sua duração e sua dissolução. direito de praticá-la em grupo ou individualmente.
2. O casamento não será válido senão com o livre e
pleno consentimento dos nubentes. Artigo XIX
Toda pessoa tem direito à liberdade de opinião e
O casamento, como todas as instituições sociais, varia expressão; este direito inclui a liberdade de, sem inter-
com o tempo e os povos, que evoluem e adquirem novas ferência, ter opiniões e de procurar, receber e transmitir
culturas. Há quem o defina como um ato, outros como informações e ideias por quaisquer meios e independente-
um contato. Basicamente, casamento é a união, devida- mente de fronteiras.
mente formalizada conforme a lei, com a finalidade de Silva49 entende que a liberdade de expressão pode
construir família. A principal finalidade do casamento é ser vista sob diversos enfoques, como o da liberdade de
estabelecer a comunhão plena de vida, impulsionada comunicação, ou liberdade de informação, que consiste
pelo amor e afeição existente entre o casal e baseada na em um conjunto de direitos, formas, processos e veículos
igualdade de direitos e deveres dos cônjuges e na mútua que viabilizam a coordenação livre da criação, expressão
assistência.45 Não é aceitável o casamento que se esta-
LEGISLAÇÃO ESPECÍFICA

46 MORAES, Alexandre de. Direitos humanos


beleça à força para algum dos nubentes, sendo exigido fundamentais: teoria geral, comentários aos artigos 1º a 5º
o livre e pleno consentimento de ambos. Não obstante, da Constituição da República Federativa do Brasil, doutrina e
é coerente que a lei traga limitações como a idade, pois jurisprudência. São Paulo: Atlas, 1997.
o casamento é uma instituição séria, base da família, e 47 SILVA, José Afonso da. Curso de direito
somente a maturidade pode permitir compreender tal constitucional positivo. 25. ed. São Paulo: Malheiros, 2006.
importância. 48 BALERA, Wagner (Coord.). Comentários à
Declaração Universal dos Direitos do Homem. Brasília: Fortium, 2008.
45 GONÇALVES, Carlos Roberto. Direito civil 49 SILVA, José Afonso da. Curso de direito
brasileiro. 6. ed. São Paulo: Saraiva, 2009. v. 6. constitucional positivo. 25. ed. São Paulo: Malheiros, 2006.

43
e difusão da informação e do pensamento. Contudo, o a Não obstante, se introduz a dimensão do Estado So-
manifestação do pensamento não pode ocorrer de forma cial, de forma que ao cidadão é garantida a prestação
ilimitada, devendo se pautar na verdade e no respeito de serviços públicos. Isto se insere na segunda dimensão
dos direitos à honra, à intimidade e à imagem dos de- de direitos humanos, referentes aos direitos econômi-
mais membros da sociedade. cos, sociais e culturais - sem os quais não se consolida a
igualdade material.
Artigo XX
1. Toda pessoa tem direito à liberdade de reunião e Artigo XXII
associação pacíficas. Toda pessoa, como membro da sociedade, tem direito
O direito de reunião pode ser exercido independente- à segurança social e à realização, pelo esforço nacional,
mente de autorização estatal, mas deve se dar de manei- pela cooperação internacional e de acordo com a organi-
ra pacífica, por exemplo, sem utilização de armas. zação e recursos de cada Estado, dos direitos econômi-
2. Ninguém pode ser obrigado a fazer parte de uma cos, sociais e culturais indispensáveis à sua dignidade e
associação. ao livre desenvolvimento da sua personalidade.
Por sua vez, “a liberdade de associação para fins lí- Direitos econômicos, sociais e culturais compõem a
citos, vedada a de caráter paramilitar, é plena. Portanto, segunda dimensão de direitos fundamentais. O Pacto
ninguém poderá ser compelido a associar-se e, uma vez internacional de Direitos Econômicos, Sociais e Cultu-
associado, será livre, também, para decidir se permanece rais de 1966 é o documento que especifica e descreve
associado ou não”50. tais direitos. de uma maneira geral, são direitos que não
dependem puramente do indivíduo para a implementa-
Artigo XXI ção, exigindo prestações positivas estatais, geralmente
1. Toda pessoa tem o direito de tomar parte no gover- externadas por políticas públicas (escolhas políticas a
no de seu país, diretamente ou por intermédio de respeito de áreas que necessitam de investimento maior
representantes livremente escolhidos. ou menos para proporcionar um bom índice de desen-
2. Toda pessoa tem igual direito de acesso ao serviço volvimento social, diminuindo desigualdades). Entre ou-
público do seu país. tros direitos, envolvem o trabalho, a educação, a saúde, a
3. A vontade do povo será a base da autoridade alimentação, a moradia, o lazer, etc. Como são inúmeras
as áreas que necessitam de investimento estatal, natural-
do governo; esta vontade será expressa em eleições
mente o atendimento a estes direitos se dá de maneira
periódicas e legítimas, por sufrágio universal, por
gradual.
voto secreto ou processo equivalente que assegure
a liberdade de voto.
Artigo XXIII
“Na sociedade moderna, nascida de transformações
1. Toda pessoa tem direito ao trabalho, à livre esco-
que culminaram na Revolução Francesa, o indivíduo é vis-
lha de emprego, a condições justas e favoráveis
to como homem (pessoa privada) e como cidadão (pes-
de trabalho e à proteção contra o desemprego.
soa pública). O termo cidadão designava originalmente o
2. Toda pessoa, sem qualquer distinção, tem direito a
habitante da cidade. Com a consolidação da sociedade igual remuneração por igual trabalho.
burguesa, passa a indicar a ação política e a participação 3. Toda pessoa que trabalhe tem direito a uma remu-
do sujeito na vida da sociedade”51. neração justa e satisfatória, que lhe assegure, as-
Democracia (do grego, demo+kratos) é um regime de sim como à sua família, uma existência compatível
governo em que o poder de tomar decisões políticas está com a dignidade humana, e a que se acrescentarão,
com os cidadãos, de forma direta (quando um cidadão se se necessário, outros meios de proteção social.
reúne com os demais e, juntos, eles tomam a decisão po- 4. Toda pessoa tem direito a organizar sindicatos e
lítica) ou indireta (quando ao cidadão é dado o poder de neles ingressar para proteção de seus interesses.
eleger um representante). Uma democracia pode existir O trabalho é um instrumento fundamental para as-
num sistema presidencialista ou parlamentarista, republi- segurar a todos uma existência digna: de um lado por
cano ou monárquico - somente importa que seja dado proporcionar a remuneração com a qual a pessoa ad-
aos cidadãos o poder de tomar decisões políticas (por si quirirá bens materiais para sua subsistência, de outro
só ou por seu representante eleito), nos termos que este por gerar por si só o sentimento de importância para a
artigo da Declaração prevê. A principal classificação das sociedade por parte daquele que faz algo útil nela. No
democracias é a que distingue a direta da indireta - a) entanto, a geração de empregos não se dá automati-
direta, também chamada de pura, na qual o cidadão ex- camente, cabendo aos Estados desenvolverem políticas
pressa sua vontade por voto direto e individual em casa econômicas para diminuir os índices de desemprego o
questão relevante; b) indireta, também chamada repre- máximo possível.
LEGISLAÇÃO ESPECÍFICA

sentativa, em que os cidadãos exercem individualmente A remuneração é a retribuição financeira pelo traba-
o direito de voto para escolher representante(s) e aque- lho realizado. Nesta esfera também é necessário o res-
le(s) que for(em) mais escolhido(s) representa(m) todos peito ao princípio da igualdade, por não ser justo que
os eleitores. uma pessoa que desempenhe as mesmas funções que a
50 LENZA, Pedro. Curso de direito constitucional outra receba menos por um fator externo, característico
esquematizado. 15. ed. São Paulo: Saraiva, 2011. dela, como sexo ou raça. No âmbito do serviço público
51 SCHLESENER, Anita Helena. Cidadania e é mais fácil controlar tal aspecto, mas são inúmeras as
política. In: CARDI, Cassiano; et. al. Para filosofar. São Paulo:
empresas privadas que pagam menor salário a mulheres
Scipione, 2000.

44
e que não chegam a ser levadas à justiça por isso. Não 2. A maternidade e a infância têm direito a cuidados
obstante, a remuneração deve ser suficiente para propor- e assistência especiais. Todas as crianças nascidas
cionar uma existência digna, com o necessário para man- dentro ou fora do matrimônio, gozarão da mesma
ter assegurados ao menos minimamente todos os direitos proteção social.
humanos previstos na Declaração. A proteção da maternidade tem sentido porque sem
Os sindicatos são bastante comuns na seara trabalhis- isto o mundo não continua. É preciso que as crianças se-
ta e, como visto, a todos é garantida a liberdade de asso- jam protegidas com atenção especial para que se tornem
ciação, não podendo ninguém ser impedido ou forçado a adultos capazes de proporcionar uma melhora no plane-
ingressar ou sair de um sindicato. ta.

Artigo XXIV Artigo XXVI


Toda pessoa tem direito a repouso e lazer, inclusive a 1. Toda pessoa tem direito à instrução. A instrução
limitação razoável das horas de trabalho e férias pe- será gratuita, pelo menos nos graus elementares e
riódicas remuneradas. fundamentais. A instrução elementar será obrigató-
Por mais que o trabalho seja um direito humano, nem ria. A instrução técnico-profissional será acessível a
somente dele é feita a vida de uma pessoa. Desta forma, todos, bem como a instrução superior, esta baseada
assegura-se horários livres para que a pessoa desfrute de no mérito.
momentos de lazer e descanso, bem como impede-se a 2. A instrução será orientada no sentido do pleno
fixação de uma jornada de trabalho muito exaustiva. São desenvolvimento da personalidade humana e do
medidas que asseguram isto a previsão de descanso se- fortalecimento do respeito pelos direitos humanos
manal remunerado, a limitação do horário de trabalho, e pelas liberdades fundamentais. A instrução pro-
a concessão de férias remuneradas anuais, entre outras. moverá a compreensão, a tolerância e a amizade
Quanto aos artigos XXIII e XXIV, tem-se que é forneci- entre todas as nações e grupos raciais ou religiosos,
do “[...] um conjunto mínimo de direitos dos trabalhado- e coadjuvará as atividades das Nações Unidas em
res. De forma geral, os dispositivos em comento versam prol da manutenção da paz.
sobre o direito ao trabalho, principal meio de sobrevivên- 3. Os pais têm prioridade de direito na escolha do gê-
nero de instrução que será ministrada a seus filhos.
cia dos indivíduos que ‘vendem’ força de trabalho em tro-
A Declaração Universal de 1948 divide a disponibili-
ca de uma remuneração justa. Ademais, estabelecem a
dade e a obrigatoriedade da educação em níveis. Aquela
liberdade do cidadão de escolher o trabalho e, uma vez
educação que é considerada essencial, qual seja, a ele-
obtido o emprego, o direito de nele encontrar condições
mentar, deve ser gratuita e obrigatória. Já a educação
justas, tanto no tocante à remuneração, como no que diz
fundamental, de grande importância, deve ser gratuita,
respeito ao limite de horas trabalhadas e períodos de re-
mas não é obrigatória. Esta nomenclatura adotada pela
pouso (disposição constante do artigo XXIV da Declara-
Declaração equipara-se ao ensino fundamental e ao en-
ção). Garantem ainda o direito dos trabalhadores de se sino médio no Brasil, sendo elementar o primeiro e fun-
unirem em associação, com o objetivo de defesa de seus damental o segundo. A educação técnico-profissional
interesses”52. refere-se às escolas voltadas ao ensino de algum ofício,
não complexo a ponto de exigir formação superior e,
Artigo XXV justamente por isso, possuem menor duração e menor
1. Toda pessoa tem direito a um padrão de vida capaz custo; ao passo que a educação superior é a que se dá
de assegurar a si e a sua família saúde e bem estar, no âmbito das universidades, formando profissionais de
inclusive alimentação, vestuário, habitação, cuida- maior especialidade numa área profissional, com amplo
dos médicos e os serviços sociais indispensáveis, e conhecimento, razão pela qual dura mais tempo e é mais
direito à segurança em caso de desemprego, doença, onerosa. As duas últimas são de maior custo e não podem
invalidez, viuvez, velhice ou outros casos de perda ser instituídas de tal forma que sejam garantidas vagas
dos meios de subsistência fora de seu controle. para todas as pessoas em sociedade, entretanto, exige-se
O ideal é que todas as pessoas possuam um padrão um critério justo para a seleção dos ingressos, o qual seja
de vida suficiente para garantir sua dignidade em todas as baseado no mérito (os mais capacitados conseguirão as
esferas: alimentação, vestuário, moradia, saúde, etc. Bem vagas de ensino técnico-profissional e superior).
se sabe que é um objetivo constante do Estado Democrá- Ainda, a Declaração de 1948 deixa clara que a educa-
tico de Direito proporcionar que pessoas cheguem o mais ção não envolve apenas o aprendizado do conteúdo pro-
próximo possível - e cada vez mais - desta circunstância. gramático das matérias comuns como matemática, por-
Fala-se em segurança no sentido de segurança públi- tuguês, história e geografia, mas também a compreensão
ca, de dever do Estado de preservar a ordem pública e de abordagens sobre assuntos que possam contribuir
a incolumidade das pessoas e do patrimônio público e para a formação da personalidade da pessoa humana e
LEGISLAÇÃO ESPECÍFICA

privado53. Neste conceito enquadra-se a seguridade so- conscientizá-la de seu papel social.
cial, na qual o Estado, custeado pela coletividade e pelos Não obstante, da parte final da Declaração extrai-se
cofres públicos, garante a manutenção financeira dos que a consciência de que a educação não é apenas a formal,
por algum motivo não possuem condição de trabalhar. aprendida nos estabelecimentos de ensino, mas também
52 BALERA, Wagner (Coord.). Comentários à a informal, transmitida no ambiente familiar e nas demais
Declaração Universal dos Direitos do Homem. Brasília: Fortium, áreas de contato da pessoa, como igreja, clubes e, no-
2008. tadamente, a residência. Por isso, os pais têm um papel
53 LENZA, Pedro. Curso de direito constitucional direto na escolha dos meios de educação de seus filhos.
esquematizado. 15. ed. São Paulo: Saraiva, 2011.

45
Artigo XXVII 3. Esses direitos e liberdades não podem, em hipótese
1. Toda pessoa tem o direito de participar livremente alguma, ser exercidos contrariamente aos propósi-
da vida cultural da comunidade, de fruir as artes tos e princípios das Nações Unidas.
e de participar do processo científico e de seus be- Explica Canotilho57 que “a ideia de deveres funda-
nefícios. mentais é suscetível de ser entendida como o ‘outro
2. Toda pessoa tem direito à proteção dos interesses lado’ dos direitos fundamentais. Como ao titular de um
morais e materiais decorrentes de qualquer produ- direito fundamental corresponde um dever por parte de
ção científica, literária ou artística da qual seja autor. um outro titular, poder-se-ia dizer que o particular está
Os conflitos que se dão entre a liberdade e a proprie- vinculado aos direitos fundamentais como destinatário
dade intelectual se evidenciam, principalmente, sob o de um dever fundamental. Neste sentido, um direito
aspecto da liberdade de expressão, na esfera específica fundamental, enquanto protegido, pressuporia um de-
da liberdade de comunicação ou informação, que, nos di- ver correspondente”. Esta é a ideia que a Declaração de
zeres de Silva54, “compreende a liberdade de informar e 1948 busca trazer: não será assegurada nenhuma liber-
a liberdade de ser informado”. Sob o enfoque do direito dade que contrarie a lei ou os demais direitos de outras
à liberdade e do direito de acesso à cultura, seria livre pessoas, isto é, os preceitos universais consagrados pelas
a divulgação de toda e qualquer informação e o acesso Nações Unidas.
aos dados disponíveis, independentemente da fonte ou
da autoria. De outro lado, há o direito de propriedade in- Artigo XXX
telectual, o qual possui um caráter dualista: moral, que Nenhuma disposição da presente Declaração pode ser
nunca prescreve porque o autor de uma obra nunca dei- interpretada como o reconhecimento a qualquer Estado,
xará de ser considerado como tal, e patrimonial, que pres- grupo ou pessoa, do direito de exercer qualquer ativida-
creve, perdendo o autor o direito de explorar benefícios de ou praticar qualquer ato destinado à destruição de
econômicos de sua obra55. Cada vez mais esta dualidade quaisquer dos direitos e liberdades aqui estabelecidos.
entre direitos se encontra em conflito, uma vez que a evo- “A colidência entre os direitos afirmados na Declara-
lução tecnológica trouxe meios para a cópia em massa de ção é natural. Busca-se com o presente artigo evitar que,
conteúdos protegidos pela propriedade intelectual. no eventual choque entre duas normas garantistas, os
sujeitos nela mencionados se valham de uma interpre-
Artigo XVIII tação tendente a infirmar qualquer das disposições da
Toda pessoa tem direito a uma ordem social e inter- Declaração ao argumento de que estão respeitando um
nacional em que os direitos e liberdades estabelecidos na direito em detrimento de outro”58. Nenhum direito hu-
presente Declaração possam ser plenamente realizados. mano é ilimitado: se o fossem, seria impossível garantir
Como já destacado, o sistema de proteção dos direi- um sistema no qual todas as pessoas tivessem tais direi-
tos humanos tem caráter global e cada Estado que as- tos plenamente respeitados, afinal, estes necessariamen-
sumiu compromisso perante a ONU ao integrá-la deve te colidiriam com os direitos das outras pessoas, os quais
garantir o respeito a estes direitos no âmbito de seu ter- teriam que ser violados. Este é um dos sentidos do princí-
ritório. Com isso, a pessoa estará numa ordem social e in- pio da relatividade dos direitos humanos - os direitos hu-
ternacional na qual seus direitos humanos sejam assegu- manos não podem ser utilizados como um escudo para
rados, preservando-se sua dignidade. Em outras palavras, práticas ilícitas ou como argumento para afastamento ou
“devidamente emparelhadas, portanto, a ordem social e diminuição da responsabilidade por atos ilícitos, assim os
a ordem internacional se manifestam, a seu modo, como direitos humanos não são ilimitados e encontram seus li-
as duas faces das instituições humanitárias, tanto estatais mites nos demais direitos igualmente consagrados como
quanto particulares, orientando seus passos a serviço da humanos. Isto vale tanto para os indivíduos, numa atitu-
comunidade humana”56. de perante os demais, quanto para os Estados, ao exter-
nar o compromisso global assumido perante a ONU.
Artigo XXIX
1. Toda pessoa tem deveres para com a comunidade,
em que o livre e pleno desenvolvimento de sua per-
#FicaDica
sonalidade é possível. O direito de reunião deve ser pacífico e poderá
2. No exercício de seus direitos e liberdades, toda pessoa sofrer restrições previstas em lei. O intuito é
estará sujeita apenas às limitações determinadas preservação da segurança nacional e da ordem
pela lei, exclusivamente com o fim de assegurar o pública.
devido reconhecimento e respeito dos direitos e li-
berdades de outrem e de satisfazer às justas exigên-
cias da moral, da ordem pública e do bem-estar de
LEGISLAÇÃO ESPECÍFICA

uma sociedade democrática.


54 SILVA, José Afonso da. Curso de direito
constitucional positivo. 25. ed. São Paulo: Malheiros, 2006.
55 PAESANI, Liliana Minardi. Direito e Internet: 57 CANOTILHO, José Joaquim Gomes.
liberdade de informação, privacidade e responsabilidade civil. Direito constitucional e teoria da constituição. 2. ed. Coimbra:
3. ed. São Paulo: Atlas, 2006. Almedina, 1998.
56 BALERA, Wagner (Coord.). Comentários à 58 BALERA, Wagner (Coord.). Comentários à
Declaração Universal dos Direitos do Homem. Brasília: Fortium, Declaração Universal dos Direitos do Homem. Brasília: Fortium,
2008. 2008.

46
c) Assistência (princípios 10 a 14): aproveitar os bene-
LEI ORDINÁRIA Nº 10.741, DE 1º DE fícios dos cuidados e da proteção da família e da comu-
OUTUBRO DE 2003. (ESTATUTO DO IDOSO); nidade, conforme os valores culturais de cada sociedade;
DOS DIREITOS FUNDAMENTAIS; DAS acesso a cuidados de saúde que o garantam bem-estar
MEDIDAS DE PROTEÇÃO; DOS CRIMES; físico, mental e emocional, inclusive preventivos; utiliza-
ção de meios adequados de assistência em âmbito insti-
tucional em prol de proteção, reabilitação e estimulação
social e mental num ambiente humano e seguro; acesso
O tratamento do idoso no campo dos direitos a serviços jurídicos; gozo dos direitos humanos e liberda-
humanos des fundamentais onde quer que se encontrem (lar pró-
Os idosos costumam ser afastados do convívio social prio ou instituições de assistência), com pleno respeito
à medida em que adquirem mais dificuldades de saú-
de sua dignidade e do seu direito de decidir seu próprio
de, naturais ao processo de envelhecimento. A lógica da
destino;
“descartabilidade” do ser humano e da produtividade em
d) Realização pessoal (princípios 15 e 16): possibilida-
massa que teima em persistir na sociedade leva à intensi-
ficação deste movimento de rejeição, cabendo ao Direito de de procurar oportunidades com vista ao pleno desen-
assumir uma postura de promoção da igualdade material volvimento do seu potencial; acesso aos recursos edu-
dos idosos, por medidas judiciais, legislativas e executivas cativos, culturais, espirituais e recreativos da sociedade;
pertinentes. e) Dignidade (princípios 17 e 18): possibilidade de vi-
No plano da proteção internacional dos direitos huma- ver com dignidade e segurança, sem serem explorados
nos deste grupo de pessoas, já ocorreu algum avanço, mas ou maltratados física ou mentalmente; tratamento de
este é ainda distante se comparado ao sistema de proteção forma justa, independentemente da sua idade, gênero,
de outros grupos vulneráveis específicos, como mulheres e origem racial ou étnica, deficiência ou outra condição, e
pessoas portadoras de deficiência. sendo devidamente valorizados.
Em relação à ONU, ainda não há Convenção específica Por sua vez, no âmbito regional, destaca-se a men-
de proteção, mas apenas normativas principiológicas não ção do Protocolo à Convenção Americana sobre Direi-
diretamente coativas, que podem ser combinadas com tos Humanos: “Artigo 18 - Proteção de pessoas idosas,
normas genéricas como as dos Pactos Internacionais de PCADH. Toda pessoa tem direito à proteção especial na
1966. Neste sentido, de forma mais relevante, em 1991 so- velhice. Nesse sentido, os Estados Partes comprometem
brevieram os Princípios das Nações Unidas para as Pessoas se a adotar de maneira progressiva as medidas necessá-
Idosas; e em 2002, na II Conferência Internacional de Madri rias a fim de pôr em prática este direito e, especialmente,
sobre o Envelhecimento, surgiram a Declaração Política e o a: a) Proporcionar instalações adequadas, bem como ali-
Plano de Ação Internacional de Madri sobre Envelhecimen- mentação e assistência médica especializada, às pessoas
to (MIPAA), estes de ordem um pouco mais pragmática. de idade avançada que careçam delas e não estejam em
Em janeiro de 2010, o Comitê Consultivo do Conselho condições de provê-las por seus próprios meios; b) Exe-
de Direitos Humanos das Nações Unidas publicou estudo cutar programas trabalhistas específicos destinados a
apontando a necessidade de uma convenção internacional dar a pessoas idosas a possibilidade de realizar atividade
específica, o que indica que futuramente é possível que tal produtiva adequada às suas capacidades, respeitando
documento seja elaborado e ratificado pelos países-mem- sua vocação ou desejos; c) Promover a formação de or-
bros da ONU.
ganizações sociais destinadas a melhorar a qualidade de
Vale colacionar alguns trechos dos Princípios das Na-
vida das pessoas idosas”.
ções Unidas para as Pessoas Idosas, de ordem mais genéri-
Ainda, a OEA aprovou em 15 de junho de 2015 a
ca, identificando as pretendidas políticas da ONU em favor
deste grupo vulnerável ao dividir em categorias as esferas Convenção Interamericana sobre a Proteção dos Direitos
de proteção específica a ser conferida: Humanos das Pessoas Idosas, ainda não incorporada ao
a) Independência (princípios 1 a 6): acesso à alimen- ordenamento interno brasileiro. Logo, a proteção especí-
tação, água, alojamento, vestuário e cuidados de saúde fica destes direitos avançou substancialmente no ano de
adequados, recebendo recursos para tanto (apoio do Es- 2015 dentro do sistema interamericano, primeiro sistema
tado, da comunidade e da família); direito ao trabalho ou a a criar um tratado internacional específico para o resguar-
outro rendimento; direito de decidir junto com a socieda- de das pessoas idosas. Conforme o disposto no primeiro
de quando é o momento de deixar de ter uma vida ativa artigo do documento: “O objetivo da Convenção é pro-
de participação social e política, dedicando-se ao repouso mover, proteger e assegurar o reconhecimento e o pleno
com dignidade; acesso a programas de educação e forma- gozo e exercício, em condições de igualdade, de todos
ção; convivência em ambientes seguros e adaptáveis às os direitos humanos e liberdades fundamentais do idoso,
suas necessidades e preferências; residência em seu pró- a fim de contribuir para sua plena inclusão, integração e
LEGISLAÇÃO ESPECÍFICA

prio domicílio tanto quanto possível; participação na sociedade”. Frisa-se, ademais, no mesmo
b) Participação (princípios 7 a 9): permanência na inte- dispositivo, que o disposto em tal Convenção Interame-
gração em sociedade, participando na formulação e execu- ricana não deve ser interpretado como uma limitação a
ção de políticas que o afetem diretamente, compartilhando direitos ou benefícios mais amplos ou adicionais reco-
suas experiências, conhecimentos e aptidões; possibilidade nhecidos pelo direito internacional ou pelas legislações
de prestar serviços à comunidade, inclusive voluntaria- internas dos Estados-partes em favor dos idosos.
mente, no que for de seu interesse e capacidade; cons- Também, no artigo 3º da Convenção são trazidos
trução de movimentos e associações de idosos; princípios gerais que lhe são inerentes, a saber, a pro-

47
moção e defesa dos direitos humanos e liberdades fun- Art. 230. A família, a sociedade e o Estado têm o de-
damentais do idoso; a valorização do idoso, seu papel ver de amparar as pessoas idosas, assegurando sua
na sociedade e sua contribuição ao desenvolvimento; a participação na comunidade, defendendo sua digni-
dignidade, independência, protagonismo e autonomia dade e bem-estar e garantindo-lhes o direito à vida.
do idoso; a igualdade e não discriminação; a participa- § 1º Os programas de amparo aos idosos serão exe-
ção, integração e inclusão plena e efetiva na sociedade; cutados preferencialmente em seus lares.
o bem-estar e cuidado; a segurança física, econômica e § 2º Aos maiores de sessenta e cinco anos é garan-
social; a autorrealização; a equidade e igualdade de gê- tida a gratuidade dos transportes coletivos urbanos.
nero e enfoque do curso de vida; a solidariedade e o for-
talecimento da proteção familiar e comunitária; o bom Ainda, a segunda parte do artigo 229, CF preconiza
tratamento e a atuação preferencial; o enfoque diferen- que “[...] os filhos maiores têm o dever de ajudar e ampa-
cial para o gozo efetivo dos direitos do idoso; o respeito rar os pais na velhice, carência ou enfermidade”. Conso-
e a valorização da diversidade cultural; a proteção judi- lida o dever de solidariedade familiar, compensando os
cial efetiva; bem como a responsabilidade do Estado e a pais que criaram seus filhos quando tinham condições e
participação da família e da comunidade na integração que hoje se encontram na posição de necessitados. Con-
ativa, plena e produtiva do idoso dentro da sociedade, tudo, este dever de amparo não é exclusivo da família,
bem como em seu cuidado e atenção, de acordo com a conforme se extrai do artigo 230, CF.
legislação interna. O fato de uma pessoa ter se tornado idosa não a
Por fim, são elencados na Convenção alguns direitos transforma numa parte dispensável da sociedade, que
protegidos: igualdade e não discriminação por razões de merece isolamento. Pelo contrário, suas experiências de-
idade (artigo 5º); direito à vida e à dignidade na velhice vem ser valorizadas e incorporadas nas práticas sociais,
(artigo 6º); direito à independência e à autonomia (arti- tornando-as mais adequadas. Assim, Estado, família e
go 7º); direito à participação e integração comunitária sociedade possuem o dever compartilhado de conferir
(artigo 8º); direito à segurança e a uma vida sem nenhum assistência aos idosos.
tipo de violência (artigo 9º); direito a não ser submetido O amparo aos idosos inclui múltiplas facetas: parti-
à tortura nem a penas ou tratamentos cruéis, desuma- cipação na comunidade, dignidade, bem-estar e vida.
nos ou degradantes (artigo 10); direito a manifestar con- Além disso, os programas de assistência, que prestam
sentimento livre e informado no âmbito da saúde (artigo este amparo, devem ser executados de preferência em
11); direitos do idoso que recebe serviços de cuidado seus lares.
de longo prazo (artigo 12); direito à liberdade pessoal O artigo 230, CF também assegura de forma expres-
(artigo 13); direito à liberdade de expressão e opinião sa a gratuidade dos transportes coletivos urbanos aos
e ao acesso à informação (artigo 14); direito à naciona- maiores de 65 anos.
lidade e à liberdade de circulação (artigo 15); direito à
intimidade e à privacidade (art. 16); direito à seguridade Histórico do estatuto do idoso
social (art. 17); direito ao trabalho (artigo 18); direito à A Lei nº 10.741, de 01 de outubro de 2003, que ins-
saúde (artigo 19); direito à educação (artigo 20); direito titui o Estatuto do Idoso, dispõe sobre papel da família,
à cultura (artigo 21); direito à recreação, ao esporte e ao da comunidade, da sociedade e do Poder Público de as-
segurar ao idoso, com absoluta prioridade, a efetivação
lazer (artigo 22); direito à propriedade (artigo 23); direito
do direito à vida, à saúde, à alimentação, à educação, à
à moradia (artigo 24); direito a um meio ambiente sau-
cultura, ao esporte, ao lazer, ao trabalho, à cidadania, à
dável (artigo 25); direito à acessibilidade e à mobilidade
liberdade, à dignidade, ao respeito e à convivência fami-
pessoal (artigo 26); direitos políticos (artigo 27); direito
liar e comunitária.
de reunião e de associação (artigo 28); situações de risco
O panorama demográfico mundial tem apresentado
e emergências humanitárias (artigo 29); igual reconheci-
mudanças nos últimos anos devido ao declínio das taxas
mento como pessoa perante a lei (artigo 30); e acesso à de fertilidade e mortalidade e como decorrência a lon-
justiça (artigo 31). gevidade tem-se apresentado como um fenômeno real.
Não se pode deixar de reconhecer, veja-se, que os O Brasil tem sido surpreendido por uma significativa
avanços das indústrias médica e farmacológica, a prá- mudança demográfica. Atualmente apresenta cerca de
tica de bons hábitos alimentares e esportivos/recreati- 15 milhões de idosos e, segundo projeção do IBGE, no
vos, a gradativa melhoria das condições de saneamento ano de 2025, será o sexto país mais idoso do mundo,
básico e as políticas voltadas à previdência social têm apenas perdendo para a Suíça, França, Estados Unidos,
promovido um processo geral de envelhecimento das Uruguai, Argentina, China, com um contingente de 34
populações de alguns países (o que se soma, geralmen- milhões de idosos, cerca de 15% da população.
te, à queda nos índices de natalidade). O direito interna- Diante dessa realidade, diferentes segmentos como
cional dos direitos humanos, portanto, não pode deixar
LEGISLAÇÃO ESPECÍFICA

a saúde, transporte, habitação, previdência social e edu-


desamparada esta nova realidade em rápido processo cação precisam ser redimensionados para atender esse
evolutivo. novo perfil populacional.
A sociedade política também assumiu sua responsa-
O tratamento do idoso no direito constitucional bilidade diante desse novo panorama demográfico bra-
Estabelece a Constituição Federal em seu artigo 230, sileiro, elaborou a Política Nacional do Idoso e o Estatuto
inserido no título VII (Ordem Social), capítulo VII sobre do Idoso. São leis elaboradas para preservar os direitos
a proteção da Família, da Criança, do Adolescente, do do idoso e evitar que essa faixa etária sofra discrimina-
Jovem e do Idoso: ções e seja marginalizado na sociedade brasileira.

48
É flagrante na sociedade brasileira um discurso favo- § 2° O direito ao respeito consiste na inviolabilidade
rável ao idoso, porém inserido em uma realidade prática da integridade física, psíquica e moral, abrangendo a
incompatível, ora reforçado pelo paternalismo, ora pelo preservação da imagem, da identidade, da autono-
assistencialismo, ora potencializando essa faixa etária, mas mia, de valores, ideias e crenças, dos espaços e dos
sem oferecer um real espaço social. objetos pessoais.
Por isso, o Estatuto do Idoso surge como instrumento § 3° É dever de todos zelar pela dignidade do idoso,
essencial ao resgate da dignidade inerente a todos idosos, colocando-o a salvo de qualquer tratamento desu-
mediante inclusão social e reforço de direitos fundamen- mano, violento, aterrorizante, vexatório ou constran-
tais. O fato é que a população brasileira está envelhecen- gedor.
do e necessário se faz garantir que este envelhecimento O Estado e a sociedade devem assegurar à pessoa
ocorra dentro dos princípios norteadores do atual texto idosa a liberdade, o respeito e a dignidade. O §1º
constitucional. A intervenção legislativa representada nes- especifica o direito à liberdade, que compreende o
te estatuto busca efetividade em resposta às necessidades direito de ir e vir, opinião e expressão, crença e culto
emergentes desse segmento da população. religioso, esportes e diversões, vida familiar e comu-
nitária, vida política, refúgio, auxílio e orientação. O
TÍTULO II §2º especifica o direito ao respeito como inviolabili-
Dos Direitos Fundamentais dade de qualquer tipo de integridade física, psíquica
e moral. O §3º especifica o direito à dignidade como
CAPÍTULO I o direito de não ser posto em qualquer tipo de tra-
Do Direito à Vida tamento desumano, violento, aterrorizante, vexatório
ou constrangedor.
Art. 8° O envelhecimento é um direito personalíssimo
e a sua proteção um direito social, nos termos desta CAPÍTULO III
Lei e da legislação vigente. Dos Alimentos
O envelhecimento é colocado como um direito perso-
nalíssimo. A ideia é de que envelhecer é um processo
Art. 11. Os alimentos serão prestados ao idoso na
natural que não deve ser combatido, evitado ou nega-
forma da lei civil.
do a todo custo. É, ainda, direito social, ou seja, direito
Art. 12. A obrigação alimentar é solidária, podendo o
humano de segunda dimensão.
idoso optar entre os prestadores.
Art. 9° É obrigação do Estado, garantir à pessoa idosa
Art. 13.As transações relativas a alimentos poderão
a proteção à vida e à saúde, mediante efetivação de
ser celebradas perante o Promotor de Justiça ou De-
políticas sociais públicas que permitam um envelheci-
mento saudável e em condições de dignidade. fensor Público, que as referendará, e passarão a ter
Cabe ao Estado desenvolver políticas públicas que ga- efeito de título executivo extrajudicial nos termos da
rantam o direito à vida e à saúde das pessoas idosas. lei processual civil.
Políticas públicas são conjuntos de programas, ações e Art. 14. Se o idoso ou seus familiares não possuírem
atividades desenvolvidas pelo Estado diretamente ou condições econômicas de prover o seu sustento, im-
indiretamente, com a participação de entes públicos põe-se ao Poder Público esse provimento, no âmbito
ou privados, que visam assegurar determinado direito da assistência social.
de cidadania, de forma difusa ou para determinado A obrigação alimentar decorre da intervenção do po-
seguimento social, cultural, étnico ou econômico. As der estatal, dos familiares, cônjuges e até mesmo da
políticas públicas específicas em relação ao idoso fa- sociedade (ao realizar atos de caridade), em relação
zem parte da Política Nacional do Idoso. a quem se encontra impossibilitado de fazê-lo por
meios próprios. Sendo importante notar que os ali-
CAPÍTULO II mentos não abrangem tão somente a alimentação,
Do Direito à Liberdade, ao Respeito e à Dignidade mas também outras utilidades necessárias para o
usufruto de uma vida digna.
Art. 10. É obrigação do Estado e da sociedade, asse- O artigo 1695 do Código Civil Brasileiro estabelece
gurar à pessoa idosa a liberdade, o respeito e a digni- que “são devidos os alimentos quando quem os pre-
dade, como pessoa humana e sujeito de direitos civis, tende não tem bens suficientes, nem pode prover,
políticos, individuais e sociais, garantidos na Consti- pelo seu trabalho, à própria mantença, e aquele, de
tuição e nas leis. quem se reclamam, pode fornecê-los, sem desfalque
§ 1° O direito à liberdade compreende, entre outros, do necessário ao seu sustento”.
os seguintes aspectos: E ainda o artigo 1694, também do Código Civil, dis-
I – faculdade de ir, vir e estar nos logradouros públi- põe que “podem os parentes, os cônjuges ou com-
LEGISLAÇÃO ESPECÍFICA

cos e espaços comunitários, ressalvadas as restrições panheiros pedir uns aos outros os alimentos de que
legais; necessitam para viver de modo compatível com a sua
II – opinião e expressão; condição social, inclusive para atender às necessida-
III – crença e culto religioso; des de sua educação”.
IV – prática de esportes e de diversões; Por fim, estabelece o §1° do dispositivo supramen-
V – participação na vida familiar e comunitária; cionado: “os alimentos devem ser fixados na propor-
VI – participação na vida política, na forma da lei; ção das necessidades do reclamante e dos recursos
VII – faculdade de buscar refúgio, auxílio e orientação. da pessoa obrigada”.

49
Desses dispositivos legais, pode-se extrair que são II – atendimento geriátrico e gerontológico em am-
dois os pressupostos principais para se reclamar ali- bulatórios;
mentos, quais sejam, a necessidade de quem plei- III – unidades geriátricas de referência, com pessoal
teia em consonância com a possibilidade de quem especializado nas áreas de geriatria e gerontologia
os fornecerá. social;
O reconhecimento da solidariedade implicaria admi- IV – atendimento domiciliar, incluindo a internação,
tir que todos os obrigados fossem responsáveis de para a população que dele necessitar e esteja impossi-
igual modo e por igual valor. Como regra, não há so- bilitada de se locomover, inclusive para idosos abriga-
lidariedade de alimentos no direito de família. Con- dos e acolhidos por instituições públicas, filantrópicas
tudo, por força da lei especial, é incontestável que o ou sem fins lucrativos e eventualmente conveniadas
Estatuto do Idoso disciplinou de forma contrária às com o Poder Público, nos meios urbano e rural;
Leis Civis de 1916 e 2002, adotando como política V – reabilitação orientada pela geriatria e gerontolo-
pública (art. 3º), a obrigação da família, da comu- gia, para redução das sequelas decorrentes do agravo
nidade, da sociedade e do Poder Público assegurar da saúde.
ao idoso, com absoluta prioridade a efetivação do O §1º fixa as formas especiais de tratamento da po-
direito à alimentação. Para tanto, mudou a natureza pulação idosa, percebendo-se que a atuação se dá na
da obrigação alimentícia de conjunta para solidária, vertente de controle da população para viabilizar um
com o objetivo de beneficiar sobremaneira a cele- melhor tratamento, o qual deve ser especializado e
ridade do processo, evitando discussões acerca do se dar tanto em ambientes especializados quanto de
ingresso dos demais devedores, não escolhidos pelo forma domiciliar.
credor-idoso para figurarem no polo passivo. Ex.: Se Obs.: geriatria é a especialidade médica que estuda e
um idoso tiver três filhos e um deles não pagar o
trata das doenças ligadas ao envelhecimento; e ge-
valor devido, poderá exigir do outro.
rontologia é o estudo dos fenômenos fisiológicos,
psicológicos e sociais relacionados ao envelhecimen-
CAPÍTULO IV
to do ser humano.
Do Direito à Saúde
§ 2° Incumbe ao Poder Público fornecer aos idosos,
Art. 15. É assegurada a atenção integral à saúde do gratuitamente, medicamentos, especialmente os de
idoso, por intermédio do Sistema Único de Saúde – uso continuado, assim como próteses, órteses e ou-
SUS, garantindo-lhe o acesso universal e igualitário, tros recursos relativos ao tratamento, habilitação ou
em conjunto articulado e contínuo das ações e ser- reabilitação.
viços, para a prevenção, promoção, proteção e re- O idoso tem direito a tratamento gratuito da saúde,
cuperação da saúde, incluindo a atenção especial às inclusive recebendo medicamentos, próteses, órteses
doenças que afetam preferencialmente os idosos. e outros recursos.
De forma semelhante à Constituição Federal quanto § 3° É vedada a discriminação do idoso nos planos
ao direito à saúde em geral, o Estatuto do Idoso fixa: de saúde pela cobrança de valores diferenciados em
- Direito à atenção integral: é a atenção plena, em razão da idade.
todas dimensões, tanto paliativa quanto preventiva A questão da discriminação do idoso nos planos de
– os objetivos, afinal, são prevenção, promoção, pro- saúde é complexa, porque não inviabiliza, segundo
teção e recuperação da saúde; a jurisprudência pátria, de forma plena o reajuste
- Por meio do Sistema Único de Saúde – SUS: é o por faixa etária. O STJ disse que é possível reajustar
sistema público de saúde brasileiro, um dos maiores por faixa etária se houver previsão contratual, se não
do mundo, mas a sua presença não exclui a possibili- forem aplicados percentuais desarrazoados, se pre-
dade de atuação privada no setor (até mesmo devido enchidos os requisitos da Lei nº 9.656/1998 (dispõe
à previsão sobre atenção integral); sobre os planos e seguros privados de assistência à
- Assegurado o acesso universal e igualitário: o aces- saúde), observância da boa-fé objetiva que impede
so ao sistema público de saúde deve ser garantido a reajustes absurdos e desproporcionais (REsp 866.840/
todas pessoas e em condições de igualdade material, SP, j. 07/06/2011).
ou seja, considerando as específicas necessidades das § 4° Os idosos portadores de deficiência ou com limi-
pessoas (por isso, é devido o tratamento diferenciado tação incapacitante terão atendimento especializado,
ao idoso e ao idoso com deficiência ou limitação);
nos termos da lei.
- Conjunto articulado e contínuo: as práticas de pre-
As pessoas idosas com condições peculiares devem
venção, promoção, proteção e recuperação devem
receber tratamento diferenciado, especializado.
ser adotadas de forma constante e planejada e, para
LEGISLAÇÃO ESPECÍFICA

§ 5º É vedado exigir o comparecimento do idoso en-


tanto, interligada;
- Atenção especial às doenças que afetam a popula- fermo perante os órgãos públicos, hipótese na qual
ção idosa: liga-se ao aspecto da igualdade material e será admitido o seguinte procedimento:
ao da atenção especial. I - quando de interesse do poder público, o agente
§ 1° A prevenção e a manutenção da saúde do idoso promoverá o contato necessário com o idoso em sua
serão efetivadas por meio de: residência; ou
I – cadastramento da população idosa em base ter- II - quando de interesse do próprio idoso, este se fará
ritorial; representar por procurador legalmente constituído.

50
§ 6º É assegurado ao idoso enfermo o atendimento Art. 18. As instituições de saúde devem atender aos
domiciliar pela perícia médica do Instituto Nacional critérios mínimos para o atendimento às necessida-
do Seguro Social - INSS, pelo serviço público de saú- des do idoso, promovendo o treinamento e a capa-
de ou pelo serviço privado de saúde, contratado ou citação dos profissionais, assim como orientação a
conveniado, que integre o Sistema Único de Saúde cuidadores familiares e grupos de autoajuda.
- SUS, para expedição do laudo de saúde necessá- Caberá às instituições de saúde atender a critérios
rio ao exercício de seus direitos sociais e de isenção mínimos que viabilizem o atendimento especializa-
tributária. do, devendo treinar e capacitar outras entidades que
Os §§ 5º e 6º foram incluídos pela Lei nº 12.896/2013 atuem paralelamente a elas.
e trata do comparecimento do idoso enfermo em ór- Art. 19. Os casos de suspeita ou confirmação de vio-
gãos públicos, inclusive para realização de perícias: lência praticada contra idosos serão objeto de notifi-
- Interesse do poder público – agente público deve cação compulsória pelos serviços de saúde públicos
ir até a residência; e privados à autoridade sanitária, bem como serão
- Interesse pessoal – representação por procurador. obrigatoriamente comunicados por eles a quaisquer
dos seguintes órgãos:
* As perícias do INSS devem se realizar em domicílio.
I – autoridade policial;
§ 7º Em todo atendimento de saúde, os maiores de
II – Ministério Público;
oitenta anos terão preferência especial sobre os de-
III – Conselho Municipal do Idoso;
mais idosos, exceto em caso de emergência.
IV – Conselho Estadual do Idoso;
Trata-se de prioridade especial no atendimento de V – Conselho Nacional do Idoso.
saúde. § 1° Para os efeitos desta Lei, considera-se violência
Art. 16. Ao idoso internado ou em observação é as- contra o idoso qualquer ação ou omissão praticada
segurado o direito a acompanhante, devendo o ór- em local público ou privado que lhe cause morte,
gão de saúde proporcionar as condições adequadas dano ou sofrimento físico ou psicológico.
para a sua permanência em tempo integral, segundo § 2° Aplica-se, no que couber, à notificação compul-
o critério médico. sória prevista no caput deste artigo, o disposto na Lei
Parágrafo único. Caberá ao profissional de saúde res- no 6.259, de 30 de outubro de 1975 (dispõe sobre a
ponsável pelo tratamento conceder autorização para organização das ações de Vigilância Epidemiológica,
o acompanhamento do idoso ou, no caso de impos- sobre o Programa Nacional de Imunizações, estabe-
sibilidade, justificá-la por escrito. lece normas relativas à notificação compulsória de
Embora o direito a acompanhante no tratamento doenças, e dá outras providências).
de saúde seja assegurado, caberá ao profissional de O artigo 19 foi alterado e teve dispositivos incluídos
saúde responsável decidir quanto ele é possível, jus- pela Lei nº 12.461/2011.
tificando por escrito eventual impossibilidade. Sua redação é um pouco confusa e inadequada. Trabalha
Art. 17. Ao idoso que esteja no domínio de suas fa- com notificações que devem ser comunicadas a órgãos
culdades mentais é assegurado o direito de optar competentes tanto práticas de violência (por qualquer
pelo tratamento de saúde que lhe for reputado mais ação ou omissão praticada em local público ou privado,
favorável. sendo violência todo caso de morte, dano ou sofrimento
Parágrafo único. Não estando o idoso em condições físico ou psicológico) quanto de doenças epidemiológi-
de proceder à opção, esta será feita: cas (com risco de transmissão em larga escala). Cabe a
I – pelo curador, quando o idoso for interditado; comunicação aos órgãos descritos nos incisos do caput
II – pelos familiares, quando o idoso não tiver cura- – autoridade policial, Ministério Público, Conselhos Mu-
dor ou este não puder ser contatado em tempo hábil; nicipal, Estadual e Federal; além dos órgãos de vigilância
sanitária e outros fixados na Lei nº 6.259/1975.
III – pelo médico, quando ocorrer iminente risco de
vida e não houver tempo hábil para consulta a cura-
CAPÍTULO V
dor ou familiar;
Da Educação, Cultura, Esporte e Lazer
IV – pelo próprio médico, quando não houver cura-
dor ou familiar conhecido, caso em que deverá co- Art. 20. O idoso tem direito a educação, cultura, es-
municar o fato ao Ministério Público. porte, lazer, diversões, espetáculos, produtos e ser-
O idoso não perde sua autonomia para a tomada de viços que respeitem sua peculiar condição de idade.
decisões, de modo que caberá a ele decidir sobre Art. 21. O Poder Público criará oportunidades de
sua própria saúde sempre que estiver em suas ple- acesso do idoso à educação, adequando currículos,
nas faculdades. Havendo incapacidade, permanente metodologias e material didático aos programas
ou transitória, a ordem de tomada de decisões é a educacionais a ele destinados.
LEGISLAÇÃO ESPECÍFICA

seguinte: § 1o Os cursos especiais para idosos incluirão conte-


- Situações não emergenciais – se houver curador, údo relativo às técnicas de comunicação, computa-
ele tem prioridade, se não houver a decisão cabe ao ção e demais avanços tecnológicos, para sua integra-
familiar, mas se não houver nenhum dos dois caberá ção à vida moderna.
ao médico, que informará ao Ministério Público. § 2° Os idosos participarão das comemorações de ca-
- Situações de urgência – risco de vida + ausência de ráter cívico ou cultural, para transmissão de conheci-
tempo hábil – cabe ao médico decidir. mentos e vivências às demais gerações, no sentido da
preservação da memória e da identidade culturais.

51
Art. 22. Nos currículos mínimos dos diversos níveis Art. 28. O Poder Público criará e estimulará progra-
de ensino formal serão inseridos conteúdos voltados mas de:
ao processo de envelhecimento, ao respeito e à valo- I – profissionalização especializada para os idosos,
rização do idoso, de forma a eliminar o preconceito e aproveitando seus potenciais e habilidades para ati-
a produzir conhecimentos sobre a matéria. vidades regulares e remuneradas;
Art. 23. A participação dos idosos em atividades cul- II – preparação dos trabalhadores para a aposenta-
turais e de lazer será proporcionada mediante des- doria, com antecedência mínima de 1 (um) ano, por
contos de pelo menos 50% (cinquenta por cento) meio de estímulo a novos projetos sociais, conforme
nos ingressos para eventos artísticos, culturais, es- seus interesses, e de esclarecimento sobre os direitos
portivos e de lazer, bem como o acesso preferencial sociais e de cidadania;
aos respectivos locais. III – estímulo às empresas privadas para admissão de
Art. 24. Os meios de comunicação manterão espaços idosos ao trabalho.
ou horários especiais voltados aos idosos, com fina- Como corolário da garantia de proteção ao idoso
lidade informativa, educativa, artística e cultural, e ao e do princípio da dignidade que lhes deve ser con-
público sobre o processo de envelhecimento. ferido o idoso tem direito ao exercício de atividade
Art. 25. As instituições de educação superior oferta- profissional, respeitadas suas condições físicas, inte-
rão às pessoas idosas, na perspectiva da educação lectuais e psíquicas.
ao longo da vida, cursos e programas de extensão, O trabalho é um direito fundamental da pessoa hu-
presenciais ou a distância, constituídos por ativida- mana que não deve encontrar óbice em seu tempo
des formais e não formais. de vida. A idade somente pode ser considerada um
Parágrafo único. O poder público apoiará a criação empecilho ao exercício do trabalho somente quando
de universidade aberta para as pessoas idosas e in- em benefício do idoso.
centivará a publicação de livros e periódicos, de con-
teúdo e padrão editorial adequados ao idoso, que CAPÍTULO VII
facilitem a leitura, considerada a natural redução da Da Previdência Social
capacidade visual.
O exercício ao direito à educação, cultura, esporte, Art. 29. Os benefícios de aposentadoria e pensão do
lazer, diversões, espetáculos, produtos e serviços dos Regime Geral da Previdência Social observarão, na
idosos deve respeitar sua peculiar condição de idade, sua concessão, critérios de cálculo que preservem o
fundado efetivamente em razões de igualdade ma- valor real dos salários sobre os quais incidiram con-
terial, a fim de proporcionar um tratamento justo ao tribuição, nos termos da legislação vigente.
idoso em seu meio social. Parágrafo único. Os valores dos benefícios em ma-
O respeito à peculiar condição de idade implica em nutenção serão reajustados na mesma data de re-
afirmar que na medida de suas condições físicas e ajuste do salário-mínimo, pro rata, de acordo com
emocionais de usufruir de educação, cultura, espor- suas respectivas datas de início ou do seu último re-
te, lazer, diversões, espetáculos, produtos e serviços ajustamento, com base em percentual definido em
aos idosos devem ser destinados oportunidades de regulamento, observados os critérios estabelecidos
participar do meio social em que vive sem lhe acres- pela Lei no 8.213, de 24 de julho de 1991.
centar fardo algum tendo em vista a sua vulnerabili- Art. 30. A perda da condição de segurado não será
dade, condição esta alçada em caráter de princípio a considerada para a concessão da aposentadoria por
ser buscado por todos. idade, desde que a pessoa conte com, no mínimo,
Constitui obrigação do Poder Público criar oportu- o tempo de contribuição correspondente ao exigido
nidades de acesso do idoso à educação, adequando para efeito de carência na data de requerimento do
currículos, metodologias e material didático aos pro- benefício.
gramas educacionais a ele destinados. O idoso que Parágrafo único. O cálculo do valor do benefício pre-
necessitar obter formação escolar regular deve ter visto no caput observará o disposto no caput e § 2o
acesso a processo educativo como as demais pesso- do art. 3o da Lei no 9.876, de 26 de novembro de
as na sociedade. 1999, ou, não havendo salários-de-contribuição re-
colhidos a partir da competência de julho de 1994, o
CAPÍTULO VI disposto no art. 35 da Lei no 8.213, de 1991.
Da Profissionalização e do Trabalho Art. 31. O pagamento de parcelas relativas a bene-
fícios, efetuado com atraso por responsabilidade da
Art. 26. O idoso tem direito ao exercício de atividade Previdência Social, será atualizado pelo mesmo índi-
profissional, respeitadas suas condições físicas, inte- ce utilizado para os reajustamentos dos benefícios
lectuais e psíquicas. do Regime Geral de Previdência Social, verificado no
LEGISLAÇÃO ESPECÍFICA

Art. 27. Na admissão do idoso em qualquer trabalho período compreendido entre o mês que deveria ter
ou emprego, é vedada a discriminação e a fixação sido pago e o mês do efetivo pagamento.
de limite máximo de idade, inclusive para concur- Art. 32. O Dia Mundial do Trabalho, 1o de Maio, é a
sos, ressalvados os casos em que a natureza do car- data-base dos aposentados e pensionistas.
go o exigir. A data base de uma categoria profissional é a data
Parágrafo único. O primeiro critério de desempate destinada a correção salarial e a discussão e revisão
em concurso público será a idade, dando-se prefe- das condições de trabalho fixadas em acordo, con-
rência ao de idade mais elevada. venção ou dissídio coletivo.

52
Este Estatuto ao dispor sobre o regime da Previdên- CAPÍTULO IX
cia Social reconhece que este constitui direito essen- Da Habitação
cial a conferir dignidade ao trabalhador em especial
aos idosos que por muito tempo contribuíram a fim Art. 37. O idoso tem direito a moradia digna, no seio
de fazer jus a este benefício. da família natural ou substituta, ou desacompanhado
A Previdência Social constitui um seguro coletivo de seus familiares, quando assim o desejar, ou, ainda,
que visa a cobertura de riscos sociais, cujo objetivo em instituição pública ou privada.
está delineado no art. 1° da Lei 8.213/1991, que é § 1o A assistência integral na modalidade de entida-
assegurar aos seus beneficiários meios indispensá- de de longa permanência será prestada quando ve-
veis de manutenção, por motivo de incapacidade, rificada inexistência de grupo familiar, casa-lar, aban-
desemprego voluntário, idade avançada, tempo de dono ou carência de recursos financeiros próprios ou
contribuição, encargos familiares e prisão ou morte da família.
daqueles de quem dependiam economicamente. § 2o Toda instituição dedicada ao atendimento ao
idoso fica obrigada a manter identificação externa
CAPÍTULO VIII visível, sob pena de interdição, além de atender toda
Da Assistência Social a legislação pertinente.
§ 3o As instituições que abrigarem idosos são obri-
Art. 33. A assistência social aos idosos será prestada, gadas a manter padrões de habitação compatíveis
de forma articulada, conforme os princípios e dire- com as necessidades deles, bem como provê-los
trizes previstos na Lei Orgânica da Assistência Social, com alimentação regular e higiene indispensáveis às
na Política Nacional do Idoso, no Sistema Único de normas sanitárias e com estas condizentes, sob as
Saúde e demais normas pertinentes. penas da lei.
Art. 34. Aos idosos, a partir de 65 (sessenta e cin- Art. 38. Nos programas habitacionais, públicos ou
co) anos, que não possuam meios para prover sua subsidiados com recursos públicos, o idoso goza de
subsistência, nem de tê-la provida por sua família, prioridade na aquisição de imóvel para moradia pró-
é assegurado o benefício mensal de 1 (um) salário- pria, observado o seguinte:
-mínimo, nos termos da Lei Orgânica da Assistência I - reserva de pelo menos 3% (três por cento) das uni-
Social – Loas. dades habitacionais residenciais para atendimento
Parágrafo único. O benefício já concedido a qualquer aos idosos; (Redação dada pela Lei nº 12.418, de 2011)
membro da família nos termos do caput não será II – implantação de equipamentos urbanos comuni-
computado para os fins do cálculo da renda familiar tários voltados ao idoso;
III – eliminação de barreiras arquitetônicas e urbanís-
per capita a que se refere a Loas.
ticas, para garantia de acessibilidade ao idoso;
Art. 35. Todas as entidades de longa permanência, ou
IV – critérios de financiamento compatíveis com os
casa-lar, são obrigadas a firmar contrato de presta-
rendimentos de aposentadoria e pensão.
ção de serviços com a pessoa idosa abrigada.
Parágrafo único.As unidades residenciais reservadas
§ 1° No caso de entidades filantrópicas, ou casa-lar,
para atendimento a idosos devem situar-se, prefe-
é facultada a cobrança de participação do idoso no
rencialmente, no pavimento térreo. (Incluído pela Lei
custeio da entidade.
nº 12.419, de 2011)
§ 2° O Conselho Municipal do Idoso ou o Conselho
As ações do Estado visando conferir a pessoa idosa
Municipal da Assistência Social estabelecerá a forma
moradia digna devem alcançar os programas habi-
de participação prevista no § 1o, que não poderá ex- tacionais, públicos ou subsidiados com recursos pú-
ceder a 70% (setenta por cento) de qualquer benefí- blicos, sendo que nestes programas o idoso deve ter
cio previdenciário ou de assistência social percebido prioridade na aquisição de imóvel para moradia pró-
pelo idoso. pria. Alguns requisitos devem ser observados nestes
§ 3° Se a pessoa idosa for incapaz, caberá a seu re- programas visando minimizar a situação do idoso e
presentante legal firmar o contrato a que se refere o a conferir-lhe moradia digna, como a reserva de pelo
caput deste artigo. menos 3% (três por cento) das unidades habitacio-
Art. 36. O acolhimento de idosos em situação de ris- nais residenciais para atendimento aos idosos.
co social, por adulto ou núcleo familiar, caracteriza a
dependência econômica, para os efeitos legais. CAPÍTULO X
Já a assistência tem caráter não-contributivo visan- Do Transporte
do ao amparo das pessoas necessitadas, conforme
LEGISLAÇÃO ESPECÍFICA

a previsão do art. 230 da CRFB/88, que a assistência Art. 39. Aos maiores de 65 (sessenta e cinco) anos
social será prestada a quem dela necessitar indepen- fica assegurada a gratuidade dos transportes cole-
dente de contribuição.A assistência tem caráter não- tivos públicos urbanos e semiurbanos, exceto nos
-contributivo visando ao amparo das pessoas neces- serviços seletivos e especiais, quando prestados pa-
sitadas, conforme a previsão do art. 230 da CRFB/88, ralelamente aos serviços regulares.
que a assistência social será prestada a quem dela § 1° Para ter acesso à gratuidade, basta que o ido-
necessitar independente de contribuição. so apresente qualquer documento pessoal que faça
prova de sua idade.

53
§ 2° Nos veículos de transporte coletivo de que trata II – por falta, omissão ou abuso da família, curador ou
este artigo, serão reservados 10% (dez por cento) dos entidade de atendimento;
assentos para os idosos, devidamente identificados com III – em razão de sua condição pessoal.
a placa de reservado preferencialmente para idosos. Medidas de proteção são providências que serão to-
§ 3° No caso das pessoas compreendidas na faixa etária madas para resguardar a vítima de violação, exposta
entre 60 (sessenta) e 65 (sessenta e cinco) anos, ficará a situação de risco. O artigo 43 leva em consideração
a critério da legislação local dispor sobre as condições a adoção da doutrina da proteção integral pelo es-
para exercício da gratuidade nos meios de transporte tatuto do idoso. As medidas de proteção se aplicam
previstos no caput deste artigo. ao idoso em situação de risco, que surgiu em razão
Art. 40. No sistema de transporte coletivo interestadual de qualquer dos responsáveis por seu cuidado (Esta-
observar-se-á, nos termos da legislação específica: do, família, sociedade). Existem inúmeras situações
I – a reserva de 2 (duas) vagas gratuitas por veículo de risco que justificam a adoção de medidas, por
para idosos com renda igual ou inferior a 2 (dois) sa- exemplo, agressões físicas, psíquicas, abandono ma-
lários-mínimos; terial ou afetivo (fundado no dever de solidariedade
II – desconto de 50% (cinquenta por cento), no míni- familiar), maus-tratos em geral. Além das medidas de
mo, no valor das passagens, para os idosos que exce- proteção em favor do agredido, deverão ser tomadas
derem as vagas gratuitas, com renda igual ou inferior a medidas de punição com relação ao agressor.
2 (dois) salários-mínimos.
Parágrafo único. Caberá aos órgãos competentes de- CAPÍTULO II
finir os mecanismos e os critérios para o exercício dos Das Medidas Específicas de Proteção
direitos previstos nos incisos I e II.
Art. 41. É assegurada a reserva, para os idosos, nos ter- Art. 44. As medidas de proteção ao idoso previstas
mos da lei local, de 5% (cinco por cento) das vagas nos nesta Lei poderão ser aplicadas, isolada ou cumulati-
estacionamentos públicos e privados, as quais deverão vamente, e levarão em conta os fins sociais a que se
ser posicionadas de forma a garantir a melhor como- destinam e o fortalecimento dos vínculos familiares
didade ao idoso. e comunitários.
Art. 42. São asseguradas a prioridade e a segurança Nada impede a aplicação cumulada de mais de uma
do idoso nos procedimentos de embarque e desem- medida de proteção. Caberá a adoção de quantas e
barque nos veículos do sistema de transporte coletivo. quais forem necessárias para remover o idoso da si-
(Redação dada pela Lei nº 12.899, de 2013) tuação de risco.
Está presente no Estatuto o direito fundamental do Art. 45. Verificada qualquer das hipóteses previstas
idoso ao transporte. Estabelece-se que os idosos no art. 43, o Ministério Público ou o Poder Judiciário,
maiores de 65 (sessenta e cinco) anos tem assegurado a requerimento daquele, poderá determinar, dentre
a gratuidade dos transportes coletivos públicos urba- outras, as seguintes medidas:
nos e semiurbanos, exceto nos serviços seletivos e es- I – encaminhamento à família ou curador, mediante
peciais, quando prestados paralelamente aos serviços termo de responsabilidade;
regulares. II – orientação, apoio e acompanhamento temporários;
Assim, essa gratuidade compreende o transporte ur- III – requisição para tratamento de sua saúde, em re-
bano coletivo, aquele transporte público oferecido a gime ambulatorial, hospitalar ou domiciliar;
todas as pessoas a fim de que se desloquem diaria- IV – inclusão em programa oficial ou comunitário de
mente na realização de suas atividades diversas e o auxílio, orientação e tratamento a usuários depen-
transporte semiurbano que tem as características do dentes de drogas lícitas ou ilícitas, ao próprio idoso
transporte coletivo, mas é prestado em áreas que ul- ou à pessoa de sua convivência que lhe cause per-
trapassa os limites de municípios, em grandes centros turbação;
metropolitanos. Essa benesse não alcança os serviços V – abrigo em entidade;
de transportes seletivos destinado a determinadas ca- VI – abrigo temporário.
tegorias de pessoas, como servidores de determina- O Ministério Público é a instituição responsável por
do órgão ou empresa pública, estudantes etc., como determinar ou requerer as medidas de proteção. Não
também não alcança o serviço de transporte especial, é qualquer medida que o Ministério Público pode
como os ônibus executivos que oferecem serviço e determinar diretamente, mas apenas aquelas que
atendimento diferenciado ao usuário. não caracterizem restrições involuntárias que afetem
o direito de liberdade (por exemplo, não pode de ofí-
TÍTULO III cio determinar a internação compulsória).
Das Medidas de Proteção As medidas descritas no rol do artigo 45 são exem-
LEGISLAÇÃO ESPECÍFICA

plificativas. Por exemplo, é possível aplicar as medi-


CAPÍTULO I das protetivas de urgência da Lei Maria da Penha e
Das Disposições Gerais as medidas cautelares do CPP, quando necessário
(como quando o idoso for vítima de crime).
Art. 43. As medidas de proteção ao idoso são aplicá- Interessante observar que no artigo 45 se prevê a
veis sempre que os direitos reconhecidos nesta Lei fo- possibilidade de que a medida de proteção afete
rem ameaçados ou violados: pessoa da convivência do idoso, caso se trate de de-
I – por ação ou omissão da sociedade ou do Estado; pendente químico que lhe causa perturbação.

54
TÍTULO VI Art. 100. Constitui crime punível com reclusão de 6
Dos Crimes (seis) meses a 1 (um) ano e multa:
I – obstar o acesso de alguém a qualquer cargo pú-
CAPÍTULO I blico por motivo de idade;
Disposições Gerais II – negar a alguém, por motivo de idade, emprego
ou trabalho;
Art. 93. Aplicam-se subsidiariamente, no que couber, III – recusar, retardar ou dificultar atendimento ou
as disposições da Lei no 7.347, de 24 de julho de 1985. deixar de prestar assistência à saúde, sem justa cau-
Art. 94. Aos crimes previstos nesta Lei, cuja pena má- sa, a pessoa idosa;
xima privativa de liberdade não ultrapasse 4 (quatro) IV – deixar de cumprir, retardar ou frustrar, sem justo
anos, aplica-se o procedimento previsto na Lei no 9.099, motivo, a execução de ordem judicial expedida na
de 26 de setembro de 1995, e, subsidiariamente, no ação civil a que alude esta Lei;
que couber, as disposições do Código Penal e do Códi- V – recusar, retardar ou omitir dados técnicos in-
go de Processo Penal. (Vide ADI 3.096-5 - STF) dispensáveis à propositura da ação civil objeto desta
Lei, quando requisitados pelo Ministério Público.
CAPÍTULO II Art. 101. Deixar de cumprir, retardar ou frustrar, sem
Dos Crimes em Espécie justo motivo, a execução de ordem judicial expedida
nas ações em que for parte ou interveniente o idoso:
Art. 95. Os crimes definidos nesta Lei são de ação Pena – detenção de 6 (seis) meses a 1 (um) ano e
penal pública incondicionada, não se lhes aplicando multa.
os arts. 181 e 182 do Código Penal. Art. 102. Apropriar-se de ou desviar bens, proven-
Art. 96. Discriminar pessoa idosa, impedindo ou difi- tos, pensão ou qualquer outro rendimento do idoso,
cultando seu acesso a operações bancárias, aos meios dando-lhes aplicação diversa da de sua finalidade:
de transporte, ao direito de contratar ou por qualquer Pena – reclusão de 1 (um) a 4 (quatro) anos e multa.
outro meio ou instrumento necessário ao exercício da Art. 103. Negar o acolhimento ou a permanência do
cidadania, por motivo de idade: idoso, como abrigado, por recusa deste em outorgar
procuração à entidade de atendimento:
Pena – reclusão de 6 (seis) meses a 1 (um) ano e multa.
Pena – detenção de 6 (seis) meses a 1 (um) ano e
§ 1o Na mesma pena incorre quem desdenhar, hu-
multa.
milhar, menosprezar ou discriminar pessoa idosa, por
Art. 104. Reter o cartão magnético de conta bancária
qualquer motivo.
relativa a benefícios, proventos ou pensão do idoso,
§ 2o A pena será aumentada de 1/3 (um terço) se a ví-
bem como qualquer outro documento com objetivo
tima se encontrar sob os cuidados ou responsabilidade
de assegurar recebimento ou ressarcimento de dí-
do agente.
vida:
Art. 97. Deixar de prestar assistência ao idoso, quan- Pena – detenção de 6 (seis) meses a 2 (dois) anos e
do possível fazê-lo sem risco pessoal, em situação de multa.
iminente perigo, ou recusar, retardar ou dificultar sua Art. 105. Exibir ou veicular, por qualquer meio de
assistência à saúde, sem justa causa, ou não pedir, nes- comunicação, informações ou imagens depreciativas
ses casos, o socorro de autoridade pública: ou injuriosas à pessoa do idoso:
Pena – detenção de 6 (seis) meses a 1 (um) ano e mul- Pena – detenção de 1 (um) a 3 (três) anos e multa.
ta. Art. 106. Induzir pessoa idosa sem discernimento de
Parágrafo único. A pena é aumentada de metade, se seus atos a outorgar procuração para fins de admi-
da omissão resulta lesão corporal de natureza grave, e nistração de bens ou deles dispor livremente:
triplicada, se resulta a morte. Pena – reclusão de 2 (dois) a 4 (quatro) anos.
Art. 98. Abandonar o idoso em hospitais, casas de saú- Art. 107. Coagir, de qualquer modo, o idoso a doar,
de, entidades de longa permanência, ou congêneres, contratar, testar ou outorgar procuração:
ou não prover suas necessidades básicas, quando obri- Pena – reclusão de 2 (dois) a 5 (cinco) anos.
gado por lei ou mandado: Art. 108. Lavrar ato notarial que envolva pessoa ido-
Pena – detenção de 6 (seis) meses a 3 (três) anos e sa sem discernimento de seus atos, sem a devida re-
multa. presentação legal:
Art. 99. Expor a perigo a integridade e a saúde, física Pena – reclusão de 2 (dois) a 4 (quatro) anos.
ou psíquica, do idoso, submetendo-o a condições de-
sumanas ou degradantes ou privando-o de alimentos TÍTULO VII
e cuidados indispensáveis, quando obrigado a fazê-lo, Disposições Finais e Transitórias
LEGISLAÇÃO ESPECÍFICA

ou sujeitando-o a trabalho excessivo ou inadequado:


Pena – detenção de 2 (dois) meses a 1 (um) ano e Art. 109. Impedir ou embaraçar ato do representante
multa. do Ministério Público ou de qualquer outro agente
§ 1o Se do fato resulta lesão corporal de natureza gra- fiscalizador:
ve: Pena – reclusão de 6 (seis) meses a 1 (um) ano e
Pena – reclusão de 1 (um) a 4 (quatro) anos. multa.
§ 2o Se resulta a morte:
Pena – reclusão de 4 (quatro) a 12 (doze) anos.

55
2) Registro
LEI ORDINÁRIA Nº 10.826, DE 22 DE Posse ou guarda - artigo 5º.
DEZEMBRO DE 2003. (ESTATUTO DO A finalidade é autorizar o proprietário a manter a
DESARMAMENTO); arma de fogo exclusivamente no interior de sua casa, do-
micílio ou local de trabalho.
A falta de registro leva à criminalização - artigo 12.
Posse irregular de arma: delito previsto no artigo 12.
“As regras para se comprar uma arma e os mecanis- A posse irregular é a posse sem registro. Trata-se de cri-
mos de controle destas no Brasil sempre foram falhos ou me comum (qualquer pessoa pode praticar), de perigo
praticamente inexistentes. Isto gerou, por muitos anos, abstrato (presume-se o perigo), de conteúdo múltiplo ou
uma grande entrada de armas em circulação no país. O variado (mais de um verbo no tipo - possuir ou guardar),
fácil acesso às armas de fogo sempre transformou os unissubjetivo (pode ser praticado por uma só pessoa),
conflitos existentes na sociedade brasileira em tragédias. doloso, para o qual não se admite tentativa.
Em 1997, apareceram os primeiros movimentos pró-
-desarmamento no Brasil e o controle de armas de fogo 3) SINARM
começou a entrar na pauta de preocupações nacional. As armas de fogo possuem algumas características
Neste mesmo ano, houve a primeira mudança na legis- como: marca, calibre, quantidade de cartuchos (balas), e
lação, ainda bastante insipiente frente à realidade brasi- outras mais complexas, como tipo da coronha, raias, etc.
leira. Afinal, mais de 80% dos crimes eram cometidos por Existem ainda as armas comuns como garruchas e revol-
armas de fogo. veres, que se diferenciam das armas automáticas, como
Os movimentos não pararam. Organizações passa- pistolas, metralhadoras e outras impróprias para o uso
ram a realizar eventos e atos públicos chamando a aten- comum, que são utilizadas pelas policias em operações
ção da população brasileira. Somando-se a isso, os da- especiais. Cabe ao SINARM catalogar e registrar todas as
dos e pesquisas que apareciam mostravam relação direta armas em circulação no Brasil.
entre o fácil acesso às armas de fogo e o aumento do Assim, o Sistema Nacional de Armas (SINARM), ins-
número de homicídios, comprovando que quanto mais tituído no Ministério da Justiça, no âmbito da Polícia Fe-
armas em circulação, mais morte. deral, com circunscrição em todo o território nacional, é
Em junho de 2003, foi organizada uma Marcha Si- responsável pelo controle de armas de fogo em poder da
lenciosa, com sapatos de vítimas de armas de fogo em população, conforme previsto na Lei nº 10.826/03 (Esta-
frente ao congresso nacional. Este fato chamou bastante tuto do Desarmamento).
atenção da mídia e da opinião pública. Os legisladores
tomaram para si o tema e criaram uma comissão mista, 4) Porte
com deputados federais e senadores para formular uma Autoriza a pessoa a ter a arma consigo fora de casa
nova lei. Esta comissão analisou todos os projetos que ou do trabalho. Para ter porte, precisa ter posse.
falavam sobre o tema nas duas casas e reescreveram uma O porte de uso para pessoas comuns em regra não é
lei conjunta: o Estatuto do Desarmamento. permitido - artigo 10.
Depois de redigido, faltava a aprovação, tanto no Se- O porte para funcionários de segurança e coleciona-
nado quanto na Câmara dos Deputados. O Estatuto foi dores que participam de eventos esportivos é eventual,
facilmente aprovado no Senado, mas logo em seguida ou seja, somente é aceito em algumas situações - artigos
ficou, mais de 3 meses parado esperando a aprovação 6º e 9º.
na Câmara dos Deputados. Lá enfrentou o poderosíssimo Magistratura e Ministério Público possuem porte
lobby das armas, ou seja, deputados federais que na sua funcional, assegurado nas respectivas leis orgânicas.
maioria tiveram as campanhas financiadas pelas indús- No porte ilegal não interessa se a arma é permitida
trias de armas e munições, a chamada Bancada da Bala. ou de uso restrito, se há registro ou não. Significa ter a
No entanto, a pressão popular foi mais forte e o Es- arma consigo fora dos limites do trabalho e da residên-
tatuto foi aprovado em outubro de 2004 na Câmara dos cia, sem autorização para isso, o que já constitui ato ilí-
Deputados. Voltou para o Senado novamente onde outra cito. Logo, uma pessoa pode ter a posse legal ou regular
vez foi aprovado rapidamente. No dia 23 de Dezembro (arma registrada) e praticar o crime de porte ilegal, pre-
o Estatuto do Desarmamento foi sancionado pelo presi- visto no artigo 14. Caso a posse seja ilegal, o delito é o
dente Luis Inácio Lula da Silva”59. do artigo 16.
Alguns pontos essenciais do Estatuto merecem des- O porte ilegal de arma do artigo 14 é um crime co-
taque em separado: mum (qualquer pessoa pode cometer), de merda condu-
ta (não depende de resultado), de perigo abstrato (não
1) Armas precisa sacar a arma), conteúdo múltiplo (13 núcleos de
LEGISLAÇÃO ESPECÍFICA

O estatuto do desarmamento se aplica apenas às tipo), unissubjetivo (basta ser praticado por 1 pessoa).
armas de fogo, munições e acessórios. Não se aplica às Seu objeto material é a arma ou acessório de uso permi-
armas brancas. tido devidamente numerado.
As armas podem ser próprias quando fabricadas O tipo do artigo 16 se aplica tanto ao porte quanto
para serem armas desde a sua origem, ou impróprias à posse de arma de uso restrito (mesma classificação do
quando não tem como finalidade ser arma mas ser artigo 14). O parágrafo único traz 6 ações diferentes que
usada como tal. constituem crimes autônomos.
59 http://www.deolhonoestatuto.org.br/

56
5) Prazo de regularização da arma ou entrega - LEI No 10.826, DE 22 DE DEZEMBRO DE 2003.
artigos 30 a 32
O prazo limite foi prorrogado e já se encerrou em Dispõe sobre registro, posse e comercialização de
31 de dezembro de 2009 - artigo 20 (Lei nº 11706/08). armas de fogo e munição, sobre o Sistema Nacional
Trata-se de abolitio criminis temporária: o fato deixa de de Armas – Sinarm, define crimes e dá outras provi-
ser considerado crime por algum tempo. dências.
Os artigos 30 a 32 se aplicam só à posse, não ao
porte. O artigo 30 fala que só se aplica à arma de uso CAPÍTULO I
permitido. O artigo 31 fala em arma regularmente ad- DO SISTEMA NACIONAL DE ARMAS
quirida, presumindo-se logicamente que só se aplica às
armas de uso permitido, porque não é possível adquirir Art. 1º O Sistema Nacional de Armas – Sinarm, ins-
regularmente arma de uso proibido. Já o artigo 32 não tituído no Ministério da Justiça, no âmbito da Po-
é expresso quanto à aplicação restrita às armas de uso lícia Federal, tem circunscrição em todo o território
permitido. Isto criou uma divergência nos tribunais, que nacional.
majoritariamente (inclusive STJ) têm decidido que não se Art. 2º Ao Sinarm compete:
aplica às armas de uso proibido. I – identificar as características e a propriedade de ar-
mas de fogo, mediante cadastro;
6) Exame pericial II – cadastrar as armas de fogo produzidas, importa-
Posição amplamente majoritária diz que o exame pe- das e vendidas no País;
ricial é indispensável. A minoritária parte do pressuposto III – cadastrar as autorizações de porte de arma de
de que o que a polícia diz que é arma, é arma. O laudo fogo e as renovações expedidas pela Polícia Federal;
é nulo se o exame pericial for feito pelos policiais que IV – cadastrar as transferências de propriedade, ex-
fizeram a prisão em flagrante. travio, furto, roubo e outras ocorrências suscetíveis de
alterar os dados cadastrais, inclusive as decorrentes de
7) Arma com defeito fechamento de empresas de segurança privada e de
Se o defeito existia e não era possível disparar a arma, transporte de valores;
a doutrina majoritária diz que não há crime, porque não V – identificar as modificações que alterem as caracte-
existe arma de fogo; a doutrina minoritária diz que há rísticas ou o funcionamento de arma de fogo;
crime, porque o objetivo da lei é proteger a segurança VI – integrar no cadastro os acervos policiais já exis-
pública. tentes;
VII – cadastrar as apreensões de armas de fogo, inclu-
8) Arma sem munição sive as vinculadas a procedimentos policiais e judiciais;
Existem 3 posições: exige munição, porque não há VIII – cadastrar os armeiros em atividade no País, bem
crime sem potencialidade lesiva; no se exige munição, como conceder licença para exercer a atividade;
desde que ela esteja ao alcance (ex: arma no porta-malas IX – cadastrar mediante registro os produtores, ataca-
e munição no bolso); não interessa se a arma está com distas, varejistas, exportadores e importadores autori-
munição ou não por causa da objetividade jurídica, que zados de armas de fogo, acessórios e munições;
é proteger a segurança e a incolumidade (majoritária, re- X – cadastrar a identificação do cano da arma, as ca-
comendável para o concurso da PRF). racterísticas das impressões de raiamento e de micro-
estriamento de projétil disparado, conforme marcação
9) Concurso de crimes e testes obrigatoriamente realizados pelo fabricante;
- Posse de mais de 1 arma: jurisprudência diz que é XI – informar às Secretarias de Segurança Pública dos
um só crime. Estados e do Distrito Federal os registros e autoriza-
- Posse de 1 arma e de munição de calibre diferente: ções de porte de armas de fogo nos respectivos terri-
2 crimes em concurso formal. tórios, bem como manter o cadastro atualizado para
- Posse só de munição: 1 só crime independente da consulta.
quantidade. Parágrafo único. As disposições deste artigo não al-
cançam as armas de fogo das Forças Armadas e Au-
10) Disparo xiliares, bem como as demais que constem dos seus
Previsto no artigo 15. Trata-se de crime comum, de registros próprios.
mera conduta, unissubjetivo, de perigo abstrato. Seu ob-
jeto jurídico é a proteção da incolumidade pública. Seu CAPÍTULO II
objeto material é a arma de fogo ou munição. O crime é DO REGISTRO
subsidiário, pois é preciso que não se tenha como fina-
LEGISLAÇÃO ESPECÍFICA

lidade a prática de outro crime (ex: tentativa de homicí- Art. 3º É obrigatório o registro de arma de fogo no
dio). Consuma-se com o disparo. órgão competente.
Parágrafo único. As armas de fogo de uso restrito se-
rão registradas no Comando do Exército, na forma do
regulamento desta Lei.
Art. 4º Para adquirir arma de fogo de uso permiti-
do o interessado deverá, além de declarar a efe-
tiva necessidade, atender aos seguintes requisitos:

57
I – comprovação de idoneidade, com a apresentação § 3ºO proprietário de arma de fogo com certificados
de certidões negativas de antecedentes criminais for- de registro de propriedade expedido por órgão esta-
necidas pela Justiça Federal, Estadual, Militar e Eleito- dual ou do Distrito Federal até a data da publicação
ral e de não estar respondendo a inquérito policial ou desta Lei que não optar pela entrega espontânea pre-
a processo criminal, que poderão ser fornecidas por vista no art. 32 desta Lei deverá renová-lo mediante o
meios eletrônicos; pertinente registro federal, até o dia 31 de dezembro
II – apresentação de documento comprobatório de de 2008, ante a apresentação de documento de iden-
ocupação lícita e de residência certa; tificação pessoal e comprovante de residência fixa,
III – comprovação de capacidade técnica e de apti- ficando dispensado do pagamento de taxas e do cum-
dão psicológica para o manuseio de arma de fogo, primento das demais exigências constantes dos incisos
atestadas na forma disposta no regulamento desta I a III do caput do art. 4º desta Lei.
Lei. § 4ºPara fins do cumprimento do disposto no § 3º
§ 1º O Sinarm expedirá autorização de compra de deste artigo, o proprietário de arma de fogo poderá
arma de fogo após atendidos os requisitos anterior- obter, no Departamento de Polícia Federal, certificado
mente estabelecidos, em nome do requerente e para a de registro provisório, expedido na rede mundial de
arma indicada, sendo intransferível esta autorização. computadores - internet, na forma do regulamento e
§ 2ºA aquisição de munição somente poderá ser obedecidos os procedimentos a seguir:
feita no calibre correspondente à arma registrada I - emissão de certificado de registro provisório pela
internet, com validade inicial de 90 (noventa) dias; e
e na quantidade estabelecida no regulamento desta
II - revalidação pela unidade do Departamento de
Lei.
Polícia Federal do certificado de registro provisório
§ 3º A empresa que comercializar arma de fogo em
pelo prazo que estimar como necessário para a emissão
território nacional é obrigada a comunicar a venda definitiva do certificado de registro de propriedade.
à autoridade competente, como também a manter
banco de dados com todas as características da arma CAPÍTULO III
e cópia dos documentos previstos neste artigo. DO PORTE
§ 4º A empresa que comercializa armas de fogo,
acessórios e munições responde legalmente por essas Art. 6º É proibido o porte de arma de fogo em todo
mercadorias, ficando registradas como de sua pro- o território nacional, salvo para os casos previstos
priedade enquanto não forem vendidas. em legislação própria e para:
§ 5º A comercialização de armas de fogo, acessórios e I – os integrantes das Forças Armadas;
munições entre pessoas físicas somente será efetiva- II – os integrantes de órgãos referidos nos incisos I, II,
da mediante autorização do Sinarm. III, IV e V do caput do art. 144 da Constituição Fede-
§ 6º A expedição da autorização a que se refere o § 1º ral e os da Força Nacional de Segurança Pública
será concedida, ou recusada com a devida fundamen- (FNSP);
tação, no prazo de 30 (trinta) dias úteis, a contar da III – os integrantes das guardas municipais das capi-
data do requerimento do interessado. tais dos Estados e dos Municípios com mais de 500.000
§ 7º O registro precário a que se refere o § 4º prescin- (quinhentos mil) habitantes, nas condições estabeleci-
de do cumprimento dos requisitos dos incisos I, II e III das no regulamento desta Lei;
deste artigo. IV - os integrantes das guardas municipais dos Mu-
§ 8ºEstará dispensado das exigências constantes nicípios com mais de 50.000 (cinquenta mil) e menos
do inciso III do caput deste artigo, na forma do re- de 500.000 (quinhentos mil) habitantes, quando em
gulamento, o interessado em adquirir arma de fogo serviço;
de uso permitido que comprove estar autorizado a V – os agentes operacionais da Agência Brasileira de
portar arma com as mesmas características daquela Inteligência e os agentes do Departamento de Se-
a ser adquirida. gurança do Gabinete de Segurança Institucional
da Presidência da República;
Art. 5º O certificado de Registro de Arma de Fogo,
VI – os integrantes dos órgãos policiais referidos no
com validade em todo o território nacional, autoriza
art. 51, IV, e no art. 52, XIII, da Constituição Federal;
o seu proprietário a manter a arma de fogo exclusiva-
VII – os integrantes do quadro efetivo dos agentes e
mente no interior de sua residência ou domicílio, ou guardas prisionais, os integrantes das escoltas de
dependência desses, ou, ainda, no seu local de traba- presos e as guardas portuárias;
lho, desde que seja ele o titular ou o responsável legal VIII – as empresas de segurança privada e de trans-
pelo estabelecimento ou empresa. porte de valores constituídas, nos termos desta Lei;
§ 1º O certificado de registro de arma de fogo será
LEGISLAÇÃO ESPECÍFICA

IX – para os integrantes das entidades de desporto


expedido pela Polícia Federal e será precedido de legalmente constituídas, cujas atividades esporti-
autorização do Sinarm. vas demandem o uso de armas de fogo, na forma
§ 2º Os requisitos de que tratam os incisos I, II e III do do regulamento desta Lei, observando-se, no que cou-
art. 4º deverão ser comprovados periodicamente, ber, a legislação ambiental.
em período não inferior a 3 (três) anos, na con- X - integrantes das Carreiras de Auditoria da Re-
formidade do estabelecido no regulamento desta Lei, ceita Federal do Brasil e de Auditoria-Fiscal do
para a renovação do Certificado de Registro de Arma Trabalho, cargos de Auditor-Fiscal e Analista Tri-
de Fogo. butário.

58
XI - os tribunais do Poder Judiciário descritos no art. § 6ºO caçador para subsistência que der outro uso à
92 da Constituição Federal e os Ministérios Públicos sua arma de fogo, independentemente de outras ti-
da União e dos Estados, para uso exclusivo de servi- pificações penais, responderá, conforme o caso, por
dores de seus quadros pessoais que efetivamente este- porte ilegal ou por disparo de arma de fogo de uso
jam no exercício de funções de segurança, na forma de permitido.
regulamento a ser emitido pelo Conselho Nacional de § 7ºAos integrantes das guardas municipais dos Mu-
Justiça - CNJ e pelo Conselho Nacional do Ministério nicípios que integram regiões metropolitanas será au-
Público - CNMP. torizado porte de arma de fogo, quando em serviço.
§ 1ºAs pessoas previstas nos incisos I, II, III, V e VI do Art. 7º As armas de fogo utilizadas pelos empregados
caput deste artigo terão direito de portar arma de fogo das empresas de segurança privada e de transpor-
de propriedade particular ou fornecida pela respectiva te de valores, constituídas na forma da lei, serão de
corporação ou instituição, mesmo fora de serviço, nos propriedade, responsabilidade e guarda das res-
termos do regulamento desta Lei, com validade em pectivas empresas, somente podendo ser utilizadas
âmbito nacional para aquelas constantes dos incisos quando em serviço, devendo essas observar as con-
I, II, V e VI. dições de uso e de armazenagem estabelecidas pelo
§ 1o-A (Revogado) órgão competente, sendo o certificado de registro e
§ 1º-B. Os integrantes do quadro efetivo de agentes a autorização de porte expedidos pela Polícia Federal
e guardas prisionais poderão portar arma de fogo de em nome da empresa.
propriedade particular ou fornecida pela respectiva § 1º O proprietário ou diretor responsável de empre-
corporação ou instituição, mesmo fora de serviço, des- sa de segurança privada e de transporte de valores
de que estejam: responderá pelo crime previsto no parágrafo único do
I - submetidos a regime de dedicação exclusiva; art. 13 desta Lei, sem prejuízo das demais sanções ad-
II - sujeitos à formação funcional, nos termos do re- ministrativas e civis, se deixar de registrar ocorrência
gulamento; e policial e de comunicar à Polícia Federal perda, furto,
III - subordinados a mecanismos de fiscalização e de roubo ou outras formas de extravio de armas de fogo,
acessórios e munições que estejam sob sua guarda,
controle interno.
nas primeiras 24 (vinte e quatro) horas depois de ocor-
§ 1º-C. (VETADO).
rido o fato.
§ 2ºA autorização para o porte de arma de fogo aos
§ 2º A empresa de segurança e de transporte de valo-
integrantes das instituições descritas nos incisos V, VI,
res deverá apresentar documentação comprobatória
VII e X do caput deste artigo está condicionada à com-
do preenchimento dos requisitos constantes do art. 4º
provação do requisito a que se refere o inciso III do
desta Lei quanto aos empregados que portarão arma
caput do art. 4º desta Lei nas condições estabelecidas
de fogo.
no regulamento desta Lei.
§ 3º A listagem dos empregados das empresas referi-
§ 3º A autorização para o porte de arma de fogo das
das neste artigo deverá ser atualizada semestralmente
guardas municipais está condicionada à formação junto ao Sinarm.
funcional de seus integrantes em estabelecimentos de Art. 7º-A.As armas de fogo utilizadas pelos servido-
ensino de atividade policial, à existência de mecanis- res das instituições descritas no inciso XI do art. 6º
mos de fiscalização e de controle interno, nas condi- serão de propriedade, responsabilidade e guarda
ções estabelecidas no regulamento desta Lei, observa- das respectivas instituições, somente podendo ser
da a supervisão do Ministério da Justiça. utilizadas quando em serviço, devendo estas observar
§ 4º Os integrantes das Forças Armadas, das polícias as condições de uso e de armazenagem estabelecidas
federais e estaduais e do Distrito Federal, bem como pelo órgão competente, sendo o certificado de registro
os militares dos Estados e do Distrito Federal, ao exer- e a autorização de porte expedidos pela Polícia Fede-
cerem o direito descrito no art. 4º, ficam dispensados ral em nome da instituição.
do cumprimento do disposto nos incisos I, II e III do § 1ºA autorização para o porte de arma de fogo de que
mesmo artigo, na forma do regulamento desta Lei. trata este artigo independe do pagamento de taxa.
§ 5ºAos residentes em áreas rurais, maiores de 25 § 2ºO presidente do tribunal ou o chefe do Ministério
(vinte e cinco) anos que comprovem depender do em- Público designará os servidores de seus quadros pes-
prego de arma de fogo para prover sua subsistência soais no exercício de funções de segurança que pode-
alimentar familiar será concedido pela Polícia Federal rão portar arma de fogo, respeitado o limite máximo
o porte de arma de fogo, na categoria caçador para de 50% (cinquenta por cento) do número de servidores
subsistência, de uma arma de uso permitido, de tiro que exerçam funções de segurança.
LEGISLAÇÃO ESPECÍFICA

simples, com 1 (um) ou 2 (dois) canos, de alma lisa e § 3ºO porte de arma pelos servidores das instituições
de calibre igual ou inferior a 16 (dezesseis), desde que de que trata este artigo fica condicionado à apresen-
o interessado comprove a efetiva necessidade em re- tação de documentação comprobatória do preenchi-
querimento ao qual deverão ser anexados os seguintes mento dos requisitos constantes do art. 4º desta Lei,
documentos: bem como à formação funcional em estabelecimentos
I - documento de identificação pessoal; de ensino de atividade policial e à existência de meca-
II - comprovante de residência em área rural; e nismos de fiscalização e de controle interno, nas con-
III - atestado de bons antecedentes. dições estabelecidas no regulamento desta Lei.

59
§ 4ºA listagem dos servidores das instituições de que Art. 11-A.O Ministério da Justiça disciplinará a forma
trata este artigo deverá ser atualizada semestralmen- e as condições do credenciamento de profissionais
te no Sinarm. pela Polícia Federal para comprovação da aptidão
§ 5ºAs instituições de que trata este artigo são obri- psicológica e da capacidade técnica para o manuseio
gadas a registrar ocorrência policial e a comunicar à de arma de fogo.
Polícia Federal eventual perda, furto, roubo ou outras § 1ºNa comprovação da aptidão psicológica, o valor
formas de extravio de armas de fogo, acessórios e mu- cobrado pelo psicólogo não poderá exceder ao valor
nições que estejam sob sua guarda, nas primeiras 24 médio dos honorários profissionais para realização de
(vinte e quatro) horas depois de ocorrido o fato. avaliação psicológica constante do item 1.16 da tabela
Art. 8º As armas de fogo utilizadas em entidades des- do Conselho Federal de Psicologia.
portivas legalmente constituídas devem obedecer às § 2ºNa comprovação da capacidade técnica, o valor
condições de uso e de armazenagem estabelecidas cobrado pelo instrutor de armamento e tiro não pode-
pelo órgão competente, respondendo o possuidor ou o rá exceder R$ 80,00 (oitenta reais), acrescido do custo
autorizado a portar a arma pela sua guarda na forma da munição.
do regulamento desta Lei. § 3ºA cobrança de valores superiores aos previstos nos
Art. 9º Compete ao Ministério da Justiça a autori- §§ 1º e 2º deste artigo implicará o descredenciamento
zação do porte de arma para os responsáveis pela do profissional pela Polícia Federal.
segurança de cidadãos estrangeiros em visita ou
sediados no Brasil e, ao Comando do Exército, nos
CAPÍTULO IV
DOS CRIMES E DAS PENAS
termos do regulamento desta Lei, o registro e a con-
cessão de porte de trânsito de arma de fogo para
Posse irregular de arma de fogo de uso permitido
colecionadores, atiradores e caçadores e de repre-
Art. 12. Possuir ou manter sob sua guarda arma de
sentantes estrangeiros em competição internacional
fogo, acessório ou munição, de uso permitido, em de-
oficial de tiro realizada no território nacional.
sacordo com determinação legal ou regulamentar, no
Art. 10. A autorização para o porte de arma de
interior de sua residência ou dependência desta, ou,
fogo de uso permitido, em todo o território nacio- ainda no seu local de trabalho, desde que seja o titular
nal, é de competência da Polícia Federal e somente ou o responsável legal do estabelecimento ou empresa:
será concedida após autorização do Sinarm. Pena – detenção, de 1 (um) a 3 (três) anos, e multa.
§ 1º A autorização prevista neste artigo poderá ser
concedida com eficácia temporária e territorial limi- Omissão de cautela
tada, nos termos de atos regulamentares, e dependerá Art. 13. Deixar de observar as cautelas necessárias
de o requerente: para impedir que menor de 18 (dezoito) anos ou pes-
I – demonstrar a sua efetiva necessidade por exercício soa portadora de deficiência mental se apodere de
de atividade profissional de risco ou de ameaça à sua arma de fogo que esteja sob sua posse ou que seja de
integridade física; sua propriedade:
II – atender às exigências previstas no art. 4º desta Lei; Pena – detenção, de 1 (um) a 2 (dois) anos, e multa.
III – apresentar documentação de propriedade de Parágrafo único. Nas mesmas penas incorrem o pro-
arma de fogo, bem como o seu devido registro no ór- prietário ou diretor responsável de empresa de segu-
gão competente. rança e transporte de valores que deixarem de registrar
§ 2º A autorização de porte de arma de fogo, prevista ocorrência policial e de comunicar à Polícia Federal
neste artigo, perderá automaticamente sua eficácia perda, furto, roubo ou outras formas de extravio de
caso o portador dela seja detido ou abordado em es- arma de fogo, acessório ou munição que estejam sob
tado de embriaguez ou sob efeito de substâncias quí- sua guarda, nas primeiras 24 (vinte quatro) horas de-
micas ou alucinógenas. pois de ocorrido o fato.
Art. 11. Fica instituída a cobrança de taxas, nos va-
lores constantes do Anexo desta Lei, pela prestação de Porte ilegal de arma de fogo de uso permitido
serviços relativos: Art. 14. Portar, deter, adquirir, fornecer, receber, ter em
I – ao registro de arma de fogo; depósito, transportar, ceder, ainda que gratuitamente,
II – à renovação de registro de arma de fogo; emprestar, remeter, empregar, manter sob guarda ou
III – à expedição de segunda via de registro de arma ocultar arma de fogo, acessório ou munição, de uso
de fogo; permitido, sem autorização e em desacordo com de-
IV – à expedição de porte federal de arma de fogo; terminação legal ou regulamentar:
V – à renovação de porte de arma de fogo; Pena – reclusão, de 2 (dois) a 4 (quatro) anos, e multa.
Parágrafo único. O crime previsto neste artigo é ina-
LEGISLAÇÃO ESPECÍFICA

VI – à expedição de segunda via de porte federal de


arma de fogo. fiançável, salvo quando a arma de fogo estiver regis-
§ 1º Os valores arrecadados destinam-se ao custeio trada em nome do agente. (Vide Adin 3.112-1)
e à manutenção das atividades do Sinarm, da Polícia
Federal e do Comando do Exército, no âmbito de suas Disparo de arma de fogo
respectivas responsabilidades. Art. 15. Disparar arma de fogo ou acionar munição em
§ 2ºSão isentas do pagamento das taxas previstas nes- lugar habitado ou em suas adjacências, em via pública
te artigo as pessoas e as instituições a que se referem ou em direção a ela, desde que essa conduta não te-
os incisos I a VII e X e o § 5º do art. 6º desta Lei. nha como finalidade a prática de outro crime:

60
Pena – reclusão, de 2 (dois) a 4 (quatro) anos, e multa. Art. 21. Os crimes previstos nos arts. 16, 17 e 18 são
Parágrafo único. O crime previsto neste artigo é ina- insuscetíveis de liberdade provisória. (Adin 3.112-1 –
fiançável. (Vide Adin 3.112-1) declarou inconstitucional a vedação de liberdade pro-
visória)
Posse ou porte ilegal de arma de fogo de uso res-
trito CAPÍTULO V
Art. 16. Possuir, deter, portar, adquirir, fornecer, receber, DISPOSIÇÕES GERAIS
ter em depósito, transportar, ceder, ainda que gratuita-
mente, emprestar, remeter, empregar, manter sob sua Art. 22. O Ministério da Justiça poderá celebrar convê-
guarda ou ocultar arma de fogo, acessório ou munição nios com os Estados e o Distrito Federal para o cum-
de uso proibido ou restrito, sem autorização e em de- primento do disposto nesta Lei.
sacordo com determinação legal ou regulamentar: Art. 23.A classificação legal, técnica e geral bem
Pena – reclusão, de 3 (três) a 6 (seis) anos, e multa. como a definição das armas de fogo e demais produ-
Parágrafo único. Nas mesmas penas incorre quem: tos controlados, de usos proibidos, restritos, permitidos
I – suprimir ou alterar marca, numeração ou qualquer ou obsoletos e de valor histórico serão disciplinadas
sinal de identificação de arma de fogo ou artefato; em ato do chefe do Poder Executivo Federal, mediante
II – modificar as características de arma de fogo, de proposta do Comando do Exército.
forma a torná-la equivalente a arma de fogo de uso § 1º Todas as munições comercializadas no País de-
proibido ou restrito ou para fins de dificultar ou de verão estar acondicionadas em embalagens com sis-
qualquer modo induzir a erro autoridade policial, pe- tema de código de barras, gravado na caixa, visando
rito ou juiz; possibilitar a identificação do fabricante e do adqui-
III – possuir, detiver, fabricar ou empregar artefato ex- rente, entre outras informações definidas pelo regula-
plosivo ou incendiário, sem autorização ou em desa- mento desta Lei.
cordo com determinação legal ou regulamentar; § 2º Para os órgãos referidos no art. 6º, somente serão
IV – portar, possuir, adquirir, transportar ou fornecer expedidas autorizações de compra de munição com
arma de fogo com numeração, marca ou qualquer ou- identificação do lote e do adquirente no culote dos
tro sinal de identificação raspado, suprimido ou adul- projéteis, na forma do regulamento desta Lei.
terado; § 3º As armas de fogo fabricadas a partir de 1 (um)
V – vender, entregar ou fornecer, ainda que gratuita- ano da data de publicação desta Lei conterão disposi-
mente, arma de fogo, acessório, munição ou explosivo tivo intrínseco de segurança e de identificação, grava-
a criança ou adolescente; e do no corpo da arma, definido pelo regulamento desta
VI – produzir, recarregar ou reciclar, sem autorização Lei, exclusive para os órgãos previstos no art. 6º.
legal, ou adulterar, de qualquer forma, munição ou ex- § 4ºAs instituições deensino policial e as guardas mu-
plosivo. nicipais referidas nos incisos III e IV do caput do art.
6º desta Lei e no seu § 7º poderão adquirir insumos e
Comércio ilegal de arma de fogo máquinas de recarga de munição para o fim exclusivo
Art. 17. Adquirir, alugar, receber, transportar, conduzir, de suprimento de suas atividades, mediante autoriza-
ocultar, ter em depósito, desmontar, montar, remontar, ção concedida nos termos definidos em regulamento.
adulterar, vender, expor à venda, ou de qualquer forma Art. 24. Excetuadas as atribuições a que se refere o
utilizar, em proveito próprio ou alheio, no exercício de art. 2º desta Lei, compete ao Comando do Exército
atividade comercial ou industrial, arma de fogo, aces- autorizar e fiscalizar a produção, exportação, im-
sório ou munição, sem autorização ou em desacordo portação, desembaraço alfandegário e o comércio
com determinação legal ou regulamentar: de armas de fogo e demais produtos controlados,
Pena – reclusão, de 4 (quatro) a 8 (oito) anos, e multa. inclusive o registro e o porte de trânsito de arma de
Parágrafo único. Equipara-se à atividade comercial ou fogo de colecionadores, atiradores e caçadores.
industrial, para efeito deste artigo, qualquer forma de Art. 25.As armas de fogo apreendidas, após a ela-
prestação de serviços, fabricação ou comércio irregular boração do laudo pericial e sua juntada aos au-
ou clandestino, inclusive o exercido em residência. tos, quando não mais interessarem à persecução
penal serão encaminhadas pelo juiz competente
Tráfico internacional de arma de fogo ao Comando do Exército, no prazo máximo de 48
Art. 18. Importar, exportar, favorecer a entrada ou saí- (quarenta e oito) horas, para destruição ou doação
da do território nacional, a qualquer título, de arma de aos órgãos de segurança pública ou às Forças Arma-
fogo, acessório ou munição, sem autorização da auto- das, na forma do regulamento desta Lei.
§ 1ºAs armas de fogo encaminhadas ao Comando do
LEGISLAÇÃO ESPECÍFICA

ridade competente:
Pena – reclusão de 4 (quatro) a 8 (oito) anos, e multa. Exército que receberem parecer favorável à doação,
Art. 19. Nos crimes previstos nos arts. 17 e 18, a pena é obedecidos o padrão e a dotação de cada Força Ar-
aumentada da metade se a arma de fogo, acessório ou mada ou órgão de segurança pública, atendidos os
munição forem de uso proibido ou restrito. critérios de prioridade estabelecidos pelo Ministério
Art. 20. Nos crimes previstos nos arts. 14, 15, 16, 17 e da Justiça e ouvido o Comando do Exército, serão ar-
18, a pena é aumentada da metade se forem pratica- roladas em relatório reservado trimestral a ser enca-
dos por integrante dos órgãos e empresas referidas nos minhado àquelas instituições, abrindo-se-lhes prazo
arts. 6º, 7º e 8º desta Lei. para manifestação de interesse.

61
§ 2ºO Comando do Exército encaminhará a relação Art. 32. Os possuidores e proprietários de arma de
das armas a serem doadas ao juiz competente, que fogo poderão entregá-la, espontaneamente, me-
determinará o seu perdimento em favor da instituição diante recibo, e, presumindo-se de boa-fé, serão
beneficiada. indenizados, na forma do regulamento, ficando ex-
§ 3ºO transporte dasarmas de fogo doadas será de tinta a punibilidade de eventual posse irregular da
responsabilidade da instituição beneficiada, que pro- referida arma.
cederá ao seu cadastramento no Sinarm ou no Sigma Art. 33. Será aplicada multa de R$ 100.000,00 (cem
§ 4º(VETADO) mil reais) a R$ 300.000,00 (trezentos mil reais), con-
§ 5ºO Poder Judiciário instituirá instrumentos para o forme especificar o regulamento desta Lei:
encaminhamento ao Sinarm ou ao Sigma, conforme I – à empresa de transporte aéreo, rodoviário, ferroviá-
se trate de arma de uso permitido ou de uso restrito, rio, marítimo, fluvial ou lacustre que deliberadamente,
semestralmente, da relação de armas acauteladas em por qualquer meio, faça, promova, facilite ou permita
juízo, mencionando suas características e o local onde o transporte de arma ou munição sem a devida au-
se encontram. torização ou com inobservância das normas de segu-
Art. 26. São vedadas a fabricação, a venda, a co- rança;
mercialização e a importação de brinquedos, ré- II – à empresa de produção ou comércio de armamen-
plicas e simulacros de armas de fogo, que com es- tos que realize publicidade para venda, estimulando o
tas se possam confundir. uso indiscriminado de armas de fogo, exceto nas pu-
Parágrafo único. Excetuam-se da proibição as réplicas blicações especializadas.
e os simulacros destinados à instrução, ao adestra- Art. 34. Os promotores de eventos em locais fecha-
mento, ou à coleção de usuário autorizado, nas condi- dos, com aglomeração superior a 1000 (um mil)
ções fixadas pelo Comando do Exército. pessoas, adotarão, sob pena de responsabilidade, as
Art. 27. Caberá ao Comando do Exército autorizar, providências necessárias para evitar o ingresso de
excepcionalmente, a aquisição de armas de fogo pessoas armadas, ressalvados os eventos garantidos
pelo inciso VI do art. 5º da Constituição Federal.
de uso restrito.
Parágrafo único. As empresas responsáveis pela pres-
Parágrafo único. O disposto neste artigo não se aplica
tação dos serviços de transporte internacional e in-
às aquisições dos Comandos Militares.
terestadual de passageiros adotarão as providências
Art. 28.É vedado ao menor de 25 (vinte e cinco)
necessárias para evitar o embarque de passageiros
anos adquirir arma de fogo, ressalvados os inte-
armados.
grantes das entidades constantes dos incisos I, II, III, V,
VI, VII e X do caput do art. 6º desta Lei.
CAPÍTULO VI
Art. 29. As autorizações de porte de armas de fogo
DISPOSIÇÕES FINAIS
já concedidas expirar-se-ão 90 (noventa) dias após a
publicação desta Lei. Art. 35. É proibida a comercialização de arma de
Parágrafo único. O detentor de autorização com pra- fogo e munição em todo o território nacional, sal-
zo de validade superior a 90 (noventa) dias poderá vo para as entidades previstas no art. 6º desta Lei.
renová-la, perante a Polícia Federal, nas condições dos § 1º Este dispositivo, para entrar em vigor, dependerá
arts. 4º, 6º e 10 desta Lei, no prazo de 90 (noventa) de aprovação mediante referendo popular, a ser reali-
dias após sua publicação, sem ônus para o requerente. zado em outubro de 2005.
Art. 30.Os possuidores e proprietários de arma de fogo § 2º Em caso de aprovação do referendo popular, o
de uso permitido ainda não registrada deverão solici- disposto neste artigo entrará em vigor na data de pu-
tar seu registro até o dia 31 de dezembro de 2008, me- blicação de seu resultado pelo Tribunal Superior Elei-
diante apresentação de documento de identificação toral.
pessoal e comprovante de residência fixa, acompa- Art. 36. É revogada a Lei nº 9.437, de 20 de fevereiro
nhados de nota fiscal de compra ou comprovação da de 1997.
origem lícita da posse, pelos meios de prova admitidos Art. 37. Esta Lei entra em vigor na data de sua publi-
em direito, ou declaração firmada na qual constem as cação.
características da arma e a sua condição de proprie-
tário, ficando este dispensado do pagamento de taxas Brasília, 22 de dezembro de 2003; 182º da Indepen-
e do cumprimento das demais exigências constantes dência e 115º da República.
dos incisos I a III do caput do art. 4º desta Lei.
Parágrafo único.Para fins do cumprimento do disposto
no caput deste artigo, o proprietário de arma de fogo
LEGISLAÇÃO ESPECÍFICA

poderá obter, no Departamento de Polícia Federal,


certificado de registro provisório, expedido na forma
do § 4º do art. 5º desta Lei.
Art. 31. Os possuidores e proprietários de armas de
fogo adquiridas regularmente poderão, a qual-
quer tempo, entregá-las à Polícia Federal, me-
diante recibo e indenização, nos termos do regula-
mento desta Lei.

62
II - as armas de fogo das empresas de segurança pri-
DECRETO Nº 5.123, DE 1º DE JULHO DE 2004; vada e de transporte de valores; e
III - as armas de fogo de uso permitido dos integran-
tes dos órgãos, instituições e corporações menciona-
dos no inciso II do art. 6° da Lei no 10.826, de 2003.
Regulamenta a Lei no 10.826, de 22 de dezembro de
§ 3oA apreensão das armas de fogo a que se refere
2003, que dispõe sobre registro, posse e comercialização
o inciso II do §1° deste artigo deverá ser imediata-
de armas de fogo e munição, sobre o Sistema Nacional
mente comunicada à Policia Federal, pela autoridade
de Armas - SINARM e define crimes.
competente, podendo ser recolhidas aos depósitos do
O PRESIDENTE DA REPÚBLICA, no uso da atribuição
Comando do Exército, para guarda, a critério da mes-
que lhe confere o art. 84, inciso IV, da Constituição, e ten-
ma autoridade.
do em vista o disposto na Lei no 10.826, de 22 de dezem-
§ 4 ° O cadastramento das armas de fogo de que trata
bro de 2003,
o inciso I do § 1° observará as especificações e os pro-
cedimentos estabelecidos pelo Departamento de Po-
DECRETA:
lícia Federal. (Incluído pelo Decreto nº 6.715, de 2008).
Art. 2° O SIGMA, instituído no Ministério da Defesa, no
CAPÍTULO I
âmbito do Comando do Exército, com circunscrição
DOS SISTEMAS DE CONTROLE DE ARMAS DE FOGO
em todo o território nacional, tem por finalidade man-
ter cadastro geral, permanente e integrado das armas
Art. 1° O Sistema Nacional de Armas - SINARM, ins-
de fogo importadas, produzidas e vendidas no país,
tituído no Ministério da Justiça, no âmbito da Polícia
de competência do SIGMA, e das armas de fogo que
Federal, com circunscrição em todo o território na-
constem dos registros próprios.
cional e competência estabelecida pelo caput e inci-
§ 1° Serão cadastradas no SIGMA:
sos do art. 2° da Lei no 10.826, de 22 de dezembro
I - as armas de fogo institucionais, de porte e portá-
de 2003, tem por finalidade manter cadastro geral,
teis, constantes de registros próprios:
integrado e permanente das armas de fogo importa-
a) das Forças Armadas;
das, produzidas e vendidas no país, de competência
b) das Polícias Militares e Corpos de Bombeiros Mili-
do SINARM, e o controle dos registros dessas armas.
tares;
§ 1° Serão cadastradas no SINARM:
c) da Agência Brasileira de Inteligência; e
I - as armas de fogo institucionais, constantes de re-
d) do Gabinete de Segurança Institucional da Presi-
gistros próprios:
dência da República;
a) da Polícia Federal;
II - as armas de fogo dos integrantes das Forças Ar-
b) da Polícia Rodoviária Federal;
madas, da Agência Brasileira de Inteligência e do Ga-
c) das Polícias Civis;
binete de Segurança Institucional da Presidência da
d) dos órgãos policiais da Câmara dos Deputados e
República, constantes de registros próprios;
do Senado Federal, referidos nos arts. 51, inciso IV, e
III - as informações relativas às exportações de armas
52, inciso XIII da Constituição;
de fogo, munições e demais produtos controlados,
e) dos integrantes do quadro efetivo dos agentes e
devendo o Comando do Exército manter sua atuali-
guardas prisionais, dos integrantes das escoltas de
zação;
presos e das Guardas Portuárias;
IV - as armas de fogo importadas ou adquiridas no
f) das Guardas Municipais; e
país para fins de testes e avaliação técnica; e
g) dos órgãos públicos não mencionados nas alíneas
V - as armas de fogo obsoletas.
anteriores, cujos servidores tenham autorização legal
§ 2° Serão registradas no Comando do Exército e ca-
para portar arma de fogo em serviço, em razão das
dastradas no SIGMA:
atividades que desempenhem, nos termos do caput
I - as armas de fogo de colecionadores, atiradores e
do art. 6o da Lei no 10.826, de 2003.
caçadores; e
II - as armas de fogo apreendidas, que não constem
II - as armas de fogo das representações diplomáticas.
dos cadastros do SINARM ou Sistema de Gerencia-
Art. 3° Entende-se por registros próprios, para os fins
mento Militar de Armas - SIGMA, inclusive as vincu-
deste Decreto, os feitos pelas instituições, órgãos e
ladas a procedimentos policiais e judiciais, mediante
corporações em documentos oficiais de caráter per-
comunicação das autoridades competentes à Polícia
manente.
Federal;
Art. 4° A aquisição de armas de fogo, diretamente da
III - as armas de fogo de uso restrito dos integrantes
fábrica, será precedida de autorização do Comando
dos órgãos, instituições e corporações mencionados
do Exército.
LEGISLAÇÃO ESPECÍFICA

no inciso II do art. 6o da Lei no 10.826, de 2003; e


Art. 5° Os dados necessários ao cadastro mediante
IV - as armas de fogo de uso restrito, salvo aquelas
registro, a que se refere o inciso IX do art. 2° da Lei
mencionadas no inciso II, do §1°, do art. 2° deste De-
no 10.826, de 2003, serão fornecidos ao SINARM pelo
creto.
Comando do Exército.
§ 2° Serão registradas na Polícia Federal e cadastra-
Art. 6° Os dados necessários ao cadastro da identifica-
das no SINARM:
ção do cano da arma, das características das impres-
I - as armas de fogo adquiridas pelo cidadão com
sões de raiamento e microestriamento de projetil
atendimento aos requisitos do art. 4o da Lei no
disparado, a marca do percutor e extrator no estojo
10.826, de 2003;

63
do cartucho deflagrado pela arma de que trata o in- VI - comprovar, em seu pedido de aquisição do Certifi-
ciso X do art. 2° da Lei no 10.826, de 2003, serão cado de Registro de Arma de Fogo e periodicamente, a
disciplinados em norma específica da Polícia Federal, capacidade técnica para o manuseio de arma de fogo;
ouvido o Comando do Exército, cabendo às fábricas e (Redação dada pelo Decreto nº 8.935, de 2016)
de armas de fogo o envio das informações necessá- VII - comprovar aptidão psicológica para o manuseio
rias ao órgão responsável da Polícia Federal. de arma de fogo, atestada em laudo conclusivo for-
Parágrafo único. A norma específica de que trata este necido por psicólogo do quadro da Polícia Federal
artigo será expedida no prazo de cento e oitenta dias. ou por esta credenciado.
Art. 7° As fábricas de armas de fogo fornecerão à Po- VIII - na hipótese de residência habitada também por
lícia Federal, para fins de cadastro, quando da saída criança, adolescente ou pessoa com deficiência men-
do estoque, relação das armas produzidas, que de- tal, apresentar declaração de que a sua residência
vam constar do SINARM, na conformidade do art. 2o possui cofre ou local seguro com tranca para arma-
da Lei no 10.826, de 2003, com suas características e zenamento.(Incluído pelo Decreto nº 9.685, de 2019)
os dados dos adquirentes. § 1ºPresume-se a veracidade dos fatos e das circuns-
Art. 8° As empresas autorizadas a comercializar ar- tâncias afirmadas na declaração de efetiva necessi-
mas de fogo encaminharão à Polícia Federal, quaren- dade a que se refere o inciso I do caput, a qual será
ta e oito horas após a efetivação da venda, os dados examinada pela Polícia Federal nos termos deste ar-
que identifiquem a arma e o comprador. tigo.(Redação dada pelo Decreto nº 9.685, de 2019)
Art. 9° Os dados do SINARM e do SIGMA serão in- § 2oO indeferimento do pedido deverá ser funda-
terligados e compartilhados no prazo máximo de um mentado e comunicado ao interessado em docu-
ano. mento próprio.
Parágrafo único .Os Ministros da Justiça e da Defesa § 3oO comprovante de capacitação técnica, de que
estabelecerão no prazo máximo de um ano os níveis trata o inciso VI do caput, deverá ser expedido por
de acesso aos cadastros mencionados no caput. instrutor de armamento e tiro credenciado pela Polí-
cia Federal e deverá atestar, necessariamente: (Reda-
CAPÍTULO II ção dada pelo Decreto nº 6.715, de 2008).
DA ARMA DE FOGO I - conhecimento da conceituação e normas de segu-
Seção I rança pertinentes à arma de fogo;
Das Definições II - conhecimento básico dos componentes e partes
da arma de fogo; e
Art. 10.Arma de fogo de uso permitido é aquela cuja III - habilidade do uso da arma de fogo demonstra-
utilização é autorizada a pessoas físicas, bem como a da, pelo interessado, em estande de tiro credenciado
pessoas jurídicas, de acordo com as normas do Co- pelo Comando do Exército.
mando do Exército e nas condições previstas na Lei § 4° Após a apresentação dos documentos referidos
no 10.826, de 2003. nos incisos III a VII do caput, havendo manifestação
Art. 11.Arma de fogo de uso restrito é aquela de uso favorável do órgão competente mencionada no §1o,
exclusivo das Forças Armadas, de instituições de se- será expedida, pelo SINARM, no prazo máximo de
gurança pública e de pessoas físicas e jurídicas habi- trinta dias, em nome do interessado, a autorização
litadas, devidamente autorizadas pelo Comando do para a aquisição da arma de fogo indicada.
Exército, de acordo com legislação específica. § 5oÉ intransferível a autorização para a aquisição da
arma de fogo, de que trata o §4o deste artigo.
Seção II § 6oEstá dispensado da comprovação dos requisitos
Da Aquisição e do Registro da Arma de Fogo de a que se referem os incisos VI e VII do caput o inte-
Uso Permitido ressado em adquirir arma de fogo de uso permiti-
do que comprove estar autorizado a portar arma da
Art. 12. Para adquirir arma de fogo de uso permitido mesma espécie daquela a ser adquirida, desde que
o interessado deverá: o porte de arma de fogo esteja válido e o interessa-
I - declarar efetiva necessidade; do tenha se submetido a avaliações em período não
II - ter, no mínimo, vinte e cinco anos; superior a um ano, contado do pedido de aquisição.
III - apresentar original e cópia, ou cópia autentica- (Incluído pelo Decreto nº 6.715, de 2008).
da, de documento de identificação pessoal; (Redação § 7ºPara a aquisição de armas de fogo de uso permi-
dada pelo Decreto nº 6.715, de 2008). tido, considera-se presente a efetiva necessidade nas
IV - comprovar, em seu pedido de aquisição do Cer- seguintes hipóteses: (Incluído pelo Decreto nº 9.685,
tificado de Registro de Arma de Fogo e periodica- de 2019)
LEGISLAÇÃO ESPECÍFICA

mente, a idoneidade e a inexistência de inquérito I - agentes públicos, inclusive os inativos: (Incluído


policial ou processo criminal, por meio de certidões pelo Decreto nº 9.685, de 2019)
de antecedentes criminais da Justiça Federal, Esta- a) da área de segurança pública; (Incluído pelo De-
dual, Militar e Eleitoral, que poderão ser fornecidas creto nº 9.685, de 2019)
por meio eletrônico; (Redação dada pelo Decreto nº b) integrantes das carreiras da Agência Brasileira de
8.935, de 2016) Inteligência; (Incluído pelo Decreto nº 9.685, de 2019)
V - apresentar documento comprobatório de ocupa- c) da administração penitenciária; (Incluído pelo De-
ção lícita e de residência certa; creto nº 9.685, de 2019)

64
d) do sistema socioeducativo, desde que lotados nas Art. 14.É obrigatório o registro da arma de fogo, no
unidades de internação a que se refere o inciso VI do SINARM ou no SIGMA, excetuadas as obsoletas.
caput do art. 112 da Lei nº 8.069, de 13 de julho de Art. 15.O registro da arma de fogo de uso permitido
1990; e (Incluído pelo Decreto nº 9.685, de 2019) deverá conter, no mínimo, os seguintes dados:
e) envolvidos no exercício de atividades de poder I - do interessado:
de polícia administrativa ou de correição em caráter a) nome, filiação, data e local de nascimento;
permanente;(Incluído pelo Decreto nº 9.685, de 2019) b) endereço residencial;
II - militares ativos e inativos; (Incluído pelo Decreto c) endereço da empresa ou órgão em que trabalhe;
nº 9.685, de 2019) d) profissão;
III - residentes em área rural; (Incluído pelo Decreto e) número da cédula de identidade, data da expedição,
nº 9.685, de 2019) órgão expedidor e Unidade da Federação; e
IV - residentes em áreas urbanas com elevados índi- f) número do Cadastro de Pessoa Física - CPF ou Ca-
ces de violência, assim consideradas aquelas localiza- dastro Nacional de Pessoa Jurídica - CNPJ;
II - da arma:
das em unidades federativas com índices anuais de
a) número do cadastro no SINARM;
mais de dez homicídios por cem mil habitantes, no
b) identificação do fabricante e do vendedor;
ano de 2016, conforme os dados do Atlas da Violên-
c) número e data da nota Fiscal de venda;
cia 2018, produzido pelo Instituto de Pesquisa Econô- d) espécie, marca, modelo e número de série;
mica Aplicada e pelo Fórum Brasileiro de Segurança e) calibre e capacidade de cartuchos;
Pública;(Incluído pelo Decreto nº 9.685, de 2019) f) tipo de funcionamento;
V - titulares ou responsáveis legais de estabelecimen- g) quantidade de canos e comprimento;
tos comerciais ou industriais; e(Incluído pelo Decreto h) tipo de alma (lisa ou raiada);
nº 9.685, de 2019) i) quantidade de raias e sentido; e
VI - colecionadores, atiradores e caçadores, devida- j) número de série gravado no cano da arma.
mente registrados no Comando do Exército.(Incluído Parágrafo único.Os dados de que tratam o inciso I e a
pelo Decreto nº 9.685, de 2019) alínea “b” do inciso II do caput serão substituídos pelo
§ 8ºO disposto no § 7º se aplica para a aquisição de número de matrícula funcional, na hipótese em que o
até quatro armas de fogo de uso permitido e não ex- cadastro no SIGMA ou no SINARM estiver relaciona-
clui a caracterização da efetiva necessidade se pre- do com armas de fogo pertencentes a integrantes da
sentes outros fatos e circunstâncias que a justifiquem, Agência Brasileira de Inteligência.(Incluído pelo Decre-
inclusive para a aquisição de armas de fogo de uso to nº 9.685, de 2019)
permitido em quantidade superior a esse limite, con- Art. 16. O Certificado de Registro de Arma de Fogo
forme legislação vigente. (Incluído pelo Decreto nº expedido pela Polícia Federal, precedido de cadastro
9.685, de 2019) no SINARM, tem validade em todo o território nacio-
§ 9º Constituem razões para o indeferimento do pedi- nal e autoriza o seu proprietário a manter a arma de
do ou para o cancelamento do registro:(Incluído pelo fogo exclusivamente no interior de sua residência ou
Decreto nº 9.685, de 2019) dependência desta, ou, ainda, no seu local de trabalho,
I - a ausência dos requisitos a que se referem os inci- desde que seja ele o titular ou o responsável legal pelo
sos I a VII do caput; e (Incluído pelo Decreto nº 9.685, estabelecimento ou empresa. (Redação dada pelo De-
de 2019) creto nº 6.715, de 2008).
II - quando houver comprovação de que o § 1° Para os efeitos do disposto no caput deste artigo
considerar-se-á titular do estabelecimento ou empresa
requerente:(Incluído pelo Decreto nº 9.685, de 2019)
todo aquele assim definido em contrato social, e res-
a) prestou a declaração de efetiva necessidade com
ponsável legal o designado em contrato individual de
afirmações falsas;(Incluído pelo Decreto nº 9.685, de trabalho, com poderes de gerência.
2019) § 2ºOs requisitos de que tratam os incisos IV, V, VI e VII
b) mantém vínculo com grupos criminosos; e(Incluído do caput do art. 12 deverão ser comprovados, periodi-
pelo Decreto nº 9.685, de 2019) camente, a cada dez anos, junto à Polícia Federal, para
c) age como pessoa interposta de quem não preen- fins de renovação do Certificado de Registro.(Redação
che os requisitos a que se referem os incisos I a VII do dada pelo Decreto nº 9.685, de 2019)
caput.(Incluído pelo Decreto nº 9.685, de 2019) § 2º- A.(Revogado pelo Decreto nº 9.685, de 2019)
§ 10° A inobservância do disposto no inciso VIII do § 3º (Revogado pelo Decreto nº 6.715, de 2008).
caput sujeitará o interessado à pena prevista no art. § 4º O disposto nos § 2º e § 2º-A não se aplica, para a
13 da Lei nº 10.826, de 2003. (Incluído pelo Decreto aquisição e a renovação do Certificado de Registro de
nº 9.685, de 2019) Arma de Fogo, aos integrantes dos órgãos, das institui-
Art. 13. A transferência de propriedade da arma de ções e das corporações, mencionados nos incisos I e II
LEGISLAÇÃO ESPECÍFICA

fogo, por qualquer das formas em direito admitidas, do caput do art. 6º da Lei nº 10.826, de 2003.(Redação
entre particulares, sejam pessoas físicas ou jurídicas, dada pelo Decreto nº 8.935, de 2016)
estará sujeita à prévia autorização da Polícia Federal, Art. 17.O proprietário de arma de fogo é obrigado a
aplicando-se ao interessado na aquisição as disposi- comunicar, imediatamente, à unidade policial local, o
ções do art. 12 deste Decreto. extravio, furto ou roubo de arma de fogo ou do Certi-
Parágrafo único.A transferência de arma de fogo re- ficado de Registro de Arma de Fogo, bem como a sua
gistrada no Comando do Exército será autorizada recuperação. (Redação dada pelo Decreto nº 6.715,
pela instituição e cadastrada no SIGMA. de 2008).

65
§ 1° A unidade policial deverá, em quarenta e oito Seção IV
horas, remeter as informações coletadas à Polícia Do Comércio Especializado de Armas de Fogo e
Federal, para fins de cadastro no SINARM. (Redação Munições
dada pelo Decreto nº 6.715, de 2008).
§ 2° No caso de arma de fogo de uso restrito, a Polí- Art. 19.É proibida a venda de armas de fogo, muni-
cia Federal repassará as informações ao Comando do ções e demais produtos controlados, de uso restrito,
Exército, para fins de cadastro no SIGMA. (Redação no comércio.
dada pelo Decreto nº 6.715, de 2008). Art. 20.O estabelecimento que comercializar arma de
§ 3oNos casos previstos no caput, o proprietário de- fogo de uso permitido em território nacional é obri-
verá, também, comunicar o ocorrido à Polícia Federal gado a comunicar à Polícia Federal, mensalmente, as
ou ao Comando do Exército, encaminhando, se for o vendas que efetuar e a quantidade de armas em es-
caso, cópia do Boletim de Ocorrência. toque, respondendo legalmente por essas mercado-
rias, que ficarão registradas como de sua proprieda-
Seção III de, de forma precária, enquanto não forem vendidas,
Da Aquisição e Registro da Arma de Fogo de Uso sujeitos seus responsáveis às penas previstas em lei.
(Redação dada pelo Decreto nº 6.715, de 2008).
Restrito
Art. 21.A comercialização de acessórios de armas
de fogo e de munições, incluídos estojos, espoletas,
Art. 18.Compete ao Comando do Exército autorizar a
pólvora e projéteis, só poderá ser efetuada em esta-
aquisição e registrar as armas de fogo de uso restrito.
belecimento credenciado pela Polícia Federal e pelo
§ 1oAs armas de que trata o caput serão cadastradas comando do Exército que manterão um cadastro dos
no SIGMA e no SINARM, conforme o caso. comerciantes.
§ 2oO registro de arma de fogo de uso restrito, de § 1oQuando se tratar de munição industrializada,
que trata o caput deste artigo, deverá conter as se- a venda ficará condicionada à apresentação pelo
guintes informações: adquirente, do Certificado de Registro de Arma de
I - do interessado: Fogo válido, e ficará restrita ao calibre corresponden-
a) nome, filiação, data e local de nascimento; te à arma registrada.
b) endereço residencial; § 2oOs acessórios e a quantidade de munição que
c) endereço da empresa ou órgão em que trabalhe; cada proprietário de arma de fogo poderá adquirir
d) profissão; serão fixados em Portaria do Ministério da Defesa,
e) número da cédula de identidade, data da expedi- ouvido o Ministério da Justiça.
ção, órgão expedidor e Unidade da Federação; e § 3oO estabelecimento mencionado no caput deste
f) número do Cadastro de Pessoa Física - CPF ou Ca- artigo deverá manter à disposição da Polícia Federal
dastro Nacional de Pessoa Jurídica - CNPJ; e do Comando do Exército os estoques e a relação
II - da arma: das vendas efetuadas mensalmente, pelo prazo de
a) número do cadastro no SINARM; cinco anos.
b) identificação do fabricante e do vendedor;
c) número e data da nota Fiscal de venda; CAPÍTULO III
d) espécie, marca, modelo e número de série; DO PORTE E DO TRÂNSITO DA ARMA DE FOGO
e) calibre e capacidade de cartuchos; Seção I
f) tipo de funcionamento; Do Porte
g) quantidade de canos e comprimento;
h) tipo de alma (lisa ou raiada); Art. 22.O Porte de Arma de Fogo de uso permitido,
vinculado ao prévio registro da arma e ao cadastro
i) quantidade de raias e sentido; e
no SINARM, será expedido pela Polícia Federal, em
j) número de série gravado no cano da arma.
todo o território nacional, em caráter excepcional,
§ 3ºOs requisitos de que tratam os incisos IV, V, VI e
desde que atendidos os requisitos previstos nos in-
VII do caput do art. 12 deverão ser comprovados, pe-
cisos I, II e III do § 1o do art. 10 da Lei no 10.826, de
riodicamente, a cada dez anos, junto ao Comando do 2003. (Redação dada pelo Decreto nº 6.715, de 2008).
Exército, para fins de renovação do Certificado de Re- Parágrafo único.A taxa estipulada para o Porte de
gistro.(Redação dada pelo Decreto nº 9.685, de 2019) Arma de Fogo somente será recolhida após a análise
§ 4oNão se aplica aos integrantes dos órgãos, insti- e a aprovação dos documentos apresentados.
tuições e corporações mencionados nos incisos I e II Art. 23.O Porte de Arma de Fogo é documento obri-
do art. 6o da Lei no 10.826, de 2003, o disposto no § gatório para a condução da arma e deverá conter os
3o deste artigo. seguintes dados:
LEGISLAÇÃO ESPECÍFICA

§ 5ºOs dados de que tratam o inciso I e a alínea “b” I - abrangência territorial;


do inciso II do § 2º serão substituídos pelo número II - eficácia temporal;
de matrícula funcional, na hipótese em que o cadas- III - características da arma;
tro no SIGMA ou no SINARM estiver relacionado com IV - número do cadastro da arma no SINARM; (Reda-
armas de fogo pertencentes a integrantes da Agên- ção dada pelo Decreto nº 6.715, de 2008).
cia Brasileira de Inteligência.(Incluído pelo Decreto nº V - identificação do proprietário da arma; e
9.685, de 2019) VI - assinatura, cargo e função da autoridade con-
cedente.

66
Art. 24.O Porte de Arma de Fogo é pessoal, intrans- Art. 28.O proprietário de arma de fogo de uso permi-
ferível e revogável a qualquer tempo, sendo válido tido registrada, em caso de mudança de domicílio ou
apenas com relação à arma nele especificada e com outra situação que implique o transporte da arma,
a apresentação do documento de identificação do deverá solicitar guia de trânsito à Polícia Federal para
portador. (Redação dada pelo Decreto nº 6.715, de as armas de fogo cadastradas no SINARM, na forma
2008). estabelecida pelo Departamento de Polícia Federal.
Art. 24-A.Para portar a arma de fogo adquirida nos (Redação dada pelo Decreto nº 6.715, de 2008).
termos do § 6o do art. 12, o proprietário deverá soli- Art. 29.Observado o princípio da reciprocidade pre-
citar a expedição do respectivo documento de porte, visto em convenções internacionais, poderá ser auto-
que observará o disposto no art. 23 e terá a mesma rizado o Porte de Arma de Fogo pela Polícia Federal,
validade do documento referente à primeira arma. a diplomatas de missões diplomáticas e consulares
(Incluído pelo Decreto nº 6.715, de 2008). acreditadas junto ao Governo Brasileiro, e a agentes
Art. 25.O titular do Porte de Arma de Fogo deverá de segurança de dignitários estrangeiros durante a
comunicar imediatamente: permanência no país, independentemente dos requi-
I - a mudança de domicílio, ao órgão expedidor do sitos estabelecidos neste Decreto.
Porte de Arma de Fogo; e Art. 29-A.Caberá ao Departamento de Polícia Federal
II - o extravio, furto ou roubo da arma de fogo, à estabelecer os procedimentos relativos à concessão
Unidade Policial mais próxima e, posteriormente, à e renovação do Porte de Arma de Fogo. (Incluído
Polícia Federal. pelo Decreto nº 6.715, de 2008).
Parágrafo único.A inobservância do disposto nes-
te artigo implicará na suspensão do Porte de Arma Seção II
de Fogo, por prazo a ser estipulado pela autoridade Dos Atiradores, Caçadores e Colecionadores
concedente. Subseção I
Art. 26.O titular de porte de arma de fogo para de- Da Prática de Tiro Desportivo
fesa pessoal concedido nos termos do art. 10 da Lei
no 10.826, de 2003, não poderá conduzi-la osten- Art. 30.As agremiações esportivas e as empresas de
instrução de tiro, os colecionadores, atiradores e ca-
sivamente ou com ela adentrar ou permanecer em
çadores serão registrados no Comando do Exército,
locais públicos, tais como igrejas, escolas, estádios
ao qual caberá estabelecer normas e verificar o cum-
desportivos, clubes, agências bancárias ou outros
primento das condições de segurança dos depósitos
locais onde haja aglomeração de pessoas em virtu-
das armas de fogo, munições e equipamentos de re-
de de eventos de qualquer natureza. (Redação dada
carga.
pelo Decreto nº 6.715, de 2008).
§ 1oAs armas pertencentes às entidades menciona-
§ 1oA inobservância do disposto neste artigo im-
das no caput e seus integrantes terão autorização
plicará na cassação do Porte de Arma de Fogo e na
para porte de trânsito (guia de tráfego) a ser expedi-
apreensão da arma, pela autoridade competente, da pelo Comando do Exército.
que adotará as medidas legais pertinentes. § 2oA prática de tiro desportivo por menores de
§ 2oAplica-se o disposto no §1o deste artigo, quando dezoito anos deverá ser autorizada judicialmente e
o titular do Porte de Arma de Fogo esteja portando deve restringir-se aos locais autorizados pelo Co-
o armamento em estado de embriaguez ou sob o mando do Exército, utilizando arma da agremiação
efeito de drogas ou medicamentos que provoquem ou do responsável quando por este acompanhado.
alteração do desempenho intelectual ou motor. § 3oA prática de tiro desportivo por maiores de de-
Art. 27.Será concedido pela Polícia Federal, nos ter- zoito anos e menores de vinte e cinco anos pode ser
mos do § 5o do art. 6o da Lei no 10.826, de 2003, feita utilizando arma de sua propriedade, registrada
o Porte de Arma de Fogo, na categoria “caçador de com amparo na Lei no 9.437, de 20 de fevereiro de
subsistência”, de uma arma portátil, de uso permiti- 1997, de agremiação ou arma registrada e cedida por
do, de tiro simples, com um ou dois canos, de alma outro desportista.
lisa e de calibre igual ou inferior a 16, desde que o § 4oAs entidades de tiro desportivo e as empresas de
interessado comprove a efetiva necessidade em re- instrução de tiro poderão fornecer a seus associados
querimento ao qual deverão ser anexados os seguin- e clientes, desde que obtida autorização específica e
tes documentos: obedecidas as condições e requisitos estabelecidos
I - documento comprobatório de residência em área em ato do Comando do Exército, munição recarrega-
rural ou certidão equivalente expedida por órgão da para uso exclusivo nas dependências da institui-
municipal; (Redação dada pelo Decreto nº 6.715, de ção em provas, cursos e treinamento.(Incluído pelo
LEGISLAÇÃO ESPECÍFICA

2008). Decreto nº 9.685, de 2019)


II - original e cópia, ou cópia autenticada, do do- Art. 31.A entrada de arma de fogo e munição no país,
cumento de identificação pessoal; e (Redação dada como bagagem de atletas, para competições inter-
pelo Decreto nº 6.715, de 2008). nacionais será autorizada pelo Comando do Exército.
III - atestado de bons antecedentes. § 1oO Porte de Trânsito das armas a serem utilizadas
Parágrafo único.Aplicam-se ao portador do Porte de por delegações estrangeiras em competição oficial
Arma de Fogo mencionado neste artigo as demais de tiro no país será expedido pelo Comando do Exér-
obrigações estabelecidas neste Decreto. cito.

67
§ 2oOs responsáveis e os integrantes pelas delega- de arma de seus agentes públicos ou políticos esta-
ções estrangeiras e brasileiras em competição oficial belecidos em lei própria, na forma do caput do art.
de tiro no país transportarão suas armas desmuni- 6o da Lei no 10.826, de 2003, deverão encaminhar
ciadas. à Polícia Federal a relação dos autorizados a portar
arma de fogo, observando-se, no que couber, o dis-
Subseção II posto no art. 26. (Incluído pelo Decreto nº 6.715, de
Dos Colecionadores e Caçadores 2008).
§ 4oNão será concedida a autorização para o porte
Art. 32.O Porte de Trânsito das armas de fogo de de arma de fogo de que trata o art. 22 a integrantes
colecionadores e caçadores será expedido pelo Co- de órgãos, instituições e corporações não autoriza-
mando do Exército. dos a portar arma de fogo fora de serviço, exceto se
Parágrafo único.Os colecionadores e caçadores comprovarem o risco à sua integridade física, obser-
transportarão suas armas desmuniciadas. vando-se o disposto no art. 11 da Lei no 10.826, de
Subseção III 2003. (Incluído pelo Decreto nº 6.715, de 2008).
Dos Integrantes e das Instituições Mencionadas no § 5oO porte de que tratam os incisos V, VI e X do
Art. 6o da Lei no 10.826, de 2003 caput do art. 6o da Lei no 10.826, de 2003, e aquele
Art. 33.O Porte de Arma de Fogo é deferido aos mi- previsto em lei própria, na forma do caput do men-
litares das Forças Armadas, aos policiais federais e cionado artigo, serão concedidos, exclusivamente,
estaduais e do Distrito Federal, civis e militares, aos para defesa pessoal, sendo vedado aos seus respec-
Corpos de Bombeiros Militares, bem como aos poli- tivos titulares o porte ostensivo da arma de fogo. (In-
ciais da Câmara dos Deputados e do Senado Federal cluído pelo Decreto nº 6.715, de 2008).
em razão do desempenho de suas funções institu- § 6oA vedação prevista no parágrafo 5o não se aplica
cionais. aos servidores designados para execução da ativida-
§ 1oO Porte de Arma de Fogo das praças das Forças de fiscalizatória do Instituto Brasileiro do Meio Am-
Armadas e dos Policiais e Corpos de Bombeiros Mi- biente e dos Recursos Naturais Renováveis - IBAMA
litares é regulado em norma específica, por atos dos e do Instituto Chico Mendes de Conservação da Bio-
Comandantes das Forças Singulares e dos Coman- diversidade - Instituto Chico Mendes. (Incluído pelo
dantes-Gerais das Corporações. Decreto nº 6.817, de 2009)
§ 2oOs integrantes das polícias civis estaduais e das Art. 35.Poderá ser autorizado, em casos excepcionais,
Forças Auxiliares, quando no exercício de suas fun-
pelo órgão competente, o uso, em serviço, de arma
ções institucionais ou em trânsito, poderão portar
de fogo, de propriedade particular do integrante dos
arma de fogo fora da respectiva unidade federativa,
órgãos, instituições ou corporações mencionadas no
desde que expressamente autorizados pela institui-
inciso II do art. 6o da Lei no 10.826, de 2003.
ção a que pertençam, por prazo determinado, con-
§ 1oA autorização mencionada no caput será regula-
forme estabelecido em normas próprias.
mentada em ato próprio do órgão competente.
Art. 33-A.A autorização para o porte de arma de
§ 2oA arma de fogo de que trata este artigo deverá
fogo previsto em legislação própria, na forma do
ser conduzida com o seu respectivo Certificado de
caput do art. 6o da Lei no 10.826, de 2003, está con-
dicionada ao atendimento dos requisitos previstos Registro.
no inciso III do caput do art. 4o da mencionada Lei. Art. 35-A.As armas de fogo particulares de que trata
(Incluído pelo Decreto nº 6.715, de 2008). o art. 35, e as institucionais não brasonadas, deverão
Art. 34.Os órgãos, instituições e corporações men- ser conduzidas com o seu respectivo Certificado de
cionados nos incisos I, II, III, V, VI, VII e X do caput Registro ou termo de cautela decorrente de autori-
do art. 6º da Lei nº 10.826, de 2003, estabelecerão, zação judicial para uso, sob pena de aplicação das
em normativos internos, os procedimentos relativos sanções penais cabíveis. (Incluído pelo Decreto nº
às condições para a utilização das armas de fogo de 6.715, de 2008).
sua propriedade, ainda que fora do serviço. (Reda- Art. 36.A capacidade técnica e a aptidão psicológica
ção dada pelo Decreto nº 6.146, de 2007 para o manuseio de armas de fogo, para os integran-
§ 1oAs instituições mencionadas no inciso IV do art. tes das instituições descritas nos incisos III, IV, V, VI,
6o da Lei no 10.826, de 2003, estabelecerão em nor- VII e X do caput do art. 6º da Lei nº10.826, de 2003,
mas próprias os procedimentos relativos às condi- serão atestadas pela própria instituição, depois de
ções para a utilização, em serviço, das armas de fogo cumpridos os requisitos técnicos e psicológicos es-
de sua propriedade. tabelecidos pela Polícia Federal. (Redação dada pelo
§ 2oAs instituições, órgãos e corporações nos pro- Decreto nº 6.146, de 2007
LEGISLAÇÃO ESPECÍFICA

cedimentos descritos no caput, disciplinarão as Parágrafo único.Caberá à Polícia Federal expedir o


normas gerais de uso de arma de fogo de sua pro- Porte de Arma de Fogo para os guardas portuários.
priedade, fora do serviço, quando se tratar de locais (Redação dada pelo Decreto nº 8.935, de 2016)
onde haja aglomeração de pessoas, em virtude de Art. 37.Os integrantes das Forças Armadas e os ser-
evento de qualquer natureza, tais como no interior vidores dos órgãos, instituições e corporações men-
de igrejas, escolas, estádios desportivos, clubes, pú- cionados nos incisos II, V, VI e VII do caput do art. 6º
blicos e privados. da Lei nº 10.826, de 2003, transferidos para a reserva
§ 3oOs órgãos e instituições que tenham os portes remunerada ou aposentados, para conservarem a

68
autorização de porte de arma de fogo de sua pro- I - conceder autorização para o funcionamento dos
priedade deverão submeter-se, a cada cinco anos, cursos de formação de guardas municipais;
aos testes de avaliação psicológica a que faz menção II - fixar o currículo dos cursos de formação;
o inciso III do caput do art. 4º da Lei nº 10.826, de III - conceder Porte de Arma de Fogo;
2003(Redação dada pelo Decreto nº 8.935, de 2016) IV - fiscalizar os cursos mencionados no inciso II; e
§ 1oO cumprimento destes requisitos será atestado V - fiscalizar e controlar o armamento e a munição
pelas instituições, órgãos e corporações de vinculação. utilizados.
§ 2oNão se aplicam aos integrantes da reserva não Parágrafo único.As competências previstas nos incisos
remunerada das Forças Armadas e Auxiliares, as prer- I e II deste artigo não serão objeto de convênio.
rogativas mencionadas no caput. Art. 41.Compete ao Comando do Exército autorizar a
aquisição de armas de fogo e de munições para as
Subseção IV Guardas Municipais.
Das Empresas de Segurança Privada e de Transpor- Art. 42.O Porte de Arma de Fogo aos profissionais ci-
te de Valores tados nos incisos III e IV, do art. 6o, da Lei no 10.826,
de 2003, será concedido desde que comprovada a re-
Art. 38.A autorização para o uso de arma de fogo alização de treinamento técnico de, no mínimo, ses-
expedida pela Polícia Federal, em nome das empre- senta horas para armas de repetição e cem horas para
sas de segurança privada e de transporte de valores, arma semi-automática.
será precedida, necessariamente, da comprovação § 1oO treinamento de que trata o caput desse artigo
do preenchimento de todos os requisitos constantes deverá ter, no mínimo, sessenta e cinco por cento de
do art. 4o da Lei no 10.826, de 2003, pelos emprega- conteúdo prático.
dos autorizados a portar arma de fogo. § 2oO curso de formação dos profissionais das Guar-
§ 1oA autorização de que trata o caput é válida ape- das Municipais deverá conter técnicas de tiro defensi-
nas para a utilização da arma de fogo em serviço. vo e defesa pessoal.
§ 3oOs profissionais da Guarda Municipal deverão ser
§ 2oAs empresas de que trata o caput encaminharão,
submetidos a estágio de qualificação profissional por,
trimestralmente, à Polícia Federal, para cadastro no
no mínimo, oitenta horas ao ano.
SINARM, a relação nominal dos empregados auto-
§ 4oNão será concedido aos profissionais das Guardas
rizados a portar arma de fogo. (Redação dada pelo
Municipais Porte de Arma de Fogo de calibre restrito,
Decreto nº 6.715, de 2008).
privativos das forças policiais e forças armadas.
§ 3oA transferência de armas de fogo, por qualquer
Art. 43.O profissional da Guarda Municipal com Porte
motivo, entre estabelecimentos da mesma empresa
de Arma de Fogo deverá ser submetido, a cada dois
ou para empresa diversa, deverão ser previamente anos, a teste de capacidade psicológica e, sempre
autorizados pela Polícia Federal. que estiver envolvido em evento de disparo de arma
§ 4oDurante o trâmite do processo de transferên- de fogo em via pública, com ou sem vítimas, deverá
cia de armas de fogo de que trata o § 3o, a Polícia apresentar relatório circunstanciado, ao Comando da
Federal poderá, em caráter excepcional, autorizar a Guarda Civil e ao Órgão Corregedor para justificar o
empresa adquirente a utilizar as armas em fase de motivo da utilização da arma.
aquisição, em seus postos de serviço, antes da expe- Art. 44.A Polícia Federal poderá conceder Porte de
dição do novo Certificado de Registro. (Incluído pelo Arma de Fogo, nos termos no §3o do art. 6o, da Lei
Decreto nº 6.715, de 2008). no 10.826, de 2003, às Guardas Municipais dos mu-
Art. 39.É de responsabilidade das empresas de segu- nicípios que tenham criado corregedoria própria e
rança privada e de transportes de valores a guarda e autônoma, para a apuração de infrações disciplinares
armazenagem das armas, munições e acessórios de atribuídas aos servidores integrantes do Quadro da
sua propriedade, nos termos da legislação específica. Guarda Municipal.
Parágrafo único.A perda, furto, roubo ou outras for- Parágrafo único.A concessão a que se refere o ca-
mas de extravio de arma de fogo, acessório e mu- put dependerá, também, da existência de Ouvidoria,
nições que estejam sob a guarda das empresas de como órgão permanente, autônomo e independente,
segurança privada e de transporte de valores deverá com competência para fiscalizar, investigar, auditorar
ser comunicada à Polícia Federal, no prazo máximo e propor políticas de qualificação das atividades de-
de vinte e quatro horas, após a ocorrência do fato, senvolvidas pelos integrantes das Guardas Municipais.
sob pena de responsabilização do proprietário ou di- Art. 45. (Revogado pelo Decreto nº 5.871, de 2006).
retor responsável.
CAPÍTULO IV
Subseção V DAS DISPOSIÇÕES GERAIS, FINAIS E TRANSITÓRIAS
LEGISLAÇÃO ESPECÍFICA

Das guardas Municipais Seção I


Das Disposições Gerais
Art. 40.Cabe ao Ministério da Justiça, por intermédio
da Polícia Federal, diretamente ou mediante convê- Art. 46.O Ministro da Justiça designará as autoridades
nio com os órgãos de segurança pública dos Estados, policiais competentes, no âmbito da Polícia Federal,
do Distrito Federal ou dos Municípios, nos termos do para autorizar a aquisição e conceder o Porte de
§ 3o do art. 6o da Lei no 10.826, de 2003:(Redação Arma de Fogo, que terá validade máxima de cinco
dada pelo Decreto nº 6.715, de 2008). anos.

69
Art. 47.O Ministério da Justiça, por intermédio da Po- IV - expedir regulamentação específica para o con-
lícia Federal, poderá celebrar convênios com os ór- trole da fabricação, importação, comércio, trânsito e
gãos de segurança pública dos Estados e do Distrito utilização de simulacros de armas de fogo, conforme
Federal para possibilitar a integração, ao SINARM, o art. 26 da Lei no 10.826, de 2003.
dos acervos policiais de armas de fogo já existentes, Art. 51.A importação de armas de fogo, munições e
em cumprimento ao disposto no inciso VI do art. 2o acessórios de uso restrito está sujeita ao regime de
da Lei no 10.826, de 2003. (Redação dada pelo De- licenciamento não-automático prévio ao embarque
creto nº 6.715, de 2008). da mercadoria no exterior e dependerá da anuência
Art. 48.Compete ao Ministério da Defesa e ao Minis- do Comando do Exército.
tério da Justiça: § 1oA autorização é concedida por meio do Certifica-
I - estabelecer as normas de segurança a serem ob- do Internacional de Importação.
servadas pelos prestadores de serviços de transporte § 2oA importação desses produtos somente será au-
aéreo de passageiros, para controlar o embarque de torizada para os órgãos de segurança pública e para
passageiros armados e fiscalizar o seu cumprimento; colecionadores, atiradores e caçadores nas condi-
II - regulamentar as situações excepcionais do in- ções estabelecidas em normas específicas.
teresse da ordem pública, que exijam de policiais Art. 52.Os interessados pela importação de armas
federais, civis e militares, integrantes das Forças Ar- de fogo, munições e acessórios, de uso restrito, ao
madas e agentes do Departamento de Segurança do preencherem a Licença de Importação no Sistema In-
Gabinete de Segurança Institucional da Presidência tegrado de Comércio Exterior - SISCOMEX, deverão
da República, o Porte de Arma de Fogo a bordo de informar as características específicas dos produtos
aeronaves; e importados, ficando o desembaraço aduaneiro sujei-
III - estabelecer, nas ações preventivas com vistas à to à satisfação desse requisito.
segurança da aviação civil, os procedimentos de res- Art. 53.As importações realizadas pelas Forças Arma-
trição e condução de armas por pessoas com a prer- das dependem de autorização prévia do Ministério
rogativa de Porte de Arma de Fogo em áreas restritas da Defesa e serão por este controladas.
Art. 54.A importação de armas de fogo, munições e
aeroportuárias, ressalvada a competência da Polícia
acessórios de uso permitido e demais produtos con-
Federal, prevista no inciso III do §1o do art. 144 da
trolados está sujeita, no que couber, às condições es-
Constituição.
tabelecidas nos arts. 51 e 52 deste Decreto.
Parágrafo único.As áreas restritas aeroportuárias são
Art. 55.A Secretaria da Receita Federal e o Comando
aquelas destinadas à operação de um aeroporto,
do Exército fornecerão à Polícia Federal, as informa-
cujos acessos são controlados, para os fins de segu-
ções relativas às importações de que trata o art. 54
rança e proteção da aviação civil. e que devam constar do cadastro de armas do SI-
Art. 49.A classificação legal, técnica e geral e a defi- NARM.
nição das armas de fogo e demais produtos contro- Art. 56.O Comando do Exército poderá autorizar a
lados, de uso restrito ou permitido são as constan- entrada temporária no país, por prazo definido, de
tes do Regulamento para a Fiscalização de Produtos armas de fogo, munições e acessórios para fins de
Controlados e sua legislação complementar. demonstração, exposição, conserto, mostruário ou
Parágrafo único.Compete ao Comando do Exército testes, mediante requerimento do interessado ou de
promover a alteração do Regulamento mencionado seus representantes legais ou, ainda, das representa-
no caput, com o fim de adequá-lo aos termos deste ções diplomáticas do país de origem.
Decreto. § 1oA importação sob o regime de admissão tempo-
Art. 50.Compete, ainda, ao Comando do Exército: rária deverá ser autorizada por meio do Certificado
I - autorizar e fiscalizar a produção e o comércio de Internacional de Importação.
armas, munições e demais produtos controlados, em § 2oTerminado o evento que motivou a importação,
todo o território nacional; o material deverá retornar ao seu país de origem,
II - estabelecer as dotações em armamento e muni- não podendo ser doado ou vendido no território na-
ção das corporações e órgãos previstos nos incisos II, cional, exceto a doação para os museus das Forças
III, IV, V, VI e VII do art. 6o da Lei no 10.826, de 2003; e Armadas e das instituições policiais.
III - estabelecer normas, ouvido o Ministério da Justi- § 3oA Receita Federal fiscalizará a entrada e saída
ça, em cento e oitenta dias: desses produtos.
a) para que todas as munições estejam acondicio- § 4oO desembaraço alfandegário das armas e mu-
nadas em embalagens com sistema de código de nições trazidas por agentes de segurança de digni-
barras, gravado na caixa, visando possibilitar a iden- tários estrangeiros, em visita ao país, será feito pela
tificação do fabricante e do adquirente; Receita Federal, com posterior comunicação ao Co-
LEGISLAÇÃO ESPECÍFICA

b) para que as munições comercializadas para os ór- mando do Exército.


gãos referidos no art. 6o da Lei no 10.826, de 2003, Art. 57.Fica vedada a importação de armas de fogo,
contenham gravação na base dos estojos que per- seus acessórios e peças, de munições e seus compo-
mita identificar o fabricante, o lote de venda e o ad- nentes, por meio do serviço postal e similares.
quirente; Parágrafo único.Fica autorizada, em caráter excep-
c) para definir os dispositivos de segurança e identi- cional, a importação de peças de armas de fogo, com
ficação previstos no §3o do art. 23 da Lei no 10.826, exceção de armações, canos e ferrolho, por meio do
de 2003; e serviço postal e similares.

70
Art. 58.O Comando do Exército autorizará a expor- Comando do Exército, no prazo máximo de quarenta
tação de armas, munições e demais produtos con- e oito horas, para destruição ou doação aos órgãos
trolados. de segurança pública ou às Forças Armadas.(Reda-
§ 1oA autorização das exportações enquadradas nas ção dada pelo Decreto nº 8.938, de 2016)
diretrizes de exportação de produtos de defesa rege- § 1ºA doação de que trata este artigo restringe-se às
-se por legislação específica, a cargo do Ministério armas de fogo portáteis previstas no art. 3º, caput,
da Defesa. incisos XXXVII, XLIX, LIII e LXI, do Anexo ao Decreto
§ 2oConsidera-se autorizada a exportação quando nº 3.665, de 20 de novembro de 2000 - Regulamento
efetivado o respectivo Registro de Exportação, no para a Fiscalização de Produtos Controlados (R-105).
Sistema de Comércio Exterior - SISCOMEX. (Redação dada pelo Decreto nº 8.938, de 2016)
Art. 59.O exportador de armas de fogo, munições § 2ºOs órgãos de segurança pública ou das Forças
ou demais produtos controlados deverá apresentar Armadas responsáveis pela apreensão manifestarão
como prova da venda ou transferência do produto, interesse pelas armas de fogo de que trata o § 1º,
um dos seguintes documentos: respectivamente, ao Ministério da Justiça e Cidada-
I - Licença de Importação (LI), expedida por autorida- nia ou ao Comando do Exército, no prazo de até dez
de competente do país de destino; ou dias, contado da data de envio das armas ao Coman-
II - Certificado de Usuário Final (End User), expedi- do do Exército, na forma prevista no caput.(Redação
do por autoridade competente do país de destino, dada pelo Decreto nº 8.938, de 2016)
quando for o caso. § 3º A relação das armas a serem doadas e a indica-
Art. 60.As exportações de armas de fogo, munições ção das instituições beneficiárias serão elaboradas,
ou demais produtos controlados considerados de desde que:(Redação dada pelo Decreto nº 8.938, de
valor histórico somente serão autorizadas pelo Co- 2016)
mando do Exército após consulta aos órgãos com- I - verificada a necessidade de destinação do
petentes. armamento;(Redação dada pelo Decreto nº 8.938, de
Parágrafo único. O Comando do Exército estabelece- 2016)
rá, em normas específicas, os critérios para definição II - obedecidos o padrão e a dotação de cada órgão;
do termo “valor histórico”. e(Redação dada pelo Decreto nº 8.938, de 2016)
Art. 61.O Comando do Exército cadastrará no SIGMA III - atendidos os critérios de priorização estabeleci-
os dados relativos às exportações de armas, muni- dos pelo Ministério da Justiça e Cidadania, nos ter-
ções e demais produtos controlados, mantendo-os mos do § 1º do art. 25 da Lei nº 10.826, de 2003.
devidamente atualizados. (Redação dada pelo Decreto nº 8.938, de 2016)
Art. 62.Fica vedada a exportação de armas de fogo, § 4ºOs critérios de que trata o inciso III do § 3º de-
de seus acessórios e peças, de munição e seus com- verão considerar a priorização de atendimento ao
ponentes, por meio do serviço postal e similares. órgão que efetivou a apreensão. (Redação dada pelo
Art. 63.O desembaraço alfandegário de armas e mu- Decreto nº 8.938, de 2016)
nições, peças e demais produtos controlados será § 5ºA análise da presença dos requisitos estabeleci-
autorizado pelo Comando do Exército. dos no § 3º será realizada no prazo de atécinco dias,
Parágrafo único. O desembaraço alfandegário de contado da data de manifestação de interesse de
que trata este artigo abrange: que trata o § 2º, pela Secretaria Nacional de Segu-
I - operações de importação e exportação, sob qual- rança Pública do Ministério da Justiça e Cidadania,
quer regime; caso a manifestação tenha sido apresentada pelos
II - internação de mercadoria em entrepostos adua- órgãos de segurança pública, ou pelo Comando do
neiros; Exército, caso a manifestação tenha sido apresenta-
III - nacionalização de mercadoria entrepostadas; da pelas Forças Armadas. (Incluído pelo Decreto nº
IV - ingresso e saída de armamento e munição de 8.938, de 2016)
atletas brasileiros e estrangeiros inscritos em compe- § 6ºCumpridos os requisitos de que trata o § 3º, o
tições nacionais ou internacionais; Comando do Exército encaminhará, no prazo de até
V - ingresso e saída de armamento e munição; vinte dias, a relação das armas ao juiz competente,
VI - ingresso e saída de armamento e munição de que determinará o seu perdimento em favor da ins-
órgãos de segurança estrangeiros, para participação tituição beneficiária. (Incluído pelo Decreto nº 8.938,
em operações, exercícios e instruções de natureza de 2016)
oficial; e § 7ºNa hipótese de não haver manifestação expressa
VII - as armas de fogo, munições, suas partes e peças, do órgão que realizou a apreensão das armas de que
trazidos como bagagem acompanhada ou desacom- trata o § 1º, os demais órgãos de segurança pública
panhada.
LEGISLAÇÃO ESPECÍFICA

ou das Forças Armadas poderão manifestar interesse


Art. 64.O desembaraço alfandegário de armas de pelas armas, no prazo de trinta dias, contado da data
fogo e munição somente será autorizado após o de recebimento do relatório a que se refere o art. 25,
cumprimento de normas específicas sobre marcação, § 1º, da Lei nº 10.826, de 2003, cabendo-lhes enca-
a cargo do Comando do Exército. minhar pedido de doação ao Comando do Exército.
Art. 65.As armas de fogo apreendidas, observados os (Incluído pelo Decreto nº 8.938, de 2016)
procedimentos relativos à elaboração do laudo pe- § 8ºO Comando do Exército apreciará o pedido de
ricial e quando não mais interessarem à persecução doação de que trata o § 7º, observados os requisitos
penal, serão encaminhadas pelo juiz competente ao estabelecidos no § 3º, e encaminhará, no prazo de

71
sessenta dias, contado da data de divulgação do re- ciar sua transferência no prazo máximo de sessenta
latório a que se refere o art. 25, § 1º, da Lei nº 10.826, dias, aplicando-se, ao interessado na aquisição, as
de 2003, a relação das armas a serem doadas, para disposições do art. 4o da Lei no 10.826, de 2003. (In-
que o juiz competente determine o seu perdimento, cluído pelo Decreto nº 6.715, de 2008).
nos termos do § 6º. (Incluído pelo Decreto nº 8.938, § 2oA cassação da autorização de posse ou de porte
de 2016) de arma de fogo será determinada a partir do indi-
§ 9ºAs armas de fogo de valor histórico ou obsoletas, ciamento do investigado no inquérito policial ou do
objetos de doação nos termos deste artigo, poderão recebimento da denúncia ou queixa pelo juiz. (Inclu-
ser destinadas pelo juiz competente a museus das ído pelo Decreto nº 6.715, de 2008).
Forças Armadas ou de instituições policiais, indica- § 3oAplica-se o disposto neste artigo a todas as ar-
dos pelo Comando do Exército.(Incluído pelo Decre- mas de fogo de propriedade do indiciado ou acusa-
to nº 8.938, de 2016) do. (Incluído pelo Decreto nº 6.715, de 2008).
§ 10.As armas de fogo de uso permitido apreendidas Art. 67-B.No caso do não-atendimento dos requisi-
poderão ser devolvidas pela autoridade competente tos previstos no art. 12, para a renovação do Certi-
aos seus legítimos proprietários se cumpridos os re- ficado de Registro da arma de fogo, o proprietário
quisitos estabelecidos no art. 4º da Lei nº10.826, de deverá entregar a arma à Polícia Federal, mediante
2003. (Incluído pelo Decreto nº 8.938, de 2016) indenização na forma do art. 68, ou providenciar sua
§ 11.A decisão sobre o destino final das armas de transferência para terceiro, no prazo máximo de ses-
fogo não doadas nos termos deste Decreto caberá senta dias, aplicando-se, ao interessado na aquisição,
ao Comando do Exército, que deverá concluir pela as disposições do art. 4o da Lei no 10.826, de 2003.
sua destruição ou pela doação às Forças Armadas. (Incluído pelo Decreto nº 6.715, de 2008).
(Incluído pelo Decreto nº 8.938, de 2016) Parágrafo único.A inobservância do disposto no ca-
§ 12.Ato conjunto do Ministro de Estado da Defe- put implicará a apreensão da arma de fogo pela Po-
sa e do Ministro de Estado da Justiça e Cidadania lícia Federal ou órgão público por esta credenciado,
disciplinará o procedimento de doação de munições aplicando-se ao proprietário as sanções penais cabí-
e acessórios apreendidos. (Incluído pelo Decreto nº veis. (Incluído pelo Decreto nº 6.715, de 2008).
8.938, de 2016)
Art. 67-C.Quaisquer cadastros constantes do SIGMA
Art. 66.A solicitação de informações sobre a origem
ou do SINARM, na hipótese em que estiverem rela-
de armas de fogo, munições e explosivos deverá ser
cionados com integrantes da Agência Brasileira de
encaminhada diretamente ao órgão controlador da
Inteligência, deverão possuir exclusivamente o nú-
Polícia Federal ou do Comando do Exército.
mero de matrícula funcional como dado de qualifi-
Art. 67.No caso de falecimento ou interdição do pro-
cação pessoal, incluídos os relativos à aquisição e à
prietário de arma de fogo, o administrador da heran-
venda de armamento e à comunicação de extravio,
ça ou curador, conforme o caso, deverá providenciar
furto ou roubo de arma de fogo ou seus documen-
a transferência da propriedade da arma mediante
alvará judicial ou autorização firmada por todos os tos. (Incluído pelo Decreto nº 9.685, de 2019)
herdeiros, desde que maiores e capazes, aplicando- ,
-se ao herdeiro ou interessado na aquisição as dis- Seção II
posições do art. 12. (Redação dada pelo Decreto nº Das Disposições Finais e Transitórias
6.715, de 2008).
§ 1oO administrador da herança ou o curador comu- Art. 68.O valor da indenização de que tratam os arts.
nicará à Polícia Federal ou ao Comando do Exército, 31 e 32 da Lei no 10.826, de 2003, bem como o pro-
conforme o caso, a morte ou interdição do proprie- cedimento para pagamento, será fixado pelo Minis-
tário da arma de fogo. (Redação dada pelo Decreto tério da Justiça.
nº 6.715, de 2008). Parágrafo único.Os recursos financeiros necessários
§ 2oNos casos previstos no caput deste artigo, a arma para o cumprimento do disposto nos arts. 31 e 32
deverá permanecer sob a guarda e responsabilidade da Lei nº 10.826, de 2003, serão custeados por dota-
do administrador da herança ou curador, depositada ção específica constante do orçamento do Ministério
em local seguro, até a expedição do Certificado de da Justiça.(Redação dada pelo Decreto nº 7.473, de
Registro e entrega ao novo proprietário. 2011)
§ 3oA inobservância do disposto no § 2o implicará Art. 69.Presumir-se-á a boa-fé dos possuidores e
a apreensão da arma pela autoridade competente, proprietários de armas de fogo que espontaneamen-
aplicando-se ao administrador da herança ou ao te entregá-las na Polícia Federal ou nos postos de
curador as sanções penais cabíveis. (Redação dada recolhimento credenciados, nos termos do art. 32 da
LEGISLAÇÃO ESPECÍFICA

pelo Decreto nº 6.715, de 2008). Lei no 10.826, de 2003. (Redação dada pelo Decreto
Art. 67-A.Serão cassadas as autorizações de posse nº 7.473, de 2011)
e de porte de arma de fogo do titular a quem seja Art. 70.A entrega da arma de fogo, acessório ou mu-
imputada a prática de crime doloso. (Incluído pelo nição, de que tratam os arts. 31 e 32 da Lei nº 10.826,
Decreto nº 6.715, de 2008). de 2003, deverá ser feita na Polícia Federal ou nos
§ 1oNos casos previstos no caput, o proprietário de- órgãos e entidades credenciados pelo Ministério
verá entregar a arma de fogo à Polícia Federal, me- da Justiça.(Redação dada pelo Decreto nº 7.473, de
diante indenização na forma do art. 68, ou providen- 2011)

72
§ 1oPara o transporte da arma de fogo até o local de § 2oNo ato do preenchimento do formulário pela
entrega, será exigida guia de trânsito, expedida pela rede mundial de computadores - Internet, o reque-
Polícia Federal, ou órgão por ela credenciado, con- rente deverá escolher a unidade da Polícia Federal, ou
tendo as especificações mínimas estabelecidas pelo órgão por ela credenciado, na qual entregará pesso-
Ministério da Justiça. (Redação dada pelo Decreto nº almente a documentação exigida para o registro ou
7.473, de 2011) renovação. (Incluído pelo Decreto nº 6.715, de 2008).
§ 2oA guia de trânsito poderá ser expedida pela rede § 3oCaso o requerente deixe de apresentar a docu-
mundial de computadores - Internet, na forma disci- mentação exigida para o registro ou renovação na uni-
plinada pelo Departamento de Polícia Federal. (Inclu- dade da Polícia Federal, ou órgão por ela credenciado,
ído pelo Decreto nº 6.715, de 2008). escolhida dentro do prazo de noventa dias, o certifi-
§ 3oA guia de trânsito não autoriza o porte da arma, cado de registro provisório, que será expedido pela
mas apenas o seu transporte, desmuniciada e acon- rede mundial de computadores - Internet uma única
dicionada de maneira que não possa ser feito o seu vez, perderá a validade, tornando irregular a posse da
pronto uso e, somente, no percurso nela autorizado. arma. (Incluído pelo Decreto nº 6.715, de 2008).
(Incluído pelo Decreto nº 6.715, de 2008). § 4oNo caso da perda de validade do certificado de
§ 4oO transporte da arma de fogo sem a guia de registro provisório, o interessado deverá se dirigir
trânsito ou o transporte com a guia, mas sem a ob- imediatamente à unidade da Polícia Federal, ou órgão
servância do que nela estiver estipulado, poderá su- por ela credenciado, para a regularização de sua situa-
jeitar o infrator às sanções penais cabíveis.(Incluído ção. (Incluído pelo Decreto nº 6.715, de 2008).
pelo Decreto nº 6.715, de 2008). § 5oAplica-se o disposto no art. 70-B à renovação dos
Art. 70-A.Para o registro da arma de fogo de uso per- registros de arma de fogo cujo certificado tenha sido
mitido ainda não registrada de que trata o art. 30 expedido pela Polícia Federal, inclusive aqueles com
da Lei no 10.826, de 2003, deverão ser apresenta- vencimento até o prazo previsto no § 3odo art. 5o da
dos pelo requerente os documentos previstos no art. Lei no 10.826, de 2003, ficando o proprietário isento
70-C e original e cópia, ou cópia autenticada, da nota do pagamento de taxa nas condições e prazos da Ta-
fiscal de compra ou de comprovação da origem lícita
bela constante do Anexo à referida Lei. (Incluído pelo
da posse, pelos meios de prova admitidos em direi-
Decreto nº 6.715, de 2008).
to, ou declaração firmada na qual constem as carac-
§ 6oNos requerimentos de registro ou de renovação
terísticas da arma e a sua condição de proprietário.
de Certificado de Registro de Arma de Fogo em que
(Incluído pelo Decreto nº 6.715, de 2008).
se constate a existência de cadastro anterior em nome
Art. 70-B.Para a renovação do Certificado de Registro
de terceiro, será feita no SINARM a transferência da
de Arma de Fogo de que trata o § 3o do art. 5o da Lei
no 10.826, de 2003, deverão ser apresentados pelo arma para o novo proprietário. (Incluído pelo Decreto
requerente os documentos previstos no art. 70-C e nº 6.715, de 2008).
cópia do referido Certificado ou, se for o caso, do § 7oNos requerimentos de registro ou de renovação
boletim de ocorrência comprovando o seu extravio. de Certificado de Registro de Arma de Fogo em que
(Incluído pelo Decreto nº 6.715, de 2008). se constate a existência de cadastro anterior em nome
Art. 70-C.Para a renovação do Certificado de Registro de terceiro e a ocorrência de furto, roubo, apreensão
de Arma de Fogo ou para o registro da arma de fogo ou extravio, será feita no SINARM a transferência da
de que tratam, respectivamente, o § 3o do art. 5o e o arma para o novo proprietário e a respectiva arma de
art. 30 da Lei no 10.826, de 2003, o requerente deve- fogo deverá ser entregue à Polícia Federal para poste-
rá: (Incluído pelo Decreto nº 6.715, de 2008). rior encaminhamento à autoridade policial ou judicial
I - ter, no mínimo, vinte e cinco anos de idade; (Inclu- competente. (Incluído pelo Decreto nº 6.715, de 2008).
ído pelo Decreto nº 6.715, de 2008). § 8oNo caso do requerimento de renovação do Certi-
II - apresentar originais e cópias, ou cópias autenti- ficado de Registro de que trata o § 6o, além dos docu-
cadas, do documento de identificação pessoal e do mentos previstos no art. 70-B, deverá ser comprovada
comprovante de residência fixa; (Incluído pelo Decre- a origem lícita da posse, pelos meios de prova admi-
to nº 6.715, de 2008). tidos em direito, ou, ainda, apresentada declaração
III -apresentar o formulário SINARM devidamente firmada na qual constem as características da arma e
preenchido; e (Incluído pelo Decreto nº 6.715, de a sua condição de proprietário. (Incluído pelo Decreto
2008). nº 6.715, de 2008).
IV - apresentar o certificado de registro provisório § 9oNos casos previstos neste artigo, além dos da-
e comprovar os dados pessoais informados, caso o dos de identificação do proprietário, o Certificado de
procedimento tenha sido iniciado pela rede mundial Registro provisório e o definitivo deverão conter, no
de computadores - Internet. (Incluído pelo Decreto mínimo, o número de série da arma de fogo, a marca,
LEGISLAÇÃO ESPECÍFICA

nº 6.715, de 2008). a espécie e o calibre. (Incluído pelo Decreto nº 6.715,


§ 1oO procedimento de registro da arma de fogo, de 2008).
ou sua renovação, poderá ser iniciado por meio do Art. 70-D.Não se aplicam as disposições do § 6o do art.
preenchimento do formulário SINARM na rede mun- 70-C às armas de fogo cujos Certificados de Registros
dial de computadores - Internet, cujo comprovante tenham sido expedidos pela Polícia Federal a partir da
de preenchimento impresso valerá como certificado vigência deste Decreto e cujas transferências de pro-
de registro provisório, pelo prazo de noventa dias. priedade dependam de prévia autorização. (Incluído
(Incluído pelo Decreto nº 6.715, de 2008). pelo Decreto nº 6.715, de 2008).

73
Art. 70-E.As armas de fogo entregues na campanha I - a documentação comprobatória do preenchi-
do desarmamento não serão submetidas a perícia, mento dos requisitos constantes do art. 4o da Lei no
salvo se estiverem com o número de série ilegível ou 10.826, de 2003, quanto aos empregados que porta-
houver dúvidas quanto à sua caracterização como rão arma de fogo; ou
arma de fogo, podendo, nesse último caso, serem II - semestralmente, ao SINARM, a listagem atualiza-
submetidas a simples exame de constatação. (Incluí- da de seus empregados.
do pelo Decreto nº 6.715, de 2008). Art. 73. (Revogado pelo Decreto nº 6.146, de 2007
Parágrafo único.As armas de fogo de que trata o Art. 74.Os recursos arrecadados em razão das taxas
caput serão, obrigatoriamente, destruídas. (Incluído e das sanções pecuniárias de caráter administrativo
pelo Decreto nº 6.715, de 2008). previstas neste Decreto serão aplicados na forma
Art. 70-F.Não poderão ser registradas ou terem seu prevista no § 1o do art. 11 da Lei no 10.826, de 2003.
registro renovado as armas de fogo adulteradas ou Parágrafo único.As receitas destinadas ao SINARM
com o número de série suprimido. (Incluído pelo serão recolhidas ao Banco do Brasil S.A., na conta
Decreto nº 6.715, de 2008). “Fundo para Aparelhamento e Operacionalização das
Atividades-Fim da Polícia Federal”, e serão alocadas
Parágrafo único.Nos prazos previstos nos arts. 5o, §
para o reaparelhamento, manutenção e custeio das
3o, e 30 da Lei no 10.826, de 2003, as armas de que
atividades de controle e fiscalização da circulação de
trata o caput serão recolhidas, mediante indeniza-
armas de fogo e de repressão a seu tráfico ilícito,a
ção, e encaminhadas para destruição.(Incluído pelo
cargo da Polícia Federal. (Redação dada pelo Decreto
Decreto nº 6.715, de 2008). nº 6.715, de 2008).
Art. 70-G.Compete ao Ministério da Justiça estabe- Art. 75.Serão concluídos em sessenta dias, a partir da
lecer os procedimentos necessários à execução da publicação deste Decreto, os processos de doação,
campanha do desarmamento e ao Departamento em andamento no Comando do Exército, das armas
de Polícia Federal a regularização de armas de fogo. de fogo apreendidas e recolhidas na vigência da Lei
(Redação dada pelo Decreto nº 7.473, de 2011) no 9.437, de 20 de fevereiro de 1997.
Art. 70-H.As disposições sobre entrega de armas de Art. 76.Este Decreto entra em vigor na data de sua
que tratam os arts. 31 e 32 da Lei no 10.826, de 2003, publicação.
não se aplicam às empresas de segurança privada Art. 77.Ficam revogados os Decretos nos 2.222, de 8
e transporte de valores. (Incluído pelo Decreto nº de maio de 1997, 2.532, de 30 de março de 1998, e
6.715, de 2008). 3.305, de 23 de dezembro de 1999.
Art. 71.Será aplicada pelo órgão competente pela Brasília, 1º de julho de 2004; 183º da Independência
fiscalização multa no valor de: e 116º da República.
I - R$ 100.000,00 (cem mil reais):
a) à empresa de transporte aéreo, rodoviário, fer-
roviário, marítimo, fluvial ou lacustre que permita o LEI Nº 11.340, DE 7 DE AGOSTO DE 2006
transporte de arma de fogo, munição ou acessórios, (LEI MARIA DA PENHA);
sem a devida autorização, ou com inobservância das
normas de segurança; e
b) à empresa de produção ou comércio de arma- Na abertura deste material, apresentamos um resu-
mentos que realize publicidade estimulando a venda mo das considerações da doutrinadora Maria Berenice
e o uso indiscriminado de armas de fogo, acessórios Dias60 sobre a Lei Maria da Penha. Se trabalho contribui
e munição, exceto nas publicações especializadas; por levantar a discussão sobre a efetividade do referido
diploma, demonstrando-a por meio de uma explicação
II - R$ 200.000,00 (duzentos mil reais), sem prejuízo
detalhada sobre o procedimento que deve ser seguido
das sanções penais cabíveis:
nas ações penais que envolvem a violência doméstica.
a) à empresa de transporte aéreo, rodoviário, ferro-
Neste ponto é esclarecedor, a partir do momento no qual
viário, marítimo, fluvial ou lacustre que deliberada-
explica de maneira suficiente e breve os papeis da auto-
mente, por qualquer meio, faça, promova ou facili- ridade policial, do Ministério Público, do magistrado, dos
te o transporte de arma ou munição sem a devida advogados, da vítima e do agressor em se tratando de
autorização ou com inobservância das normas de crimes no âmbito da relação familiar.
segurança; e Contribui ao destacar a importância da figura do
b) à empresa de produção ou comércio de arma- tratamento psicológico e hospitalar do agressor, o que
mentos, na reincidência da hipótese mencionada no pode contribuir para o aumento de denúncias e para a
inciso I, alínea “b”; e diminuição da violência doméstica. De fato, muitas vezes
III - R$ 300.000,00 (trezentos mil reais), sem prejuízo a vítima deixa de fazer a denúncia porque o agressor é o
LEGISLAÇÃO ESPECÍFICA

das sanções penais cabíveis, na hipótese de reinci- responsável pelo sustento do lar.
dência da conduta prevista na alínea “a”, do inciso I, Por outro lado, é de se considerar que o artigo traz
e nas alíneas “a” e “b”, do inciso II. apenas a posição da autora no tocante à espécie de ação
Art. 72.A empresa de segurança e de transporte de penal aplicável no caso de lesões corporais leves ou cul-
valores ficará sujeita às penalidades de que trata 60 DIAS, Maria Berenice. A efetividade da lei
o art. 23 da Lei no 7.102, de 20 de junho de 1983, Maria da Penha. Revista Brasileira de Ciências Cri-
quando deixar de apresentar, nos termos do art. 7o, minais, São Paulo, n. 64, ano 14, p. 297-312, jan./fev.
§§ 2o e 3o, da Lei no 10.826, de 2003:
2007.

74
posas cometidas no âmbito da relação familiar. Para Dias, A Lei Maria da Penha criou ainda nova hipótese de
a ação penal em tais casos será sempre incondicionada, prisão preventiva, visando garantir a execução das medi-
diante do afastamento da Lei n. 9.099/95. Referido en- das de urgência (artigo 42). Com isso, a prisão preventiva
tendimento tem sido abarcado nas principais cortes bra- deixou de ser restrita aos crimes apenados com reclusão.
sileiras, inclusive resultando em súmula do STJ: Ela pode ser decretada de ofício pelo juiz, a requerimento
Súmula 542, STJ. A ação penal relativa ao crime do Ministério Público ou mediante representação da au-
de lesão corporal resultante de violência doméstica toridade policial (artigo 20).
contra a mulher é pública incondicionada.
Em que pesem as controvérsias, a Lei Maria da Pe- 3) Sua constitucionalidade
nha foi fundamental para uma mudança no modo pelo Há quem sustente a inconstitucionalidade da lei, sob
qual a sociedade encarava a violência doméstica contra a dois argumentos principais:
mulher, que muitas vezes era vista com indiferença. Ante- a) afronta ao princípio da igualdade porque o homem
riormente, a denúncia da violência poucas vezes gerava a não pode ser sujeito passivo;
punição efetiva do agressor, o que levava aos constantes b) definição de competências, transbordando os limi-
casos de reincidência. tes da lei, porque tal definição deve ser feita pelo Poder
A Lei Maria da Penha trouxe instrumentos importan- Judiciário.
tes para uma postura proativa do Estado perante o pro- O primeiro argumento não se justifica porque, sob
blema da violência doméstica contra a mulher, dando-lhe um aspecto histórico, a mulher sempre foi colocada em
instrumentos de atuação mais eficientes para a realização posição menos favorável que o homem, o que levou ao
da justiça em seu significado mais profundo, não apenas contexto de inferioridade e submissão que leva à vio-
como a aplicação fria e cega de regras, mas como instru- lência doméstica, sendo, portanto, necessárias ações
mentos de mudança social em prol da emancipação do afirmativas para promover a efetividade do princípio da
ser humano. igualdade.
Já o segundo deve ser afastado porque não é a pri-
1) Uma justificativa meira vez que o legislador cria competências específicas
A Lei Maria da Penha foi recebida pelos juristas com (no caso, estabeleceu a criação dos JVDFM e a compe-
desconfiança, constituindo objeto de várias críticas, que tência cível e criminal das Varas Criminais até que esta
em geral buscam desqualificá-la, suscitando dúvidas,
ocorra) e, como houve o afastamento da aplicação da Lei
apontando erros, identificando imprecisões e até mesmo
n. 9.099/95, a definição de competência deixou de per-
proclamando inconstitucionalidades, tudo isto servindo
tencer exclusivamente à esfera do Judiciário.
de motivo para impedir sua efetividade.
O STF julgou nas ADI 4424 e ADC 19:
No entanto, todas estas críticas apenas demonstram
- o artigo 1º da Lei é constitucional, logo ela não fere
uma injustificável resistência às mudanças na postura de
os princípios constitucionais da igualdade e proporcio-
enfrentamento da violência doméstica, que sempre foi
nalidade (não é desproporcional ou ilegítimo o uso do
alvo de absoluto descaso por parte do ordenamento ju-
rídico, principalmente a partir do momento no qual a le- sexo como critério de diferenciação, visto que a mulher
são corporal leve passou a ser considerada como crime é eminentemente vulnerável no tocante a constrangi-
de pequeno potencial ofensivo (podendo os conflitos ser mentos físicos, morais e psicológicos sofridos em âmbito
solucionados de forma consensual). Além disso, tornou- privado);
-se popular a punição com o pagamento de cestas bási- - o artigo 33 da Lei da mesma forma é constitucional,
cas, o que banalizou ainda mais a violência doméstica e portanto, enquanto não forem organizados os Juizados
a integridade física da vítima. A Lei Maria da Penha veio de Violência Doméstica e Familiar, compete às varas cri-
para mudar esta perspectiva. minais o julgamento destas causas;
- também é constitucional o artigo 44 da Lei, assim,
2) Os avanços aos crimes praticados com violência doméstica e familiar
A Lei Maria da Penha trouxe benefícios significativos e contra a mulher, não se aplica a Lei 9.099/95 (Precedente
de efeito imediato. O maior avanço foi a criação dos Jui- STF, HC 106.212/MS, Plenário, 24/03/2011);
zados de Violência Doméstica e Familiar contra a Mulher - os artigos 12, I; 16 e 41 da Lei Maria da Penha foram
(JVDFM), com competência cível e criminal (artigo 14). interpretados conforme a Constituição para assentar a
O ideal seria que os JVDFM fossem instalados em natureza incondicionada da ação penal em caso de crime
todas as comarcas imediatamente, com especialistas de lesão corporal, praticado mediante violência domésti-
( juízes, promotores e defensores) no atendimento das ca e familiar contra a mulher.
demandas, equipes de atendimento multidisciplinar in-
tegrada por profissionais das áreas psicossocial, jurídica 4) Competência
e de saúde (artigo 29) e serviço de assistência judiciária A violência doméstica está fora do âmbito dos Juiza-
LEGISLAÇÃO ESPECÍFICA

(artigo 34). No entanto, até que isto ocorra foi atribuída dos Especiais Criminais, e estes não poderão mais apre-
às Varas Criminais competência cível e criminal (artigos ciar tal matéria. A instalação dos JDFM é imprescindível e
11 e 33), o que se justifica diante do afastamento da apli- deve ser feita logo que possível.
cação da Lei 9.099/95 (Juizados Especiais Cíveis e Crimi- Destaca-se que cada denúncia de violência doméstica
nais) (artigo 41). pode gerar duas demandas, porque tanto o expediente
Outro avanço se encontra no artigo 27, que garante à para a adoção de medidas protetivas de urgência quanto
vítima o acesso aos serviços da Defensoria Pública e à as- o inquérito policial são enviados pela autoridade policial
sistência judiciária tanto na fase policial como na judicial. ao juiz e ao Ministério Público.

75
5) Fase policial mentos. Realizado o acordo, prossegue o inquérito poli-
Anteriormente, o único meio de afastar o agressor do cial, pois o acordo não significa a renúncia à representa-
lar era a ação cautelar de separação de corpos. ção. Na audiência estarão o Ministério Público (artigo 25)
Com a Lei Maria da Penha, passaram a ser necessárias e as partes com seus advogados (artigo 27).
diversas providências quando comunicada a violência Após, esgota-se a atividade do JVDFM ou da Vara Cri-
doméstica: registra-se a ocorrência, com oitiva da vítima minal no tocante às medidas de urgência. Controvérsias
(artigo 12, I), oportunidade na qual esta é informada dos quanto ao adimplemento do acordo no toante a matéria
direitos e serviços disponíveis existentes (artigo 11, V), in- cível ou de Direito de Família devem ser discutidas nas va-
clusive medidas protetivas disponíveis (artigo. 12, §1°); a ras Cíveis ou de Família.
vítima é encaminhada ao hospital com transporte segu- O inquérito policial continua independente do defe-
ro e acompanhamento para retirar seus pertences do lar rimento de medida protetiva ou de acordo realizado em
(artigo 11); instaura-se o inquérito policial (artigo 12, VII); juízo, devendo ser remetido à justiça quando encerrado e
a polícia toma por termo o pedido de medidas urgen- distribuído ao mesmo juízo que apreciou a medida caute-
tes (artigo 12, §1°), formalizando-se a representação na lar, que será a este apensada. Em seguida, os autos serão
mesma ocasião (artigo 12, I); a autoridade policial pode remetidos ao Ministério Público para oferecimento de de-
solicitar a prisão do agressor (artigo 20). núncia.
Para a busca de medidas protetivas faz-se necessária
somente a ouvida da ofendida, anexadas apenas as pro- 7) Ministério Público
vas que estiverem disponíveis e em sua posse (artigo 12, A participação do Ministério Público é indispensável e
§2°). Logo, não é preciso tomar depoimento do agressor ele tem legitimidade para agir como parte, intervindo nas
ou de testemunhas e nem realizar exame de corpo de ações cíveis e criminais (artigo 25). Pode, ainda, exercer a
delito, providências que devem instruir exclusivamente o defesa dos interesses e direitos transindividuais (artigo 37).
inquérito policial. Devem ser comunicadas ao promotor as medidas adota-
No inquérito policial é determinada a realização do das (artigo 22, §1°), podendo ele requerer outras providên-
exame de corpo de delito e outros que se fizerem neces- cias ou a substituição das medidas (artigo 19), bem como a
sários (artigo 12, IV) e são colhidos os depoimentos do prisão do agressor (artigo 20).
Quando a vítima manifestar o interesse em desistir da
agressor e das testemunhas (artigo 12, VI).
ação, o Ministério Público deverá estar presente (artigo 16).
6) Procedimento judicial
8) A polêmica sobre o delito de lesão corporal
O pedido de medidas de urgência é encaminhado à
A Lei n. 9.099/95 estabeleceu que a lesão corporal leve
justiça em até 48 horas, quando é autuado e distribuído
a lesão culposa são delitos de menor potencial ofensivo
às Varas Criminais, enquanto não existir juízo especializa-
(artigo 88), restando condicionadas à representação. Entre-
do na comarca.
tanto, não houve modificação no Código Penal.
O juiz pode deferir medidas cautelares em sede de Já a Lei n. 11.340 (Lei Maria da Penha), em seu artigo 41,
liminar (tenham ela sido requeridas pela ofendida ou afastou a aplicação da Lei n. 9.099/95 aos crimes praticados
pelo Ministério Público ou não, conforme os artigos 12, com violência doméstica e familiar contra a mulher, inde-
III; 18; 19 e 19, §3°), designar audiência de justificação pendente da pena prevista.
ou indeferi-las de plano. Assim, o juiz pode determinar Desta forma, não é possível falar em ação penal pú-
de ofício as medidas que entender de direito (artigos 20, blica condicionada à representação nas lesões corporais
22, §4°, 23 e 24), por exemplo, afastamento do agressor leves cometidas no âmbito das relações familiares, diante
do lar, impedimento de que este se aproxime da casa, do afastamento por lei posterior da lei que prevê nestes
vedação de comunicação com a família, suspensão de termos.
visitas, encaminhamento da mulher e dos filhos a lugar Além disso, o aumento da pena do delito de lesão
seguro, fixação de alimentos provisórios ou provisionais, corporal para 3 anos (artigo 44) afasta a possibilidade de
restituição de bens da ofendida, suspensão de procura- aplicação de medidas de despenalização e suspensão con-
ção por esta outorgada ao agressor, proibição temporá- dicional do processo, somente cabíveis em delitos que te-
ria da venda de bens comuns etc. nham por pena mínima cominada igual ou inferior a 1 ano.
Para garantir a efetividade destas medidas, o juiz O entendimento foi consolidado na súmula 542 do Su-
pode, a qualquer momento, utilizar força policial (arti- perior Tribunal da Justiça: “A ação penal relativa ao crime
go 22, §3°) ou decretar a prisão preventiva do agressor de lesão corporal resultante de violência doméstica contra
(artigo 20). a mulher é pública incondicionada”.
O magistrado pode, ainda, determinar a inclusão
da vítima em programas assistenciais (artigo 9°, §1°). À 9) Necessidade de representação e possibilidade de
ofendida é assegurado o acesso prioritário à remoção, renúncia
LEGISLAÇÃO ESPECÍFICA

se ela for funcionária pública, e, se trabalhar na iniciativa Pela Lei Maria da Penha, nos crimes de ação penal pú-
privada, a manutenção do vínculo empregatício por até blica condicionada, a vítima pode renunciar à representa-
seis meses de for necessário o afastamento do local de ção (artigo 16). Esta representação é tomada por termo
trabalho (artigo 9°, §2°). pela autoridade policial quando ela registra a ocorrência
Deferida ou não a medida protetiva, é recomendável (artigo 12, I). No entanto, só há esta possibilidade nos de-
a designação de audiência para se ouvir o agressor e para litos que o Código Penal classifica como de ação pública
tentar resolver consensualmente os temas como guarda condicionada à representação, por exemplo, nos crimes
dos filhos, regulamentação de visitas, definição dos ali- contra a liberdade sexual e no de ameaça.

76
A vontade de desistir deve ser comunicada pela ofen- lência contra a Mulher e de outros tratados interna-
dida ao cartório da Vara na qual foi distribuída a medida cionais ratificados pela República Federativa do Brasil;
protetiva de urgência, comunicando-se ao juiz que rea- dispõe sobre a criação dos Juizados de Violência Do-
lizará audiência, o mais rápido possível, na qual deverá méstica e Familiar contra a Mulher; e estabelece medi-
estar presente o Ministério Público. Após a renúncia, de- das de assistência e proteção às mulheres em situação
verá haver comunicação à autoridade policial para que de violência doméstica e familiar.
arquive o inquérito policial. Se o inquérito já tiver sido
remetido ao juízo, a extinção somente pode ocorrer até Art. 2o Toda mulher, independentemente de classe,
o recebimento da denúncia. raça, etnia, orientação sexual, renda, cultura, nível
educacional, idade e religião, goza dos direitos fun-
10) Dos delitos e das penas damentais inerentes à pessoa humana, sendo-lhe
A Lei Maria da Penha não fez alterações relevantes no asseguradas as oportunidades e facilidades para vi-
Código Penal, limitando-se a aumentar a pena máxima e ver sem violência, preservar sua saúde física e mental e
diminuir a pena mínima do delito de lesão corporal: de seu aperfeiçoamento moral, intelectual e social.
seis meses a um ano para de três meses a três anos. Além
disso, estabeleceu uma majorante (artigo 129, §9°, CP) e Art. 3o Serão asseguradas às mulheres as condições
uma agravante (artigo 61, II, CP). para o exercício efetivo dos direitos à vida, à segu-
Não deve ser considerado como de ação penal pú- rança, à saúde, à alimentação, à educação, à cultura,
blica condicionada à representação os crimes de lesões à moradia, ao acesso à justiça, ao esporte, ao lazer, ao
corporais leves ou culposas, diante do afastamento da trabalho, à cidadania, à liberdade, à dignidade, ao res-
Lei n. 9.099/95. Assim, são crimes de ação penal pública peito e à convivência familiar e comunitária.
incondicionada, motivo pelo qual não é possível a renún- § 1o O poder público desenvolverá políticas que visem
cia ou a desistência. garantir os direitos humanos das mulheres no âm-
Não incidindo a Lei n. 9.099/95 também não há possi- bito das relações domésticas e familiares no sentido de
bilidade de suspensão condicional do processo, compo- resguardá-las de toda forma de negligência, discri-
sição de danos ou aplicação imediata de pena não-pri- minação, exploração, violência, crueldade e opressão.
vativa de liberdade. Neste sentido, reforça o artigo 17 da § 2o Cabe à família, à sociedade e ao poder público
Lei Maria da Penha. criar as condições necessárias para o efetivo exer-
Igualmente, por conta do afastamento da Lei dos Jui- cício dos direitos enunciados no caput.
Art. 4o Na interpretação desta Lei, serão considerados
zados Especiais, não pode o Ministério Público propor
os fins sociais a que ela se destina e, especialmente,
transação penal ou aplicar imediatamente a pena restriti-
as condições peculiares das mulheres em situação de
va de direito ou multa.
violência doméstica e familiar.
Entretanto, é possível a suspensão condicional da
pena (artigo 77, CP) e a sua substituição por medida res-
TÍTULO II
tritiva de direitos (artigo 43, CP), isto porque tais bene-
DA VIOLÊNCIA DOMÉSTICA E FAMILIAR CONTRA A
fícios estão previstos no Código Penal, aplicável na Lei
MULHER
Maria da Penha.
CAPÍTULO I
LEI Nº 11.340, DE 7 DE AGOSTO DE 2006. DISPOSIÇÕES GERAIS
Cria mecanismos para coibir a violência doméstica Art. 5o Para os efeitos desta Lei, configura violência
e familiar contra a mulher, nos termos do § 8º do art. doméstica e familiar contra a mulher qualquer
226 da Constituição Federal, da Convenção sobre a Eli- ação ou omissão baseada no gênero que lhe cause
minação de Todas as Formas de Discriminação contra as morte, lesão, sofrimento físico, sexual ou psicoló-
Mulheres e da Convenção Interamericana para Prevenir, gico e dano moral ou patrimonial:
Punir e Erradicar a Violência contra a Mulher; dispõe sobre I - no âmbito da unidade doméstica, compreendi-
a criação dos Juizados de Violência Doméstica e Familiar da como o espaço de convívio permanente de pessoas,
contra a Mulher; altera o Código de Processo Penal, o Có- com ou sem vínculo familiar, inclusive as esporadica-
digo Penal e a Lei de Execução Penal; e dá outras provi- mente agregadas;
dências. II - no âmbito da família, compreendida como a co-
O PRESIDENTE DA REPÚBLICA Faço saber que o Con- munidade formada por indivíduos que são ou se con-
gresso Nacional decreta e eu sanciono a seguinte Lei: sideram aparentados, unidos por laços naturais, por
afinidade ou por vontade expressa;
TÍTULO I
LEGISLAÇÃO ESPECÍFICA

III - em qualquer relação íntima de afeto, na qual o


DISPOSIÇÕES PRELIMINARES agressor conviva ou tenha convivido com a ofendida,
independentemente de coabitação.
Art. 1o Esta Lei cria mecanismos para coibir e preve- Parágrafo único. As relações pessoais enunciadas nes-
nir a violência doméstica e familiar contra a mu- te artigo independem de orientação sexual.
lher, nos termos do § 8o do art. 226 da Constituição
Federal, da Convenção sobre a Eliminação de Todas as Art. 6o A violência doméstica e familiar contra a mu-
Formas de Violência contra a Mulher, da Convenção lher constitui uma das formas de violação dos direi-
Interamericana para Prevenir, Punir e Erradicar a Vio- tos humanos.

77
CAPÍTULO II III - o respeito, nos meios de comunicação social, dos
DAS FORMAS DE VIOLÊNCIA DOMÉSTICA E FAMILIAR valores éticos e sociais da pessoa e da família, de
CONTRA A MULHER forma a coibir os papéis estereotipados que legiti-
mem ou exacerbem a violência doméstica e familiar,
Art. 7o São formas de violência doméstica e familiar de acordo com o estabelecido no inciso III do art. 1o,
contra a mulher, entre outras: no inciso IV do art. 3o e no inciso IV do art. 221 da
I - a violência física, entendida como qualquer con- Constituição Federal;
duta que ofenda sua integridade ou saúde corporal; IV - a implementação de atendimento policial espe-
II - a violência psicológica, entendida como qual- cializado para as mulheres, em particular nas Dele-
quer conduta que lhe cause dano emocional e di- gacias de Atendimento à Mulher;
minuição da auto-estima ou que lhe prejudique e V - a promoção e a realização de campanhas edu-
perturbe o pleno desenvolvimento ou que vise de- cativas de prevenção da violência doméstica e fa-
gradar ou controlar suas ações, comportamentos, miliar contra a mulher, voltadas ao público escolar
crenças e decisões, mediante ameaça, constrangimen- e à sociedade em geral, e a difusão desta Lei e dos
to, humilhação, manipulação, isolamento, vigilância instrumentos de proteção aos direitos humanos das
constante, perseguição contumaz, insulto, chantagem, mulheres;
ridicularização, exploração e limitação do direito de ir VI - a celebração de convênios, protocolos, ajustes,
e vir ou qualquer outro meio que lhe cause prejuízo termos ou outros instrumentos de promoção de
à saúde psicológica e à autodeterminação; parceria entre órgãos governamentais ou entre estes
III - a violência sexual, entendida como qualquer e entidades não-governamentais, tendo por objetivo a
conduta que a constranja a presenciar, a manter implementação de programas de erradicação da vio-
ou a participar de relação sexual não desejada, lência doméstica e familiar contra a mulher;
mediante intimidação, ameaça, coação ou uso da for- VII - a capacitação permanente das Polícias Civil e
ça; que a induza a comercializar ou a utilizar, de Militar, da Guarda Municipal, do Corpo de Bombeiros
qualquer modo, a sua sexualidade, que a impeça de e dos profissionais pertencentes aos órgãos e às áreas
usar qualquer método contraceptivo ou que a force enunciados no inciso I quanto às questões de gênero
ao matrimônio, à gravidez, ao aborto ou à prostitui- e de raça ou etnia;
ção, mediante coação, chantagem, suborno ou mani- VIII - a promoção de programas educacionais que
pulação; ou que limite ou anule o exercício de seus disseminem valores éticos de irrestrito respeito à dig-
direitos sexuais e reprodutivos; nidade da pessoa humana com a perspectiva de gêne-
IV - a violência patrimonial, entendida como qual- ro e de raça ou etnia;
quer conduta que configure retenção, subtração, IX - o destaque, nos currículos escolares de todos os
destruição parcial ou total de seus objetos, instru- níveis de ensino, para os conteúdos relativos aos di-
mentos de trabalho, documentos pessoais, bens, va- reitos humanos, à equidade de gênero e de raça ou
lores e direitos ou recursos econômicos, incluindo os etnia e ao problema da violência doméstica e familiar
destinados a satisfazer suas necessidades; contra a mulher.
V - a violência moral, entendida como qualquer con-
duta que configure calúnia, difamação ou injúria. CAPÍTULO II
DA ASSISTÊNCIA À MULHER EM SITUAÇÃO DE VIO-
TÍTULO III
LÊNCIA DOMÉSTICA E FAMILIAR
DA ASSISTÊNCIA À MULHER EM SITUAÇÃO DE VIO-
LÊNCIA DOMÉSTICA E FAMILIAR
Art. 9o A assistência à mulher em situação de violência
doméstica e familiar será prestada de forma articu-
CAPÍTULO I
lada e conforme os princípios e as diretrizes previstos
DAS MEDIDAS INTEGRADAS DE PREVENÇÃO
na Lei Orgânica da Assistência Social, no Sistema Úni-
co de Saúde, no Sistema Único de Segurança Pública,
Art. 8o A política pública que visa coibir a violência do-
méstica e familiar contra a mulher far-se-á por meio entre outras normas e políticas públicas de proteção, e
de um conjunto articulado de ações da União, dos emergencialmente quando for o caso.
Estados, do Distrito Federal e dos Municípios e de § 1o O juiz determinará, por prazo certo, a inclusão da
ações não-governamentais, tendo por diretrizes: mulher em situação de violência doméstica e familiar
I - a integração operacional do Poder Judiciário, no cadastro de programas assistenciais do gover-
do Ministério Público e da Defensoria Pública com as no federal, estadual e municipal.
áreas de segurança pública, assistência social, saúde, § 2o O juiz assegurará à mulher em situação de violên-
LEGISLAÇÃO ESPECÍFICA

educação, trabalho e habitação; cia doméstica e familiar, para preservar sua integrida-
II - a promoção de estudos e pesquisas, estatísticas de física e psicológica:
e outras informações relevantes, com a perspectiva I - acesso prioritário à remoção quando servidora
de gênero e de raça ou etnia, concernentes às causas, pública, integrante da administração direta ou indi-
às consequências e à frequência da violência domés- reta;
tica e familiar contra a mulher, para a sistematização II - manutenção do vínculo trabalhista, quando ne-
de dados, a serem unificados nacionalmente, e a ava- cessário o afastamento do local de trabalho, por até
liação periódica dos resultados das medidas adotadas; seis meses.

78
§ 3o A assistência à mulher em situação de violência § 1o O pedido da ofendida será tomado a termo pela
doméstica e familiar compreenderá o acesso aos be- autoridade policial e deverá conter:
nefícios decorrentes do desenvolvimento científico e I - qualificação da ofendida e do agressor;
tecnológico, incluindo os serviços de contracepção II - nome e idade dos dependentes;
de emergência, a profilaxia das Doenças Sexual- III - descrição sucinta do fato e das medidas proteti-
mente Transmissíveis (DST) e da Síndrome da Imuno- vas solicitadas pela ofendida.
deficiência Adquirida (AIDS) e outros procedimentos § 2o A autoridade policial deverá anexar ao documento
médicos necessários e cabíveis nos casos de violência referido no § 1o o boletim de ocorrência e cópia de
sexual. todos os documentos disponíveis em posse da ofendida.
§ 3o Serão admitidos como meios de prova os laudos ou
CAPÍTULO III prontuários médicos fornecidos por hospitais e postos
DO ATENDIMENTO PELA AUTORIDADE POLICIAL de saúde.

Art. 10. Na hipótese da iminência ou da prática de TÍTULO IV


violência doméstica e familiar contra a mulher, a au- DOS PROCEDIMENTOS
toridade policial que tomar conhecimento da ocor-
rência adotará, de imediato, as providências legais CAPÍTULO I
cabíveis. DISPOSIÇÕES GERAIS
Parágrafo único. Aplica-se o disposto no caput deste
artigo ao descumprimento de medida protetiva de Art. 13. Ao processo, ao julgamento e à execução das
urgência deferida. causas cíveis e criminais decorrentes da prática de vio-
lência doméstica e familiar contra a mulher aplicar-
Art. 11. No atendimento à mulher em situação de -se-ão as normas dos Códigos de Processo Penal e
violência doméstica e familiar, a autoridade policial Processo Civil e da legislação específica relativa à
deverá, entre outras providências: criança, ao adolescente e ao idoso que não conflitarem
I - garantir proteção policial, quando necessário, com o estabelecido nesta Lei.
comunicando de imediato ao Ministério Público e ao
Poder Judiciário;
Art. 14. Os Juizados de Violência Doméstica e Fa-
II - encaminhar a ofendida ao hospital ou posto de
miliar contra a Mulher, órgãos da Justiça Ordiná-
saúde e ao Instituto Médico Legal;
ria com competência cível e criminal, poderão ser
III - fornecer transporte para a ofendida e seus de-
criados pela União, no Distrito Federal e nos Territó-
pendentes para abrigo ou local seguro, quando hou-
rios, e pelos Estados, para o processo, o julgamento e a
ver risco de vida;
execução das causas decorrentes da prática de violência
IV - se necessário, acompanhar a ofendida para as-
doméstica e familiar contra a mulher.
segurar a retirada de seus pertences do local da
ocorrência ou do domicílio familiar; Parágrafo único. Os atos processuais poderão reali-
V - informar à ofendida os direitos a ela conferidos zar-se em horário noturno, conforme dispuserem as
nesta Lei e os serviços disponíveis. normas de organização judiciária.

Art. 12. Em todos os casos de violência doméstica e Art. 15. É competente, por opção da ofendida, para os
familiar contra a mulher, feito o registro da ocorrência, processos cíveis regidos por esta Lei, o Juizado:
deverá a autoridade policial adotar, de imediato, os I - do seu domicílio ou de sua residência;
seguintes procedimentos, sem prejuízo daqueles pre- II - do lugar do fato em que se baseou a demanda;
vistos no Código de Processo Penal: III - do domicílio do agressor.
I - ouvir a ofendida, lavrar o boletim de ocorrência
e tomar a representação a termo, se apresentada; Art. 16. Nas ações penais públicas condicionadas à
II - colher todas as provas que servirem para o escla- representação da ofendida de que trata esta Lei, só
recimento do fato e de suas circunstâncias; será admitida a renúncia à representação perante
III - remeter, no prazo de 48 (quarenta e oito) horas, o juiz, em audiência especialmente designada com tal
expediente apartado ao juiz com o pedido da ofendi- finalidade, antes do recebimento da denúncia e ouvido
da, para a concessão de medidas protetivas de ur- o Ministério Público.
gência;
IV - determinar que se proceda ao exame de corpo Art. 17. É vedada a aplicação, nos casos de violência
de delito da ofendida e requisitar outros exames pe- doméstica e familiar contra a mulher, de penas de ces-
LEGISLAÇÃO ESPECÍFICA

riciais necessários; ta básica ou outras de prestação pecuniária, bem


V - ouvir o agressor e as testemunhas; como a substituição de pena que implique o pagamento
VI - ordenar a identificação do agressor e fazer jun- isolado de multa.
tar aos autos sua folha de antecedentes criminais,
indicando a existência de mandado de prisão ou regis-
tro de outras ocorrências policiais contra ele;
VII - remeter, no prazo legal, os autos do inquérito
policial ao juiz e ao Ministério Público.

79
CAPÍTULO II I - suspensão da posse ou restrição do porte de
DAS MEDIDAS PROTETIVAS DE URGÊNCIA armas, com comunicação ao órgão competente, nos
termos da Lei no 10.826, de 22 de dezembro de 2003;
Seção I II - afastamento do lar, domicílio ou local de con-
Disposições Gerais vivência com a ofendida;
III - proibição de determinadas condutas, entre as
Art. 18. Recebido o expediente com o pedido da ofen- quais:
dida, caberá ao juiz, no prazo de 48 (quarenta e a) aproximação da ofendida, de seus familiares e das
oito) horas: testemunhas, fixando o limite mínimo de distância
I - conhecer do expediente e do pedido e decidir entre estes e o agressor;
sobre as medidas protetivas de urgência; b) contato com a ofendida, seus familiares e testemu-
II - determinar o encaminhamento da ofendida ao nhas por qualquer meio de comunicação;
órgão de assistência judiciária, quando for o caso; c) frequentação de determinados lugares a fim de
III - comunicar ao Ministério Público para que ado- preservar a integridade física e psicológica da ofen-
te as providências cabíveis. dida;
IV - restrição ou suspensão de visitas aos depen-
Art. 19. As medidas protetivas de urgência poderão dentes menores, ouvida a equipe de atendimento
ser concedidas pelo juiz, a requerimento do Ministé- multidisciplinar ou serviço similar;
rio Público ou a pedido da ofendida. V - prestação de alimentos provisionais ou provisó-
§ 1o As medidas protetivas de urgência poderão ser rios.
concedidas de imediato, independentemente de § 1o As medidas referidas neste artigo não impe-
audiência das partes e de manifestação do Minis- dem a aplicação de outras previstas na legislação
tério Público, devendo este ser prontamente comu- em vigor, sempre que a segurança da ofendida ou as
nicado. circunstâncias o exigirem, devendo a providência ser
§ 2o As medidas protetivas de urgência serão apli- comunicada ao Ministério Público.
cadas isolada ou cumulativamente, e poderão ser § 2o Na hipótese de aplicação do inciso I, encontrando-
substituídas a qualquer tempo por outras de maior -se o agressor nas condições mencionadas no caput e
eficácia, sempre que os direitos reconhecidos nesta Lei incisos do art. 6o da Lei no 10.826, de 22 de dezembro
forem ameaçados ou violados. de 2003, o juiz comunicará ao respectivo órgão, cor-
§ 3o Poderá o juiz, a requerimento do Ministério Públi- poração ou instituição as medidas protetivas de ur-
co ou a pedido da ofendida, conceder novas medidas gência concedidas e determinará a restrição do porte
protetivas de urgência ou rever aquelas já concedidas, de armas, ficando o superior imediato do agressor res-
se entender necessário à proteção da ofendida, de seus ponsável pelo cumprimento da determinação judicial,
familiares e de seu patrimônio, ouvido o Ministério sob pena de incorrer nos crimes de prevaricação ou de
Público. desobediência, conforme o caso.
§ 3o Para garantir a efetividade das medidas protetivas
Art. 20. Em qualquer fase do inquérito policial ou da de urgência, poderá o juiz requisitar, a qualquer mo-
instrução criminal, caberá a prisão preventiva do mento, auxílio da força policial.
agressor, decretada pelo juiz, de ofício, a requerimen- § 4o Aplica-se às hipóteses previstas neste artigo, no
to do Ministério Público ou mediante representação que couber, o disposto no caput e nos §§ 5o e 6º do art.
da autoridade policial. 461 da Lei nº 5.869, de 11 de janeiro de 1973 (Código
Parágrafo único. O juiz poderá revogar a prisão pre- de Processo Civil).
ventiva se, no curso do processo, verificar a falta de
motivo para que subsista, bem como de novo decre- Seção III
tá-la, se sobrevierem razões que a justifiquem. Das Medidas Protetivas de Urgência à Ofendida

Art. 21. A ofendida deverá ser notificada dos atos Art. 23. Poderá o juiz, quando necessário, sem prejuí-
processuais relativos ao agressor, especialmente dos zo de outras medidas:
pertinentes ao ingresso e à saída da prisão, sem pre- I - encaminhar a ofendida e seus dependentes a pro-
juízo da intimação do advogado constituído ou do de- grama oficial ou comunitário de proteção ou de
fensor público. atendimento;
Parágrafo único. A ofendida não poderá entregar inti- II - determinar a recondução da ofendida e a de seus
mação ou notificação ao agressor. dependentes ao respectivo domicílio, após afasta-
mento do agressor;
LEGISLAÇÃO ESPECÍFICA

Seção II III - determinar o afastamento da ofendida do lar,


Das Medidas Protetivas de Urgência que Obrigam o sem prejuízo dos direitos relativos a bens, guarda dos
Agressor filhos e alimentos;
IV - determinar a separação de corpos.
Art. 22. Constatada a prática de violência doméstica e
familiar contra a mulher, nos termos desta Lei, o juiz Art. 24. Para a proteção patrimonial dos bens da
poderá aplicar, de imediato, ao agressor, em conjunto sociedade conjugal ou daqueles de propriedade
ou separadamente, as seguintes medidas protetivas particular da mulher, o juiz poderá determinar, li-
de urgência, entre outras: minarmente, as seguintes medidas, entre outras:

80
I - restituição de bens indevidamente subtraídos pelo TÍTULO V
agressor à ofendida; DA EQUIPE DE ATENDIMENTO MULTIDISCIPLINAR
II - proibição temporária para a celebração de atos e
contratos de compra, venda e locação de propriedade Art. 29. Os Juizados de Violência Doméstica e Fa-
em comum, salvo expressa autorização judicial; miliar contra a Mulher que vierem a ser criados
III - suspensão das procurações conferidas pela ofen- poderão contar com uma equipe de atendimento
dida ao agressor; multidisciplinar, a ser integrada por profissionais es-
IV - prestação de caução provisória, mediante depó- pecializados nas áreas psicossocial, jurídica e de saú-
sito judicial, por perdas e danos materiais decorrentes de.
da prática de violência doméstica e familiar contra a
ofendida. Art. 30. Compete à equipe de atendimento mul-
Parágrafo único. Deverá o juiz oficiar ao cartório com- tidisciplinar, entre outras atribuições que lhe forem
petente para os fins previstos nos incisos II e III deste reservadas pela legislação local, fornecer subsídios
artigo. por escrito ao juiz, ao Ministério Público e à Defen-
soria Pública, mediante laudos ou verbalmente em
Seção IV audiência, e desenvolver trabalhos de orientação,
Do Crime de Descumprimento de Medidas Protetivas encaminhamento, prevenção e outras medidas, vol-
de Urgência Descumprimento de Medidas Protetivas tados para a ofendida, o agressor e os familiares, com
de Urgência especial atenção às crianças e aos adolescentes.

Art. 24-A. Descumprir decisão judicial que defere Art. 31. Quando a complexidade do caso exigir ava-
medidas protetivas de urgência previstas nesta Lei: liação mais aprofundada, o juiz poderá determinar a
Pena – detenção, de 3 (três) meses a 2 (dois) anos. manifestação de profissional especializado, me-
§ 1o A configuração do crime independe da competên- diante a indicação da equipe de atendimento multi-
cia civil ou criminal do juiz que deferiu as medidas. disciplinar.
§ 2o Na hipótese de prisão em flagrante, apenas a
autoridade judicial poderá conceder fiança. Art. 32. O Poder Judiciário, na elaboração de sua pro-
§ 3o O disposto neste artigo não exclui a aplicação de posta orçamentária, poderá prever recursos para a
outras sanções cabíveis. criação e manutenção da equipe de atendimento
CAPÍTULO III multidisciplinar, nos termos da Lei de Diretrizes Or-
DA ATUAÇÃO DO MINISTÉRIO PÚBLICO çamentárias.

Art. 25. O Ministério Público intervirá, quando não TÍTULO VI


for parte, nas causas cíveis e criminais decorrentes da DISPOSIÇÕES TRANSITÓRIAS
violência doméstica e familiar contra a mulher.
Art. 33. Enquanto não estruturados os Juizados de Vio-
Art. 26. Caberá ao Ministério Público, sem prejuízo de lência Doméstica e Familiar contra a Mulher, as varas
outras atribuições, nos casos de violência doméstica e criminais acumularão as competências cível e cri-
familiar contra a mulher, quando necessário: minal para conhecer e julgar as causas decorrentes
I - requisitar força policial e serviços públicos de da prática de violência doméstica e familiar contra a
saúde, de educação, de assistência social e de seguran- mulher, observadas as previsões do Título IV desta Lei,
ça, entre outros; subsidiada pela legislação processual pertinente.
II - fiscalizar os estabelecimentos públicos e parti- Parágrafo único. Será garantido o direito de prefe-
culares de atendimento à mulher em situação de vio- rência, nas varas criminais, para o processo e o julga-
lência doméstica e familiar, e adotar, de imediato, as mento das causas referidas no caput.
medidas administrativas ou judiciais cabíveis no tocan-
te a quaisquer irregularidades constatadas; TÍTULO VII
III - cadastrar os casos de violência doméstica e fami- DISPOSIÇÕES FINAIS
liar contra a mulher.
Art. 34. A instituição dos Juizados de Violência
CAPÍTULO IV Doméstica e Familiar contra a Mulher poderá ser
DA ASSISTÊNCIA JUDICIÁRIA acompanhada pela implantação das curadorias ne-
cessárias e do serviço de assistência judiciária.
Art. 27. Em todos os atos processuais, cíveis e criminais,
LEGISLAÇÃO ESPECÍFICA

a mulher em situação de violência doméstica e familiar Art. 35. A União, o Distrito Federal, os Estados e os
deverá estar acompanhada de advogado, ressalvado Municípios poderão criar e promover, no limite das
o previsto no art. 19 desta Lei. respectivas competências:
I - centros de atendimento integral e multidisci-
Art. 28. É garantido a toda mulher em situação de vio- plinar para mulheres e respectivos dependentes em
lência doméstica e familiar o acesso aos serviços de situação de violência doméstica e familiar;
Defensoria Pública ou de Assistência Judiciária II - casas-abrigos para mulheres e respectivos depen-
Gratuita, nos termos da lei, em sede policial e judi- dentes menores em situação de violência doméstica e
cial, mediante atendimento específico e humanizado. familiar;

81
III - delegacias, núcleos de defensoria pública, servi- Art. 43. A alínea f do inciso II do art. 61 do Decreto-Lei
ços de saúde e centros de perícia médico-legal espe- no 2.848, de 7 de dezembro de 1940 (Código Penal),
cializados no atendimento à mulher em situação de passa a vigorar com a seguinte redação:
violência doméstica e familiar; “Art. 61. ..................................................
IV - programas e campanhas de enfrentamento da .................................................................
violência doméstica e familiar; II - ............................................................
V - centros de educação e de reabilitação para os .................................................................
agressores. f) com abuso de autoridade ou prevalecendo-se de re-
lações domésticas, de coabitação ou de hospitalidade,
Art. 36. A União, os Estados, o Distrito Federal e os ou com violência contra a mulher na forma da lei es-
Municípios promoverão a adaptação de seus órgãos pecífica;
e de seus programas às diretrizes e aos princípios ........................................................... ” (NR)
desta Lei.
Art. 44. O art. 129 do Decreto-Lei nº 2.848, de 7 de de-
Art. 37. A defesa dos interesses e direitos transindivi- zembro de 1940 (Código Penal), passa a vigorar com
duais previstos nesta Lei poderá ser exercida, concor- as seguintes alterações:
rentemente, pelo Ministério Público e por associa- “Art. 129. ..................................................
ção de atuação na área, regularmente constituída ..................................................................
há pelo menos um ano, nos termos da legislação § 9o Se a lesão for praticada contra ascendente, des-
civil. cendente, irmão, cônjuge ou companheiro, ou com
Parágrafo único. O requisito da pré-constituição po- quem conviva ou tenha convivido, ou, ainda, prevale-
derá ser dispensado pelo juiz quando entender que cendo-se o agente das relações domésticas, de coabi-
não há outra entidade com representatividade ade- tação ou de hospitalidade:
quada para o ajuizamento da demanda coletiva. Pena - detenção, de 3 (três) meses a 3 (três) anos.
..................................................................
Art. 38. As estatísticas sobre a violência doméstica § 11. Na hipótese do § 9o deste artigo, a pena será au-
e familiar contra a mulher serão incluídas nas bases mentada de um terço se o crime for cometido contra
pessoa portadora de deficiência.” (NR)
de dados dos órgãos oficiais do Sistema de Justiça e
Segurança a fim de subsidiar o sistema nacional de
Art. 45. O art. 152 da Lei no 7.210, de 11 de julho de
dados e informações relativo às mulheres.
1984 (Lei de Execução Penal), passa a vigorar com a
Parágrafo único. As Secretarias de Segurança Pública
seguinte redação:
dos Estados e do Distrito Federal poderão remeter suas
“Art. 152. ...................................................
informações criminais para a base de dados do Minis-
Parágrafo único. Nos casos de violência doméstica
tério da Justiça.
contra a mulher, o juiz poderá determinar o compare-
cimento obrigatório do agressor a programas de recu-
Art. 39. A União, os Estados, o Distrito Federal e os peração e reeducação.” (NR)
Municípios, no limite de suas competências e nos ter-
mos das respectivas leis de diretrizes orçamentárias, Art. 46. Esta Lei entra em vigor 45 (quarenta e cinco)
poderão estabelecer dotações orçamentárias espe- dias após sua publicação.
cíficas, em cada exercício financeiro, para a imple-
mentação das medidas estabelecidas nesta Lei. Brasília, 7 de agosto de 2006; 185o da Independência e
118o da República.
Art. 40. As obrigações previstas nesta Lei não ex-
cluem outras decorrentes dos princípios por ela
adotados.

Art. 41. Aos crimes praticados com violência domésti-


ca e familiar contra a mulher, independentemente da
pena prevista, não se aplica a Lei no 9.099, de 26 de
setembro de 1995.

Art. 42. O art. 313 do Decreto-Lei no 3.689, de 3 de


outubro de 1941 (Código de Processo Penal), passa a
LEGISLAÇÃO ESPECÍFICA

vigorar acrescido do seguinte inciso IV:


“Art. 313. .................................................
................................................................
IV - se o crime envolver violência doméstica e familiar
contra a mulher, nos termos da lei específica, para ga-
rantir a execução das medidas protetivas de urgência.”
(NR)

82
§ 5º A adoção será assistida pelo Poder Público, na
LEI ORDINÁRIA Nº 8.069, DE 13 DE forma da lei, que estabelecerá casos e condições de sua
JULHO DE 1990 (ESTATUTO DA CRIANÇA efetivação por parte de estrangeiros.
E DO ADOLESCENTE); DOS DIREITOS § 6º Os filhos, havidos ou não da relação do ca-
samento, ou por adoção, terão os mesmos direitos e
FUNDAMENTAIS; DA PRÁTICA DO ATO
qualificações, proibidas quaisquer designações discrimi-
INFRACIONAL; natórias relativas à filiação.
§ 7º No atendimento dos direitos da criança e do ado-
lescente levar-se-á em consideração o disposto no art.
Noções introdutórias e disciplina constitucional 20461.
§ 8º A lei estabelecerá:
Art. 227. É dever da família, da sociedade e do Es- I - o estatuto da juventude, destinado a regular os
tado assegurar à criança, ao adolescente e ao jovem, com direitos dos jovens;
absoluta prioridade, o direito à vida, à saúde, à alimen- II - o plano nacional de juventude, de duração de-
tação, à educação, ao lazer, à profissionalização, à cultu- cenal, visando à articulação das várias esferas do poder
ra, à dignidade, ao respeito, à liberdade e à convivência público para a execução de políticas públicas.
familiar e comunitária, além de colocá-los a salvo de toda
forma de negligência, discriminação, exploração, vio- No caput do artigo 227, CF se encontra uma das prin-
lência, crueldade e opressão. cipais diretrizes do direito da criança e do adolescente
§ 1º O Estado promoverá programas de assistência que é o princípio da prioridade absoluta. Significa que
integral à saúde da criança, do adolescente e do jovem, cada criança e adolescente deve receber tratamento es-
admitida a participação de entidades não governa- pecial do Estado e ser priorizado em suas políticas públi-
mentais, mediante políticas específicas e obedecendo aos cas, pois são o futuro do país e as bases de construção
seguintes preceitos: da sociedade.
I - aplicação de percentual dos recursos públicos desti- A Lei nº 8.069, de 13 de julho de 1990 dispõe sobre
nados à saúde na assistência materno-infantil; o Estatuto da Criança e do Adolescente e dá outras pro-
II - criação de programas de prevenção e atendi- vidências, seguindo em seus dispositivos a ideologia do
mento especializado para as pessoas portadoras de de- princípio da absoluta prioridade.
ficiência física, sensorial ou mental, bem como de inte- No §1º do artigo 227 aborda-se a questão da assis-
gração social do adolescente e do jovem portador de tência à saúde da criança e do adolescente. Do inciso I
deficiência, mediante o treinamento para o trabalho e a se depreende a intrínseca relação entre a proteção da
convivência, e a facilitação do acesso aos bens e serviços criança e do adolescente com a proteção da maternidade
coletivos, com a eliminação de obstáculos arquitetônicos e e da infância, mencionada no artigo 6º, CF. Já do inciso
de todas as formas de discriminação. II se depreende a proteção de outro grupo vulnerável,
§ 2º A lei disporá sobre normas de construção dos que é a pessoa portadora de deficiência, valendo lembrar
logradouros e dos edifícios de uso público e de fabrica- que o Decreto nº 6.949, de 25 de agosto de 2009, que
ção de veículos de transporte coletivo, a fim de garantir promulga a Convenção Internacional sobre os Direitos
acesso adequado às pessoas portadoras de deficiência. das Pessoas com Deficiência e seu Protocolo Facultati-
§ 3º O direito a proteção especial abrangerá os se- vo, assinados em Nova York, em 30 de março de 2007,
guintes aspectos: foi promulgado após aprovação no Congresso Nacional
I - idade mínima de quatorze anos para admissão nos moldes da Emenda Constitucional nº 45/2004, tendo
ao trabalho, observado o disposto no art. 7º, XXXIII; força de norma constitucional e não de lei ordinária. A
II - garantia de direitos previdenciários e trabalhistas; preocupação com o direito da pessoa portadora de defi-
III - garantia de acesso do trabalhador adolescente e ciência se estende ao §2º do artigo 227, CF: “a lei dispo-
jovem à escola; rá sobre normas de construção dos logradouros e dos
IV - garantia de pleno e formal conhecimento da edifícios de uso público e de fabricação de veículos de
atribuição de ato infracional, igualdade na relação transporte coletivo, a fim de garantir acesso adequado às
processual e defesa técnica por profissional habilita- pessoas portadoras de deficiência”.
do, segundo dispuser a legislação tutelar específica;
61 Art. 204. As ações governamentais na área da assistência
V - obediência aos princípios de brevidade, excepcio- social serão realizadas com recursos do orçamento da seguridade
nalidade e respeito à condição peculiar de pessoa em social, previstos no art. 195, além de outras fontes, e organizadas
desenvolvimento, quando da aplicação de qualquer medi- com base nas seguintes diretrizes: I - descentralização político-ad-
da privativa da liberdade; ministrativa, cabendo a coordenação e as normas gerais à esfera
VI - estímulo do Poder Público, através de assistência federal e a coordenação e a execução dos respectivos programas às
LEGISLAÇÃO ESPECÍFICA

jurídica, incentivos fiscais e subsídios, nos termos da esferas estadual e municipal, bem como a entidades beneficentes e
de assistência social; II - participação da população, por meio de
lei, ao acolhimento, sob a forma de guarda, de criança ou
organizações representativas, na formulação das políticas e no con-
adolescente órfão ou abandonado; trole das ações em todos os níveis. Parágrafo único. É facultado aos
VII - programas de prevenção e atendimento espe- Estados e ao Distrito Federal vincular a programa de apoio à inclu-
cializado à criança, ao adolescente e ao jovem dependen- são e promoção social até cinco décimos por cento de sua receita
te de entorpecentes e drogas afins. tributária líquida, vedada a aplicação desses recursos no pagamento
§ 4º A lei punirá severamente o abuso, a violência e de: I - despesas com pessoal e encargos sociais; II - serviço da dívi-
a exploração sexual da criança e do adolescente. da; III - qualquer outra despesa corrente não vinculada diretamente
aos investimentos ou ações apoiados.

83
A proteção especial que decorre do princípio da prio- Ademais, a proteção à criança, ao adolescente e ao
ridade absoluta está prevista no §3º do artigo 227. Li- jovem representa incumbência atribuída não só ao Es-
ga-se, ainda, à proteção especial, a previsão do §4º do tado, mas também à família e à sociedade. Sendo assim,
artigo 227: “A lei punirá severamente o abuso, a violência há se prestar bastante atenção nas provas de concurso,
e a exploração sexual da criança e do adolescente”. tendo em vista que só se costuma colocar o Estado como
Tendo em vista o direito de toda criança e adolescente observador da “Doutrina da Proteção Integral”, sendo
de ser criado no seio de uma família, o §5º do artigo 227 que isso também compete à família e à sociedade.
da Constituição prevê que “a adoção será assistida pelo Nesta frequência, o direito à proteção especial abran-
Poder Público, na forma da lei, que estabelecerá casos e gerá os seguintes aspectos (art. 227, §3º, CF):
condições de sua efetivação por parte de estrangeiros”. - A idade mínima de dezesseis anos para admissão ao
Neste sentido, a Lei nº 12.010, de 3 de agosto de 2009, trabalho, salvo a partir dos quatorze anos, na condição
dispõe sobre a adoção. de aprendiz (inciso I de acordo com o art. 7º, XXXIII, CF,
A igualdade entre os filhos, quebrando o paradigma da pós-alteração promovida pela Emenda Constitucional nº
Constituição anterior e do até então vigente Código Civil 20/98);
de 1916 consta no artigo 227, § 6º, CF: “os filhos, havidos - A garantia de direitos previdenciários e trabalhistas
ou não da relação do casamento, ou por adoção, terão os (inciso II);
mesmos direitos e qualificações, proibidas quaisquer de- - A garantia de acesso ao trabalhador adolescente e
signações discriminatórias relativas à filiação”. jovem à escola (inciso III);
Quando o artigo 227 dispõe no § 7º que “no atendi- - A garantia de pleno e formal conhecimento da atri-
mento dos direitos da criança e do adolescente levar-se-á buição do ato infracional, igualdade na relação proces-
em consideração o disposto no art. 204” tem em vista a sual e defesa técnica por profissional habilitado, segundo
adoção de práticas de assistência social, com recursos da dispuser a legislação tutelar específica (inciso IV);
seguridade social, em prol da criança e do adolescente. - A obediência aos princípios de brevidade, excepcio-
Por seu turno, o artigo 227, § 8º, CF, preconiza: “A lei nalidade e respeito à condição peculiar de pessoa em de-
estabelecerá: I - o estatuto da juventude, destinado a senvolvimento, quando da aplicação de qualquer medida
regular os direitos dos jovens; II - o plano nacional de privativa de liberdade (inciso V);
juventude, de duração decenal, visando à articulação - O estímulo do Poder Público, através de assistência
das várias esferas do poder público para a execução de jurídica, incentivos fiscais e subsídios, nos termos da lei,
políticas públicas”. A Lei nº 12.852, de 5 de agosto de
ao acolhimento, sob a forma de guarda, de criança ou
2013, institui o Estatuto da Juventude e dispõe sobre os
adolescente órfão ou abandonado (inciso VI);
direitos dos jovens, os princípios e diretrizes das políticas
- Programas de prevenção e atendimento especializa-
públicas de juventude e o Sistema Nacional de Juventude
do à criança, ao adolescente e ao jovem dependente de
- SINAJUVE. Mais informações sobre a Política menciona-
entorpecentes e drogas afins (inciso VII).
da no inciso II e sobre a Secretaria e o Conselho Nacional
Prosseguindo, o parágrafo sexto, do art. 227, da
de Juventude que direcionam a implementação dela po-
Constituição, garante o “Princípio da Igualdade entre os
dem ser obtidas na rede62.
Filhos”, ao dispor que os filhos, havidos ou não da relação
Aprofundando o tema, a cabeça do art. 227, da Lei
Fundamental, preconiza ser dever da família, da socieda- do casamento, ou por adoção, terão os mesmos direitos
de e do Estado assegurar à criança, ao adolescente e ao e qualificações, proibidas quaisquer designações discri-
jovem, com absoluta prioridade, o direito à vida, à saúde, minatórias relativas à filiação.
à alimentação, à educação, ao lazer, à profissionalização, Assim, com a Constituição Federal, os filhos não têm
à cultura, à dignidade, ao respeito, à liberdade e à convi- mais “valor” para efeito de direitos alimentícios e suces-
vência familiar e comunitária, além de colocá-los a salvo sórios. Não se pode falar em um filho receber metade
de toda forma de negligência, discriminação, exploração, da parte que originalmente lhe cabia por ser “bastardo”,
violência, crueldade e opressão. enquanto aquele fruto da sociedade conjugal receber a
A leitura do art. 227, caput, da Constituição Federal quantia integral. Aliás, nem mesmo a expressão “filho
permite concluir que se adotou, neste país, a chamada bastardo” pode mais ser utilizada, por representar uma
“Doutrina da Proteção Integral da Criança”, ao lhe asse- forma de discriminação designatória.
gurar a absoluta prioridade em políticas públicas, medi- Também, o art. 229 traz uma “via de mão dupla” entre
das sociais, decisões judiciais, respeito aos direitos hu- pais e filhos, isto é, os pais têm o dever de assistir, criar e
manos, e observância da dignidade da pessoa humana. educar os filhos menores, e os filhos maiores têm o dever
Neste sentido, o parágrafo único, do art. 5º, do “Estatuto de ajudar e amparar os pais na velhice, carência ou en-
da Criança e do Adolescente”, prevê que a garantia de fermidade. Tal dispositivo, inclusive, permite que os filhos
prioridade compreende a primazia de receber proteção peçam alimentos aos pais, e que os pais peçam alimentos
LEGISLAÇÃO ESPECÍFICA

e socorro em quaisquer circunstâncias (alínea “a”), a pre- aos filhos.


cedência de atendimento nos serviços públicos ou de re- Por fim, há se mencionar o acrescentado parágrafo
levância pública (alínea “b”), a preferência na formulação oitavo (pela Emenda Constitucional nº 65/2010), ao art.
e na execução das políticas sociais públicas (alínea “c”), e 227, da Constituição Federal, segundo o qual a lei es-
a destinação privilegiada de recursos públicos nas áreas tabelecerá o estatuto da juventude, destinado a regular
relacionadas com a proteção à infância e à juventude (alí- os direitos dos jovens (inciso I), e o plano nacional de
nea “d”). juventude, de duração decenal, visando à articulação
das várias esferas do poder público para a execução de
62 http://www.juventude.gov.br/politica

84
políticas públicas (inciso II). Nada obstante a exigência a Infância, mas mantém a sigla que a tornara conhecida
constitucional desde 2010, somente bem recentemente o em todo o mundo – UNICEF. Desde então, sobrevieram
Estatuto da Juventude foi aprovado (Lei nº 12.852/2013), no âmbito das Nações Unidas documentos bastante re-
como visto acima, carecendo, ainda, o Plano Nacional de levantes sobre a condição jurídica peculiar da criança, já
Juventude de maior regulamentação infraconstitucional. estudados neste material.
No Brasil, no final do século XIX e início do século XX,
Evolução histórica foi instituído no Rio de Janeiro o Instituto de Proteção e
Assistência à Infância, primeiro estabelecimento público
Na Grécia antiga, a criança era colocada numa posi- nacional de atendimento a crianças e adolescentes. Em
ção de inferioridade, tida como um ser irracional, sem ca- seguida, veio a Lei nº 4.242/1921, que autorizou o go-
pacidade de tomar qualquer tipo de decisão. Trata-se de verno a organizar o Serviço de Assistência e Proteção à
marco da cultura grega, que enxergava apenas poucos Infância Abandonada e Dellinquente. Em 1927 foi apro-
homens de posses como cidadãos. Estes homens con- vado o primeiro Código de Menores. Em 1941, durante
centravam para si o pátrio poder, isto é, o poder do pai. o governo Vargas, foi criado o Serviço de Assistência ao
Devido ao pátrio poder, o pai de família concentrava em Menor, cujo fim era dar tratamento penal teoricamen-
suas mãos plena possibilidade de gerir a vida das crian- te diferenciado aos menores (na prática, eram tratados
ças e adolescentes e estes não tinham nenhuma possibi- como criminosos comuns). Em 1964 surge a Política Na-
lidade de participar destas decisões. Na Idade Média se cional do Bem-estar do Menor (Lei nº 4.513/1964), que
manteve o sistema do “pátrio poder”. As crianças eram criou a FUNABEM. Surge novo Código de Menores em
submetidas ao absoluto poder do pai e seus destinos se- 1979 (Lei nº 6.697), cujo objeto era a proteção e vigilân-
guiam a mesma sorte. cia de crianças e adolescentes em situação irregular. Na
A partir da Idade Moderna, com o Renascimento e o década de 80 começa um movimento de reelaboração
Iluminismo, as crianças e os adolescentes saíram ligei- da concepção de infância e juventude. O destaque re-
ramente da margem social. A moral da época passa a percute na Constituição Federal de 1988 e no Estatuto
impor aos pais o dever de educar seus filhos. Entretanto, da Criança e do Adolescente de 1990, que revogou o
a educação costumava ser oferecida apenas aos homens. Código de Menores e substituiu a doutrina da situação
Aqueles que possuíam melhores condições enviavam irregular pela doutrina da proteção integral63.
seus filhos para estudarem nas universidades que co-
meçavam a despontar na Europa, aqueles que possuíam Relações jurídicas no direito da criança e do ado-
condições piores ao menos passavam a ensinar seus ofí- lescente
cios a estes jovens. Já as meninas permaneciam margina-
lizadas das atividades educacionais e profissionalizantes, “As relações jurídicas são formas qualificadas de rela-
apenas lhes era ensinado como desempenhar atividades ções interpessoais, indicando, assim, a ligação entre pes-
domésticas. soas, em razão de algum objeto, devidamente regulada
Desde o final da Revolução Francesa e, com destaque, pelo direito. Desta forma, o Direito da Criança e do Ado-
a partir da Revolução Industrial, que alterou substancial- lescente, sob o aspecto objetivo e formal, representa a
mente os modos e métodos de produção, a criança e o disciplina das relações jurídicas entre Crianças e Adoles-
adolescente passam a ocupar papel central na sociedade, centes, de um lado, e de outro, a família, a comunidade,
desempenhando atividades trabalhistas de caráter equi- a sociedade e o próprio Estado. [...] Percebemos que a
valente a dos adultos. Foram vítimas de inúmeros aciden- intenção dos doutrinadores e do próprio legislador foi,
tes de trabalho, morriam em meio à insalubridade das fá- sempre, criar uma doutrina da proteção integral não so-
bricas, então movidas predominantemente a carvão. Foi mente para a Criança, como, ainda, para o Adolescente,
apenas com a emergência da Organização Internacional ambos ainda em desenvolvimento, posto que, somente
do Trabalho – OIT, em 1919, que aos poucos se consoli- com o término da adolescência é que o menor comple-
dou uma consciência a respeito da necessidade de se li- tará o processo de aquisição de mecanismos mentais re-
mitar a participação das crianças e adolescentes no espa- lacionados ao pensamento, percepção, reconhecimento,
ço de trabalho. Este foi o estopim para o reconhecimento classificação etc. [...] Com isso, o Estatuto da Criança e do
da condição especial da criança e do adolescente. Adolescente, sabiamente, se preocupou em envolver não
Internacionalmente, a proteção efetiva da criança e somente a família, mas, ainda, a comunidade, a socie-
do adolescente começa a tomar corpo com o reconhe- dade e o próprio Estado, para que todos, em conjunto,
cimento internacional dos direitos humanos e a funda- exerçam seus direitos e deveres sem oprimir aqueles que,
ção da UNICEF. A UNICEF, inicialmente conhecida como em condição inferior, viviam a mercê da sociedade. Mas,
Fundo Internacional de Emergência das Nações Unidas qual a razão dessa inclusão tão abrangente? Pois bem,
para as Crianças, foi criada em dezembro de 1946 para a intenção do Estatuto da Criança e do Adolescente foi
LEGISLAÇÃO ESPECÍFICA

ajudar as crianças da Europa vítimas da II Guerra Mun- conferir ao menor, de forma integral, todas as condições
dial. No início da década de 50 o seu mandato foi alar- para que o mesmo possa desenvolver-se plenamente,
gado para responder às necessidades das crianças e das evitando-se, com isso, que haja alguma deficiência em
mães nos países em desenvolvimento. Em 1953, torna-se sua formação. Desta forma, a melhor solução apresen-
uma agência permanente das Nações Unidas, e passa a
ocupar-se especialmente das crianças dos países mais 63 DEZEM, Guilherme Madeira; AGUIRRE, João Ricardo
pobres da África, Ásia, América Latina e Médio Oriente. Brandão; FULLER, Paulo Henrique Aranda. Estatuto da Criança e
do Adolescente. São Paulo: Revista dos Tribunais, 2009. (Coleção
Passa então a designar-se Fundo das Nações Unidas para
Elementos do Direito)

85
tada pelo legislador foi incluir todos os segmentos da c) Princípio da dignidade da pessoa humana: A dig-
sociedade, para que ninguém ficasse isento de qualquer nidade da pessoa humana é o valor-base de interpretação
responsabilidade, uma vez que a doutrina da proteção de qualquer sistema jurídico, internacional ou nacional,
integral apresentada pelo Estatuto da Criança e do Ado- que possa se considerar compatível com os valores éti-
lescente exige a participação de todos, sem qualquer ex- cos, notadamente da moral, da justiça e da democracia.
ceção”64. Com efeito, o objeto formal do direito da criança Pensar em dignidade da pessoa humana significa, acima
e do adolescente é a proteção jurídica especial da criança de tudo, colocar a pessoa humana como centro e norte
e do adolescente. Já o objeto material é a própria criança para qualquer processo de interpretação jurídico, seja na
ou adolescente. elaboração da norma, seja na sua aplicação.
Sem pretender estabelecer uma definição fechada ou
Princípios plena, é possível conceituar dignidade da pessoa huma-
na como o principal valor do ordenamento ético e, por
Não se pode olvidar que os princípios sempre desem- consequência, jurídico que pretende colocar a pessoa
penharam um importante papel social, mas foi somente humana como um sujeito pleno de direitos e obriga-
na atual dogmática jurídica que eles adquiriram normati- ções na ordem internacional e nacional, cujo desrespeito
vidade. Hoje em dia, os princípios servem para condensar acarreta a própria exclusão de sua personalidade.
valores, dar unidade ao sistema e condicionar a ativida- Aponta Barroso67: “o princípio da dignidade da pes-
de do intérprete. Os princípios são normas jurídicas, não soa humana identifica um espaço de integridade moral
meros conteúdos axiológicos, aceitando aplicação autô- a ser assegurado a todas as pessoas por sua só existên-
noma65. cia no mundo. É um respeito à criação, independente da
Em resumo, a teoria dos princípios chega à presente crença que se professe quanto à sua origem. A dignidade
fase do Pós-positivismo com os seguintes resultados já relaciona-se tanto com a liberdade e valores do espírito
consolidados: a passagem dos princípios da especulação como com as condições materiais de subsistência”.
metafísica e abstrata para o campo concreto e positivo O Ministro Alberto Luiz Bresciani de Fontan Pereira,
do Direito, com baixíssimo teor de densidade normati-
do Tribunal Superior do Trabalho, trouxe interessante
va; a transição crucial da ordem jusprivatista (sua antiga
conceito numa das decisões que relatou: “a dignidade
inserção nos Códigos) para a órbita juspublicística (seu
consiste na percepção intrínseca de cada ser humano a
ingresso nas Constituições); a suspensão da distinção
respeito dos direitos e obrigações, de modo a assegurar,
clássica entre princípios e normas; o deslocamento dos
sob o foco de condições existenciais mínimas, a partici-
princípios da esfera da jusfilosofia para o domínio da
pação saudável e ativa nos destinos escolhidos, sem que
Ciência Jurídica; a proclamação de sua normatividade; a
isso importe destilação dos valores soberanos da demo-
perda de seu caráter de normas programáticas; o reco-
nhecimento definitivo de sua positividade e concretude cracia e das liberdades individuais. O processo de valo-
por obra sobretudo das Constituições; a distinção en- rização do indivíduo articula a promoção de escolhas,
tre regras e princípios, como espécies diversificadas do posturas e sonhos, sem olvidar que o espectro de abran-
gênero norma, e, finalmente, por expressão máxima de gência das liberdades individuais encontra limitação em
todo esse desdobramento doutrinário, o mais significa- outros direitos fundamentais, tais como a honra, a vida
tivo de seus efeitos: a total hegemonia e preeminência privada, a intimidade, a imagem. Sobreleva registrar que
dos princípios66. essas garantias, associadas ao princípio da dignidade da
No campo do direito da criança e do adolescente, al- pessoa humana, subsistem como conquista da humani-
guns princípios assumem destaque, entre eles: dade, razão pela qual auferiram proteção especial con-
a) Princípio da prioridade absoluta: previsto nos ar- sistente em indenização por dano moral decorrente de
tigos 227, CF e 4º, ECA preconiza que é dever de todos sua violação”68.
– Estado, sociedade, comunidade e família – assegurar Para Reale69, a evolução histórica demonstra o do-
com absoluta prioridade direitos fundamentais às crian- mínio de um valor sobre o outro, ou seja, a existência
ças e adolescentes. Por isso, estabelece-se com primazia de uma ordem gradativa entre os valores; mas existem
a adoção de políticas públicas, a destinação de recursos os valores fundamentais e os secundários, sendo que o
e a prestação de serviços essenciais àqueles que se en- valor fonte é o da pessoa humana. Nesse sentido, são
contram na faixa etária inferior a 18 anos. os dizeres de Reale70: “partimos dessa ideia, a nosso ver
b) Princípio da proteção integral: previsto no artigo básica, de que a pessoa humana é o valor-fonte de to-
1º, ECA estabelece que a proteção da criança e do ado- dos os valores. O homem, como ser natural biopsíquico,
lescente não pode se restringir às situações de irregula- é apenas um indivíduo entre outros indivíduos, um ente
ridade, o que teria um caráter estigmatizante, mas deve animal entre os demais da mesma espécie. O homem,
abranger todas as situações de vida pelas quais passa a considerado na sua objetividade espiritual, enquanto ser
criança e o adolescente, mesmo as regulares. Neste sen-
LEGISLAÇÃO ESPECÍFICA

tido, ao se assegurar direitos na regularidade, evita-se 67 BARROSO, Luís Roberto. Interpretação e aplicação da
que a criança e o adolescente caiam em irregularidade. Constituição. 7. ed. São Paulo: Saraiva, 2009, p. 382.
68 BRASIL. Tribunal Superior do Trabalho. Recurso de Re-
64 MENDES, Moacyr Pereira. As relações jurídicas decorren- vista n. 259300-59.2007.5.02.0202. Relator: Alberto Luiz Bresciani
tes do Estatuto da Criança e do Adolescente. Âmbito Jurídico, Rio de Fontan Pereira. Brasília, 05 de setembro de 2012j1. Disponível
Grande, XII, n. 70, nov. 2009. em: www.tst.gov.br. Acesso em: 17 nov. 2012.
65 Ibid., p.327. 69 REALE, Miguel. Filosofia do direito. 19. ed. São Paulo:
66 BONAVIDES, Paulo. Curso de direito constitucional. 26. Saraiva, 2002, p. 228.
ed. São Paulo: Malheiros, 2011, p. 294. 70 Ibid., p. 220.

86
que só realiza no sentido de seu dever ser, é o que cha- dade, criando a necessidade de se ter figuras aptas a de-
mamos de pessoa. Só o homem possui a dignidade origi- cidir e responder por eles, como se estas figuras fossem
nária de ser enquanto deve ser, pondo-se essencialmente sempre e inevitavelmente imbuídas das melhores inten-
como razão determinante do processo histórico”. ções em relação à criança e ao adolescente.
Quando a Constituição Federal assegura a dignidade No entender de Kopelman, para que toda esta legis-
da pessoa humana como um dos fundamentos da Repú- lação fosse realmente válida seria necessário definir me-
blica, faz emergir uma nova concepção de proteção de lhor, de maneira bem precisa, o que se entende por um
cada membro do seu povo. Tal ideologia de forte fulcro padrão mínimo de benefício ou o que é ‘o melhor’ para
humanista guia a afirmação de todos os direitos funda- os interesses da criança ou do adolescente, de modo que
mentais e confere a eles posição hierárquica superior às a definição não fique em aberto para a interpretação de
normas organizacionais do Estado, de modo que é o Es- quem detém o poder de decidir em nome deles. Além
tado que está para o povo, devendo garantir a dignidade disso, estas definições deveriam estar em constante revi-
de seus membros, e não o inverso. são, para que não acabem sendo ultrapassadas, frente à
d) Princípio da participação popular: previsto no evolução histórico-social dos fatos que geraram a neces-
artigo 227, §§ 3º e 7º e no artigo 204, II, CF, assegura a sidade de sua criação.
participação popular, através de organizações represen- Superados estes dois pontos, que apesar de poten-
tativas, na elaboração de políticas públicas direcionadas à cialmente limitantes do processo de discussão da auto-
infância e à juventude. nomia da criança e do adolescente não podem ser sim-
e) Princípio da excepcionalidade: previsto no artigo plesmente ignorados, como se não existissem, chega-se
227, §3º, V, CF assegura que quando da imposição de me- ao terceiro e mais importante: a interpretação do concei-
dida privativa de liberdade esta não será imposta a não to de autonomia à luz do momento de desenvolvimen-
ser que se trate de um caso excepcional, em que nenhu- to em que uma determinada criança ou adolescente se
ma outra medida sócio-educativa possa ser utilizada. encontra.
f) Princípio da brevidade: previsto no artigo 227, §3º, Nesse sentido, diversas características do desenvolvi-
V, CF assegura que quando da aplicação de medida pri- mento devem ser levadas em consideração:
vativa de liberdade esta não se estenderá no tempo, de- 1. Trata-se de um processo que evolui continuamente
vendo ser a mais breve possível, perdurando apenas pelo à medida que habilidades se aperfeiçoam, novas capa-
prazo necessário para a ressocialização do adolescente. cidades são adquiridas, novas vivências são acumuladas
No caso, o ECA limita a aplicação de medidas desta natu- e integradas e, portanto, passível de rápidas e extremas
reza ao prazo máximo de 3 anos. mudanças no tempo;
2. A aquisição das competências é progressiva, não se
g) Princípio da condição peculiar da pessoa em de-
dá saltos, como se se tratasse de compartimentos estan-
senvolvimento: a criança e o adolescente estão em pro-
ques, e segue sempre uma ordem preestabelecida, sen-
cesso de formação e de transformação física e psíquica,
do, portanto, razoavelmente previsível;
logo, possuem uma condição peculiar que deve ser res-
3. Os tempos e o ritmo em que o desenvolvimento se
peitada quando da aplicação da lei.
processa são muito individualizados, fazendo com que
dois indivíduos de uma mesma idade possam estar em
Autonomia da criança e do adolescente
momentos diferentes de desenvolvimento;
4. No caso específico da inteligência, o desenvolvi-
Coloca-se o trecho do trabalho de Cláudio Leone71 em mento é extremamente influenciável por fatores extrín-
que reflete sobre a construção da autonomia do infante: secos ao indivíduo: as experiências, os estímulos, o am-
“Conceitualmente, a análise do respeito à autonomia biente, a educação, a cultura, etc., o que também acaba
de uma criança ou de um adolescente só tem sentido se por reforçar sua evolução extremamente individualizada.
for conduzida a partir do conhecimento da evolução de Segundo Piaget, a capacidade de operar o pensa-
suas competências nas diferentes idades. É de conheci- mento concreto estendendo-o à compreensão do outro
mento de todos que a criança nasce totalmente depen- e às possíveis consequências de boa parte dos seus atos
dente de cuidados alheios e que passa por um processo se aperfeiçoa na idade escolar, entre os 6 e os 11 anos de
de desenvolvimento progressivo que a leva a alcançar a vida. Este amadurecimento se completa na adolescência,
completa independência na maturidade, o que, nas so- com a capacidade crescente de abstração que a criança
ciedades modernas, se situa por volta dos vinte anos de desenvolve nesta fase da existência. Como consequência,
idade. é possível admitir que é na segunda fase da adolescência,
Entretanto, para que este processo de análise de sua em geral a partir dos 15 anos, que o indivíduo atingiria
autonomia transcorra de maneira isenta, fundamental- as competências necessárias para o exercício de sua au-
mente centrado nas peculiaridades do desenvolvimento tonomia, competências estas que necessitariam apenas
do ser humano, o primeiro ponto a ser considerado é a
LEGISLAÇÃO ESPECÍFICA

serem lapidadas ao longo das vivências e de uma maior


necessidade de abdicar de alguns conceitos preestabele- experiência de vida.
cidos, como é o caso da atitude paternalista. [...] Entretanto, isto não significa que a autonomia da
O segundo ponto a considerar neste percurso, em criança e do adolescente só possa (ou deva) ser respeita-
geral decorrente do primeiro, é a própria legislação que, da a partir desta fase.
mesmo tendo o melhor dos intuitos, praticamente nivela Compete ao pediatra e aos demais profissionais de
todos os menores a uma mesma condição: a de incapaci- saúde, utilizando suas competências profissionais, defi-
nir já desde os primeiros anos de vida em que etapa a
71 LEONE, Cláudio. A criança, o adolescente e a autonomia.
criança se encontra ao longo do seu processo evolutivo,
Revista Bioética, v. 6, n. 1.

87
tentando diferenciar se se está diante de uma tomada O artigo 228, CF dispõe: “são penalmente inimputá-
de decisão ditada apenas pelo receio do desconhecido, veis os menores de dezoito anos, sujeitos às normas da
por um capricho ou vontade decorrente apenas de sua vi- legislação especial”. Percebe-se que a normativa não está
são egocêntrica, natural em determinadas idades, ou se a no rol de cláusulas pétreas, razão pela qual seria possível
mesma já é o resultado de uma reflexão mais amadureci- uma emenda constitucional que alterasse a menoridade
da. São estes extremos que dão a entender a ampla gama penal. Inclusive, há projetos de lei neste sentido.
de estágios de desenvolvimento, portanto de autonomia,
que entre eles podem se apresentar. [...] Comentários à lei
Novamente, cabe enfatizar que o risco que se corre ao
se utilizar definições bastante precisas como estas é o de Parte geral
acabar classificando um indivíduo de maneira dicotômica,
no caso específico da autonomia, como sendo capaz ou Título I
incapaz, desistindo assim de uma possível análise de sua Das Disposições Preliminares
real capacidade.
Consequentemente, a ausência de uma ou de mais Art. 1º Esta Lei dispõe sobre a proteção integral à
das características anteriormente citadas não deve ser uti- criança e ao adolescente.
lizada para qualificar a criança ou o adolescente como in- O princípio da proteção integral se associa ao princí-
capaz. Deve, isto sim, servir de embasamento para que se pio da prioridade absoluta, colacionado no artigo 4º do
possa tentar entender como suas decisões se originaram. ECA e no artigo 227, CF. “Com a positivação desse prin-
Em face de situações específicas, individualizadas, cípio tem-se também a positivação da proteção integral,
como ocorre no dia-a-dia da prática pediátrica, esta é a que se opõe à antiga e superada doutrina da situação
única forma que o profissional tem de realmente respeitar irregular, que era prevista no antigo Código de Menores
a autonomia da criança ou do adolescente. e especificava que sua incidência se restringia aos me-
A interpretação adequada da legislação e o dimensio- nores em situação irregular, apresentando um conjunto
namento correto da decisão dos pais ou responsáveis de- de normas destinadas ao tratamento e prevenção dessas
penderão fundamentalmente deste tipo de análise da au- situações”72.
tonomia da criança ou adolescente. Deste modo, mesmo Basicamente, tinha-se na doutrina da situação irre-
que resulte em situações de conflito entre as posições, gular que era necessário disciplinar um estatuto jurídico
servirá de embasamento para um trabalho, muitas vezes da criança e do adolescente que apenas abordasse situa-
exaustivo, de apresentação, de reflexão e de discussão ções em que ele estivesse irregular, seja por uma despro-
de argumentos e fatos, capaz de conduzir a uma decisão teção, como no caso de abandono, ou pela violação da
amadurecida e o mais isenta possível, que, respeitando lei, como nos casos de atos infracionais.
a posição da criança ou do adolescente, poderá efetiva- Entretanto, o direito evoluiu e passou a contemplar
mente redundar em seu benefício. uma noção de proteção mais ampla da criança e do ado-
No leque das diferentes situações da prática pediátri- lescente, que não apenas abordasse situações de irregu-
ca, que se estende desde o recém-nascido no limite de laridade (embora ainda o fizesse), mas que abrangesse
viabilidade ao qual se quer prestar cuidados intensivos de todo o arcabouço jurídico protetivo da criança e do ado-
validade questionável naquelas circunstâncias, passando lescente.
pelas pesquisas científicas que envolvem crianças e ado-
lescentes, até a criança cujo pátrio poder pertence a pais Art. 2º Considera-se criança, para os efeitos desta Lei,
adolescentes, portanto autônomos nas decisões que lhes a pessoa até doze anos de idade incompletos, e adoles-
dizem respeito, todas estas situações, onde nem sempre cente aquela entre doze e dezoito anos de idade.
o real interesse que está em jogo é o da criança, mas sim Parágrafo único. Nos casos expressos em lei, aplica-se
o dos responsáveis por ela, clarificam que não há uma excepcionalmente este Estatuto às pessoas entre dezoito e
única resposta ou solução mágica, perfeita, para a ques- vinte e um anos de idade.
tão da autonomia da criança e do adolescente. O Estatuto da Criança e do Adolescente opta por ca-
Na realidade, o que deve existir é a construção conjun- tegorizar separadamente estas duas categorias de meno-
ta de uma verdade para aquele momento, amadurecida res. Criança é aquele que tem até 12 anos de idade (na
no crescimento e evolução de todos: juízes e legisladores, data de aniversário de 12 anos, passa a ser adolescente),
pais ou responsáveis, médicos e profissionais de saúde adolescente é aquele que tem entre 12 e 18 anos (na
e, principalmente, a criança ou o adolescente, como par- data de aniversário de 18 anos, passa a ser maior). Em
te de um processo de interação franco, sincero, isento e situações excepcionais o ECA se aplica ao maior de 18
realmente participativo que de fato respeite a autonomia, anos, até os 21 anos de idade, por exemplo, no caso do
qualquer que seja o nível de competência que a criança menor infrator sujeito a internação em fundação CASA
LEGISLAÇÃO ESPECÍFICA

ou o adolescente estejam apresentando para tal”. que tenha 17 anos e 11 meses na data do ato infracio-
nal poderá ficar detido até o limite de seus 20 anos e 11
Imputabilidade penal meses (eis que 3 anos é o tempo máximo de internação).

Art. 228, CF. São penalmente inimputáveis os me-


nores de dezoito anos, sujeitos às normas da legislação 72 DEZEM, Guilherme Madeira; AGUIRRE, João Ricardo
especial. Brandão; FULLER, Paulo Henrique Aranda. Estatuto da Criança e
do Adolescente. São Paulo: Revista dos Tribunais, 2009. (Coleção
Elementos do Direito)

88
Art. 3º A criança e o adolescente gozam de todos os Deste artigo 3º do ECA é possível, ainda, extrair o
direitos fundamentais inerentes à pessoa humana, destaque ao princípio da igualdade, no sentido de que
sem prejuízo da proteção integral de que trata esta Lei, há plena igualdade na garantia de direitos entre todas as
assegurando-se-lhes, por lei ou por outros meios, todas crianças e adolescentes, não sendo permitido qualquer
as oportunidades e facilidades, a fim de lhes facultar tipo de discriminação.
o desenvolvimento físico, mental, moral, espiritual e
social, em condições de liberdade e de dignidade. A leitura dos artigos 4º e 5º, em conjunto com outros
Parágrafo único. Os direitos enunciados nesta Lei dispositivos do ECA, por sua vez, permite detectar a pre-
aplicam-se a todas as crianças e adolescentes, sem dis- sença de um tríplice sistema de garantias.
criminação de nascimento, situação familiar, idade, sexo, Assim, o Estatuto da Criança e do Adolescente adota
raça, etnia ou cor, religião ou crença, deficiência, condição uma estrutura que contempla três sistemas de garantia
pessoal de desenvolvimento e aprendizagem, condição – primário, secundário e terciário.
econômica, ambiente social, região e local de moradia ou a) Sistema primário – artigos 4º e 87, ECA – aborda
outra condição que diferencie as pessoas, as famílias ou a políticas públicas de atendimento de crianças e adoles-
comunidade em que vivem. centes.
O artigo 3º volta-se à concretização dos direitos da
criança e do adolescente. Concretização significa viabi- Art. 4º É dever da família, da comunidade, da socie-
lização prática, consecução real dos fins que a lei des- dade em geral e do poder público assegurar, com ab-
creve. Como se percebe pela leitura até o momento, soluta prioridade, a efetivação dos direitos referentes à
o legislador brasileiro preocupou-se em elaborar uma vida, à saúde, à alimentação, à educação, ao esporte, ao
legislação cujo objetivo é concretizar estes direitos da lazer, à profissionalização, à cultura, à dignidade, ao res-
criança e do adolescente. Entretanto, a lei é apenas uma peito, à liberdade e à convivência familiar e comunitária.
carta de intenções. É necessário colocar seu conteúdo Parágrafo único. A garantia de prioridade com-
em prática, porque sozinha ela nada faz. preende:
A implementação na prática dos direitos da criança a) primazia de receber proteção e socorro em
e do adolescente depende da adoção de posturas por quaisquer circunstâncias;
parte de todos aqueles colocados como responsáveis b) precedência de atendimento nos serviços públicos
para tanto: Estado, sociedade, comunidade e família. Es- ou de relevância pública;
pecificamente no que se refere ao Estado, mostra-se es- c) preferência na formulação e na execução das
sencial que ele desenvolve políticas públicas adequadas políticas sociais públicas;
em respeito à peculiar condição do infante. d) destinação privilegiada de recursos públicos nas
“O Direito da Criança e do Adolescente deve ter con- áreas relacionadas com a proteção à infância e à juven-
dições suficientemente próprias de promoção e concre- tude.
tização de direitos. Para isso deve-se desvencilhar do O artigo 4º do ECA colaciona em seu caput teor idên-
dogmatismo e do mero positivismo jurídico acrítico. O tico ao do caput do artigo 227, CF, onde se encontra
Direito da Criança e do Adolescente enquanto ramo au- uma das principais diretrizes do direito da criança e do
tônomo do direito é responsável por ressignificar a atua- adolescente que é o princípio da prioridade absoluta.
ção estatal, principalmente no campo das políticas pú- Significa que cada criança e adolescente deve receber
tratamento especial do Estado e ser priorizado em suas
blicas e impõe corresponsabilidades compartilhadas”73.
políticas públicas, pois são o futuro do país e as bases de
Vale ressaltar que às crianças e aos adolescentes são
construção da sociedade.
garantidos os mesmos direitos fundamentais que aos
Explica Liberati74: “Por absoluta prioridade, devemos
adultos, entretanto, o ECA aprofunda alguns direitos
entender que a criança e o adolescente deverão estar
fundamentais em espécie, abordando-os na vertente da
em primeiro lugar na escala de preocupação dos go-
condição especial dos que pertencem a este grupo.
vernantes; devemos entender que, primeiro, devem ser
As crianças e adolescentes gozam de igualdade de
atendidas todas as necessidades das crianças e adoles-
direitos em relação às demais pessoas, podendo usufruir
centes [...]. Por absoluta prioridade, entende-se que, na
de todos eles. O próprio estatuto contempla em seu
área administrativa, enquanto não existirem creches, es-
título II os direitos fundamentais da criança e do ado- colas, postos de saúde, atendimento preventivo e emer-
lescente, entre eles incluindo-se: vida, saúde, liberdade, gencial às gestantes dignas moradias e trabalho, não
respeito, dignidade, convivência familiar e comunitá- se deveria asfaltar ruas, construir praças, sambódromos
ria, educação, cultura, esporte, lazer, profissionalização monumentos artísticos etc., porque a vida, a saúde, o lar,
e proteção no trabalho. Não se trata de rol taxativo de a prevenção de doenças são importantes que as obras
LEGISLAÇÃO ESPECÍFICA

direitos fundamentais garantidos à criança e ao adoles- de concreto que ficam par a demonstrar o poder do go-
cente, eis que ele possui todos os direitos humanos e vernante”.
fundamentais que as demais pessoas. O título II do ECA O parágrafo único do artigo 4º especifica a abran-
tem por objetivo aprofundar especificidades acerca de gência da absoluta prioridade, esclarecendo que é ne-
algumas das categorias de direitos fundamentais asse- cessário conferir atendimento prioritário às crianças e
gurados à criança e ao adolescente. aos adolescentes diante de situações de perigo e risco
73 http://t.boletimjuridico.com.br/doutrina/texto.as- 74 LIBERATI, Wilson Donizeti. O Estatuto da Criança e do
p?id=2236 Adolescente: Comentários. São Paulo: IBPS.

89
(como no salvamento em incêndios e enchentes, etc.), O artigo 5º ressalta o verdadeiro objetivo geral do
bem como nos serviços públicos em geral (chegada aos ECA: proteger a criança de qualquer forma de negli-
hospitais, por exemplo). Além disso, devem ser prioriza- gência, discriminação, exploração, violência, crueldade
das políticas públicas que favoreçam a criança e o adoles- e opressão. Neste sentido, coloca-se a possibilidade de
cente e também devem ser reservados recursos próprios responsabilização de todos que atentarem contra esse
prioritariamente a eles. propósito. A responsabilização poderá se dar em qual-
quer uma das três esferas, isolada ou cumulativamente:
Art. 87. São linhas de ação da política de atendi- penal, respondendo por crimes e contravenções penais
mento: todo aquele que praticá-lo contra criança e adolescente,
I - políticas sociais básicas; bem como respondendo por atos infracionais as crianças
II - serviços, programas, projetos e benefícios de as- e adolescentes que atentarem um contra o outro; civil,
sistência social de garantia de proteção social e de preven- estabelecendo-se o dever de indenizar por danos cau-
ção e redução de violações de direitos, seus agravamentos sados a crianças e a adolescentes, que se estende a toda
ou reincidências; e qualquer pessoa física ou jurídica que o faça, inclusive
III - serviços especiais de prevenção e atendimento o próprio Estado; e administrativa, impondo-se penas
médico e psicossocial às vítimas de negligência, maus-tra- disciplinares a funcionários sujeitos a regime jurídico ad-
tos, exploração, abuso, crueldade e opressão; ministrativo em trabalhos privados ou em cargos, empre-
IV - serviço de identificação e localização de pais, gos e funções públicos.
responsável, crianças e adolescentes desaparecidos;
V - proteção jurídico-social por entidades de defesa Art. 6º Na interpretação desta Lei levar-se-ão em conta
dos direitos da criança e do adolescente. os fins sociais a que ela se dirige, as exigências do bem
VI - políticas e programas destinados a prevenir ou comum, os direitos e deveres individuais e coletivos,
abreviar o período de afastamento do convívio fami- e a condição peculiar da criança e do adolescente como
liar e a garantir o efetivo exercício do direito à convi- pessoas em desenvolvimento.
vência familiar de crianças e adolescentes;
VII - campanhas de estímulo ao acolhimento sob É pacífico que o processo de interpretação hoje faz
forma de guarda de crianças e adolescentes afastados do parte do Direito, principalmente se considerada a cons-
convívio familiar e à adoção, especificamente inter-racial, tante evolução da sociedade, demandando diariamente
de crianças maiores ou de adolescentes, com necessidades
por novos modos de aplicação das normas. Como a so-
específicas de saúde ou com deficiências e de grupos de ir-
ciedade é dinâmica e o Direito existe para servi-la, cabe a
mãos.
ele adequar-se às novas exigências sociais, aplicando-se
O artigo 87 descreve linhas de ação na política de
da maneira mais justa à vasta gama de casos concretos.
atendimento, que compõem a delimitação do princípio da
Sobre a interpretação, explica Gonçalves75: “Quando o
prioridade absoluta na vertente da priorização na adoção
fato é típico e se enquadra perfeitamente no conceito
de políticas públicas e na delimitação de recursos financei-
abstrato da norma, dá-se o fenômeno da subsunção. Há
ros para execução de tais políticas.
casos, no entanto, em que tal enquadramento não ocor-
b) Sistema secundário – artigos 98 e 101, ECA – abor- re, não encontrando o juiz nenhuma norma aplicável à
da as medidas de proteção destinadas à criança e ao ado- hipótese sub judice. Deve, então, proceder à integração
lescente em situação de risco pessoal ou social. normativa, mediante o emprego da analogia, dos costu-
Obs.: as medidas de proteção são estudadas adiante mes e dos princípios gerais do direito. [...] Para verificar
neste material. se a norma é aplicável ao caso em julgamento (subsun-
c) Sistema terciário – artigo 112, ECA – aborda as ção) ou se deve proceder à integração normativa, o juiz
medidas socioeducativas, destinadas à responsabilização procura descobrir o sentido da norma, interpretando-a.
penal do adolescente infrator, isto é, àquele entre 12 e 18 Interpretar é descobrir o sentido e o alcance da norma
anos que comete atos infracionais. jurídica”.
Obs.: as medidas socioeducativas são estudadas adian- A hermenêutica possui 3 categorias de métodos.
te neste material. Quanto às fontes ou origem, a interpretação pode ser
O sistema tríplice deve operar de forma harmônica, autêntica ou legislativa, jurisprudencial ou judicial e dou-
com o acionamento gradual de cada um deles. Nas situa- trinária. Quanto aos meios, pode ser gramatical ou literal,
ções em que a criança ou adolescente escape ao sistema examinando o texto normativo linguísticamente; lógica
primário de prevenção, ou seja, nos casos de ineficácia das ou racional, apurando o sentido e a finalidade da nor-
políticas públicas específicas, deve ser acionado o sistema ma; sistemática, analisando a lei de maneira comparati-
secundário, operado predominantemente pelo Conselho va com outras leis pertencentes à mesma província do
Tutelas. Por sua vez, em casos extremos, é necessário partir Direito (livro, título, capítulo, seção, parágrafo); histórica,
LEGISLAÇÃO ESPECÍFICA

para a adoção de medidas socioeducativas, operadas pre- baseando-se na verificação dos antecedentes do proces-
dominantemente pelo Ministério Público e pelo Judiciário. so legislativo; sociológica, adaptando o sentido ou fi-
nalidade da norma às novas exigências sociais (artigo
Art. 5º Nenhuma criança ou adolescente será objeto de 5°, LINDB). Quanto aos resultados pode ser declarativa,
qualquer forma de negligência, discriminação, explo- quando o texto legal corresponde ao pensamento do le-
ração, violência, crueldade e opressão, punido na forma gislador; extensiva ou ampliativa, quando o alcance da lei
da lei qualquer atentado, por ação ou omissão, aos seus
direitos fundamentais. 75 GONÇALVES, Carlos Roberto. Direito Civil Brasileiro. 9.
ed. São Paulo: Saraiva, 2011, v. 1.

90
é mais amplo que o indicado pelo seu texto; e restritiva, § 8º A gestante tem direito a acompanhamento sau-
na qual se limita o campo de aplicação da lei. Nenhum dável durante toda a gestação e a parto natural cuidado-
destes métodos se opera isoladamente76. so, estabelecendo-se a aplicação de cesariana e outras
O artigo 6º do ECA, tal como o artigo 5º da LINDB, intervenções cirúrgicas por motivos médicos.
expressa o método de interpretação sociológico, cha- § 9º A atenção primária à saúde fará a busca ativa da
mando atenção à interpretação da lei levando em conta gestante que não iniciar ou que abandonar as consultas de
os seus fins sociais, as exigências do bem comum, os di- pré-natal, bem como da puérpera que não comparecer às
reitos e deveres individuais e coletivos, e vai além: exi- consultas pós-parto.
ge que se leve em conta a condição peculiar da criança § 10. Incumbe ao poder público garantir, à gestante e
e do adolescente. Logo, ao se interpretar o ECA não se à mulher com filho na primeira infância que se encontrem
pode nunca perder de vista que o seu objeto material, a sob custódia em unidade de privação de liberdade, am-
criança e o adolescente, é extremamente peculiar, dota- biência que atenda às normas sanitárias e assistenciais do
do de especificidades as quais sempre se deve atentar. Sistema Único de Saúde para o acolhimento do filho, em
articulação com o sistema de ensino competente, visando
Título II ao desenvolvimento integral da criança.
Dos Direitos Fundamentais
Art. 8º-A. Fica instituída a Semana Nacional de Pre-
Capítulo I venção da Gravidez na Adolescência, a ser realizada anual-
Do Direito à Vida e à Saúde mente na semana que incluir o dia 1º de fevereiro, com
o objetivo de disseminar informações sobre medidas pre-
Art. 7º A criança e o adolescente têm direito a prote- ventivas e educativas que contribuam para a redução da
ção à vida e à saúde, mediante a efetivação de políticas incidência da gravidez na adolescência.(Incluído pela Lei nº
sociais públicas que permitam o nascimento e o desen- 13.798, de 2019)
volvimento sadio e harmonioso, em condições dignas Parágrafo único. As ações destinadas a efetivar o dis-
posto no caput deste artigo ficarão a cargo do poder pú-
de existência.
blico, em conjunto com organizações da sociedade civil,
e serão dirigidas prioritariamente ao público adolescente.
Art. 8º É assegurado a todas as mulheres o acesso
(Incluído pela Lei nº 13.798, de 2019)
aos programas e às políticas de saúde da mulher e
de planejamento reprodutivo e, às gestantes, nutrição Art. 9º O poder público, as instituições e os emprega-
adequada, atenção humanizada à gravidez, ao parto e dores propiciarão condições adequadas ao aleitamento
ao puerpério e atendimento pré-natal, perinatal e pós- materno, inclusive aos filhos de mães submetidas a
-natal integral no âmbito do Sistema Único de Saúde. medida privativa de liberdade.
§ 1º O atendimento pré-natal será realizado por pro- § 1º Os profissionais das unidades primárias de saúde de-
fissionais da atenção primária. senvolverão ações sistemáticas, individuais ou coletivas, visan-
§ 2º Os profissionais de saúde de referência da ges- do ao planejamento, à implementação e à avaliação de ações
tante garantirão sua vinculação, no último trimestre da de promoção, proteção e apoio ao aleitamento materno e à
gestação, ao estabelecimento em que será realizado o alimentação complementar saudável, de forma contínua.
parto, garantido o direito de opção da mulher. § 2º Os serviços de unidades de terapia intensiva neo-
§ 3º Os serviços de saúde onde o parto for realizado natal deverão dispor de banco de leite humano ou unidade
assegurarão às mulheres e aos seus filhos recém-nas- de coleta de leite humano.
cidos alta hospitalar responsável e contrarreferência na
atenção primária, bem como o acesso a outros serviços e a Art. 10. Os hospitais e demais estabelecimentos de
grupos de apoio à amamentação. atenção à saúde de gestantes, públicos e particulares, são
§ 4º Incumbe ao poder público proporcionar assis- obrigados a:
tência psicológica à gestante e à mãe, no período pré e I - manter registro das atividades desenvolvidas, através
pós-natal, inclusive como forma de prevenir ou minorar as de prontuários individuais, pelo prazo de dezoito anos;
consequências do estado puerperal. II - identificar o recém-nascido mediante o registro de
§ 5º A assistência referida no § 4º deste artigo deverá sua impressão plantar e digital e da impressão digital da
ser prestada também a gestantes e mães que manifes- mãe, sem prejuízo de outras formas normatizadas pela au-
tem interesse em entregar seus filhos para adoção, bem toridade administrativa competente;
como a gestantes e mães que se encontrem em situação III - proceder a exames visando ao diagnóstico e tera-
de privação de liberdade. pêutica de anormalidades no metabolismo do recém-nasci-
do, bem como prestar orientação aos pais;
§ 6º A gestante e a parturiente têm direito a 1 (um)
LEGISLAÇÃO ESPECÍFICA

IV - fornecer declaração de nascimento onde constem


acompanhante de sua preferência durante o período do
necessariamente as intercorrências do parto e do desenvol-
pré-natal, do trabalho de parto e do pós-parto imediato.
vimento do neonato;
§ 7º A gestante deverá receber orientação sobre alei- V - manter alojamento conjunto, possibilitando ao neo-
tamento materno, alimentação complementar saudável e nato a permanência junto à mãe.
crescimento e desenvolvimento infantil, bem como sobre VI - acompanhar a prática do processo de amamen-
formas de favorecer a criação de vínculos afetivos e de tação, prestando orientações quanto à técnica adequada,
estimular o desenvolvimento integral da criança. enquanto a mãe permanecer na unidade hospitalar, utili-
zando o corpo técnico já existente.
76 Ibid.

91
Art. 11. É assegurado acesso integral às linhas de § 3º A atenção odontológica à criança terá função
cuidado voltadas à saúde da criança e do adolescente, educativa protetiva e será prestada, inicialmente, antes
por intermédio do Sistema Único de Saúde, observado o de o bebê nascer, por meio de aconselhamento pré-natal,
princípio da equidade no acesso a ações e serviços para e, posteriormente, no sexto e no décimo segundo anos de
promoção, proteção e recuperação da saúde. vida, com orientações sobre saúde bucal.
§ 1º A criança e o adolescente com deficiência serão § 4º A criança com necessidade de cuidados odon-
atendidos, sem discriminação ou segregação, em suas tológicos especiais será atendida pelo Sistema Único de
necessidades gerais de saúde e específicas de habilitação Saúde.
e reabilitação. § 5º É obrigatória a aplicação a todas as crianças, nos
§ 2º Incumbe ao poder público fornecer gratuita- seus primeiros dezoito meses de vida, de protocolo ou
mente, àqueles que necessitarem, medicamentos, órte- outro instrumento construído com a finalidade de faci-
ses, próteses e outras tecnologias assistivas relativas litar a detecção, em consulta pediátrica de acompanha-
ao tratamento, habilitação ou reabilitação para crianças e mento da criança, de risco para o seu desenvolvimento
adolescentes, de acordo com as linhas de cuidado voltadas psíquico.
às suas necessidades específicas.
§ 3º Os profissionais que atuam no cuidado diário ou Capítulo II
frequente de crianças na primeira infância receberão for- Do Direito à Liberdade, ao Respeito e à Dignidade
mação específica e permanente para a detecção de sinais
de risco para o desenvolvimento psíquico, bem como Art. 15. A criança e o adolescente têm direito à liber-
para o acompanhamento que se fizer necessário. dade, ao respeito e à dignidade como pessoas huma-
nas em processo de desenvolvimento e como sujeitos de
Art. 12. Os estabelecimentos de atendimento à saú- direitos civis, humanos e sociais garantidos na Constitui-
de, inclusive as unidades neonatais, de terapia intensiva ção e nas leis.
e de cuidados intermediários, deverão proporcionar con- Entre os direitos fundamentais garantidos à criança
dições para a permanência em tempo integral de um e ao adolescente que são especificados e aprofundados
dos pais ou responsável, nos casos de internação de no ECA estão os direitos à liberdade, ao respeito e à dig-
criança ou adolescente. nidade.

Art. 13. Os casos de suspeita ou confirmação de cas- Art. 16. O direito à liberdade compreende os seguin-
tigo físico, de tratamento cruel ou degradante e de tes aspectos:
maus-tratos contra criança ou adolescente serão obriga- I - ir, vir e estar nos logradouros públicos e espaços
toriamente comunicados ao Conselho Tutelar da res- comunitários, ressalvadas as restrições legais;
pectiva localidade, sem prejuízo de outras providências II - opinião e expressão;
legais. III - crença e culto religioso;
§ 1º As gestantes ou mães que manifestem interesse IV - brincar, praticar esportes e divertir-se;
em entregar seus filhos para adoção serão obrigatoria- V - participar da vida familiar e comunitária, sem
mente encaminhadas, sem constrangimento, à Justiça da discriminação;
Infância e da Juventude. VI - participar da vida política, na forma da lei;
§ 2º Os serviços de saúde em suas diferentes portas VII - buscar refúgio, auxílio e orientação.
de entrada, os serviços de assistência social em seu com- O artigo 16 aborda diversas facetas do direito de
ponente especializado, o Centro de Referência Especiali- liberdade: locomoção, opinião e expressão, religiosa e
zado de Assistência Social (Creas) e os demais órgãos do política. Cria, ainda, duas facetes específicas deste direi-
Sistema de Garantia de Direitos da Criança e do Adoles- to: liberdade para brincar e divertir-se e liberdade para
cente deverão conferir máxima prioridade ao atendimen- buscar refúgio, auxílio e orientação, processos estes es-
to das crianças na faixa etária da primeira infância com senciais para o desenvolvimento do infante.
suspeita ou confirmação de violência de qualquer natu-
reza, formulando projeto terapêutico singular que inclua Art. 17. O direito ao respeito consiste na inviola-
intervenção em rede e, se necessário, acompanhamento bilidade da integridade física, psíquica e moral da
domiciliar. criança e do adolescente, abrangendo a preservação da
imagem, da identidade, da autonomia, dos valores, ideias
Art. 14. O Sistema Único de Saúde promoverá pro- e crenças, dos espaços e objetos pessoais.
gramas de assistência médica e odontológica para a
prevenção das enfermidades que ordinariamente afetam Art. 18. É dever de todos velar pela dignidade da
LEGISLAÇÃO ESPECÍFICA

a população infantil, e campanhas de educação sanitária criança e do adolescente, pondo-os a salvo de qual-
para pais, educadores e alunos. quer tratamento desumano, violento, aterrorizante,
§ 1º É obrigatória a vacinação das crianças nos casos vexatório ou constrangedor.
recomendados pelas autoridades sanitárias. Os direitos ao respeito e à dignidade abrangem a
§ 2º O Sistema Único de Saúde promoverá a aten- proteção da criança e do adolescente em todas facetas
ção à saúde bucal das crianças e das gestantes, de forma de sua integridade: física, psíquica e moral.
transversal, integral e intersetorial com as demais linhas
de cuidado direcionadas à mulher e à criança.

92
Art. 18-A. A criança e o adolescente têm o direito de Os críticos da “Lei da Palmada” apontam que ela adota
ser educados e cuidados sem o uso de castigo físico uma posição extrema e impõe uma indevida intervenção
ou de tratamento cruel ou degradante, como formas de do Estado nos ambientes familiares, retirando o poder
correção, disciplina, educação ou qualquer outro pre- disciplinar garantido aos pais na educação de seus filhos.
texto, pelos pais, pelos integrantes da família ampliada, Os defensores da “Lei da Palmada” utilizam estudos
pelos responsáveis, pelos agentes públicos executores de de psicólogos e educadores para argumentar que não é
medidas socioeducativas ou por qualquer pessoa encarre- necessário utilizar qualquer tipo de agressão física, mes-
gada de cuidar deles, tratá-los, educá-los ou protegê-los. mo a mais leve, para educar uma criança.
Parágrafo único. Para os fins desta Lei, considera-se:
I - castigo físico: ação de natureza disciplinar ou Capítulo III
punitiva aplicada com o uso da força física sobre a Do Direito à Convivência Familiar e Comunitária
criança ou o adolescente que resulte em:
a) sofrimento físico; ou Seção I
b) lesão; Disposições Gerais
II - tratamento cruel ou degradante: conduta ou for-
ma cruel de tratamento em relação à criança ou ao ado- Quando se aborda o direito à convivência familiar e
lescente que: comunitária no ECA confere-se destaque à distinção en-
a) humilhe; ou tre família natural e substituta e aos procedimentos que
b) ameace gravemente; ou caracterizam a inserção e a retirada da criança e do ado-
c) ridicularize. lescente destes ambientes.
Art. 18-B. Os pais, os integrantes da família ampliada, Art. 19. É direito da criança e do adolescente ser
os responsáveis, os agentes públicos executores de me- criado e educado no seio de sua família e, excepcio-
didas socioeducativas ou qualquer pessoa encarrega- nalmente, em família substituta, assegurada a convi-
da de cuidar de crianças e de adolescentes, tratá-los, vência familiar e comunitária, em ambiente que garanta
educá-los ou protegê-los que utilizarem castigo físico ou seu desenvolvimento integral.
tratamento cruel ou degradante como formas de correção, § 1o Toda criança ou adolescente que estiver inserido
disciplina, educação ou qualquer outro pretexto estarão em programa de acolhimento familiar ou institucio-
sujeitos, sem prejuízo de outras sanções cabíveis, às se- nal terá sua situação reavaliada, no máximo, a cada 3
guintes medidas, que serão aplicadas de acordo com a
(três) meses, devendo a autoridade judiciária competen-
gravidade do caso:
te, com base em relatório elaborado por equipe inter-
I - encaminhamento a programa oficial ou comuni-
profissional ou multidisciplinar, decidir de forma fun-
tário de proteção à família;
damentada pela possibilidade de reintegração familiar
II - encaminhamento a tratamento psicológico ou
ou pela colocação em família substituta, em quaisquer
psiquiátrico;
das modalidades previstas no art. 28 desta Lei.
III - encaminhamento a cursos ou programas de
§ 2o A permanência da criança e do adolescente em
orientação;
IV - obrigação de encaminhar a criança a tratamen- programa de acolhimento institucional não se prolonga-
to especializado; rá por mais de 18 (dezoito meses), salvo comprovada
V - advertência. necessidade que atenda ao seu superior interesse, devida-
Parágrafo único. As medidas previstas neste artigo se- mente fundamentada pela autoridade judiciária.
rão aplicadas pelo Conselho Tutelar, sem prejuízo de ou- § 3º A manutenção ou a reintegração de criança
tras providências legais. ou adolescente à sua família terá preferência em re-
lação a qualquer outra providência, caso em que será
Os artigos 18-A e 18-B foram incluídos no ECA pela esta incluída em serviços e programas de proteção, apoio
Lei nº 13.010, de 26 de junho de 2014, que estabelece o e promoção, nos termos do § 1o do art. 23, dos incisos I e
direito da criança e do adolescente de serem educados e IV do caput do art. 101 e dos incisos I a IV do caput do art.
cuidados sem o uso de castigos físicos ou de tratamento 129 desta Lei.
cruel ou degradante. Também ficou conhecida como “Lei § 4º Será garantida a convivência da criança e do
do Menino Bernardo”77 e “Lei da Palmada”. adolescente com a mãe ou o pai privado de liberdade,
Em que pesem as aparentes boas intenções da lei no por meio de visitas periódicas promovidas pelo responsá-
sentido de evitar situações extremas como a do menino vel ou, nas hipóteses de acolhimento institucional, pela
Bernardo, assassinado após incontáveis ameaças e agres- entidade responsável, independentemente de autorização
sões físicas por parte de seus responsáveis, seu conteúdo judicial.
§ 5o Será garantida a convivência integral da crian-
LEGISLAÇÃO ESPECÍFICA

é bastante criticado. Afinal, é claro que a lei coloca todo


e qualquer tipo de agressão física no mesmo patamar. ça com a mãe adolescente que estiver em acolhimen-
Considerado o teor da lei, mesmo uma palmada numa to institucional.
criança é proibida. § 6o A mãe adolescente será assistida por equipe es-
pecializada multidisciplinar.
77 O nome da lei é uma homenagem ao menino Bernardo Como se depreende do artigo 19, a família natural é
Boldrini, morto em abril de 2014, aos 11 anos, em Três Passos (RS). a regra e a família substituta é a exceção. A criança e o
Os acusados são o pai e a madrasta do menino, com ajuda de uma adolescente podem ser inseridos em programa de aco-
amiga e do irmão dela. Segundo as investigações, Bernardo procu-
lhimento familiar ou institucional, pelo limite temporal
rou ajuda para denunciar as ameaças que sofria.

93
de 18 meses, cujo caráter é o de permitir a sua retirada § 1o O apadrinhamento consiste em estabelecer e pro-
de potencial ou efetiva situação de risco. Durante este porcionar à criança e ao adolescente vínculos externos
programa, reavaliado a cada 3 meses, se verificará se é à instituição para fins de convivência familiar e co-
possível a reinserção no ambiente da família natural (o munitária e colaboração com o seu desenvolvimento
que é preferencial) ou se é o caso de colocação definitiva nos aspectos social, moral, físico, cognitivo, educacional e
em família substituta. financeiro.
§ 2o (VETADO).
Art. 19-A. A gestante ou mãe que manifeste interes- § 3o Pessoas jurídicas podem apadrinhar criança ou
se em entregar seu filho para adoção, antes ou logo adolescente a fim de colaborar para o seu desenvolvimen-
após o nascimento, será encaminhada à Justiça da In- to.
fância e da Juventude. § 4o O perfil da criança ou do adolescente a ser apa-
§ 1o A gestante ou mãe será ouvida pela equipe in- drinhado será definido no âmbito de cada programa
terprofissional da Justiça da Infância e da Juventude, que de apadrinhamento, com prioridade para crianças ou
apresentará relatório à autoridade judiciária, conside- adolescentes com remota possibilidade de reinserção
rando inclusive os eventuais efeitos do estado gestacional familiar ou colocação em família adotiva.
e puerperal. § 5o Os programas ou serviços de apadrinhamento
§ 2o De posse do relatório, a autoridade judiciária apoiados pela Justiça da Infância e da Juventude poderão
poderá determinar o encaminhamento da gestan- ser executados por órgãos públicos ou por organiza-
te ou mãe, mediante sua expressa concordância, à rede ções da sociedade civil.
pública de saúde e assistência social para atendimento § 6o Se ocorrer violação das regras de apadrinha-
especializado. mento, os responsáveis pelo programa e pelos serviços de
§ 3o A busca à família extensa, conforme definida acolhimento deverão imediatamente notificar a autori-
nos termos do parágrafo único do art. 25 desta Lei, respei- dade judiciária competente.
tará o prazo máximo de 90 (noventa) dias, prorrogá- Os programas de apadrinhamento visam permitir a
vel por igual período. convivência comunitária da criança e do adolescente que
§ 4o Na hipótese de não haver a indicação do ge- estão no sistema de adoção, especialmente com relação
nitor e de não existir outro representante da família àqueles que dificilmente sairão dele.
extensa apto a receber a guarda, a autoridade judiciária
competente deverá decretar a extinção do poder fami- Art. 20. Os filhos, havidos ou não da relação do ca-
liar e determinar a colocação da criança sob a guarda samento, ou por adoção, terão os mesmos direitos e
provisória de quem estiver habilitado a adotá-la ou qualificações, proibidas quaisquer designações dis-
de entidade que desenvolva programa de acolhimento criminatórias relativas à filiação.
familiar ou institucional. O artigo 20 destaca a igualdade entre todos os filhos,
§ 5o Após o nascimento da criança, a vontade da mãe sejam eles havidos dentro ou fora do casamento, sejam
ou de ambos os genitores, se houver pai registral ou pai eles inseridos em família natural ou substituta.
indicado, deve ser manifestada na audiência a que se
refere o § 1o do art. 166 desta Lei, garantido o sigilo sobre A disciplina sobre a perda e suspensão do poder de
a entrega. família no ECA se encontra em dois blocos, o primeiro
§ 6o (VETADO). do artigo 21 ao 24, que aborda questões materiais, e o
§ 7o Os detentores da guarda possuem o prazo de segundo do artigo 155 a 163, que foca em questões pro-
15 (quinze) dias para propor a ação de adoção, conta- cedimentais:
do do dia seguinte à data do término do estágio de con-
vivência. Art. 21. O poder familiar será exercido, em igualda-
§ 8o Na hipótese de desistência pelos genitores - ma- de de condições, pelo pai e pela mãe, na forma do que
nifestada em audiência ou perante a equipe interprofissio- dispuser a legislação civil, assegurado a qualquer deles o
nal - da entrega da criança após o nascimento, a crian- direito de, em caso de discordância, recorrer à autoridade
ça será mantida com os genitores, e será determinado pela judiciária competente para a solução da divergência.
Justiça da Infância e da Juventude o acompanhamento
familiar pelo prazo de 180 (cento e oitenta) dias. Art. 22. Aos pais incumbe o dever de sustento, guar-
§ 9o É garantido à mãe o direito ao sigilo sobre o da e educação dos filhos menores, cabendo-lhes ainda,
nascimento, respeitado o disposto no art. 48 desta Lei. no interesse destes, a obrigação de cumprir e fazer cumprir
§ 10. (VETADO). as determinações judiciais.
O artigo 19-A trata do procedimento aplicável nos ca- Parágrafo único. A mãe e o pai, ou os responsáveis,
sos em que a família biológica pretenda entregar recém- têm direitos iguais e deveres e responsabilidades com-
LEGISLAÇÃO ESPECÍFICA

-nascido para adoção, assegurando-se o acompanha- partilhados no cuidado e na educação da criança, deven-
mento por equipe multidisciplinar e também a tomada do ser resguardado o direito de transmissão familiar de
de providências de busca de pessoa da família extensa suas crenças e culturas, assegurados os direitos da criança
apta a assumir o encargo. estabelecidos nesta Lei.

Art. 19-B. A criança e o adolescente em programa de Art. 23. A falta ou a carência de recursos materiais
acolhimento institucional ou familiar poderão participar não constitui motivo suficiente para a perda ou a sus-
de programa de apadrinhamento. pensão do poder familiar.

94
§ 1º Não existindo outro motivo que por si só auto- Artigo 1.634, CC. Compete aos pais, quanto à pessoa
rize a decretação da medida, a criança ou o adolescen- dos filhos menores:
te será mantido em sua família de origem, a qual deverá I – dirigir-lhes a criação e educação;
obrigatoriamente ser incluída em serviços e programas II – tê-los em sua companhia e guarda;
oficiais de proteção, apoio e promoção. III – conceder-lhes ou negar-lhes consentimento para
§ 2º A condenação criminal do pai ou da mãe não im- casarem;
plicará a destituição do poder familiar, exceto na hipótese IV – nomear-lhes tutor por testamento ou documento
de condenação por crime doloso sujeito à pena de reclu- autêntico, se o outro dos pais não lhe sobreviver, ou o so-
são contra outrem igualmente titular do mesmo poder fa- brevivo não puder exercer o poder familiar;
miliar ou contra filho, filha ou outro descendente. V – representá-los, até aos dezesseis anos, nos atos da
vida civil, e assisti-los, após essa idade, nos atos em que
Art. 24. A perda e a suspensão do poder familiar serão decre- forem partes, suprindo-lhes o consentimento;
tadas judicialmente, em procedimento contraditório, nos casos VI – reclamá-los de quem ilegalmente os detenha;
previstos na legislação civil, bem como na hipótese de descumpri- VII – exigir que lhes prestem obediência, respeito e os
mento injustificado dos deveres e obrigações a que alude o art. 22. serviços próprios de sua idade e condição.

O instituto do poder familiar surgiu no direito romano e Em relação às suas características, o poder familiar é
era conhecido naquela época como pátrio poder, pois o pai indisponível e decorre da paternidade natural ou legal,
exercia mais poderes sobre os filhos do que a mãe, o que por isso, não há a possibilidade de seu titular o transferir
vinha a representar um poder absoluto por parte do geni- a terceiros por iniciativa própria, tampouco existindo a
tor, inclusive sobre a vida e a morte dos próprios filhos78. viabilidade de ocorrer a sua prescrição pelo desuso. O
O Código Civil de 1916 atribuiu o poder familiar ao pai, poder familiar é indelegável, sendo, em regra, irrenunciá-
que era considerado o chefe da sociedade conjugal, e a vel e sempre intransferível. Logo, não sendo possível ao
mãe possuía um papel secundário, conforme apontava o titular do poder familiar abrir mão de seu dever, a renún-
artigo 380 do Código Civil de 1916 que dizia que “durante cia é inviável, existindo apenas uma exceção, qual seja, a
o casamento compete o pátrio poder aos pais, exercendo- decorrente da adoção.
-o o marido com a colaboração da mulher [...]”.
Existe apenas uma exceção, que é o caso do pedido
De acordo com Santos Neto79: “[...] o exercício da auto-
de colocação do menor em família substituta, disponi-
ridade parental pela mãe era admitido apenas em caráter
bilizando o filho para a adoção, caso em que os direitos
excepcional. Ao homem era dada, em condições normais,
e deveres decorrentes do poder familiar serão exercidos
a titularidade exclusiva do direito em pauta. Sua vontade
por novos titulares, ou seja, pelos pais adotivos.
prevalecia e contra ela não havia remédio previsto, salvo, é
A esse respeito, os direitos e deveres dos pais dispos-
claro, no caso de comportamento abusivo e contrário aos
tos nos artigos 1.630 a 1.638 do Código Civil regulam,
interesses dos menores”.
O Código Civil de 2002, por seu turno, seguindo a toa- dentre os deveres e poderes decorrentes do poder fa-
da da Constituição Federal de 1988, trouxe significativas miliar, também os de ordem pessoal, ou seja, o cuida-
modificações ao instituto em análise, surgindo então o do existencial do menor, a educação, a correição, e os
chamado poder familiar, onde ambos os pais exercem de de ordem material, que envolvem a administração dos
forma igualitária o poder sobre os filhos menores, equili- bens dos filhos. Os principais atributos do poder familiar
brando dessa forma a relação familiar. são a guarda, a criação e a educação, que se refletem
O artigo 1.630 do atual Código Civil, sem definir o po- nos deveres dos pais para com os filhos; tanto que, não
der familiar, dispõe que “os filhos estão sujeitos ao poder cumprindo algum desses atributos, o detentor do poder
familiar enquanto menores”, ou seja, enquanto não com- poderá sofrer sanções cíveis e até criminais.
pletarem dezoito anos ou não alcançarem a maioridade O poder familiar, conforme disposição do próprio or-
civil por meio de uma das formas previstas no artigo 5º, denamento civil, não é um instituto irrevogável e pode
parágrafo único e seus incisos, do mesmo diploma legal. ser extinto, suspenso ou destituído a qualquer tempo.
No mesmo sentido, o citado artigo 21, ECA. Basicamente, as formas de extinção se aplicam quando
O poder familiar, conhecido também como autoridade o exercício do poder familiar não é mais necessário; ao
parental, é um conjunto de direitos e deveres que são atri- passo que as regras de perda e suspensão constituem
buídos aos pais para que esses administrem de forma legal casos de privação do exercício do poder familiar pelo
a pessoa dos filhos e também de seus bens. descumprimento de seus deveres.
“O poder familiar pode ser definido como um conjunto Sendo assim, o Estado poderá interferir na relação
de direitos e obrigações, quanto às pessoas e bens do filho familiar, com o objetivo de resguardar os interesses do
menor não emancipado, exercido, em igualdade de condi- menor, sendo que a lei disciplina os casos em que o titu-
ções, por ambos os pais, para que possam desempenhar os lar do poder familiar ficará privado de exercê-lo, seja de
LEGISLAÇÃO ESPECÍFICA

encargos que a norma jurídica lhes impõe, tendo em vista forma temporária ou até mesmo definitiva.
o interesse e a proteção do filho”80.
Artigo 1.637, CC. Se o pai, ou a mãe, abusar de sua
78 AKEL, Ana Carolina Silveira. Guarda Compartilhada: um autoridade, faltando aos deveres a eles inerentes ou arrui-
Avanço para a Família. 2. ed. São Paulo: Atlas, 2010. nando os bens dos filhos, cabe ao juiz, requerendo algum
79 SANTOS NETO, José Antônio de Paula. Do Pátrio Poder. parente, ou o Ministério Público, adotar a medida que lhe
São Paulo: Revista dos Tribunais, 1994.
pareça reclamada pela segurança do menor e seus have-
80 DINIZ, Maria Helena. Curso de Direito Civil Brasileiro:
res, até suspendendo o poder familiar, quando convenha.
Direito de Família. 22. ed. São Paulo: Saraiva, 2007. v. 5.

95
Parágrafo único. Suspende-se igualmente o exercício Artigo 1.635, CC. Extingue-se o poder familiar: I – pela
do poder familiar ao pai ou à mãe condenados por sen- morte dos pais ou do filho; II – pela emancipação, nos termos
tença irrecorrível, em virtude de crime cuja pena exceda a do artigo 5º, parágrafo único; III – pela maioridade; IV – pela
dois anos de prisão. adoção; V – por decisão judicial, na forma do artigo 1.638.

Nestes casos, a suspensão não será definitiva, é ape- A extinção do poder familiar é mais complexa, pois
nas uma sanção imposta pelo Poder Judiciário visando nesta situação os pais, extintos do poder, não poderão
preservar os interesses dos filhos, assim, diante da com- requerer a reintegração do poder familiar se houve inter-
provação de que os problemas que levaram à suspensão ferência deles para sua extinção. Na maioria dos casos,
desapareceram, o poder familiar poderá retornar aos é possível identificar facilmente a existência da extinção
genitores. do poder familiar, por se tratarem de hipóteses objetivas.
Assim sendo, os pais podem ser suspensos de exer-
cer os direitos e deveres decorrentes do poder familiar Seção II
quando ficar evidenciado perante a autoridade compe- Da Família Natural
tente o abuso. Como visto, existe, também, a probabili-
dade de se decretar a suspensão do poder familiar, caso Dos artigos 25 a 27 o ECA aborda a família natural,
nos seguintes termos:
um dos pais seja condenado por crime cuja pena exceda
a dois anos de prisão.
Art. 25. Entende-se por família natural a comunidade
Há, ainda, as hipóteses de perda ou destituição do
formada pelos pais ou qualquer deles e seus descen-
poder familiar, que é a sanção mais grave imposta aos
dentes.
pais que faltarem com os deveres em relação aos filhos: Parágrafo único. Entende-se por família extensa ou
ampliada aquela que se estende para além da unida-
Artigo 1.638, CC. Perderá por ato judicial o poder fa- de pais e filhos ou da unidade do casal, formada por
miliar o pai ou a mãe que: parentes próximos com os quais a criança ou adolescen-
I – castigar imoderadamente o filho; te convive e mantém vínculos de afinidade e afetividade.
II – deixar o filho em abandono; A família natural é composta por pais e filhos que for-
III – praticar atos contrários à moral e aos bons cos- mam vínculo entre si desde o nascimento, por questão
tumes; biológica.
IV – incidir, reiteradamente, nas faltas previstas no ar- O conceito de família pode ser visto de uma maneira
tigo antecedente. mais ampla, o que se denomina família extensa ou am-
pliada. Por exemplo, avós e tios que sejam muito próxi-
Nessa linha de pensamento, os artigos 24 e 22 do mos e participem diretamente do convívio familiar, for-
Estatuto da Criança e do Adolescente, citados anterior- mando para com a criança e o adolescente vínculos de
mente, além de preverem a suspensão e destituição do afinidade e afetividade.
poder familiar, trazem também os motivos que poderão
acarretá-las. Art. 26. Os filhos havidos fora do casamento pode-
São bastante frequentes nos casos de pais que per- rão ser reconhecidos pelos pais, conjunta ou separada-
dem o poder familiar os atos de violência e espanca- mente, no próprio termo de nascimento, por testamen-
mento; assim como o de abandono, no qual os meno- to, mediante escritura ou outro documento público,
res, ao se verem abandonados, começam a relacionar-se qualquer que seja a origem da filiação.
com pessoas delinquentes e usuárias de drogas, que Parágrafo único. O reconhecimento pode preceder o
não vão colaborar em nada com seu desenvolvimento e nascimento do filho ou suceder-lhe ao falecimento, se
crescimento. Nessas situações, os detentores do poder deixar descendentes.
Como o artigo 20 estabelece a igualdade entre os fi-
familiar podem sofrer a perda do poder familiar.
lhos havidos dentro ou fora do casamento, sentido em
Cabe aqui consignar que, no caso da perda do poder
que se compreende que tanto os filhos inseridos no ma-
familiar, se no decorrer do tempo o menor não vier a
trimônio quanto os que não o estão fazem parte da famí-
ser adotado por outra família e as causas que levaram
lia natural, o artigo 26 tece detalhes sobre a possibilidade
a perda do poder desaparecerem, os genitores poderão de reconhecimento do vínculo de filiação, que pode se
requerer judicialmente a reintegração do poder familiar, dar antes mesmo do nascimento do filho ou extrapolar a
desde que comprovem realmente que o motivo que os sua vida, devendo ser feito em documento público.
levou a perder esse poder já não existe mais.
Por seu turno, pelo que se verifica na análise da legis- Art. 27. O reconhecimento do estado de filiação é
lação, quando o legislador estabeleceu as hipóteses de
LEGISLAÇÃO ESPECÍFICA

direito personalíssimo, indisponível e imprescritível,


extinção do poder familiar, em regra, o fez por perceber podendo ser exercitado contra os pais ou seus herdeiros,
que a pessoa que a ele se encontrava sujeita adquiriu sem qualquer restrição, observado o segredo de Justiça.
maturidade o suficiente para guiar a sua vida, não ha- O direito à filiação é personalíssimo, indisponível e
vendo razão para que permaneça tal vínculo. Há casos, imprescritível. Mesmo que um filho passe a vida toda ou
entretanto, que a violação aos direitos inerentes ao po- boa parte de sua vida sem buscar o seu reconhecimento,
der familiar tornou irreversível o seu restabelecimento, ele não se perde. Logo, a ação de investigação de pater-
como ocorre na adoção. nidade é imprescritível, pois também o é o vínculo que
ela reconhece em caso de procedência.

96
Seção III Art. 30. A colocação em família substituta não admiti-
Da Família Substituta rá transferência da criança ou adolescente a terceiros
ou a entidades governamentais ou não-governamen-
Subseção I tais, sem autorização judicial.
Disposições Gerais
Art. 31. A colocação em família substituta estrangei-
Dos artigos 28 a 32 aborda-se a família substituta, ra constitui medida excepcional, somente admissível na
nos seguintes termos: modalidade de adoção.

Art. 28. A colocação em família substituta far-se- Art. 32. Ao assumir a guarda ou a tutela, o responsá-
-á mediante guarda, tutela ou adoção, independente- vel prestará compromisso de bem e fielmente desempe-
mente da situação jurídica da criança ou adolescente, nos nhar o encargo, mediante termo nos autos.
termos desta Lei.
§ 1o Sempre que possível, a criança ou o adolescente Existem três formas de colocação em família substi-
será previamente ouvido por equipe interprofissional, tuta: guarda, tutela e adoção. Neste sentido, a criança é
respeitado seu estágio de desenvolvimento e grau de com- retirada da esfera do poder familiar de ambos os pais (o
preensão sobre as implicações da medida, e terá sua opi- que pode acontecer na tutela e na adoção), ou então per-
nião devidamente considerada. manece vinculado ao poder familiar de ambos genitores
§ 2o Tratando-se de maior de 12 (doze) anos de enquanto apenas um ou um terceiro exerce a guarda (o
idade, será necessário seu consentimento, colhido em que ocorre apenas na guarda). Trata-se de situação ex-
audiência. cepcional, eis que em regra a criança deve permanecer
§ 3o Na apreciação do pedido levar-se-á em conta na família natural, vinculada ao poder familiar atribuído a
o grau de parentesco e a relação de afinidade ou de ambos os pais.
afetividade, a fim de evitar ou minorar as consequências Neste tipo de circunstância, deve-se buscar sempre
decorrentes da medida. ouvir a criança ou o adolescente. Caso já possua 12 anos
§ 4o Os grupos de irmãos serão colocados sob completos, a oitiva é obrigatória. Trata-se de respeito à
adoção, tutela ou guarda da mesma família subs- autonomia da criança e do adolescente.
tituta, ressalvada a comprovada existência de risco de Os irmãos devem permanecer unidos em qualquer cir-
abuso ou outra situação que justifique plenamente a cunstância, sendo a separação de irmãos medida excep-
excepcionalidade de solução diversa, procurando-se, cional. Por exemplo, se um casal estiver disposto a adotar
em qualquer caso, evitar o rompimento definitivo dos 4 filhos e existirem 4 irmãos, será dada prioridade a ele,
vínculos fraternais. passando na frente dos demais candidatos à adoção.
§ 5o A colocação da criança ou adolescente em fa-
mília substituta será precedida de sua preparação gra- Subseção II
dativa e acompanhamento posterior, realizados pela Da Guarda
equipe interprofissional a serviço da Justiça da Infância e
da Juventude, preferencialmente com o apoio dos técnicos Art. 33. A guarda obriga a prestação de assistência
responsáveis pela execução da política municipal de ga- material, moral e educacional à criança ou adolescen-
rantia do direito à convivência familiar. te, conferindo a seu detentor o direito de opor-se a tercei-
§ 6o Em se tratando de criança ou adolescente in- ros, inclusive aos pais.
dígena ou proveniente de comunidade remanescente de § 1º A guarda destina-se a regularizar a posse de
quilombo, é ainda obrigatório: fato, podendo ser deferida, liminar ou incidentalmente,
I - que sejam consideradas e respeitadas sua identida- nos procedimentos de tutela e adoção, exceto no de adoção
de social e cultural, os seus costumes e tradições, bem por estrangeiros.
como suas instituições, desde que não sejam incompatíveis § 2º Excepcionalmente, deferir-se-á a guarda, fora dos
com os direitos fundamentais reconhecidos por esta Lei e casos de tutela e adoção, para atender a situações pecu-
pela Constituição Federal; liares ou suprir a falta eventual dos pais ou responsá-
II - que a colocação familiar ocorra prioritariamente vel, podendo ser deferido o direito de representação para a
no seio de sua comunidade ou junto a membros da prática de atos determinados.
mesma etnia; § 3º A guarda confere à criança ou adolescente a con-
III - a intervenção e oitiva de representantes do órgão dição de dependente, para todos os fins e efeitos de direi-
federal responsável pela política indigenista, no caso de to, inclusive previdenciários.
crianças e adolescentes indígenas, e de antropólogos, pe- § 4o Salvo expressa e fundamentada determinação em
LEGISLAÇÃO ESPECÍFICA

rante a equipe interprofissional ou multidisciplinar que irá contrário, da autoridade judiciária competente, ou quan-
acompanhar o caso. do a medida for aplicada em preparação para adoção, o
deferimento da guarda de criança ou adolescente a ter-
Art. 29. Não se deferirá colocação em família substi- ceiros não impede o exercício do direito de visitas pelos
tuta a pessoa que revele, por qualquer modo, incompa- pais, assim como o dever de prestar alimentos, que serão
tibilidade com a natureza da medida ou não ofereça objeto de regulamentação específica, a pedido do interes-
ambiente familiar adequado. sado ou do Ministério Público.

97
Art. 34. O poder público estimulará, por meio de as- pais forem culpados pela separação e se a hipótese for
sistência jurídica, incentivos fiscais e subsídios, o acolhi- de ruptura da vida em comum ou de separação por mo-
mento, sob a forma de guarda, de criança ou adolescente tivo de doença mental. A regra amolda-se ao princípio
afastado do convívio familiar. do melhor interesse da criança, identificado como direito
§ 1o A inclusão da criança ou adolescente em pro- fundamental na Constituição Federal (art. 5º, §2º), em ra-
gramas de acolhimento familiar terá preferência a seu zão da ratificação pela Convenção Internacional sobre os
acolhimento institucional, observado, em qualquer caso, o Direitos da Criança – ONU/89”83.
caráter temporário e excepcional da medida, nos ter- O juiz antes de decidir o mérito de uma ação de guar-
mos desta Lei. da, separação ou divórcio, tem que determinar a guarda
§ 2o Na hipótese do § 1o deste artigo a pessoa ou ca- provisória do menor a um dos pais, o qual não se trata
sal cadastrado no programa de acolhimento familiar po- de um modelo de guarda, mas de definir uma situação
derá receber a criança ou adolescente mediante guarda, momentânea em que a prole se encontra. Somente com
observado o disposto nos arts. 28 a 33 desta Lei. o julgamento do mérito será estabelecida a guarda de-
§ 3º A União apoiará a implementação de serviços finitiva, que deverá adotar um dos modelos de guarda
de acolhimento em família acolhedora como política permitida no ordenamento jurídico brasileiro.
pública, os quais deverão dispor de equipe que organize o A guarda definitiva expressa o modelo de guarda
acolhimento temporário de crianças e de adolescentes em adotado pelos pais. Porém, mesmo tratando-se de guar-
residências de famílias selecionadas, capacitadas e acom- da definitiva, tal modelo poderá ser alterado a qualquer
panhadas que não estejam no cadastro de adoção. tempo, pois o que regula o instituto da guarda é o me-
§ 4º Poderão ser utilizados recursos federais, esta- lhor interesse do menor e, não sendo isso possível, a
duais, distritais e municipais para a manutenção dos guarda é passível de modificação.
serviços de acolhimento em família acolhedora, facultan- a) Guarda Comum ou Guarda Originária
do-se o repasse de recursos para a própria família acolhe- A guarda comum ou guarda originária não é uma
dora. guarda judicial, existindo quando os genitores possuem
vínculo matrimonial ou vivem em união estável e moram
Art. 35. A guarda poderá ser revogada a qualquer juntos com seus filhos, situação na qual exercem plena-
tempo, mediante ato judicial fundamentado, ouvido o Mi- mente e simultaneamente todos os poderes inerentes
nistério Público. do poder familiar e, consequentemente, a guarda. Não
existe a figura do guardião e nem arbitramento judicial
sobre a questão. Ambos pais exercem juntos a guarda
Na definição de Santos Neto81 a guarda trata-se de
em plenitude.
um “direito consistente na posse de menor oponível a
b) Guarda Unilateral ou Guarda Única
terceiros e que acarreta deveres de vigilância em relação
Observando-se sempre o princípio do melhor interes-
a este”.
se do menor, a guarda excepcionalmente poderá ser atri-
No entender de Ishida82, a guarda é vinculada ao po-
buída de forma unilateral a uma terceira pessoa quando
der familiar, “todavia, pode ocorrer a separação dos dois
os genitores não estiverem em condições de exercê-la,
institutos, por exemplo, com a separação judicial do ma- pois, conforme determina o artigo 1.583, §1º, do Código
rido e da mulher, onde o poder familiar continua per- Civil, “compreende-se por guarda unilateral a atribuída a
tencendo aos dois, no entanto um só poderá ficar com um só dos genitores ou a alguém que o substitua”. Neste
a guarda da prole”. ou seja, tanto o pai como a mãe são viés, dispõe o artigo 1.584, §5º, do mesmo diploma legal:
detentores do poder familiar mesmo após a separação, “Se o juiz verificar que o filho não deve permanecer sob
mas nos tipos de guarda comum, apenas um terá a guar- a guarda do pai ou da mãe, deferirá a guarda à pessoa
da do filho. que revele compatibilidade com a natureza da medida,
Logo, a dissolução do vínculo conjugal não exclui o considerados, de preferência, o grau de parentesco e as
poder familiar, mas pode excluir a guarda de um dos pais, relações de afinidade e afetividade”. Vale ressaltar que
reservando-o apenas o direito de visitas, dependendo da a guarda unilateral também é possível quando nenhum
modalidade de guarda adotada. Com a dissolução da dos pais tem condições de exercê-la, por exemplo, atri-
união conjugal, hoje se estabeleceu que a prole poderá buindo a guarda aos avós ou aos tios.
ficar com o genitor que tiver melhor condições de assistir Usualmente, nesse contexto, podemos constatar a
o filho. existência da guarda unilateral, que é atribuída somente
“Mesmo que a mãe seja considerada culpada pela a um dos genitores, e da guarda compartilhada, que é
separação, pode o juiz deferir-lhe a guarda dos filhos atribuída a ambos, sendo que a previsão das duas moda-
menores, se estiver comprovado que o pai, por exemplo, lidades de guarda encontra-se no artigo 1.583, da Lei n.
é alcoólatra e não tem condições de cuidar bem deles. 11.698/2008, que constitui que “a guarda será unilateral
Não se indaga, portanto, quem deu causa à separação ou compartilhada”.
LEGISLAÇÃO ESPECÍFICA

e quem é o cônjuge inocente, mas qual deles revela me- A Lei alterou a redação do artigo 1.583 e passou a
lhores condições para exercer a guarda dos filhos me- estabelecer nos incisos do §2º do dispositivo algumas
nores, cujos interesses foram colocados em primeiro das situações que deverão ser consideradas pelo magis-
plano. A solução será, portanto, a mesma se ambos os trado ao atribuir a guarda a um dos genitores: afeto nas
relações com o genitor e com o grupo familiar; saúde e
81 SANTOS NETO, José Antônio de Paula. Do Pátrio Poder. segurança; e educação.
São Paulo: Revista dos Tribunais, 1994.
82 ISHIDA, Valter Kenji. Estatuto da Criança e do Adoles- 83 GONÇALVES, Carlos Roberto. Direito Civil Brasileiro: Di-
cente: Doutrina e Jurisprudência. 9. ed. São Paulo: Atlas, 2008. reito de Família. 6. ed. São Paulo: Saraiva, 2009. v. 6.

98
Na guarda unilateral atribuída a apenas um dos pais, Segundo Akel85, “a guarda compartilhada surgiu da
apesar de um dos genitores não ser o guardião, conti- necessidade de se encontrar uma maneira que fosse ca-
nuam ambos a exercerem a guarda jurídica. A diferença paz de fazer com que os pais, que não mais convivem,
é que, em virtude da guarda, o genitor guardião tem o e seus filhos mantivessem os vínculos afetivos latentes,
poder de decisão, enquanto o genitor não guardião tem mesmo após o rompimento”.
o poder de fiscalização, podendo contestar a decisão do Com esse propósito, a recente Lei nº 11.698/2008
genitor guardião e até mesmo recorrer à justiça, caso instituiu expressamente no ordenamento jurídico o ins-
entenda que a decisão tomada não seja a melhor para tituto da guarda compartilhada. Embora tenha sido san-
a prole, conforme prevê o artigo 1.583, §3º, do Código cionada em 13 de junho de 2008 e publicada no Diário
Civil: “a guarda unilateral obriga o pai ou a mãe que não Oficial da União em 16 de junho do mesmo ano, por
a detenha a supervisionar os interesses dos filhos”. força da vacatio legis instituída no artigo 2º, a lei so-
c) Guarda Alternada ou Guarda Partilhada mente entrou em vigor no país 60 (sessenta) dias após a
Nesse modelo de guarda, cada um dos genitores terá sua publicação, ou seja, em 16 de agosto de 2008 (vide
a possibilidade de ter sobre sua guarda o menor ou ado- anexo).
lescente de forma alternada e exclusiva, ou seja, o casal A nova lei trás em seu bojo o conceito de guarda
determinará o período em que o menor ficará com o pai compartilhada nos seguintes termos: “compreende-se
ou com a mãe, existindo dessa forma sempre uma alter- por [...] guarda compartilhada a responsabilização con-
nância na guarda jurídica do menor. O período em que a junta e o exercício de direitos e deveres do pai e da mãe
guarda ficará com o pai ou com a mãe na guarda alter- que não vivam sob o mesmo teto, concernentes ao po-
nada, poderá ser de um dia, uma semana, uma parte da der familiar dos filhos comuns”.
semana, um mês, um ano, ou até mais, dependendo do Assim, de acordo com o novo diploma legal, pode-se
acordo dos genitores, sendo que, ao término desse pe- verificar que na guarda compartilhada os pais terão os
ríodo, os papéis se invertem. Os direitos e deveres deste mesmos direitos e deveres com relação ao filho, ou seja,
modelo de guarda ficarão sempre com o cônjuge que as tarefas serão divididas de forma igualitária, não so-
estiver com a guarda do menor, cabendo ao outro os di- brecarregando somente um dos genitores.
reitos inerentes do não guardião, ou seja, o de visita e o
de fiscalização84. Subseção III
d) Guarda Dividida Da Tutela
Na guarda dividida, são os próprios pais que contes-
tam e procuram novos meios de garantir uma maior par- Art. 36. A tutela será deferida, nos termos da lei civil, a
ticipação na vida da prole, pois neste modelo o menor pessoa de até 18 (dezoito) anos incompletos.
vive em um lar fixo e determinado e recebe periodica- Parágrafo único. O deferimento da tutela pressupõe
mente a visita do pai ou da mãe que não tem a guarda. a prévia decretação da perda ou suspensão do poder
Na guarda dividida o filho tem um lar fixo e recebe familiar e implica necessariamente o dever de guar-
nele a visita de ambos os genitores em tempos diferen- da.
tes, sendo que a guarda é exercida por aquele que esti-
ver com a criança, o que é diferente da guarda unilateral, Art. 37. O tutor nomeado por testamento ou qual-
pois nela a guarda é de um dos genitores e o infante quer documento autêntico, conforme previsto no pará-
vai, em dias determinados, receber a visita do outro não grafo único do art. 1.729 da Lei no 10.406, de 10 de janei-
guardião. ro de 2002 - Código Civil, deverá, no prazo de 30 (trinta)
e) Guarda por Aninhamento ou Nidação dias após a abertura da sucessão, ingressar com pedido
A guarda por aninhamento, conhecida também como destinado ao controle judicial do ato, observando o pro-
guarda por nidação, ocorre quando a prole possui um lar cedimento previsto nos arts. 165 a 170 desta Lei.
fixo e os pais se revezam, mudando-se para a casa do(s) Parágrafo único. Na apreciação do pedido, serão ob-
filho(s) em períodos alternados de tempo, para conviver servados os requisitos previstos nos arts. 28 e 29 desta
e atender as suas necessidades. Basicamente, os pais se Lei, somente sendo deferida a tutela à pessoa indicada na
revezam na residência do filho. disposição de última vontade, se restar comprovado que
f) Guarda Compartilhada a medida é vantajosa ao tutelando e que não existe outra
Considerando a evolução da família, especialmente a pessoa em melhores condições de assumi-la.
da mulher na sociedade, bem como o grande número de
separações ocorridas nos últimos anos, o ordenamento Art. 38. Aplica-se à destituição da tutela o disposto no
jurídico busca com o instituto da guarda compartilhada art. 24.
evitar prejuízos ainda maiores aos filhos de casal separa-
do, que, além de não terem o convívio diário com um dos A tutela é forma de colocação de criança e adoles-
LEGISLAÇÃO ESPECÍFICA

genitores, têm que vivenciar os problemas conjugais de cente em família substituta. Pressupõe, ao contrário da
seus pais e ainda se tornarem, em muitos casos, vítimas guarda, a prévia destituição ou suspensão do poder fa-
da Síndrome da Alienação Parental. miliar dos pais (família natural). Visa essencialmente a
suprir carência de representação legal, assumindo o tu-
tor tal munus na ausência dos genitores. Na hipótese de
84 COSTA, Luiz Jorge Valente Pontes. Guarda Conjunta: em os pais serem falecidos, tiverem sido destituídos ou sus-
busca do maior interesse do menor. Disponível em: <http://jus.uol.
com.br/revista/texto/13965/guarda-conjunta-em-busca-do-maior- 85 AKEL, Ana Carolina Silveira. Guarda Compartilhada: um
-interesse-do-menor/2>. Acesso em: 16 out. 2010. Avanço para a Família. 2. ed. São Paulo: Atlas, 2010.

99
pensos do poder familiar, ou houverem aderido expres- § 5o Nos casos do § 4o deste artigo, desde que de-
samente ao pedido de colocação em família substituta, monstrado efetivo benefício ao adotando, será assegu-
este poderá ser formulado diretamente em cartório, em rada a guarda compartilhada, conforme previsto no art.
petição assinada pelos próprios requerentes, dispensada 1.584 da Lei no10.406, de 10 de janeiro de 2002 - Código
a assistência por advogado (art. 166, ECA). Em outras cir- Civil.
cunstâncias, deve passar pelo crivo do Judiciário. § 6o A adoção poderá ser deferida ao adotante que,
após inequívoca manifestação de vontade, vier a falecer
Subseção IV no curso do procedimento, antes de prolatada a sentença.
Da Adoção
Art. 43. A adoção será deferida quando apresentar reais
A disciplina do ECA a respeito da adoção também se
vantagens para o adotando e fundar-se em motivos legí-
divide em dois blocos, um voltado a aspectos materiais,
timos.
do artigo 39 ao 52-D, e outro voltado a aspectos procedi-
mentais, notadamente no que se refere à habilitação para
a adoção, do artigo 197-A a 197-D. Art. 44. Enquanto não der conta de sua administra-
ção e saldar o seu alcance, não pode o tutor ou o curador
Art. 39. A adoção de criança e de adolescente reger-se- adotar o pupilo ou o curatelado.
-á segundo o disposto nesta Lei.
§ 1o A adoção é medida excepcional e irrevogável, Art. 45. A adoção depende do consentimento dos pais
à qual se deve recorrer apenas quando esgotados os recur- ou do representante legal do adotando.
sos de manutenção da criança ou adolescente na família § 1º O consentimento será dispensado em relação à
natural ou extensa, na forma do parágrafo único do art. criança ou adolescente cujos pais sejam desconhecidos ou
25 desta Lei. tenham sido destituídos do poder familiar.
§ 2o É vedada a adoção por procuração. § 2º Em se tratando de adotando maior de doze anos
§ 3o Em caso de conflito entre direitos e interesses do de idade, será também necessário o seu consentimento.
adotando e de outras pessoas, inclusive seus pais biológi-
cos, devem prevalecer os direitos e os interesses do ado- Art. 46. A adoção será precedida de estágio de convi-
tando. vência com a criança ou adolescente, pelo prazo máxi-
mo de 90 (noventa) dias, observadas a idade da criança
Art. 40. O adotando deve contar com, no máximo, de-
ou adolescente e as peculiaridades do caso.
zoito anos à data do pedido, salvo se já estiver sob a
§ 1o O estágio de convivência poderá ser dispensado
guarda ou tutela dos adotantes.
se o adotando já estiver sob a tutela ou guarda legal do
Art. 41. A adoção atribui a condição de filho ao adota- adotante durante tempo suficiente para que seja possível
do, com os mesmos direitos e deveres, inclusive sucessó- avaliar a conveniência da constituição do vínculo.
rios, desligando-o de qualquer vínculo com pais e paren- § 2o A simples guarda de fato não autoriza, por si só, a
tes, salvo os impedimentos matrimoniais. dispensa da realização do estágio de convivência.
§ 1º Se um dos cônjuges ou concubinos adota o filho § 2o-A. O prazo máximo estabelecido no caput deste
do outro, mantêm-se os vínculos de filiação entre o ado- artigo pode ser prorrogado por até igual período, mediante
tado e o cônjuge ou concubino do adotante e os respec- decisão fundamentada da autoridade judiciária.
tivos parentes. § 3o Em caso de adoção por pessoa ou casal residente
§ 2º É recíproco o direito sucessório entre o adotado, ou domiciliado fora do País, o estágio de convivência será
seus descendentes, o adotante, seus ascendentes, des- de, no mínimo, 30 (trinta) dias e, no máximo, 45 (quarenta
cendentes e colaterais até o 4º grau, observada a ordem e cinco) dias, prorrogável por até igual período, uma única
de vocação hereditária. vez, mediante decisão fundamentada da autoridade judi-
ciária.
Art. 42. Podem adotar os maiores de 18 (dezoito) § 3o-A. Ao final do prazo previsto no § 3o deste artigo,
anos, independentemente do estado civil.
deverá ser apresentado laudo fundamentado pela equipe
§ 1º Não podem adotar os ascendentes e os irmãos
mencionada no § 4o deste artigo, que recomendará ou não
do adotando.
o deferimento da adoção à autoridade judiciária.
§ 2o Para adoção conjunta, é indispensável que os
adotantes sejam casados civilmente ou mantenham § 4o O estágio de convivência será acompanhado pela
união estável, comprovada a estabilidade da família. equipe interprofissional a serviço da Justiça da Infância e da
§ 3º O adotante há de ser, pelo menos, dezesseis anos Juventude, preferencialmente com apoio dos técnicos res-
mais velho do que o adotando. ponsáveis pela execução da política de garantia do direito à
LEGISLAÇÃO ESPECÍFICA

§ 4o Os divorciados, os judicialmente separados e os convivência familiar, que apresentarão relatório minucioso


ex-companheiros podem adotar conjuntamente, contanto acerca da conveniência do deferimento da medida.
que acordem sobre a guarda e o regime de visitas e desde § 5o O estágio de convivência será cumprido no territó-
que o estágio de convivência tenha sido iniciado na cons- rio nacional, preferencialmente na comarca de residência
tância do período de convivência e que seja comprovada da criança ou adolescente, ou, a critério do juiz, em cidade
a existência de vínculos de afinidade e afetividade com limítrofe, respeitada, em qualquer hipótese, a competência
aquele não detentor da guarda, que justifiquem a excep- do juízo da comarca de residência da criança.
cionalidade da concessão.

100
Art. 47. O vínculo da adoção constitui-se por sen- § 4o Sempre que possível e recomendável, a prepara-
tença judicial, que será inscrita no registro civil mediante ção referida no § 3o deste artigo incluirá o contato com
mandado do qual não se fornecerá certidão. crianças e adolescentes em acolhimento familiar ou ins-
§ 1º A inscrição consignará o nome dos adotantes titucional em condições de serem adotados, a ser reali-
como pais, bem como o nome de seus ascendentes. zado sob a orientação, supervisão e avaliação da equipe
§ 2º O mandado judicial, que será arquivado, cancela- técnica da Justiça da Infância e da Juventude, com apoio
rá o registro original do adotado. dos técnicos responsáveis pelo programa de acolhimen-
§ 3o A pedido do adotante, o novo registro poderá ser to e pela execução da política municipal de garantia do
lavrado no Cartório do Registro Civil do Município de sua direito à convivência familiar.
residência. § 5o Serão criados e implementados cadastros esta-
§ 4o Nenhuma observação sobre a origem do ato po- duais e nacional de crianças e adolescentes em condi-
derá constar nas certidões do registro. ções de serem adotados e de pessoas ou casais habilita-
§ 5o A sentença conferirá ao adotado o nome do ado- dos à adoção.
tante e, a pedido de qualquer deles, poderá determinar a § 6o Haverá cadastros distintos para pessoas ou casais
modificação do prenome. residentes fora do País, que somente serão consultados
§ 6o Caso a modificação de prenome seja requerida na inexistência de postulantes nacionais habilitados nos
pelo adotante, é obrigatória a oitiva do adotando, obser- cadastros mencionados no § 5o deste artigo.
vado o disposto nos §§ 1o e 2o do art. 28 desta Lei. § 7o As autoridades estaduais e federais em matéria
§ 7o A adoção produz seus efeitos a partir do trânsito de adoção terão acesso integral aos cadastros, incumbin-
em julgado da sentença constitutiva, exceto na hipótese do-lhes a troca de informações e a cooperação mútua,
prevista no § 6o do art. 42 desta Lei, caso em que terá para melhoria do sistema.
força retroativa à data do óbito. § 8o A autoridade judiciária providenciará, no prazo
§ 8o O processo relativo à adoção assim como outros de 48 (quarenta e oito) horas, a inscrição das crianças e
a ele relacionados serão mantidos em arquivo, admitin- adolescentes em condições de serem adotados que não
do-se seu armazenamento em microfilme ou por outros tiveram colocação familiar na comarca de origem, e das
meios, garantida a sua conservação para consulta a qual- pessoas ou casais que tiveram deferida sua habilitação à
quer tempo. adoção nos cadastros estadual e nacional referidos no § 5o
§ 9º Terão prioridade de tramitação os processos de
deste artigo, sob pena de responsabilidade.
adoção em que o adotando for criança ou adolescente
§ 9o Compete à Autoridade Central Estadual zelar
com deficiência ou com doença crônica.
pela manutenção e correta alimentação dos cadastros,
§ 10. O prazo máximo para conclusão da ação de ado-
com posterior comunicação à Autoridade Central Federal
ção será de 120 (cento e vinte) dias, prorrogável uma úni-
Brasileira.
ca vez por igual período, mediante decisão fundamenta-
§ 10. Consultados os cadastros e verificada a ausência
da da autoridade judiciária.
de pretendentes habilitados residentes no País com perfil
Art. 48. O adotado tem direito de conhecer sua ori- compatível e interesse manifesto pela adoção de crian-
gem biológica, bem como de obter acesso irrestrito ao ça ou adolescente inscrito nos cadastros existentes, será
processo no qual a medida foi aplicada e seus eventuais realizado o encaminhamento da criança ou adolescente
incidentes, após completar 18 (dezoito) anos. à adoção internacional.
Parágrafo único. O acesso ao processo de adoção po- § 11. Enquanto não localizada pessoa ou casal in-
derá ser também deferido ao adotado menor de 18 (dezoi- teressado em sua adoção, a criança ou o adolescente,
to) anos, a seu pedido, assegurada orientação e assistência sempre que possível e recomendável, será colocado sob
jurídica e psicológica. guarda de família cadastrada em programa de acolhi-
mento familiar.
Art. 49. A morte dos adotantes não restabelece o poder § 12. A alimentação do cadastro e a convocação cri-
familiar dos pais naturais. teriosa dos postulantes à adoção serão fiscalizadas pelo
Ministério Público.
Art. 50. A autoridade judiciária manterá, em cada co- § 13. Somente poderá ser deferida adoção em favor
marca ou foro regional, um registro de crianças e ado- de candidato domiciliado no Brasil não cadastrado pre-
lescentes em condições de serem adotados e outro de viamente nos termos desta Lei quando:
pessoas interessadas na adoção. I - se tratar de pedido de adoção unilateral;
§ 1º O deferimento da inscrição dar-se-á após prévia II - for formulada por parente com o qual a criança ou
consulta aos órgãos técnicos do juizado, ouvido o Minis- adolescente mantenha vínculos de afinidade e afetividade;
tério Público. III - oriundo o pedido de quem detém a tutela ou guar-
§ 2º Não será deferida a inscrição se o interessado
LEGISLAÇÃO ESPECÍFICA

da legal de criança maior de 3 (três) anos ou adolescente,


não satisfazer os requisitos legais, ou verificada qualquer desde que o lapso de tempo de convivência comprove a fi-
das hipóteses previstas no art. 29. xação de laços de afinidade e afetividade, e não seja cons-
§ 3o A inscrição de postulantes à adoção será prece- tatada a ocorrência de má-fé ou qualquer das situações
dida de um período de preparação psicossocial e jurídi- previstas nos arts. 237 ou 238 desta Lei.
ca, orientado pela equipe técnica da Justiça da Infância e § 14. Nas hipóteses previstas no § 13 deste artigo, o
da Juventude, preferencialmente com apoio dos técnicos candidato deverá comprovar, no curso do procedimento,
responsáveis pela execução da política municipal de ga- que preenche os requisitos necessários à adoção, confor-
rantia do direito à convivência familiar. me previsto nesta Lei.

101
§ 15. Será assegurada prioridade no cadastro a pes- V - os documentos em língua estrangeira serão devida-
soas interessadas em adotar criança ou adolescente com mente autenticados pela autoridade consular, observados
deficiência, com doença crônica ou com necessidades os tratados e convenções internacionais, e acompanhados
específicas de saúde, além de grupo de irmãos. da respectiva tradução, por tradutor público juramentado;
VI - a Autoridade Central Estadual poderá fazer exigên-
Art. 51. Considera-se adoção internacional aquela cias e solicitar complementação sobre o estudo psicossocial
na qual o pretendente possui residência habitual em país- do postulante estrangeiro à adoção, já realizado no país de
-parte da Convenção de Haia, de 29 de maio de 1993, acolhida;
Relativa à Proteção das Crianças e à Cooperação em Ma- VII - verificada, após estudo realizado pela Autoridade
téria de Adoção Internacional, promulgada pelo Decreto Central Estadual, a compatibilidade da legislação estrangeira
no 3.087, de 21 junho de 1999, e deseja adotar criança em com a nacional, além do preenchimento por parte dos pos-
outro país-parte da Convenção. tulantes à medida dos requisitos objetivos e subjetivos neces-
§ 1o A adoção internacional de criança ou adolescen- sários ao seu deferimento, tanto à luz do que dispõe esta Lei
te brasileiro ou domiciliado no Brasil somente terá lugar como da legislação do país de acolhida, será expedido laudo
quando restar comprovado: de habilitação à adoção internacional, que terá validade por,
I - que a colocação em família adotiva é a solução no máximo, 1 (um) ano;
VIII - de posse do laudo de habilitação, o interessado será
adequada ao caso concreto;
autorizado a formalizar pedido de adoção perante o Juízo
II - que foram esgotadas todas as possibilidades de
da Infância e da Juventude do local em que se encontra a
colocação da criança ou adolescente em família adotiva
criança ou adolescente, conforme indicação efetuada pela
brasileira, com a comprovação, certificada nos autos, da
Autoridade Central Estadual.
inexistência de adotantes habilitados residentes no Brasil § 1o Se a legislação do país de acolhida assim o autorizar,
com perfil compatível com a criança ou adolescente, após admite-se que os pedidos de habilitação à adoção interna-
consulta aos cadastros mencionados nesta Lei; cional sejam intermediados por organismos credenciados.
III - que, em se tratando de adoção de adolescente, § 2o Incumbe à Autoridade Central Federal Brasileira o
este foi consultado, por meios adequados ao seu estágio credenciamento de organismos nacionais e estrangeiros
de desenvolvimento, e que se encontra preparado para a encarregados de intermediar pedidos de habilitação à ado-
medida, mediante parecer elaborado por equipe interpro- ção internacional, com posterior comunicação às Autorida-
fissional, observado o disposto nos §§ 1o e 2o do art. 28 des Centrais Estaduais e publicação nos órgãos oficiais de
desta Lei. imprensa e em sítio próprio da internet.
§ 2o Os brasileiros residentes no exterior terão prefe- § 3o Somente será admissível o credenciamento de or-
rência aos estrangeiros, nos casos de adoção internacio- ganismos que:
nal de criança ou adolescente brasileiro. I - sejam oriundos de países que ratificaram a Convenção
§ 3o A adoção internacional pressupõe a intervenção de Haia e estejam devidamente credenciados pela Autori-
das Autoridades Centrais Estaduais e Federal em matéria dade Central do país onde estiverem sediados e no país de
de adoção internacional. acolhida do adotando para atuar em adoção internacional
no Brasil;
Art. 52. A adoção internacional observará o proce- II - satisfizerem as condições de integridade moral, com-
dimento previsto nos arts. 165 a 170 desta Lei, com as petência profissional, experiência e responsabilidade exigidas
seguintes adaptações: pelos países respectivos e pela Autoridade Central Federal
I - a pessoa ou casal estrangeiro, interessado em Brasileira;
adotar criança ou adolescente brasileiro, deverá formu- III - forem qualificados por seus padrões éticos e sua for-
lar pedido de habilitação à adoção perante a Autoridade mação e experiência para atuar na área de adoção interna-
Central em matéria de adoção internacional no país de cional;
acolhida, assim entendido aquele onde está situada sua IV - cumprirem os requisitos exigidos pelo ordenamento
jurídico brasileiro e pelas normas estabelecidas pela Autori-
residência habitual;
dade Central Federal Brasileira.
II - se a Autoridade Central do país de acolhida con-
§ 4o Os organismos credenciados deverão ainda:
siderar que os solicitantes estão habilitados e aptos para
I - perseguir unicamente fins não lucrativos, nas condi-
adotar, emitirá um relatório que contenha informações
ções e dentro dos limites fixados pelas autoridades compe-
sobre a identidade, a capacidade jurídica e adequação tentes do país onde estiverem sediados, do país de acolhida e
dos solicitantes para adotar, sua situação pessoal, familiar pela Autoridade Central Federal Brasileira;
e médica, seu meio social, os motivos que os animam e II - ser dirigidos e administrados por pessoas qualificadas
sua aptidão para assumir uma adoção internacional;
LEGISLAÇÃO ESPECÍFICA

e de reconhecida idoneidade moral, com comprovada for-


III - a Autoridade Central do país de acolhida enviará mação ou experiência para atuar na área de adoção inter-
o relatório à Autoridade Central Estadual, com cópia para nacional, cadastradas pelo Departamento de Polícia Federal
a Autoridade Central Federal Brasileira; e aprovadas pela Autoridade Central Federal Brasileira, me-
IV - o relatório será instruído com toda a documen- diante publicação de portaria do órgão federal competente;
tação necessária, incluindo estudo psicossocial elaborado III - estar submetidos à supervisão das autoridades com-
por equipe interprofissional habilitada e cópia autentica- petentes do país onde estiverem sediados e no país de aco-
da da legislação pertinente, acompanhada da respectiva lhida, inclusive quanto à sua composição, funcionamento
prova de vigência; e situação financeira;

102
IV - apresentar à Autoridade Central Federal Brasileira, Art. 52-A. É vedado, sob pena de responsabilidade e
a cada ano, relatório geral das atividades desenvolvidas, descredenciamento, o repasse de recursos provenientes
bem como relatório de acompanhamento das adoções in- de organismos estrangeiros encarregados de intermediar
ternacionais efetuadas no período, cuja cópia será enca- pedidos de adoção internacional a organismos nacio-
minhada ao Departamento de Polícia Federal; nais ou a pessoas físicas.
V - enviar relatório pós-adotivo semestral para a Au- Parágrafo único. Eventuais repasses somente poderão
toridade Central Estadual, com cópia para a Autoridade ser efetuados via Fundo dos Direitos da Criança e do Ado-
Central Federal Brasileira, pelo período mínimo de 2 (dois) lescente e estarão sujeitos às deliberações do respectivo
anos. O envio do relatório será mantido até a juntada de Conselho de Direitos da Criança e do Adolescente.
cópia autenticada do registro civil, estabelecendo a cida-
dania do país de acolhida para o adotado; Art. 52-B. A adoção por brasileiro residente no ex-
VI - tomar as medidas necessárias para garantir que terior em país ratificante da Convenção de Haia, cujo pro-
os adotantes encaminhem à Autoridade Central Federal cesso de adoção tenha sido processado em conformidade
Brasileira cópia da certidão de registro de nascimento es- com a legislação vigente no país de residência e atendido o
trangeira e do certificado de nacionalidade tão logo lhes disposto na Alínea “c” do Artigo 17 da referida Convenção,
sejam concedidos. será automaticamente recepcionada com o reingresso
§ 5o A não apresentação dos relatórios referidos no § no Brasil.
4 deste artigo pelo organismo credenciado poderá acar-
o
§ 1o Caso não tenha sido atendido o disposto na Alí-
retar a suspensão de seu credenciamento. nea “c” do Artigo 17 da Convenção de Haia, deverá a sen-
§ 6o O credenciamento de organismo nacional ou es- tença ser homologada pelo Superior Tribunal de Justiça.
trangeiro encarregado de intermediar pedidos de ado- § 2o O pretendente brasileiro residente no exterior
ção internacional terá validade de 2 (dois) anos. em país não ratificante da Convenção de Haia, uma vez
§ 7o A renovação do credenciamento poderá ser con- reingressado no Brasil, deverá requerer a homologação da
cedida mediante requerimento protocolado na Autorida- sentença estrangeira pelo Superior Tribunal de Justiça.
de Central Federal Brasileira nos 60 (sessenta) dias ante-
riores ao término do respectivo prazo de validade. Art. 52-C. Nas adoções internacionais, quando o
§ 8o Antes de transitada em julgado a decisão que Brasil for o país de acolhida, a decisão da autoridade
concedeu a adoção internacional, não será permitida a competente do país de origem da criança ou do adoles-
§ 9o Transitada em julgado a decisão, a autoridade cente será conhecida pela Autoridade Central Estadual
judiciária determinará a expedição de alvará com auto- que tiver processado o pedido de habilitação dos pais
rização de viagem, bem como para obtenção de passa- adotivos, que comunicará o fato à Autoridade Central
porte, constando, obrigatoriamente, as características da Federal e determinará as providências necessárias à expe-
criança ou adolescente adotado, como idade, cor, sexo, dição do Certificado de Naturalização Provisório.
eventuais sinais ou traços peculiares, assim como foto re- § 1o A Autoridade Central Estadual, ouvido o Minis-
cente e a aposição da impressão digital do seu polegar tério Público, somente deixará de reconhecer os efeitos
direito, instruindo o documento com cópia autenticada daquela decisão se restar demonstrado que a adoção é
da decisão e certidão de trânsito em julgado. manifestamente contrária à ordem pública ou não atende
§ 10. A Autoridade Central Federal Brasileira poderá, ao interesse superior da criança ou do adolescente.
a qualquer momento, solicitar informações sobre a situa- § 2o Na hipótese de não reconhecimento da adoção,
ção das crianças e adolescentes adotados. prevista no § 1o deste artigo, o Ministério Público deverá
§ 11. A cobrança de valores por parte dos organismos imediatamente requerer o que for de direito para resguar-
credenciados, que sejam considerados abusivos pela Au- dar os interesses da criança ou do adolescente, comuni-
toridade Central Federal Brasileira e que não estejam cando-se as providências à Autoridade Central Estadual,
devidamente comprovados, é causa de seu descreden- que fará a comunicação à Autoridade Central Federal Bra-
ciamento. sileira e à Autoridade Central do país de origem.
§ 12. Uma mesma pessoa ou seu cônjuge não podem
ser representados por mais de uma entidade credenciada Art. 52-D. Nas adoções internacionais, quando o Brasil
para atuar na cooperação em adoção internacional. for o país de acolhida e a adoção não tenha sido deferida
§ 13. A habilitação de postulante estrangeiro ou do- no país de origem porque a sua legislação a delega ao país
miciliado fora do Brasil terá validade máxima de 1 (um) de acolhida, ou, ainda, na hipótese de, mesmo com decisão,
ano, podendo ser renovada. a criança ou o adolescente ser oriundo de país que não te-
§ 14. É vedado o contato direto de representantes de nha aderido à Convenção referida, o processo de adoção
organismos de adoção, nacionais ou estrangeiros, com seguirá as regras da adoção nacional.
dirigentes de programas de acolhimento institucional
LEGISLAÇÃO ESPECÍFICA

ou familiar, assim como com crianças e adolescentes em A adoção no Código Civil de 1916 era tratada em seu
condições de serem adotados, sem a devida autorização capítulo V. A adoção seguia um critério de ter um filho
judicial. para que a família tivesse sucessão e para configurar a fa-
§ 15. A Autoridade Central Federal Brasileira poderá mília da época que só tinha uma configuração costumeira
limitar ou suspender a concessão de novos credencia- se houvesse pai, mãe e filhos. A idade de 30 (trinta) anos
mentos sempre que julgar necessário, mediante ato ad- para que as pessoas pudessem não se arrepender do fei-
ministrativo fundamentado. to. Quando se falava do casamento criava-se o critério
de que tenha se passado 05 (cinco) anos de matrimônio

103
para que o casal possa adotar, pois o ideal era que os Os seguintes princípios e diretrizes devem guiar a de-
filhos fossem consanguíneos. A adoção poderia se dis- cisão pela adoção:
solver por vontade das partes. Caso houvessem filhos - Condição da criança e do adolescente como ser de
naturais reconhecidos ou legitimados os filhos adotivos direitos: As crianças são detentoras de direitos previstos
não participavam da sucessão além de não se desfazer o na lei 8.069/90 e na Constituição Federal de 1988;
vínculo com os parentes naturais, somente no que dizia - Proteção integral e prioritária: Toda e qualquer nor-
respeito ao pátrio poder. ma contida dentro da lei 8.069/90 deve ser voltada a pro-
Houve um grande salto no que concerne a adoção teção integral dos interesses do menor;
que foi a letra trazida pela Constituição Federal de 1988 - Responsabilidade primária e solidaria do poder pú-
em seu artigo 227 e com o ECA, que versa sobre todas as blico: tanto nos casos ressalvados pelo E.C.A (Estatuto da
garantias constitucionais e no que tange a adoção vem Criança e do Adolescente) quanto nos casos que a CF/88
dos artigos 39 ao 52-D falando sobre a regularização da (Constituição Federal de 1988) e demais objetos legais
adoção e traz como princípio basilar o melhor interesse específicos, é, dever dos três poderes atuarem no que for
da criança, além de ter como fundamento o afeto de pai necessário para a proteção do menor;
para filho sem que haja qualquer diferenciação para com - Interesse superior da criança e do adolescente: me-
os filhos biológicos. diante a necessidade da intervenção estatal é necessário
Ainda no que concerne a adoção, foi criada para dar que se dê privilégio ao interesse do que for melhor para
efetividade ao ECA a Lei nº 12.010 de 03 de agosto de a criança e/ou adolescente. O primeiro interesse a ser ob-
2009, que tem o intuito de alterar o direito à convivên- servado é o que trouxer melhor benefício a estes;
cia familiar mostrando com clareza como deve ocorrer a - Privacidade: A promoção dos direitos deve ser sem-
adoção. pre feita respeitando a privação e o direito da imagem, a
Para ser candidato a adoção deve ter cumprido uma sua vida privada e a intimidade;
série de requisitos para se tornar hábil, estes requisitos - Intervenção precoce: Ao primeiro sinal de perigo e
são elencados na lei. O primeiro deles é a qualificação risco ao menor, o Estado deve intervir para reverter a si-
completa: da pessoa que deseja adotar ou das pessoas tuação presente;
que desejam no caso de casais juntamente com os dados - Intervenção mínima;
familiares, dados que vão completar não só a ficha do ca- - Proporcionalidade e atualidade: Ser a atitude toma-
sal ou pessoa que deseja adotar, mais da família onde o da de acordo com a proporção de perigo existente no
menor vai residir e ter sua formação os documentos pes- momento, evitando e revertendo o risco de morte;
soais como cópias de certidão de nascimento (quando - Responsabilidade parental: a intervenção deve ser a
solteiros), de certidão de casamento (quando casados), princípio para que os pais tomem responsabilidade sobre
cópias de declaração de período de união estável, Cópias seus filhos;
de Certidão de identidade (RG), Comprovante da renda
- Prevalência da família: Primeiro se dará a preferência
da pessoa ou casal, comprovante que demostre que as
a família natural para que a criança continue com seus
condições vão suprir a necessidade de se incluir mais um
pais depois a preferência vai ser da família contínua (fa-
membro naquele lar, comprovante de moradia em que
mília natural a primeira e extensa a segunda), em último
prove a pessoa ter sua residência que acolherá o novo
caso a criança/adolescente vai ser direcionada a família
integrante.
substituta.
O interessado em adotar deve juntar atestados de
- Obrigatoriedade da informação: respeitando o es-
saúde física onde a pessoa vai demonstrar ser capaz de
cuidar e garantir uma boa vida para o adotando e atesta- tágio de desenvolvimento e compreensão da criança ou
do de saúde mental, comprovando que a pessoa é capaz adolescente, além de seus pais e responsáveis devem ser
legalmente. informados o motivo em que se dá a intervenção e como
Nos aspectos judiciais inerentes ao caso devem ser esta se processou;
juntados Certidão de antecedentes criminais, que de- - Oitiva obrigatória e participação: a criança ou ado-
mostra conduta legal do indivíduo perante a sociedade, lescente separados ou na companhia de seus pais, res-
Certidão negativa de distribuição civil, além da manifes- ponsáveis ou pessoas por ela indicados; bem como seus
tação do M.P (Ministério Público) apresentando quesi- pais e responsáveis, tem o direito de serem ouvidas e sua
tos a serem respondidos pela equipe interdisciplinar, opinião deve ser considerada pelas autoridades do judi-
designação de audiência para oitiva de testemunhas e ciário.
requerentes, solicitar juntada de documentos comple- - Afastamento da criança e do adolescente do con-
mentares que sejam necessários. Deve ser obrigatório o vívio familiar: É um processo contencioso que importara
estudo psicossocial onde se testa a capacidade dos in- aos pais o direito do contraditório e da ampla defesa,
divíduos para se tornarem pai ou mãe, o de incluir mais este procedimento é de competência do judiciário (pro-
cedimento judicial) artigo 28 §§ 1° e 2° do E.C.A;
LEGISLAÇÃO ESPECÍFICA

um membro junto aos filhos já existente. Os programas


de capacitação também devem ser aderido a este siste- - Acolhimento familiar: medida de proteção criada
ma, levando esclarecimentos aos pretendentes a adoção para o amparo da criança até que seja resolvida a si-
além, de manter contato com a criança em regime de tuação;
acolhimento familiar. - Guia de acolhimento: será elaborado pelo poder Ju-
Sendo os adotantes inscritos são chamados por or- diciário um guia para encaminhar à criança a entidade
dem cronológica de acordo com a habilitação, e que só de acolhimento (guia deve conter a causa da retirada e
poderá ser dispensada se for pelo melhor interesse do permanência do menor fora da sua família natural além
adotando. de todos os dados)

104
- Plano individual de atendimento: cada criança vai § 1º O acesso ao ensino obrigatório e gratuito é
ter um plano a ser desenvolvido visando sua adaptação direito público subjetivo.
a nova família; § 2º O não oferecimento do ensino obrigatório pelo
- Destituição do poder familiar: Promovida pelo M.P poder público ou sua oferta irregular importa responsabi-
(Ministério público) sendo para isto necessário prévio lidade da autoridade competente.
aviso a entidade de acolhimento familiar, ou, responsá- § 3º Compete ao poder público recensear os educan-
veis pela execução da política municipal de garantia do dos no ensino fundamental, fazer-lhes a chamada e zelar,
direito a convivência familiar. junto aos pais ou responsável, pela frequência à escola.
- Cadastro de crianças a adolescentes à regime ins-
titucional e familiar: é um cadastro para crianças e ado- Art. 55. Os pais ou responsável têm a obrigação de ma-
lescentes que necessitem de acolhimento familiar ou tricular seus filhos ou pupilos na rede regular de ensino.
institucional devido a algum registro de maus tratos.
A Lei no 13.509 de 22 de novembro de 2017 surgiu Art. 56. Os dirigentes de estabelecimentos de ensi-
no fundamental comunicarão ao Conselho Tutelar os
com o propósito de maximizar a efetividade das dispo-
casos de:
sições do ECA, inclusive no âmbito da adoção, possuin-
I - maus-tratos envolvendo seus alunos;
do a peculiaridade de ser mais rigorosa no que tange II - reiteração de faltas injustificadas e de evasão es-
ao cumprimento de prazos para maior celeridade do colar, esgotados os recursos escolares;
processo. III - elevados níveis de repetência.

Capítulo IV Art. 57. O poder público estimulará pesquisas, experiên-


Do Direito à Educação, à Cultura, ao Esporte e ao cias e novas propostas relativas a calendário, seriação,
Lazer currículo, metodologia, didática e avaliação, com vistas
à inserção de crianças e adolescentes excluídos do ensino
Art. 53. A criança e o adolescente têm direito à edu- fundamental obrigatório.
cação, visando ao pleno desenvolvimento de sua pessoa,
preparo para o exercício da cidadania e qualificação para Art. 58. No processo educacional respeitar-se-ão os va-
o trabalho, assegurando-se-lhes: lores culturais, artísticos e históricos próprios do contexto
I - igualdade de condições para o acesso e perma- social da criança e do adolescente, garantindo-se a estes
nência na escola; a liberdade da criação e o acesso às fontes de cultura.
II - direito de ser respeitado por seus educadores;
III - direito de contestar critérios avaliativos, po- Art. 59. Os municípios, com apoio dos estados e da
dendo recorrer às instâncias escolares superiores; União, estimularão e facilitarão a destinação de recursos
IV - direito de organização e participação em enti- e espaços para programações culturais, esportivas e de
lazer voltadas para a infância e a juventude.
dades estudantis;
V - acesso à escola pública e gratuita próxima de
Capítulo V
sua residência.
Do Direito à Profissionalização e à Proteção no
Parágrafo único. É direito dos pais ou responsáveis Trabalho
ter ciência do processo pedagógico, bem como participar
da definição das propostas educacionais. Art. 60. É proibido qualquer trabalho a menores de
quatorze anos de idade, salvo na condição de aprendiz.
Art. 54. É dever do Estado assegurar à criança e ao Preconiza o artigo 7º, XXXIII, CF a “proibição de traba-
adolescente: lho noturno, perigoso ou insalubre a menores de de-
I - ensino fundamental, obrigatório e gratuito, in- zoito e de qualquer trabalho a menores de dezesseis
clusive para os que a ele não tiveram acesso na idade anos, salvo na condição de aprendiz, a partir de quator-
própria; ze anos”.
II - progressiva extensão da obrigatoriedade e gra- Portanto, em decorrência da própria norma constitu-
tuidade ao ensino médio; cional, nenhuma criança ou adolescente pode trabalhar
III - atendimento educacional especializado aos por- antes dos 14 anos de idade. Evidentemente que há algu-
tadores de deficiência, preferencialmente na rede regu- mas exceções a esta regra, devidamente fiscalizadas pelo
lar de ensino; Conselho Tutelar, como é o caso dos artistas mirins.
IV - atendimento em creche e pré-escola às crianças Entre 14 anos e 16 anos de idade somente será possí-
de zero a cinco anos de idade; vel o trabalho na condição de menor aprendiz, cuja natu-
LEGISLAÇÃO ESPECÍFICA

reza é de ensino técnico-profissional, viabilizando a futura


V - acesso aos níveis mais elevados do ensino, da
inserção do adolescente no mercado de trabalho.
pesquisa e da criação artística, segundo a capacidade
A partir dos 16 anos, o menor pode trabalhar, mas não
de cada um; no período noturno ou em condições de periculosidade e
VI - oferta de ensino noturno regular, adequado às insalubridade.
condições do adolescente trabalhador;
VII - atendimento no ensino fundamental, através de Art. 61. A proteção ao trabalho dos adolescentes é re-
programas suplementares de material didático-escolar, gulada por legislação especial, sem prejuízo do disposto
transporte, alimentação e assistência à saúde. nesta Lei.

105
Art. 62. Considera-se aprendizagem a formação téc- deverá assegurar ao adolescente que dele participe con-
nico-profissional ministrada segundo as diretrizes e ba- dições de capacitação para o exercício de atividade
ses da legislação de educação em vigor. regular remunerada.
§ 1º Entende-se por trabalho educativo a atividade
Art. 63. A formação técnico-profissional obedecerá aos laboral em que as exigências pedagógicas relativas ao
seguintes princípios: desenvolvimento pessoal e social do educando prevale-
I - garantia de acesso e frequência obrigatória ao cem sobre o aspecto produtivo.
ensino regular; § 2º A remuneração que o adolescente recebe pelo
II - atividade compatível com o desenvolvimento do trabalho efetuado ou a participação na venda dos produ-
adolescente; tos de seu trabalho não desfigura o caráter educativo.
III - horário especial para o exercício das atividades. Os programas sociais voltados à capacitação dos
Aquele que trabalha na condição de menor aprendiz adolescentes devem sempre ter por objetivo educá-lo
é obrigado a frequentar a escola, devendo ser facilitadas
para que ele adquira condições de inserir-se no mercado
as condições para que o faça, notadamente pelo estabe-
de trabalho. Deve ser ensinado, logo, dele não se deve
lecimento de horário especial de trabalho. Além disso, a
cobrar tanta produtividade, mas sim deve ser avaliado
atividade laboral deve ser compatível com as atividades
de ensino, até mesmo por se tratar de ensino técnico- pelo seu aprendizado. O fato do trabalho ser remunera-
-profissionalizante. do não desvirtua este propósito.
Ex.: um jovem pode trabalhar no período matutino,
frequentar o SENAI na parte da tarde e ir ao colégio no Art. 69. O adolescente tem direito à profissionaliza-
ensino médio noturno. ção e à proteção no trabalho, observados os seguintes
aspectos, entre outros:
Art. 64. Ao adolescente até quatorze anos de idade é I - respeito à condição peculiar de pessoa em desen-
assegurada bolsa de aprendizagem. volvimento;
Toda criança e adolescente que necessitar receberá II - capacitação profissional adequada ao mercado de
fomento para que não se desvincule das atividades de trabalho.
ensino. Trata-se de incentivo àquele que sem auxílio aca- Com efeito, profissionalização e proteção no traba-
baria entrando em situação irregular e trabalhando. lho são direitos fundamentais garantidos ao adolescente,
exigindo-se neste campo que sua condição peculiar ine-
Art. 65. Ao adolescente aprendiz, maior de quatorze rente ao processo de aprendizado seja respeitada e que
anos, são assegurados os direitos trabalhistas e previ- o trabalho sirva para permitir a sua inserção no mercado
denciários.
de trabalho.
Uma vez que o adolescente está autorizado a traba-
lhar, mesmo que na condição de menor aprendiz, possui
Título III
direitos trabalhistas e previdenciários.
Da Prevenção
Art. 66. Ao adolescente portador de deficiência é as-
segurado trabalho protegido. Capítulo I
O adolescente que possui deficiência não pode ser Disposições Gerais
exposto a uma situação de risco em decorrência da ati-
vidade laboral. Art. 70. É dever de todos prevenir a ocorrência de
ameaça ou violação dos direitos da criança e do ado-
Art. 67. Ao adolescente empregado, aprendiz, em re- lescente.
gime familiar de trabalho, aluno de escola técnica, assisti-
do em entidade governamental ou não-governamental, é Art. 70-A. A União, os Estados, o Distrito Federal e os
vedado trabalho: Municípios deverão atuar de forma articulada na ela-
I - noturno, realizado entre as vinte e duas horas de boração de políticas públicas e na execução de ações
um dia e as cinco horas do dia seguinte; destinadas a coibir o uso de castigo físico ou de tra-
II - perigoso, insalubre ou penoso; tamento cruel ou degradante e difundir formas não
III - realizado em locais prejudiciais à sua formação violentas de educação de crianças e de adolescentes,
e ao seu desenvolvimento físico, psíquico, moral e social; tendo como principais ações:
IV - realizado em horários e locais que não permitam I - a promoção de campanhas educativas permanentes
a frequência à escola. para a divulgação do direito da criança e do adolescente
O menor aprendiz está proibido de trabalhar no pe- de serem educados e cuidados sem o uso de castigo físico
ríodo noturno, em trabalho que o coloque exposto a pe-
LEGISLAÇÃO ESPECÍFICA

ou de tratamento cruel ou degradante e dos instrumentos


riculosidade (ex.: em andaimes, em áreas com risco de
de proteção aos direitos humanos;
incêndio ou choques), insalubridade (ex.: em freezers de
frigoríficos, expostos a radiação) ou penosidade (ex.: ex- II - a integração com os órgãos do Poder Judiciário, do
cesso de força física exigida). Ministério Público e da Defensoria Pública, com o Con-
selho Tutelar, com os Conselhos de Direitos da Criança e
Art. 68. O programa social que tenha por base o do Adolescente e com as entidades não governamentais
trabalho educativo, sob responsabilidade de entidade go- que atuam na promoção, proteção e defesa dos direitos da
vernamental ou não-governamental sem fins lucrativos, criança e do adolescente;

106
III - a formação continuada e a capacitação dos profis- Parágrafo único. Os responsáveis pelas diversões e
sionais de saúde, educação e assistência social e dos demais espetáculos públicos deverão afixar, em lugar visível e de
agentes que atuam na promoção, proteção e defesa dos di- fácil acesso, à entrada do local de exibição, informação
reitos da criança e do adolescente para o desenvolvimento destacada sobre a natureza do espetáculo e a faixa etária
das competências necessárias à prevenção, à identificação de especificada no certificado de classificação.
evidências, ao diagnóstico e ao enfrentamento de todas as
formas de violência contra a criança e o adolescente; Art. 75. Toda criança ou adolescente terá acesso às
IV - o apoio e o incentivo às práticas de resolução pacífi- diversões e espetáculos públicos classificados como
ca de conflitos que envolvam violência contra a criança e o adequados à sua faixa etária.
adolescente; Parágrafo único. As crianças menores de dez anos so-
V - a inclusão, nas políticas públicas, de ações que visem a mente poderão ingressar e permanecer nos locais de apre-
garantir os direitos da criança e do adolescente, desde a aten- sentação ou exibição quando acompanhadas dos pais ou
ção pré-natal, e de atividades junto aos pais e responsáveis responsável.
com o objetivo de promover a informação, a reflexão, o de-
bate e a orientação sobre alternativas ao uso de castigo físico Art. 76. As emissoras de rádio e televisão somente exi-
ou de tratamento cruel ou degradante no processo educativo; birão, no horário recomendado para o público infanto
VI - a promoção de espaços intersetoriais locais para a juvenil, programas com finalidades educativas, artís-
articulação de ações e a elaboração de planos de atuação ticas, culturais e informativas.
conjunta focados nas famílias em situação de violência, com Parágrafo único. Nenhum espetáculo será apresentado
participação de profissionais de saúde, de assistência social e ou anunciado sem aviso de sua classificação, antes de sua
de educação e de órgãos de promoção, proteção e defesa dos transmissão, apresentação ou exibição.
direitos da criança e do adolescente.
Parágrafo único. As famílias com crianças e adolescentes Art. 77. Os proprietários, diretores, gerentes e funcio-
com deficiência terão prioridade de atendimento nas ações e nários de empresas que explorem a venda ou aluguel de
políticas públicas de prevenção e proteção. fitas de programação em vídeo cuidarão para que não
haja venda ou locação em desacordo com a classifica-
Art. 70-B. As entidades, públicas e privadas, que atuem nas ção atribuída pelo órgão competente.
áreas a que se refere o art. 71, dentre outras, devem contar, Parágrafo único. As fitas a que alude este artigo deve-
em seus quadros, com pessoas capacitadas a reconhecer rão exibir, no invólucro, informação sobre a natureza da
e comunicar ao Conselho Tutelar suspeitas ou casos de obra e a faixa etária a que se destinam.
maus-tratos praticados contra crianças e adolescentes.
Parágrafo único. São igualmente responsáveis pela co- Art. 78. As revistas e publicações contendo material
municação de que trata este artigo, as pessoas encarregadas, impróprio ou inadequado a crianças e adolescentes de-
por razão de cargo, função, ofício, ministério, profissão ou verão ser comercializadas em embalagem lacrada, com
ocupação, do cuidado, assistência ou guarda de crianças e a advertência de seu conteúdo.
adolescentes, punível, na forma deste Estatuto, o injustificado Parágrafo único. As editoras cuidarão para que as ca-
retardamento ou omissão, culposos ou dolosos. pas que contenham mensagens pornográficas ou obscenas
sejam protegidas com embalagem opaca.
Art. 71. A criança e o adolescente têm direito a infor-
mação, cultura, lazer, esportes, diversões, espetáculos e Art. 79. As revistas e publicações destinadas ao pú-
produtos e serviços que respeitem sua condição peculiar de blico infanto-juvenil não poderão conter ilustrações,
pessoa em desenvolvimento. fotografias, legendas, crônicas ou anúncios de bebidas
alcoólicas, tabaco, armas e munições, e deverão res-
Art. 72. As obrigações previstas nesta Lei não excluem peitar os valores éticos e sociais da pessoa e da família.
da prevenção especial outras decorrentes dos princípios por
ela adotados. Art. 80. Os responsáveis por estabelecimentos que ex-
plorem comercialmente bilhar, sinuca ou congênere ou
Art. 73. A inobservância das normas de prevenção im- por casas de jogos, assim entendidas as que realizem
portará em responsabilidade da pessoa física ou jurídica, apostas, ainda que eventualmente, cuidarão para que não
nos termos desta Lei. seja permitida a entrada e a permanência de crianças
e adolescentes no local, afixando aviso para orientação
Capítulo II do público.
Da Prevenção Especial
Seção II
Seção I Dos Produtos e Serviços
LEGISLAÇÃO ESPECÍFICA

Da informação, Cultura, Lazer, Esportes, Diversões


e Espetáculos Art. 81. É proibida a venda à criança ou ao adoles-
cente de:
Art. 74. O poder público, através do órgão competente, I - armas, munições e explosivos;
regulará as diversões e espetáculos públicos, infor- II - bebidas alcoólicas;
mando sobre a natureza deles, as faixas etárias a que III - produtos cujos componentes possam causar de-
não se recomendem, locais e horários em que sua apre- pendência física ou psíquica ainda que por utilização
sentação se mostre inadequada. indevida;

107
IV - fogos de estampido e de artifício, exceto aqueles III - serviços especiais de prevenção e atendimento
que pelo seu reduzido potencial sejam incapazes de provocar médico e psicossocial às vítimas de negligência, maus-tra-
qualquer dano físico em caso de utilização indevida; tos, exploração, abuso, crueldade e opressão;
V - revistas e publicações a que alude o art. 78; IV - serviço de identificação e localização de pais, res-
VI - bilhetes lotéricos e equivalentes. ponsável, crianças e adolescentes desaparecidos;
V - proteção jurídico-social por entidades de defesa
Art. 82. É proibida a hospedagem de criança ou ado- dos direitos da criança e do adolescente;
lescente em hotel, motel, pensão ou estabelecimento VI - políticas e programas destinados a prevenir ou abre-
congênere, salvo se autorizado ou acompanhado pelos viar o período de afastamento do convívio familiar e a
pais ou responsável. garantir o efetivo exercício do direito à convivência fami-
liar de crianças e adolescentes;
Seção III VII - campanhas de estímulo ao acolhimento sob forma
Da Autorização para Viajar de guarda de crianças e adolescentes afastados do convívio
familiar e à adoção, especificamente inter-racial, de crianças
Art. 83. Nenhuma criança ou adolescente menor de 16 (de- maiores ou de adolescentes, com necessidades específicas de
zesseis) anos poderá viajar para fora da comarca onde reside saúde ou com deficiências e de grupos de irmãos.
desacompanhado dos pais ou dos responsáveis sem expressa
autorização judicial. (Redação dada pela Lei nº 13.812, de 2019) Art. 88. São diretrizes da política de atendimento:
§ 1º A autorização não será exigida quando: I - municipalização do atendimento;
a) tratar-se de comarca contígua à da residência da crian- II - criação de conselhos municipais, estaduais e nacio-
ça ou do adolescente menor de 16 (dezesseis) anos, se na nal dos direitos da criança e do adolescente, órgãos delibera-
mesma unidade da Federação, ou incluída na mesma região tivos e controladores das ações em todos os níveis, assegura-
metropolitana; (Redação dada pela Lei nº 13.812, de 2019) da a participação popular paritária por meio de organizações
b) a criança ou o adolescente menor de 16 (dezesseis) anos representativas, segundo leis federal, estaduais e municipais;
estiver acompanhado: (Redação dada pela Lei nº 13.812, de 2019)
III - criação e manutenção de programas específicos,
1) de ascendente ou colateral maior, até o terceiro
observada a descentralização político-administrativa;
grau, comprovado documentalmente o parentesco;
IV - manutenção de fundos nacional, estaduais e mu-
2) de pessoa maior, expressamente autorizada pelo
nicipais vinculados aos respectivos conselhos dos direitos da
pai, mãe ou responsável.
criança e do adolescente;
§ 2º A autoridade judiciária poderá, a pedido dos pais ou
V - integração operacional de órgãos do Judiciário,
responsável, conceder autorização válida por dois anos.
Ministério Público, Defensoria, Segurança Pública e As-
Art. 84. Quando se tratar de viagem ao exterior, a auto- sistência Social, preferencialmente em um mesmo local,
rização é dispensável, se a criança ou adolescente: para efeito de agilização do atendimento inicial a adolescente
I - estiver acompanhado de ambos os pais ou respon- a quem se atribua autoria de ato infracional;
sável; VI - integração operacional de órgãos do Judiciário,
II - viajar na companhia de um dos pais, autorizado Ministério Público, Defensoria, Conselho Tutelar e en-
expressamente pelo outro através de documento com carregados da execução das políticas sociais básicas e de
firma reconhecida. assistência social, para efeito de agilização do atendimento
de crianças e de adolescentes inseridos em programas de aco-
Art. 85. Sem prévia e expressa autorização judicial, ne- lhimento familiar ou institucional, com vista na sua rápida
nhuma criança ou adolescente nascido em território na- reintegração à família de origem ou, se tal solução se mostrar
cional poderá sair do País em companhia de estrangeiro comprovadamente inviável, sua colocação em família substi-
residente ou domiciliado no exterior. tuta, em quaisquer das modalidades previstas no art. 28 desta
Lei;
Parte Especial VII - mobilização da opinião pública para a indispen-
sável participação dos diversos segmentos da sociedade;
Título I VIII - especialização e formação continuada dos pro-
Da Política de Atendimento fissionais que trabalham nas diferentes áreas da atenção à
primeira infância, incluindo os conhecimentos sobre direitos
Capítulo I da criança e sobre desenvolvimento infantil;
Disposições Gerais IX - formação profissional com abrangência dos di-
versos direitos da criança e do adolescente que favoreça a
Art. 86. A política de atendimento dos direitos da criança intersetorialidade no atendimento da criança e do adolescen-
e do adolescente far-se-á através de um conjunto articula- te e seu desenvolvimento integral;
LEGISLAÇÃO ESPECÍFICA

do de ações governamentais e não-governamentais, da X - realização e divulgação de pesquisas sobre desen-


União, dos estados, do Distrito Federal e dos municípios. volvimento infantil e sobre prevenção da violência.

Art. 87. São linhas de ação da política de atendimento: Art. 89. A função de membro do conselho nacional e dos
I - políticas sociais básicas; conselhos estaduais e municipais dos direitos da criança e do
II - serviços, programas, projetos e benefícios de assistên- adolescente é considerada de interesse público relevante
cia social de garantia de proteção social e de prevenção e redu- e não será remunerada.
ção de violações de direitos, seus agravamentos ou reincidências;

108
Capítulo II d) tenha em seus quadros pessoas inidôneas.
Das Entidades de Atendimento e) não se adequar ou deixar de cumprir as resoluções e
deliberações relativas à modalidade de atendimento pres-
Seção I tado expedidas pelos Conselhos de Direitos da Criança e do
Disposições Gerais Adolescente, em todos os níveis.
§ 2o O registro terá validade máxima de 4 (quatro)
Art. 90. As entidades de atendimento são respon- anos, cabendo ao Conselho Municipal dos Direitos da
sáveis pela manutenção das próprias unidades, assim Criança e do Adolescente, periodicamente, reavaliar o ca-
como pelo planejamento e execução de programas de pro- bimento de sua renovação, observado o disposto no § 1o
teção e socioeducativos destinados a crianças e adolescen- deste artigo.
tes, em regime de:
I - orientação e apoio sociofamiliar; Art. 92. As entidades que desenvolvam programas de
II - apoio socioeducativo em meio aberto; acolhimento familiar ou institucional deverão adotar os se-
III - colocação familiar; guintes princípios:
IV - acolhimento institucional; I - preservação dos vínculos familiares e promoção
V - prestação de serviços à comunidade; da reintegração familiar;
VI - liberdade assistida; II - integração em família substituta, quando esgo-
VII - semiliberdade; e tados os recursos de manutenção na família natural ou
VIII - internação. extensa;
§ 1o As entidades governamentais e não governa- III - atendimento personalizado e em pequenos
mentais deverão proceder à inscrição de seus programas, grupos;
especificando os regimes de atendimento, na forma de- IV - desenvolvimento de atividades em regime de
finida neste artigo, no Conselho Municipal dos Direitos coeducação;
da Criança e do Adolescente, o qual manterá registro das V - não desmembramento de grupos de irmãos;
inscrições e de suas alterações, do que fará comunicação VI - evitar, sempre que possível, a transferência para
ao Conselho Tutelar e à autoridade judiciária. outras entidades de crianças e adolescentes abrigados;
§ 2o Os recursos destinados à implementação e ma- VII - participação na vida da comunidade local;
nutenção dos programas relacionados neste artigo serão VIII - preparação gradativa para o desligamento;
previstos nas dotações orçamentárias dos órgãos públi- IX - participação de pessoas da comunidade no pro-
cos encarregados das áreas de Educação, Saúde e Assis- cesso educativo.
tência Social, dentre outros, observando-se o princípio § 1o O dirigente de entidade que desenvolve progra-
da prioridade absoluta à criança e ao adolescente preco- ma de acolhimento institucional é equiparado ao guar-
nizado pelo caput do art. 227 da Constituição Federal e dião, para todos os efeitos de direito.
pelo caput e parágrafo único do art. 4o desta Lei. § 2o Os dirigentes de entidades que desenvolvem pro-
§ 3o Os programas em execução serão reavaliados gramas de acolhimento familiar ou institucional remete-
pelo Conselho Municipal dos Direitos da Criança e do rão à autoridade judiciária, no máximo a cada 6 (seis) me-
Adolescente, no máximo, a cada 2 (dois) anos, constituin- ses, relatório circunstanciado acerca da situação de cada
do-se critérios para renovação da autorização de funcio- criança ou adolescente acolhido e sua família, para fins da
namento: reavaliação prevista no § 1o do art. 19 desta Lei.
I - o efetivo respeito às regras e princípios desta Lei, § 3o Os entes federados, por intermédio dos Pode-
bem como às resoluções relativas à modalidade de atendi- res Executivo e Judiciário, promoverão conjuntamente
mento prestado expedidas pelos Conselhos de Direitos da a permanente qualificação dos profissionais que atuam
Criança e do Adolescente, em todos os níveis; direta ou indiretamente em programas de acolhimento
II - a qualidade e eficiência do trabalho desenvolvido, institucional e destinados à colocação familiar de crianças
atestadas pelo Conselho Tutelar, pelo Ministério Público e e adolescentes, incluindo membros do Poder Judiciário,
pela Justiça da Infância e da Juventude; Ministério Público e Conselho Tutelar.
III - em se tratando de programas de acolhimento ins- § 4o Salvo determinação em contrário da autorida-
titucional ou familiar, serão considerados os índices de su- de judiciária competente, as entidades que desenvolvem
cesso na reintegração familiar ou de adaptação à família programas de acolhimento familiar ou institucional, se ne-
substituta, conforme o caso. cessário com o auxílio do Conselho Tutelar e dos órgãos
de assistência social, estimularão o contato da criança ou
Art. 91. As entidades não-governamentais somente adolescente com seus pais e parentes, em cumprimento
poderão funcionar depois de registradas no Conselho ao disposto nos incisos I e VIII do caput deste artigo.
§ 5o As entidades que desenvolvem programas de
LEGISLAÇÃO ESPECÍFICA

Municipal dos Direitos da Criança e do Adolescente, o


qual comunicará o registro ao Conselho Tutelar e à auto- acolhimento familiar ou institucional somente poderão
ridade judiciária da respectiva localidade. receber recursos públicos se comprovado o atendimento
§ 1o Será negado o registro à entidade que: dos princípios, exigências e finalidades desta Lei.
a) não ofereça instalações físicas em condições adequa- § 6o O descumprimento das disposições desta Lei pelo
das de habitabilidade, higiene, salubridade e segurança; dirigente de entidade que desenvolva programas de aco-
b) não apresente plano de trabalho compatível com os lhimento familiar ou institucional é causa de sua destitui-
princípios desta Lei; ção, sem prejuízo da apuração de sua responsabilidade
c) esteja irregularmente constituída; administrativa, civil e criminal.

109
§ 7o Quando se tratar de criança de 0 (zero) a 3 (três) XVIII - manter programas destinados ao apoio e
anos em acolhimento institucional, dar-se-á especial acompanhamento de egressos;
atenção à atuação de educadores de referência estáveis XIX - providenciar os documentos necessários ao
e qualitativamente significativos, às rotinas específicas e exercício da cidadania àqueles que não os tiverem;
ao atendimento das necessidades básicas, incluindo as de XX - manter arquivo de anotações onde constem data
afeto como prioritárias. e circunstâncias do atendimento, nome do adolescente,
seus pais ou responsável, parentes, endereços, sexo, idade,
Art. 93. As entidades que mantenham programa de acompanhamento da sua formação, relação de seus per-
acolhimento institucional poderão, em caráter excepcional tences e demais dados que possibilitem sua identifica-
e de urgência, acolher crianças e adolescentes sem prévia ção e a individualização do atendimento.
determinação da autoridade competente, fazendo comu- § 1o Aplicam-se, no que couber, as obrigações cons-
nicação do fato em até 24 (vinte e quatro) horas ao Juiz tantes deste artigo às entidades que mantêm programas
da Infância e da Juventude, sob pena de responsabilidade. de acolhimento institucional e familiar.
Parágrafo único. Recebida a comunicação, a autorida- § 2º No cumprimento das obrigações a que alude este
de judiciária, ouvido o Ministério Público e se necessário artigo as entidades utilizarão preferencialmente os recur-
com o apoio do Conselho Tutelar local, tomará as medidas sos da comunidade.
necessárias para promover a imediata reintegração fami-
liar da criança ou do adolescente ou, se por qualquer razão Art. 94-A. As entidades, públicas ou privadas, que
não for isso possível ou recomendável, para seu encami- abriguem ou recepcionem crianças e adolescentes, ainda
nhamento a programa de acolhimento familiar, institucio- que em caráter temporário, devem ter, em seus quadros,
nal ou a família substituta, observado o disposto no § 2o profissionais capacitados a reconhecer e reportar ao
do art. 101 desta Lei. Conselho Tutelar suspeitas ou ocorrências de maus-tratos.

Art. 94. As entidades que desenvolvem programas Seção II


de internação têm as seguintes obrigações, entre ou- Da Fiscalização das Entidades
tras:
I - observar os direitos e garantias de que são titu- Art. 95. As entidades governamentais e não-governa-
lares os adolescentes; mentais referidas no art. 90 serão fiscalizadas pelo Ju-
diciário, pelo Ministério Público e pelos Conselhos Tu-
II - não restringir nenhum direito que não tenha sido
telares.
objeto de restrição na decisão de internação;
III - oferecer atendimento personalizado, em peque-
Art. 96. Os planos de aplicação e as prestações de
nas unidades e grupos reduzidos;
contas serão apresentados ao estado ou ao município,
IV - preservar a identidade e oferecer ambiente de
conforme a origem das dotações orçamentárias.
respeito e dignidade ao adolescente;
V - diligenciar no sentido do restabelecimento e da
Art. 97. São medidas aplicáveis às entidades de aten-
preservação dos vínculos familiares; dimento que descumprirem obrigação constante do art. 94,
VI - comunicar à autoridade judiciária, periodica- sem prejuízo da responsabilidade civil e criminal de
mente, os casos em que se mostre inviável ou impossível seus dirigentes ou prepostos:
o reatamento dos vínculos familiares; I - às entidades governamentais:
VII - oferecer instalações físicas em condições ade- a) advertência;
quadas de habitabilidade, higiene, salubridade e se- b) afastamento provisório de seus dirigentes;
gurança e os objetos necessários à higiene pessoal; c) afastamento definitivo de seus dirigentes;
VIII - oferecer vestuário e alimentação suficientes e d) fechamento de unidade ou interdição de progra-
adequados à faixa etária dos adolescentes atendidos; ma.
IX - oferecer cuidados médicos, psicológicos, odon- II - às entidades não-governamentais:
tológicos e farmacêuticos; a) advertência;
X - propiciar escolarização e profissionalização; b) suspensão total ou parcial do repasse de verbas
XI - propiciar atividades culturais, esportivas e de públicas;
lazer; c) interdição de unidades ou suspensão de progra-
XII - propiciar assistência religiosa àqueles que dese- ma;
jarem, de acordo com suas crenças; d) cassação do registro.
XIII - proceder a estudo social e pessoal de cada caso; § 1o Em caso de reiteradas infrações cometidas por en-
XIV - reavaliar periodicamente cada caso, com in- tidades de atendimento, que coloquem em risco os direi-
tervalo máximo de seis meses, dando ciência dos resulta- tos assegurados nesta Lei, deverá ser o fato comunicado
LEGISLAÇÃO ESPECÍFICA

dos à autoridade competente; ao Ministério Público ou representado perante autoridade


XV - informar, periodicamente, o adolescente inter- judiciária competente para as providências cabíveis, inclu-
nado sobre sua situação processual; sive suspensão das atividades ou dissolução da entidade.
XVI - comunicar às autoridades competentes todos os § 2o As pessoas jurídicas de direito público e as orga-
casos de adolescentes portadores de moléstias infecto- nizações não governamentais responderão pelos danos
contagiosas; que seus agentes causarem às crianças e aos adolescen-
XVII - fornecer comprovante de depósito dos perten- tes, caracterizado o descumprimento dos princípios nor-
ces dos adolescentes; teadores das atividades de proteção específica.

110
Título II IX - responsabilidade parental: a intervenção deve ser
Das Medidas de Proteção efetuada de modo que os pais assumam os seus deveres
para com a criança e o adolescente;
Capítulo I X - prevalência da família: na promoção de direitos e na
Disposições Gerais proteção da criança e do adolescente deve ser dada preva-
lência às medidas que os mantenham ou reintegrem na sua
Art. 98. As medidas de proteção à criança e ao adoles- família natural ou extensa ou, se isso não for possível, que
cente são aplicáveis sempre que os direitos reconhecidos promovam a sua integração em família adotiva;
nesta Lei forem ameaçados ou violados: XI - obrigatoriedade da informação: a criança e o ado-
I - por ação ou omissão da sociedade ou do Estado; lescente, respeitado seu estágio de desenvolvimento e capa-
II - por falta, omissão ou abuso dos pais ou respon- cidade de compreensão, seus pais ou responsável devem ser
sável; informados dos seus direitos, dos motivos que determina-
III - em razão de sua conduta. ram a intervenção e da forma como esta se processa;
XII - oitiva obrigatória e participação: a criança e o ado-
Capítulo II lescente, em separado ou na companhia dos pais, de res-
Das Medidas Específicas de Proteção ponsável ou de pessoa por si indicada, bem como os seus
pais ou responsável, têm direito a ser ouvidos e a participar
Art. 99. As medidas previstas neste Capítulo poderão nos atos e na definição da medida de promoção dos direitos
ser aplicadas isolada ou cumulativamente, bem como e de proteção, sendo sua opinião devidamente considerada
substituídas a qualquer tempo. pela autoridade judiciária competente, observado o dispos-
to nos §§ 1o e 2o do art. 28 desta Lei.
Art. 100. Na aplicação das medidas levar-se-ão em
conta as necessidades pedagógicas, preferindo-se aque- Art. 101. Verificada qualquer das hipóteses previstas no
las que visem ao fortalecimento dos vínculos familiares e art. 98, a autoridade competente poderá determinar, dentre
comunitários. outras, as seguintes medidas:
Parágrafo único. São também princípios que regem a I - encaminhamento aos pais ou responsável, me-
aplicação das medidas: diante termo de responsabilidade;
I - condição da criança e do adolescente como sujeitos II - orientação, apoio e acompanhamento temporá-
de direitos: crianças e adolescentes são os titulares dos di- rios;
reitos previstos nesta e em outras Leis, bem como na Cons-
III - matrícula e frequência obrigatórias em estabele-
tituição Federal;
cimento oficial de ensino fundamental;
II - proteção integral e prioritária: a interpretação e
IV - inclusão em serviços e programas oficiais ou
aplicação de toda e qualquer norma contida nesta Lei deve
comunitários de proteção, apoio e promoção da família,
ser voltada à proteção integral e prioritária dos direitos de
da criança e do adolescente;
que crianças e adolescentes são titulares;
V - requisição de tratamento médico, psicológico ou
III - responsabilidade primária e solidária do poder pú-
psiquiátrico, em regime hospitalar ou ambulatorial;
blico: a plena efetivação dos direitos assegurados a crian-
ças e a adolescentes por esta Lei e pela Constituição Fede- VI - inclusão em programa oficial ou comunitário
ral, salvo nos casos por esta expressamente ressalvados, é de auxílio, orientação e tratamento a alcoólatras e to-
de responsabilidade primária e solidária das 3 (três) esferas xicômanos;
de governo, sem prejuízo da municipalização do atendi- VII - acolhimento institucional;
mento e da possibilidade da execução de programas por VIII - inclusão em programa de acolhimento familiar;
entidades não governamentais; IX - colocação em família substituta.
IV - interesse superior da criança e do adolescente: a § 1o O acolhimento institucional e o acolhimento fa-
intervenção deve atender prioritariamente aos interesses miliar são medidas provisórias e excepcionais, utilizáveis
e direitos da criança e do adolescente, sem prejuízo da como forma de transição para reintegração familiar ou,
consideração que for devida a outros interesses legítimos não sendo esta possível, para colocação em família substi-
no âmbito da pluralidade dos interesses presentes no caso tuta, não implicando privação de liberdade.
concreto; § 2o Sem prejuízo da tomada de medidas emergenciais
V - privacidade: a promoção dos direitos e proteção da para proteção de vítimas de violência ou abuso sexual e
criança e do adolescente deve ser efetuada no respeito pela das providências a que alude o art. 130 desta Lei, o afasta-
intimidade, direito à imagem e reserva da sua vida privada; mento da criança ou adolescente do convívio familiar é de
VI - intervenção precoce: a intervenção das autoridades competência exclusiva da autoridade judiciária e importa-
competentes deve ser efetuada logo que a situação de pe- rá na deflagração, a pedido do Ministério Público ou de
quem tenha legítimo interesse, de procedimento judicial
LEGISLAÇÃO ESPECÍFICA

rigo seja conhecida;


VII - intervenção mínima: a intervenção deve ser exer- contencioso, no qual se garanta aos pais ou ao responsá-
cida exclusivamente pelas autoridades e instituições cuja vel legal o exercício do contraditório e da ampla defesa.
ação seja indispensável à efetiva promoção dos direitos e à § 3o Crianças e adolescentes somente poderão ser en-
proteção da criança e do adolescente; caminhados às instituições que executam programas de
VIII - proporcionalidade e atualidade: a intervenção acolhimento institucional, governamentais ou não, por
deve ser a necessária e adequada à situação de perigo em meio de uma Guia de Acolhimento, expedida pela au-
que a criança ou o adolescente se encontram no momento toridade judiciária, na qual obrigatoriamente constará,
em que a decisão é tomada; dentre outros:

111
I - sua identificação e a qualificação completa de seus § 11. A autoridade judiciária manterá, em cada comar-
pais ou de seu responsável, se conhecidos; ca ou foro regional, um cadastro contendo informações
II - o endereço de residência dos pais ou do responsá- atualizadas sobre as crianças e adolescentes em regime
vel, com pontos de referência; de acolhimento familiar e institucional sob sua respon-
III - os nomes de parentes ou de terceiros interessados sabilidade, com informações pormenorizadas sobre a si-
em tê-los sob sua guarda; tuação jurídica de cada um, bem como as providências
IV - os motivos da retirada ou da não reintegração ao tomadas para sua reintegração familiar ou colocação em
convívio familiar. família substituta, em qualquer das modalidades previs-
§ 4o Imediatamente após o acolhimento da criança tas no art. 28 desta Lei.
ou do adolescente, a entidade responsável pelo progra- § 12. Terão acesso ao cadastro o Ministério Público,
ma de acolhimento institucional ou familiar elaborará um o Conselho Tutelar, o órgão gestor da Assistência Social
plano individual de atendimento, visando à reintegração e os Conselhos Municipais dos Direitos da Criança e do
familiar, ressalvada a existência de ordem escrita e fun- Adolescente e da Assistência Social, aos quais incumbe
damentada em contrário de autoridade judiciária com- deliberar sobre a implementação de políticas públicas
petente, caso em que também deverá contemplar sua que permitam reduzir o número de crianças e adolescen-
colocação em família substituta, observadas as regras e tes afastados do convívio familiar e abreviar o período de
princípios desta Lei. permanência em programa de acolhimento.
§ 5o O plano individual será elaborado sob a respon-
sabilidade da equipe técnica do respectivo programa Art. 102. As medidas de proteção de que trata este Capítulo
de atendimento e levará em consideração a opinião da serão acompanhadas da regularização do registro civil.
criança ou do adolescente e a oitiva dos pais ou do res- § 1º Verificada a inexistência de registro anterior, o as-
ponsável. sento de nascimento da criança ou adolescente será feito
§ 6o Constarão do plano individual, dentre outros: à vista dos elementos disponíveis, mediante requisição
I - os resultados da avaliação interdisciplinar; da autoridade judiciária.
II - os compromissos assumidos pelos pais ou respon- § 2º Os registros e certidões necessários à regulariza-
sável; e ção de que trata este artigo são isentos de multas, custas
III - a previsão das atividades a serem desenvolvidas e emolumentos, gozando de absoluta prioridade.
com a criança ou com o adolescente acolhido e seus pais § 3o Caso ainda não definida a paternidade, será de-
ou responsável, com vista na reintegração familiar ou, flagrado procedimento específico destinado à sua ave-
caso seja esta vedada por expressa e fundamentada deter- riguação, conforme previsto pela Lei no 8.560, de 29 de
minação judicial, as providências a serem tomadas para dezembro de 1992.
§ 4o Nas hipóteses previstas no § 3o deste artigo, é
sua colocação em família substituta, sob direta supervisão
dispensável o ajuizamento de ação de investigação de
da autoridade judiciária.
paternidade pelo Ministério Público se, após o não com-
§ 7o O acolhimento familiar ou institucional ocorrerá
parecimento ou a recusa do suposto pai em assumir a
no local mais próximo à residência dos pais ou do res-
paternidade a ele atribuída, a criança for encaminhada
ponsável e, como parte do processo de reintegração fa-
para adoção.
miliar, sempre que identificada a necessidade, a família
§ 5º Os registros e certidões necessários à inclusão, a
de origem será incluída em programas oficiais de orien-
qualquer tempo, do nome do pai no assento de nascimen-
tação, de apoio e de promoção social, sendo facilitado to são isentos de multas, custas e emolumentos, gozando
e estimulado o contato com a criança ou com o adoles- de absoluta prioridade.
cente acolhido. § 6º São gratuitas, a qualquer tempo, a averbação
§ 8o Verificada a possibilidade de reintegração fami- requerida do reconhecimento de paternidade no assento
liar, o responsável pelo programa de acolhimento familiar de nascimento e a certidão correspondente.
ou institucional fará imediata comunicação à autoridade
judiciária, que dará vista ao Ministério Público, pelo pra- As normas de prevenção do ECA são destinadas a
zo de 5 (cinco) dias, decidindo em igual prazo. crianças e adolescentes em situação de risco. Existirá si-
§ 9o Em sendo constatada a impossibilidade de re- tuação de risco quando a criança ou o adolescente esti-
integração da criança ou do adolescente à família de verem privados de assistência. Essa assistência pode ser
origem, após seu encaminhamento a programas oficiais material (quando não se tem onde dormir, o que comer,
ou comunitários de orientação, apoio e promoção so- vestir etc.), moral (quando a criança ou o adolescente
cial, será enviado relatório fundamentado ao Ministério permanece em local inadequado, como locais de prática
Público, no qual conste a descrição pormenorizada das de jogo, prostituição etc.) ou jurídica (quando não tem
providências tomadas e a expressa recomendação, subs- quem o represente).
crita pelos técnicos da entidade ou responsáveis pela
LEGISLAÇÃO ESPECÍFICA

O menor que pratica ato infracional está em situação


execução da política municipal de garantia do direito à de risco por estar privado de assistência moral. A situa-
convivência familiar, para a destituição do poder familiar, ção de risco pode decorrer de ação ou omissão do Poder
ou destituição de tutela ou guarda. Público; ação ou omissão dos pais ou dos responsáveis;
§ 10. Recebido o relatório, o Ministério Público terá o por conduta própria.
prazo de 15 (quinze) dias para o ingresso com a ação de O art. 101 do ECA traz um rol das medidas protetivas
destituição do poder familiar, salvo se entender necessá- diante da situação de risco. Essas medidas poderão ser
ria a realização de estudos complementares ou de outras aplicadas tanto para a criança quanto para o adolescente.
providências indispensáveis ao ajuizamento da demanda. São elas:

112
- encaminhamento da criança e do adolescente aos Art. 108. A internação, antes da sentença, pode ser de-
pais ou responsáveis, mediante termo ou responsabili- terminada pelo prazo máximo de quarenta e cinco dias.
dade; Parágrafo único. A decisão deverá ser fundamentada e
- orientação, apoio e acompanhamentos temporários basear-se em indícios suficientes de autoria e materia-
por pessoa nomeada pelo Juiz; lidade, demonstrada a necessidade imperiosa da medida.
- matrícula e frequência obrigatória em estabeleci-
mento oficial de ensino fundamental (o Juiz determina Art. 109. O adolescente civilmente identificado não
aos pais a obrigação); será submetido a identificação compulsória pelos ór-
- inclusão em programa comunitário ou oficial de au- gãos policiais, de proteção e judiciais, salvo para efeito
xílio à família, à criança e ao adolescente; de confrontação, havendo dúvida fundada.
- requisição de tratamento médico, psicológico ou
psiquiátrico em regime hospitalar (internação) ou ambu- O adolescente não é preso, é apreendido.
latorial (consultas periódicas); A internação é a medida mais gravosa para o adoles-
- abrigo em entidade (não se fala em orfanato). A cente. O ECA permite a internação provisória durante o
doutrina chama de “Tutela de Estado” quando a criança processo. É fixado o prazo máximo de 45 dias. Os funda-
está em abrigo sob a proteção do Estado; mentos para que o Juiz decrete essa internação provisó-
- colocação em família substituta (é utilizada somente ria são: indícios suficientes de autoria e materialidade e
em situações muito graves). necessidade da medida.
O Juiz pode aplicar essas medidas isolada ou cumu- Esse prazo de internação provisória será descontado
lativamente. Pode, também, substituir uma medida pela na internação definitiva. Em nenhuma hipótese a criança
outra a qualquer tempo (art. 99 do ECA). Antes de aplicar poderá ser internada. Criança, que é todo aquele menor
qualquer uma dessas medidas, o Juiz deverá ouvir os pais de 12 anos, não se sujeita a medida sócio-educativa, mas
ou responsáveis, realizar estudo social do caso e ouvir o apenas a medida de proteção.
MP. Essa oitiva do MP é obrigatória, sob pena de nulida-
de (art. 204 do ECA). Esse rol do art. 101 é taxativo. Capítulo III
Das Garantias Processuais
Título III
Da Prática de Ato Infracional Art. 110. Nenhum adolescente será privado de sua li-
berdade sem o devido processo legal.
Capítulo I
Disposições Gerais
Art. 111. São asseguradas ao adolescente, entre outras,
as seguintes garantias:
Art. 103. Considera-se ato infracional a conduta
I - pleno e formal conhecimento da atribuição de
descrita como crime ou contravenção penal.
ato infracional, mediante citação ou meio equivalente;
Art. 104. São penalmente inimputáveis os menores II - igualdade na relação processual, podendo con-
de dezoito anos, sujeitos às medidas previstas nesta Lei. frontar-se com vítimas e testemunhas e produzir todas as
Parágrafo único. Para os efeitos desta Lei, deve ser con- provas necessárias à sua defesa;
siderada a idade do adolescente à data do fato. III - defesa técnica por advogado;
IV - assistência judiciária gratuita e integral aos
Art. 105. Ao ato infracional praticado por criança necessitados, na forma da lei;
corresponderão as medidas previstas no art. 101. V - direito de ser ouvido pessoalmente pela autorida-
de competente;
Capítulo II VI - direito de solicitar a presença de seus pais ou
Dos Direitos Individuais responsável em qualquer fase do procedimento.

Art. 106. Nenhum adolescente será privado de sua Capítulo IV


liberdade senão em flagrante de ato infracional ou Das Medidas Socioeducativas
por ordem escrita e fundamentada da autoridade judi-
ciária competente. Seção I
Parágrafo único. O adolescente tem direito à identifi- Disposições Gerais
cação dos responsáveis pela sua apreensão, devendo
ser informado acerca de seus direitos. Art. 112. Verificada a prática de ato infracional, a au-
toridade competente poderá aplicar ao adolescente as se-
LEGISLAÇÃO ESPECÍFICA

Art. 107. A apreensão de qualquer adolescente e o lo- guintes medidas:


cal onde se encontra recolhido serão incontinenti comuni- I - advertência;
cados à autoridade judiciária competente e à família do II - obrigação de reparar o dano;
apreendido ou à pessoa por ele indicada. III - prestação de serviços à comunidade;
Parágrafo único. Examinar-se-á, desde logo e sob pena IV - liberdade assistida;
de responsabilidade, a possibilidade de liberação ime- V - inserção em regime de semi-liberdade;
diata. VI - internação em estabelecimento educacional;
VII - qualquer uma das previstas no art. 101, I a VI.

113
§ 1º A medida aplicada ao adolescente levará em con- Seção III
ta a sua capacidade de cumpri-la, as circunstâncias e a Da Obrigação de Reparar o Dano
gravidade da infração.
§ 2º Em hipótese alguma e sob pretexto algum, será Art. 116. Em se tratando de ato infracional com re-
admitida a prestação de trabalho forçado. flexos patrimoniais, a autoridade poderá determinar, se
§ 3º Os adolescentes portadores de doença ou defi- for o caso, que o adolescente restitua a coisa, promova o
ciência mental receberão tratamento individual e es- ressarcimento do dano, ou, por outra forma, compense o
pecializado, em local adequado às suas condições. prejuízo da vítima.
Parágrafo único. Havendo manifesta impossibilidade, a
Art. 113. Aplica-se a este Capítulo o disposto nos arts. medida poderá ser substituída por outra adequada.
99 e 100.
Obrigação de reparar o dano (art. 116 do ECA). Há um
Art. 114. A imposição das medidas previstas nos in- pressuposto: o ato infracional deve ter causado um dano
cisos II a VI do art. 112 pressupõe a existência de provas à vítima. Essa reparação é para a vítima que sofreu o dano.
suficientes da autoria e da materialidade da infração, É uma medida voltada para o adolescente, então deve ser
ressalvada a hipótese de remissão, nos termos do art. 127. estabelecida de acordo com a possibilidade de cumpri-
Parágrafo único. A advertência poderá ser aplicada mento pelo adolescente (ex.: devolução da coisa furtada,
sempre que houver prova da materialidade e indícios sufi- pequenos serviços a título de reparação etc.).
cientes da autoria. A jurisprudência admite que essa reparação de dano
pode ser aplicada à criança (ex.: devolução da coisa fur-
tada).
As medidas socioeducativas dependem de um proce-
dimento judicial, só podendo ser aplicadas pelo Juiz. O
Seção IV
ECA apresenta dois critérios genéricos para a aplicação
Da Prestação de Serviços à Comunidade
de medida socioeducativa:
- capacidade do adolescente para cumprir a medida; Art. 117. A prestação de serviços comunitários consiste
- circunstâncias e gravidade da infração. na realização de tarefas gratuitas de interesse geral,
A internação é uma exceção, existindo hipóteses le- por período não excedente a seis meses, junto a entida-
gais para sua aplicação. des assistenciais, hospitais, escolas e outros estabelecimen-
A medida de segurança não poderá ser aplicada ao tos congêneres, bem como em programas comunitários ou
adolescente, tendo em vista ser medida para maior de governamentais.
idade que apresenta periculosidade. No caso de adoles- Parágrafo único. As tarefas serão atribuídas conforme
cente doente mental, será aplicada medida de proteção, as aptidões do adolescente, devendo ser cumpridas durante
podendo ser requisitado tratamento médico. jornada máxima de oito horas semanais, aos sábados, do-
O Juiz poderá cumular medidas socioeducativas, des- mingos e feriados ou em dias úteis, de modo a não prejudi-
de que sejam compatíveis (ex.: prestação de serviço à car a frequência à escola ou à jornada normal de trabalho.
comunidade cumulada com reparação de danos). Com
exceção da internação, o Juiz poderá substituir as me- Disposta no art. 117 do ECA, o adolescente será obri-
didas socioeducativas de acordo com o caso concreto, gado a prestar serviços em benefício da coletividade. São
visto não haver taxatividade. tarefas gratuitas de interesse geral junto a entidades as-
Se o Promotor discordar com a medida socioeducati- sistenciais, hospitais, escolas ou estabelecimentos congê-
va aplicada, deverá entrar com recurso de apelação. Essa neres.
apelação do ECA possui juízo de retratação, ou seja, o Como a medida é mais gravosa, a lei fixa um prazo
Juiz pode voltar atrás na decisão. O Tribunal competente máximo de 6 meses para essa prestação e um máximo
para julgar essa apelação é o TJ. de 8 horas semanais. Essas 8 horas poderão ser esta-
belecidas discricionariamente, desde que não prejudi-
Seção II quem a frequência ao trabalho e à escola. Deverá ser
Da Advertência levada em conta a aptidão do adolescente para a apli-
cação da medida.
Art. 115. A advertência consistirá em admoestação
Seção V
verbal, que será reduzida a termo e assinada.
Da Liberdade Assistida
Disposta no art. 115 do ECA, é uma medida sócio-e-
Art. 118. A liberdade assistida será adotada sempre
ducativa que consiste em uma admoestação verbal que
LEGISLAÇÃO ESPECÍFICA

que se afigurar a medida mais adequada para o fim


é aplicada pelo Juiz ao adolescente e que é reduzida a de acompanhar, auxiliar e orientar o adolescente.
termo. É destinada a atos de menor gravidade. § 1º A autoridade designará pessoa capacitada para
Para a aplicação da advertência, o Juiz deve levar em acompanhar o caso, a qual poderá ser recomendada por
consideração a prova da materialidade e indícios sufi- entidade ou programa de atendimento.
cientes de autoria. É a única medida que o Juiz poderá § 2º A liberdade assistida será fixada pelo prazo míni-
aplicar fundamentando-se somente em indícios de au- mo de seis meses, podendo a qualquer tempo ser prorro-
toria. gada, revogada ou substituída por outra medida, ouvido
o orientador, o Ministério Público e o defensor.

114
Art. 119. Incumbe ao orientador, com o apoio e a su- Seção VII
pervisão da autoridade competente, a realização dos se- Da Internação
guintes encargos, entre outros:
I - promover socialmente o adolescente e sua família, Art. 121. A internação constitui medida privativa da
fornecendo-lhes orientação e inserindo-os, se necessário, liberdade, sujeita aos princípios de brevidade, excepcio-
em programa oficial ou comunitário de auxílio e assistên- nalidade e respeito à condição peculiar de pessoa em
cia social; desenvolvimento.
II - supervisionar a frequência e o aproveitamento es- § 1º Será permitida a realização de atividades exter-
colar do adolescente, promovendo, inclusive, sua matrícu- nas, a critério da equipe técnica da entidade, salvo ex-
la; pressa determinação judicial em contrário.
III - diligenciar no sentido da profissionalização do § 2º A medida não comporta prazo determinado, de-
adolescente e de sua inserção no mercado de trabalho; vendo sua manutenção ser reavaliada, mediante decisão
IV - apresentar relatório do caso. fundamentada, no máximo a cada seis meses.
§ 3º Em nenhuma hipótese o período máximo de in-
É a última medida em que o adolescente permanece ternação excederá a três anos.
com sua família. O Juiz irá determinar um acompanha- § 4º Atingido o limite estabelecido no parágrafo an-
mento permanente ao adolescente, designando, para terior, o adolescente deverá ser liberado, colocado em
isso, um orientador, que poderá ser substituído a qual- regime de semiliberdade ou de liberdade assistida.
quer tempo. A lei fixa um prazo mínimo de 6 meses para § 5º A liberação será compulsória aos vinte e um anos
a duração dessa medida. O orientador terá as seguintes de idade.
obrigações legais: § 6º Em qualquer hipótese a desinternação será pre-
- promover socialmente o adolescente, bem como a cedida de autorização judicial, ouvido o Ministério Pú-
sua família, inserindo-os em programas sociais. Promo- blico.
ver socialmente é fazer com que o adolescente realize § 7o A determinação judicial mencionada no § 1o po-
atividades valorizadas socialmente (teatro, música etc.); derá ser revista a qualquer tempo pela autoridade judi-
- supervisionar a frequência e o aproveitamento es- ciária.
colar do adolescente;
- profissionalizar o adolescente (nos termos da EC n. 20); Art. 122. A medida de internação só poderá ser aplica-
- apresentar relatório do caso ao Juiz. da quando:
I - tratar-se de ato infracional cometido mediante gra-
Seção VI ve ameaça ou violência a pessoa;
Do Regime de Semiliberdade II - por reiteração no cometimento de outras infrações
graves;
Art. 120. O regime de semiliberdade pode ser determi- III - por descumprimento reiterado e injustificável
nado desde o início, ou como forma de transição para o da medida anteriormente imposta.
meio aberto, possibilitada a realização de atividades ex- § 1o O prazo de internação na hipótese do inciso III
ternas, independentemente de autorização judicial. deste artigo não poderá ser superior a 3 (três) meses,
§ 1º São obrigatórias a escolarização e a profissio- devendo ser decretada judicialmente após o devido pro-
nalização, devendo, sempre que possível, ser utilizados os cesso legal.
recursos existentes na comunidade. § 2º Em nenhuma hipótese será aplicada a internação,
§ 2º A medida não comporta prazo determinado havendo outra medida adequada.
aplicando-se, no que couber, as disposições relativas à in-
ternação. Art. 123. A internação deverá ser cumprida em entida-
de exclusiva para adolescentes, em local distinto daquele
Disposta no art. 120 do ECA, é uma medida que destinado ao abrigo, obedecida rigorosa separação por cri-
importa em privação de liberdade ao adolescente que térios de idade, compleição física e gravidade da infração.
pratica um ato infracional mais grave. O adolescente é Parágrafo único. Durante o período de internação, in-
retirado de sua família e colocado em um estabeleci- clusive provisória, serão obrigatórias atividades pedagó-
mento apropriado de semiliberdade, podendo realizar gicas.
atividades externas (estudar, trabalhar etc.) somente com
autorização do diretor do estabelecimento, não havendo Art. 124. São direitos do adolescente privado de liber-
necessidade de autorização judicial. Pode ser usada tanto dade, entre outros, os seguintes:
como medida principal quanto como medida progressiva I - entrevistar-se pessoalmente com o representante do
ou regressiva.
LEGISLAÇÃO ESPECÍFICA

Ministério Público;
A semiliberdade não tem prazo fixado em lei, nem II - peticionar diretamente a qualquer autoridade;
mínimo nem máximo. A doutrina e a jurisprudência de- III - avistar-se reservadamente com seu defensor;
terminam a aplicação da medida por analogia dos prazos IV - ser informado de sua situação processual, sempre
da internação, tendo como prazo máximo 3 anos. Há a que solicitada;
obrigatoriedade de escolarização e profissionalização na V - ser tratado com respeito e dignidade;
semiliberdade. VI - permanecer internado na mesma localidade ou
naquela mais próxima ao domicílio de seus pais ou res-
ponsável;

115
VII - receber visitas, ao menos, semanalmente; O art. 123 dispõe que o local para a internação deve
VIII - corresponder-se com seus familiares e amigos; ser distinto do abrigo, devendo-se obedecer a separação
IX - ter acesso aos objetos necessários à higiene e as- por idade, composição física (tamanho), sexo e gravida-
seio pessoal; de do ato infracional. Há, também, a obrigatoriedade de
X - habitar alojamento em condições adequadas de hi- realização de atividades pedagógicas.
giene e salubridade; O art. 124 dispõe sobre direitos específicos dos ado-
XI - receber escolarização e profissionalização; lescentes:
XII - realizar atividades culturais, esportivas e de lazer: - entrevista pessoal com o representante do MP;
XIII - ter acesso aos meios de comunicação social; - entrevista reservada com seu defensor, dentre ou-
XIV - receber assistência religiosa, segundo a sua cren- tros.
ça, e desde que assim o deseje; As visitas podem ser suspensas pelo juiz, sob o fun-
XV - manter a posse de seus objetos pessoais e dispor damento de segurança e proteção do menor, entretan-
de local seguro para guardá-los, recebendo comprovante to, em nenhuma hipótese o menor poderá ficar inco-
daqueles porventura depositados em poder da entidade; municável.
XVI - receber, quando de sua desinternação, os docu-
mentos pessoais indispensáveis à vida em sociedade. Capítulo V
§ 1º Em nenhum caso haverá incomunicabilidade. Da Remissão
§ 2º A autoridade judiciária poderá suspender tempo-
rariamente a visita, inclusive de pais ou responsável, se Art. 126. Antes de iniciado o procedimento judicial
existirem motivos sérios e fundados de sua prejudiciali- para apuração de ato infracional, o representante do
dade aos interesses do adolescente. Ministério Público poderá conceder a remissão, como
forma de exclusão do processo, atendendo às circuns-
Art. 125. É dever do Estado zelar pela integridade físi- tâncias e consequências do fato, ao contexto social, bem
ca e mental dos internos, cabendo-lhe adotar as medidas como à personalidade do adolescente e sua maior ou me-
adequadas de contenção e segurança.
nor participação no ato infracional.
Parágrafo único. Iniciado o procedimento, a conces-
Disposta no art. 121 e seguintes do ECA, é a medida
são da remissão pela autoridade judiciária importará na
reservada para os atos infracionais de natureza grave. O
suspensão ou extinção do processo.
ECA estabelece princípios específicos para a internação,
pois é medida de privação de liberdade sempre excep-
Art. 127. A remissão não implica necessariamente
cional.
o reconhecimento ou comprovação da responsabili-
A internação deve durar o menor tempo possível
dade, nem prevalece para efeito de antecedentes, po-
(princípio da brevidade), é uma medida de exceção que
dendo incluir eventualmente a aplicação de qualquer das
só deverá ser utilizada em último caso (princípio da ex-
cepcionalidade) e deve seguir o princípio do respeito à medidas previstas em lei, exceto a colocação em regime
condição peculiar do adolescente como pessoa em de- de semi-liberdade e a internação.
senvolvimento. Em nenhuma hipótese pode ser aplicada
à criança. Art. 128. A medida aplicada por força da remissão
O ECA estabelece hipóteses de internação para: poderá ser revista judicialmente, a qualquer tempo,
- prática de ato infracional mediante grave ameaça ou mediante pedido expresso do adolescente ou de seu re-
violência à pessoa; presentante legal, ou do Ministério Público.
- reiteração de infrações graves;
- descumprimento reiterado e injustificado da medi- Tem por conceito o perdão, a indulgência ao menor.
da anteriormente imposta (é uma hipótese de regressão). Podem conceder remissão tanto o MP quanto o Juiz.
Neste caso, a internação não pode ultrapassar o prazo São hipóteses de natureza jurídica diferentes. A remis-
de 3 meses. são judicial é forma de extinção ou de suspensão do
Nas duas primeiras hipóteses, o prazo máximo para processo (portanto, pressupõe o processo em curso). Já
internação é de 3 anos. Por força desse prazo, o ECA po- a remissão ministerial é forma de exclusão do processo
derá atingir o maior de 18 anos. Em rigor, todas as medi- (logo, deve ser concedida antes do processo - adminis-
das sócio-educativas poderão atingir o maior de 18 anos. trativamente). Quando a remissão é concedida pelo MP,
A medida só poderá ser aplicada com o devido pro- segue-se o seguinte procedimento:
cesso legal e em nenhuma hipótese poderá ser aplica- - o menor é ouvido pelo Promotor que concederá a
da à criança. Quando o adolescente completar 21 anos, remissão;
LEGISLAÇÃO ESPECÍFICA

a liberação será obrigatória. Caso o adolescente tenha - o Promotor encaminha a remissão para homologa-
passado por internação provisória, esses dias serão com- ção pelo Juiz;
putados na internação (detração). A diferença entre se- - se o Juiz não aceitar a remissão, deverá remeter
mi-liberdade e internação é que, nesta, o adolescente para o Procurador de Justiça, que poderá insistir na re-
depende de autorização expressa do juiz para praticar missão ou designar outro representante do MP para
atividades externas, ou seja, o adolescente internado so- apresentar representação contra o menor. Essa remissão
mente se ausentará do estabelecimento em que se achar concedida pelo MP é causa de exclusão do processo,
se autorizado pelo juiz. visto que, ao conceder a remissão, inexiste o processo.

116
Quando a remissão é concedida pelo Juiz, segue-se Parágrafo único. Da medida cautelar constará, ain-
o seguinte procedimento: da, a fixação provisória dos alimentos de que necessitem a
- o Promotor oferece a representação; criança ou o adolescente dependentes do agressor.
- na audiência de apresentação, o menor será ouvido
pelo Juiz, que poderá decidir pela remissão; Capítulo I
- o representante do MP deverá, obrigatoriamente, Disposições Gerais
ser ouvido sobre a possibilidade da remissão antes de
ela ser aplicada. A remissão concedida pelo Juiz causa Art. 131. O Conselho Tutelar é órgão permanente e
extinção do processo. Havendo discordância por parte autônomo, não jurisdicional, encarregado pela sociedade
do MP, este deverá ingressar com uma apelação para re- de zelar pelo cumprimento dos direitos da criança e do
formar a decisão do Juiz. adolescente, definidos nesta Lei.
Tanto a doutrina quanto a jurisprudência admitem a
cumulação da remissão com uma medida sócio-educa- Art. 132. Em cada Município e em cada Região Admi-
tiva que seja compatível (ex.: reparação do dano, adver-
nistrativa do Distrito Federal haverá, no mínimo, 1 (um)
tência etc.). Neste caso, a remissão é causa de suspensão
Conselho Tutelar como órgão integrante da administra-
do processo.
ção pública local, composto de 5 (cinco) membros, esco-
O ECA traz quatro requisitos genéricos para a aplica-
lhidos pela população local para mandato de 4 (quatro)
ção da remissão, devendo ficar a critério do membro do
MP ou do Juiz a sua concessão. São eles: anos, permitida 1 (uma) recondução, mediante novo
- circunstâncias e conseqüências do fato; processo de escolha.
- contexto social em que o fato foi praticado;
- personalidade do agente; Art. 133. Para a candidatura a membro do Conselho
- maior ou menor participação no ato infracional. Tutelar, serão exigidos os seguintes requisitos:
A remissão, quer concedida pelo MP quer pelo Juiz, I - reconhecida idoneidade moral;
não implica confissão de culpa. Existe uma divergência na II - idade superior a vinte e um anos;
doutrina em considerar a remissão como um acordo ou III - residir no município.
não. A posição majoritária entende que a remissão não
é um acordo, tendo em vista a lei falar em concessão e, Art. 134. Lei municipal ou distrital disporá sobre
ainda, pelo fato de não haver nenhum prejuízo para o o local, dia e horário de funcionamento do Conselho
adolescente, não possuindo a remissão nenhum efeito, Tutelar, inclusive quanto à remuneração dos respectivos
podendo ser concedida quantas vezes forem necessárias. membros, aos quais é assegurado o direito a:
I - cobertura previdenciária;
Título IV II - gozo de férias anuais remuneradas, acrescidas de
Das Medidas Pertinentes aos Pais ou Responsável 1/3 (um terço) do valor da remuneração mensal;
III - licença-maternidade;
Art. 129. São medidas aplicáveis aos pais ou respon- IV - licença-paternidade;
sável: V - gratificação natalina.
I - encaminhamento a serviços e programas oficiais ou Parágrafo único. Constará da lei orçamentária muni-
comunitários de proteção, apoio e promoção da família; cipal e da do Distrito Federal previsão dos recursos neces-
II - inclusão em programa oficial ou comunitário de sários ao funcionamento do Conselho Tutelar e à remune-
auxílio, orientação e tratamento a alcoólatras e toxicôma- ração e formação continuada dos conselheiros tutelares.
nos;
III - encaminhamento a tratamento psicológico ou psi-
Art. 135. O exercício efetivo da função de conselheiro
quiátrico;
constituirá serviço público relevante e estabelecerá pre-
IV - encaminhamento a cursos ou programas de orien-
sunção de idoneidade moral.
tação;
V - obrigação de matricular o filho ou pupilo e acom-
panhar sua frequência e aproveitamento escolar;
Capítulo II
VI - obrigação de encaminhar a criança ou adolescente
Das Atribuições do Conselho
a tratamento especializado;
VII - advertência; Art. 136. São atribuições do Conselho Tutelar:
VIII - perda da guarda; I - atender as crianças e adolescentes nas hipóteses
IX - destituição da tutela; previstas nos arts. 98 e 105, aplicando as medidas previs-
tas no art. 101, I a VII;
LEGISLAÇÃO ESPECÍFICA

X - suspensão ou destituição do poder familiar.


Parágrafo único. Na aplicação das medidas previstas II - atender e aconselhar os pais ou responsável, apli-
nos incisos IX e X deste artigo, observar-se-á o disposto cando as medidas previstas no art. 129, I a VII;
nos arts. 23 e 24. III - promover a execução de suas decisões, podendo
para tanto:
Art. 130. Verificada a hipótese de maus-tratos, opres- a) requisitar serviços públicos nas áreas de saúde, edu-
são ou abuso sexual impostos pelos pais ou responsável, cação, serviço social, previdência, trabalho e segurança;
a autoridade judiciária poderá determinar, como medida b) representar junto à autoridade judiciária nos casos
cautelar, o afastamento do agressor da moradia comum. de descumprimento injustificado de suas deliberações.

117
IV - encaminhar ao Ministério Público notícia de fato Capítulo V
que constitua infração administrativa ou penal contra os Dos Impedimentos
direitos da criança ou adolescente;
V - encaminhar à autoridade judiciária os casos de sua Art. 140. São impedidos de servir no mesmo Con-
competência; selho marido e mulher, ascendentes e descendentes,
VI - providenciar a medida estabelecida pela autorida- sogro e genro ou nora, irmãos, cunhados, durante o
de judiciária, dentre as previstas no art. 101, de I a VI, para cunhadio, tio e sobrinho, padrasto ou madrasta e en-
o adolescente autor de ato infracional; teado.
VII - expedir notificações; Parágrafo único. Estende-se o impedimento do conse-
VIII - requisitar certidões de nascimento e de óbito de lheiro, na forma deste artigo, em relação à autoridade ju-
criança ou adolescente quando necessário; diciária e ao representante do Ministério Público com atua-
IX - assessorar o Poder Executivo local na elaboração ção na Justiça da Infância e da Juventude, em exercício na
da proposta orçamentária para planos e programas de comarca, foro regional ou distrital.
atendimento dos direitos da criança e do adolescente;
X - representar, em nome da pessoa e da família, con- Título VI
tra a violação dos direitos previstos no art. 220, § 3º, inciso Do Acesso à Justiça
II, da Constituição Federal;
XI - representar ao Ministério Público para efeito das Capítulo I
ações de perda ou suspensão do poder familiar, após esgo- Disposições Gerais
tadas as possibilidades de manutenção da criança ou do
adolescente junto à família natural; Art. 141. É garantido o acesso de toda criança ou
XII - promover e incentivar, na comunidade e nos gru- adolescente à Defensoria Pública, ao Ministério Públi-
pos profissionais, ações de divulgação e treinamento para co e ao Poder Judiciário, por qualquer de seus órgãos.
o reconhecimento de sintomas de maus-tratos em crian- § 1º. A assistência judiciária gratuita será prestada aos
ças e adolescentes. que dela necessitarem, através de defensor público ou ad-
Parágrafo único. Se, no exercício de suas atribuições, vogado nomeado.
o Conselho Tutelar entender necessário o afastamento do § 2º As ações judiciais da competência da Justiça da
convívio familiar, comunicará incontinenti o fato ao Mi- Infância e da Juventude são isentas de custas e emolu-
nistério Público, prestando-lhe informações sobre os moti- mentos, ressalvada a hipótese de litigância de má-fé.
vos de tal entendimento e as providências tomadas para a
Art. 142. Os menores de dezesseis anos serão repre-
orientação, o apoio e a promoção social da família.
sentados e os maiores de dezesseis e menores de vinte
e um anos assistidos por seus pais, tutores ou curadores,
Art. 137. As decisões do Conselho Tutelar somente po-
na forma da legislação civil ou processual.
derão ser revistas pela autoridade judiciária a pedido
Parágrafo único. A autoridade judiciária dará curador
de quem tenha legítimo interesse.
especial à criança ou adolescente, sempre que os interes-
ses destes colidirem com os de seus pais ou responsável,
Capítulo III ou quando carecer de representação ou assistência legal
Da Competência ainda que eventual.
Art. 138. Aplica-se ao Conselho Tutelar a regra de com- Art. 143. E vedada a divulgação de atos judiciais, po-
petência constante do art. 147. liciais e administrativos que digam respeito a crianças e
adolescentes a que se atribua autoria de ato infracional.
Capítulo IV Parágrafo único. Qualquer notícia a respeito do fato
Da Escolha dos Conselheiros não poderá identificar a criança ou adolescente, vedando-
-se fotografia, referência a nome, apelido, filiação, paren-
Art. 139. O processo para a escolha dos membros do tesco, residência e, inclusive, iniciais do nome e sobrenome.
Conselho Tutelar será estabelecido em lei municipal e
realizado sob a responsabilidade do Conselho Munici- Art. 144. A expedição de cópia ou certidão de atos a
pal dos Direitos da Criança e do Adolescente, e a fis- que se refere o artigo anterior somente será deferida pela
calização do Ministério Público. autoridade judiciária competente, se demonstrado o inte-
§ 1o O processo de escolha dos membros do Conse- resse e justificada a finalidade.
lho Tutelar ocorrerá em data unificada em todo o territó-
rio nacional a cada 4 (quatro) anos, no primeiro domingo O homem necessita do convívio social, não é um ser
do mês de outubro do ano subsequente ao da eleição
LEGISLAÇÃO ESPECÍFICA

capaz de viver de maneira autônoma e totalmente des-


presidencial. vinculada dos demais. Neste sentido, a imposição de re-
§ 2o A posse dos conselheiros tutelares ocorrerá no dia gramentos e normas permitiu que a sociedade atingisse
10 de janeiro do ano subsequente ao processo de escolha. o atual grau de evolução.
§ 3o No processo de escolha dos membros do Conse- Obviamente, no ambiente social surgem conflitos de
lho Tutelar, é vedado ao candidato doar, oferecer, prome- interesses. Afinal, nem sempre os bens e valores existem
ter ou entregar ao eleitor bem ou vantagem pessoal de em quantidade suficiente para atender a todas as pessoas.
qualquer natureza, inclusive brindes de pequeno valor. Inicialmente, estes conflitos eram solucionados pelos pró-
prios envolvidos, na denominada fase da autotutela.

118
Contudo, a solução possibilidade pela autotutela era A noção de jurisdição inclusiva também se aplica
bastante insatisfatória e fazia com que prevalecesse a lei à tutela jurisdicional da criança e do adolescente. Basi-
do mais forte. Então, surgiu o Estado apresentando um camente, refere-se à propiciação de uma jurisdição que
melhor sistema para a solução dos conflitos. esteja atenta às peculiaridades das minorias e dos grupos
O Estado assumiu para si o poder-dever de dizer o Di- vulneráveis. No caso, as crianças e adolescentes são con-
reito, de solucionar os conflitos, conhecido como jurisdi- siderados um grupo vulnerável devido à condição espe-
ção. Assim, o Estado irá elaborar as leis (direito material) cial que ocupam.
e prever como elas serão aplicadas (direito processual). A
autotutela para a ser punida como regra geral e o Estado Capítulo II
exerce a heterotutela por meio da atividade jurisdicional. Da Justiça da Infância e da Juventude
Jurisdição é o poder-dever do Estado de dizer o Di-
reito. Sendo assim, trata-se de atividade estatal exercida Seção I
por intermédio de um agente constituído com compe- Disposições Gerais
tência para exercê-la, o juiz.
Nos primórdios da humanidade não existia o Direito e Art. 145. Os estados e o Distrito Federal poderão criar
nem existiam as leis, de modo que a justiça era feita pelas varas especializadas e exclusivas da infância e da juven-
próprias mãos, na denominada autotutela. Com a evolu- tude, cabendo ao Poder Judiciário estabelecer sua pro-
ção das instituições, o Estado avocou para si o poder-de- porcionalidade por número de habitantes, dotá-las de in-
ver de solucionar os litígios, o que é feito pela jurisdição. fra-estrutura e dispor sobre o atendimento, inclusive em
O poder-dever de dizer o direito é uno, apenas exis- plantões.
tindo uma separação de funções: o Legislativo regula-
menta normas gerais e abstratas (função legislativa) e o Seção II
Judiciário as aplica no caso concreto (função jurisdicio- Do Juiz
nal).
Entretanto, vale destacar que na sociedade contem- Art. 146. A autoridade a que se refere esta Lei é o Juiz
porânea, devido às inúmeras mazelas que se apresen- da Infância e da Juventude, ou o juiz que exerce essa fun-
taram envolvendo o abarrotamento de processos pelo ção, na forma da lei de organização judiciária local.
Judiciário, passou-se a incentivar a adoção de métodos
de autocomposição, como conciliação, mediação e arbi- Art. 147. A competência será determinada:
tragem. I - pelo domicílio dos pais ou responsável;
Tradicionalmente, são enumerados pela doutrina os II - pelo lugar onde se encontre a criança ou adolescen-
seguintes princípios inerentes à jurisdição: investidura, te, à falta dos pais ou responsável.
porque somente exerce jurisdição quem ocupa o cargo § 1º. Nos casos de ato infracional, será competente a
de juiz; aderência ao território, posto que juízes somente
autoridade do lugar da ação ou omissão, observadas as
têm autoridade no território nacional e nos limites de sua
regras de conexão, continência e prevenção.
competência; indelegabilidade, não podendo o Poder Ju-
§ 2º A execução das medidas poderá ser delega-
diciário delegar sua competência; inafastabilidade, pois
da à autoridade competente da residência dos pais ou
a lei não pode excluir da apreciação do Poder Judiciário
responsável, ou do local onde sediar-se a entidade que
nenhuma lesão ou ameaça a direito.
abrigar a criança ou adolescente.
Embora a jurisdição seja una, em termos doutriná-
§ 3º Em caso de infração cometida através de trans-
rios é possível classificá-la: a) quanto ao objeto – penal,
missão simultânea de rádio ou televisão, que atinja mais
trabalhista e civil (a civil é subsidiária, envolvendo todo
direito material que não seja penal ou trabalhista, não de uma comarca, será competente, para aplicação da pe-
somente questões inerentes ao direito civil); b) quanto nalidade, a autoridade judiciária do local da sede esta-
ao organismo que a exerce – comum (estadual ou fede- dual da emissora ou rede, tendo a sentença eficácia para
ral) ou especial (trabalhista, militar, eleitoral); c) quanto à todas as transmissoras ou retransmissoras do respectivo
hierarquia – superior e inferior. estado.
Neste sentido, com vistas a instrumentalizar a jurisdi-
ção, impedindo que ela seja exercida de maneira caótica, Art. 148. A Justiça da Infância e da Juventude é com-
ela é distribuída entre juízos e foros (órgãos competen- petente para:
tes em localidades determinadas). A esta distribuição das I - conhecer de representações promovidas pelo Minis-
parcelas de jurisdição dá-se o nome de competência. tério Público, para apuração de ato infracional atribuído a
As tutelas jurisdicionais diferenciadas, por sua vez, adolescente, aplicando as medidas cabíveis;
são aquelas que apresentam procedimentos diversos do II - conceder a remissão, como forma de suspensão ou
LEGISLAÇÃO ESPECÍFICA

comum. Possuem procedimentos ditos especiais, os quais extinção do processo;


buscam garantir um processo mais rápido e compatível III - conhecer de pedidos de adoção e seus incidentes;
com as necessidades específicas do direito em discussão. IV - conhecer de ações civis fundadas em interesses in-
No âmbito do direito da criança e do adolescente, tem-se dividuais, difusos ou coletivos afetos à criança e ao adoles-
o estabelecimento de uma tutela jurisdicional diferencia- cente, observado o disposto no art. 209;
da, eis que existem inúmeras regras específicas aplicáveis V - conhecer de ações decorrentes de irregularidades
aos processos que envolvem de algum modo criança ou em entidades de atendimento, aplicando as medidas ca-
adolescente. bíveis;

119
VI - aplicar penalidades administrativas nos casos de Art. 151. Compete à equipe interprofissional dentre ou-
infrações contra norma de proteção à criança ou adoles- tras atribuições que lhe forem reservadas pela legislação
cente; local, fornecer subsídios por escrito, mediante laudos, ou
VII - conhecer de casos encaminhados pelo Conselho verbalmente, na audiência, e bem assim desenvolver tra-
Tutelar, aplicando as medidas cabíveis. balhos de aconselhamento, orientação, encaminhamento,
Parágrafo único. Quando se tratar de criança ou ado- prevenção e outros, tudo sob a imediata subordinação à
lescente nas hipóteses do art. 98, é também competente a autoridade judiciária, assegurada a livre manifestação do
Justiça da Infância e da Juventude para o fim de: ponto de vista técnico.
a) conhecer de pedidos de guarda e tutela; Parágrafo único. Na ausência ou insuficiência de servi-
b) conhecer de ações de destituição do poder familiar, dores públicos integrantes do Poder Judiciário responsáveis
perda ou modificação da tutela ou guarda; pela realização dos estudos psicossociais ou de quaisquer
c) suprir a capacidade ou o consentimento para o ca- outras espécies de avaliações técnicas exigidas por esta Lei
samento; ou por determinação judicial, a autoridade judiciária po-
d) conhecer de pedidos baseados em discordância pa- derá proceder à nomeação de perito, nos termos do art.
terna ou materna, em relação ao exercício do poder fa- 156 da Lei no 13.105, de 16 de março de 2015 (Código de
miliar; Processo Civil).
e) conceder a emancipação, nos termos da lei civil,
quando faltarem os pais; Capítulo III
f) designar curador especial em casos de apresentação Dos Procedimentos
de queixa ou representação, ou de outros procedimentos
judiciais ou extrajudiciais em que haja interesses de crian- Seção I
ça ou adolescente; Disposições Gerais
g) conhecer de ações de alimentos;
h) determinar o cancelamento, a retificação e o supri- Art. 152. Aos procedimentos regulados nesta Lei apli-
mento dos registros de nascimento e óbito. cam-se subsidiariamente as normas gerais previstas na
legislação processual pertinente.
§ 1o É assegurada, sob pena de responsabilidade, prio-
Art. 149. Compete à autoridade judiciária disciplinar,
ridade absoluta na tramitação dos processos e proce-
através de portaria, ou autorizar, mediante alvará:
dimentos previstos nesta Lei, assim como na execução dos
I - a entrada e permanência de criança ou adolescente,
atos e diligências judiciais a eles referentes.
desacompanhado dos pais ou responsável, em:
§ 2º Os prazos estabelecidos nesta Lei e aplicáveis aos
a) estádio, ginásio e campo desportivo;
seus procedimentos são contados em dias corridos, ex-
b) bailes ou promoções dançantes;
cluído o dia do começo e incluído o dia do vencimento,
c) boate ou congêneres;
vedado o prazo em dobro para a Fazenda Pública e o Mi-
d) casa que explore comercialmente diversões eletrô-
nistério Público.
nicas;
e) estúdios cinematográficos, de teatro, rádio e televi- Art. 153. Se a medida judicial a ser adotada não cor-
são. responder a procedimento previsto nesta ou em outra lei,
II - a participação de criança e adolescente em: a autoridade judiciária poderá investigar os fatos e or-
a) espetáculos públicos e seus ensaios; denar de ofício as providências necessárias, ouvido o
b) certames de beleza. Ministério Público.
§ 1º Para os fins do disposto neste artigo, a autorida- Parágrafo único. O disposto neste artigo não se apli-
de judiciária levará em conta, dentre outros fatores: ca para o fim de afastamento da criança ou do adolescen-
a) os princípios desta Lei; te de sua família de origem e em outros procedimentos
b) as peculiaridades locais; necessariamente contenciosos.
c) a existência de instalações adequadas;
d) o tipo de freqüência habitual ao local; Art. 154. Aplica-se às multas o disposto no art. 214.
e) a adequação do ambiente a eventual participação
ou freqüência de crianças e adolescentes; A tutela sócio-individual abrange aspectos do direito
f) a natureza do espetáculo. da criança e do adolescente voltado à criança e ao ado-
§ 2º As medidas adotadas na conformidade deste ar- lescente individualmente concebidos, isto é, pensados
tigo deverão ser fundamentadas, caso a caso, vedadas as como sujeitos de direitos individuais que possam ser por
determinações de caráter geral. eles exercidos.
A tutela sócio-educativa abrange aspectos do direito
LEGISLAÇÃO ESPECÍFICA

Seção III da criança e do adolescente voltados às atividades de en-


Dos Serviços Auxiliares sino e aprendizagem, tanto no que se refere à educação
formal quanto em relação à educação informal.
Art. 150. Cabe ao Poder Judiciário, na elaboração de A tutela coletiva volta-se à proteção de direitos di-
sua proposta orçamentária, prever recursos para manu- fusos e coletivos da criança e do adolescente. Aos di-
tenção de equipe interprofissional, destinada a assessorar reitos difusos e coletivos são conferidos mecanismos de
a Justiça da Infância e da Juventude. tutela específicos para sua proteção, bem como atribuí-
da competência para tanto a órgãos determinados que

120
exercerão um papel representativo. No Brasil, destacam- § 1º A citação será pessoal, salvo se esgotados todos
-se instituições como o Ministério Público e a Defensoria os meios para sua realização.
Pública. Sem prejuízo, como visto, há remédios consti- § 2º O requerido privado de liberdade deverá ser ci-
tucionais que se voltam à proteção de interesses desta tado pessoalmente.
categoria, como o mandado de segurança coletivo e a § 3o Quando, por 2 (duas) vezes, o oficial de justiça
própria ação popular, sem falar na ação civil pública, tam- houver procurado o citando em seu domicílio ou residên-
bém mencionada no texto constitucional. cia sem o encontrar, deverá, havendo suspeita de ocul-
Considerados os diferentes tipos de tutelas inseridas tação, informar qualquer pessoa da família ou, em sua
no direito da criança e do adolescente, justifica-se a tute- falta, qualquer vizinho do dia útil em que voltará a fim de
la jurisdicional diferenciada, adaptada à condição em de- efetuar a citação, na hora que designar, nos termos do art.
senvolvimento da criança e do adolescente, que deve ser 252 e seguintes da Lei no 13.105, de 16 de março de 2015
ágil, efetiva, atenta às peculiaridades do caso concreto. (Código de Processo Civil).
Os procedimentos especiais do ECA se referem a: per- § 4o Na hipótese de os genitores encontrarem-se em
da e suspensão de poder familiar, destituição de tutela,
local incerto ou não sabido, serão citados por edital no
colocação em família substituta, apuração de ato infra-
prazo de 10 (dez) dias, em publicação única, dispensado o
cional atribuído a adolescente, apuração de irregularida-
envio de ofícios para a localização.
des em atendimento, apuração de infração administrati-
va às normas de proteção da criança e do adolescente e
habilitação em adoção. Art. 159. Se o requerido não tiver possibilidade de
constituir advogado, sem prejuízo do próprio sustento e
Seção II de sua família, poderá requerer, em cartório, que lhe seja
Da Perda e da Suspensão do Poder Familiar nomeado dativo, ao qual incumbirá a apresentação de
resposta, contando-se o prazo a partir da intimação do
Art. 155. O procedimento para a perda ou a suspen- despacho de nomeação.
são do poder familiar terá início por provocação do Mi- Parágrafo único. Na hipótese de requerido privado de
nistério Público ou de quem tenha legítimo interesse. liberdade, o oficial de justiça deverá perguntar, no mo-
mento da citação pessoal, se deseja que lhe seja nomeado
Art. 156. A petição inicial indicará: defensor.
I - a autoridade judiciária a que for dirigida;
II - o nome, o estado civil, a profissão e a residência Art. 160. Sendo necessário, a autoridade judiciária re-
do requerente e do requerido, dispensada a qualificação quisitará de qualquer repartição ou órgão público a apre-
em se tratando de pedido formulado por representante do sentação de documento que interesse à causa, de ofício
Ministério Público; ou a requerimento das partes ou do Ministério Público.
III - a exposição sumária do fato e o pedido;
IV - as provas que serão produzidas, oferecendo, desde Art. 161. Se não for contestado o pedido e tiver sido
logo, o rol de testemunhas e documentos. concluído o estudo social ou a perícia realizada por equi-
pe interprofissional ou multidisciplinar, a autoridade ju-
Art. 157. Havendo motivo grave, poderá a autorida- diciária dará vista dos autos ao Ministério Público,
de judiciária, ouvido o Ministério Público, decretar a sus- por 5 (cinco) dias, salvo quando este for o requerente, e
pensão do poder familiar, liminar ou incidentalmente, decidirá em igual prazo.
até o julgamento definitivo da causa, ficando a criança ou § 1º A autoridade judiciária, de ofício ou a requeri-
adolescente confiado a pessoa idônea, mediante termo de mento das partes ou do Ministério Público, determinará
responsabilidade.
a oitiva de testemunhas que comprovem a presença de
§ 1o Recebida a petição inicial, a autoridade judiciária
uma das causas de suspensão ou destituição do poder
determinará, concomitantemente ao despacho de citação
familiar previstas nos arts. 1.637 e 1.638 da Lei no 10.406,
e independentemente de requerimento do interessado, a
de 10 de janeiro de 2002 (Código Civil), ou no art. 24
realização de estudo social ou perícia por equipe interpro-
fissional ou multidisciplinar para comprovar a presença de desta Lei.
uma das causas de suspensão ou destituição do poder fa- § 2o (Revogado).
miliar, ressalvado o disposto no § 10 do art. 101 desta Lei, § 3o Se o pedido importar em modificação de guarda,
e observada a Lei no 13.431, de 4 de abril de 2017. será obrigatória, desde que possível e razoável, a oitiva da
§ 2o Em sendo os pais oriundos de comunidades indí- criança ou adolescente, respeitado seu estágio de desen-
genas, é ainda obrigatória a intervenção, junto à equipe volvimento e grau de compreensão sobre as implicações
da medida.
LEGISLAÇÃO ESPECÍFICA

interprofissional ou multidisciplinar referida no § 1o des-


te artigo, de representantes do órgão federal responsável § 4º É obrigatória a oitiva dos pais sempre que eles
pela política indigenista, observado o disposto no § 6o do forem identificados e estiverem em local conhecido,
art. 28 desta Lei. ressalvados os casos de não comparecimento perante a
Justiça quando devidamente citados.
Art. 158. O requerido será citado para, no prazo de § 5o Se o pai ou a mãe estiverem privados de liberdade,
dez dias, oferecer resposta escrita, indicando as provas a a autoridade judicial requisitará sua apresentação para a
serem produzidas e oferecendo desde logo o rol de teste- oitiva.
munhas e documentos.

121
Art. 162. Apresentada a resposta, a autoridade judi- Art. 166. Se os pais forem falecidos, tiverem sido des-
ciária dará vista dos autos ao Ministério Público, por tituídos ou suspensos do poder familiar, ou houverem
cinco dias, salvo quando este for o requerente, designan- aderido expressamente ao pedido de colocação em família
do, desde logo, audiência de instrução e julgamento. substituta, este poderá ser formulado diretamente em cartó-
§ 1º (Revogado). rio, em petição assinada pelos próprios requerentes, dispensa-
§ 2o Na audiência, presentes as partes e o Ministério da a assistência de advogado.
Público, serão ouvidas as testemunhas, colhendo-se oral- § 1o Na hipótese de concordância dos pais, o juiz:
mente o parecer técnico, salvo quando apresentado por I - na presença do Ministério Público, ouvirá as partes, de-
escrito, manifestando-se sucessivamente o requerente, o vidamente assistidas por advogado ou por defensor público,
requerido e o Ministério Público, pelo tempo de 20 (vinte) para verificar sua concordância com a adoção, no prazo má-
minutos cada um, prorrogável por mais 10 (dez) minutos. ximo de 10 (dez) dias, contado da data do protocolo da peti-
§ 3o A decisão será proferida na audiência, podendo ção ou da entrega da criança em juízo, tomando por termo as
a autoridade judiciária, excepcionalmente, designar data declarações; e
para sua leitura no prazo máximo de 5 (cinco) dias. II - declarará a extinção do poder familiar.
§ 4o Quando o procedimento de destituição de poder § 2o O consentimento dos titulares do poder familiar será
familiar for iniciado pelo Ministério Público, não haverá precedido de orientações e esclarecimentos prestados pela
necessidade de nomeação de curador especial em favor da equipe interprofissional da Justiça da Infância e da Juventude,
criança ou adolescente. em especial, no caso de adoção, sobre a irrevogabilidade da
medida.
Art. 163. O prazo máximo para conclusão do pro- § 3o São garantidos a livre manifestação de vontade dos
cedimento será de 120 (cento e vinte) dias, e caberá detentores do poder familiar e o direito ao sigilo das infor-
ao juiz, no caso de notória inviabilidade de manutenção mações.
do poder familiar, dirigir esforços para preparar a criança § 4o O consentimento prestado por escrito não terá vali-
ou o adolescente com vistas à colocação em família subs- dade se não for ratificado na audiência a que se refere o § 1o
tituta. deste artigo.
Parágrafo único. A sentença que decretar a perda ou a § 5o O consentimento é retratável até a data da realiza-
suspensão do poder familiar será averbada à margem do ção da audiência especificada no § 1o deste artigo, e os pais
registro de nascimento da criança ou do adolescente. podem exercer o arrependimento no prazo de 10 (dez) dias,
contado da data de prolação da sentença de extinção do po-
Seção III der familiar.
Da Destituição da Tutela § 6o O consentimento somente terá valor se for dado após
o nascimento da criança.
Art. 164. Na destituição da tutela, observar-se-á o pro- § 7o A família natural e a família substituta receberão a
cedimento para a remoção de tutor previsto na lei proces- devida orientação por intermédio de equipe técnica inter-
sual civil e, no que couber, o disposto na seção anterior. profissional a serviço da Justiça da Infância e da Juventude,
preferencialmente com apoio dos técnicos responsáveis pela
A destituição da tutela pode, assim, ser decretada ju- execução da política municipal de garantia do direito à con-
dicialmente, em procedimento contraditório, nos casos vivência familiar.
previstos na legislação civil, bem como na hipótese de
descumprimento injustificado dos deveres e obrigações. Art. 167. A autoridade judiciária, de ofício ou a requeri-
mento das partes ou do Ministério Público, determinará a
Seção IV realização de estudo social ou, se possível, perícia por equi-
Da Colocação em Família Substituta pe interprofissional, decidindo sobre a concessão de guarda
provisória, bem como, no caso de adoção, sobre o estágio de
Dos artigos 165 a 170 estão descritos procedimentos convivência.
adotados na colocação em família substituta: Parágrafo único. Deferida a concessão da guarda provi-
sória ou do estágio de convivência, a criança ou o adolescente
Art. 165. São requisitos para a concessão de pedidos será entregue ao interessado, mediante termo de responsa-
de colocação em família substituta: bilidade.
I - qualificação completa do requerente e de seu even-
tual cônjuge, ou companheiro, com expressa anuência Art. 168. Apresentado o relatório social ou o laudo pericial,
deste; e ouvida, sempre que possível, a criança ou o adolescente, dar-
II - indicação de eventual parentesco do requerente e -se-á vista dos autos ao Ministério Público, pelo prazo de
de seu cônjuge, ou companheiro, com a criança ou adoles- cinco dias, decidindo a autoridade judiciária em igual prazo.
LEGISLAÇÃO ESPECÍFICA

cente, especificando se tem ou não parente vivo;


III - qualificação completa da criança ou adolescente e Art. 169. Nas hipóteses em que a destituição da tutela, a
de seus pais, se conhecidos; perda ou a suspensão do poder familiar constituir pressuposto
IV - indicação do cartório onde foi inscrito nascimento, lógico da medida principal de colocação em família substitu-
anexando, se possível, uma cópia da respectiva certidão; ta, será observado o procedimento contraditório previsto nas
V - declaração sobre a existência de bens, direitos ou Seções II e III deste Capítulo.
rendimentos relativos à criança ou ao adolescente. Parágrafo único. A perda ou a modificação da guarda po-
Parágrafo único. Em se tratando de adoção, observar- derá ser decretada nos mesmos autos do procedimento,
-se-ão também os requisitos específicos. observado o disposto no art. 35.

122
Art. 170. Concedida a guarda ou a tutela, observar-se-á o Art. 176. Sendo o adolescente liberado, a autoridade
disposto no art. 32, e, quanto à adoção, o contido no art. 47. policial encaminhará imediatamente ao representante do
Parágrafo único. A colocação de criança ou adolescente Ministério Público cópia do auto de apreensão ou boletim
sob a guarda de pessoa inscrita em programa de acolhimen- de ocorrência.
to familiar será comunicada pela autoridade judiciária à enti-
dade por este responsável no prazo máximo de 5 (cinco) dias. Art. 177. Se, afastada a hipótese de flagrante, houver
indícios de participação de adolescente na prática de ato
Seção V infracional, a autoridade policial encaminhará ao repre-
Da Apuração de Ato Infracional Atribuído a sentante do Ministério Público relatório das investigações e
Adolescente demais documentos.

Art. 171. O adolescente apreendido por força de ordem Art. 178. O adolescente a quem se atribua autoria de ato
judicial será, desde logo, encaminhado à autoridade judiciá- infracional não poderá ser conduzido ou transportado em
ria. compartimento fechado de veículo policial, em condições
atentatórias à sua dignidade, ou que impliquem risco à sua
Art. 172. O adolescente apreendido em flagrante de ato integridade física ou mental, sob pena de responsabilidade.
infracional será, desde logo, encaminhado à autoridade po-
licial competente. Art. 179. Apresentado o adolescente, o representante
Parágrafo único. Havendo repartição policial especializa- do Ministério Público, no mesmo dia e à vista do auto de
da para atendimento de adolescente e em se tratando de ato apreensão, boletim de ocorrência ou relatório policial, devi-
infracional praticado em co-autoria com maior, prevalecerá damente autuados pelo cartório judicial e com informação
a atribuição da repartição especializada, que, após as provi- sobre os antecedentes do adolescente, procederá imediata
dências necessárias e conforme o caso, encaminhará o adulto e informalmente à sua oitiva e, em sendo possível, de seus
à repartição policial própria. pais ou responsável, vítima e testemunhas.
Parágrafo único. Em caso de não apresentação, o repre-
Art. 173. Em caso de flagrante de ato infracional come- sentante do Ministério Público notificará os pais ou respon-
tido mediante violência ou grave ameaça a pessoa, a auto- sável para apresentação do adolescente, podendo requisitar
ridade policial, sem prejuízo do disposto nos arts. 106, pará- o concurso das polícias civil e militar.
grafo único, e 107, deverá:
I - lavrar auto de apreensão, ouvidos as testemunhas e o Art. 180. Adotadas as providências a que alude o artigo
adolescente; anterior, o representante do Ministério Público poderá:
II - apreender o produto e os instrumentos da infração; I - promover o arquivamento dos autos;
III - requisitar os exames ou perícias necessários à com- II - conceder a remissão;
provação da materialidade e autoria da infração. III - representar à autoridade judiciária para aplicação
Parágrafo único. Nas demais hipóteses de flagrante, a de medida sócio-educativa.
lavratura do auto poderá ser substituída por boletim de ocor-
rência circunstanciada. Art. 181. Promovido o arquivamento dos autos ou concedida
a remissão pelo representante do Ministério Público, mediante
Art. 174. Comparecendo qualquer dos pais ou responsá- termo fundamentado, que conterá o resumo dos fatos, os autos
vel, o adolescente será prontamente liberado pela autorida- serão conclusos à autoridade judiciária para homologação.
de policial, sob termo de compromisso e responsabilidade de § 1º Homologado o arquivamento ou a remissão, a au-
sua apresentação ao representante do Ministério Público, no toridade judiciária determinará, conforme o caso, o cum-
mesmo dia ou, sendo impossível, no primeiro dia útil imedia- primento da medida.
to, exceto quando, pela gravidade do ato infracional e sua § 2º Discordando, a autoridade judiciária fará remessa
repercussão social, deva o adolescente permanecer sob inter- dos autos ao Procurador-Geral de Justiça, mediante despa-
nação para garantia de sua segurança pessoal ou manuten- cho fundamentado, e este oferecerá representação, desig-
ção da ordem pública. nará outro membro do Ministério Público para apresentá-la,
ou ratificará o arquivamento ou a remissão, que só então
Art. 175. Em caso de não liberação, a autoridade policial estará a autoridade judiciária obrigada a homologar.
encaminhará, desde logo, o adolescente ao representante do
Ministério Público, juntamente com cópia do auto de apreen- Art. 182. Se, por qualquer razão, o representante do Mi-
são ou boletim de ocorrência. nistério Público não promover o arquivamento ou conceder
§ 1º Sendo impossível a apresentação imediata, a au- a remissão, oferecerá representação à autoridade judiciária,
toridade policial encaminhará o adolescente à entidade de
LEGISLAÇÃO ESPECÍFICA

propondo a instauração de procedimento para aplicação da


atendimento, que fará a apresentação ao representante do medida sócio-educativa que se afigurar a mais adequada.
Ministério Público no prazo de vinte e quatro horas. § 1º A representação será oferecida por petição, que
§ 2º Nas localidades onde não houver entidade de aten- conterá o breve resumo dos fatos e a classificação do ato
dimento, a apresentação far-se-á pela autoridade policial. infracional e, quando necessário, o rol de testemunhas,
À falta de repartição policial especializada, o adolescente podendo ser deduzida oralmente, em sessão diária insta-
aguardará a apresentação em dependência separada da lada pela autoridade judiciária.
destinada a maiores, não podendo, em qualquer hipóte- § 2º A representação independe de prova pré-consti-
se, exceder o prazo referido no parágrafo anterior. tuída da autoria e materialidade.

123
Art. 183. O prazo máximo e improrrogável para a con- Art. 187. Se o adolescente, devidamente notificado, não
clusão do procedimento, estando o adolescente internado comparecer, injustificadamente à audiência de apresenta-
provisoriamente, será de quarenta e cinco dias. ção, a autoridade judiciária designará nova data, determi-
nando sua condução coercitiva.
Art. 184. Oferecida a representação, a autoridade ju-
diciária designará audiência de apresentação do adoles- Art. 188. A remissão, como forma de extinção ou sus-
cente, decidindo, desde logo, sobre a decretação ou ma- pensão do processo, poderá ser aplicada em qualquer fase
nutenção da internação, observado o disposto no art. 108 do procedimento, antes da sentença.
e parágrafo.
§ 1º O adolescente e seus pais ou responsável serão Art. 189. A autoridade judiciária não aplicará qualquer
cientificados do teor da representação, e notificados a medida, desde que reconheça na sentença:
comparecer à audiência, acompanhados de advogado. I - estar provada a inexistência do fato;
§ 2º Se os pais ou responsável não forem localizados, II - não haver prova da existência do fato;
a autoridade judiciária dará curador especial ao adoles- III - não constituir o fato ato infracional;
cente. IV - não existir prova de ter o adolescente concorrido
§ 3º Não sendo localizado o adolescente, a autorida- para o ato infracional.
de judiciária expedirá mandado de busca e apreensão, Parágrafo único. Na hipótese deste artigo, estando o
determinando o sobrestamento do feito, até a efetiva adolescente internado, será imediatamente colocado em
apresentação. liberdade.
§ 4º Estando o adolescente internado, será requisita-
da a sua apresentação, sem prejuízo da notificação dos Art. 190. A intimação da sentença que aplicar medida
pais ou responsável. de internação ou regime de semi-liberdade será feita:
I - ao adolescente e ao seu defensor;
Art. 185. A internação, decretada ou mantida pela au- II - quando não for encontrado o adolescente, a seus
toridade judiciária, não poderá ser cumprida em estabele- pais ou responsável, sem prejuízo do defensor.
cimento prisional. § 1º Sendo outra a medida aplicada, a intimação far-
§ 1º Inexistindo na comarca entidade com as carac- -se-á unicamente na pessoa do defensor.
§ 2º Recaindo a intimação na pessoa do adolescen-
terísticas definidas no art. 123, o adolescente deverá ser
te, deverá este manifestar se deseja ou não recorrer da
imediatamente transferido para a localidade mais próxi-
sentença.
ma.
§ 2º Sendo impossível a pronta transferência, o ado-
Seção V-A
lescente aguardará sua remoção em repartição policial,
Da Infiltração de Agentes de Polícia para a Inves-
desde que em seção isolada dos adultos e com instala-
tigação de Crimes contra a Dignidade Sexual de
ções apropriadas, não podendo ultrapassar o prazo má-
Criança e de Adolescente
ximo de cinco dias, sob pena de responsabilidade.
Art. 190-A. A infiltração de agentes de polícia na inter-
Art. 186. Comparecendo o adolescente, seus pais ou
net com o fim de investigar os crimes previstos nos arts.
responsável, a autoridade judiciária procederá à oitiva dos
240, 241, 241-A, 241-B, 241-C e 241-D desta Lei e nos
mesmos, podendo solicitar opinião de profissional quali- arts. 154-A, 217-A, 218, 218-A e 218-B do Decreto-Lei nº
ficado. 2.848, de 7 de dezembro de 1940 (Código Penal), obede-
§ 1º Se a autoridade judiciária entender adequada a cerá às seguintes regras:
remissão, ouvirá o representante do Ministério Público, I – será precedida de autorização judicial devidamen-
proferindo decisão. te circunstanciada e fundamentada, que estabelecerá os
§ 2º Sendo o fato grave, passível de aplicação de me- limites da infiltração para obtenção de prova, ouvido o
dida de internação ou colocação em regime de semi-li- Ministério Público;
berdade, a autoridade judiciária, verificando que o adoles- II – dar-se-á mediante requerimento do Ministério Pú-
cente não possui advogado constituído, nomeará defensor, blico ou representação de delegado de polícia e conterá a
designando, desde logo, audiência em continuação, po- demonstração de sua necessidade, o alcance das tarefas
dendo determinar a realização de diligências e estudo do dos policiais, os nomes ou apelidos das pessoas investiga-
caso. das e, quando possível, os dados de conexão ou cadastrais
§ 3º O advogado constituído ou o defensor nomeado, que permitam a identificação dessas pessoas;
no prazo de três dias contado da audiência de apresenta- III – não poderá exceder o prazo de 90 (noventa) dias,
ção, oferecerá defesa prévia e rol de testemunhas. sem prejuízo de eventuais renovações, desde que o total
§ 4º Na audiência em continuação, ouvidas as teste-
LEGISLAÇÃO ESPECÍFICA

não exceda a 720 (setecentos e vinte) dias e seja demons-


munhas arroladas na representação e na defesa prévia, trada sua efetiva necessidade, a critério da autoridade ju-
cumpridas as diligências e juntado o relatório da equipe dicial.
interprofissional, será dada a palavra ao representante do § 1º A autoridade judicial e o Ministério Público po-
Ministério Público e ao defensor, sucessivamente, pelo derão requisitar relatórios parciais da operação de infil-
tempo de vinte minutos para cada um, prorrogável por tração antes do término do prazo de que trata o inciso II
mais dez, a critério da autoridade judiciária, que em se- do § 1º deste artigo.
guida proferirá decisão. § 2º Para efeitos do disposto no inciso I do § 1º deste
artigo, consideram-se:

124
I – dados de conexão: informações referentes a hora, Art. 192. O dirigente da entidade será citado para, no
data, início, término, duração, endereço de Protocolo de prazo de dez dias, oferecer resposta escrita, podendo jun-
Internet (IP) utilizado e terminal de origem da conexão; tar documentos e indicar as provas a produzir.
II – dados cadastrais: informações referentes a nome e
endereço de assinante ou de usuário registrado ou auten- Art. 193. Apresentada ou não a resposta, e sendo ne-
ticado para a conexão a quem endereço de IP, identifica- cessário, a autoridade judiciária designará audiência de
ção de usuário ou código de acesso tenha sido atribuído no instrução e julgamento, intimando as partes.
momento da conexão. § 1º Salvo manifestação em audiência, as partes e o
§ 3º A infiltração de agentes de polícia na internet Ministério Público terão cinco dias para oferecer alega-
não será admitida se a prova puder ser obtida por outros ções finais, decidindo a autoridade judiciária em igual
meios. prazo.
§ 2º Em se tratando de afastamento provisório ou de-
Art. 190-B. As informações da operação de infiltração finitivo de dirigente de entidade governamental, a auto-
serão encaminhadas diretamente ao juiz responsável pela ridade judiciária oficiará à autoridade administrativa ime-
autorização da medida, que zelará por seu sigilo. diatamente superior ao afastado, marcando prazo para a
Parágrafo único. Antes da conclusão da operação, o substituição.
acesso aos autos será reservado ao juiz, ao Ministério Pú- § 3º Antes de aplicar qualquer das medidas, a auto-
blico e ao delegado de polícia responsável pela operação, ridade judiciária poderá fixar prazo para a remoção das
com o objetivo de garantir o sigilo das investigações. irregularidades verificadas. Satisfeitas as exigências, o
processo será extinto, sem julgamento de mérito.
Art. 190-C. Não comete crime o policial que oculta a § 4º A multa e a advertência serão impostas ao diri-
sua identidade para, por meio da internet, colher indícios gente da entidade ou programa de atendimento.
de autoria e materialidade dos crimes previstos nos arts.
240, 241, 241-A, 241-B, 241-C e 241-D desta Lei e nos Seção VII
arts. 154-A, 217-A, 218, 218-A e 218-B do Decreto-Lei nº Da Apuração de Infração Administrativa às Nor-
2.848, de 7 de dezembro de 1940 (Código Penal).
mas de Proteção à Criança e ao Adolescente
Parágrafo único. O agente policial infiltrado que deixar
de observar a estrita finalidade da investigação responde-
Art. 194. O procedimento para imposição de penali-
rá pelos excessos praticados.
dade administrativa por infração às normas de proteção
à criança e ao adolescente terá início por representação
Art. 190-D. Os órgãos de registro e cadastro público
do Ministério Público, ou do Conselho Tutelar, ou auto de
poderão incluir nos bancos de dados próprios, mediante
infração elaborado por servidor efetivo ou voluntário cre-
procedimento sigiloso e requisição da autoridade judicial,
as informações necessárias à efetividade da identidade fic- denciado, e assinado por duas testemunhas, se possível.
tícia criada. § 1º No procedimento iniciado com o auto de infra-
Parágrafo único. O procedimento sigiloso de que trata ção, poderão ser usadas fórmulas impressas, especifican-
esta Seção será numerado e tombado em livro específico. do-se a natureza e as circunstâncias da infração.
§ 2º Sempre que possível, à verificação da infração
Art. 190-E. Concluída a investigação, todos os atos ele- seguir-se-á a lavratura do auto, certificando-se, em caso
trônicos praticados durante a operação deverão ser regis- contrário, dos motivos do retardamento.
trados, gravados, armazenados e encaminhados ao juiz e
ao Ministério Público, juntamente com relatório circuns- Art. 195. O requerido terá prazo de dez dias para apre-
tanciado. sentação de defesa, contado da data da intimação, que
Parágrafo único. Os atos eletrônicos registrados citados será feita:
no caput deste artigo serão reunidos em autos apartados e I - pelo autuante, no próprio auto, quando este for la-
apensados ao processo criminal juntamente com o inqué- vrado na presença do requerido;
rito policial, assegurando-se a preservação da identidade II - por oficial de justiça ou funcionário legalmente ha-
do agente policial infiltrado e a intimidade das crianças e bilitado, que entregará cópia do auto ou da representação
dos adolescentes envolvidos. ao requerido, ou a seu representante legal, lavrando cer-
tidão;
Seção VI III - por via postal, com aviso de recebimento, se não for
Da Apuração de Irregularidades em Entidade de encontrado o requerido ou seu representante legal;
Atendimento IV - por edital, com prazo de trinta dias, se incerto ou
não sabido o paradeiro do requerido ou de seu represen-
LEGISLAÇÃO ESPECÍFICA

Art. 191. O procedimento de apuração de irregulari- tante legal.


dades em entidade governamental e não-governamental
terá início mediante portaria da autoridade judiciária ou Art. 196. Não sendo apresentada a defesa no prazo
representação do Ministério Público ou do Conselho Tute- legal, a autoridade judiciária dará vista dos autos do Mi-
lar, onde conste, necessariamente, resumo dos fatos. nistério Público, por cinco dias, decidindo em igual prazo.
Parágrafo único. Havendo motivo grave, poderá a au-
toridade judiciária, ouvido o Ministério Público, decretar Art. 197. Apresentada a defesa, a autoridade judiciária
liminarmente o afastamento provisório do dirigente da procederá na conformidade do artigo anterior, ou, sendo
entidade, mediante decisão fundamentada. necessário, designará audiência de instrução e julgamento.

125
Parágrafo único. Colhida a prova oral, manifestar-se- § 3o É recomendável que as crianças e os adolescentes
-ão sucessivamente o Ministério Público e o procurador acolhidos institucionalmente ou por família acolhedora se-
do requerido, pelo tempo de vinte minutos para cada um, jam preparados por equipe interprofissional antes da inclu-
prorrogável por mais dez, a critério da autoridade judiciá- são em família adotiva.
ria, que em seguida proferirá sentença.
Art. 197-D. Certificada nos autos a conclusão da par-
Seção VIII ticipação no programa referido no art. 197-C desta Lei, a
Da Habilitação de Pretendentes à Adoção autoridade judiciária, no prazo de 48 (quarenta e oito) ho-
ras, decidirá acerca das diligências requeridas pelo Minis-
Art. 197-A. Os postulantes à adoção, domiciliados no tério Público e determinará a juntada do estudo psicosso-
Brasil, apresentarão petição inicial na qual conste: cial, designando, conforme o caso, audiência de instrução
I - qualificação completa; e julgamento.
II - dados familiares; Parágrafo único. Caso não sejam requeridas diligên-
III - cópias autenticadas de certidão de nascimento ou cias, ou sendo essas indeferidas, a autoridade judiciária
casamento, ou declaração relativa ao período de união es- determinará a juntada do estudo psicossocial, abrindo a
tável; seguir vista dos autos ao Ministério Público, por 5 (cinco)
IV - cópias da cédula de identidade e inscrição no Ca- dias, decidindo em igual prazo.
dastro de Pessoas Físicas;
V - comprovante de renda e domicílio; Art. 197-E. Deferida a habilitação, o postulante será
VI - atestados de sanidade física e mental; inscrito nos cadastros referidos no art. 50 desta Lei, sen-
VII - certidão de antecedentes criminais; do a sua convocação para a adoção feita de acordo com
VIII - certidão negativa de distribuição cível. ordem cronológica de habilitação e conforme a disponibili-
dade de crianças ou adolescentes adotáveis.
Art. 197-B. A autoridade judiciária, no prazo de 48 § 1o A ordem cronológica das habilitações somente po-
derá deixar de ser observada pela autoridade judiciária nas
(quarenta e oito) horas, dará vista dos autos ao Ministério
hipóteses previstas no § 13 do art. 50 desta Lei, quando
Público, que no prazo de 5 (cinco) dias poderá:
comprovado ser essa a melhor solução no interesse do ado-
I - apresentar quesitos a serem respondidos pela equi-
tando.
pe interprofissional encarregada de elaborar o estudo téc-
§ 2o A habilitação à adoção deverá ser renovada no mí-
nico a que se refere o art. 197-C desta Lei;
nimo trienalmente mediante avaliação por equipe inter-
II - requerer a designação de audiência para oitiva dos
profissional.
postulantes em juízo e testemunhas;
§ 3o Quando o adotante candidatar-se a uma nova ado-
III - requerer a juntada de documentos complementa- ção, será dispensável a renovação da habilitação, bastando
res e a realização de outras diligências que entender ne- a avaliação por equipe interprofissional.
cessárias. § 4o Após 3 (três) recusas injustificadas, pelo habilitado,
à adoção de crianças ou adolescentes indicados dentro do
Art. 197-C. Intervirá no feito, obrigatoriamente, equi- perfil escolhido, haverá reavaliação da habilitação conce-
pe interprofissional a serviço da Justiça da Infância e da dida.
Juventude, que deverá elaborar estudo psicossocial, que § 5o A desistência do pretendente em relação à guarda
conterá subsídios que permitam aferir a capacidade e o para fins de adoção ou a devolução da criança ou do ado-
preparo dos postulantes para o exercício de uma paterni- lescente depois do trânsito em julgado da sentença de ado-
dade ou maternidade responsável, à luz dos requisitos e ção importará na sua exclusão dos cadastros de adoção.
princípios desta Lei.
§ 1o É obrigatória a participação dos postulantes em Art. 197-F. O prazo máximo para conclusão da habili-
programa oferecido pela Justiça da Infância e da Juventu- tação à adoção será de 120 (cento e vinte) dias, prorrogá-
de, preferencialmente com apoio dos técnicos responsáveis vel por igual período, mediante decisão fundamentada da
pela execução da política municipal de garantia do direito autoridade judiciária.
à convivência familiar e dos grupos de apoio à adoção de-
vidamente habilitados perante a Justiça da Infância e da Capítulo IV
Juventude, que inclua preparação psicológica, orientação Dos Recursos
e estímulo à adoção inter-racial, de crianças ou de ado-
lescentes com deficiência, com doenças crônicas ou com Art. 198. Nos procedimentos afetos à Justiça da Infân-
necessidades específicas de saúde, e de grupos de irmãos. cia e da Juventude, inclusive os relativos à execução das
§ 2o Sempre que possível e recomendável, a etapa obri- medidas socioeducativas, adotar-se-á o sistema recursal da
gatória da preparação referida no § 1o deste artigo inclui-
LEGISLAÇÃO ESPECÍFICA

Lei no 5.869, de 11 de janeiro de 1973 (Código de Processo


rá o contato com crianças e adolescentes em regime de Civil), com as seguintes adaptações: (Redação dada pela Lei
acolhimento familiar ou institucional, a ser realizado sob nº 12.594, de 2012)
orientação, supervisão e avaliação da equipe técnica da I - os recursos serão interpostos independentemente de
Justiça da Infância e da Juventude e dos grupos de apoio à preparo;
adoção, com apoio dos técnicos responsáveis pelo progra- II - em todos os recursos, salvo nos embargos de decla-
ma de acolhimento familiar e institucional e pela execução ração, o prazo para o Ministério Público e para a defesa
da política municipal de garantia do direito à convivência será sempre de 10 (dez) dias; (Redação dada pela Lei nº
familiar. 12.594, de 2012)

126
III - os recursos terão preferência de julgamento e dis- IV - promover, de ofício ou por solicitação dos interes-
pensarão revisor; sados, a especialização e a inscrição de hipoteca legal e
VII - antes de determinar a remessa dos autos à supe- a prestação de contas dos tutores, curadores e quaisquer
rior instância, no caso de apelação, ou do instrumento, no administradores de bens de crianças e adolescentes nas
caso de agravo, a autoridade judiciária proferirá despacho hipóteses do art. 98;
fundamentado, mantendo ou reformando a decisão, no V - promover o inquérito civil e a ação civil pública
prazo de cinco dias; para a proteção dos interesses individuais, difusos ou co-
VIII - mantida a decisão apelada ou agravada, o escrivão letivos relativos à infância e à adolescência, inclusive os
remeterá os autos ou o instrumento à superior instância den- definidos no art. 220, § 3º inciso II, da Constituição Federal;
tro de vinte e quatro horas, independentemente de novo pe- VI - instaurar procedimentos administrativos e, para
dido do recorrente; se a reformar, a remessa dos autos depen- instruí-los:
derá de pedido expresso da parte interessada ou do Ministério a) expedir notificações para colher depoimentos ou es-
Público, no prazo de cinco dias, contados da intimação. clarecimentos e, em caso de não comparecimento injustifi-
cado, requisitar condução coercitiva, inclusive pela polícia
Art. 199. Contra as decisões proferidas com base no art.
civil ou militar;
149 caberá recurso de apelação.
b) requisitar informações, exames, perícias e docu-
mentos de autoridades municipais, estaduais e federais,
Art. 199-A. A sentença que deferir a adoção produz
efeito desde logo, embora sujeita a apelação, que será rece- da administração direta ou indireta, bem como promover
bida exclusivamente no efeito devolutivo, salvo se se tratar inspeções e diligências investigatórias;
de adoção internacional ou se houver perigo de dano irre- c) requisitar informações e documentos a particulares
parável ou de difícil reparação ao adotando. e instituições privadas;
VII - instaurar sindicâncias, requisitar diligências inves-
Art. 199-B. A sentença que destituir ambos ou qualquer tigatórias e determinar a instauração de inquérito policial,
dos genitores do poder familiar fica sujeita a apelação, que para apuração de ilícitos ou infrações às normas de prote-
deverá ser recebida apenas no efeito devolutivo. ção à infância e à juventude;
VIII - zelar pelo efetivo respeito aos direitos e garantias
Art. 199-C. Os recursos nos procedimentos de adoção e legais assegurados às crianças e adolescentes, promoven-
de destituição de poder familiar, em face da relevância das do as medidas judiciais e extrajudiciais cabíveis;
questões, serão processados com prioridade absoluta, de- IX - impetrar mandado de segurança, de injunção e
vendo ser imediatamente distribuídos, ficando vedado que habeas corpus, em qualquer juízo, instância ou tribunal,
aguardem, em qualquer situação, oportuna distribuição, e na defesa dos interesses sociais e individuais indisponíveis
serão colocados em mesa para julgamento sem revisão e afetos à criança e ao adolescente;
com parecer urgente do Ministério Público. X - representar ao juízo visando à aplicação de penali-
dade por infrações cometidas contra as normas de prote-
Art. 199-D. O relator deverá colocar o processo em ção à infância e à juventude, sem prejuízo da promoção da
mesa para julgamento no prazo máximo de 60 (sessenta) responsabilidade civil e penal do infrator, quando cabível;
dias, contado da sua conclusão. XI - inspecionar as entidades públicas e particulares de
Parágrafo único. O Ministério Público será intimado da atendimento e os programas de que trata esta Lei, ado-
data do julgamento e poderá na sessão, se entender neces- tando de pronto as medidas administrativas ou judiciais
sário, apresentar oralmente seu parecer. necessárias à remoção de irregularidades porventura ve-
rificadas;
Art. 199-E. O Ministério Público poderá requerer a ins-
XII - requisitar força policial, bem como a colaboração
tauração de procedimento para apuração de responsabili-
dos serviços médicos, hospitalares, educacionais e de as-
dades se constatar o descumprimento das providências e do
sistência social, públicos ou privados, para o desempenho
prazo previstos nos artigos anteriores.
de suas atribuições.
Capítulo V § 1º A legitimação do Ministério Público para as ações
Do Ministério Público cíveis previstas neste artigo não impede a de terceiros,
nas mesmas hipóteses, segundo dispuserem a Constitui-
Art. 200. As funções do Ministério Público previstas nes- ção e esta Lei.
ta Lei serão exercidas nos termos da respectiva lei orgânica. § 2º As atribuições constantes deste artigo não ex-
cluem outras, desde que compatíveis com a finalidade do
Art. 201. Compete ao Ministério Público: Ministério Público.
§ 3º O representante do Ministério Público, no exercí-
LEGISLAÇÃO ESPECÍFICA

I - conceder a remissão como forma de exclusão do pro-


cesso; cio de suas funções, terá livre acesso a todo local onde se
II - promover e acompanhar os procedimentos relativos encontre criança ou adolescente.
às infrações atribuídas a adolescentes; § 4º O representante do Ministério Público será res-
III - promover e acompanhar as ações de alimentos e ponsável pelo uso indevido das informações e documen-
os procedimentos de suspensão e destituição do poder fa- tos que requisitar, nas hipóteses legais de sigilo.
miliar, nomeação e remoção de tutores, curadores e guar- § 5º Para o exercício da atribuição de que trata o inci-
diães, bem como oficiar em todos os demais procedimen- so VIII deste artigo, poderá o representante do Ministério
tos da competência da Justiça da Infância e da Juventude; Público:

127
a) reduzir a termo as declarações do reclamante, ins- I - do ensino obrigatório;
taurando o competente procedimento, sob sua presidên- II - de atendimento educacional especializado aos por-
cia; tadores de deficiência;
b) entender-se diretamente com a pessoa ou autorida- III - de atendimento em creche e pré-escola às crianças
de reclamada, em dia, local e horário previamente notifi- de zero a cinco anos de idade;
cados ou acertados; IV - de ensino noturno regular, adequado às condições
c) efetuar recomendações visando à melhoria dos ser- do educando;
viços públicos e de relevância pública afetos à criança e V - de programas suplementares de oferta de material
ao adolescente, fixando prazo razoável para sua perfeita didático-escolar, transporte e assistência à saúde do edu-
adequação. cando do ensino fundamental;
VI - de serviço de assistência social visando à proteção
Art. 202. Nos processos e procedimentos em que não à família, à maternidade, à infância e à adolescência, bem
for parte, atuará obrigatoriamente o Ministério Público como ao amparo às crianças e adolescentes que dele ne-
na defesa dos direitos e interesses de que cuida esta Lei, cessitem;
hipótese em que terá vista dos autos depois das partes,
VII - de acesso às ações e serviços de saúde;
podendo juntar documentos e requerer diligências, usando
VIII - de escolarização e profissionalização dos adoles-
os recursos cabíveis.
centes privados de liberdade.
IX - de ações, serviços e programas de orientação,
Art. 203. A intimação do Ministério Público, em qual-
quer caso, será feita pessoalmente. apoio e promoção social de famílias e destinados ao pleno
exercício do direito à convivência familiar por crianças e
Art. 204. A falta de intervenção do Ministério Público adolescentes.
acarreta a nulidade do feito, que será declarada de ofício X - de programas de atendimento para a execução das
pelo juiz ou a requerimento de qualquer interessado. medidas socioeducativas e aplicação de medidas de prote-
ção. (Incluído pela Lei nº 12.594, de 2012)
Art. 205. As manifestações processuais do representan- § 1o As hipóteses previstas neste artigo não excluem
te do Ministério Público deverão ser fundamentadas. da proteção judicial outros interesses individuais, difusos
ou coletivos, próprios da infância e da adolescência, pro-
Capítulo VI tegidos pela Constituição e pela Lei.
Do Advogado § 2o A investigação do desaparecimento de crianças
ou adolescentes será realizada imediatamente após noti-
Art. 206. A criança ou o adolescente, seus pais ou res- ficação aos órgãos competentes, que deverão comunicar
ponsável, e qualquer pessoa que tenha legítimo interesse o fato aos portos, aeroportos, Polícia Rodoviária e com-
na solução da lide poderão intervir nos procedimentos de panhias de transporte interestaduais e internacionais,
que trata esta Lei, através de advogado, o qual será inti- fornecendo-lhes todos os dados necessários à identifica-
mado para todos os atos, pessoalmente ou por publicação ção do desaparecido.
oficial, respeitado o segredo de justiça.
Parágrafo único. Será prestada assistência judiciária Art. 209. As ações previstas neste Capítulo serão pro-
integral e gratuita àqueles que dela necessitarem. postas no foro do local onde ocorreu ou deva ocorrer a
ação ou omissão, cujo juízo terá competência absoluta
Art. 207. Nenhum adolescente a quem se atribua a para processar a causa, ressalvadas a competência da
prática de ato infracional, ainda que ausente ou foragido, Justiça Federal e a competência originária dos tribunais
será processado sem defensor. superiores.
§ 1º Se o adolescente não tiver defensor, ser-lhe-á no-
meado pelo juiz, ressalvado o direito de, a todo tempo,
Art. 210. Para as ações cíveis fundadas em interesses
constituir outro de sua preferência.
coletivos ou difusos, consideram-se legitimados concor-
§ 2º A ausência do defensor não determinará o adia-
rentemente:
mento de nenhum ato do processo, devendo o juiz no-
mear substituto, ainda que provisoriamente, ou para o só I - o Ministério Público;
efeito do ato. II - a União, os estados, os municípios, o Distrito Fede-
§ 3º Será dispensada a outorga de mandato, quando ral e os territórios;
se tratar de defensor nomeado ou, sido constituído, tiver III - as associações legalmente constituídas há pelo
sido indicado por ocasião de ato formal com a presença menos um ano e que incluam entre seus fins institucionais
da autoridade judiciária. a defesa dos interesses e direitos protegidos por esta Lei,
dispensada a autorização da assembleia, se houver prévia
LEGISLAÇÃO ESPECÍFICA

Capítulo VII autorização estatutária.


Da Proteção Judicial dos Interesses Individuais, Di- § 1º Admitir-se-á litisconsórcio facultativo entre os
fusos e Coletivos Ministérios Públicos da União e dos estados na defesa
dos interesses e direitos de que cuida esta Lei.
Art. 208. Regem-se pelas disposições desta Lei as ações § 2º Em caso de desistência ou abandono da ação
de responsabilidade por ofensa aos direitos assegurados à por associação legitimada, o Ministério Público ou outro
criança e ao adolescente, referentes ao não oferecimento legitimado poderá assumir a titularidade ativa.
ou oferta irregular:

128
Art. 211. Os órgãos públicos legitimados poderão to- Art. 218. O juiz condenará a associação autora a pagar
mar dos interessados compromisso de ajustamento de sua ao réu os honorários advocatícios arbitrados na conformi-
conduta às exigências legais, o qual terá eficácia de título dade do § 4º do art. 20 da Lei n.º 5.869, de 11 de janeiro de
executivo extrajudicial. 1973 (Código de Processo Civil), quando reconhecer que a
pretensão é manifestamente infundada.
Art. 212. Para defesa dos direitos e interesses protegi- Parágrafo único. Em caso de litigância de má-fé, a as-
dos por esta Lei, são admissíveis todas as espécies de ações sociação autora e os diretores responsáveis pela proposi-
pertinentes. tura da ação serão solidariamente condenados ao décuplo
§ 1º Aplicam-se às ações previstas neste Capítulo as das custas, sem prejuízo de responsabilidade por perdas e
normas do Código de Processo Civil. danos.
§ 2º Contra atos ilegais ou abusivos de autoridade
pública ou agente de pessoa jurídica no exercício de atri- Art. 219. Nas ações de que trata este Capítulo, não ha-
buições do poder público, que lesem direito líquido e verá adiantamento de custas, emolumentos, honorários
certo previsto nesta Lei, caberá ação mandamental, que periciais e quaisquer outras despesas.
se regerá pelas normas da lei do mandado de segurança.
Art. 220. Qualquer pessoa poderá e o servidor público
Art. 213. Na ação que tenha por objeto o cumprimen- deverá provocar a iniciativa do Ministério Público, pres-
to de obrigação de fazer ou não fazer, o juiz concederá a tando-lhe informações sobre fatos que constituam objeto
tutela específica da obrigação ou determinará providên- de ação civil, e indicando-lhe os elementos de convicção.
cias que assegurem o resultado prático equivalente ao do
adimplemento. Art. 221. Se, no exercício de suas funções, os juízos e tri-
§ 1º Sendo relevante o fundamento da demanda e bunais tiverem conhecimento de fatos que possam ensejar
havendo justificado receio de ineficácia do provimento a propositura de ação civil, remeterão peças ao Ministério
final, é lícito ao juiz conceder a tutela liminarmente ou Público para as providências cabíveis.
após justificação prévia, citando o réu.
§ 2º O juiz poderá, na hipótese do parágrafo anterior Art. 222. Para instruir a petição inicial, o interessado
ou na sentença, impor multa diária ao réu, independen- poderá requerer às autoridades competentes as certidões
temente de pedido do autor, se for suficiente ou com- e informações que julgar necessárias, que serão fornecidas
patível com a obrigação, fixando prazo razoável para o no prazo de quinze dias.
cumprimento do preceito.
§ 3º A multa só será exigível do réu após o trânsito em Art. 223. O Ministério Público poderá instaurar, sob sua
julgado da sentença favorável ao autor, mas será devida presidência, inquérito civil, ou requisitar, de qualquer pes-
desde o dia em que se houver configurado o descum- soa, organismo público ou particular, certidões, informa-
primento. ções, exames ou perícias, no prazo que assinalar, o qual
não poderá ser inferior a dez dias úteis.
Art. 214. Os valores das multas reverterão ao fundo § 1º Se o órgão do Ministério Público, esgotadas to-
gerido pelo Conselho dos Direitos da Criança e do Adoles- das as diligências, se convencer da inexistência de fun-
cente do respectivo município. damento para a propositura da ação cível, promoverá o
§ 1º As multas não recolhidas até trinta dias após o arquivamento dos autos do inquérito civil ou das peças
trânsito em julgado da decisão serão exigidas através de informativas, fazendo-o fundamentadamente.
execução promovida pelo Ministério Público, nos mes- § 2º Os autos do inquérito civil ou as peças de infor-
mos autos, facultada igual iniciativa aos demais legitima- mação arquivados serão remetidos, sob pena de se in-
dos. correr em falta grave, no prazo de três dias, ao Conselho
§ 2º Enquanto o fundo não for regulamentado, o di- Superior do Ministério Público.
nheiro ficará depositado em estabelecimento oficial de § 3º Até que seja homologada ou rejeitada a promo-
crédito, em conta com correção monetária. ção de arquivamento, em sessão do Conselho Superior
do Ministério público, poderão as associações legitima-
Art. 215. O juiz poderá conferir efeito suspensivo aos das apresentar razões escritas ou documentos, que serão
recursos, para evitar dano irreparável à parte. juntados aos autos do inquérito ou anexados às peças de
informação.
Art. 216. Transitada em julgado a sentença que impu- § 4º A promoção de arquivamento será submetida a
ser condenação ao poder público, o juiz determinará a re- exame e deliberação do Conselho Superior do Ministério
messa de peças à autoridade competente, para apuração Público, conforme dispuser o seu regimento.
LEGISLAÇÃO ESPECÍFICA

da responsabilidade civil e administrativa do agente a que § 5º Deixando o Conselho Superior de homologar a


se atribua a ação ou omissão. promoção de arquivamento, designará, desde logo, ou-
tro órgão do Ministério Público para o ajuizamento da
Art. 217. Decorridos sessenta dias do trânsito em jul- ação.
gado da sentença condenatória sem que a associação au-
tora lhe promova a execução, deverá fazê-lo o Ministério Art. 224. Aplicam-se subsidiariamente, no que couber,
Público, facultada igual iniciativa aos demais legitimados. as disposições da Lei n.º 7.347, de 24 de julho de 1985.

129
Título VII Art. 234. Deixar a autoridade competente, sem justa
Dos Crimes e Das Infrações Administrativas causa, de ordenar a imediata liberação de criança ou ado-
lescente, tão logo tenha conhecimento da ilegalidade da
Capítulo I apreensão:
Dos Crimes Pena - detenção de seis meses a dois anos.

Seção I Art. 235. Descumprir, injustificadamente, prazo fixado


Disposições Gerais nesta Lei em benefício de adolescente privado de liberda-
de:
Art. 225. Este Capítulo dispõe sobre crimes praticados Pena - detenção de seis meses a dois anos.
contra a criança e o adolescente, por ação ou omissão, sem
prejuízo do disposto na legislação penal. Art. 236. Impedir ou embaraçar a ação de autoridade
judiciária, membro do Conselho Tutelar ou representante
Art. 226. Aplicam-se aos crimes definidos nesta Lei do Ministério Público no exercício de função prevista nesta
as normas da Parte Geral do Código Penal e, quanto ao Lei:
processo, as pertinentes ao Código de Processo Penal. Pena - detenção de seis meses a dois anos.

Art. 227. Os crimes definidos nesta Lei são de ação pú- Art. 237. Subtrair criança ou adolescente ao poder de
blica incondicionada. quem o tem sob sua guarda em virtude de lei ou ordem
judicial, com o fim de colocação em lar substituto:
Seção II Pena - reclusão de dois a seis anos, e multa.
Dos Crimes em Espécie
Art. 238. Prometer ou efetivar a entrega de filho ou
Art. 228. Deixar o encarregado de serviço ou o dirigen- pupilo a terceiro, mediante paga ou recompensa:
te de estabelecimento de atenção à saúde de gestante de Pena - reclusão de um a quatro anos, e multa.
manter registro das atividades desenvolvidas, na forma e Parágrafo único. Incide nas mesmas penas quem ofe-
prazo referidos no art. 10 desta Lei, bem como de fornecer rece ou efetiva a paga ou recompensa.
à parturiente ou a seu responsável, por ocasião da alta
médica, declaração de nascimento, onde constem as in- Art. 239. Promover ou auxiliar a efetivação de ato des-
tercorrências do parto e do desenvolvimento do neonato: tinado ao envio de criança ou adolescente para o exterior
Pena - detenção de seis meses a dois anos. com inobservância das formalidades legais ou com o fito
Parágrafo único. Se o crime é culposo: de obter lucro:
Pena - detenção de dois a seis meses, ou multa.
Pena - reclusão de quatro a seis anos, e multa.
Parágrafo único. Se há emprego de violência, grave
Art. 229. Deixar o médico, enfermeiro ou dirigente de
ameaça ou fraude:
estabelecimento de atenção à saúde de gestante de iden-
Pena - reclusão, de 6 (seis) a 8 (oito) anos, além da
tificar corretamente o neonato e a parturiente, por ocasião
pena correspondente à violência.
do parto, bem como deixar de proceder aos exames referi-
dos no art. 10 desta Lei:
Art. 240. Produzir, reproduzir, dirigir, fotografar, filmar
Pena - detenção de seis meses a dois anos.
Parágrafo único. Se o crime é culposo: ou registrar, por qualquer meio, cena de sexo explícito ou
Pena - detenção de dois a seis meses, ou multa. pornográfica, envolvendo criança ou adolescente:
Pena – reclusão, de 4 (quatro) a 8 (oito) anos, e multa.
Art. 230. Privar a criança ou o adolescente de sua liber- § 1o Incorre nas mesmas penas quem agencia, facilita,
dade, procedendo à sua apreensão sem estar em flagrante recruta, coage, ou de qualquer modo intermedeia a parti-
de ato infracional ou inexistindo ordem escrita da autori- cipação de criança ou adolescente nas cenas referidas no
dade judiciária competente: caputdeste artigo, ou ainda quem com esses contracena.
Pena - detenção de seis meses a dois anos. § 2o Aumenta-se a pena de 1/3 (um terço) se o agente
Parágrafo único. Incide na mesma pena aquele que comete o crime:
procede à apreensão sem observância das formalidades I – no exercício de cargo ou função pública ou a pre-
legais. texto de exercê-la;
II – prevalecendo-se de relações domésticas, de coabi-
Art. 231. Deixar a autoridade policial responsável pela tação ou de hospitalidade; ou
apreensão de criança ou adolescente de fazer imediata co- III – prevalecendo-se de relações de parentesco consan-
güíneo ou afim até o terceiro grau, ou por adoção, de tutor,
LEGISLAÇÃO ESPECÍFICA

municação à autoridade judiciária competente e à família


do apreendido ou à pessoa por ele indicada: curador, preceptor, empregador da vítima ou de quem, a
Pena - detenção de seis meses a dois anos. qualquer outro título, tenha autoridade sobre ela, ou com
seu consentimento.
Art. 232. Submeter criança ou adolescente sob sua au-
toridade, guarda ou vigilância a vexame ou a constrangi- Art. 241. Vender ou expor à venda fotografia, vídeo
mento: ou outro registro que contenha cena de sexo explícito ou
Pena - detenção de seis meses a dois anos. pornográfica envolvendo criança ou adolescente:
Pena – reclusão, de 4 (quatro) a 8 (oito) anos, e multa.

130
Art. 241-A. Oferecer, trocar, disponibilizar, transmitir, Parágrafo único. Nas mesmas penas incorre quem:
distribuir, publicar ou divulgar por qualquer meio, inclu- I – facilita ou induz o acesso à criança de material
sive por meio de sistema de informática ou telemático, contendo cena de sexo explícito ou pornográfica com o
fotografia, vídeo ou outro registro que contenha cena de fim de com ela praticar ato libidinoso;
sexo explícito ou pornográfica envolvendo criança ou ado- II – pratica as condutas descritas no caput deste arti-
lescente: go com o fim de induzir criança a se exibir de forma por-
Pena – reclusão, de 3 (três) a 6 (seis) anos, e multa. nográfica ou sexualmente explícita.
§ 1o Nas mesmas penas incorre quem:
I – assegura os meios ou serviços para o armazena- Art. 241-E. Para efeito dos crimes previstos nesta
mento das fotografias, cenas ou imagens de que trata o Lei, a expressão “cena de sexo explícito ou pornográfica”
caput deste artigo; compreende qualquer situação que envolva criança ou
II – assegura, por qualquer meio, o acesso por rede de adolescente em atividades sexuais explícitas, reais ou si-
computadores às fotografias, cenas ou imagens de que muladas, ou exibição dos órgãos genitais de uma criança
trata o caput deste artigo. ou adolescente para fins primordialmente sexuais.
§ 2o As condutas tipificadas nos incisos I e II do §
Art. 242. Vender, fornecer ainda que gratuitamente
1 deste artigo são puníveis quando o responsável legal
o
ou entregar, de qualquer forma, a criança ou adolescente
pela prestação do serviço, oficialmente notificado, deixa
arma, munição ou explosivo:
de desabilitar o acesso ao conteúdo ilícito de que trata o
Pena - reclusão, de 3 (três) a 6 (seis) anos.
caput deste artigo.
Art. 243. Vender, fornecer, servir, ministrar ou entregar,
Art. 241-B. Adquirir, possuir ou armazenar, por qual- ainda que gratuitamente, de qualquer forma, a criança ou
quer meio, fotografia, vídeo ou outra forma de registro a adolescente, bebida alcoólica ou, sem justa causa, outros
que contenha cena de sexo explícito ou pornográfica en- produtos cujos componentes possam causar dependência
volvendo criança ou adolescente: física ou psíquica:
Pena – reclusão, de 1 (um) a 4 (quatro) anos, e multa. Pena - detenção de 2 (dois) a 4 (quatro) anos, e multa,
§ 1o A pena é diminuída de 1 (um) a 2/3 (dois terços) se o fato não constitui crime mais grave.
se de pequena quantidade o material a que se refere o
caput deste artigo. Art. 244. Vender, fornecer ainda que gratuitamente ou
§ 2o Não há crime se a posse ou o armazenamento entregar, de qualquer forma, a criança ou adolescente fo-
tem a finalidade de comunicar às autoridades competen- gos de estampido ou de artifício, exceto aqueles que, pelo
tes a ocorrência das condutas descritas nos arts. 240, 241, seu reduzido potencial, sejam incapazes de provocar qual-
241-A e 241-C desta Lei, quando a comunicação for feita quer dano físico em caso de utilização indevida:
por: Pena - detenção de seis meses a dois anos, e multa.
I – agente público no exercício de suas funções;
II – membro de entidade, legalmente constituída, que Art. 244-A. Submeter criança ou adolescente, como tais
inclua, entre suas finalidades institucionais, o recebimen- definidos no caput do art. 2o desta Lei, à prostituição ou à
to, o processamento e o encaminhamento de notícia dos exploração sexual:
crimes referidos neste parágrafo; Pena – reclusão de quatro a dez anos e multa, além da
III – representante legal e funcionários responsáveis perda de bens e valores utilizados na prática criminosa em
de provedor de acesso ou serviço prestado por meio de favor do Fundo dos Direitos da Criança e do Adolescente
rede de computadores, até o recebimento do material da unidade da Federação (Estado ou Distrito Federal) em
relativo à notícia feita à autoridade policial, ao Ministério que foi cometido o crime, ressalvado o direito de terceiro
de boa-fé.
Público ou ao Poder Judiciário.
§ 1o Incorrem nas mesmas penas o proprietário, o ge-
§ 3o As pessoas referidas no § 2o deste artigo deverão
rente ou o responsável pelo local em que se verifique a
manter sob sigilo o material ilícito referido.
submissão de criança ou adolescente às práticas referidas
no caputdeste artigo.
Art. 241-C. Simular a participação de criança ou ado- § 2o Constitui efeito obrigatório da condenação a cas-
lescente em cena de sexo explícito ou pornográfica por sação da licença de localização e de funcionamento do
meio de adulteração, montagem ou modificação de foto- estabelecimento.
grafia, vídeo ou qualquer outra forma de representação
visual: Art. 244-B. Corromper ou facilitar a corrupção de me-
Pena – reclusão, de 1 (um) a 3 (três) anos, e multa. nor de 18 (dezoito) anos, com ele praticando infração penal
Parágrafo único. Incorre nas mesmas penas quem
LEGISLAÇÃO ESPECÍFICA

ou induzindo-o a praticá-la:
vende, expõe à venda, disponibiliza, distribui, publica ou Pena - reclusão, de 1 (um) a 4 (quatro) anos.
divulga por qualquer meio, adquire, possui ou armazena o § 1o Incorre nas penas previstas no caput deste artigo
material produzido na forma do caput deste artigo. quem pratica as condutas ali tipificadas utilizando-se de quais-
quer meios eletrônicos, inclusive salas de bate-papo da internet.
Art. 241-D. Aliciar, assediar, instigar ou constranger, § 2o As penas previstas no caput deste artigo são au-
por qualquer meio de comunicação, criança, com o fim mentadas de um terço no caso de a infração cometida ou
de com ela praticar ato libidinoso: induzida estar incluída no rol do art. 1o da Lei no 8.072, de
Pena – reclusão, de 1 (um) a 3 (três) anos, e multa. 25 de julho de 1990.

131
Capítulo II § 2º Se comprovada a reincidência em período infe-
Das Infrações Administrativas rior a 30 (trinta) dias, o estabelecimento será definitiva-
mente fechado e terá sua licença cassada.
Art. 245. Deixar o médico, professor ou responsável por
estabelecimento de atenção à saúde e de ensino fundamen- Art. 251. Transportar criança ou adolescente, por qual-
tal, pré-escola ou creche, de comunicar à autoridade com- quer meio, com inobservância do disposto nos arts. 83, 84
petente os casos de que tenha conhecimento, envolvendo e 85 desta Lei:
suspeita ou confirmação de maus-tratos contra criança ou Pena - multa de três a vinte salários de referência, apli-
adolescente: cando-se o dobro em caso de reincidência.
Pena - multa de três a vinte salários de referência, apli-
cando-se o dobro em caso de reincidência. Art. 252. Deixar o responsável por diversão ou espetáculo
público de afixar, em lugar visível e de fácil acesso, à entrada
Art. 246. Impedir o responsável ou funcionário de enti- do local de exibição, informação destacada sobre a natureza
dade de atendimento o exercício dos direitos constantes nos da diversão ou espetáculo e a faixa etária especificada no
incisos II, III, VII, VIII e XI do art. 124 desta Lei: certificado de classificação:
Pena - multa de três a vinte salários de referência, apli- Pena - multa de três a vinte salários de referência, apli-
cando-se o dobro em caso de reincidência. cando-se o dobro em caso de reincidência.

Art. 247. Divulgar, total ou parcialmente, sem autoriza- Art. 253. Anunciar peças teatrais, filmes ou quaisquer re-
ção devida, por qualquer meio de comunicação, nome, ato presentações ou espetáculos, sem indicar os limites de idade
ou documento de procedimento policial, administrativo ou a que não se recomendem:
judicial relativo a criança ou adolescente a que se atribua Pena - multa de três a vinte salários de referência, du-
ato infracional: plicada em caso de reincidência, aplicável, separadamente,
Pena - multa de três a vinte salários de referência, apli- à casa de espetáculo e aos órgãos de divulgação ou publi-
cidade.
cando-se o dobro em caso de reincidência.
§ 1º Incorre na mesma pena quem exibe, total ou par-
Art. 254. Transmitir, através de rádio ou televisão, espe-
cialmente, fotografia de criança ou adolescente envolvido
táculo em horário diverso do autorizado ou sem aviso de sua
em ato infracional, ou qualquer ilustração que lhe diga res-
classificação:
peito ou se refira a atos que lhe sejam atribuídos, de forma
Pena - multa de vinte a cem salários de referência; dupli-
a permitir sua identificação, direta ou indiretamente.
cada em caso de reincidência a autoridade judiciária poderá
§ 2º Se o fato for praticado por órgão de imprensa
determinar a suspensão da programação da emissora por
ou emissora de rádio ou televisão, além da pena prevista até dois dias.
neste artigo, a autoridade judiciária poderá determinar a
apreensão da publicação ou a suspensão da programação Art. 255. Exibir filme, trailer, peça, amostra ou congêne-
da emissora até por dois dias, bem como da publicação do re classificado pelo órgão competente como inadequado às
periódico até por dois números. (Expressão declarada in- crianças ou adolescentes admitidos ao espetáculo:
constitucional pela ADIN 869-2). Pena - multa de vinte a cem salários de referência; na
reincidência, a autoridade poderá determinar a suspensão
Art. 248. Deixar de apresentar à autoridade judiciária de do espetáculo ou o fechamento do estabelecimento por até
seu domicílio, no prazo de cinco dias, com o fim de regula- quinze dias.
rizar a guarda, adolescente trazido de outra comarca para
a prestação de serviço doméstico, mesmo que autorizado Art. 256. Vender ou locar a criança ou adolescente fita de
pelos pais ou responsável: programação em vídeo, em desacordo com a classificação
Pena - multa de três a vinte salários de referência, apli- atribuída pelo órgão competente:
cando-se o dobro em caso de reincidência, independente- Pena - multa de três a vinte salários de referência; em
mente das despesas de retorno do adolescente, se for o caso. caso de reincidência, a autoridade judiciária poderá deter-
minar o fechamento do estabelecimento por até quinze dias.
Art. 249. Descumprir, dolosa ou culposamente, os deve-
res inerentes ao poder familiar ou decorrente de tutela ou Art. 257. Descumprir obrigação constante dos arts. 78 e
guarda, bem assim determinação da autoridade judiciária 79 desta Lei:
ou Conselho Tutelar: Pena - multa de três a vinte salários de referência, dupli-
Pena - multa de três a vinte salários de referência, apli- cando-se a pena em caso de reincidência, sem prejuízo de
cando-se o dobro em caso de reincidência. apreensão da revista ou publicação.
LEGISLAÇÃO ESPECÍFICA

Art. 250. Hospedar criança ou adolescente desacompa- Art. 258. Deixar o responsável pelo estabelecimento ou o
nhado dos pais ou responsável, ou sem autorização escrita empresário de observar o que dispõe esta Lei sobre o acesso
desses ou da autoridade judiciária, em hotel, pensão, motel de criança ou adolescente aos locais de diversão, ou sobre
ou congênere: sua participação no espetáculo:
Pena – multa. Pena - multa de três a vinte salários de referência; em
§ 1º Em caso de reincidência, sem prejuízo da pena de caso de reincidência, a autoridade judiciária poderá deter-
multa, a autoridade judiciária poderá determinar o fecha- minar o fechamento do estabelecimento por até quinze
mento do estabelecimento por até 15 (quinze) dias. dias.

132
Art. 258-A. Deixar a autoridade competente de pro- § 2o Os conselhos nacional, estaduais e municipais dos
videnciar a instalação e operacionalização dos cadastros direitos da criança e do adolescente fixarão critérios de
previstos no art. 50 e no § 11 do art. 101 desta Lei: utilização, por meio de planos de aplicação, das dotações
Pena - multa de R$ 1.000,00 (mil reais) a R$ 3.000,00 subsidiadas e demais receitas, aplicando necessariamente
(três mil reais). percentual para incentivo ao acolhimento, sob a forma de
Parágrafo único. Incorre nas mesmas penas a autori- guarda, de crianças e adolescentes e para programas de
dade que deixa de efetuar o cadastramento de crianças e atenção integral à primeira infância em áreas de maior
de adolescentes em condições de serem adotadas, de pes- carência socioeconômica e em situações de calamidade.
soas ou casais habilitados à adoção e de crianças e adoles- § 3º O Departamento da Receita Federal, do Ministé-
centes em regime de acolhimento institucional ou familiar. rio da Economia, Fazenda e Planejamento, regulamentará
a comprovação das doações feitas aos fundos, nos ter-
Art. 258-B. Deixar o médico, enfermeiro ou dirigen- mos deste artigo.
te de estabelecimento de atenção à saúde de gestante de § 4º O Ministério Público determinará em cada co-
efetuar imediato encaminhamento à autoridade judiciária marca a forma de fiscalização da aplicação, pelo Fundo
de caso de que tenha conhecimento de mãe ou gestante Municipal dos Direitos da Criança e do Adolescente, dos
interessada em entregar seu filho para adoção: incentivos fiscais referidos neste artigo.
Pena - multa de R$ 1.000,00 (mil reais) a R$ 3.000,00 § 5o Observado o disposto no § 4o do art. 3o da Lei
(três mil reais). n 9.249, de 26 de dezembro de 1995, a dedução de que
o

Parágrafo único. Incorre na mesma pena o funcionário trata o inciso I do caput:


de programa oficial ou comunitário destinado à garantia I - será considerada isoladamente, não se submetendo
do direito à convivência familiar que deixa de efetuar a a limite em conjunto com outras deduções do imposto; e
comunicação referida no caput deste artigo. II - não poderá ser computada como despesa operacio-
nal na apuração do lucro real.
Art. 258-C. Descumprir a proibição estabelecida no
Art. 260-A. A partir do exercício de 2010, ano-calen-
inciso II do art. 81:
dário de 2009, a pessoa física poderá optar pela doação de
Pena - multa de R$ 3.000,00 (três mil reais) a R$
que trata o inciso II do caput do art. 260 diretamente em
10.000,00 (dez mil reais);
sua Declaração de Ajuste Anual.
Medida Administrativa - interdição do estabelecimento
§ 1o A doação de que trata o caput poderá ser deduzi-
comercial até o recolhimento da multa aplicada.
da até os seguintes percentuais aplicados sobre o imposto
apurado na declaração:
Disposições Finais e Transitórias
I - (VETADO);
II - (VETADO);
Art. 259. A União, no prazo de noventa dias contados
III - 3% (três por cento) a partir do exercício de 2012.
da publicação deste Estatuto, elaborará projeto de lei dis- § 2o A dedução de que trata o caput:
pondo sobre a criação ou adaptação de seus órgãos às I - está sujeita ao limite de 6% (seis por cento) do im-
diretrizes da política de atendimento fixadas no art. 88 e posto sobre a renda apurado na declaração de que trata o
ao que estabelece o Título V do Livro II. inciso II do caput do art. 260;
Parágrafo único. Compete aos estados e municípios II - não se aplica à pessoa física que:
promoverem a adaptação de seus órgãos e programas às a) utilizar o desconto simplificado;
diretrizes e princípios estabelecidos nesta Lei. b) apresentar declaração em formulário; ou
c) entregar a declaração fora do prazo;
Art. 260. Os contribuintes poderão efetuar doações aos III - só se aplica às doações em espécie; e
Fundos dos Direitos da Criança e do Adolescente nacional, IV - não exclui ou reduz outros benefícios ou deduções
distrital, estaduais ou municipais, devidamente comprova- em vigor.
das, sendo essas integralmente deduzidas do imposto de § 3o O pagamento da doação deve ser efetuado até a
renda, obedecidos os seguintes limites: data de vencimento da primeira quota ou quota única do
I - 1% (um por cento) do imposto sobre a renda devido imposto, observadas instruções específicas da Secretaria
apurado pelas pessoas jurídicas tributadas com base no da Receita Federal do Brasil.
lucro real; e § 4o O não pagamento da doação no prazo estabe-
II - 6% (seis por cento) do imposto sobre a renda apu- lecido no § 3o implica a glosa definitiva desta parcela de
rado pelas pessoas físicas na Declaração de Ajuste Anual, dedução, ficando a pessoa física obrigada ao recolhimento
observado o disposto no art. 22 da Lei no 9.532, de 10 de da diferença de imposto devido apurado na Declaração
dezembro de 1997.
LEGISLAÇÃO ESPECÍFICA

de Ajuste Anual com os acréscimos legais previstos na le-


§ 1º - (Revogado) gislação.
§ 1o-A. Na definição das prioridades a serem atendidas § 5o A pessoa física poderá deduzir do imposto apu-
com os recursos captados pelos fundos nacional, estaduais rado na Declaração de Ajuste Anual as doações feitas, no
e municipais dos direitos da criança e do adolescente, se- respectivo ano-calendário, aos fundos controlados pelos
rão consideradas as disposições do Plano Nacional de Pro- Conselhos dos Direitos da Criança e do Adolescente mu-
moção, Proteção e Defesa do Direito de Crianças e Adoles- nicipais, distrital, estaduais e nacional concomitantemen-
centes à Convivência Familiar e Comunitária e as do Plano te com a opção de que trata o caput, respeitado o limite
Nacional pela Primeira Infância. previsto no inciso II do art. 260.

133
Art. 260-B. A doação de que trata o inciso I do art. 260 Art. 260-G. Os órgãos responsáveis pela administração
poderá ser deduzida: das contas dos Fundos dos Direitos da Criança e do Ado-
I - do imposto devido no trimestre, para as pessoas ju- lescente nacional, estaduais, distrital e municipais devem:
rídicas que apuram o imposto trimestralmente; e I - manter conta bancária específica destinada exclusi-
II - do imposto devido mensalmente e no ajuste anual, vamente a gerir os recursos do Fundo;
para as pessoas jurídicas que apuram o imposto anual- II - manter controle das doações recebidas; e
mente. III - informar anualmente à Secretaria da Receita Fede-
Parágrafo único. A doação deverá ser efetuada dentro ral do Brasil as doações recebidas mês a mês, identificando
do período a que se refere a apuração do imposto. os seguintes dados por doador:
a) nome, CNPJ ou CPF;
Art. 260-C. As doações de que trata o art. 260 desta Lei b) valor doado, especificando se a doação foi em espé-
podem ser efetuadas em espécie ou em bens. cie ou em bens.
Parágrafo único. As doações efetuadas em espécie de-
vem ser depositadas em conta específica, em instituição Art. 260-H. Em caso de descumprimento das obriga-
financeira pública, vinculadas aos respectivos fundos de ções previstas no art. 260-G, a Secretaria da Receita Fe-
que trata o art. 260. deral do Brasil dará conhecimento do fato ao Ministério
Público.
Art. 260-D. Os órgãos responsáveis pela administra-
ção das contas dos Fundos dos Direitos da Criança e do Art. 260-I. Os Conselhos dos Direitos da Criança e do
Adolescente nacional, estaduais, distrital e municipais de- Adolescente nacional, estaduais, distrital e municipais di-
vem emitir recibo em favor do doador, assinado por pessoa vulgarão amplamente à comunidade:
competente e pelo presidente do Conselho corresponden- I - o calendário de suas reuniões;
te, especificando: II - as ações prioritárias para aplicação das políticas de
I - número de ordem; atendimento à criança e ao adolescente;
II - nome, Cadastro Nacional da Pessoa Jurídica (CNPJ) III - os requisitos para a apresentação de projetos a
e endereço do emitente; serem beneficiados com recursos dos Fundos dos Direitos
da Criança e do Adolescente nacional, estaduais, distrital
III - nome, CNPJ ou Cadastro de Pessoas Físicas (CPF)
ou municipais;
do doador;
IV - a relação dos projetos aprovados em cada ano-ca-
IV - data da doação e valor efetivamente recebido; e
lendário e o valor dos recursos previstos para implementa-
V - ano-calendário a que se refere a doação.
ção das ações, por projeto;
§ 1o O comprovante de que trata o caput deste artigo
V - o total dos recursos recebidos e a respectiva desti-
pode ser emitido anualmente, desde que discrimine os va-
nação, por projeto atendido, inclusive com cadastramento
lores doados mês a mês.
na base de dados do Sistema de Informações sobre a In-
§ 2o No caso de doação em bens, o comprovante
fância e a Adolescência; e
deve conter a identificação dos bens, mediante descrição VI - a avaliação dos resultados dos projetos beneficia-
em campo próprio ou em relação anexa ao comprovante, dos com recursos dos Fundos dos Direitos da Criança e do
informando também se houve avaliação, o nome, CPF ou Adolescente nacional, estaduais, distrital e municipais.
CNPJ e endereço dos avaliadores.
Art. 260-J. O Ministério Público determinará, em cada
Art. 260-E. Na hipótese da doação em bens, o doador Comarca, a forma de fiscalização da aplicação dos incen-
deverá: tivos fiscais referidos no art. 260 desta Lei.
I - comprovar a propriedade dos bens, mediante docu- Parágrafo único. O descumprimento do disposto nos
mentação hábil; arts. 260-G e 260-I sujeitará os infratores a responder por
II - baixar os bens doados na declaração de bens e di- ação judicial proposta pelo Ministério Público, que poderá
reitos, quando se tratar de pessoa física, e na escrituração, atuar de ofício, a requerimento ou representação de qual-
no caso de pessoa jurídica; e quer cidadão.
III - considerar como valor dos bens doados:
a) para as pessoas físicas, o valor constante da última Art. 260-K. A Secretaria de Direitos Humanos da Pre-
declaração do imposto de renda, desde que não exceda o sidência da República (SDH/PR) encaminhará à Secretaria
valor de mercado; da Receita Federal do Brasil, até 31 de outubro de cada
b) para as pessoas jurídicas, o valor contábil dos bens. ano, arquivo eletrônico contendo a relação atualizada dos
Parágrafo único. O preço obtido em caso de leilão Fundos dos Direitos da Criança e do Adolescente nacio-
não será considerado na determinação do valor dos bens nal, distrital, estaduais e municipais, com a indicação dos
LEGISLAÇÃO ESPECÍFICA

doados, exceto se o leilão for determinado por autoridade respectivos números de inscrição no CNPJ e das contas
judiciária. bancárias específicas mantidas em instituições financeiras
públicas, destinadas exclusivamente a gerir os recursos dos
Art. 260-F. Os documentos a que se referem os arts. Fundos.
260-D e 260-E devem ser mantidos pelo contribuinte por
um prazo de 5 (cinco) anos para fins de comprovação da Art. 260-L. A Secretaria da Receita Federal do Brasil
dedução perante a Receita Federal do Brasil. expedirá as instruções necessárias à aplicação do disposto
nos arts. 260 a 260-K.

134
Art. 261. A falta dos conselhos municipais dos direitos Art. 266. Esta Lei entra em vigor noventa dias após sua
da criança e do adolescente, os registros, inscrições e alte- publicação.
rações a que se referem os arts. 90, parágrafo único, e 91 Parágrafo único. Durante o período de vacância deve-
desta Lei serão efetuados perante a autoridade judiciária rão ser promovidas atividades e campanhas de divulgação
da comarca a que pertencer a entidade. e esclarecimentos acerca do disposto nesta Lei.
Parágrafo único. A União fica autorizada a repassar aos
estados e municípios, e os estados aos municípios, os recur- Art. 267. Revogam-se as Leis nº 4.513, de 1964, e
sos referentes aos programas e atividades previstos nesta 6.697, de 10 de outubro de 1979 (Código de Menores), e
Lei, tão logo estejam criados os conselhos dos direitos da as demais disposições em contrário.
criança e do adolescente nos seus respectivos níveis.
Brasília, 13 de julho de 1990; 169º da Independência e
Art. 262. Enquanto não instalados os Conselhos Tute- 102º da República.
lares, as atribuições a eles conferidas serão exercidas pela
autoridade judiciária.

Art. 263. O Decreto-Lei nº 2.848, de 7 de dezembro de EXERCÍCIOS COMENTADOS


1940 (Código Penal), passa a vigorar com as seguintes al-
terações: 1. (FCC/2014 - Prefeitura de Recife/PE - Procurador)
1) Art. 121 (...) Nos termos do art. 226 da Constituição Federal, “a famí-
§ 4º No homicídio culposo, a pena é aumentada de um lia, base da sociedade, tem especial proteção do Estado”.
terço, se o crime resulta de inobservância de regra técnica Entre os aspectos abrangidos pelo direito à proteção es-
de profissão, arte ou ofício, ou se o agente deixa de prestar pecial, segundo o texto constitucional, encontram-se os
imediato socorro à vítima, não procura diminuir as con- seguintes:
sequências do seu ato, ou foge para evitar prisão em fla-
grante. Sendo doloso o homicídio, a pena é aumentada de a) garantia de direitos previdenciários e trabalhistas; e
um terço, se o crime é praticado contra pessoa menor de obediência aos princípios de brevidade, excepciona-
catorze anos. lidade e respeito à condição peculiar de pessoa em
2) Art. 129 (...)
desenvolvimento, quando da aplicação de qualquer
§ 7º Aumenta-se a pena de um terço, se ocorrer qual-
medida privativa da liberdade.
quer das hipóteses do art. 121, § 4º.
b) garantia de direitos previdenciários e trabalhistas; e
§ 8º Aplica-se à lesão culposa o disposto no § 5º do
acesso universal à educação infantil, em creche e pré-
art. 121.
-escola, às crianças até 5 (cinco) anos de idade.
3) Art. 136 (...)
c) erradicação do analfabetismo; e estímulo do Poder Pú-
§ 3º Aumenta-se a pena de um terço, se o crime é pra-
blico, através de assistência jurídica, incentivos fiscais
ticado contra pessoa menor de catorze anos.
4) Art. 213 (...) e subsídios, nos termos da lei, ao acolhimento, sob a
Parágrafo único. Se a ofendida é menor de catorze forma de guarda, de criança ou adolescente órfão ou
anos: abandonado.
Pena - reclusão de quatro a dez anos. d) punição severa ao abuso, à violência e à exploração
5) Art. 214 (...) sexual da criança e do adolescente; e garantia às pre-
Parágrafo único. Se o ofendido é menor de catorze anos: sidiárias de condições para que possam permanecer
Pena - reclusão de três a nove anos.” com seus filhos durante o período de amamentação.
e) punição severa ao abuso, à violência e à exploração
Art. 264. O art. 102 da Lei n.º 6.015, de 31 de dezem- sexual da criança e do adolescente; e estímulo do Po-
bro de 1973, fica acrescido do seguinte item: der Público, através de assistência jurídica, incentivos
“Art. 102 (...) fiscais e subsídios, nos termos da lei, ao acolhimento,
6º) a perda e a suspensão do pátrio poder.” sob a forma de guarda, de criança ou adolescente ór-
fão ou abandonado.
Art. 265. A Imprensa Nacional e demais gráficas da
União, da administração direta ou indireta, inclusive fun- Resposta: Letra A. O artigo 227, §3º, CF fixa os as-
dações instituídas e mantidas pelo poder público federal pectos que abrangem a proteção especial da criança
promoverão edição popular do texto integral deste Estatu- e do adolescente: “I - idade mínima de quatorze anos
to, que será posto à disposição das escolas e das entidades para admissão ao trabalho, observado o disposto no
de atendimento e de defesa dos direitos da criança e do art. 7º, XXXIII; II - garantia de direitos previdenciários
e trabalhistas; III - garantia de acesso do trabalhador
LEGISLAÇÃO ESPECÍFICA

adolescente.
adolescente e jovem à escola; IV - garantia de pleno
Art. 265-A. O poder público fará periodicamente am- e formal conhecimento da atribuição de ato infracio-
pla divulgação dos direitos da criança e do adolescente nos nal, igualdade na relação processual e defesa técnica
meios de comunicação social. por profissional habilitado, segundo dispuser a legis-
Parágrafo único. A divulgação a que se refere o caput lação tutelar específica; V - obediência aos princípios
será veiculada em linguagem clara, compreensível e ade- de brevidade, excepcionalidade e respeito à condição
quada a crianças e adolescentes, especialmente às crian- peculiar de pessoa em desenvolvimento, quando da
ças com idade inferior a 6 (seis) anos. aplicação de qualquer medida privativa da liberdade;

135
VI - estímulo do Poder Público, através de assistência que é vedada a adoção por procuração, pois a adoção
jurídica, incentivos fiscais e subsídios, nos termos da é ato personalíssimo (art. 39, § 2º, ECA); a alternativa
lei, ao acolhimento, sob a forma de guarda, de criança “d” está incorreta porque é possível a adoção conjunta
ou adolescente órfão ou abandonado; VII - programas desde que preencha os requisitos de serem casados
de prevenção e atendimento especializado à criança, civilmente ou mantenham união estável, comprovada
ao adolescente e ao jovem dependente de entorpe- a estabilidade da família (art. 42, § 1º, ECA); e a alter-
centes e drogas afins”. nativa “e” está incorreta porque pode ser dispensado
o estágio de convivência quando o adotando já estiver
2. (Alternative Concursos/2017 - Prefeitura de Sul sob a tutela ou guarda do adotante (art. 46, § 1º, ECA).
Brasil/SC - Agente Educativo) De acordo com o Estatu-
to da Criança e do Adolescente, Lei n.º 8.069/90, art. 60, 4. (FCC/2016 - AL-MS - Agente de Polícia Legislativo)
é proibido qualquer trabalho a menores: Sobre a prática de ato infracional à luz do Estatuto da
Criança e do Adolescente, é INCORRETO afirmar que a
a) De quatorze anos de idade, inclusive na condição de
aprendiz. a) medida socioeducativa de internação pode ser deter-
b) De quatorze anos de idade, salvo na condição de minada por descumprimento reiterado e injustificável
aprendiz. da medida anteriormente imposta.
c) De dezesseis anos de idade, salvo na condição de b) internação, antes da sentença, poderá ser determinada
aprendiz. pelo prazo máximo de quarenta e cinco dias.
d) De dezesseis anos de idade, inclusive na condição de c) medida socioeducativa de internação não poderá ex-
aprendiz. ceder em nenhuma hipótese três anos, liberando-se
e) De dezessete anos de idade, inclusive na condição de compulsoriamente o menor infrator aos vinte e um
aprendiz. anos de idade.
d) medida socioeducativa de liberdade assistida será
Resposta: Letra B. Em que pese o teor do art. 64 do fixada pelo prazo mínimo de trinta dias, podendo a
ECA, que poderia dar a entender que um menor de 14 qualquer tempo ser prorrogada, revogada ou substi-
anos pode trabalhar, prevalece o que diz o texto da tuída por outra medida, ouvido o orientador, o Minis-
Constituição Federal: “Art. 7º São direitos dos traba- tério Público e o defensor.
lhadores urbanos e rurais, além de outros que visem à e) remissão não implica necessariamente o reconheci-
melhoria de sua condição social: [...] XXXIII - proibição mento ou comprovação da responsabilidade, nem
de trabalho noturno, perigoso ou insalubre a menores prevalece para efeito de antecedentes, podendo in-
de dezoito e de qualquer trabalho a menores de de- cluir eventualmente a aplicação de qualquer das me-
zesseis anos, salvo na condição de aprendiz, a partir didas previstas em lei, exceto a colocação em regime
de quatorze anos”. Logo, o menor pode trabalhar em de semiliberdade e a internação.
qualquer serviço, desde que não seja noturno, peri-
goso e insalubre, dos 16 aos 18 anos; e entre 14 e 16 Resposta: Letra D. A lei exige como prazo mínimo de
anos apenas pode trabalhar como aprendiz. medida socioeducativa o período de 6 meses, confor-
me art. 118, § 2º, ECA, não 30 dias conforme a alter-
3. (FCC/2016 - AL-MS - Agente de Polícia Legislativo) nativa “d”, razão pela qual está incorreta. A alternativa
Sobre a adoção, nos termos preconizados pelo Estatuto “a” está prevista no art. 122, § 1º, ECA; a alternativa “b”
da Criança e do Adolescente, está prevista no art. 108 do ECA; a alternativa “c” está
prevista no art. 121, §§ 3º e 5º, ECA; a alternativa “e”
a) o adotante deve ser, no mínimo, 18 anos mais velho está prevista no art. 127 ECA.
que o adotando.
b) é permitida a adoção por procuração. 5. (COMPERVE/2016 - Câmara de Natal/RN - Guarda
c) se um dos cônjuges adota o filho do outro, mantêm-se Legislativo) As crianças e os adolescentes, qualificados
os vínculos de filiação entre o adotado e o cônjuge do pelo direito hoje vigente como pessoas em desenvolvi-
adotante e os respectivos parentes. mento, receberam do direito positivo brasileiro, tutela
d) é vedada a adoção conjunta pelos divorciados, separa- especial através da Lei nº 8.069, de 13 de julho de 1990,
dos judicialmente e pelos ex-companheiros. mais conhecida como Estatuto da Criança e do Adoles-
e) o estágio de convivência que precede a adoção não cente. Seguindo as diretrizes traçadas pela Constituição
poderá, em nenhuma hipótese, ser dispensado pela de 1988, o Estatuto da Criança e do Adolescente trouxe a
autoridade judiciária. previsão normativa da absoluta prioridade e de variados
direitos fundamentais. Em tal seara, foi determinado que
LEGISLAÇÃO ESPECÍFICA

Resposta: Letra C. Neste sentido, disciplina o art. 41, as crianças e os adolescentes têm direito,
§ 1º, ECA: “Se um dos cônjuges ou concubinos adota o
filho do outro, mantêm-se os vínculos de filiação entre a) à liberdade, de forma a compreender a liberdade de
o adotado e o cônjuge ou concubino do adotante e ir, vir e estar nos logradouros públicos e espaços co-
os respectivos parentes”. A alternativa “a” está errada munitários, ressalvadas as restrições legais; a liberdade
porque o adotante deve ser, pelo menos, 16 anos mais de opinião e de expressão; a liberdade de brincar e
velho que o adotado e possuir pelo menos 18 anos de praticar esportes, a liberdade de participar da vida
(art. 42, § 3º, ECA); a alternativa “b” está incorreta por- familiar e comunitária; a liberdade de buscar refúgio,

136
auxílio e orientação, excetuadas dessa tutela a liberda- e) a pessoa até 14 anos incompletos, desde que não te-
de de crença e culto religioso e de participar da vida nha cometido nenhum crime.
política.
b) ao respeito, consistente na inviolabilidade da sua in- Resposta: Letra C. O Estatuto da Criança e do Adoles-
tegridade física, psíquica e moral, abrangendo a pre- cente opta por categorizar separadamente estas duas
servação da imagem, da identidade, da autonomia, categorias de menores. Criança é aquele que tem até
de seus valores, ideias e crenças, excluída a tutela dos 12 anos de idade (na data de aniversário de 12 anos,
seus espaços e objetos pessoais. passa a ser adolescente), adolescente é aquele que
c) de serem educados e cuidados sem o uso de castigo fí- tem entre 12 e 18 anos (na data de aniversário de 18
sico ou de tratamento cruel ou degradante, como for- anos, passa a ser maior), conforme o artigo 2º do ECA.
mas de correção, disciplina, educação ou a qualquer
outro pretexto, por parte dos pais, de integrantes da 7) (Câmara dos Deputados - Analista Legislativo -
família ampliada, dos responsáveis, dos agentes pú- CESPE/2014) Julgue o próximo item , referente ao dis-
blicos executores de medidas socioeducativas ou por posto no Estatuto da Criança e do Adolescente (ECA) e às
qualquer pessoa encarregada de cuidar deles, tratá- atribuições do conselho tutelar.
-los, educá-los ou protegê-los. As disposições do ECA aplicam-se apenas a crianças, in-
d) de serem criados e educados no seio de sua família divíduos até doze anos de idade incompletos, e a ado-
biológica, não se admitindo a sua inserção em família lescentes, indivíduos entre doze e dezoito anos de idade.
substituta, assegurada a convivência familiar e comu-
nitária, em ambiente que garanta seu desenvolvimen- ( ) Certo ( ) Errado
to integral.
Resposta: Errado. Preconiza o artigo 2º, parágrafo
Resposta: Letra C. Nestes termos, preconiza o artigo único, ECA: “Nos casos expressos em lei, aplica-se ex-
18-A do ECA: “A criança e o adolescente têm o direi- cepcionalmente este Estatuto às pessoas entre dezoito
to de ser educados e cuidados sem o uso de castigo anos e vinte e um anos de idade”.
físico ou de tratamento cruel ou degradante, como
formas de correção, disciplina, educação ou qualquer 8) (Câmara dos Deputados - Analista Legislativo -
outro pretexto, pelos pais, pelos integrantes da famí- CESPE/2014) Acerca de ato infracional e medidas so-
lia ampliada, pelos responsáveis, pelos agentes pú- cioeducativas, bem como dos crimes e infrações prati-
blicos executores de medidas socioeducativas ou por cados contra a criança e o adolescente, julgue o item a
qualquer pessoa encarregada de cuidar deles, tratá- seguir.
-los, educá-los ou protegê-los”. A alternativa “a” está Caso um adolescente que faça parte de um grupo for-
errada porque o artigo 16 do ECA fixa que o direito mado por adultos e que já tenha praticado, comprova-
à liberdade envolve os seguintes aspectos: “I - ir, vir damente, diversos roubos com uso de arma de fogo seja
e estar nos logradouros públicos e espaços comuni- apreendido, a ele deverá ser imposta – após o devido
tários, ressalvadas as restrições legais; II - opinião e procedimento judicial – a medida socioeducativa deno-
expressão; III - crença e culto religioso; IV - brincar, minada liberdade assistida.
praticar esportes e divertir-se; V - participar da vida
familiar e comunitária, sem discriminação; VI - parti- ( ) Certo ( ) Errado
cipar da vida política, na forma da lei; VII - buscar
refúgio, auxílio e orientação”. A alternativa “b” está er- Resposta: Errado. Nos termos do artigo 118, ECA, “a
rada porque o artigo 17 do ECA prevê que “o direito liberdade assistida será adotada sempre que se afigu-
ao respeito consiste na inviolabilidade da integridade rar a medida mais adequada para o fim de acompa-
física, psíquica e moral da criança e do adolescente, nhar, auxiliar e orientar o adolescente”. Ocorre que o
abrangendo a preservação da imagem, da identidade, adolescente praticou crimes com violência ou grave
da autonomia, dos valores, ideias e crenças, dos espa- ameaça, razão pela qual cabe internação, conforme
ços e objetos pessoais”. A alternativa “d” está errada artigo 122, I, ECA: “A medida de internação só poderá
porque o artigo 18 do ECA assegura que “é direito da ser aplicada quando: I - tratar-se de ato infracional co-
criança e do adolescente ser criado e educado no seio metido mediante grave ameaça ou violência a pessoa”.
de sua família e, excepcionalmente, em família substi-
tuta, assegurada a convivência familiar e comunitária, 9) (Câmara dos Deputados - Técnico Legislativo - CES-
em ambiente que garanta seu desenvolvimento inte- PE/2014) Paulo e João foram surpreendidos nas depen-
gral”. dências da Câmara dos Deputados quando subtraíam
carteiras e celulares dos casacos e bolsas de pessoas que
LEGISLAÇÃO ESPECÍFICA

6. (FUNRIO/2016 - IF-PA - Assistente de Alunos) ali transitavam. Paulo tem dezessete anos e teve acesso
Segundo o Estatuto da Criança e do Adolescente (Lei ao local por intermédio de João, que é servidor da Casa.
8.069/90), é considerado criança Com base nessa situação hipotética, julgue o item a seguir.
As medidas socioeducativas aplicáveis a Paulo incluem a
a) a pessoa até seis anos incompletos de idade. advertência, a obrigação de reparar o dano e a prestação
b) a pessoa até oito anos incompletos de idade. de serviços a comunidade.
c) a pessoa até 12 anos incompletos de idade.
d) a pessoa até 18 anos incompletos de idade. ( ) Certo ( ) Errado

137
Resposta: Certo. Prevê o artigo 112, ECA: “Verificada Normas gerais de circulação e conduta.
a prática de ato infracional, a autoridade competente O Código de Trânsito Brasileiro estabelece normas
poderá aplicar ao adolescente as seguintes medidas: I de circulação em relação aos usuários das vias terrestres,
- advertência; II - obrigação de reparar o dano; III - pres- bem como a forma de organização das vias para circula-
tação de serviços à comunidade; IV - liberdade assisti- ção dos veículos.
da; V - inserção em regime de semi-liberdade; VI - inter- O art. 26 preceitua que os usuários das vias terrestres
nação em estabelecimento educacional; VII - qualquer devem:
uma das previstas no art. 101, I a VI”. - abster-se de todo ato que possa constituir perigo ou
obstáculo para o trânsito de veículos, de pessoas ou de
animais, ou ainda causar danos a propriedades públicas
LEI 9.503, DE 23 DE SETEMBRO DE 1997. ou privadas;
(CÓDIGO DE TRÂNSITO BRASILEIRO); - abster-se de obstruir o trânsito ou torná-lo perigo-
DISPOSIÇÕES PRELIMINARES; DO SISTEMA so, atirando, depositando ou abandonando na via objetos
NACIONAL DE TRÂNSITO; DAS NORMAS ou substâncias, ou nela criando qualquer outro obstáculo.
Os usuários das vias terrestres, portanto, devem
GERAIS DE CIRCULAÇÃO E CONDUTA; DA abster-se de praticar qualquer conduta que possa trazer
HABILITAÇÃO; DOS CRIMES DE TRÂNSITO. qualquer risco a todos que possam circular na via, inclu-
sive animais.
Também não podem praticar qualquer conduta que
Sistema Nacional de Trânsito: disposições gerais; possa ocasionar danos nas propriedades, sejam elas pú-
composição e competência do Sistema Nacional de blicas como as ruas e avenidas, por exemplo ou privadas
Trânsito. como os imóveis.
O Sistema Nacional de Trânsito, conforme preceitua Ademais, os usuários também devem abster-se de
o art. 5º do Código de Trânsito, é o conjunto de órgãos deixar qualquer objeto na via que possa ocasionar qual-
e entidades da União, dos Estados, do Distrito Federal e quer tipo de risco.
dos Municípios. Dentre outras das normas de conduta previstas pelo
Compete ao SINETRAN, o exercício das atividades de CTB estão:
planejamento, administração, normatização, pesquisa, - Observar as condições do veículo, mantendo equi-
registro e licenciamento de veículos, formação, habilita- pamentos em boas condições de funcionamento, bem
ção e reciclagem de condutores, educação, engenharia, como atentando para a existência de combustível sufi-
operação do sistema viário, policiamento, fiscalização, ciente, de forma que não haja qualquer parada imprevista
julgamento de infrações e de recursos e aplicação de pe- do veículo na via.
nalidades. Art. 27. Antes de colocar o veículo em circulação
Seus objetivos básicos estão estabelecidos no art. 6º nas vias públicas, o condutor deverá verificar a exis-
tência e as boas condições de funcionamento dos equi-
e são os seguintes:
pamentos de uso obrigatório, bem como assegurar-se
- estabelecer diretrizes da Política Nacional de Trân-
da existência de combustível suficiente para chegar ao
sito, com vistas à segurança, à fluidez, ao conforto, à de-
local de destino.
fesa ambiental e à educação para o trânsito, e fiscalizar
- Cabe ao condutor ter domínio de seu veículo, com a
seu cumprimento;
observância dos cuidados do trânsito, conforme previsto
- fixar, mediante normas e procedimentos, a padroni-
no art. 28.
zação de critérios técnicos, financeiros e administrativos
Art. 28. O condutor deverá, a todo momento, ter
para a execução das atividades de trânsito; domínio de seu veículo, dirigindo-o com atenção e cui-
- estabelecer a sistemática de fluxos permanentes de in- dados indispensáveis à segurança do trânsito.
formações entre os seus diversos órgãos e entidades, a fim Outro dos deveres do condutor é manter o domínio
de facilitar o processo decisório e a integração do Sistema. do seu veículo. Deve dirigir com cuidado e atenção indis-
É composto pelos seguintes órgãos e entidades pre- pensáveis para a manutenção da segurança no trânsito.
vistos no art. 7º do Código mencionado acima: Nas vias terrestres, tendo em vista o excesso de veícu-
- o Conselho Nacional de Trânsito - CONTRAN, coor- los, devem ser observadas normas de circulação.
denador do Sistema e órgão máximo normativo e con- Destaca-se que a circulação deve ocorrer pelo lado
sultivo; direito, admitindo exceções, desde que devidamente si-
- os Conselhos Estaduais de Trânsito - CETRAN e o nalizadas.
Conselho de Trânsito do Distrito Federal - CONTRANDI- O condutor deve também guardar distância lateral e
FE, órgãos normativos, consultivos e coordenadores; frontal em relação aos demais veículos e em relação à via.
- os órgãos e entidades executivos de trânsito da Art. 29. O trânsito de veículos nas vias terrestres
LEGISLAÇÃO ESPECÍFICA

União, dos Estados, do Distrito Federal e dos Municípios; abertas à circulação obedecerá às seguintes normas:
- os órgãos e entidades executivos rodoviários da I - a circulação far-se-á pelo lado direito da via,
União, dos Estados, do Distrito Federal e dos Municípios; admitindo-se as exceções devidamente sinalizadas;
- a Polícia Rodoviária Federal; II - o condutor deverá guardar distância de segu-
- as Polícias Militares dos Estados e do Distrito Fe- rança lateral e frontal entre o seu e os demais veículos,
deral; e bem como em relação ao bordo da pista, considerando-
- as Juntas Administrativas de Recursos de Infrações -se, no momento, a velocidade e as condições do local,
- JARI. da circulação, do veículo e as condições climáticas;

138
- Quando não houver sinalização da via, a pre- de trânsito, gozam de livre circulação, estacionamen-
ferência de passagem do condutor será da seguinte to e parada, quando em serviço de urgência e devida-
forma: mente identificados por dispositivos regulamentares
- daquele que estiver circulando na rodovia de um de alarme sonoro e iluminação vermelha intermiten-
fluxo único; te, observadas as seguintes disposições:
- na rotatória, a preferência será daquele que estiver a) quando os dispositivos estiverem acionados,
nela circulando; indicando a proximidade dos veículos, todos os con-
- nas outras situações, a preferência será do condu- dutores deverão deixar livre a passagem pela faixa
tor que vier pela direita. da esquerda, indo para a direita da via e parando, se
- Quando veículos, transitando por fluxos que se necessário;
cruzem, se aproximarem de local não sinalizado, terá b) os pedestres, ao ouvir o alarme sonoro, deve-
preferência de passagem: rão aguardar no passeio, só atravessando a via quan-
a) no caso de apenas um fluxo ser proveniente de do o veículo já tiver passado pelo local;
rodovia, aquele que estiver circulando por ela; c) o uso de dispositivos de alarme sonoro e de
b) no caso de rotatória, aquele que estiver circu- iluminação vermelha intermitente só poderá ocorrer
lando por ela; quando da efetiva prestação de serviço de urgência;
c) nos demais casos, o que vier pela direita do d) a prioridade de passagem na via e no cruza-
condutor; mento deverá se dar com velocidade reduzida e com
Em uma pista de rolamento em que haja várias faixas os devidos cuidados de segurança, obedecidas as de-
de circulação no mesmo sentido, os veículos mais lentos mais normas deste Código;
devem deslocar-se pela direita. Também devem manter- Quando se tratar de um veículo de utilidade pública,
-se na pista da direita aqueles veículos de maior porte, ele poderá parar e estacionar no local para prestação do
de forma que a esquerda fique livre para o deslocamento serviço. Deverá, porém, sinalizar sobre esta parada.
em maior velocidade. Os veículos prestadores de serviços de utilidade
- Quando uma pista de rolamento comportar vá- pública, quando em atendimento na via, gozam de
rias faixas de circulação no mesmo sentido, são as da livre parada e estacionamento no local da prestação
direita destinadas ao deslocamento dos veículos mais
de serviço, desde que devidamente sinalizados, de-
lentos e de maior porte, quando não houver faixa es-
vendo estar identificados na forma estabelecida pelo
pecial a eles destinada, e as da esquerda, destinadas
CONTRAN;
à ultrapassagem e ao deslocamento dos veículos de
Norma de circulação e conduta de grande importân-
maior velocidade;
cia e a que deve ser destinada muita atenção é sobre a
Outra regra de conduta de grande relevância: os veí-
ultrapassagem.
culos não poderão de forma injustificada transitar nas
Isto porque aquele que pretende fazer uma ultrapas-
calçadas, passeios e acostamentos. A exceção, porém,
sagem deverá observar o seguinte:
será para saída dos imóveis ou de áreas especiais de es-
tacionamento. - que o veículo que venha atrás também não preten-
Trânsito de veículos sobre passeios, calçadas e da ultrapassá-lo;
nos acostamentos, só poderá ocorrer para que se - que o veículo que venha logo à frente também não
adentre ou se saia dos imóveis ou áreas especiais de esteja efetuando uma ultrapassagem;
estacionamento; - que haja espaço suficiente na pista para que realize
a ultrapassagem, sem que haja qualquer risco de invasão
Veículos especiais da pista contrária;
Os veículos de batedores terão prioridade de passagem. Ainda: deverá o condutor indicar com antecedência
Em caso de veículos que prestem socorro, há prio- a manobra que pretende realizar, podendo fazê-lo por
ridades que lhe são garantidas como de livre circulação, meio da seta ou até mesmo utilizando-se do gesto con-
estacionamento e parada. vencional com o braço.
Estes veículos devem, porém, acionar dispositivos de A ultrapassagem de outro veículo em movimento
alarme sonoro e iluminação vermelha para que os de- deverá ser feita pela esquerda, obedecida a sinaliza-
mais condutores possam atentar-se da necessidade de ção regulamentar e as demais normas estabelecidas
sua passagem e deixar livre o lado esquerdo, inclusive, neste Código, exceto quando o veículo a ser ultra-
se necessário estacionando o carro para não impedir o passado estiver sinalizando o propósito de entrar à
trânsito do carro de socorro. esquerda;
Inclusive, para a passagem de veículos especiais, até X - todo condutor deverá, antes de efetuar uma
mesmo os pedestres devem atentar-se para as normas ultrapassagem, certificar-se de que:
LEGISLAÇÃO ESPECÍFICA

de conduta, devendo aguardar para realização da traves- a) nenhum condutor que venha atrás haja come-
sia, ainda que esteja aberta em seu favor. çado uma manobra para ultrapassá-lo;
Os veículos precedidos de batedores terão priori- b) quem o precede na mesma faixa de trânsito não
dade de passagem, respeitadas as demais normas de haja indicado o propósito de ultrapassar um terceiro;
circulação; c) a faixa de trânsito que vai tomar esteja livre
Os veículos destinados a socorro de incêndio e numa extensão suficiente para que sua manobra não
salvamento, os de polícia, os de fiscalização e opera- ponha em perigo ou obstrua o trânsito que venha em
ção de trânsito e as ambulâncias, além de prioridade sentido contrário;

139
XI - todo condutor ao efetuar a ultrapassagem de- Rodovias de pista dupla:
verá: - Para automóveis, camionetas e motocicletas: 110
a) indicar com antecedência a manobra pretendi- km/h;
da, acionando a luz indicadora de direção do veículo - Demais veículos: 90 km/h;
ou por meio de gesto convencional de braço;
b) afastar-se do usuário ou usuários aos quais ul- Rodovias de pista simples:
trapassa, de tal forma que deixe livre uma distância - Para automóveis, camionetas e mot;ocicletas: 100
lateral de segurança; km/h;
c) retomar, após a efetivação da manobra, a fai- - Para os demais veículos: 90 km/h.
xa de trânsito de origem, acionando a luz indicadora Estradas: 60 km/h.
de direção do veículo ou fazendo gesto convencional
de braço, adotando os cuidados necessários para não Transporte de crianças
pôr em perigo ou obstruir o trânsito dos veículos que Outra norma de conduta de grande relevância e inci-
ultrapassou; dência em provas trata do transporte de crianças com ida-
XII - os veículos que se deslocam sobre trilhos te- de inferior a dez anos que deve ocorrer sempre no banco
rão preferência de passagem sobre os demais, respei- traseiro, conforme preceitua o art. 64 do CTB.
tadas as normas de circulação. Sobre o transporte de crianças vale ressaltar que a
Outra importante norma de conduta se refere à proi- Resolução CONTRAN 277/2008 determinam que além do
bição de que o condutor ou passageiros deixem a porta transporte de crianças (até dez anos) ter que ocorrer no
do veículo aberta ou mesmo desçam do carro sem certi- banco traseiro, deverão sê-lo com cinto de segurança ou
ficarem-se de que há segurança para este desembarque. sistema de retenção equivalente.
Inclusive, o Código de Trânsito determina que em- Importa esclarecer que isto deve ocorrer da seguinte
barque e desembarque sempre devem ocorrer pelo lado forma:
da calçada, exceto se tratar-se do condutor. - Crianças com até um ano de idade: deverão ser trans-
Art. 49. O condutor e os passageiros não deverão portadas com dispositivo de retenção denominado “bebê
abrir a porta do veículo, deixá-la aberta ou descer conforto ou conversível”;
do veículo sem antes se certificarem de que isso não - Crianças com idade superior a um ano e inferior ou
constitui perigo para eles e para outros usuários da igual a quatro anos deverão ser transportadas com dispo-
via. sitivo de retenção denominado “cadeirinha”;
Parágrafo único. O embarque e o desembarque Crianças com idade superior a quatro anos e inferior
devem ocorrer sempre do lado da calçada, exceto ou igual a sete anos e meio deverão ser transportadas com
para o condutor. dispositivo de retenção denominado “assento de elevação”;
Classificação das vias abertas Crianças com idade superior a sete anos e meio e in-
O art. 60 traz importante classificação referente às ferior ou igual a dez anos deverão utilizar o cinto de segu-
vias abertas. Sendo que primeiramente se dividem em: rança do veículo.
vias urbanas e vias rurais.
Por sua vez, as vias urbanas podem ser: de trânsito Habilitação.
rápido, via arterial; via coletora e via local. Para que a pessoa possa obter sua habilitação para
Já as vias rurais se subdividem em rodovias e estra- condução de veículos automotor e elétrico deverá realizar
das. exames perante o órgão ou entidade executivos do Estado
O Código de Trânsito traz as velocidades permitidas ou Distrito Federal.
em cada uma das vias. O órgão competente para realização destes exames
Art. 60. As vias abertas à circulação, de acordo será do domicílio ou residência do candidato ou na sede
com sua utilização, classificam-se em: estadual ou distrital do próprio órgão, conforme previsto
I - vias urbanas: no art. 140 do CTB.
a) via de trânsito rápido; Para tanto, a pessoa deve preencher os seguintes re-
b) via arterial; quisitos:
c) via coletora; - ser penalmente imputável: a pessoa deverá ter ca-
d) via local; pacidade para poder ser responsabilizada por infrações
II - vias rurais: penais;
a) rodovias; - saber ler e escrever;
b) estradas. - possuir Carteira de Identidade ou equivalente.
O CTB determina que a velocidade máxima das vias A habilitação será concedida nas categorias da seguin-
será indicada por meio de sinalização (art. 61). Em sua te forma (art. 143):
ausência, porém, as velocidades vigentes serão: - Categoria A - condutor de veículo motorizado de
LEGISLAÇÃO ESPECÍFICA

Vias urbanas: duas ou três rodas, com ou sem carro lateral;


- 80 km/h: vias de trânsito rápido; - Categoria B - condutor de veículo motorizado, não
- 60 km/h: vias arteriais; abrangido pela categoria A, cujo peso bruto total não ex-
- 40 km/h: vias coletoras; ceda a três mil e quinhentos quilogramas e cuja lotação
- 30 km/h: vias locais. não exceda a oito lugares, excluído o do motorista;
Vale atenta-se para a velocidade das vias rurais, ten- - Categoria C - condutor de veículo motorizado utiliza-
do em vista terem sofrido importante alteração pela Lei do em transporte de carga, cujo peso bruto total exceda a
13.281/2016, conforme segue: três mil e quinhentos quilogramas;

140
Infrações. Penalidades. Medidas e processos ad-
FIQUE ATENTO! ministrativos.
Para que a pessoa busque sua habilitação Infrações de trânsito são as condutas que não ob-
na Categoria C, deverá estar habilitada há servem qualquer preceito previsto no Código de Trânsito
um ano na categoria B e não ter cometido Brasileiro, a legislação complementar ou qualquer das
nenhuma infração grave ou gravíssima, ou ser resoluções do CONTRAN, conforme determina o art. 161
reincidente em infrações médias, durante os do CTB.
últimos doze meses. O infrator estará sujeito às penalidades e medidas
administrativas determinadas no Código.
Os artigos 162 a 255 preceituam todas as condutas
consideradas infrações de trânsito, as respectivas penali-
- Categoria D - condutor de veículo motorizado utili- dades e medidas administrativas.
zado no transporte de passageiros, cuja lotação exceda a
Para consulta e estudo das infrações, veja este link:
oito lugares, excluído o do motorista;
http://www.planalto.gov.br/ccivil_03/leis/L9503.htm
- Categoria E - condutor de combinação de veículos
O art. 256 estabelece quais são as penalidades a que
em que a unidade tratora se enquadre nas categorias B,
estarão sujeitos os infratores de trânsito.
C ou D e cuja unidade acoplada, reboque, semirreboque,
trailer ou articulada tenha 6.000 kg (seis mil quilogramas) São as seguintes:
ou mais de peso bruto total, ou cuja lotação exceda a 8 - advertência por escrito;
(oito) lugares. - multa;
Ainda, importa dizer que o indivíduo para habilitar-se - suspensão do direito de dirigir;
nas categorias D e E ou ainda quando pretender conduzir - cassação da Carteira Nacional de Habilitação;
veículos de transporte coletivo de passageiros, escolares, - cassação da Permissão para Dirigir;
de emergência ou de produtos perigosos deverá preen- - frequência obrigatória em curso de reciclagem.
cher os seguintes requisitos (art. 145 CTB): Importa esclarecer que a pessoa que pratique uma
I - ser maior de vinte e um anos; infração de trânsito poderá sofrer uma penalidade, o
II - estar habilitado: que não impede que também sofra uma pena, caso esta
a) no mínimo há dois anos na categoria B, ou no infração constitua um ilício penal, ou seja, um crime de
mínimo há um ano na categoria C, quando pretender trânsito (art. 256, § 1º CTB).
habilitar-se na categoria D; e As penalidades podem ser impostas conta o condu-
b) no mínimo há um ano na categoria C, quando tor, proprietário, embarcador e transportador.
pretender habilitar-se na categoria E; O condutor, responderá sempre que a infração de-
III - não ter cometido nenhuma infração grave correr de ato praticado na direção do veículo (art. 257,
ou gravíssima ou ser reincidente em infrações médias § 3º CTB);
durante os últimos doze meses; O proprietário responderá por infrações sempre que
IV - ser aprovado em curso especializado e em a responsabilidade decorrer de prévia regularização e
curso de treinamento de prática veicular em situação preenchimento de formalidades exigidas para o trânsito
de risco, nos termos da normatização do CONTRAN. do veículo (art. 257, § 2º CTB);
Quando o candidato for aprovado, após a realização O embarcador responde pela infração relativa ao
dos exames, para dirigir receberá a Permissão para Dirigir transporte de carga com excesso de peso nos eixos ou
que terá validade de um ano (art. 148, § 2º). peso bruto total, desde que seja o único remetente da
Assim, transcorrido o primeiro ano, receberá a Car-
carga e se o peso declarado for inferior ao que houver
teira Nacional de Habilitação, desde que não tenha co-
sido informado na documentação por ele apresentada
metido qualquer infração de natureza grave ou gravíssi-
(art. 257, § 4º CTB);
ma ou que não seja reincidente em infração média (art.
O transportador responderá pela infração se o trans-
148, § 3º).
O condutor deverá sempre que estiver dirigindo por- porte de carga em excesso de peso for proveniente de
tar a Permissão para Dirigir ou a Carteira Nacional de Ha- mais de um embarcador e ultrapassar o peso bruto total.
bilitação, conforme determina o art. 159, § 1º CTB). Observe que nesta situação, fica excluída a responsabili-
Importante regra está prevista no art. 160 CTB. O dade do embarcador. (art. 257, § 5º CTB).
condutor condenado por delito de trânsito deverá ser O transportador responderá solidariamente ao em-
submetido a novos exames para que que possa voltar barcador pela infração relativa ao excesso do peso bruto
a dirigir, de acordo com as normas estabelecidas pelo total, se o peso declarado na documentação for superior
CONTRAN. ao limite legal (art. 257, § 6º CTB).
LEGISLAÇÃO ESPECÍFICA

Inclusive, em caso de acidente grave, o condutor A cada infração cometida são computados os se-
envolvido poderá ser submetido a outros exames que guintes números de pontos (art. 259 CTB):
poderão ser determinados pela autoridade executiva es- I - gravíssima - sete pontos;
tadual de trânsito. II - grave - cinco pontos;
Destaca-se que a autoridade de trânsito poderá tam- III - média - quatro pontos;
bém apreender o documento de habilitação do condutor IV - leve - três pontos.
até a sua aprovação nos exames realizados (art. 160, § § A aplicação das penalidades será realizada da se-
1º e 2º). guinte forma:

141
Suspensão para dirigir (art. 261 CTB): Julgamento (art. 281 CTB): caberá à autoridade de
- sempre que o infrator atingir a contagem de 20 trânsito por meio da análise do auto de infração.
(vinte) pontos, no período de 12 (doze) meses; Será arquivado o auto:
II - por transgressão às normas estabelecidas neste - se considerado inconsistente ou irregular;
Código, cujas infrações preveem, de forma específica, a - se, no prazo de trinta dias, não for expedida a noti-
penalidade de suspensão do direito de dirigir ficação da autuação.
Cassação do documento de habilitação ocorrerá(art. Aplicação da penalidade (art. 282 CTB): na sequência
263 CTB) deverá ser expedida notificação ao proprietário do veícu-
I - quando, suspenso o direito de dirigir, o infrator lo ou ao infrator.
conduzir qualquer veículo;
II - no caso de reincidência, no prazo de doze meses, Crimes de Trânsito.
das infrações previstas no inciso III do art. 162 e nos arts. Nos crimes de trânsito serão aplicadas as normas
163, 164, 165, 173, 174 e 175; gerais do Código Penal e do Código de Processo Penal,
III - quando condenado judicialmente por delito de bem como a Lei 9.099/95.
trânsito, observado o disposto no art. 160. Quando se tratar de crime de trânsito em que ocor-
O condutor poderá requerer sua reabilitação, trans- rer lesão corporal culposa poderão ser aplicadas a com-
corridos dois anos da data em que houve a cassação da posição de danos; aplicação imediata de penas restritivas
CNH. Para tanto, deverá submeter-se a todos os exames de direitos e multa; suspensão condicional do processo.
necessários (art. 263, § 2º CTB). Tratam-se de benefícios estabelecidos em lei que po-
Destaca-se que as penalidades de suspensão do di- derão ser oferecidos ao réu.
reito de dirigir e de cassação da CNH, deverão advir de Porém, não fará jus a referidos benefícios aquele que
decisão fundamentada da autoridade de trânsito, profe- houver praticado o crime nas seguintes circunstâncias
rida em processo administrativo, assegurado ao infrator (art. 291, § 1º CTB):
- sob a influência de álcool ou qualquer outra subs-
o direito à ampla defesa. (art. 265 CTB).
tância psicoativa que determine dependência;
Finalmente, o infrator será submetido a curso de re-
- participando, em via pública, de corrida, disputa ou
ciclagem nas seguintes situações (art. 268 CTB):
competição automobilística, de exibição ou demonstra-
I - quando, sendo contumaz, for necessário à sua
ção de perícia em manobra de veículo automotor, não
reeducação;
autorizada pela autoridade competente;
II - quando suspenso do direito de dirigir;
- transitando em velocidade superior à máxima per-
III - quando se envolver em acidente grave para o
mitida para a via em 50 km/h (cinquenta quilômetros por
qual haja contribuído, independentemente de processo hora).
judicial; As penalidades de suspensão ou a proibição de se
IV - quando condenado judicialmente por delito de obter a permissão ou a habilitação para dirigir veículo
trânsito; automotor poderão ser impostas isolada ou cumulativa-
V - a qualquer tempo, se for constatado que o con- mente com outras penalidades (art. 292 CTB).
dutor está colocando em risco a segurança do trânsito; A penalidade de suspensão ou de proibição de se
VI - em outras situações a serem definidas pelo CON- obter a permissão ou a habilitação, para dirigir veículo
TRAN. automotor, tem a duração de dois meses a cinco anos
Dentre as medidas administrativas podem ocorrer: (art. 293 CTB).
retenção do veículo; remoção do veículo; recolhimento
da CNH ou do Certificado de Registro, conforme previsto Fique Atento!:
no art. 269 CTB. - 48 horas: é o prazo que o réu terá, para entregar a
Processo Administrativo Permissão para Dirigir ou a Carteira de Habilitação, após
O processo administrativo terá as seguintes fases: o trânsito em julgado da sentença condenatória,
- autuação; Durante a investigação ou ação penal, o juiz poderá
- julgamento; determinar de forma motivada decretar, em decisão mo-
- aplicação das penalidades. tivada, a suspensão da permissão ou da habilitação para
Autuação (art. 280 CTB): ocorrerá com a lavratura do dirigir veículo automotor, ou a proibição de sua obtenção
auto de infração decorrente da infração cometida. (art. 294 CTB).
No auto deverá constar: Circunstâncias que sempre agravam as penalidades
I- tipificação da infração; dos crimes de trânsito (art. 298 CTB):
II - local, data e hora do cometimento da infração; I - com dano potencial para duas ou mais pessoas ou
III - caracteres da placa de identificação do veículo, com grande risco de grave dano patrimonial a terceiros;
sua marca e espécie, e outros elementos julgados neces- II - utilizando o veículo sem placas, com placas falsas
LEGISLAÇÃO ESPECÍFICA

sários à sua identificação; ou adulteradas;


IV - o prontuário do condutor, sempre que possível; III - sem possuir Permissão para Dirigir ou Carteira
V - identificação do órgão ou entidade e da autorida- de Habilitação;
de ou agente autuador ou equipamento que comprovar IV - com Permissão para Dirigir ou Carteira de Habi-
a infração; litação de categoria diferente da do veículo;
VI - assinatura do infrator, sempre que possível, va- V - quando a sua profissão ou atividade exigir cui-
lendo esta como notificação do cometimento da infra- dados especiais com o transporte de passageiros ou de
ção. carga;

142
VI - utilizando veículo em que tenham sido adulte- Ademais, o § 4º acima mencionado, informa que se-
rados equipamentos ou características que afetem a sua rão fixadas metas no âmbito dos Estados e do Distrito
segurança ou o seu funcionamento de acordo com os Federal pelo CONTRAN de cada um destes.
limites de velocidade prescritos nas especificações do fa- § 5o Antes de submeterem as propostas ao Contran,
bricante; os Cetran, o Contrandife e o Departamento de Polícia Ro-
VII - sobre faixa de trânsito temporária ou perma- doviária Federal realizarão consulta ou audiência pública
nentemente destinada a pedestres para manifestação da sociedade sobre as metas a serem
O condutor que se envolver em acidente de trânsito propostas.
com vítima não será preso em flagrante e nem poderá Por sua vez, o § 5º afirma que antes de definirem tais
lhe ser imposta fiança, desde que preste imediato e inte- metas no âmbito dos Estados, o Departamento de Polí-
gral socorro à vítima (art. 301 CTB). cia Rodoviária Federal deverá realizar consultas públicas
Os crimes de trânsito estão previstos nos artigos 302 para que a sociedade possa participar no intuito de ma-
à 312-A do Código de Trânsito Brasileiro. nifestar-se sobre as metas propostas.
A Lei 13.614 promulgada em 11 de janeiro de 2018 § 6o As propostas dos Cetran, do Contrandife e do De-
tem por objetivo a criação do Plano Nacional de Redução
partamento de Polícia Rodoviária Federal serão encami-
de Mortes e Lesões no Trânsito (Pnatrans).
nhadas ao Contran até o dia 1o de agosto de cada ano,
Assim, foram acrescentados, por força da referida lei,
acompanhadas de relatório analítico a respeito do cum-
dispositivos à Lei no 9.503, de 23 de setembro de 1997
(Código de Trânsito Brasileiro), para estabelecer regras primento das metas fixadas para o ano anterior e de expo-
sobre o regime de metas de redução de índice de mortos sição de ações, projetos ou programas, com os respectivos
no trânsito por grupos de habitantes e de índice de mor- orçamentos, por meio dos quais se pretende cumprir as
tos no trânsito por grupos de veículos. metas propostas para o ano seguinte.
É de conhecimento notório que as mortes de trân- § 7o As metas fixadas serão divulgadas em setembro,
sito, nos últimos anos, atingiram níveis muito altos em durante a Semana Nacional de Trânsito, assim como o de-
virtude da inobservância das regras de trânsito, bem sempenho, absoluto e relativo, de cada Estado e do Distrito
como imprudência dos motoristas na condução de veí- Federal no cumprimento das metas vigentes no ano ante-
culos automotores motivada também pelo uso de álcool rior, detalhados os dados levantados e as ações realiza-
e substâncias ilícitas. das por vias federais, estaduais e municipais, devendo tais
Assim, foi criado o Plano Nacional de Redução de informações permanecer à disposição do público na rede
Mortes e Lesões no Trânsito (Pnatrans). O art. 326-A inse- mundial de computadores, em sítio eletrônico do órgão
rido no Código de Trânsito Brasileiro preceitua que os in- máximo executivo de trânsito da União
tegrantes do SINETRAN devem objetivar o cumprimento As propostas para fixação das metas deverão ser en-
de metas anuais para a redução de índice de mortos por caminhadas anualmente até 1º de agosto, acompanha-
grupos de veículos e também por grupo de habitantes. das de relatório que informe os avanços obtidos no ano
Art. 326-A. A atuação dos integrantes do Sistema Na- anterior.
cional de Trânsito, no que se refere à política de segurança Em setembro de cada ano, durante a Semana Nacio-
no trânsito, deverá voltar-se prioritariamente para o cum- nal de Trânsito, as metas serão divulgadas, juntamente
primento de metas anuais de redução de índice de mortos com o relatório referente ao trabalho de cada Estado no
por grupo de veículos e de índice de mortos por grupo de ano anterior para redução dos índices de mortes.
habitantes, ambos apurados por Estado e por ano, deta- § 8o O Contran, ouvidos o Departamento de Polícia
lhando-se os dados levantados e as ações realizadas por Rodoviária Federal e demais órgãos do Sistema Nacional
vias federais, estaduais e municipais. de Trânsito, definirá as fórmulas para apuração dos índi-
§ 1o O objetivo geral do estabelecimento de metas é,
ces de que trata este artigo, assim como a metodologia
ao final do prazo de dez anos, reduzir à metade, no míni-
para a coleta e o tratamento dos dados estatísticos neces-
mo, o índice nacional de mortos por grupo de veículos e o
sários para a composição dos termos das fórmulas.
índice nacional de mortos por grupo de habitantes, relati-
vamente aos índices apurados no ano da entrada em vigor § 9o Os dados estatísticos coletados em cada Estado
da lei que cria o Plano Nacional de Redução de Mortes e e no Distrito Federal serão tratados e consolidados pelo
Lesões no Trânsito (Pnatrans). respectivo órgão ou entidade executivos de trânsito, que
Conforme se verifica do § 1º, o objetivo geral do Pla- os repassará ao órgão máximo executivo de trânsito da
no é a redução pela metade do número de mortos por União até o dia 1o de março, por meio do sistema de regis-
grupo de veículos e o índice nacional de mortos por gru- tro nacional de acidentes e estatísticas de trânsito.
pos de habitantes no prazo já determinado de dez anos. § 10. Os dados estatísticos sujeitos à consolidação
§ 2o As metas expressam a diferença a menor, em pelo órgão ou entidade executivos de trânsito do Estado
base percentual, entre os índices mais recentes, oficial- ou do Distrito Federal compreendem os coletados naquela
LEGISLAÇÃO ESPECÍFICA

mente apurados, e os índices que se pretende alcançar. circunscrição:


§ 3o A decisão que fixar as metas anuais estabelecerá I - pela Polícia Rodoviária Federal e pelo órgão execu-
as respectivas margens de tolerância. tivo rodoviário da União;
§ 4o As metas serão fixadas pelo Contran para cada II - pela Polícia Militar e pelo órgão ou entidade exe-
um dos Estados da Federação e para o Distrito Federal, cutivos rodoviários do Estado ou do Distrito Federal;
mediante propostas fundamentadas dos Cetran, do Con- III - pelos órgãos ou entidades executivos rodoviá-
trandife e do Departamento de Polícia Rodoviária Federal, rios e pelos órgãos ou entidades executivos de trânsito dos
no âmbito das respectivas circunscrições. Municípios.

143
§ 11. O cálculo dos índices, para cada Estado e para
o Distrito Federal, será feito pelo órgão máximo executivo
de trânsito da União, ouvidos o Departamento de Polícia HORA DE PRATICAR!
Rodoviária Federal e demais órgãos do Sistema Nacional
de Trânsito. 1) (TCE-PA - Auditor de Controle Externo - Área Admi-
§ 12. Os índices serão divulgados oficialmente até o nistrativa - Serviço Social - CESPE/2016) Julgue o item
dia 31 de março de cada ano. subsecutivo, acerca do Estatuto da Criança e do Adoles-
§ 13. Com base em índices parciais, apurados no de- cente (ECA).
correr do ano, o Contran, os Cetran e o Contrandife po- De acordo com o ECA, é considerada criança a pessoa com
derão recomendar aos integrantes do Sistema Nacional até doze anos de idade incompletos.
de Trânsito alterações nas ações, projetos e programas
em desenvolvimento ou previstos, com o fim de atingir as ( ) Certo ( ) Errado
metas fixadas para cada um dos Estados e para o Distrito
Federal. 2) (DPU - Assistente Social - CESPE/2016) Segundo as
§ 14. A partir da análise de desempenho a que se re- normas contidas na legislação social voltada para os direi-
fere o § 7o deste artigo, o Contran elaborará e divulgará, tos sociais e proteção de crianças e adolescentes, julgue o
também durante a Semana Nacional de Trânsito: seguinte item.
I - duas classificações ordenadas dos Estados e do Para o Sistema Nacional de Atendimento Socioeducativo,
Distrito Federal, uma referente ao ano analisado e outra é prioritária a aplicação de medidas privativas ou restritivas
que considere a evolução do desempenho dos Estados e do de liberdade em estabelecimento educacional, de modo a
Distrito Federal desde o início das análises; garantir a inclusão social dos egressos do sistema socioe-
II - relatório a respeito do cumprimento do objetivo ducativo.
geral do estabelecimento de metas previsto no § 1o deste
artigo. ( ) Certo ( ) Errado
Os índices serão acompanhados e com base nas
parciais informadas anualmente, o Contran, os Cetrans e 3) (TJ-DFT - Analista Judiciário - Judiciária - CESPE/2015)
o Contrandife poderão recomendar aos integrantes no Julgue o próximo item, de acordo com o disposto no Códi-
SINETRAN, alterações nas ações, projetos e programas, go de Defesa do Consumidor e no Estatuto da Criança e do
sempre com o objetivo de alcance das metas fixadas para Adolescente (ECA).
cada um dos Estados e Distrito Federal. De acordo com o ECA, o conselho tutelar pode aplicar, con-
forme a gravidade do caso, medida de encaminhamento a
tratamento psicológico ou psiquiátrico aos pais que apli-
quem castigo físico ou tratamento cruel ou degradante
como formas de disciplina ou correção do comportamento
de criança ou adolescente.

( ) Certo ( ) Errado

4) (DPE-PE - Defensor Público - CESPE/2015) No item


abaixo, é apresentada uma situação hipotética, seguida de
uma assertiva a ser julgada conforme as normas do ECA e o
entendimento do STJ.
Alberto, adolescente condenado a cumprir medida socioe-
ducativa de internação, diante da inexistência de estabeleci-
mento apropriado na cidade de residência de seus pais, foi
custodiado em unidade distante, em razão da superlotação
da unidade mais próxima. Nessa situação, houve violação
ao direito absoluto do adolescente previsto no ECA: Alberto
deveria ter sido enviado para a localidade mais próxima do
domicílio dos seus pais, mesmo que a unidade de custódia
estivesse superlotada.

( ) Certo ( ) Errado
LEGISLAÇÃO ESPECÍFICA

144
5) (DPE-PE - Defensor Público - CESPE/2015) No item
abaixo, é apresentada uma situação hipotética, seguida de
uma assertiva a ser julgada conforme as normas do ECA e o
GABARITO
entendimento do STJ.
Marcelino, maior imputável, fotografou sua sobrinha, de
1 Certo
treze anos de idade, enquanto ela tomava banho. As fo-
tos mostravam as partes íntimas da adolescente e algumas 2 Errado
imagens mostravam apenas os órgãos genitais da garota. 3 Errado
Apurou-se que Marcelino jamais praticou qualquer ato libi-
dinoso com a sobrinha nem divulgou o material fotográfico 4 Errado
obtido e que ele utilizava as fotos apenas para satisfazer a 5 Certo
própria lascívia. Nessa situação, Marcelino responderá por
crime previsto no ECA, uma vez que registrou cena porno- 6 E
gráfica envolvendo adolescente. 7 C
8 D
( ) Certo ( ) Errado

6) (PC-PI – AGENTE DE POLÍCIA – NUCEPE – 2018) A


segurança pública, dever do Estado, direito e responsabi-
lidade de todos, é exercida para a preservação da ordem
pública e da incolumidade das pessoas e do patrimônio,
através dos seguintes órgãos:

a) Polícia Federal, Polícia Rodoviária Federal, Polícia Fer-


roviária Federal e Polícias Civis, tão somente.
b) somente pela Polícia Federal, Polícias Civis e Polícias
Militares.
c) Polícia Federal, Polícia Rodoviária Federal, Polícias Civis,
Polícias Militares e Corpos de Bombeiros Militares, tão
somente.
d) Polícia Federal, Polícia Rodoviária Federal, Polícia Fer-
roviária Federal, Polícias Civis, Polícias Militares, Cor-
pos de Bombeiros Militares e Ministério Público.
e) Polícia Federal, Polícia Rodoviária Federal, Polícia Fer-
roviária Federal, Polícias Civis, Polícias Militares e Cor-
pos de Bombeiros Militares, tão somente.

7) A Administração Pública direta e indireta do Municí-


pio, obedecerá aos princípios dispostos na Lei Orgânica,
exceto:

a)legalidade
b) impessoalidade
c) contraditório
d) moralidade
e) publicidade

8) O Vereador, membro do poder legislativo do municí-


pio, não poderá licenciar-se:

a) por motivo de doença;


b) para tratar, sem remuneração, de interesse particular,
desde que o afastamento não ultrapasse 120 (cento e
vinte) dias por sessão legislativa.
LEGISLAÇÃO ESPECÍFICA

c)para desempenhar missões temporárias, de caráter cul-


tural ou de interesse do Município
d) para desempenhar missões temporárias, de caráter
cultural, do Estado a qual o município esta vinculado.

145
ANOTAÇÕES

_______________________________________________________________________________________________________

_______________________________________________________________________________________________________

_______________________________________________________________________________________________________

_______________________________________________________________________________________________________

_______________________________________________________________________________________________________

_______________________________________________________________________________________________________

_______________________________________________________________________________________________________

_______________________________________________________________________________________________________

_______________________________________________________________________________________________________

_______________________________________________________________________________________________________

_______________________________________________________________________________________________________

_______________________________________________________________________________________________________

_______________________________________________________________________________________________________

_______________________________________________________________________________________________________

_______________________________________________________________________________________________________

_______________________________________________________________________________________________________

_______________________________________________________________________________________________________

_______________________________________________________________________________________________________

_______________________________________________________________________________________________________

_______________________________________________________________________________________________________

_______________________________________________________________________________________________________

_______________________________________________________________________________________________________

_______________________________________________________________________________________________________

_______________________________________________________________________________________________________

_______________________________________________________________________________________________________

_______________________________________________________________________________________________________

_______________________________________________________________________________________________________

_______________________________________________________________________________________________________

_______________________________________________________________________________________________________
LEGISLAÇÃO ESPECÍFICA

_______________________________________________________________________________________________________

_______________________________________________________________________________________________________

_______________________________________________________________________________________________________

_______________________________________________________________________________________________________

_______________________________________________________________________________________________________

146

Você também pode gostar